Essentials of Medical Pharmacology [8 ed.] 9789352704996

Untainted Version

1,076 100 256MB

English Pages 1081

Report DMCA / Copyright

DOWNLOAD PDF FILE

Table of contents :
Bookmarks
drugs acting on PNS. 25.skeletal muscle relaxants
y12. 5-HT & ITS ANTAGONISTS & DRUG THERAPY OF MIGRAINE
y11. HISTAMINE & ANTI-HISTAMINICS
ysec.3 AUTACOIDS AND RELATED DRUGS
y10. ANTIADRENERGIC DRUGS & DRUGS FOR GLAUCOMA
y9.ADRENERGIC TRANSMISSION & ADRENERGIC DRUGS
y8. ANTICHOLINERGIC DRUGS AND DRUGS ACTING ON AUTONOMIC GANGLIA
y7.CHOLINERGIC TRANSMISSION & CHOLINERGIC DRUGS
ysec2. ANS: GENERAL CONSIDERATIONS
y6. ADVERSE DRUG EFFECTS
y5. ASPECTS OF PHARMACOTHERAPY, CLINICAL PHARMACOLOGY & DRUG DEVELOPMENT
y4.PHARMACODYNAMICS: MECHANISM OF DRUG ACTION & RECEPTOR PHARMACOLOGY
y3. PHARMACOKINETICS: METABOLISM & EXCRETION OF DRUGS, KINETICS OF ELIMINATION
y2.PHARMACOKINETICS: MEMBRANE TRANSPORT, ABSORPTION & DISTRIBUTION OF DRUGS
y1. Introduction, routes of drug administration
list of abbreviations
contents
INDEX
References
Drugs & FDC
Drugs in breast feeding
Prescribing in pregnancy
solution to problem directed study
71. Drug interactions
70. Vaccines, anti-sera & immunoglobulins
69. Vitamins
68. Chelating agents
67. Anti-septics,disinfectants & ectoparasiticides
66.Drugs acting on skin and mucous membranes
Sec14...65.Immunosuppressant drugs
sec13..64.Anti-cancer drugs
63. Anti-helminthic drugs
62. Anti-amoebic & other anti-protozoal drugs
61. Anti malarial drugs
60. Antiviral drugs(anti-retrovirus)
59. Anti-viral drugs(non-retroviral)
58. Anti-fungal drugs
57. Anti leprotic drugs
56. Anti tubercular drugs
55. Macrolide , lincosamide, glycopeptide,....
54. Aminoglycoside antibiotics
53. Tetracyclines & chloroamphenicol
52. beta lactam antibiotics
51. sulphonamides, cotrimoxazole & quinolones
sec.12 antimicrobial drugs.50.general considerations
49. drugs for constipation & diarrhoea
48. Anti-emetic,prokinetic & digestant drugs
sec.11. gastrointestinal drugs47. drugs for peptic ulcers & GE reflux
46. Hypolipidaemic drugs
45. Drugs affecting coagulation, bleeding & thrombosis
sec.10drugs affecting blood & blood formation.44. hematinics & Ep
43. Anti-diuretics
42. Diuretics
sec.9 drugs acting on kidney
41. Anti-hypertensive drugs
40. anti-anginal and other anti-ischaemic drugs
39. Anti-arrhythmic drugs
38. Cardiac glycosides & drugs for cardiac failure
37. Nitric oxide and vasoactive peptide signal molecules
36. Drugs affecting RAAS
sec. 8cardiovascular drugs general considerations
35. CNS stimulants & cognition enhancers
34. Opioid analgesics & antagonists
33. anti-depressant & anti-anxiety drugs
32. Anti-psychotic & anti-maniac drugs
31. Anti- prakinsonian drugs
30. Anti-epileptics
29. Sedative- hypnotics
28. Ethyl & methyl alcohol
27. General anaesthetics
26. Local anaesthetics
24. hormones & drugs affecting calcium balance
23. Oxytocin & other drugs acting on uterus
22. Estrogens , progestins & contraceptives
y21. Androgens & related drugs, drugs for erectile dysfunction
y20. corticosteroids
y19. Insulin, oral antidiabetic drugs & glucagon
y18. Thyroid hormones and thyroid inhibitors
y17. Anterior pituitary hormones
sec. Harmones and related drugs introduction
ysec.4. Respiratory system drugs
y15. Antirheumatoid & antigout drugs
y14. NSAIDS
y13.PGs,LTs & PAFs
mebooksfree.com
Recommend Papers

Essentials of Medical Pharmacology [8 ed.]
 9789352704996

  • 0 0 0
  • Like this paper and download? You can publish your own PDF file online for free in a few minutes! Sign Up
File loading please wait...
Citation preview

,

Essentials of

Medical Pharmacology

Essentials of

Medical Pharmacology Eighth Edition

KD TRIPATHI MD Ex-Director-Professor and Head of Pharmacology Maulana Azad Medical College and associated LN and GB Pant Hospitals New Delhi, India

t·-·

JAYPEE BROTHERS MEDICAL PUBLISHERS The Health Sciences Publisher New Delhi I London I Panama

t

Glbt

Jaypee Brothers Medical Publishers (P) Ltd

Headquarters Jaypee Brothers Medical Publishers (P) Ltd 4838124, Ansari Road, Daryaganj New Delhi 110 002, India Phone: +91 -11-43574357 Fax: +91-11 -43574314 Email: [email protected]

Overseas Offices J.P. Medical ltd 83 Victoria Street, London SW1 H OHW (UK) Phone: +44 20 3170 8910 Fax: +44 (0)20 3008 61 80 Email: [email protected]

Jaypee-Highlights Medical Publishers Inc City of Knowledge, Bid. 235, 2nd Floor, Clayton Panama City, Panama Phone: +1 507-301-0496 Fax: +1 507-301-0499 Email: [email protected]

Jaypee Brothers Medical Publishers (P) Ltd 17I1-B Babar Road, Block-B, Shyamoli Mohammadpur, Dhaka-1207 Bangladesh Mobile: +08801912003485 Email: [email protected]

Jaypee Brothers Medical Publishers (P) ltd Bhotahity, Kathmandu, Nepal Phone: +977-9741 283608 Email: [email protected]

Website: www.jaypeebrothers.com Website: www.jaypeedigital.com

© 2019, KO Tripathi Managing Editor: M . Tripathi

The views and opinions expressed in this book are solely those of the original contributor(s)/author(s) and do not necessarily represent those of editor(s) of the book. All rights reserved. No part of this publication may be reproduced, stored or transmitted in any form or by any means, electronic, mechanical, photocopying, recording or otherwise, without the prior permission in writing o1 the publishers. All brand names and product names used in this book are trade names, service marks, trademarks or registered trademarks of their respective owners. The publisher is not associated with any product or vendor mentioned in this book. Medical knowledge and practice change constantly. This book is designed to provide accurate, authoritative information about the subject matter in question. However, readers are advised to check the most current ln1ormation available on procedures included and check information from the manu1acturer of each product to be administered, to verity the recommended dose, formula, method and duration of administration, adverse effects and contraindications. It is the responsibility o1 the practitioner to take all appropriate safety precautions. Neither the publisher nor the author(s)leditor(s) assume any liability for any injury and/or damage to persons or property arising from or related to use of material in this book. This book is sold on the understanding that the publisher is not engaged in providing pro1essional medical services. If such advice or services are required, the services of a competent medical professional should be sought. Every effort has been made where necessary to contact holders of copyright to obtain permission to reproduce copyright material. If any have been inadvertently overlooked, the publisher will be pleased to make the necessary arrangements at the first opportunity. The CD/DVD-ROM (if any) provided in the sealed envelope with this book is complimentary and tree of cost. Not meant for sale.

Inquiries tor bulk sales may be solicited at: jaypee @jaypeebrothers.com

Essentials of Medical Pharmacology First Edition: Second Edition: Third Edition: Fourth Edition: Fifth Edition: Sixth Edition: Seventh Edition: Reprint: Eighth Edition:

1985 1988 1994 1999, Updated Reprint: 2001 2003 2008 2013 2014 2019

ISBN: 978-93-5270-499-6 Printed at Replika Press Pvt. ltd.

Preface Medical pharmacology is a unique blend of basic pharmacology, clinical pharmacology and pharmacotherapeutics. The subject is highl y dynam ic wi th concepts and priority drugs changing rapidly. Innovations and developments are happening at an unprecedented pace. Several ne w molecula r targets for drug action have been identified a nd novel drugs produced to attack them. On the other hand, a huge body o f evidence has been generated to quantify impact of various drugs and reg imens o n well defined therapeutic end points, so that practice of medic ine is transforming from ' impression based ' to 'evidence based'. The prese nt edition focuses on evidence based medicine by referring to numerous large randomized tria ls and other studies wh ich have shaped current therapeutic practices. By evaluating suc h evidences, professional bodies, eminent health institutes, expert committees and WHO have formu lated the rapeutic guidelines for treati ng many conditions, as well as for use of specific drugs. The latest guidelines have been summarized and included in the present edition along wi th other deve lopments and the core content. Adopting the 'prototype drug' approach and a structured, syste matic and user-fri endly format, all chapters have been thoroughl y revised and updated. In this edi tion, drug classifications have been presented as eye-catching charts which help create pictorial memory. A new chapter on 'Nitric Oxide and Vasoacti ve Peptide Signal Molecules' has been added a long with some recently introduced drugs which act through receptors for these molecules or by a ltering their turnover. Priority has been accorded to drugs that are marketed in India, and their leadi ng brand names are mentioned along with dosage forms. All recently released drugs are included, while those not commercially available or infrequently used have been excluded or described in small type. India specific information on drugs and diseases finds a place in relevant topics. Treatment of diseases like TB, leprosy, HIV-AI DS, malaria, Kala- azar which are covered under WHO and National Health Programmes arc described as per the latest recommendations of these organizations. Several new figures, c harts, tables and highlight boxes have been added and many older ones have been revised/im proved. The recent material and data has been authenticated by quoting its source. A brief list of useful references fo r further reading is provided at the end of the book. The 'Problem Directed Study' at the end of most chapters provides an exerc ise in therapeutic decision making. 1 thank my colleagues and students for providing va luable inputs and raising thought ful queries. As ever, the driving force behind this book has been Shri Jitendar P Vij (Group Chairman) and Mr Ankit Vij (Managing Director) of M/s Jaypee Brothers Med ical Publishers (P) Ltd, New Delhi, India. The staff of Jaypee Brothers, especially Ms Sunita Katia (Executive Assista nt to Group Chairman and Publishing Manager), Ms Geeta Srivastava (Proof Reader), Mr Manoj Pahuja (Graphic Designer) and Mr Kapil Dev Sharma (DTP Operator) deserve special commendation for excellent production of this text. Cooperation and participation of my wife has been pivotal.

New Delhi June 201 8

KD Tripathi

Extract from Preface to the First Edition Phannacology is both a basic and an applied science. It forms the backbone of rational therapeutics. Whereas the medical student and the presc ribing physician are primarily concerned with the applied aspects, correct and skilful application of drugs is impossible without a proper understanding of their basic pharmacology. Medical pham1acology, therefore, must include both funda mental background and clinical pha rmacologica l information. Objective and quantitati ve data on the use of drugs in man, i.e., relationship between plasma concentration and intensity of therapeutic/toxic act io ns, p lasma half li ves, relati ve efficacy of diffe rent medications a nd incidence of adverse effects etc., are being obtained with the a im of optimising drug therapy. The concepts regarding mechani sm of action of dru gs are changing. In addition, new drugs are being introduced in different countri es at an explosive pace. A plethora o f information thus appears to be important. However, trying to impart all this to a med ical student wou ld be cou nter-productive. One of the important aims of this book is to delineate the essentia l information about drugs. The opening sentence in each chapter defines the class of dmgs considered. A 'prototype' approach has been foll owed by describing the representati ve drug of a class followed by features by which individual members differ from it. Leading trade names have been included. Clinically relevant drug interactions have been mentioned. Clear-cut guide lines on selection of drugs and thei r c linical status have been outlined on the basis of current information. Original, simple a nd self-explanatory illustrations, tables and flowcharts have been used with impunity. Selected chemical structures are depicted. Recent developments have been incorporated . However, discretion has been used in including only few of the multitude of new drugs not yet avai lable in India. T his is based on the ir likelihood of being marketed soon. The information and views have been arranged in a n orderly seq uence of distinct stateme nts. I hope thi s man ageable vo lume book wou ld se rve to di spe l awe towards pharmacology from the minds of medical students and provide a conc ise and uptodate informati on source for prescribers who wish to remain informed of the current concepts and developments concerning drugs. My sincere thanks are due to my colleagues for the ir val uable comments and suggestions.

New Delhi I st Jan ., 1985

KD Tripathi

Contents SECTION 1

General Pharmacological Principles 1. Introduction, Routes of Drug Administration 2. Pharmacokinetics: Membrane Transport, Absorption and Distribution of Drugs

1 15

3. Pharmacokinetics: Metabolism and Excretion of Drugs, Kinetics of Elimination

28

4. Pharmacodynamics: Mechanism of Drug Action; Receptor Pharmacology

45

5. Aspects of Pharmacotherapy, Clinical Pharmacology and Drug Development

71

6. Adverse Drug Effects

92

SECTION 2

Drugs Acting on Autonomic Nervous System Autonomic Nervous System: General Considerations

7. Cholinergic Transmission and Cholinergic Drugs

103

110

8. Anticholinergic Drugs and Drugs Acting on Autonomic Ganglia

124

9. Adrenergic Transmission and Adrenergic Drugs

136

10. Antiadrenergic Drugs (Adrenergic Receptor Antagonists) and Drugs for Glaucoma

153

SECTION 3

Autacoids and Related Drugs 11. Histamine and Antihistaminics

174

12. 5-Hydroxytryptamine, its Antagonists and Drug Therapy of Migraine

185

13. Prostaglandins, Leukotrienes (Eicosanoids) and Platelet Activating Factor

197

14. Nonsteroidal Antiinflammatory Drugs and Antipyretic-Analgesics

209

15. Antirheumatoid and Antigout Drugs

227

SECTION 4

Respiratory System Drugs 16. Drugs for Cough and Bronchial Asthma

237

viii

ESSENTIALS OF MEDICAL PHARMACOLOGY

SECTION

s

Hormones and Related Drugs 17. 18. 19. 20. 21. 22. 23. 24.

Introduction

255

Anterior Pituitary Hormones

257 267 280 306 320 330 354 360

Thyroid Hormones and Thyroid Inhibitors Insulin, Oral Antidiabetic Drugs and Glucagon Corticosteroids Androgens and Related Drugs, Drugs for Erectile Dysfunction Estrogens, Progestins and Contraceptives Oxytocin and Other Drugs Acting on Uterus Hormones and Drugs Affecting Calcium Balance

SECTION 6

Drugs Acting on Peripheral (Somatic) Nervous System 25. Skeletal Muscle Relaxants 26. Local Anaesthetics

373 386

SECTION 7

Drugs Acting on Central Nervous System 27. General Anaesthetics 28. Ethyl and Methyl Alcohols

29. 30. 31. 32. 33. 34. 35.

Sedative-Hypnotics Antiepileptic Drugs Antiparkinsonian Drugs Drugs Used in Mental Illness: Antipsychotic and Antimanic Drugs Drugs Used in Mental Illness: Antidepressant and Antianxiety Drugs Opioid Analgesics and Antagonists CNS Stimulants and Cognition Enhancers

399 415 424 438 452 462 481 497 515

SECTION 8

Cardiovascular Drugs Cardiac Electrophysiological Considerations

36. Drugs Affecting Renin-Angiotensin System 37. Nitric Oxide and Vasoactive Peptide Signal Molecules

521 524 540

ix

CONTENTS 38. Cardiac Glycosides and Drugs for Heart Failure 39. Antiarrhy thmic Drugs 40. Antiangin al and Other Anti-ischaemic Drugs

556

41. Antihype rtensive Drugs

604

570 584

SECTION 9

Drugs Acting on Kidney Relevant Physiology of Urine Formatio n

621 625

42. Diuretics

639

43. Antidiure tics

SECTION 10

Drugs Affecting Blood and Blood Form ation 44. Haematinics and Erythropo ietin

645

45. Drugs Affecting Coagulation, Bleeding and Thrombo sis

659

46. Hypolipid aemic Drugs

682

SECTION 11

Gastrointestinal Drugs 47. Drugs for Peptic Ulcer and Gastroesophageal Reflux Disease 48. Antiemet ic, Prokineti c and Digestant Drugs 49. Drugs for Constipa tion and Diarrhoea

695 709

721

SECTION 12

Antimicrobial Drugs 50. Antimicro bial Drugs: General Considerations 51. Sulfonam ides, Cotrimoxazole and Quinolon es 52. Beta-Lact am Antibiotics 53. Tetracyclines and Chloramp henicol (Broad-Spectrum Antibiotic s) 54. Aminoglycoside Antibiotic s 55. Macrolide , Lincosamide, Glycopep tide and Other Antibacte rial Antibiotics; Urinary Antisepti cs 56. Antituber cular Drugs 57. Antilepro tic Drugs

739 755 766 784 793 801 815

831

X

ESS ENTIALS OF MED ICAL PHARMACO LOGY 58. Antifungal Drugs

838

59. Antiviral Drugs {Non-retro viral)

849

60. Antiviral Drugs (Anti-retro virus)

860

61. Antimalari al Drugs

873

62. Antiamoeb ic and Other Antiprotoz oal Drugs 63. Anthelmin tic Drugs

893 906

SECTION 13

Chemo therap y of Neoplastic Diseases 64. Anticancer Drugs

915

SECTION 14

Miscellaneous Drugs 65. lmmunosu ppressant Drugs

937

66. Drugs Acting on Skin and Mucous Membrane s

946

67. Antiseptics, Disinfectants and Ectoparasit icides

957

68. Chelating Agents

964

69. Vitamins

968

70. Vaccines, Antisera and lmmuneglo bulins

978

71. Drug Interaction s

987

Appendices Appendix 1: Solution to Problem Directed Study

995

Appendix 2: Prescribing in Pregnancy

1017

Appendix 3: Drugs in Breastfeed ing

1020

Appendix 4: Drugs and Fixed Dose Combinations Banned in India

1024

Selected References for Further Reading

Index

1033

1037

List of Abbreviations Amino acid Antibody ATP-binding cassene (transporter) Amphotericin B hpid complex Adenylyl cyclase Angiotensin 11 con, ening enzyme Acetylcholine Acetylcholincstcrase Acute coronary syndromes Artemisinin-based combination therapy Adrenocorticotropic hormone Alzheimer's disease Antibody-dependent cellular cytotoxicity Advcr c drug event Antid1urctic hormone Attention deficit hyperactivity disorder Adenosine diphosphale Adrenaline Adverse drug reaction Anti diphtheritic scrum Atrial extrasystole Atrial librillation Atrial Rutter Antigen Antigasgangrene serum Antihacmophilic globulin Aromatase inhibitor Acquired nnmunodeliciency syndrome Aldosterone induced protein Alanine Amyotrophic lateral sclero is Amikacin Antimicrobial agent Amphotericin B Ampoule Adenosine mono phosphate a-Aminohydroxy methylisoxazole propiomc acid ANC Acid neutralizing capacity Ang-1/11/1II Angiotensin 1/11/111 A p Atrial natnuretic peptide ANS Autonomic nervous system ANUG Acute nccrotizing ulcerative gingi, itis AP Action potential AP-I Activator protein-I APC Antigen presenting cell APD Action potential duration aPTT Activated partial thromboplastin time AQ Amodiaquinc AR Androgen receptor AA Ab ABC ABLC AC ACE ACh AChE ACS ACT ACTH AD ADCC ADE ADH ADI-ID ADP Adr ADR ADS AES AF AFI AG AGS AJ-IG Al AIDS AIP ALA ALS Am AMA AMB amp AMP AM PA

ARB Angiotensin receptor blocker ARC AI DS related complex ARS Anti rabies serum ART Antiretrovirus therapy ARV Anliretrovirus (drug) AS Artcsunate 5-ASA 5-Amino salicyclic acid ASCVD Atherosclcrot ic cardiovascular disease AT-Ill Antithrombin Ill ATG Antithymocytc globulin ATP Adenosine triphosphale ATPase Adenosine triphosphatase ATPIII Adult treatment panel Ill ATS Antitetanic serum AUC Arca under the plasma concentration-Lime curve A-V Atriovcntricular AVP Arginine, asopressin AZT Zidov udine BA L BA BB BBB BCG DCNU BCRP BD B-ARK BHC BHP Bl BL BMD BMR BNP BOL BP BPN BSA BT BuChE BW BZD C-10 CA CAB CaBP CAD

British anti lew1sitc Dritish approv ed name Borderline leprosy Dlood-brain barrier Bacillus Calmette Guerin Bischloroethy l nitrosourea (Carmustine) 13reast cancer resistance protein tw ice daily B adrenergic receptor kinase Benzene hcxach loride Benign hypertrophy of prostate Dacillary index Borderline lepromatous leprosy Bone mineral density Basal metabolic rate Brain nartriuretic peptide 2-Bromolysergic acid diethylamide Blood pressure Bisphosphonate Body surface area Borderl ine tubcrculoid leprosy Butyryl cholinestera e Body weight Benzodiazepine Decamcthonium Catecholamine Combined androgen blockade Calcium binding protein Coronary artery disease

xii

ESSE NTIALS OF MED ICAL PHARM ACOLOGY CAM cAMP CAP cap CAse CAT CBF CBG CBS CCB CCNU CCRS CD CDC CFTR cGMP CORP CH ChE CHE CIIF Chy Chy. rem. CJ CINV CKD CL Cler Cm CMI CMV C s c.o. CoEn-A COMT

cox c.p.s. CPS C PZ CQ CRABP C RBP C rD

C REB CRF

Calmodulin s· Cyclic adcnosine monophosphate Community acquired pneumonia Capsule Carbonic anhydrase Computerizedaxial tomography Cerebral blood flow Cortisol binding globulin Colloidal bismuth subeitrate Calcium channel blocker Chloroethyl cyclohexyl nitrosourca (lomustinc) Chemokine coreceptor 5 Collecting duel/Cluster of differentiation Complement dependent cytotoxicity Cystic fibrosis transport regulator 3', 5' Cyclic guanosine monophosphatc Calcitonin gene related peptide Cholesterol Chol inestcrasc Cholesterol ester Congestive heart failure Chylomicron Chylomicron remnants Cardiac index Chemotherapy induced nausea and vomiting Chronic kidney disease Clearance Crea tinine clearance Capreomycin Cell mediated immunity Cytomegalovirus Cent ral nervous system Cardiac output Coenzyme-A Catechol-O-mcth yl transferase Cyclooxygenase Cycles per second Complex partial seizures Chlorpromazinc Chloroquine Cellular retinoic acid binding protein Cellular retinol binding protein 3'.

Crohn's disease

Cyclic AMP response element binding protein Corticotropin releasing factor cs Cycloserine CSF Ccrebrospinal fl uid CTL Cytotoxic T-lymphocytcs CTZ Chcmoreceptor trigger zone CV Cardiovascular CVP Central venous pressure CVS Cardiovascular system CWD Cell" all deficient CYP450 Cytochrome P450

DA DA-B12 DAD DAG DAM DAMP DAT dDAVP DDS DDT DEC DHA DHE DI IFA DI IFRasc DHP DHT DI DIT di DLE DMA DMARD D ICM DMPA DMPP DMT DNA DOC DOCA DOM dopa DOPAC DOSS DOTS DPD DPP-4 DPT DRC DRI DST DT DT-DA d-TC

Dopamine Deoxyadenosyl cobalamin Delayed after-depolarization Di acyl glycerol Diacctyl monoxime Diphcnyl acetoxy- -methyl piperidinc methiodide Dopamine transporter Desmopressin Diamino diphenyl sulfonc (Dapsone) Dichloro diphenyl trichloroethane Diethyl carbamazine citrate Dihydroartemisi nin Dihydroergotamine Dihydro fol ic acid Dihydrofolate reductase Dihydropyridine Dihydrotestosterone Diabetes insipidus Di iodotyrosine Decilitre Disseminated lupus erythematosus Dimcthoxy amphetamine Disease modifying antirheummic drug Dimcthoxyethyl-carbomcthoxy-P-carboline Depot medroxyprogesterone acetate Dimethy l phenyl piperazinium Dimethyl tryptamine/Diva lcntmetal Iran porter Deoxyribose nucleic acid Deoxycholate Desoxy conicosterone acetate Dimcthoxymethyl amphetamine Dihydroxyphcnyl alanine 3, 4 , Dihydroxyphenyl acetic acid Dioctyl sul fosucc inate Directly observed treatment short course D1hydropyrimidine dehydrogenase Dipcptidyl peptidasc-4 Diphtheria-pertussis-tetanus triple antigen Dose-response curve Direct renin inhibitor Drug sensitivity testing (for TB) Distal tubule Diphtheria-tetanus double antigen d-Tubocurannc

DTIC

Dacarbazi ne

DTPA DVT DY

Dicthylene triam inc pcntaacetic acid Deep vein thrombosis Dynorphin

E EACA EA D ECE c.c.f. ECG ECT

Ethambutol Epsilon amino caproic acid Early after-depolarization Endothclin converting enzyme Extracellular fl uid Electrocardiogram Electroconvuls ive therapy Erectile dysfunction

ED

xiii

ABBREVIATIONS

EDRF EDTA EEG EF EGF ELAM-I B-END eNOS ENS ENT EPAC EPEC EPO EPP EPSP ER

ERA ERP ES ESR ET ETEC Eto

Endothelium dependent relaxing factor Ethylene diamine tetraacctic acid Electroenccphalogrnm Ejection fraction Epidermal growth factor Endothelial leukocyte adhesion molecule- I B-Endorphin Endothelial nitric oxid.: synthase Enteric nervous system Extraneuronal amine transporter cAMP regulated guanine nucleotide exchange factors Enteropathogenic £. coli Erythropoietin End plate potential Excitatory postsynaptic potential Estrogen receptor Endothelin receptor antagonist Effective refractory period Extrasystole Erythrocyte sedimentation rate Endothelin Entcrotoxigenic £. coli Ethionamide

Folic acid Flavin adenine dinucleotidc 5-Flucytosine Fixed dose combination FDT Fixed duration therapy (of leprosy) FEY, Forced expiratory ,olumc in I second FFA Free fany acid FKBP FK 506 {tacrolimus) binding protein FLAP Five-lipoxygenase acti, ating protein FMN Faviin mononuclcotide FP Fcrroportin FQ Fluoroquinolonc FRase Folate reductase FSH Follicle stimulating hom1one 5-FU 5-Fluorouracil FA FAD 5-FC FDC

Genetic Gamma amino butyric acid GA BA-transporter Guanylyl cyclase Good clinical practice Granulocyte colony stimulating factor Guanosine diphosphatc Gastrocsophageal reflux disease g.f. Glomerular filtration g.f.r. Glomcrular filtration rate GH Growtth hormone GHRH grow th hormone releasing honnone Growth hormone release inhibitory hormone GHRIH GIP Gastric inhibitory pcptide/Glucosedcpendent in;,ulinotrop,c polypeptide g.i.l. Gastrointestinal tract GITS Gastrointestinal therapeutic system G

GABA GAT GC GCP G-CSF GDP GERO

Glucagon-hke peptide Glucose transporter Granulocytc macrophage colony stimulating factor GnR II Gonadotropin releasing hormone GPCR G-protcin coupled receptor G-6-PD Glucosc-6-phosphatc dchydrogenase GPI Globus pallidus intema GST Glutathione-S-transfcrase GTCS Generalised tonic-elon ic seizures GT I Glyeeryl trinitratc GTP Guanosine triphosphate

GLP GLUT GM-CSF

H

HAP Hb HBV HCG IICV HDCV HDL IIETE 5-111/\A HIV HLA HMG-CoA HMW !IPA axis HPETE hr HR HRIG IIRT 5-HT 5-HTP IIVA

lsoniazid (lsonicotinic acid hydrazidc) I lospital acquired pneumonia Haemoglobin Hepatitis B virus Human chorionicgonadotropin Hcpatiti;, C virus lluman diploid cell vaccine I ligh density lipoprotein Hydroxyeicosa tctraenoic acid 5-Hydroxyindole acetic acid Human immunodeficiency virus I luman lcucocyte antigen Hydroxymcthyl glutaryl coenzyme A I ligh molecular weight Hypothalamo-pituitary-adrenal axis 11ydropcrox y e icosatetraenoic acid Hour Ilean rate 1luman rabies immuneglobulin Hormone replacement therapy 5-1-lydro,ytryptamine 5-1Hydroxytryptophan Homo, anillic acid

Indeterminate leprosy IAP Islet amyloid polypeptide IBD lnnammatory bowel disease IBS Irritable bowel syndrome ICAM- 1 Intracellular adhesion molecule-I ICSH Interstitial cell stimulating hormone 1.cl. lntradcrmal (injection) IDL Intermediate density lipoprotcin IFN Interferon IG lmmuncglohulin IGF Insu lin-like growth factor IL Interleukin ILEU Isoleucine i.m. Intramuscular I H lsonicotin ic acid hydrazide I R International normali7ed ratio lntraocular tension i.0.1. IP, Inositol triphosphate IP4 Inositol tetrak1sphosphate IPSP Inhibitory postsynaptic potential

xiv

ESSENTIALS OF MEDICAL PHARMACOLOGY IPV IRS

Inactivated poliomyelitis vaccine Insulin response substrate Intrinsic sympathomimcuc acti, ity Isolated systolic hypenension lntema11onal unit Intrauterine contraceptivedevice Intra, enous

IA

ISH IU IUCD i.v.

JAK

Janus-kinas e

Km KTZ

Kanamycin Ketocona7olc

LA LCAT LC3-KAT LDL LES leu-E K LH liq LL LMW LOX LSD LT LVF MAbs MAC MAC MAO MAP MAPKinase

max MBC MBL

MCI MDI MDMA

MDR MDT mct-ENK mEq methyl B12 Mf MF MHC MHT Ml

MIC MIF min MIT MLCK MMF 6-MP MPPT Ml'TP MQ

Local anaesthetic Lecithin cholesterol acyl transferase Long chain B-ketoacyl-CoA-thiolase Low density lipoprotein Lower esophageal sphmctcr Leuci11e enkephalin LuteiniLing hom1one Liquid Lcpromatous leprosy low molecular weight Lipoxygcnasc Lysergic acid diethy lamide Leukotricnc Lett ventricular failure

Monoclonal an11bodies Minimal alveolar concentration Mycobacterium a1•i11111 complex Monoamine oxidase Muscle action potential \11togen acti, ated protein kinase Maximum Mmimum bactericidal concentration Multibacillary leprosy Mi ld cogniti,e impaim1cnt Manic depressivc il lness Methylene dioxy methamphetamine Multidrug resistant Muhidrug therapy (of leprosy Methionine enkephalin rni lliequivalcnt Methyl cobalamin Microfilariae \1ultifactorial Major histocompatibility complex Methylene dioxy mcthamphctam,nc Myocardial infarction Minimal inh ibitory concentration Migration inhibitory factor minimum Monoiodo tyrosine Myosin light chain kinase Mycophenolate mofctil 6-Mercaptopurine Mcthylprcdmsolonc pulse therapy 4-methy1-4-phcnyltc trahydro pynchnc Mefloquinc

MRP2

Multidrug resistance associated protein-2 Methicillin resistant staphylococcus aureus M II M clanocyte stimulating hormone rnTOR Mammalian target of raparnycin Mtx 1ethotrexate mV millivolt MW Molecular weight

MRS A

NA NA0QI ADP NAD PH

oradrenaline -acetyl-p-benLoquinoneimine icotinamide adenine dinucleotide phosphate Reduced nicotinamide adenine dinucleotide phosphate -acetyl glucosaminc N-acetyl muramic acid onadrcncrgic noncholinergic N-acetyl procainamide Noradrenergic and specific serotoncrgic antidepressant NAT -accryl transferase CEP ational cholesterol education programme orcthindrone cnanthatc EE [P cutral cndopeptidasc (Neprolysin ) ET orepinephrinc transponer IAT Nuclear factor of activaated T-cell NFkB uclear factor KB NICE National Institute for I lealthandCarc Excellence (UK)

NIS NLEP NMDA

nNOS N RTI

PY R N-REM

RTI SA ID

NSTEM I \/TS

NV RDCP NYHA

Na (sodiu111)-10dide symporter ational leprosy eradication programme -melhyl-D-aspanate eural nitric oxide synthase Nonnucleoside reverse transcriptase 111h1bnor curopcptide-Y icotimc receptor on rapid eye movement (sleep) ucleoside revcrsc transcriptase inh1bnor ,onstero1dal antnnAammatory drug Non ST-segment elevation myocardial infarction Nuclcu, tractus solitarius National vector borne diseases control programme ew York Heart Association

Organic anion transporter Organic anion transponing polypeptide oc Oral contracepti, c OCD obsessive-compulsivedisorder OCT organic cation transponcr OD Once daily OPG Ostcoprotegerin OPV Oral poliomyelitis vaccine ORS Oral rehydration salt(solution) ORT Oral rehydration therapu

OAT OATP

PABA

PAC PAF

l'araamino benzoic acid Post antibiotic effect Platelet acti,ating factor

xv

ABBREVIATIONS PAH PAI-I 2-PAM PA N PAS PBI PBL PBl's PCA PCEV PCI PC PA PD PDE PE PEMA PEP PF PFOR PG PGI2 Pgp Pl PIG PIP, PKA PKC PLA PLC Pl. ph. pMD I PnG POMC PO V PP PPARy PPH PPI ppm PPNG PRA PrEP PRF PRIii PSVT PT PTCA PTH PTMA PTP PTSD PTZ PUVA PVRV

Pulmonary ancrial hypencnsion Plasminogen activator inhibitor- I Pralidoxime Primary afferent neurone Paraamino salicylic acid Protein bound iodine l'aucibacillary leprosy Penicillin binding proteins Patient controlled anaesthesia Purified chick embryo cell vaccine (rabies) Percutaneous coronary intervention Parachloro phenyla lanme Pa rkinsons·s disease Phosphodiesterase Pulmonary embolism Phenylethyl malonamide postexposurc prophylaxis Purkinjc fibre Pyruvatc: ferredoxin oxidoreductase Prostaglandin Prostacyclin l'-glycoprotcin Protease inhibitor Phosphatidyl inositol glycan Phosphat1dyl inositol-4,5-bisphosphate l'rotem kinase: cAMP dependent Protein kinase C phospholipase A Phospholipasc C Platelet phospholipid pressurized muhidosc inhaler Penicillin G Pro--opio mclanoconin Postoperative nausea and vomiting Panial pressure Paroxysome proliferator-activ ated receptor gamma Post panum haemorrhage Proton pump 111h1bitor Pan per million Penicillinasc producing N. gonorrhoeae Plasma rcnm activity Pre-exposure prophylaxis (of HIV ) Prolactin releasing factor Prolactin release inhibitory hormone Paroxysmal supra-ventricular tachycardia Proximal tubule Percutaneous transluminal coronary angioplasty Parathyroid hormone Phenyl trimethyl ammonium Post-tetamc potentiation Post-traumatic stress disorder pentylenetetrazol Psoralen-Uhra, iolct A Puri fied verocell rabies, acc ine

QID R

RAN K RAN KL RAS RBC RB P RC RCT RE REM RGS RIG RIMA rl RMP

R A R TCP

RP RTF RTKs RXR

RyR

s SA SABE s.c.

sec SCh SCID SERCA SERDs SERM SERT SGA SGLT SHBG SIAD H s.I SLC SLE SMON SNP SN-PC S -PR SN RI s.o.s. SIP SP SPF SPRM SPS

Four times a day Rifampin (Rifampicin) Receptor for activation of nuclear factor KB RANK ligand Rcnin-angiotcnsin system Red blood cc lls Retinol binding protein Respiratory centre Randomized clinical trial Reticuloendothelial Rapid eye movement(sleep) Regulator of G-protein synthesis Rabies immuneglobulin Reversible inhibitor of MAO-A Recommended international nonproprietary name Resting membrane potential Ribonucleic acid Revised ational Tuberculosis Control Programme Refractory period Resistance transfer factor Receptor tyrosine kinases Retinoid X receptor Ryanodinc receptor Streptomycin Sinoauricular (node) Subacute bacterial cndocarditis Subcutaneous Shon course chemotherapy (of tuberculosis) Succ111ylcholine Se, ere combined immunodeficiency disease Sarcoplasmic-endoplasmic reticular calcium ATPasc Selecti, e estrogen receptor dow n regulators Sclccuvc estrogen receptor modulator Serotonin transponer Second generation antihistaminic Sodium-glucose transpon er ex hormone binding globulin Syndrome of inappropriate ADH secretion Sublingual Solute carrier s ystemic lupus erythematosus Subacutc myclo-optie ncuropathy Single nucleotide polymorphism Substantia nigra-pars compacta substantia nigra-pars reticularis Serotonin and noradrenaline rcuptake inhibitor as required Sulfonamide • pyrimethamine Substance P Sun protection factor clecuvc progesterone receptor modulator Simple panial seizures

xvi

ESSENTIALS OF MEDICA L PHARMACOLOGY SR SRS-A SSG SSI SSRls STAT ST EM ! StK SU SULT SUR susp SVR SWD

sws syr

Sustained release Slow reacting substance of anaphylaxis Sodium s tibogluconate Surgical site infection Selective serotonin rcuptake inhibitors Signal transducer and activator of transcription ST-segment elevation myocardial infarction Streptokinase Sulfonylurca Sul fotrans ferase Sulfonyl urea receptor Suspension Sustained viral response Shift work disorder Slow wave sleep syrup

TRE TRH TSJ-1

TT TTS TX

u VA UDP UFH UGDP UGT USA ' UT UTI

Half life Triiodothyroninc Thyroxine Tablet Typhoid, paratyphoid A and B vacci ne Thick ascending limb (loop of Henle) Tubercle bacilli Thyroxine bindmg globulin Transcobalamin II Tricyclic antidepressants Tissue culture infectious dose 50% Therapeutic drug monitoring Three times a day Transferrin Triglyceride 6-Thioguanine Transforming growth factor B Tetrahydrocannabinol Tetrahydro folic acid Triethylene thiophosphoram ide Threonine Transient ischaemic allacks Tumour necrosis factor a Target organ damage Train-of-four Tissue plas minogen activator Thiopurine methyl transferase Total peripheral resistance Thyro id hormone receptor

Unit Unstable angina Uridine diphosphate Unfractionated heparin University group diabetic programme UDP-glucurono syl transfcrasc United Sta tes adopted name Urea transponer Urinary tract infection

V VAL VA P VDR VES VF VIP Vil VKOR VL VLDL VMA VMAT VRE VRSA VRUT VT VTE v\VF

Volt Volume of distri bution Val ine Ventilator associated pneumonia Vit D receptor Ventricular extrasystole Ventricular fibrillation Vasoactivc intestinal peptide Vitamin Vitamin K epox idc reductase Visceral leishmaniasis Very low density lipoprotein Vanillyl mandelic acid Vesicular monoaminc transporter Vancomycin resistant enterococci Vancomycin resistant Staphylacoccus aureus Va opressin regulated urea transponer Ventricular tachycardia Venous th.romboembolism Von Wi llebrand factor

WBC WCVs W IIO WPW

White blood cells Water channel containing vesicles World Health Organization wolff-parkinson-white syndrome

V

t½ T3 T4 tab TAB TAL TB TBG TCII TCAs TCID50 T DM TDS Tf TG 6-TG TGF-B THC T HFA ThioTEPA THR TIAs TNF-a TOD TOF I-PA T PMT t.p.r. TR

Thyroid hormone response element Thyrotropin releasing hormone Thyroid stimulating hormone Tubcreuloid leprosy Transdermal therapeutic system Thromboxane

XDR-TB

Extensively drug resistant-T B

z ze (syndrome)

Zollinger-Ellison (syndrome)

GENERAL PHARMACOLOGIICAL PRINCIPLES

Chapter

1

Introduction, Routes of Drug Administration

INTRODUCTION Pharmacology Pharmacology is the science of drugs (Greek:

Pharmacon-drug; logos-discourse in). In a broad sense, it dea ls w ith interaction of exogenously administered chemical molecules with living systems, and any single chemical substance which can produce a biological response is a ' drug'. Pharmacology encompasses all aspects of knowledge about drugs, but most importantly those that are relevant to effective and safe use of drugs for medicinal purposes. For thousands of years most drugs were crude natural products of unknown composition and limited efficacy. Only the overt effects of these substances on the body were rather imprecisely known, but how the same were produced was entirely unknown. Animal experiments, primarily aimed at understanding physiological processes, were started in the 18th century. These were pioneered by F. Magendie and Claude Bernard, w ho also adapted them to study effects of certain drugs. Pharmacology as an experimental science was ushered by Rudolf Buchheim who founded the first institute of pharmacology in 1847 in Germany. In the later part of the 19th century, Oswald Schmiedeberg, regarded as the 'father of pharmacology', together with his many disciples like J Langley, T Frazer, P Ehrli ch, AJ C lark, JJ Abel propounded some

of the fundamental concepts in pharmacology. Since then drugs have been purified, chemically characterized and a vast variety of high ly potent and selective new drugs have been developed. The mechanism of action including molecular target of many drugs has been elucidated. This has been possible due to prolifi c growth of pharmacology which forms the backbone of rational therapeutics. The two main divisions of pharmacology are pharmacodynamics and pharmacokinetics.

Pharmacodynamics (Greek: dynamis-power) - What the drug does to the body. This includes physiological and biochemical effects of drugs and their mechanism of action at organ system/subcellular/ macromolecular levels, e.g.-Ad renaline • interaction with adrenocep tors • G-protein mediated stimulation of cell membrane bound adenyly l cyclase • increased intracellular cyclic 3',S'AMP • cardiac stimulation, hepatic glycogenolysis and hyperglycaemia, etc. Pharmacokinetics (Greek: Kinesis-movement)- What the body does to the drug. This refers to movement of the drug in and alteration of the drug by the body; inc ludes absorption, distribution, binding/localization/storage, biotransformation and excretion of the drug, e.g. paracetamol is rapidly and almost complete ly absorbed orally attaining peak blood levels at 30- 60 min; 25% bound to plasma protei ns,

2

GENE RAL PHARMACOLOGY wide ly and almost uniformly distributed in the body (vo lume of distribution - IL/kg); extens ively meta bo lized in the Ii ver, pri maril y by glucuron ide and sulfate conj ugation into inactive metabo li tes w hich are excreted in urine; has a plasma half life (t½) of 2-3 hours a nd a clearance value of 5 ml/kg/min.

o f drugs a nd co m parative tr ia ls w ith ot her forms of treatment; surveillance of patterns of drug use, adverse effects, etc. are a lso part of clinical pharmacology. The aim of clinical pharmacology is to generate data fo r optimum use of drugs a nd the practice of 'evidence based med icine '.

Drug (French : Drogue- a dry herb) It is the single active chemical entity present in a medicine that is used for diagnosis, prevention. treatment/cure of a disease. T his d isease oriented defi ni tion of drug does not include contraceptives or use of drugs for improvement of health. The WHO ( 1966) has given a more comprehensive defi nition- "Drug is any substance or product that is used or is intended to be used to modify or explore physiological systems or pathological states for the benefit of the recipient."

Chemotherapy

The tenn ' drugs' is being a lso used to mean addi ctive/abused/ i Ilicit substances. However, th is restri cted and derogatory sense usage is unfortunate degradation of a time honoured term, and 'drug' should refer to a substance that has some hea lth promoting/ therapeutic/ diagnostic applicati on. Nevertheless, to avo id any misinte rpretation, the term ' med icine' is being employed to designate such a substance in place of the term ' drug' . Some other important aspects of pharmacology are:

Pharmacotherapeutics It is the application of pharmacological information togethe r with know ledge of the disease fo r its prevention, mitigation or cure. Selection of the most appropriate drug, dosage and duration of treatment taking into accoun t the stage of disease a nd the specific features of a patient are a part of pharmacotherapeutics. Clinical pharmacology

It is the scientific study of drugs (both old and new) in man . It incl udes pharmacodynam ic and phannacokinetic investigation in healthy volu nteers as well as in patients. Eva luation of efficacy and safety

It is the treatment of systemic infection/ malignancy w ith specific drugs that have selec tive toxicity for the infecting organism/ malignant cell w ith no/ minimal effects on the host cells. Drugs in general, can thus be divided into:

Pharmacodynamic agents These are designed to have pharmacodynam ic effects in the recipient. Chemotherapeutic agents These are des igned to inhibit/kill invading parasites/ mal igna nt cell, but have no/ minimal pharmacodynamic effects in the rec ipient.

Pharmacy It is the art and sc ience of compounding and dispensing drugs or preparing suitable dosage forms for administration of drugs to man or anima ls. It incl udes collection, identification. p urification, isolation, synthesis, standardization and quality control of medicinal substances. The large scale manufactu re of drugs is called Pharmaceutics, which is primaril y a technological science. Toxicology

It is t he study of poisonous effect of drugs and other c he micals (household, environmenta l pollutant, industrial, agricultural, hom ic idal) wit h em phasis on detectio n, prevention and treatment of po isonings. It a lso includes the study of adverse effects of drugs, since the same substance can be a drug or a poison, depending on the dose.

NATURE OF DRUGS All drugs are chem ical entities wi th simple or complex molecules. While majority are orga ni c compounds, some are purely inorganic, like lithium carbonate, ferrous su lfate, magnesiu m hydroxide, etc. Organic drugs may be weakly

INTRODUCTION, ROUTES OF DRUG ADMINISTRATION

acidic (aspirin, penicillin) or weakly basic (morphine, chloroquine) o r no nelectrolytes (alcohol, diethyl-ether). Most drugs are no m,a lly sol ids, e.g. paracetamol, propranolol, furosemide, ampici Ilin, etc., but some such as ethanol, glyceryl trinitrate, propofol. castor oil a re liq uids, and few like nitrous oxide are gaseous. The molecular weight of majority of drugs fa l Is in the range o f I 00- I 000 D, because molec ules smaller tha n I 00 D do not generally have suffi c iently specific features in terms of shape, size, configuratio n, chirali ty, distribution of charges, etc. to selectively bind lo only one/ few closely re lated target biomolecules, to the exclusion of others. On the other hand, larger molecules than I 000 D do not readily pass through membranes/barriers in the body to reach the target sites in various tissues/cells. However, few drugs are as small as lithium ion (7 D), and some like heparin ( I 0-20 KO), gonadotropins (> JO KO), enzymes, protei ns, antibod ies (>50 KD) are much bigger. Bulky mo lecule drugs have to be adm inistered parenterally. Drugs are generally perceived to be chemical substances foreign to the body (Xcnobio ti cs). However, many endogenous chemica ls like hormones, autacoids, metabolites and nutrients are a lso used as drugs. Chemical congeners of these metabolites/signal molecules are an important class of drugs which act by modifying the synthesis, storage, degradation o r action of these metabolites/signal molecules.

SOURCES OF DRUGS Drngs are obtained from a variety of sources: 1. Plants Many plants co nta in biolog ica ll y active substances and a re the oldest source of drugs. Clues about med ic ina l plants were obtained from traditiona l systems of med icine prevalent in various pans of the world; e.g. use of opium , belladonna, ephedra. cinchona. c urare, foxg love, sarpagandha, qinghaosu has been learnt from Egyptian, Greek, Aztec. Ayurvedic, Chinese and o ther systems of medicine. Chemically the acti ve ingredients o f plants fall in several categories:

a. Alkaloids: These are al kal ine nitrogenous bases having po tent activity, and are the most im portant category of vegetab le origin drugs. Promi ne nt examples are: mo rph ine, atropine, ephedrine, ni cotine, crgotamine, reserpine, quinine, vincristine , etc. They are mostly used as the ir water so luble hydrochloride/ sulfate salts. b. Clycosides: These compounds consist of a heterocyclic nonsugar moiety (aglycone) li nked to a suga r moiety through ethe r lin kage. Card iac g lycos ides (digoxin , ouabai n) a re the best known glycosidi c drugs. The active principle of senna a nd similar plant purgatives are anthraquinone g lycos ides. Am inoglycosides (genta mic in, e t c.) a re an ti bio t ics o btained from microorganisms. and have an aminosugar in place of a sugar moiety. c. Oils: These arc viscous, inflammable liquids, insolub le in water. Fixed (nonvo lat ile) oils a re calorie yielding triglycerid es of hig her fatly acids; mostly used fo r food and as emo ll ients, e.g. groundnut oi l, coconut o il, sesame o il, etc. Castor oil is a stimula nt purgative. Essential (volatile) oils, mostly obtained from flowers or leaves by steam distillation are aromatic (fragrant) terpene hydrocarbons that have no food val ue. T hey are used as flavouring agents, cam, inatives, counterirritants and astringents; examples are eucalyptus oil, pepermint oil, nilgiri o il, etc. Clove oil is used to a l lay dental pain. Menthol, thymo l, camphor are vo latile oils that are solids at room temperature. Mineral oils are not plant products, but obtained fro m petroleum; liq uid paraffin is a lubricant laxative, soft and hard pa raffin are used as emoll ient and as o intment bases. Other plant prod ucts like tanins are astringent; gums are demulcents and act as suspending agents in liquid dosage forms. G lyceri ne is a viscous, sweet liquid used as vehicle for g um/throat pai nt. Resins and balsams are used as anti septic and in

3

4

GENERAL PHARMACOLOGY cough mixtures. The anti malarial drug artemisinin is a sesquiterpene endoperoxide obtained from a Chinese plant. 2. Animals Though animal part have been used as cures since early times, it was exploration of activity of organ extracts in the late 19th and earl y 20th century that led to introduction of animal products into medicine, e.g. adrenaline, thyroxine, insulin, liverextract(vit. 8 12).Antisera and few vaccines are also produced from animals. 3. Microbes Most anti bi ot ics are obta ined from fung i, acti nomycetes and bacteria, e.g. penic illin, gentamici n, te tracycli ne, crythromycin, polymyxin B, actinomycin D (anticancer). Some enzymes, e.g. diastase from a fungus and streptokinase from streptococci have a microbial source. Vaccines are produced by the use of microbes. 4. Minerals Few minerals, e.g. iron salts, calcium salts, lithium carbonate, magnesium/ a luminium hydroxide, iodine are used as medicinal substances. 5. Synthetic chemistry Synthetic chem istry made its debu t in the 19th century, and is now the largest source of medicines. Synthetic drugs have the advantage of purity and uniform ity of the product. They can be manufactured in any quantity as per need, in contrast to drugs from natural sources whose availability may be limited. ot only diverse congeners of nat ura lly obtained drugs (atropine substitutes, adrenergic B2 agonists, synthetic gl ucoco1ticoids/progestins/ cepha losporins, etc.) have been introduced to achieve greater selectivity of actio n or even novel type of activity, but many entirely synthetic famili es of drugs, e.g. benzodiazepines, thiazides, benzimidazoles, fluoroquinolones, etc. have been produced. Many drugs are being synthesized to target specific biomolecules, e.g. ACE inhibitors, glycoprotein lib/ I Ila receptor antagonists, HIV-reverse transcriptase inhibitors, etc. Synthetic drugs that are chiral can also be produced as single acti ve enantiomer products, which may be therapeutically superior.

6. Biotechnology Several drugs, especia ll y peptides and proteins are now produced by recombinant D A technology, e.g. human growth hormone, human insulin, altaplase, interferon, e tc. Monoclonal an tibodies, regulator peptides , erythropo ietin and other growth factors are the newer drugs of biotechnological origin. Protein therapeutics is rapidly expanding, because specifica lly designed and customized proteins can now be produced. DRUG NOMENCLATURE

A drug generally has three categories of names: (a) Chemical name It describes the substance chemically, e.g. l -(Isopropylamino)-3( 1-naphthyloxy) propan-2-ol fo r propra nolol. This is cumbersome and not suitable for use in prescribing. A code name, e.g. RO 15-1 788 (later named flu mazenil) may be assigned by the manufacturer fo r convenience and simplicity before an approved name is coined. (b) Non-proprietary name It is the name accepted by a competent scientific body/ authority, e.g. the United States Adopted ame (USA ) by the USA counci l. Similarly, there is the British Approved name (BA ) of a drug. The non-proprietary names of newer drugs arc kept un iform by an agreement to use the Recommended International onproprietary ame (rlNN) in a ll member countries of the WHO. The BAN of older drugs as well has now been modified to be commensurate with r!NN. However, many older drugs still have more than one non-proprietary names, e.g. 'mepcridine' and 'pethidine' or ' lidocaine· and 'lignocaine' for the same dru gs. Until the drug is included in a pharmacopoeia, the nonproprietary name may also be called the approved name. After its appearance in the official publication, it becomes the official name. In co mmon parlance, the term generic name is usecl in place of non proprietary name. Etymologically this is incorrect: 'generic '

INTRODUCTION, ROUTES OF DRUG ADMINISTRATION should be applied to the chemical or pharmacological g roup (or ge nus) of the compo und, e.g. phe noth ia z incs, tricycl ic antidepressants. aminoglycos ide a nt ibiotics, etc. However, thi s misnomer is w ide ly accepted and used even in official parlance. (c) Proprietary (Brand} name It is the name assigned by the manufacturer(s) and is hi s property or trade ma rk. One dru g may have multip le proprietary names, e.g. AMCARD. AMLOGARD. AMLOCOR

AMLO G1 AMLOPI ',

AMLOVAS, STAMLO for amlodipine from d ifferent

manufacturers. Bra nd names are designed to be catc hy, short, easy to remember and often suggestive, e.g. LOPRESOR s uggesting d rug for lowering blood pressure. Brand names generally differ in d ifferent countries . e.g. timo lo l maleate eye dro ps are marketed as TIMOPTIC in USA but as GLUCOMOL in India. Even the same manufacturer may market the same drug unde r different brand names in different cou ntries. In addition, combined fo rmulatio ns have their own multiple brand names. Th is is responsible for much confusion in d rug nomenc lature. There are many arg uments for us ing the nonproprietary name in prescri bing: uni formity, convenience, economy and better comprehension (propranolol, sotalol, timo lol, pindolo l, metoprolol. acebutolol, atenolol are a ll B blockers, but their brand names have no such sim ilarity). Drugs marketed un der no npro prieta ry na me (ca lled generic products) are mu ch chea per than the ir branded counterparts, partl y beca use the manufacturer invests a lo t of money in promo ting the brand name. However, when a drug is presc ribed by the generic name, the chemist is free to dispense the generic produc t from any manufacturer, but not so w hen the d rug is prescribed by a brand name. T hus, w hen it is importa nt to ensure cons istency of the product in terms of qua lity and bioavailability, etc. and especiall y when officia l control over qua lity of ma nufactured produ cts is not rigoro us, it is better to prescribe by the dependab le brand name.

DRUG COMPENDIA These are compi lation s of information on drugs in the form of monographs; w itho ut going into the theoretical concepts, mechanisms of actio n and other aspects which he lp in understanding the subject. Pharmacopoeias and Formularies are broughto ut by the Government in a co untry, hold legal status and are called offic ial compendia. In addition, some no n-official compendia are pu blished by professio na l bodies, which are supplementary and dependable sources of in formation a bo ut drugs.

Pharmacopoeias They contain descriptio n o f chemical structure molecular weight, physica l and chemical characte risti cs, solubi lity, identificati on and assay methods, standards of purity, storage conditions and dosage forms of offic ially approved drugs in a country. They are usefu l to drug manufacturers and regulatory authorities, but not to doctors, most of w hom ne ver see a p harmacopoeia. Exa mp les are Indian ( IP), British ( BP), European (Eur P), United States (US P) pham,aco poeias. Formularies G e ne rally produced in eas il y carri ed booklet form , they list indications, dose, dosage form s, con traindi catio ns, precautions, adverse effects and storage of selected d rugs that are available for medicinal use in a country. Drugs are categorized by their therapeutic class. Some rat iona l fi xed-dose drug combin ations a re inc luded . A bri e f co mm e ntary on th e drug c lass and c lini cal conditi o ns in w hic h they arc used g ene ra lly precedes spec ifics of indiv idual drugs. Brie f guide lines for treatm ent of selected conditions arc provided. While British ationa l Formula ry (B F) al so lists brand na mes w ith costs, the a tional Formul ary o f India (N FI) docs not incl ude these. Most formularies have in formative appendices as well. Formu la ri es can be considerably helpful to presc ribers. Martindale: The Complete Drug Reference (Extrapharmacopoeia) Pu blished e very 2 3 yea rs by the Royal Pharmaceu ti cal Society of G reat Britain, this no n-official compend ium is

5

6

GENE RA LPHARMACOLOGY an exha usti ve and updated compilation of unbiased information on medicines used/registered all over the world. Jt inc ludes new launches a nd contai ns pharm aceutical, pharmacological as well as therapeutic informati on on drugs, w hich can serve as a reliable reference book.

Physicians Desk Reference (PDR) and Drug: Facts and Comparisons (both from USA), etc. are other useful non-officia l compendia.

ESSENTIAL MEDICINES (DRUGS) CONCEPT The WHO has defined essential Medicines (drugs) a s " those t ha t sati sfy the p rior ity healthcare needs of the populatio n." They are selected with d ue regard to public health relevance, ev id ence on efficacy and safety, a nd comparative cost effectiveness. Essential med icines are intended to be available within the context of functioning health systems at a ll times and in adequate amoun ts, in appropriate dosage fom,s, with assured quali ty and adequate informa tion, and at a price the indi vidual and the community can afford. It has been reali zed that only a handful of medic ines out o f the multitude available can meet the health care needs of majority of the people in any cou ntry, and th at many we ll tested a nd c heaper medic ines are equa lly (or more) efficacious and safe as their newer more expensive congeners. For optimum utilization o f resources, governments (especially in developing countries) should concentrate on these medicines by ide ntifying them as Essential medicines. The W HO has laid down criteria to guide selection of an essential medic ine.* (a) Adequate data on its efficacy and safety should be ava ilable from cli nica l studies. (b) It should be available in a form in which quality, including bioavai lability, and stability on storage can be assured. (c) Its choice shou ld depend upon pattern of preva le nt diseases; ava ilability of fac ilities and trained personnel;

financ ial resources; genetic, demographic and environmenta I factors. (d) In case of two or more similar medici nes, c ho ice should be made on the basis o f their relative efficacy, safety, quality, price and avai lability. Cost-benefit ratio should be a major consideration. (e) Choice may also be inf1uenced by comparative pham,acokinetic properties and loca l fac ilities for manufacture and storage. (f) Most essential medicines shou ld be single compounds. Fixed ratio combination products should be included only when dosage of each ingredient meets the require me nts of a defined population group, and when the combination has a proven advantage in therapeutic effect, safety, adherence or in decreasing the emergence of drug resistance. (g) Selection of essential medicines should be a continuous process which should take into account the changing priorit ies for public health action, epidemiologica l conditions as well as availability of better medicines/formulations and progress in pharmacological knowledge. (h) Recently, it has been emphasized to se lect essential medicines based on rationally developed treatment guidelines.

To g uide the member countries, the WHO brought out its first Model List of Essential Drugs alo ng wi th thei r do sage forms a nd strengths in 1977 w hi ch could be adopted after suitable modifications according to local needs. This has been revised fro m time to time and the current is the 20th list (2017)s w hich has 433 medicines, includ ing 25 fi xed dose drug combinations ( FDCs). India produced its National Essential Drugs List in 1996, and has revised it in 2011, and now in 20 15 w ith the titl e "National List of Essential Medicines".£ The latest list includes 376 medi cines, of which 20 are F D Cs. These m ed ic ines ha ve been marked into 3 categories for being avai Iab le at prima ry, seconda ry and tertiary levels of health care facility. Adoptio n of the essentia l medic ines list for procurement and s upply o f medicines, especially in the pub Iic sector heal thcare system, has resulted in improved availability of medicines, c ost saving and more rat io nal use of drugs .

*The use of Essential Drugs (including the 8th model list of essential drugs); WHO Technical report series 850, 1995, Geneva. 5 www.who.int>20'h--esscntial med-list (pub. 6 Jun 20 I 7). £National list of Essential Medicines (2015) [http//www.cdsco.nic.in]

INTRODUCTION, ROUTES OF DRUG ADMINISTRAT ION

Prescription and non-prescription drugs As per drug rules, majority of drugs including all antibiotics must be sold in retai l only against a prescription issued to a patient by a registered med ical practitioner. These are called ' prescription drugs ', and in India they have been placed in the schedule H of the Drugs and Cosmetic Rule s ( 1945) as amended from time to time. However, few drugs like simple analgesics (paracetamol aspirin). antacids, laxatives (senna, lactulose), vitamins, ferrous salts, etc. are considered relati vely harmless, and can be procured without a prescription. These are ' non-prescription ' or ' over-the-counter' (OTC) drugs; can be sold even by grocery stores. Orphan Drugs These arc drugs or biological products for diagnosis/treatment/ prevention of a rare d isease or condition. or a more common disease (endemic only in resource poor countries or areas) for which there is no reasonable expectation that the cost of de,eloping and marketing it will be recovered from the sales of that drug. As per Orphan Drug Amendment ( I 983) Acl o f USA, a rare d isease/ condition is one that affects less than 0.2 mill ion people in the USA. Though these drugs may be life saving for some patients, they are commercially difficult to obtain as a medicinal product. Governments in developed countries offer tax benefits and other incenti,·es to pharmaceutical companies for developing and marketing orphan drugs. Orphan dmg starus has been awarded to many dmgs in the USA, Europe and some other countries. Few examples o f dmgs granted ·Orphan drug status arc listed in the box. Abridged list of Orphan drugs Azacitidine

lca tiba n t

Bevacizumab

lloprost

Bo rtezomib

Nilotinib

Bus u lfan

Paromomycin

Carbop rost

Rifaximin

Clo fazimine

Rituximab

Colchicine

Sodium stibogluco nate

Eltro mbopa g

Sodium thiosu lfa te

Fo mivirs e n

ThioTEPA

DOSAGE FORMS OF DRUGS Dosage fonn is a product suitable for administration of a drug to a patie nt. Every active ingredient (dru g) has to be formulated by adding o ther substances (excipients, diluents, preservatives,

vehic les, etc.) accord ing to a specific recipe and packaged into a specific 'dosage form' such as tablet, elix ir, ointment. injecti on vial, etc. w hi ch is then administered to the ubject. The dosage form provides body to the drug. demarkates si ngle doses, protects the active ingredient(s), and makes it suita ble for admi nistrati on in vari ous ways. The important dosage forms are briefly described below.

Solid dosage forms 1. Powders The drug is in a dry and finel y pulveri sed state. If the drug is for o ral admin istration, each dose has to be wrapped separately or packed in sachets; therefo re this dosage form is inconvenient a nd unpopular except w hen the quantity is several grams. e.g. oral rehydration salts. Powders for topical appl ication (dusting powders) are supplied as bulk powders in metallic or plastic containers with holes for sprinkling. Effervescent powders conta in granulated sod. bicarbonate and citric or tartaric acid. They react when dissolved in water to liberate CO2 causing bubbling. 2. Tablets The drug is powdered or granu lated, m ixed w ith bindin g agents , and other cxcipien ts, a nd compressed/ moulded into discoid, oblong or other shapes suitab le fo r swallowi ng. The ta blet may be plain or suga r coated or fi lm coated. Other specialized types of tablets are: Che wable table ts -can be c h ewed a nd swal lowed , ingred ients must be p leasent tasti ng. Dispersible rablets- the table t is dropped in a small quantity of water, wherein it disperses quickly ; the solution is then gulped. Sublingual tablets-put under the tongue, the drug is rapidly absorbed from the mouth. Enteric coated tablet- the tablet is coated with a mate rial that does not di ssolve in the acidic medium of the stomach; the tablet disintegrates only on reaching the duodenum.

Sustained/ Extended release tablets- These contain drug particles which are coated to

7

8

GENERAL PHARMACOLOGY dissolve at different rates. The active ingredient is made available for absorption over a lo nger period of time. T he durati on of action of s hort acting (2-6 hours) drugs ca n be extended to 12 hours o r more. Controlled release tablets- A semipermeable me mbrane controls the release of the drug prolong ing its duration of action.

2.

3. Pills These are archaic dosage forms in which the drug powder is mixed with honey/ syrup to make a sticky mass. This is then ro lled into spherical/oval bodies meant to be swall owed. The te rm is often loosely appl ied to tabl ets as well.

4. Capsules These are water so luble cylindrical containers made of gelatin which are fi ll ed w ith powdered or liquid medicament. The container dissolves on swallowing so that the drug is released in the sto mach. Soft ge lati ne capsules dissolve very rap idly a nd generally contain liquid medicament. Enteric coated capsules are designed to dissolve only o n reaching the ileum . Spansules are extended release capsules w hi ch are packed w ith gra nu les of the drug hav ing different coatin gs to disso lve over a range of time periods. 5. Lozenges These are tab let-like bodies o f various shapes containing the drug along with a s uitable g um , sweetening a nd Aavo uring agents. They are to be retained in the mouth and all owed to di sso lve slowly, provid ing the drug for loca l actio n in the mouth and throat. 6. Suppositories These are conical bullet- shaped dosage forms for insertion into the ana l canal, in which the drug is mixed with a mou ldable firm base that me lts at body temperature and releases the contained d rug. Oval or suitably shaped bodies for vaginal insertion are ca ll ed 'pessaries', whi le e longated penci1-1ike cones meant fo r insertion into male or fema le urethra are called bougies.

Liquid dosage forms 1. Aqueous solutions They contain the drug d issolved in water, a nd may be meant for

3.

4.

5.

oral, topical o r pare ntera l administ ration. Oral drug solutions often conta in sweetening and flavouring agents. Preservat ives have to be mostly added because shelf-life of watery solutions is short. Suspensions are dis pers ion of insolub le drugs in water with the help of a suspend ing agent. Emulsions are uniform mi xtures of two imm iscible liqu ids (mostly oil and water) in wh ich droplets of one (di spersed phase) are s us pended in the othe r (co ntinuo us phase) w ith the hel p of an amphiphili c emul sifying agent. Milk is a naturally occurring em ulsion. Both suspensio ns and emulsio ns tend to settle down o n keepi ng; should be s haken thoroughly before use. Elixirs are hydro-alcoholic sol utio ns of drugs, usually sweetened with syrup and flavoured by fruit extracts. Syrups have higher concentration of sugar and a re thicker in consistency. Drugs that deteriorate in aqueous med ium can be dispensed as 'dry syrups' wh ich is reconstituted by adding water and s haking. The reconstituted syrup must be used w ithin a few days. Linc/us is a viscous syrupy li quid meant to be licked slowly for soothing the throat. lt generally has mentho l to impart cooling sensation, and an antitussive. Drops These are relatively more concentrated so lut ions of medicame nts mea nt for oral ingestion or external application to eye, nose or ear canal. Oral drops are the prefe1Ted dosage form for infants and young ch ild re n. Eye/ nasal drops should be isotonic . Eye drops need steri lization. Drops are s uppl ied in vials w ith a nozzle o r alongwith a dropper fo r accurate dosing. Lotions T hese are solutions, suspensions or emulsions meant for external application to the skin without rubbing. They genera lly have soothing, cool ing, protective or emoll ient property and are better suited than creams o r o intments for hairy s kin. Liniments are simi lar preparations which gene rally contain cou nteri rritants, and are to be rubbed on the skin to relieve pain and cause rube faction.

INTROD UCTION, ROUTES OF DRUG ADMINISTRATION

6. Injections T hese are sterile solut ions or suspensions in aqueous or o ily medium for subcutaneous or intramuscular ad ministrati on. On ly aq ueous solutions (not suspensions) are suitable fo r intravenous (i.v.) inj ecti on, because particles in s uspension and oils injected i.v. can cause embolism. Injections are suppl ied in sea led glass ampoules or air tight rubber capped vials. Ampou les are broken just before injection, and usually contain a single dose. Drug from the vial is sucked in a syrin ge by p ie rc ing the rubber cap. Vials may be single or multi-dose. Drugs which are un stable in solution are supplied as dry powder vials. Sterile solvent is injected in the vial just before it is to be administered, and the dissolved/ suspended d rug is the n sucked out into the syringe. Some drugs like insulin are also supplied in prefilled syringes and pen injectors. Large volume i.v. infusions a re marketed in glass/polypropylene bottles.

Semisolid dosage forms 1. Ointments These are g reasy se m iso lid preparations meant fo r externa l application to the skin, eye, nasal mucosa, ear or anal canal. The drug is incorporated in an oily base, such as soft or hard paraffin, wool fat, bee's wax, etc. Ointments are not suitable fo r oozing surfaces, because they are more occlusive and do not allow evaporation of water. Rather they are good for dry, chronic lesions. Creams are similar to ointment but the base is a water in oil emuls ion. The medicament is be tter absorbed into the skin from creams than from oi ntments, and creams are cosmetically more acceptable than ointments. 2. Pastes These are nongreasy preparations of thick consistency containing hydrophilic adhesive powders such as starch, pre pared chalk, aluminium/ magnesium hydrox ide, zinc oxide, carboxy methy lcel lulose, etc. which swell by absorbing water. Pastes may contai n viscous nonoily liqu ids like glycerol or propylene glycol. Pastes can be applied to infl amed or excoriated ski n, oozing surfaces

and mucous membranes. Toothpastes are items ofpersonal hygiene, and medicated toothpastes are extensive ly used in dent istry. 3. Gels The medi ca ment is incorporated in a v isco us co llo idal solutio n of gela tin or similar materia l and is usually d ispensed in collapsible tubes. They are meant for external application to the skin or mucosa and provide longer duration contact, but are nong reasy and washable with water. Gels are suitable for appl icat ion to hairy skin, a nd a re common ly applied to oral ulcers because they are bette r retained than aqueous solutions.

Inhalations Drugs w hich are gases or volatile liquid s can be a dministered by inha lation carried into a ir or oxygen wi th the help ofa mouth piece, face mask, hood or endotracheal tube. Nonvolatile liq uids and fine pa1iicle solids can be aerosoli zed using a metered dose inhaler, jet nebulizer, rotahaler or spinha le r fo r inha lation through the mo uth. Pressurized metered dose inhalers ( PMDl s) are hand-held d evices which use a prope llant, mostly hydrofluoroal kane ( HFA), and de liver a specified dose of the drug in aeroso l fo rm per actuation. Jet nebulizers produce a mist of the drug solution generated by pressurized air o r oxygen. Rotahaler is also a portable device in wh ic h a capsul e (rotacap) containing very fine powder of the drug is punctured during actuation and the released particles are aerosolized by the inspiratory ai rfl ow of the patient. A prope llant can also be used in some spin halers. Efficacy of the aerosolized drug depends on the particle size: 1- 5 µm diameter particles deposit on the bro nc hio les and effectively de liver the drug. Larger particles settle on the oropharynx, wh ile < l µm particles donot settle anywhere and a re exhaled out.

ROUTES OF DRUG ADMINISTRATION Most drugs can be administered by a variety of routes. The choice of appropriate route in a g iven s itua tion depe nd s both o n drug as well as patient related factors. Mostly common

9

10

GENERAL PHARMACOLOGY sense considerations, feas ibility and convenience dictate the route to be used. Ro utes can be broadly d iv ided in to those for (a) Local action and (b) Systemic acti on. Factors governing choice of route 1.

2.

3. 4. 5. 6. 7.

Physical and che mical properties of the drug (solid/liquid/gas; solubility, stability, pH, irritancy). Site of desired action- localized and approachable or generalized and not approachable. Rate and extent of absorption of the drug from different routes. Effect of diges tive juices and firs t pass metabolism on the drug. Rapidity with which the response is desired (routine treatment or emergency). Accuracy of dosage requi red (i.v. and inhalational can provide fine tuning). Condition of the patient (unconscious, vomiting).

LOCAL ROUTES These routes can on ly be used fo r localized lesio ns at accessible s ites an d fo r drugs w hose syste m ic absorptio n from these si tes is m inima l o r absent. Thus, hig h concentrations are a ttained at the desired s ite without exposing the rest of the bo dy. Systemic side effects or tox ic ity are consequ ently a bsent or mi nimal. For drugs ( in suitable dosage forms) that are absorbed fro m these sites/routes, the same can ser ve as systemic route of admini stration, e.g. g lycery l trinitrate (GTN) applied on the skin as o intment or transd erma l patch fo r angina pectoris. The loca l routes are :

1. Topical This re fers to external application o f the drug to the s urface for localized action. It is often mo re con ven ient as well as reass uring to the patient. Drugs can be effi c iently d eli ve re d to th e local ized les io ns o n skin , oropharyngeal/ nasal mucosa, eyes, ear cana l, a na l c an a l o r vagin a in the fo rm of lotio n, o intment, cream, powder, rinse, paints, drops, spray, lo ze ngens, s uppos ito r ies or pesseri es. Nonabsorbable drugs g iven o ra ll y for action on g.i. mucosa (sucralfate, vancomycin), in halation of drugs for action on bronc hi (salbutamo l,

crom oly n sodium) and irrigating so lutions/jell ys (povidone iod ine, lidoca ine) appl ied to ure thra are other fo rms of topica l medicati on. 2 . Deeper tissues Certain deep areas can be approached by us ing a syringe and need le, but th e drug s ho uld be in such a fo rm that systemic absorptio n is s low, e.g. intra-art icular injection (hyd rocortisone acetate in knee j oint), infiltration around a nerve o r intratheca l injection (lidocaine), retrobu lbar inj ectio n (hydrocorti sone acetate behind the eyeba ll).

3. Arterial supply

C lose intra-arterial inj ection is used fo r contrast media in angiography; anticancer drugs can be infused in femoral o r brachia! artery to localise the effect for lim b malignancies_

SYSTEMIC ROUTES T he drug admi nistered throug h system ic routes is intended to be absorbed into the blood s tream a nd distributed a ll over, including the s ite o f action, throug h circ ulatio n (see Fig. I. l ).

1. Oral Oral ingest ion is the o ldes t a nd commo nest mo de of drug adm inistration. It is safer, more convenient, does not need assistance, nonin vas ive, often pa inless, the medicament need not be sterile and so is cheaper. Both solid dosage forms (powders, tablets, capsules, spansu les, dragees, moulded tablets, gas trointestinal therape utic systems-GITs) and liq uid dosage fo1ms (elix irs, syrups, emul sions, mixtures) can be given ora lly. Limitations of oral route of administration

• Action of drugs is s lower and thus not suitable for emergencies. • Unpalatable drugs (chloramphenicol) are difficult to administer; drug may be filled in capsules to circumvent this. • May cause nausea and vomiting. • Cannot be used for uncooperative/unconscious/ vomiting patient. • Absorption of the drug may be variable and erratic; certain drugs are not absorbed (streptomycin). • Others a re destroyed by digestive juices (penicillin G, insulin) or in liver (GTN , testosterone, lidocaine).

INTRODUCTION , ROUTES OF DRUG A DMINI STRAT ION

First pass metabolism in lungs (minor extent)

Intravenous injection

--- -- -- - First pass metabolism in intestinal wall

- 50% subjected to first pass metabolism Suppository Fig. 1.1: Vascular pathway of drugs absorbed from various systemic routes of administration and sites of first pass metabolism Note: Total drug absorbed orally is subjected to first pass metabolism in intestinal wall and liver, while approximately half of that absorbed from rectum passes through liver. Drug entering from any systemic route is exposed to first pass metabolism in lungs, but its extent is minor for most drugs.

11

12

GENERAL PHARM ACOLOGY 2. Sublingual (s.1.) or buccal T he tablet or pellet containing the drug is placed unde r the tongue or crushed in the mouth and spread over the buccal mucosa. Only lipid soluble and non-i rritating drugs can be so adm inistered. Absorption is relati ve ly rapid- action can be produced in minutes. Though it is somewhat inconvenient, one can spit the drug a fter the des ired e ffect bas been obtained. T he chief advantage is that liver is bypassed and drugs w ith high first pa s metabo lism can be absorbed directly into systemic circ ulation . Drugs given su bl ingually are-GTN, buprenorphine, desamino-oxytocin.

3 . Rectal Certain irritant and unpleasant drugs can be put into rectum as suppositories or retenti on enema for system ic e ffect. T his ro ute can a lso be used when the pa tient is having recurrent vomiting or is unconscious. However, it is rather inconvenient a nd e mbarrassing; absorption is slower, irregu lar a nd often unpredictab le, though di azepam so lution and paraceta mol suppository are rapidly and dependably absorbed from the rectum in children. D rug absorbed into external haemorrhoidal veins (about 50%) bypasses liver, but not tha t absorbed into internal hae morrhoida l veins. Recta l infla mmation ca n result from irritant drugs. Diazepam, indomethacin, paracetamo l, e rgotam ine and few other drugs are some times given rectally.

4. Cutaneous Highly lipid soluble drugs can be app lied over the sk in fo r s low a nd prolonged absorpti o n. T he liver is a lso bypassed. T he drug can be incorporated in an ointment and applied over specified area of sk in. Absorption of the drug can be enhanced by rubbing the preparation, by using an oily base a nd by an occlusive dressing.

bound to a po ly mer) is he ld in a reservo ir between an occlusive bac king fi lm and a rate con tro ll ing micropore me mbra ne, the under surface of whic h is smeared with an adhesive impregnated w ith priming dose of the drug. The adhesive layer is protected by another film that is to be peeled off j ust before appl ication. T he drug is delivered at the skin su1face by diffusion for percutaneous absorption into c irc ulation. T he mic ropore membrane is such that rate of drug de livery to skin surface is less than the slowest rate of absorption from the ski n. This offsets any variation in the rate of absorption according 10 the properties of dilTerent sites. As such, the drug is de livered at a constant and predictable rate irrespective o f site of application. Usually c hest, abdomen, uppe r arm, lower back, buttock o r mastoid region are ut ilized. Transdermal patches of GTN, fentanyl, nicotine and estrad iol are avai lable in Ind ia, while those of isosorbide d initrate, hyoscine, and c lonidine a rc marketed e lsewhe re. For different drugs, TTS have been designed to last for 1- 3 days. T hough more expensive, they prov ide smooth plasma concentrations of the drug w ithout fluctuations; mi ni mize interi nd ividual variations (drug is subjected to little first pass metabolism) and side effects. They are a lso more conven ient- many patients prefer transderma l patches to oral tablets of the same drug; patient compliance is better. Local irritation and erythema occurs in some subjects, but is generally mild: can be minimized by changing the site of application each time by rotation.

Transdermal therapeutic systems (TTS) T hese are devices in the form of adhesive patches of vario us shapes and sizes (5 20 cm2) w hich de li ver the contained drug at a constant rate into syste mic c ircu lation via the stratum corneum (Fig. 1.2). The drug (in solution or

Fig. 1.2: Illustration of a transdermal drug delivery system

INTRODUCTION, ROUTES OF DRUG ADMINISTRATION

Discontinuation has been necessa ry

in

onl y

2-7% cases.

5. Inhalation Volatile liquids and gases are given by inhalation for systemic action, e.g. general a naesthetics. Absorption takes place from the vast surface of alveoli- action is very rapid. When adm inistration is di scontinued the drug diffuses back and is rapidly elimi nated in exp ired air. Thus, controlled administration is possible with moment to mo ment adjustment. Irritant vapours (ether) cause inflammation of respiratory tract and increase secretio n.

6 . Nasal The mucous membrane of the nose can readi ly absorb many drugs; digestive juices and liver are bypassed. However, only certain drugs like GnRH agonists, calcitonin a nd dcsmo press in applied as a spray or nebulized solution have been used by thi s route. This route is being tried for some other peptide d rugs like insulin, as well as to bypass the blood-brai n barrie r. 7. Parenteral (Par- beyond. en/era/- intestinal) Conventionally. parenteral refers to administration by injection which takes the drug directly into the tissue fluid o r blood wit hou t having to cross the enteral mucosa. The limitations of oral administration a re circumvented. Drug action is faster and surer (valuable in emergencies). Gastric irritation and vomiting are no t provoked. Parenteral route s can be employed even in unconscious, uncooperative o r vomi ting patient. There are no chances of interference by food or digestive juices. Liver is bypassed. Disadvantages of parenteral routes are- the preparation has to be sterilized and is costlier, the technique is invasive and painful, assistance of another person is mostly needed (tho ug h self injection is possible, e.g. insulin by diabetics), there are chances of local tissue injury and, in general, parenteral route is more risky tha n oral. The important pare nteral routes are:

{i} Subcutaneous {s.c.} The drug is deposited in the loose s ubcutaneous tissue which is richly supplied by nerves ( irritant drugs cannot be injected) but is less vascular (absorption is slower than intramuscular). Only small volumes can be injected s.c. Self-injection is possi ble because deep pe netration is not needed. This route shou ld be avo ided in shock patients who are vasoconstricted- absorptio n wi ll be delayed. Repos itory (depot) preparations that a re aq ueous s uspensions can be injected for prolonged action. Some special fom,s of this route are:

(a) Dermojet

In this method needle is not used; a hig h velocity jet of drug solution is projected from a microfine orifice using a gun like implement. The solution passes through the superficial layers and gets deposited in the subcutaneous tiss ue. It is essentially painless and s uited for mass inoculations.

(b) Pellet implantation The drug in the form of a solid pe llet is introduced with a trochar and cannula. This provides sustained release o f the drug over weeks and months, e.g. DOCA, testosterone.

{c) Sialistic (nonbiodegradable) and biodegradable implants Cr ystalline drug is packed in tubes or capsules made of suitable materials and implanted under the skin. Slow and uniform leaching of the drug occurs over months providing constant blood levels. The no nbiodegradable implant has to be removed later on but not the biodegradable one. Th is has been tried for hormones and contraceptives (e.g. NORPLANT).

{ii) Intramuscular {i.m.} The drug is injected in one of the large skeletal muscles- deltoid, triceps, g luteus maximus, rectus femo ri s, etc. Muscle is less richl y s upplied with sensory nerves (mild irritants can be injected) and is more vascular (absorption of drugs in aqueous solution is faste r). It is less painful, but self injection is often impracticable because deep penetration is needed. Depot preparations (o ily solutions. aqueous s uspensions) can be injected

13

14

GENERAL PHARMACOLOGY by th is rou te. Intra muscular inj ections should be avoided in anticoagu la nt treated patients, because it can produce local haematoma.

(iii) Intravenous (i.v.) The drug is injected as a bolus (Greek: bolos- lump) or infused slowly over hours in one of the superficial veins. T he d rug reaches directl y into the b lood stream a nd e ffects a re prod uced immediate ly (great value in eme rgency). T he intima o f veins is insensitive and drug gets di luted w ith blood, therefo re, e ve n highly irr ita nt drugs can be inj ected i.v., but hazards are-thrombophlebitis of the injected vein and necros is o f adjoini ng tissues if extravasation occurs. T hese complications can be m inimized by di luting the drug or injecting it into a run ning i.v. line. O nly aqueous sol utions ( not suspe nsions, because drug particles can cause embolism) are to be

q;-

injected i.v. a nd there are no depot pre pa rations for this route. C hances of causing a ir embolism is another risk. T he dose of the drug requ ired is smallest (bioavailabi li ty is I00%) and even large volumes can be infused. One big advantage with th is route is- in case res po nse is accurately measurable (e.g. BP) and the drug short acting (e.g. sodium nitroprus ide), ti tration of the dose w ith the response is possi ble. However, th is is the most risky route- vital organs like hea rt, brain, etc. gel ex posed to high concentrations of the drug.

(iv) lntradermal injection

T he drug is injected into the skin rais ing a bleb (e.g. BCG vaccine. sensitiv ity testi ng) or scarring/multiple puncture of the epiderm is through a drop of the drug is done. T his route is e mployed fo r specific purposes only.

PROBLEM DIRECTED STUDY

1.1. A 5-yea r-old ch ild is brought to the hospit al with the com plai nt of feve r, cough, breath lessness and chest pain. On exa m ination he is found to be dul l, but irrit a ble wit h fast pulse (116/m in), rapid breathing (RR SO/min) and indrawing of lower chest during inspiration, wheezing, crepit ations a nd mild dehyd ratio n. Body te mperature is 40°C {104°F). The paediatrician makes a provisional diagnosis of acute pneumonia and orders releva nt haematological as well as bacte riological investigations. He decides to instit ute antibiotic t herapy. a. In case he select s a n a ntibiotic which can be given orally as well as by i.m. or i.v. injection, wh ich rout e of adm inistratio n will be most a ppropriate in t his case? b. Shou ld the pae d iatricia n a dminist e r th e antib iotic straight away or shou ld he wa it for the laboratory re po rts? (see Appendix-1 for solution)

Chapter

Pharmacokinetics: Membrane Transport, Absorption and Distribution of Drugs

2

Pharmacokinetics is the quantitative study of drug movement in, through and out o r the body. The overall scheme of pharmacokinetic processes is depicted in Fig. 2. 1. The intensity of response is related to concentration of the drug at the site of action, wh ich in turn is dependent on its pharmacokinetic properties. Pharmacokinetic considerations, therefore, determine the route(s) of admi ni stration, do e. latency of o nset, time of peak action, duration of act ion a nd frequency of admin istration of a drug.

Biological membrane This is a bilayer (about I 00 A thick) or p hospholipid and cholesterol molecules, the polar groups (glyceryl phosphate attached to erhanolamin e/choline or hydroxyl group of cholesterol) of these are oriented at Lhe two surfaces and the nonpolar hydrocarbon chains are embedded in the matrix to form a continuous sheet. This imparts high e lectrical resistance and relat ive imperm eabi lity to the membra ne.

DISTRIBUTION - STORAGE

ABSORPTION

II

l

A ll pharmaco kinctic processes involve transport of the drug across biological membranes.

EXCRETION

II Site of action

Storage tissue

Boun d ~ Release

Drug in solution

FREE DRUG

Blood

I

Metabolites

Protein bound

BIOTRANSFORMATION Fig. 2.1 : Schematic depiction of pharmacokinetic processes

-+---.}

Urine Bile Faeces Sweat Saliva

16

GENERAL PHARMACOLOGY Higher cone.

Lower cone.

0 Nonlipidsoluble drug

0 0

o"

Fig. 2.2: Illustration of the organisation of biological membrane

Extrinsic and intrins ic protein molecules are adsorbed on the lipid bilayer (Fig. 2.2). Glycoproteins or glycolipids are formed on the surface by attachment to polymeric sugars, aminosugars or sialic acids. The specific lipid and protein composition of different mem branes differs accord ing to the cell or the organelle type. The proteins are able to freely fl oat through the membrane: assoc iate and organize or vice versa. Some of the intrinsic ones, which extend through the full th ickness of the membrane, surround fine aqueous pores. Paracellular spaces or channels also exist between certain epithelial/ endothelial cells. Other adsorbed proteins have enzymatic, carrier, receptor or signal transduction properties. Lipid molecules also are capable of lateral movement. Thus, biological membranes are highly dynamic structures. Drugs are transported across the membranes by: (a) Passive diffusion and filtration (b) Specialized transport

0

Aqueous pore/ paracellular space

0 Fig. 2.3: Illustration of passive diffusion and filtration across the lipoidal biological membrane with aqueous pores

Lipid soluble drugs diffuse by dissolving in the lipoidal matrix of the membrane (Fig. 2.3), the rate of transport being proportio nal to the lipid : water partition coefficient o f the drug. A more lipid-soluble drug attains higher concentration in the membrane and diffuses quic kl y. A lso, g reater the d ifferen ce in th e concentration of the drug on the two sides of the membrane, faster is its diffusion.

Influence of pH Most drugs are weak e lectro ly tes, i.e. their ionization is pH dependent (contrast strong electrolytes that are nearly completely ionized at acidic as well as alkaline pH). The ionization of a weak acid HA is g iven by the equati on: [A- ] pH = pKa + log - [HA]

... ( 1)

Passive diffusion

pKa is the negative logarithm of acidic d isso-

The drug diffuses across the membrane in the direction of its concentration gradient (high to low), the membrane playing no acti ve role in the process. This is the most im portant mechanism for majority o f drugs; drugs are foreign substances (xenobiotics), and specialized mechanisms are developed by the body primarily for normal metabolites.

ciation constant of the weak electrolyte. If the concentration of ionized drug [A- ] is equal to concentration of unionized drug [HA], then [A- ] --=

[HA]

I

since log I is 0, under this condition pl I

=

pKa

... (2)

MEMBRANE TRANSPORT, ABSORPTION AND DISTRIBUTION OF DRU GS Thus, pKa is numerically equal to the p H at which the drug is 50% ion ized. If p H is increased by I sca le, thenlog

rA

]/ [HA] = I

or

[A- ]/ [II Al - 10

S imilarly, if pH is reduced by I scale, then[A- ] [IIAl

1/1 0

Thus, weakly acid ic drugs, which fo1m salts with cations, e.g. sod. phenobarbitone, sod. s ulfadiazi ne, pot. penic ill in-V, etc. ionize mo re at alkaline p H and I scale change in pH causes I 0 fold change in ionization. Weakly basic drugs, which form sa les w ith an ions, e.g. atropine sulfate, ephedrine HCI. ch loroquine phosphate, etc. con verse ly io nize more at acid ic pH . Ions being lipid insoluble, do not diffuse and a pH d ifference across a membrane can cause differential distribut ion of weakly acidic and weakly basic drugs o n the two sides (F ig. 2.4). pHS

pH 7

UNIONIZED

UNIONIZED

10

10

1l

1l

1

100

IONIZED

IONIZED

TOTAL = 11

TOTAL= 110

Fig. 2.4: Influence of pH difference on two sides of a biological membrane on the steady-state distribution of a weakly acidic drug with pKa = 6

(pll 7 .0) an d then only s lowly passes to the extracellular flu id. This is called ion trapping, i.e. a weak electrolyte crossing a membrane to encounter a pH from which it is not able to escape eas ily. This may contribute to gastric muco al cell dam age caused by aspirin. (c) Bas ic drugs attain higher concentratio n intracellularly (pH 7.0 vs 7.4 of plasma). (d) Acidic drugs are ionized more in alkaline u rin e- do not back d iffuse in t he kidn ey tubules and are excreted faster. Accordi ng ly, basic drugs are excreted faster if urine is acid ified. Lipi d -so luble none lectrolytes (e.g. eth anol, diethyl-ether) readi ly cross biological membranes and thei r transport is pH independent.

Filtration Filtration is passage of drugs through aqueous pores in the membrane o r through paracellular spaces. This can be accelerated if hydrodynamic flow of the solvent is occurri ng unde r hydrostatic or osm otic pressure g radient, e.g. across most capillaries including glomeruli. Li pid-insoluble drugs cross biological membranes by fil tration if their molec ular s ize is sma ller than the diameter of the po res (Fig. 2.3). Maj ority of cells (i ntestinal mucosa, RBC, etc.) have very small pores (4 A) and drugs w ith MW > 100 or 200 a re not able to penetrate. However, capillaries (except those in brain) have large paracell ular spaces (40 A) and most drugs (even albumin) can filter through these (Fig. 2.8). As s uc h, diffus ion of dru gs across cap illari es is dependent on rate o f blood flow through them rather than on lipid so lubility of the drug or pH o f the medium.

Implications of this consideration are: (a) Acidic drugs, e.g. aspirin (pKa 3.5) are largely unionized at acid gastric pH a nd are a bsorbed from s tom ach, w hil e bases, e.g. atropine (pKa I 0) are largely ionized and are absorbed only when they reach the intestines. (b) The unionized form of acidic drugs which crosses the s urface me mbrane of gastric mucosa! cell, reverts to the ionized form w ithi n the cell

Specialized transport This can be carrier mediated or by vesicula r transpon (cndocytosis, exocytosis).

Carrier transport All cell membranes express a host oftra nsmem branc p rote ins w hi c h se rve as ca rri e rs or transporters for physiologically important ions,

17

18

GENERAL PHARMACOLOGY Higher Cone.

Lower Cone. D

D

Poorly diffusible substrate

a

D D

Substrate A

D

Substrate B

D

D

Actively transported substrate

o A



Substrate A

A I' •

,1

,,

,1



0

M

0 '



'

M

Fig. 2.5: Illustration of different types of carrier mediated transport across biological membrane ABC- ATP-binding cassette transporter; SLC-Solute carrier transporter; M- Membrane A. Facilitated diffusion: the carrier (SLC) binds and moves the poorly diffusible substrate along its concentration gradient (high to low) and does not require energy 8. Primary active transport: the carrier (ABC) derives energy directly by hydrolysing ATP and moves the substrate against its concentration gradient (low to high) C. Symport: the carrier moves the substrate 'A' against its concentration gradient by utilizing energy from downhill movement of another substrate 'B' in the same direction D. Antiport: the carrier moves the substrate 'A' against its concentration gradient and is energized by the downhill movement of another substrate 'B' in the opposite direction

nutri ents, metabolites, transm itters, etc. across the membrane. At some sites, certain transporters also translocate xenobiotics, including drugs a nd thei r metabolites. In contrast to channels, wh ich open for a finite ti me and allow passage of speci fie io ns, tra nsporte rs combine transie ntly with their substrate (ion or organ ic compound) -undergo a conformational change carrying the substrate to the other side of the membrane where the substrate dissociates and the transporter returns back to its orig inal state (Fig. 2.5). Carrier transport is specific fo r the s ubstrate (or the type o f subs trate, e.g. an organi c a nion), saturable, competitively inhibited by a nalogues which utilize the same transporter, and is much s lower than the flux

through channels. Depending on requirement of energy, carrier transport is of two types: a . Facilitated diffusion The transpo rter, belonging to the super-family of solute carrier (SLC) transporters, operates passive ly without needing energy and translocates the substrate in the direction of its e lectrochemical gradient, i.e. from hi gher to lower concentratio n (F ig. 2.5A). Lt mere ly facilitates permeation of a poorly diffusible substrate, e.g. the entry of g lucose into muscle and fa t cells by the glucose transporter GLUT 4. b. Active transport It requires ene rgy, is inhibited by metabolic poisons, and transports the solute against its electrochemical gradient (low to high), resulting in selective accumulation of the

MEMBRANE TRANSPO RT, ABSORPTION AND DISTR IBUTION OF DRUGS substance on one side of the membrane. Drugs rel ated to normal metabolites can utili ze the transport processes meant for these, e.g. levodopa and methyl dopa arc actively absorbed from the gut by the aromatic am ino acid trans porter. In addition, the body has developed some re latively nonselecti ve transporters, like P-glycopro1ei11 (P-gp), to deal with xenobiotics. Acti ve transport can be primary or secondary depending on the source of the d riving force. i. Primary active transport Energy is obtained directly by the hydrolysis of ATP (Fig. 2.58). The transporters belong to the superfamily of ATP binding cassettee (A BC) transporters whose intracellular loops have ATPase activity. They mediate only emux of the solute from the cytoplasm, either to extracellular fluid or into a n intracellular o rganelli (endoplasmic reticulum , mitochondria, etc.) Encoded by the multidrug resistance 1 (M DRl) gene. P-gp is the most well known primary acti, e transporter expressed in the intest inal mucosa, rena l tubules, bi le canal iculi , choroidal epithelium. astrocyte root processes around brain capillaries (the blood-brain barrier). testicular and placental m1crovcsscls, wh ich pumps out many drugs/ metabolites and thus limit, their intestinal absorption. penetration into brain, testes and foetal tissues as well as promotes biliary and renal elimination. Many xenobiotics which induce or inhibit P-gp also ha,e a similar effect on the drug metabolizing isoenzyme CY l'3A4, indicating their synergistic role in detoxification of ,xenobiotics. Other primary active transporters o f pharmacological significance are multidrug resistance associated protein 2 (MRP 2) and breast cancer re istancc protein (BCRI').

ii. Secondary active transport In this type of active transport effected by another set of SLC trans po rters, the energy to pump one solute is derived from the downhill movement of another solute (mostly a ). When the concentration grad ients a re such that both the solutes move in the same direction (F ig. 2.5C), it is called symport o r cotransporl, but w hen they move in opposite di rections (Fig. 2.50 ), it is termed antiport or exchange lransporl. Metabolic energy ( from hydrolysis of ATP) is spent in maintaining hi gh transmembrane e lectrochemi cal gradient of the second solute (generally a+). The SLC transporters mediate both uptake and efflux of drugs and metaboli tes.

The organic anion transporting polypeptide (OATP) and organic cation transporter (OCT). highly expressed in liver canaliculi and renal tubules. are secondary active transporters important in the metabolism and e:\cretion of drugs and metabolites (especial ly glucuronidcs). The a ,Cl dependent neurotransmillcr transporters for norcpinephrine. serotonin and dopamine (NET, SERT and DAT) are active e S LC transporters that are targets for action of drug, like tricyclic antidepressants, selecti, e serotonin reuptake inhibitors (SSRls). cocaine, etc. Similarly. the Vesicular monoamine transporter (VMAT-2) of adrenergic and serotoncrgic storage vesicles transports catecholamines and 5-HT 11110 the vesicles by exchang ing with H ions, and is inhibited by reserpine. fhc absorption of glucose in intest ine, and renal t ubules is t hrough secondary acti,e transport by sodium-glucose transporters (SGLTl and GLT2).

As indicated earlier, ca rrie r transport (both facilitated diffusion and active trans po rt) is saturab lc a nd follows the Mich ae lis- Menten kinetic s. The max imal rate o f tran s port is dependent on the density of the transporter in a particular membran e, and its rate constant ( Km), i.e. the substrate conce ntration a t which rate of transport is half maximal, is governed by its affi nity fo r the s ubstrate. Ge neti c po ly morphi sm can a lter both the density and affinity of the transporter protein fo r different s ub strates and thus affect the pha rmacokineti cs of drugs . Moreover, ti ssue specific dru g d istribution ca n occu r due to the presence of s pecific tran s porters in certain cells.

Vesicular transport (endocytosis, exocytosis) Certain s ubstances w ith very large or impermeable molecules arc transported inside the cell (endocytosis) or extruded from it (exocytosis) by enclosing thei r particles into tiny vesicles. A binding protein located on the me mbrane co mpl exes wit h the s ub stance and in itiates vesic le formation (Fig. 2.6). The ves icle then detaches from the membra ne a nd may remain stored w ithin the cell. or it may release the substance in the cytoplasm, or it may move to the o pposite membrane fuse w ith it to re lease the s ubstance across the cell (exocytosis).

19

20

GENERALPHARMACOLOGY

Fig. 2.6: Illustration of vesicular transport (endocytosis and exocytosis). Endocytosis: The large molecular particle (P) binds to a binding protein (B) on the surface of the cell. T he membrane invaginates to form a vesicle which nicks off, and the vesicle may remain stored within the cell, or it may disintegrate to release the substance in the cytoplasm, or be extruded across the cell by exocytosis. Exocytosis: The particle or the transmitter/hormone(T/H) stored within intracellular vesicles, generally as a complex with a storage protein (S), is secreted by exocytosis. On activation the vesicle translocates to and fuses with the membrane. All contents of the vesicle are then poured out in the extracellular space.

Vesicu lar trans po rt is a pplicable to prote ins and other big molecules, and contributes little to transport of most drugs, barring few li ke vi t B12 whic h is absorbed from the gut after bind ing to intrinsic factor(a prote in). Most hormones (insulin, e tc.) and neurotransmitters, like noradrena line, are secreted/released from the cell/nerve end ing by exocytosis. Activation of the secretory cell/ nerve e nding prompts fus ion of the storage vesicle to the surface membrane fo ll owed by extrusion of its contents into the ex tracellular space.

ABSORPTION Absorption is movement of the drug from its site of adm inistration into the c irc ulation. ot only the fraction of the administered dose that gets absorbed, but a lso the rate of absorpti on is important. Except when given i.v., the drug has to cross biological membranes; a bsorption is governed by the above described principles. Other fac to rs affecting absorpti o n are:

MEMBRANE TRANSPORT, ABSORPTION AND DISTR IBUTION OF DRUGS

Aqueous solubility Drugs given in olid form must dissolve in the aqueous biophase before they are absorbed. For poorly water soluble drugs (aspirin, griseofulvin) rate of dissolution governs rate of absorption. Ketoconazole di ssolves at low pH: gastric acid is needed for its absorption. Obvious ly, a drug given as wate1y solution is absorbed faster than w hen the same is given in solid form or as oi ly solution. Concentration Passive diffusion depends on concentration gradient; drug given as concentrated solution is absorbed fas ter than from dilute solution. Area of absorbing surface

Larger is the surface area. faster is the ab orption.

Vascularity of the absorbing surface Blood circulation removes the drug fro m the site of abso rpti on and maintains the conce ntrati on gradient across the absorbing surface. Increased blood flow hastens d rug absorption just as wind hastens drying of c lothes. Route of administration This affects drug absorption, because each route has its own peculiarities. Oral The effective barrier to orally administered drugs is the epithelial lining of the ga trointestinal tract. which is lipoidal. onionizcd lipid soluble drugs, e.g. ethanol arc readily absorbed from stomach as well as intestine at rates proportional to their lipid : water partition coefficient. Acidic drugs, e.g. salicy lates, barbiturates, etc. are predominantly unioni zed in the acid gastri c juice a nd arc absorbed from stomach, while basic drugs, e.g. morph ine, quin ine, etc. are largely ionized and are absorbed only on reaching the duodenum. However, eve n for acidic drugs absorp tion from stomac h is s lower, because the mucosa is thick, covered w ith mucus and the s urface area is s mall. Absorbing s urface area is much larger in the small intestine due to vi lli. Thus, faster gastric emptying acce lerates drug absorption in general. Dissolution is a su1face phenomenon, therefore, particle size

of the drug in solid dosage form governs rate of dissolution and in turn rate of absorption. Presence offood dilutes the drug and re ta rds absorption. Further. certain drugs form poorly absorbed complexes with food constituents, e.g. tctracycl i nes with ca lei um present in milk; moreover food delays gastric emptying. Thus, most drugs arc absorbed better if taken in empty stomach. I lowever, there are some exceptions , e.g. fatty food greatl y en hances lumefantrine absorption. Highly ionized drugs, e.g. gentamicin, neostigmine are poorly absorbed when given o ral ly. Certain drugs are degraded in the gastrointestinal tract, e.g. penicillin G by acid. insu li n by peptidases, and arc inclTective orally. Enteric coated tablets (having acid resistant coating) and sustained release preparations (drug particles coated w ith slowly dissolving material) can be used to overcome acid !ability, gastric irritancy and brief durati on of action. T he o ral absorption of certain drugs is low because a fractio n of the absorbed drug is extruded back into t he intestinal lume n by the efflux tran s porter P-gp located in the gut epi thelium. The low oral bioavai lability of digoxin and cyclosporine is partly accounted by this mechanism. Inhibitors of P-gp like quinid ine, verapamil , erythromycin, etc. enhance, while P-gp inducers like rifampin and phenobarbitone reduce the oral bioavailability of these drugs. Absorption of a drug can be affected by other concurrently ingested drugs. This may be a luminal effect: forma tion of insoluble complexes, e.g. tetracyc lines and iron preparations with calc ium salts and antacids, phenytoin with sucralfa te. Such interaction ca n be minim ized by admi nistering the two drugs at 2 3 hr intervals. Alteratio n of gut flora by antibiotics may disrupt the enterohe patic cycling of oral contraceptives and digox in. Drugs can also alter absorption by gut wall effects: altering motility (anticho 1inergics, tricycl ic antidepressants, opioids retard motility while metoclopramide enhances it) or causi ng mucosa I damage (neomycin, methotrexate, vinbla tine).

21

22

GENERALPHARMACOLOGY Subcutaneous and Intramuscular By these routes the drug is deposited directly in the vicinity o f the capillaries. Lipid soluble drugs pass readil y across the whole surface of the capill ary endothe lium. Capi llaries having la rge paracellu la r spaces do not obstruct absorption of even large lipid insoluble mo lecules or ions (F ig. 2.9A). Very large molecu les are absorbed through lymphatics. T hus, many drugs not a bsorbed orally a re absorbed parenterally. Absorption from s.c. site is s lower than that from i.m. site, but both a rc generally faster and more consistent/ predictable than oral absorption. Application of heat and muscular exercise accelerate drug absorption by increasing blood flow, while vasoconstrictors, e.g. adrenaline injected wi th the drug (local anaesthetic) retard absorption. Incorporatio n of hya luronidase facilitates drug a bsorption from s.c. injection by promoting spread. Ma ny depot preparations, e.g. benzathine penicill in, protami ne z inc insulin, depot progestins, etc. can be given by these routes.

get absorbed through the nasolacri mal duct, e.g. timolol eye drops can produce bradycardia and precipi ta te asthma. Mucous membranes of mouth, rectum, vagina absor b lipophi lic drugs: estrogen cream applied vaginally has produced gynaecoma t ia in the ma le partner.

Bioavailability Bioavailability refers to the rate and extent of absorption of a drug fro m a dosage fo rm admin istered by any route, as determi ned by its concentration-time curve in blood or by its excretion in urine ( Fig. 2.7). It is a measure of the fraction (F) of administered dose of a drug that reaches the systemic c irculation in the uncha nged form. Bioava ilabi li ty of drug injected i.v. i I 00%, but is frequently lower after oral ingestion because(a) the drug may be incomplete ly absorbed. (b) the absorbed drug may undergo fi rst pass metabo lism in th e intesti na l wa ll/liver o r be excreted in bile.

Topical sites (skin, cornea, mucous membranes) Systemic absorption after topical application depends primaril y on lipid solubility of drugs. However, only few drugs significantly penen·ate intact skin. Hyoscine, fentanyl, GT , nicotine, testosterone, and estradiol (see p. 12) have been used in this ma nner. Corticosteroids applied over extensive areas of skin can produce systemic effects and pituitary-adrenal suppression. Absorption can be promoted by rubbing the drug incorporated in an o legenous base or by use of occlusive dressing which increases hydration of the skin. O rganophosphate insecticides coming in contact with skin can produce systemic toxicity. A braded surfaces readily absorb drugs "'' e.g. tann ic acid applied over burnt ski n has produced hepatic necrosis. Cornea is permeable to lipid soluble, unionized physostigmine but not to high ly ionized neostigmine. Drugs applied as eye drops may

i

A

zu z u
30 hr. However, when the same drug is given repeatedly or continuously over long periods, the low perfusion high capacity sites get progressively fi lled up a nd the drug becomes longer acting.

Penetration into brain and CSF The capillary endothelia l ce lls in brain have tight juncti ons a nd lack large paracellular spaces. Further, an inves tment of neural tissue (Fig. 2.9B) covers the capillaries. Together they constitute the so called blood-brain barrier (BBB). A similar bloodCSF barrier is located in the choroid plexus: capi llaries are lined by choroidal ep ithe liu m hav ing tight junctio ns. Both these barriers are lipoidal and limit the entry of nonl ipid-soluble drugs, e.g. streptomycin, neostigminc, etc. Only lipid-soluble drugs, the refore, are able to penetrate and have action o n the centra l nervous sys tem. In addition, effl ux transporters li ke P-gp and anion transporter (OATP) present in brain and choroidal vessels extrude many drugs that enter brain by other processes and serve to augment the protective barrier against potentially harmfu I xenobiotics. Do pamine does not enter brain but its precursor levodopa does; as such,

25

MEM BRANE TRANSPORT, ABSORPTION AND DISTRIBUTION OF DRUGS

Tight junctions

Endothelial cells

Glial cell processes or·- -___.:::1111,,,~ choroidal epithelium

Usual capillary

Brain capillary

Fig. 2.9: Passage of drugs across capillaries A. Usual capillary with large paracellular spaces through which even large lipid-insoluble molecules diffuse B. Capillary constituting blood brain or blood-CSF barrier. Tight junctions between capillary endothelial cells and investment of glial processes or choroidal epithelium do not allow passage of nonlipid-soluble molecules/ions

the latter is used in parkinsonism . Inflammation of meninges or brain increases permeability of these barri ers. lt has been proposed that some drugs accumul ate in the brain by utili zing the transporters for endogenous substances. There is a lso an enzymatic BBB: Monoamine oxidase (MAO), cholinesterase and some other enzymes are present in the capillary wa lls or in the cells lining th em. They do not allow catecholamines. 5-HT, acetylcho li ne. e tc. to enter brain in the active fo tm. T he BBB is deficie nt at the CTZ in the medulla oblongata (even lipid-insoluble drugs are emetic) and at certa in peri ventric ular sites(anterio r hypo thalamus). Ex it of drugs fro m the CSF and brain, however, is not dependent o n lipid-solubility and is rather unrestricted. This is due to bulk flow of CSF (alongwi th the drug dissolved in it) back into blood through the arach noid vi lli. Further, nons pecific o rgan ic an ion and cation transport processes (similar to those in renal tubule) operate at the choro id plexus.

Passage across placenta Placenta l membranes are lipoidal and allow free passage o f lipo philic drugs, while restricting hydrophi lic d rugs. The p lacental effl ux P-gp and other transpo rters li ke BCRP, MRP3 al so serve to

lim it foetal exposu re to maternally administered drugs. Placenta is a site for d rug metabolism a s well, which may lower/modi fy exposure of the foetus to the administered drug. I lowever, restricted amounts of no nlipid-soluble drugs, w hen present in high concentration or for long periods in maternal circulation, gain access to the foetus. Some influx tran sporters also operate at the placenta. Thus, it is an incomplete barrier and almost any drug taken by the mother can a ffect the foe tus o r the newbo rn (d rug ta ken just before delivery, e.g. morphine).

Plasma protein binding Most drugs possess ph ys icochemical affinity for plasma proteins and get reve rsibly bound to these. Acidic drugs generally bind to plasma a lbumin and basic drugs to a 1 acid glycoprotein. Bind ing to albumin (which is more abundant) is quantitatively more important. Extent of binding depends o n the indi vi dua l compound ; no genera li zation for a pharmacological or chemical class can be made (even small chemical change can markedl y alter protein binding), for example the binding percentage of some benzodiazepines is: Flu razepam Lorazepam

10% 90%

Alprazolam Diazepam

70% 99%

26

GE NERAL PHARMACO LOGY Increasing concentrations of the drug can progressively saturaLe the binding siLcs: fractional binding may be lower when large amounts of the drug are given. The generally expressed percentage binding refers to the usual therapeutic plasma concentrations ofa drug. The clinically significant implications of plasma protein binding are: (i) Highly plasma prote in bound drugs are largely restricted to the vascular compartment because protein bound drug does not cross membranes (except through large paracellular spaces, such as in capillaries). They tend to have smaller volumes of distribution. Drugs highly bound to plasma protein To albumin

To a,-acid glycoprotein

Barbiturates

B-blockers

Benzodiazepines

Bupivacaine Lidocaine Disopyramide

NSAIDs Valproic acid Phenytoin Penicillins

lmipramine Methadone

Sulfonamides

Prazosin

Tetracyclines

Ouinidine Verapamil

Tolbutamide Warfarin

(i i) The bound fract ion is not avai lable for action. However, it is in equilibrium w ith the free drug in plasma and dissociates when the concentration of the latter is reduced due to elimination . Plasma protein binding thus tantamounts to temporary storage of the drug. (iii) High degree of protein binding generally makes the dru g long acting, beca use bound fraction is not available for metabolism or excretion, unless it is actively extracted by liver or by kidney tubules. Glomerular filtration does not reduce the concentration of the free form in the efferent vessels, because water is also filtered. Active tubular secretion, however, removes the drug without the attendant solvent • concentration of free drug fall • bound drug dissociates and is eliminated resulting in a higher renal clearance val ue of the drug than the total renal blood flow (see Fig. 3.3).

The same is true of active transport of highly extracted drugs in liver. Plasma protein bindi ng in this situation acts as a carrier mechanism and hastens drug elimination, e.g. excretion of penici llin (elimination t½ is 30 min); metabolism of lidocaine. Highly protein bound drugs are not removed by haemodialysis and need special techniques for treatment of poisoning. (iv) The generally expressed plasma concentrations of the drug refer to bound as well as free drug. Degree of protein binding should be taken into account while relating these to concentrations of the drug that are active in vitro, e.g. MIC of an antimicrobial. (v) One drug can bind to many sites on the albumin molecule. Conversely, more than one drug can bind to the same site. This can give rise to displacement interactions among drugs bound to the same ite( ). The drug bound with higher affinity will displace that bound with lower affinity and tend to increase the concentration of its free form. This, however, is often transient becau e the displaced drug will diffuse into the tissue as well as get metabolized or excreted: the new steady-state free drug concentration is only marginally higher unless the displacement extends to tissue binding or there is concurrent inhibition of metabolism and/or excretion reducing drug clearance. The overall impact of many displacement interactions is minimal; except when the interaction is more complex. Moreover. two highly bound drugs do not necessarily displace each other-their binding sites may not overlap, e.g. probenecid and indomethaci n are highly bound to albumin bu t do not displace each other. Similarly, acidic drugs do not generally displace basic drugs and vice versa. (vi) In hypoalbuminemia. binding may be reduced and high concentrations of free drug may be atta ined, e.g. phenytoin and furosemidc. Other diseases may also alter drug binding, e.g. phenytoin and pethidi ne binding is reduced in uraemia; propranolol binding is increased in pregnant women and in patients with inflammatory disease (acute pha e reactant a 1 acidglycoprotein increases).

MEMBRANE TRAN SPORT, ABSOR PTION AND DISTRIBUTI ON OF DR UGS

Drugs concentrated in tissues Skeletal muscle, heart Liver Kidney Thy roid Brain Retina Iris Bone and teeth Adipose tissue

digoxin, emetine (bound to muscle proteins). chloroquine, tetracyclines, emetine, digoxin. digoxin, chloroquine, emetine. iodine. chlorpromazine, acetazolamide, isoniazid. chloroquine (bound to nucleoproteins). ephedrine, atropine (bound to melanin). tetracyclines, heavy metals (bound to mucopolysaccharides of connective tissue), bisphosphonates (bound to hydroxyapatite). thiopentone, ether, minocycline, phenoxybenzamine, DDT dissolve in neutral fat due to high lipid-solubility; remain stored due to poor blood supply of fat.

Tissue storage

Drugs may also accumulate in specific organs by active transport or get bound to speci fic tissue constitucms (see box). Drugs sequestra ted in va ri ous t issues are uneq ually di stributed , tend to ha ve l arger volume of distributi on and longer duration o f action. Some may exert local toxicity due to

high concentration, e.g. tetracyclines on bone and teeth, ch loroq ui ne on retina, streptomycin on vesti bular apparatus, emetine on heart and skeletal muscle. Drugs may also selectively bind to specific intracellular organelle, e.g. tetracyc line to mitochondria, chloroqui ne to nuclei .

12 frequently used drug including omeprazolc, lansoprazole phenytoin. diazepam omeprazole and fluconazole are it, inhibitors. CYP1A 1/2 Though th is s ubfamily part icipate, in the metabo lism of only few drugs like thcophylline, caffeine. paracetamol. carbamazepinc. it is more important for activat ion of procarcinogens. Polycyc lic hydrocarbons. cigarelle smoke and charbroiled meat are it, pote nt mducers. CYP2E I It catalyses oxidation o f alcohol, holothane. and formation of minor metabolite~ of few drugs, notably the hcpatotoxic N-accty l benzoquinoneiminc from paracetamol; chronic alcoholis m induces this isocnzyme.

The relative amount of di ITerent cytochrome P-450s d iffers among species and among individuals of the same species. These differences

29

30

GENERAL PHARMACOLOGY Table 3.1: Ma1or drug metabolizing CYP450 1soenzymes 1n humans with their important substrate drugs, 1nh1b1tors and inducers CYP-450 isoenzyme Drugs metabolized

Inhibitors

Inducers

CYP3A4 CYP3AS

Terfenadine, Astemizole Cisapride, Losartan Carbamazepine, Hydrocortisone Paracetamol, Diazepam Buspirone, Mifepristone Ritonavir, Saquinavir Simvastatin, Quinidine Verapamil, Lidocaine Dapsone, Nevirapine

Erythromycin Clarithromycin Ketoconazole ltraconazole Verapamil Ritonavir Fluoxetine Grape fruit juice

Barbiturates Phenytoin Carbamazepine Rifampin Glucocorticoids Nevirapine

CYP2D6

Metoprolol, Debrisoquine Nebivolol, Amitryptyline Clomipramine, Fluoxetine Paroxetine, Venlafaxine Haloperidol, Clozapine Risperidone, Codeine Propafenone, Flecainide

Qunidine Fluoxetine Pa roxetine

Phenobarbitone Rifampin

CYP2C8 CYP2C9

Phenytoin, Carbamazepine Warfarin, Tolbutamide Repaglinide, Pioglitazone Diclofenac, lbuprofem Losartan

Fluvoxamine Fluconazole Gemfibrozil Trimethoprim

Phenobarbitone Carbamazepine Rifampin

CYP2C19

Omeprazole, Lansoprazole Amitriptyline, Citalopram Phenytoin, Diazepam Propranolol, Clopidogrel

Omeprazole Fluconazole

Carbamazepine Rifampin

CYP1A1 CYP1A2

Theophylline, Catteine Paracetamol, Warfarin Carbamazepine

Fluvoxamine Fluoxetine

Polycyclic hydrocarbons Cigarette smoke Charbroiled meat Rifampin Carbamazepine

CYP2E1

Alcohol, Halothane Paracetamol'

Disulfiram Fomepizole

Chronic alcoholism lsoniazid

CYP286

Efavirenz, Nevirapine Cyclophosphamide, Methadone Sertraline, Clopidogrel

Paroxetine Sertraline Clopidogrel

Phenobarbitone Cyclophosphamide

• Generates toxic metabolite N-acetyl-p-benzoquinoneimine (NABQI) largely account for the marked interspecies and interindividual differences in rate of metabo li sm of drugs. Barbi r rates, p henothiazi nes, i mi pram ine, propranol ol, ibuprofen, paracetamol, steroids, pheny toin, benzodiazepincs, theophylline and many o ther drugs are ox idized in this w ay by C YP450. In addition few drugs like c imetidine, ranitidine, clozapine are oxidized at their , P o r S atom

by a group of flavin-monooxygenases

that are also located on hepatic endopl asmic reticulum, but are distinct from CYP enzymes. T hese enzymes are not susceptible to induction or inhibition by other drugs, and thus are no t in vo lved in drug interactions. Some other drugs, e.g. adrena line, alcohol, m crcaptopurine are oxidized by mi tocho ndrial or cytoplasmic enzymes.

(ii) Reduction This reaction is the converse of oxidation and involves cytoc hrome P-450

METABO LI SM AND EXC RETIO N OF DRUGS, KIN ET ICS OF ELIM INATION enzymes working in the opposi te direction. Alcohols. aldehydes, qui nones are reduced. Drugs primarily reduced arc chloralhydrate, chloramphenicol, halothane. warfarin.

(iii) Hydrolysis

This is cleavage of drug molecule by taking up a molecule of water.

esterase Ester + Hp

Acid + Alcohol

Similarly. amides and polypeptides are hydrolysed by amidascs and peptidases. In addition. th ere are epoxide hydrolases which detoxify epoxide metabolites of some drugs genera ted by CYP oxygenases. Hydrolysis occurs in liver, intestines, plasma and other tissues. Examples of hydrolysed drugs are choline esters, procaine, lidocaine, procainamide. aspirin. indomethacin, carbamazepine-epoxide. pethidine, oxytoci n. (iv) Cyclization This is formation of ring structure from a straight chain compound, e.g. cycloguanil from proguanil. (v) Decyclization This implies opening up of ring structure of the cyclic drug molecule, such as barbiturates. phcnytoin. This is generally a minor pathway.

Synthetic reactions These reactions involve conjugation of the drug or its phase I metabo lite with an endogenous substrate, usually derived from carbohydrate o r amino acid, to form a polar highly ionized organic acid, which is easi ly excreted in urine or bile. Conj ugation reactions have high energy requirement and are generally faste r than phase l reactions.

(i) Glucuronide conjugation This is the most important syntheti c reaction carriedout by a group of UDP-glucuronosyl transferases (UGTs). Compounds with a hyd roxyl or carboxylic acid g roup are easily conjugated with glucuronic acid which is derived from glucose. Examples arechloramphenicol, aspiri n, paracetamol, diazepa m, lorazepam, morphine, metronidazolc. ot onl y drugs but endogenous substrates like bili rubin, steroida l hormo nes and thyroxi ne utilize this

pathway. Glucuronidation increases the mo lecular weight of the drug which favours its excretion in bile. Drug glucuronidcs excreted in bile can be hydrolysed by bacteria in the gut- the liberated drug is reabsorbed and undergoe s the same fate. This ente rohcpatic cycling (see Fig. 3.2) of the d rug pro longs its action, e.g. phenolphthalei n, o ral contraceptives.

(ii) Acetylation Compounds having amino or hydrazine residues are conjugated with the help o f acetyl coenzyme-J\, e.g. s ulfonamides. isoniazid. PAS, dapsone, hyd ra lazine, clonazepam, procai namide. M ultiple genes control the N-acetyl transferases ( ATs), and rate of acetylation shows genetic polymorphism (s low and fa t acctylators). (iii) Methylation The amines and phenols can be methylated by methyl transferases ( MT); methionine and cysteine acting as methyl donors, e.g. adrenal ine, hi stamine, ni cotinic acid, methyldopa, captopril, mercapto purine. (iv) Sulfate conjugation The phenolic co mpounds and steroids are sulfated by s ulfotransferases (SU LTs). e.g. chloramphcnicol. methyldopa, adrenal and sex steroids. (v) Glycine conjugation Salicylates, nicotinic acid a nd other drugs having carboxylic acid group are conjugated with glycine, but this is not a major pathway of metabolism. (vi) Glutathione conjugation This is carried out by glutathione-S-transferasc (GST) forming a mercapturatc. It is normally a minor pathway. However. it serves to inactivate highly reactive quinonc or epox ide intermediates formed d uri ng metabol ism of certai n drugs, e.g. paracetamol. When large amount of such intermediates are formed (in poisoning or after enzyme induction), gl utathi one supply falls short- toxic adducts are formed with tissue constituents resulting in hepatic, renal and other ti ssue damage. (vii) Ribonucleoside/ nucleotide synthesis T his pathway is important for the act ivation of many purine and pyrimidine antimerabolites used in ca ncer chemotherapy.

31

32

GENERA L PHARMACOLOGY as well as in other tissues including plasma. The csterases, amidases, some flavoprotein ox idases and most conj ugases are nonmicrosomal. Reactions catalysed are: Some ox idations and redu c ti ons, man y hydrolytic reactions and all conj ugat ions except glucuronidation.

Fig. 3.1: Simultaneous and/or sequential metabolism of a drug by phase I and phase II reactions

Most dru gs a re metabolized by mul ti ple pathways, simultaneously or sequent iall y as illustrated in Fig. 3. 1. Rates of reaction by different pathways often vary considerab ly. A va riety of metabolites (some more, some less) of a drug may be produced. Stereoisomers of a drug may be metabolized differently and at difTercnt rates, e.g. S-warfarin rap idly undergoes ring oxidation, whi le R-warfarin is slowly degraded by sidechain reduction. Only a few drugs are metabolized by enzymes of intermediary metabol ism, e.g. alcoho l by dehydrogenase, allopurinol by xanthine oxidase, succinylcholine and procaine by plasma cholinesterase, adre na li ne by monoaminc oxidase. Majority of dru gs are acted on by relati vely nonspecifi c enzymes which are directed to types of molecules rather than to specific drugs. The same enzyme can metabolize many drugs. The dru g metabolising enzymes are di vided into two types: Microsomal enzymes These are located on smooth endoplasmic reticulum (a system of microtubules inside the cell), primarily in liver. also in kidney, intestinal mucosa and lungs. The monooxygenases, cytochrome P450, UGTs, cpoxide hydrolases, etc. are microsomal enzymes. They catalyse most of the oxidations. reductions, hydrolysis and glucuronide conjugation. Microsomal enzymes are inducible by drugs, certai n dietary constituents, and other agencies. Nonmicrosomal enzymes These arc present in the cytoplasm and mitochondria of hepatic cells

The nonmicrosomal enzymes are not inducible but many show genetic polymorphi sm (acetyl transferase, pseudocholinesterase) similar to the microsomal enzymes. Both microsomal and nonmicrosomal enzymes are deficient in the newborn, especially premature, maki ng them more susceptible to many drugs, e.g. chloramphenicol, opioids. This de ficit is made up in the fi rst few month s, more qu ickly in case of oxidation and other phase I reactions than in case of glucuro nide and other conjugations which take J or more months to reach adu lt levels. The amount and kind of drug metabolizing enzymes is controlled genetically and is also altered by diet. environmental factors. T hus, marked interspecies and interindividual differences are seen, e.g. cats are deficient in UGTs while dogs are deficient in ATs. Upto 6-fold difference in the rate of metabolism of a drug among normal human adults may be observed. This is one of the major causes of individual variation in drug response. Hofmann elimination This refers to inactivation of the drug in the body fluids by spontaneous molec ular rearrangement without the agency of any enzyme, e.g. atrac urium. Inhibition of Drug Metabolism

Azole antifungal drug , macrolide antibiotics and some other drugs bind to the heme iron in CYP450 and inhibit the metabolism of many drugs, as well as some endogenous substances like steroids, bi lirubin. One drug can competitively inhibit the metabolism of another if it utilizes the same enzyme or cofactors. I lowever, such interactions are not as common as one would expect, because often different drugs arc substrates for different CY P-450 isoenzymes. It is thus important to

METABO LI SM AND EXCRETION OF DRUGS, KI NETICS OF ELIMI NATION

know the CYP isoenzyme(s) that carry out the metabolism of a particular drug. A drug may be the substrate as well as inhibitor of the same CYP isoenzyme. In case the drug inhibits its own metabolism, the oral bioavailability is likely to increase (for high first pass metabolism drugs), while systemic clearance is like ly to decrease, prolonging plasma half life. On chronic dosing, the oral bioavailability of verapami l is nearly doubled and t½ is prolonged, because it inhibits its own metabolism. A drug may also inhibit one isoenzyme while being itself a substrate of another isoenzyme, e.g. quinidine is metabolized mainly by CYP3A4 but inhibits CYP2D6. Moreover, majority of drugs, at therapeutic concentrations, are metabolized by non-saturation ki netics, i.e. the enzyme is present in excess. Clinically significant inh ibition of drug metabolism occurs in case of drugs having affin ity for the same isoenzyme. specially if they are metabolized by saturation kinetics or if kinetics changes from fi rst order to zero order over the therapeutic range (capaci ty limited metabolism). The ' boosted' HIV-protease inhibitor {Pl) strategy utilizes the potent CYP3A4 inhibitory action of low dose ritonavir to lower the dose of other Pis like ataza navir, lopinavir. saquinavir given concurrently. Obviously, inhibition of drug metabolism occurs in a dose related manner and can precipitate toxicity of the object drug (whose metabolism has been inhibited). Because enzyme inhibi tion occurs by direct e lTect on the enzyme, it has a fast time course

Drugs that inhibit drug metabolizing enzymes Allopurinol Omeprazole

Propoxyphene

Amiodarone

Erythromycin

lsoniazid

Clarithromycin Chloramphenicol

Cimetidine Quinidine

Ketoconazole ltraconazole

Disulfi ram Diltiazem

Metronidazole Ciprofloxacin

Verapamil MAO inhibitors

Fluoxetine (and other SSRls)

Ritonavir (and other HIV protease inhibitors)

(within hours) compared to enzyme induction (see below).

Metabolis m of drugs wit h high hepatic extraction is dependent on liver blood flow (blood flow li mited metabolism). Propran olol reduces rate of lidocaine metabolism by decreasing hepatic blood flo w. Some other drugs whose rate of metabolism is limited by hepatic blood fl ow are morphine, propranolol, verapamil and imipramine. Microsomal Enzyme Induction Many drugs, insecti c ides and carcin ogens interact with DNA and increase the ynthesis o f microsomal enzyme protein, especiall y cytochrome P-450 and UGTs. As a re ult the rate o f metabolism of inducing drug itself (autoi nduction) and/or some other coadmi nistered drugs is accelerated. Different inducers are relatively selective for certain cytochrome P-450 isoenzyme fami lies, e.g.: • Anticonvulsants (phenobarbitone. phenytoi n, carbamazepine), rifam pin, glucocorticoids induce CYP3A isocnzymes. • Phenobarbitone and ri fampi n also induce CYP2D6 and CYP2C8/9. • Carbamazepine and rifampin, in addition, induce CYP2C l9 and CYPI A l/2. • lsoniazid and chron ic alcohol consumption induce CYP2E l. • Polycyclic hydrocarbons like 3-methylcholanthrene and bcnzopyrene fo und in cigarette s mo ke, c harcoal broiled meat, omeprazo le a nd industrial pollu tants induce CYP I A isoenzymes. • Other enzyme inducers are: chron ic alcoholism, nevirapine, griseofu lvin, DDT. S ince diffe rent C YP isoenzy me s a re involved in the metabolism of di fferent drugs, every inducer increases biotransfo rmation of certain drugs but not that o f others. However, phenoba rbitonc like indu cers o f CYP3A and CYP2D6 a ffec t the me taboli s m of a large number of drugs, because these isoenzymes act on many drugs. On the other hand

33

34

GENERAL PHARMACOLOGY inducti o n by polycycl ic hydrocarbons is limited to few drugs ( like theophy lline. warfarin) because CY P I A isocnzyme metabolizes only few drugs. Induction in volves mic rosomal enzymes in liver as well as other o rgans and increases the rate of metabolism by 2--4 fold. Inductio n takes 4- 14 days to reach its peak and is maintained till the induc ing agent is being g iven. Thereafter the enzymes return to their original value over I 3 weeks.

Possible uses of enzyme induction I. Congenital nonhaemolytic jaundi ce: It is due to deficient g lucuronidation of bilirubin: phenobarbitone hastens clearance of jaundice. 2. Cushing's syndrome: phenytoin may red uce the manifestations by e nhancing degradation of ad renal steroids which are produced in excess. 3. Chronic poisonings: by faste r metabolism of the accumulated poisonous substance. 4. Liver disease.

First-pass (Presystemic) Metabolism Consequences of microsomal enzyme induction I . Decreased intensity and/or duration of action of drugs that are inactivated by metabo lism, e.g. failu re of contraception with oral contraceptives and loss of anti- HI V acti on ofnevirapine due to ri fam pin coadmin istration. 2. Increased intensity of action of drugs that are activated by metabolism. Acute paracetamol toxicity is due to one of its metabolites- tox icity occurs at lower doses in patients receiving enzyme inducers. 3. Tolera nce- if t he dru g induces its own metabolism (autoinduction), e.g. carbamazepine. ri fampin; nevi rapinc dose needs to be doubled a fter 2 weeks. 4. Some endogenous substra tes (steroids, bilirubin) a re also metabol ized faster. 5. Precipitation o f acute intermittent porphyria: enzyme induction increases porphyrin syn thesis by derepressing delta-aminolevulinicacid synthetase. 6. Intermittent use of an inducer may interfere with adjustment of dose of another drug prescribed on regula r basis, e.g. oral anticoagu lants, ora l hypoglycaemics, antiepileptics, antihypertensives. 7. Interfere nce w ith c hroni c toxicity testing in anima ls. Drugs whose metabo li sm is significan tly affected by enzyme induction are- phenytoi n, ca rba mazepine, an tidepressants, warfari n, to lbutamide. oral contraceptives. chloramphenicol. doxycycline, theophylline, g ri seofulvin, nev irapine.

This refers to metabolism of a d rug duri ng its passage from the site of absorption into the systemic circulation. A II orally administered drugs arc exposed to drug metabo li zing enzymes in the intestinal wa ll and li ver (where they first reach through the po rta l vein). Presystemi c metabolism in the gut and liver ca11 be avoided by administering the drug through sublingual , transdermal o r parenteral routes. However. limited presystemic metabolism ca n occur in the skin (transdermally administered drug) and in lungs (for drug reaching venous blood through any route). The extent of first pass metabo lism differs for different drugs (Table 3.2) and is an important detem1inant of oral bioavailability. A drug can also be excreted as s uch into bile. The hepatic extraction ratio (ERliver)of a drug is the fraction of the a bsorbed drug prevented by the liver from reaching systemic c irculation. Both presystemic metabolism as well as direct excreti on into bi le dete rmine ERL,..,,, which is given by the equation:

ER

.. ( 1) Hepal ic blood flow

Accordingly the systemic bioavailability (F) of an orally admini tered drug w ill be:

F = fractional absorption

x (I- ER)

..(2)

When a drug wi ith high first pass metabolism is givenorally at higher dose to achieve therapeutic blood le,els. the plas ma concentration of its metabolites will be much higher compared to those resulting from parenteral dosing of the drug for the same therapeutic level If the metabolites contribute 10 the adversc effects of the drug, ora l dosing willbe less safe than parenteral.

METABOLISM AND EXCRET ION OF DRUGS, KINETICS OF ELIM INATION

Table 3.2: Extent of hepatic first pass metabolism of some important drugs High

Low

Intermediate not given orally

high oral dose

Phenobarbitone Phenylbutazone

Aspirin Quinidine

lsoprenaline Lidocaine

Propranolol

Tolbutamide Theophylline Pindolol Diazepam

Desipramine Nortriptyline Chlorpromazine Pentazocine

Hydrocortisone Testosterone

lsosorbide mononitrate

Metoprolol

Attributes of drugs with high first pass metabolism: (a) Oral dose is considerably higher than sublingual or parenteral dose. (b) There is marked individual variation in the oral do e due to differences in the extent of first pass metabolism. (c) Oral bioavailability is apparently increased in patients wi th severe liver d isease. (d) Oral bioavailability of a drug is inc reased if a nother drug competi ng w ith it in first pass metabolism is given concurrently, e.g. c hl orpromazine and propra nolol.

EXCRETION Excretion is the passage o ut or system icall y absorbed drug. Drugs and the ir metabolites are excreted in: 1. Urine Drug excretion in urine occurs via the kidney. It is the most important channel of excretion for majority of drugs (see below). 2. Faeces Apart from the unabsorbed fraction , most of the drug present in faeces is derived fro m bile. Liver acti ve ly transports into bile organic acids (especia lly drug glucuronides by OATP and MRP2), organic bases (by OCT), other lipoph ilic drugs (by P-gp) and steroids by di stinct nonspecific active transport mechan isms. Relatively larger molecules (MW > 300) are preferentially eliminated in the bile. Most of the free d rug in the gut, including that re leased by deconjugation of glucuronides

Alprenolol Verapamil Salbutamol Glyceryl trinitrate Morphine Pethidine

by enteric bacteria is reabsorbed (enterohepatic cycling) and ultimate excretion occurs in urin e (Fig. 3.2). Only the remaining is excreted in the faeces. Enterohepatic cycling contributes to longer stay of the drug in the body. Drugs that attain hig h concentrations in bile include erythromycin. ampicillin. rifampin, tetracycl ine, oral contraceptives, vecuronium, phenolphthalein. Certain drugs are excreted d irectly in colon, e.g. anthracene purgatives, heavy metals.

3. Exhaled air

Gases and volatile liquids (general anaesthetics, alcoho l) are elimi nate d by lungs, irrespective of thei r li pid so lubi li ty. Alveolar tran sfer of the gas/ vapour depend s on its partial pressure in the b lood. Lungs also serve to trap and extrude any particulate matter that enters circu lation.

4. Saliva and sweat These are of minor importance fo r drug excretion. Lithium, pot. iod ide, ri fampi n and heavy meta ls are present in these secretions in significant amounts. Most of the saliva along with the drug in it. is swallowed and meets the same fate as orall y taken drug.

5. Milk

The excretion or drug in milk is

not important for the mother. but the suck ling

infan t inadverten tly receives the drug. Most drugs enter breast milk by passive diffusion. As such. more lipid soluble and less protein bound drugs cross better. Milk has a lower pH (7.0) than plasma, basic drugs are somewhat more concentrated in it. However, the to ta l amount or

35

36

GENERAL PHARM ACOLOGY

ENTEROHEPATIC CYCLING OF DRUGS

Faeces Fig. 3.2: Enlerohepatic cycling of drugs In the liver many drgus (D), including steroids, are conjugated by the enzyme UDP-glucuronosyl transferases (UGTs) to form drug-glucuronide (DG). Part of the DG enters systemic circulation and is excreted into urine by the kidney through both glomerular filtration (GF) as well as active tubular secretion involving renal organic-anion transporting peptide (OATP). Another part of DG is actively secreted into bile by the hepatic OATP. On reaching the gut lumen via bile, a major part of DG is deconjugated by becterial hydrolytic enzymes (deconjugases) while the remaining is excreted into faeces. The released D is reabsorbed from the gut to again reach the liver through portal circulation and complete the enterohepatic cycle.

drug reaching the infa111 through breast feeding is generally small and maj ority o f drugs can be given to lactating mothers without ill effects on the infant. cvertheless, it is advisable to admin ister any drug to a lactating woman o nly when essentia l. Drugs that are safe, as well as those contraind icated d uring breast reedi ng o r need special caution a re given in Appendix-] at the end of the book.

RENAL EXCRETION T he kidney is respo nsible for excreti ng all water solubl e s ubstances. The amou nt of drug or its metabolites ultimately present in urine is the s um total of g lomerular fi ltration , tubular reabsorption and tubular secretion (Fig. 3.3).

et renal excretion

(Glomerular filtration + tubular ccrction) - tubular reabsorption

Glomerular filtration

Glomerular capillaries have pores larger than usual; al l no nprotei n bound drug ( whether lipid-solub le or inso luble) presented to the g lomerulus is fi ltered. Thus, glomeru lar filtration of a drug depends 011 its plasma protein binding and renal blood flow. G lomcrular filtration rate (g.f.r.), normall y ~ 120 ml/min, declines progressively after the age of 50, and is low in renal failure. G lomerular fi ltration of drugs dec li nes in parallel.

Tubular reabsorption Th is occurs by passive diffusion and depends o n the lipid solubili ty

METABOLISM AND EXCRETION OF DRUGS, KINETICS OF ELIMINATION

Afferent arteriole

~ ~=::~-- Efferent arteriole

Tubular secretion

Plasma protein

Tubular cell

outcome. T he effect of changes in urinary pH on d rug excretion is greatest for those having pKa val ues between 5 to 8. because only in their case pH dependent pass ive rcabsorption is significant.

-+I- ~ Peritubular

vessel

Urinary - - - - pH

Fig. 3.3 : Schematic depiction of glomerular filtration, tubular reabsorption and tubular secretion of drugs FD-free drug; BO-bound drug; UD-unionized drug; ID-ionized drug; Ox-actively secreted organic acid (or base) drug

and ionization of the drug at the existing urinary pH. Lipid-soluble drugs filtered at the glomeru lus back diffuse in the tubules because 99% of glomerular filtrate is reabsorbed, but nonlipid-soluble and high ly ionized drugs are unable to do so. Thus, rate of exc retion of such drugs, e.g. aminoglycos ide antibiotics, quaternary ammonium compounds parallels g.f.r. (or creatinine clearance). Changes in urinary pH affect tubular reabsorption of drugs that arc partially ionized• Weak bases ionize more and are less reabsorbed in acidic urine. • Weak acids ionize more and are less reabsorbed in alkaline urine. This principle is utilized for facilitating e lim ination of the drug in poisoning. i.e. urine is alkalinized in barbiturate and sal icylatc poisoning. T hough elimination of weak bases (morphine, amphetamine) can be e nhanced by acidifying urine, this is not practiced clinical ly, because acidosis can induce rhabdomyolysis, cardiotoxicity and actually worsen

This is the acti ve transfer of organic acids and bases by two separate classes of relatively nonspec ific trans porters (OAT an d OCT) wh ic h ope rate in the proximal tubules. In add ition , e mux transporters P-gp and MRP2 are located in the luminal membrane of proximal tubular cells. If renal c learance of a drug is greate r than 120 m L/ min (g.f.r.). add itional tubu lar secretion can be assumed to be occurring. Active transport of the drug across tubules reduces concentration of its free form in the tubu lar vessels and promo tes dissoc ia ti o n of protein bound drug, which then becomes available for secretion (Fig. 3.3). Thus, protein binding, which is a hindrance for g lo merular filtration of the drug, is not so (may even be fac il itatory) to excretion by tubular secretion. (a) Organic acid transport (through OAT P) operates fo r pe nicillin, probenecid , uric acid, salicylates, indomethacin, sulfinpyrazone, nitrofurantoi n, methotrexate, drug glucuronides and su Ifates. ere. (b) Organic base transport (through OCT ) operates for thiazides, am il oride. triam terene, furosemi de, quini ne, procainamide, c ho line, eimetidine, etc. Inherently both transport processes are b idirectio na 1, i.e. they ca n transport their s ubstrates from blood to tubular nu id and vice versa. However, for drugs and their me tabolites (exogenous s ubs tances) secretion into the tubu lar lumen predomi nates, w hereas a n e ndogenous substrate like uric acid is predominantly reabsorbed. Drugs utili zing the same act ive transport compete w ith each other. Probenecid is an o rganic acid which has high affinity for the tubular OATP. It blocks the active transport o f both penicilli n a nd uric acid, bu t whereas the net

37

38

GENERAL PHARMACOLOGY excretio n of the fo rmer is decreased, that of the latter is increased. This is because pe nicillin is primarily secreted whi le uric acid is primarily reabsorbed. Many drug interactions occur due 10 competition for tubular secretion, e.g. (i) Aspirin blocks uricosuric action of probenecid an d sulfinp yrazone and decrease tubular secretion of metho rrexatc. (i i) Pro benecid decreases the concentration of nitrofurantoi n in urine, increases the duration o f action of pen icillin/am pici llin and impairs secretion of methotrexate. (iii) Sulfinpyrazone inhibits excretion oftolbutam ide. (iv) Quinidine decreases renal and biliary c learance of digoxin by inhibiting efflux carrier P-gp. T ubular transport mechanisms are not well developed at birth. As a result, duration of action of many drugs, e.g. penicillin, cephalospori ns, aspirin is longer in neonates. These systems mature during infancy. Renal function aga in progress ively declines after the age of 50 years; rena l c learance of most drugs is substantially lower in the elderly (> 75 yr).

KINETICS OF ELIMINATION The knowledge of kinetics of elimination of a drug provides the basis for, as well as serves to devise rational dosage regimens and to modify them according to individual needs. T here are three fu ndamental pharmacokinetic parameters, viz bioavailability (F), volume of d istribut ion ( V) and clearance (Cl) which must be understood. The first two have a lready been considered. Drug elimination is the sumtotal of metabolic inactivation and excretion. As depicted in Fig. 2. 1, drug is e liminated only from the central com partment (blood) w hich is in eq uilibrium with peripheral compartments including the site of action. Depending upon the ability of the body to eliminate a drug, a certain fractio n of the central compartment may be considered to be totally 'cleared' of that drug in a g iven period of time to account for elimination over that period.

Fig. 3.4: Illustration of the concept of drug clearance. A fraction of the drug molecules present in plasma are removed on each passage through the organs of elimination. In the case shown, it requires 50 ml of plasma to account for the amount of drug being eliminated every minute: clearance is 50 mUmin

Clearance (CL)

The clearance of a drug is the theoretica l volume of plasma from which the drug is complete ly removed in unit time (analogy creatinine c learance; Fig. 3.4). It can be calcul ated as:

CL = Rate of elimination/C

... (3)

where C is the plasma concentration. For majority of drugs the processes involved in e limination are not saturated over the clinically obtained concentrations, they follow:

First order kinetics The rate of e limi nation is directly proportional to the drug concentration, CL remains constant; or a constant fraction of the drug pre ent in the body is eliminated in unit time. Th is applies to majority of drugs which do not sa turate the e li mination processes (transporters, enzymes. blood flow, etc.) over the therapeutic concentration range. However, if the dose is high enough. eliminati on pathways of all drugs will get saturated. Few drugs normally satu rate e li m inating mechanisms and are hand led byZero order kinetics The rate of elimination remains constant irrespective of drug concentration, Cl dec reases with increase in concentration; or a constant amount of the drug is eliminated in unit time, e.g. ethyl alcohol. This

39

METABOLISM AND EXCR ETI O N OF DRUGS, KIN ETICS OF ELIMINATION

is also called capaci1y limited elimination or Michaelis-Memen elimination. The e limination of some drugs approaches saturation over the therapeutic range, kinetics changes from first order to zero order at higher do es. As a result plasma concentrati on increases disproportionately with increase in dose (see Fig. 3.6), as occurs in case of phe nytoin, tolbutamide, theophylline. warfarin. Since the dose rate-plasma concentration plot in this case is curved, it is also called ·nonlinear elimination'.

64 32

E ? 16 ~i ozB a

Q Ol (.) a

d .J

4 2

--r- - ,-__ 1 __ 1 _ _

1 __

0.

Blood flow dependent elimination For few drugs the e liminating capac ity of a n organ of elimination (kidney, liver) far exceeds the amount of drug normally presented to it by blood circulation. The elim ination of such a drug beco mes blood flow dependent. These highly extracted drugs are a lmost completely e liminated in a si ngle passage through the organ of elimination.

Plasma half-life The Plasma half-life (t½) of a drug is the time taken for its plasma concentrat ion to be reduced to half of its o riginal value. Taking the s implest case of a dru g wh ich has rapid one compartment di stribution and first order elimination, and is given i.v. a semilog plasma concentration-t ime plot as shown in Fig. 3.5 is obtained. The plot has two slopes. • initial rapidly declining (a.) phase- due to distribution. • later less declined (B)phase- due to elimination. At least two half-lives (distribut ion t ½ and e limination t ½) can be ca lc ulated from the two slopes. The elimi nation half life derived from the B slope is simply called the ' half Ii fe · of the drug. Most drugs infact have multico111part111en1 distribution and multiexponential decay of plasma concentration-time plot. Half-lives calculated from the te rminal slopes (when plasma concentrations are , ery low)are excepttonally long. probably due to release of the drug from slow equilibrating tissues enterohepatic circulation. etc. Only the t½ calculated over the steady-state plasma concentration range is clinically relevant. It is this t½ which is commonly mentioned.

2

4

6

8

10

Time (Hours) Fig. 3.5: Semilog plasma concentration-time plot of a drug eliminated by first order kinetics after intravenous injection

Since first order kinet ics is an exponentia l process, mathematically, the elimination t ½ is ln2 t½=--

...(4)

"

Where ln2 is the natural logarithm of 2 ( or 0.693) a nd k is the e/imina1io11 rate cons1a11t of the drug, i.e. the fraction of the total amount of drug in the body which is removed per unit time. For example, if 2 g of the drug is present in the body and 0. 1 g is eliminated every hour, then

k = 0.1 /2 = 0 .05 or 5% per hour. It is calculated as: CL ... (5)

k

V V

therefore



0.693

X--

...(6)

CL

As such, hal f-life is a derived parameter from two variables V and CL, both of which may change independently. It. therefore, is not a n exact index of drug elimination. Neverthe less,

40

GENERA LPHAR MACO LOGY it is a simple and useful gu ide to the sojourn o f the d rug in the body, i.e. after I t ½- SO¾ drug is e liminated. 2 t½- 7S% (SO + 25) drug is e li minated. 3 t1/2-87.5% (50 + 25 + 12.5) drug is eliminated. 4t 1/2-93.75% (SO + 25 + 12.5 + 6.25) d rug is e liminated. Thus, nearly complete drug e limination occurs in 4- 5 half lives. For drugs eli minated byFirst o rder kineti cs-t½ remains constant because V and CL do not change with dose. Zero o rder ki netics- t ½ gets prolonged with do e because CL progressively dec rea ses as dose is increased. In fact, the concept of t ½ is meaning less fo r such drugs, because, it is no t a fi xed va lue. Half life of some representative drugs

Aspirin Penicillin-G

4 hr

Doxycycline

20 hr

30

40 hr 7 days

Phenobarbitone

90 hr

min

Repeated drug administration W he n a drug is re peated a t re lat ive ly short in te rva ls, it accumu lates in the body unt il elim ination balances input a nd a steady state plasma concentration (Cpss) is attained-

target Cpss x CL

= ---

...(9) F

W he n constant dose of a drug is repeated be fore the ex p iry of 4 t ½, it would ac hi eve hig her peak concentration, because some remnant of the previous dose will be present in the body. This continues with every dose until progressively increas ing rate of eli minati on (which increases w ith inc rease in concentra ti on) ba lances the amou nt adm ini stered over the dose inter va l. Subseq uentl y plasma concen tration plateaus

I I (/)

(/)

Cl.

u

O>

...(8)

A ller ora l ad min istration, o ften o nly a fraction (F) of the dose reaches systemic circulation 111 the active fom1. In such a case-Dose rate

Km + C where C is the plasma concen tratio n of the drug, Vmax .. is the maximum rate of d rug elimination, and K., is the pl asma co ncentrati on at w hich e limination rate is ha lf max imal.

... (7)

From this equatio n it is imp lied that doubling the dose rate would do uble the average Cpss and so on . Further, if the the rape utic plasma concentration of the d rug has been worked out and its Cl is known. the dose rate needed to ach ieve the target Cpss ca n be determinedDose rate = target Cpss x CL

( Vmax)(C) Rate o f drug eliminatio n - - - - - ... ( I 0)

Plateau principle

Digoxin Digitoxin

dose rate Cpss = - - -CL

T he dose rate-Cpss relationship is linear o nl y in case of drugs e li m inate d by first order kin e t ics. Fo r dru gs (e .g . p he nyto in) wh ic h fo llow Michae lis Me nten kin e tics, e limin ation changes from first order to zero o rder kinetics over the therape utic range. Increase in the ir dose beyond satu ra ti o n leve ls causes an increase in Cpss w hich is o ut of pro po rti o n to the cha nge in dose ra te ( F ig. 3.6). In the ir case:

Dose rat Fig. 3.6: Relationship between dose rate and average steady-state plasma concentration of drugs eliminated by first order and Michaelis Menten (zero order) kinetics

41

METABOLISM AND EXCRETIO N OF DRUGS, KINETICS OF ELIMINATION

and fluctua tes a bout an average steady-state leve l. Th is is known as the plateau principle of drug accumulation. Steady-state is reached in 4 5 half lives un less dose interval i very much longer than t½ (Fig. 3.7).

2

- -

C

- ~

- Maximum

Q ·1;, ' rr " ·•· ·,:,,,,

'§ 1.5

c

- -

- -

Average Minimum

C

8

1

(1)

E

NA> lso

lso > Adr > NA

2. Antagonist

Phentolamine

Propranolol

3. Coupling protein

Gq/Gi/Go

Gs

4 . Effector pathway

IPj DAGt, cAMP . K· channel t

cAMPi, Ca2 • channel

A

• Though inherently NA is equipotent to Adr on • receptors, in test systems with intact neuronal reuptake, it appears less potent due to faster reuptake. On the basis of relative organ specifici ty of selective agonists and antagonists, the P receptors were further subdivided into p1 and p2 subty pes. Later, P3 (atypical P) receptors were described which are more sensi ti ve to A than to Adr, and have very low affinity for the classical blockers. The Pi receptors are located on adipocytes, mediate lipolysis and

induce thermogenesis. Selective P3 agonists have been investigated as antiobesity drugs. Another location of P, receptors is on detrusor muscle of urinary bladder; activation causing relaxation. A relatively Pi selecti ve agonist mirabegron has been recently found useful in the symptomatic treatmen t of overactive bladder by relaxing the detrusor muscle.

Table 9.2: Differences between 11,, 11 and 11 , receptors

/3, 1. Location

Heart, JG cells in kidney

/3, Bronchi, blood vessels. uterus, liver, g.i.t., urinary tract, eye

/JJ Adipose tissue, Detrusor muscle of bladder

2. Selective agonist

Dobutamine

Salbutamol, terbutalin

Mirabegron

3. Selective antagonist

Metoprolol, Atenolol

ICI 118551 a-methyl propranolol

CGP 20712A (also ll,} ICI 118551 (also I\}

4 Relative potency of NA and Adr

Adr

Adr » NA

NA> Adr

NA

In the mid I970s the a receptors were demonstrated to be present prejunctionally as well. To differentiate these release inhi bitory prejunctional a receptors, a subdivision into a I and a 2 was suggested. However, the present classification into a 1 and a 2 is based on pharmacological criteria (selectiv ity of agonists and antagonist ) and not on anatomical location. Thus, a 2 receptors are located prejunctionally as well as postjunctionally in certain tissues. The vascu lar smooth muscle cells express both a 1 and a 1 receptors, and NA causes vasoconstricti on by activating both subtypes. Molecular

cloning has further identified 3 subtypes o f a 1 (atA' a I8, a t0) and 3 subty pes of a 2 (a2A, a 211, ox) receptors. Though tissue di st ribution o f sub1ypcs of a, and a , receptors has been mapped, there is lot of o, erlap. St1fficien tly subtype selective agonists or antagoni sts have also not yet been developed 10 pharmacologically exploit the molecular heterogenei ty of subtypes of a , and er., receptors.

The adrcnergic neuronal mechani sm s and action of drugs which modify them are depicted in Fig. 9.4. A summary of drugs acting through adrenergic neuronal mechanisms is presented in Table 9.4

140

DRUGS ACTING ON ANS

Table 9.3: Differences between " · and u receptors a, Location

Postjunctional on effector organs

Function subserved

GU Smooth muscle-500)

Thioperamide (H,: H3 1: 23000) lmpromidine (also H2 agonist) Tiprolisant, Clobenpropit

3. Receptor type

Gq-protein coupled

Gs-protein coupled

Gi/Go-protein coupled

4. Effector pathway

PIP2 hydrolysis • IP/ DAG : Release of Ca2• from intracellular stores; Protein kinase-C activation NO release• cGMP

Adenylyl cyclase activation cAMP t -phosphorylation of specific proteins

a. Smooth muscle (intestine, airway, uterus)--

z

0

b. Blood vessels (smooth muscle} dilatation c. Heart Atria: +ive chronotropy Ventricles: +ive inotropy d. Uterus (rat}-relaxation e. Brain (postsynaptic)-impulse

a. Brain (presynaptic)-inhibition of histamine release-sedation b. Lung, spleen, skin, gastric mucosa -decrease histamine release c. Ileum-inhibition of ACh release from myenteric plexus neurones d. Certain blood vessels-inhibit NA release-vasodilatation

)>

z

-I

I l/)

$

z n

V>

PIP,-Phosphatidyl inositol bisphosphate; IP,-lnositol trisphosphate; DAG- D1acylglycerols; NO-Nitric oxide; PGl 2-Prostacyclin; CAs-Calecholamines; cAMP -Cyclic 3', 5' adenoslne monophosphate; ACh-Acetylcholine

t:13.ld'1H3

.u, .....

176

AUTACOIDS AND RE LATED DRUGS

Antigen

FccRI

Granule Na• or Ca2•

Hist

Fig. 11 .2: Mechanism of antigen-antibody reaction induced release of histamine from mast cell In sensitized atopic individual, specific reaginic (lgE) antibody is produced and gets bound to Fe epsilon receptor I (F~ RI) on the surface of mast cells. On challenge, the antigen bridges lgE molecules resulting in transmembrane activation of a tyrosine-protein kinase (t-Pr-K) which phosphorylates and activates phospholipaseCy. Phosphatidyl inositol bisphosphate (PIP2 ) is hydrolysed and inositol trisphosphate (IP3) is generated which triggers intracellular release of Ca 2•• The Ca2• ions induce fusion of granule membrane with plasma membrane of the mast cell resulting in exocytotic release of granule contents. In the granule, positively charged histamine (Hist+) is held complexed with negatively charged protein (Prot') and heparin (Hep-) molecules. Cationic exchange with extracellular Na• (and Ca2 ·) sets histamine free to act on the target cells.

considered to be a specific 11, agonist, has shown greater affi nity and selectivity for tlic H, receptor, and is now labelled a ll, agonist. Eosinophils, mast cells and basophils are the primary cells expressing H, receptors. Activation of H, receptors enhances chemotaxis of these cell s. The H, receptor may be playing a ro le in allergic inflammation. Some H, antagonists are being explored as potential drugs for allergic inflammatory conditions like rhinitis and asthma. Intestines and brain are the other sites where H, receptors have been located.

PHARMACOLOGICAL ACTIONS 1. Blood vessels

Histamine causes marked dilatation of smaller blood vessels, including

arterioles, capi llaries and venu les. On s.c. injection flushing, especially in the bl ush area, heat, increased heart rate and cardiac output, with little or no fall in B P are produced. Rapid i.v. injection causes fall in BP which has an early short lasting H I and a slow but more persistent H2 component. With low doses only the H 1 component is man ifest since H 1 receptors have higher affinity. Fall in BP due to large doses is compl etely blocked only by a combination of H 1 and H2 antagonists. Dilatation of cranial vessels causes pulsatile headache. Like many other autacoids and ACh, vasodilatati on caused by histamine is partly (H 1 co mponent) indi rect, mediated through 'endothelium de pe nde nt relaxing factor ' (E DR F), i.e. NO; the receptor being located on the endothelial cells. H2 receptors mediating vasodilatatio n are located d irectly on the vascular smooth muscle. Larger arteries and veins are constricted by histamine. This is med iated by H 1 receptor on vascu lar smooth muscle. Histamine also causes increased capillary permeability due to se paration of endothelial cells resulting in exudation of plasma. This is primarily a H 1 response. Injected intradermally, it e licits the triple response consisting of: Red spot: due to intense capillary dilatation. Whea l: due to exudation of fluid from capillaries and venules. i.e. redness in the s urrounding Flare: area due to arteriolar dilatation mediated by axon reflex.

2. Heart

Direct effects of histamine on in situ heart are not prominent, but the isolated heart, especially of guinea pig, is stimulated- rate as well as force of contrac tion is increased. These are primarily H 2 responses but a H I mediated negati ve dromolropic (slowing of A-V conduction) effect has also been demonstrated. 3. Visceral smooth muscle Histamine causes bronchoconstriction; guinea pigs and patients of asthma are highly sensitive. Large doses cause abdominal cramps and co lic by inc reasi ng intestinal contractions. Guineapig uterus is contracted wh ile that of rat is relaxed; human uterus is not much affected as are most other visceral smooth muscles.

HISTAM INE AND ANTIHISTAM INI CS

Smooth muscle contraction is a H I response. In few instances H2 mediated relaxation is also seen, e.g. bronchial muscle of sheep, human bronchi after H 1 blockade when H 2 response is unmarked.

4. Glands Histamine causes marked increase in gastric secretion- primarily of acid but also of pepsin (see Ch. 47). This is a direct action exerted on parietal cells through H2 receptors, and is mediated by increased cAMP generation, which in turn activates the membrane proton pump (W K• ATPase). Histamine can increase other secretions also, but the effect is hardly discernable. 5. Sensory nerve endings ltching occurs when histamine is injected i.v. or intracutaneously. H igher concentrations injected more deeply cause pain. These are reflections of the capacity of histamine to stimulate nerve endings. 6. Autonomic ganglia and adrenal medulla These are stimulated and release ofAdr occurs, which can cause a secondary rise in BP.

7. CNS

Histamine does not penetrate bloodbrain barrier- no central effects are produced on i. v. injection. However, intracerebroventricular administration produces rise in BP, cardiac stimulation, behavioural arousal, hypothermia, vomiting and ADH release. These effects are mediated through both H I and H2 postsynaptic receptors.

PATHOPHYSIOLOGICAL ROLES 1. Gastric secretion Histamine has dominant physiological role in mediating secretion of HCI in the stomach (see Fig. 47. 1). onmast cell histamine occurs in gastric mucosa, possibly in cells called 'histaminocytes' situated close to the parietal cells. This histamine has high turnover rate. It is released locally under the influence of all stim uli that evoke gastric secretion (feeding, vagal stimulation, cholinergic drugs and gastrin) and activates the proton pump (WK• ATPase) through H2 receptors. H2 blockers not only suppress acid secretion induced by histamine but also markedly dimin ish

that in response to ACh and gastrin. By a mutually synergistic interaction the three secretagogues amplify responses to each other with histamine playing the dominant role. As such. antimuscarinic drugs dampen the response to histamine and gastrin as well. All three secretagogues activate the same proton pump (WK•ATPase) in the parietal cell membrane, but through their own receptors.

2. Allergic phenomena Mediation of hypersensitivity reactions was the first role ascribed to histamine. It is an important, but only one of the mediators of such phenomena. Released from mast cells following AG : AB reaction on their surface (involving IgE type of reaginic antibodies; Fig. 11.2) in immediate type of hypersensitivity reactions, histamine is causative in urticaria, angioedema, bronchoconstriction and anaphylactic shock. The H I antagonists are effective in controlling these manifestations to a considerable extent, except asthma and to a lesser extent anaphylactic fall in BP in which leukotrienes (especially LTDJ and PAF appear to be more important. Histamine is not involved in delayed or retarded type of allergic reactions. 3. As transmitter Histamine is believed to be the afferent transmitter which initiates the sensation of itch and pain at sensory nerve endi ngs. Nonmast cell histamine occurs in brain, especially hypothalamus and midbrain. It is involved in maintaining wakefulness; H1 antih istaminics owe their sedative action to blockade of th is function. In the brain H1 agonism suppresses appetite. This may explai n the appetite promoting action of certain H 1 antagonists. Histamine also appears to participate as a transmitter regulating body temperature, cardiovascular function, thirst, and possibly other functions, mainly through H2 (postsynaptic receptors) and H3 (presynaptic autoreceptors and heteroreceptors). 4. Inflammation Histamine is a medi ator of vasodi latation and other changes that occur during inflammation. It promotes adhesion of leukocytes to vascular endothelium by expressing adhesion molecule P-se/ectin on endothelial cell surface, seq uestrating leukocytes at the

177

178

AUTACOIDS AND RELATED DRUGS inflammatory site. It may also regulate microcirculation according to local needs. 5. Tissue growth and repair Because gro" ing and regenerating tissues contain high concentrations of histamine, it has been suggested to play an essential role in the process of growth and repair.

6. Headache Histamine has been implicated in certain vascular headaches, but there is no conclusive evidence. USES Histamine has no therapeutic use. In the past it used to test acid secreting capacity of stomach, hypcrrcactivity in asthmatics, and for diagnosis chromocytoma, but these pharrnacolog1cal tests and obsolete now.

has been bronchial of pheoare risky

Betahistine It is an orally active, somewhat H1 selecti ve histamine analogue, which is used to control ve1tigo in patients of Meniere's di sease. It possibly acts by causing vasodilatation in the internal ear. It is contraindicated in asthmatics and ulcer patients. VERT"

8 mg tab., 1/2 to I tab. QID.

HISTAMINE RELEASERS A variety of mechanical, chemical and immunological stimuli are capable of releasing histamine from mast cells. I. Tissue damage: trauma, stings and venoms, proteolytic enzymes, phospholipase A. 2. Antigen: antibody reaction involving lgE antibodies. 3. Polymers like dextran. polyvinyl pyrrolidone (PVP). 4. Some basic drugs- tubocurarine, morphine, atropine. pcntamidine, po lymyxin 8 , vancomycin and even some antihistaminics release histamine by displacing it from the binding site, and not by an immunological reaction. This release is not exocytotic, and does not require Ca 2' . 5. Surface acting agents like Tween 80, compound 48/80 etc. The primary action of these substances is release of histamine from mast cells, therefore they are called 'histamine liberators'. They produce an 'anaphylactoid' reaction- itching and burning sensation, flushing, urticaria, fall in BP, tachycardia, headache, colic and asthma.

H1 ANTAGONISTS (Conventional antihlstamlnics) These drugs competitively antagonize actions of histamine at the H 1 receptors. Recent evidence indicates that histamine H 1 receptor exhibits some degree of constitutive activity at certain sites, and few H I antagonists are also inverse agonists. The first H 1 antagonists were introduced in the late

1930s and have subsequently proliferated into an unnecessary motley of drugs. Nevertheless, they are frequently used for a variety of purposes. More commonly employed now are the less sedating/nonsedating second generation H 1 antihistamines added after 1980. Seemingly, H 1 antihistaminics have diverse chemical structures, but majority have a substi tuted ethylamine side c hain. They are classified and listed in Table 11.2.

PHARMACOLOGICAL ACTIONS Qualitatively all H 1 antihistaminics have similar actions, but there are quantitative differences, especially in the sedative property.

1. Antagon ism of histamine The H 1 antagonists effectively block histamine induced bronchoconstriction, contraction of intestinal and other smooth muscle and triple responseespecially wheal, flare and itch. Fall in BP produced by low doses of histamine is blocked, but additional H2 antagonists are required for complete blockade of that caused by higher doses. Pretreatment with these drugs protects animals from death due to i.v. injection of large doses of histamine. Release of Adr from adrenal medu lla in respon se to histamine is abolished. Constriction of larger blood vessel by histamine is also antagon ized. Action of histamine on gastric secretion is s ingul arly not affected by these drugs. Cyproheptadine had additional 5-HT2 receptor blocki ng activity (see p. 189).

2. Antiallergic action Many manifestations of immediate hypersensitivity (type I reactions) are suppressed. Urticaria, itching and angioedema are well controlled. Anaphy lactic fall in BP is only partially prevented. Asthma in man is practically unaffected, though anaphylactic bronchoconstriction in guinea pig is largely prevented. This tissue and species dependence of response probably reflects extent of involvement of histamine in the reaction. It is now well established that leukotrienes (LTC4, LTD4) and PAF are more important med iators for human asthma.

HISTAMIN E AND ANTIHISTAMINICS

Table 11 .2: Clinical class1flcat1on. doses and preparations of H, antih1stamirncs Drug

Dose and route

Preparations

Diphenhydramine

25-50 mg oral

BENADRYL 25, 50 mg cap., 12.5 mg/5 ml syr.

Dimenhydnnate

25-50 mg oral

DRAMAMINE 16 mg/5 ml syr, 50 mg tab, GRAVOL 50 mg tab

25-50 mg oral, i.m. (1 mg/kg)

PHENERGAN 10, 25 mg tab., 5 mg/ml elixir, 25 mg/ml inj

25-50 mg oral, i.m.

ATARAX 10, 25 mg tab., 10 mg/5 ml syr, 6 mg/ml drops, 25 mg/ml inj.

I. HIGHLY SEDATIVE

Promethazine

Hydroxyzine

II. MODERATELY SEDATIVE Pheniramine

20-50 mg oral, i.m.

AVI L 25 mg, 50 mg tab, 15 mg/5 ml syr, 22.5 mg/ml inj.

Cyproheptadine

4 mg oral

PRACTIN, CIPLACTIN 4 mg tab., 2 mg.IS ml syrup

Meclozine (Meclizine)

25-50 mg oral

In DILIGAN 12.5 mg + niacin 50 mg tab In PREGNIDOXIN 25 mg + Caffeine 20 mg tab

Cinnarizine

25-50 mg oral

STUGERON, VERTIGON 25 and 75 mg tab.

Chlorpheniramine

2-4 mg (0.1 mg/kg) oral, i.m.

PIRITON, CADISTIN 4 mg tab

Dexchlorpheniramine

2 mg oral

POLARAMINE 2 mg tab, 0.5 mg/ 5 ml syr

Triprolidine

2.5-5 mg oral

ACTIDIL 2.5 mg tab.

Clemastine

1- 2 mg oral

TAVEGYL 1 mg tab., 0.5 mg/5 ml syr

Ill. MILDSEDATIVE

IV. SECOND GENERATION ANT/HISTAMIN/CS Fexofenadine

120-180 mg oral

ALLEGRA, ALTIVA, FEXO 120, 180 mg tab

Loratadine

10 mg oral

LORFAST, LORIDIN, LORMEG, 10 mg tab, 1 mg/ml susp.

Desloratadine

5 mg oral

DESLOR, LORDAY, NEOLORIDIN 5 mg tab

Cetirizine

10 mg oral

ALERID, CETZINE, ZIRTIN, SIZON 10 mg tab, 5 mg/5 ml syr.

Levocetirizine

5-10 mg oral

LEVOSIZ, LEVOAID, TECZINE 5, 10 mg tab LEVOCET 5 mg tab, 2.5 mg/5 ml syr.

Azelastine

4 mg oral 0.28 mg intranasal

AZEP NASAL SPRAY 0.14 mg per puff nasal spray

Mizolastine

10 mg oral

ELINA 10 mg tab

Ebastine

10 mg oral

EBAST 10 mg tab

Rupatadine

10 mg oral

RUPAHIST 10 mg tab

179

180

AUTACOIDS AND RELATE D DRU GS

3. CNS The older antihistamines produce variable degree of C S depression. This appears to depend on the compound's ability to penetrate the blood-brain barrier and its affi nity for the central (compared to peripheral) HI receptors. Individual susceptibility to different agents varies considerably. The same drug and dose may incapacitate some subjects, whi le others may remain alert. An overall grading of the sedative property of H 1 antihistaminics is presented in Table 11.2. Some individuals also experience stimulant effects like restlessness and insomnia. Excitement and convulsions are frequently seen at toxic doses. The second generation antihistaminics are practicall y nonsedating. Certain (see below) H I antihistamines are effective in preventing molion sickness. It is not clear whether this is due to antagonism of histamine in the brai n or reAects antimuscarinic property of these drugs. Promethazi ne also controls vomiting of pregnancy and other causes. Promethazi ne and few other antihistaminics reduce tremor, rigidity and sialorrhoea of parkinsonism. Anticholinergic and sedative properties under! ie the benefit. Some olde r antihistami nes, especia lly cyproheptadine, have appelile s1imu/01ing efTect. Some HI antihistamines are also e ffective antitussives (see Ch. 16). 4. Anticholinergic action Many H 1 blockers in addition antagonize muscarinic actions of ACh. The antimuscarinic action can be graded as: High

Low

M inimaV A bsent

Promethazine Diphenhydramine Dimenhydrinate Pheniramine

Chlorpheniramine Triprolidine Cyproheptadine Cinnarizine

Fexofenadine Astemizole Loratadine Cetirizine Mizolastine

If used concurrently wi th atropine or its substitutes, phenothiazines, tricyclic antidepressants or disopyramide. the anticholinergic action adds up. 5 . Local anaesthetic Some drugs like pheni raminc, promethazine, di phenhydrami ne have strong while others have weak membrane stabilizing property. However, they are not

clinically suitable local anaesthetics because they cause irritation when injected s.c. Membrane stabilizing activity confers antiarrhythmic property to these compounds. 6 . BP Most antihistami nics cause a fall in BP on i.v. injection (direct smooth muscle relaxation or a adre nergic blockade). Promethazine has significant ex blocking activity. However, this is not evident on oral administration, though postural hypotension can occur in susceptible individuals. PHARMACOKINETICS

The conve ntional HI antihista minics are well abso rbed from ora l and parenteral ro utes, metabolized in the liver and excreted in urine. They are widely distributed in the body and enter brain. The newer compounds penetrate brain poorly accounting for their low/absent sedating action. Duration of action of most agents is 4-6 hours, except meclozine, chlorpheniramine, mesolastine, loratadine, cetirizine and fexofenadine which act for 12- 24 hours or more. The 1-1 1 antihistaminics are mainly administered orally, but few are avai lable for i.m./i. v. use as well.

SIDE EFFECTS AND TOXICITY Side e ffects of first generation H1 antihistaminics are frequent, but genera ll y mi ld. Individuals show marked differences in susceptibility to side effects wit h diffe re nt drugs. Some to lerance to side effects develops on repeated use. Sedation, di min ished alertness and concentration, light headedness, motor incoordination, fatigue and tendency to fa ll asleep are the most common. Objective testing shows impa irment of psychomotor performance. Patients should be cautioned not to operate motor vehicles or machinery requi ri ng constant attention. Alcohol synergises in producing these effects as do otber C S depressants. Few indi viduals, howeve r, become restless. nervous and are unable to sleep. Second generation compounds are largely free of CNS effects.

HISTAMINE AND ANTIH ISTAMIN ICS Regular use of conventional antihi stamines is not ad visable in children, because the C S depressant property may interfere w ith learning and academic tasks. Dryness of mo uth , a lteration of bowel movement, urinary hesitancy and blurring of vision can be ascribed to anticholinergic property of o lder a ntihista minics. Epigastric distress and headache may be felt. Loca l application can cause contact dermatitis. Some drugs Iike hydroxyz ine, cycliz ine a nd fexofe nadinc a re teratogenic in anima ls; but there is no evidence of excess ma lformations in humans. Caution is nevertheless to be exerc ised in prescribing an antihistaminic during pregnancy. Acute overdose produces central excitation, tremors, hallucinations, muscular incordination, convul sions, flushin g, hypotension, fever and some other features o f belladon na poisoning. Death is due to respiratory and cardiovascular fa ilure.

SECOND GENERATION ANTIHISTAMINICS The second generation antihistaminics (SGAs) may be defined as those H 1 receptor blockers marketed after 1980 w hic h have one or more of the following properties: • Absence of C S depressant property. • Higher 11 1 selectiv itiy : no anticho linergic side effects. • Additional antiall ergic mechanisms apart from histamine blockade: some also inhibit late phase a llergic reaction by modifying re lease/ action of leukotrienes, platelet activating facto r ( PA F) and cytokines, etc. As per an internationa l consensus group of experts, no compound developed so far merits labelling as ·third gem:ration antihistaminic~.

These newer drugs have the advantage of not impairing psychomotor performance (dri ving, etc. need not be contraindicated). produce no subjective effects, no s leepiness. do not potentiate alcohol o r benzodiazcpines. Some patients do complain of sedation, but incidence

is similar to that with placebo. However, they have a narrow s pectrum of therapeutic usefu lness wh ich is limited by the extent of invo lvement of hista m ine (acting through H 1 receptors) in the disease sta te. Thei r principal indicatio ns a re: ( i) Allergic rhinitis and conjuncti vitis, hay fever, pollinosis: they control sneezing, runny but not blocked nose, and red-watering-itchy eyes. ( ii) U1ticaria, dennographism, atopic eczema. (iii) Acute allergic reactions to drugs and foods. The SGAs have poor antipruritic, antiemetic and antitussive actio ns.

Fexofenadine It is the active metabo lite of terfenadine, the fi rst nonsedating SGA that was withdrawn because of seve ra l deaths due to polymorphic ventri c ul ar tachyca rdi a (Torsades de pointes) occurring with its higher doses or whe n it was coadministered w ith CYP3A4 inhibitors (crythromycin, clarithromycin, ketoconazole, itraconazole, e tc.). This toxicity is based on blockade of delayed rectifier K c ha nnels in the heart at higher concentrations. Astemizole is anot her SGA banned for the same reason. Fexofenadine has a low propensity to b lock delayed rec tifier K cha nne ls, does not prolong QTc interva l. Since it is mini mally meta bolized. no interaction with CY P3A4 inhibitors have been reported. It is largely free of arrhythmoge ni c potential, but some cases of ventricular arrhythmia in patients w ith preexisting long QT interval have been reported. Thus, it is not entirely safe in patie nts w ith long QT, bradycardia o r hypokalemia. Fexofenadine penetrates blood-brain barrier poorl y; produces minimal sedation o r impairment of psychomotor performance. It is free of arropinic side effects. Oral absorption is rapid. It is mainly excreted unchanged in urine and bile, has plasma t½ 11 - 16 ho urs and durati on of action 24 hours. Dose: For allergic rhinitis 120 mg OD; for urticaria and other skin allergies t 80 mg OD.

Loratadine

Anothe r lo ng-acting selec ti ve peripheral H 1 antago ni st w hi ch lacks CNS depressan t effects and is fast acting. It is partly metabolized by CYP3A4 to an active

181

182

AU TACOIDS AN D RELATED DRUGS metabolite with a longer t½ of 17 hr. but has not produced cardiac arrhythmia in overdose, though seiz ures a re reported. No interactio n with macrolides or antifungals has been noted. Good efficacy has been reported in urticaria and atopic dermatitis.

quick symptomatic relief lasting 12 hr. Stinging in the nose and altered taste perception are the local side effects. Some somnole nce has been reported on nasal application and a tendency to weight gain noted after oral use.

Desloratadine

is effective in allergic rhinitis and urticaria by single daily dosing des pite a t½ of 8- 10 hr and no active metabol ite.

It is the major active metabolite of loratadine elTective al half the dose. oninterfercnce wi th psychomotor perfo rmance and cardiac safety are documented.

Cetirizine

It is a metabolite of hydroxyzine with marked affinity for peripheral H I receptors; penetrates brain poorly, but mild sedation and subjective somnolence is experienced by many recipients. It is not metabolized; does not prolong card iac action potential or produce arrhy thmias when given with erythromycin/ ketoconazo le. Cetirizine in addition inhibits release of histamine and of cytotoxic medi ators from platelets as well as eosinophil chemotaxis during the secondary phase of the allergic response. Thus, it may benefit allergic disorders by other actions as wel l. It attains high and longer lasting concentration in skin, which may be responsible for superior efficacy in urticaria/atopic dermatitis. as well as for once daily dosi ng despite elimination t½ of 7-10 hr. It is indicated in upper respiratory allergies, pollinosis, urticaria and atopic dern, atitis; also used as adjuvant in seasonal asthma.

Levocetirizine is the active R(-) enantiomer of cetirizine. It is effective at ha lf the dose and appears to produce less sedation and other side effects. Azelastine

This newer H 1 blocker has good topical activity: in addition it inhibits histamine release and inflammatory reaction triggered by LTs and PAF. After intranasal application it has been shown to down regulate intracellular adhesion molecule- I (ICAM- 1) expression on nasal mucosa. Its t½ is 24 hr, but action lasts longer due to active metabolite. Its metabolism is inhibited by CYP 3A4 inhibitors. Given by nasal spray for seasonal and perennial allergic rhinitis it provides

Mizolastine This nonsedating antihistaminic

Ebastine Anothe r newer SGA that rapidly gets converted to the act ive metabolite carbastine having at½ of 10-16 hr. It is nonsedat ing and active in nasal and skin a llergies. Animal studies have fo und it to prolong Q-Tc interval which makes it liable to arrhythmogenic potential and CYP3A4 interaction, but actual reports are still few. Rupatadine This recently introd uced antihistaminic has additional PAF antagonistic property, and is indicated in a llergic rhini tis. USES The uses of H 1 antihistaminics are based on their ability to block certain effects of histami ne re leased endogeneously. as well as on thei r sedative and anticholinergic properties. 1 . Allergic disorders Antihistami nics do not suppress AG : AB reaction, but block the effects of released histamine- are only palliative. They effectively control certain immediate type of allergies, such as itching, urticaria, seasona l hay fever, a llergic conj unctivitis and angioedema of lips, eyelids, etc. However, their action is slow, therefore Adr alone is life-saving in laryngeal angioedcma, while intravenously administered antihistaminic may have adjuvant value. Similarly, they cannot be relied upon in anaphylactic shock and have a secondary place to Adr. Benefits are less marked in perennial vasomotor rhinitis, atopic dermat itis and chronic urticarias; combination with an H 2 antagonist succeeds in some cases of chronic urticaria not responding to H I antagonist alone.

HISTAMINE AND ANTIHISTAMINICS The antihistaminics are ineffective in bronchial asthma: reasons may be• Leukotrienes (LTC4, D4) and PAF are more important mediators in humans than histamine. • Concentration of antihistamines attained at the site may not be sufficient to block high concentration of histamine released locally in the bronchi. Certain newer compounds like cetirizine have adjuvant role in seasonal asthma. Antihistaminics are also ineffective in other types of humoral and cell mediated allergies because histamine is not involved. They do suppress urticaria and swellings in serum sickness, but have no effect on other components of the syndrome. Type I hypersensitivity reactions to drugs (except asthma and anaphylaxis) are suppressed. Some skin rashes also respond. 2. Other conditions involving histamine Anti histaminics block symptoms produced by histamine releasers (see p. 178); afford symptomatic relief in insect bite and ivy poisoning. Abnormal dermographism is suppressed. They also have prophylactic value in blood/saline infusion induced ri gor.

3. Pruritides

Many conventional antihistamines have antipruritic action independent of H 1 antagonism. Though relief is often incomplete, older antihistaminics chlorpheniramine, diphenhydramine, cyproheptadine remain the first choice drugs for idiopathic pruritus.

4 . Common cold

Antihistaminics do not affect the course of the illness but may afford symptomatic relief by anticholinergic (reduce rhinorrhoea) and sedative actions. The newer nonsedating antihistamines are less effective in this respect.

5 . Motion sickness Promethazine, diphenhydramine, dimenhydrinate and meclozine have prophylacti c value in milder types of motion sickness; should be taken one hour before starting the journey. Promethazine can also be used in morning sickness, drug induced and postoperative vomiting, radiation sickness.

An 'off label' (unapproved) use of cyproheptadine is often made in anorectic/convalescent patients for improving appetite. Such use in underweight children is inappropriate, because its CNS depressant action can affect learning. 6. Vertigo Vertigo is typically felt as a spinning sensation or rotation of surroundings, or sense of movement when there is none. Mild vertigo is often reported as 'dizziness'. Vertigo is mostly of vestibular origin, but may also be due to a variety of peripheral or central causes, and is generally accompanied by nausea and vomiting. The long term therapy of vertigo occurring in Meniere's disease and other conditions is imperfect. A variety of approaches have been tried. 1. Labyrinthine suppressants These drugs suppress end-organ receptors or inhibit central cholinergic pathway (in vestibular nuclei). - An1ihis1aminics (with anticholinergic action)--cinnarizine, dimenhydrinate, diphenhydramine, promcthazine. - A111icholi11ergics- hyoscine, atropine. - Amiemelic pheno1hiazines-prochlorpcrazine. 2. Vasodilators They improve blood flow 10 labyrinth and brainstem- betahistine is quite frequently used in Meniere's disease. 3. Diuretics They decrease labyrinthine fluid pressure -acetazolamide, thiazides, amiloride. 4. Anxiolytics, antidepressants These drugs appear to modify the sensation of vertigo; may also depress endorgan receptors-diazepam, amitriptyline. 5. Corticosteroids They decrease imralabyrinthine edema due to viral infection or other causes. Labyrinthine {vestibular) suppressant medications are the principal drugs for management of vertigo as well as that of attendant nausea and vomiting. The H 1 antihistaminic cinnarizine and dimenhydrinate are the most commonly used vestibular suppressants in this condition. Betahistine (see p. 178) is specifically used in Meniere's disease.

Cinnarizine:This H 1 antihistamine has additional weak antimuscarinic, anti-5HT, sedative and vasodilator properties. It modulates Ca2• fluxes and attenuates vasoconstrictor action of many endogenous mediators. The antivertigo property may be due to calcium channel blockade. Cinnarizine inhibits vestibular sensory nuclei in the inner ear, suppresses postrotatory labyrinthine

183

184

AUTACOIDS AND RELATED DRUGS

reflexes, possibly by reducing stimulated influx of Ca2+ from endolymph into the vestibular sensory cells. Beneficial effects have been reported in Meniere's disease and other types of vertigo. Side effects are sedation and mild g.i. upset. Proch/orperazine: This D2 receptor blocking neurolcptic has additional H, antihistammic and labyrinthine suppressant property which is useful in the shon-tcrm control of vcn igo as well as the associated nausea and vomiting. Parenteral prochlorperazine is the most effective drug for controlling violent ven igo and , omiting. It is also used in other types of nausea and vomiting (see Ch. 4 8)

7 . Preanaesthetic medication Promethazine has been used for its anticholinergic and sedative properties.

8. Cough Antihistaminics like chlorpheniramine, diphenhydramine and promethazine are constituents of many popular cough remedies. They have no selective cough suppressant action, but may afford symptomatic relief by sedative and anticholinergic property (see Ch. I 6).

9 . Parkinsonism

Promethazi ne and some others afford mild symptomatic relief in early cases- based on anticholinergic and sedative property.

10. Acute muscle dystonia Caused by antidopaminergic-antipsychotic drugs is promptly relieved by parenteral promethazine,

diphenhydramine or hydroxyzine. This is again based on central anticholinergic action of the drugs. 11 . As sedative, hypnotic, anxiolytic Antihistamines with C S depressant action have been used as sedative and to induce sleep. especially in children. However, promethazine has produced serious respiratory depression in young children; few deaths are on record; it is not indicated in children aged 2 years or less. For promoting sleep, antihistaminics are not as dependable as benzodiazepines. Hydroxyzi ne has been used in anxiety associated with autono mic manifestations. (Combinations of antihistaminics with antidiarrhoeals or bronchodilators, or those containing more than one amihistaminic are banned in India.)

H2 antagonist The first 8i blocker Burimamide was discovered by Black in 1972. Metiamide was the next, but both were not fo und suitable for clinical use. Cimetidine was introduced in 1977 and gained wide usage. Ranitidine, famotidine, roxatidine, and many others have been added subsequently. They are primaril y used in peptic ulcer, gastroesophageal reflux and other gastric hypersecretory states. They are described in Ch. 47. H3 antagonist Though some selective H1 antagonists have been produced, they have not found any clinical utility.

c:r PROBLEM DIRECTED STUDY

A taxi driver aged 30 years presented with sudden onset running and itchy nose, bouts of sneezing, partial nasal blockage, redness and watering from the eyes, but no fever, bodyache or malaise. He gave history of similar episodes occu rring off and on during the spring season. A diagnosis of seasonal allergic rhinitis was made and the doctor prescribed an oral antiallergic medication to be taken once a day till symptoms subside. (a) Which antiallergic medicine would be suitable for this patient? Which antiallergic drugs should be avoided? {b) Will the above medication prevent/ reduce recurrent episodes of rhinitis that this patient gets du ring spring. If not, can some medications be added to prevent/su bdue the episodes during the vulne rable season. {see Appendix-1 for solution). 11.1

Chapter

12

5-Hyd roxytrypta mine, its Antagonists and Drug Therapy of Migraine

5-HYDROXYTRYPTAMINE (5-HT, Serotonin) Serotonin was the name given to the vasoconstrictor substance which appeared in the serum when blood c lotted and Enreramine to the smooth muscle contracting substance present in enterochromaffi n cells of gut mucosa. In the earl y I950s both were shown to be 5-hydroxyttyptamine (5- HT). About 90% of body's content of 5-1-IT is localized in the enterochromaffin cells of stomach and intestines; most of the rest is in pla telets and brain. It is also fou nd in wasp and scorpion sting, and is widely distributed in invertebrates and plants (banana. pear, pineapple, tomato, stingi ng nettle, cowhage).

SYNTHESIS, STORAGE AND DESTRUCTION 5-HT is ~-aminoethyl-5-hydroxyindole. It is synthes ized from the a mino acid tryptophan and degrad ed primarily by MAO and to a small extent by a dehydrogenase (Fig. 12.1 ). There is close paralle lism between CAs and 5-HT. The decarboxylase is non-specific, acts on DOPA as we ll as 5-hydroxyt,yptophan (5HTP) to produce DA and 5-HT respecti vely. Like A, 5-HT is actively taken up by an amine pump serotonin transporter (SERT), a a+ dependent carrier, which operates at the membrane of pla te lets ( therefore, 5- HT does not circulate in free fo rm in p lasma) and serotonerg ic nerve endings. This pump is inhibited by selective serotonin reuptake inhibitors

TRYPTOPHAN

l l

Hydroxylase

~HYDROXYTRYPTOPHAN

Decarboxytase

5-HT MAO

HO~

CH 2CH 2 NH2



5-HYDROXY INDOLE ACETIC ACID (5-HIAA)

i-+

.,.H 'DROX TRYPTOPHOL

N H

Dehvdrooenase

Fig. 12.1: Synthesis and degradation of 5-hydroxytryptamine (5-HT)

(SS Rl s) and tricyclic antidepressants (TCAs). Plate lets do not syn thesize 5-HT but acqui re it by uptake during passage through intestinal blood vessels. Again like CAs, 5-HT is stored within storage vesicles, and its uptake at the vesicular membrane by vesicular monoamine transporter ( VMAT-2) is inhibited by reserpine, which causes depletion of CAs as well as 5-HT. The degrading enzyme MAO is also common for both. The isoenzyme MAO-A preferentially metabolizes 5-HT.

SEROTONERGIC (5-HT) RECEPTORS Gaddum and Picarelli ( 1957) c lassified 5-HT receptors into musc ulot ropic ( D ty pe) a nd ne urotropic (M type) on the basis o f their blockade by Dibenzyline (phe noxybenza mine) and Morph ine. The c lassical 5-HT antagoni sts methysergide and cyproheptadine blocked D

186

AUTACOIDS AND RELATED DRUGS type receptors. Subsequently 5-HT receptors were differe nti ated by their hi gh or low affinity for [3H] 5-HT in radi oligand binding studies. The present system or classifyi ng 5-HT rece ptors is based on mo lecular characterization and cloning of the recepto r c DNAs. Some subtypes of 5-HT receptors have specific tissue distribution , but certain tissues ex press mo re than one subtype. Four families o f 5-HT receptors (5-HT 1, 5-HT2 , 5-HT3 , 5-HT4 _7 ) comprisi ng of 14 receptor subtypes have so far been recognized. However, only some of these have been runctionally correlated. Selective agonists/antagonists ha ve been defined only fo r these subtypes. Know ledge of s ubtypes of 5-HT rece ptors has assumed importance because some newly developed therapeuticall y useful drugs can only be described in terms of 5-HT receptor subtype selective agoo ists or antagonists. A ll 5- HT receptors (except 5-HT3) are G protein coupled receptors which functio n through decreasing (5-HT 1) or increasing (5-HT 4, 5-HT6, 5-HT7) cAMP production, or by generating IP/ DAG (5-HT 2) as second messengers. The 5-HT3 is a ligand gated cation (Na-,K+) channel which on acti vation elicits fast depolarization. 5- HT1 Receptors

Six subtypes (5-1 IT,A. •· o. L , . ,) have been identified. The 5-HT,c receptor is now designated SHT,c. All subtypes o f 5- HT 1 receptor couple with Gi/Go protein and inhibit adenylyl cyclase; 5-HT,Ain addition activates K • channels (re ulting in hyperpolarization) and inhibits Ca2channels. These receptors function primarily as autoreceptors in brain- inhibit fi ring of 5-HT neurones or release of 5-HT from nerve endings. The most important location of 5- HT ,A receptor is so mato-dendritic synapses in raphe nuclei of brainste m; their activation serves to reduce firing of raphe neurones. I ltppocampus is another important site. The an tianxiety d rug buspirone acts as a partial agonist of 5- HT 1A receptor. The 5-HT I0 receptor has been s hown to regu la te dopaminergic to ne in subs tantia nigra- basal ganglia, and 5-IITrn 18 (the same receptor is 5-HT, 8 in rat) to cause constrict ion of cranial blood vessels. The antimigra ine drug sumarripran is a selective 5- HT 11,,rn agonist. Other functions subservcd by 5- HTm receptors are inh ibiti on of 5-HT re lease from forebrain serotonergic neurones, A release from ~ympathetic nerve endings and that of inflammatory ncuropeptides from nerve endings in cranial blood vessels.

The peripheral 5-IIT,. receptors expressed on neurones of enteric nervous system mediate t he ir s low depolarization.

5- HT2 Receptors There are 3 subtypes o f 5-HT2 receptor; all are coupled to Gq protein• activate phospholipase C and function throug h generation of IP/ DAG. 5-HT2A receptor also inhibits K ' channels resulting is slow depolarization of neurones. a-methyl 5-HT is a selective agonist for all 3 subtypes. 5- HT 2A is the most widely expressed postjunctional 5- IIT receptor (designated earlier as D type) located on vascular and visceral smooth muscle, platelets and cerebral neurones especially prefrontal conex. h mediates most of the direct actions of 5-1IT like vasoconstriction, intestinal, uterine and bronchial muscle contraction, platelet aggregation and activation of cerebral neurones. Keranserin is a 5-HT, antagonist more selective for 5-HT"". Contraction of rat gast ric fundu s is mediated by 5-I-IT28 receptor. 5-HT,c receptor is located on vascular endothelium-elicits vasod,latation through EDRF release. Choroid plexus expresses large number of 5-HTK receptors "hich may regulate CSF formation.

5-HT 3 Receptor This is the neuronal 5-HT receptor which rapidly depolarizes nerve endings by opening the cation channel located w ithin it and corresponds to the origi nal M type receptor. It mediates the indirect and reflex elTects of 5-HT at: (i) Somatic and autonomic nerve endings mediating pain. itch, coronary chemoreflex (bradycardia, fall in BP due to withdrawal of sympathetic tone, respiratory stimulation or apnoea elicited by stimulation of receptors in the coronary bed), and other viscera I reflexes. (ii) 1erve endings in myenteric plexus eliciting augmentation of peristalsis, emetic reflex. (iii) Area postrema and nucleus tractus solitarious in brainstem inducing nausea. vomiting. Ondansetron is a selective 5-HT3 antagonist which inhibits vomiting by blocking these receptors in th e bra instem as well as in gut wall. 2-Methy l 5- HT is a selective 5-HT3 agonist. 5- HT4 _ 7 Receptors The 5-H T, receptor couples to Gs protein, activates adenylyl cyclase and has been demonstrated in the mucosa, plexuses and smooth muscle o f the gut where it is probably involved in augmenting intestinal secretion and peristalsis. It is also located in brain, especially hippocampus and the colliculi where it causes slow depolarization by decreasing K" conductance. Cisapride and renzapride are selective 5-HT, agonists. The recently cloned 5-HT 5, 5- HT 6 and 5-IIT 7 receptors are closely related to the 5-IIT, receptor. These are main l)' located in speci fic brai n areas, but t heir functional role is not known. An interesting finding is that clo=api11e (atypical antipsychotic) has high a ffinity for 5-HT6 and 5-HT, receptors in addition to being a 5-HT,Aoc antagonist.

5-HT, ITS A NTAGONI STS AN D DRUG THERAPY OF M IGRAINE

Salient features of important 5-HT receptor subtypes

5-HT,

5-HT2A

5-HT,

Autoreceptors; inhibit serotonergic neural activity in brain. 5-HTu-present in raphe nuclei and hippocampus; buspirone (antianxiety) may act through these receptors. 5-HT,a,, 8-Constricts cranial blood vessels and inhibits release of inflammatory neuropeptides in them; sumatriptan (antimigraine) acts through these receptors. Previously D type receptor; most important postjunctional receptor mediating direct actions of 5-HT like vascular and visceral smooth muscle contraction, platelet aggregation, neuronal activation in brain; ketanserin blocks these receptors. Previously Mtype receptor; depolarizes neurones by gating cation channels; elicits reflex effects of 5-HT-emesis, gut peristalsis, bradycardia, transient hypotension, apnoea, pain, itch; ondansetron (antiemetic) acts by blocking these receptors. Mediate intestinal secretion, augmentation of peristalsis. Renzapride (prokinetic) is a selective 5-HT. agonist.

ACTIONS

5-HT receptor function

5-HT is a potent depolarizer of nerve endings. It thus exerts di rect as well as refl ex and indirect effects. Tachyphylaxis is common with repeated doses o f 5-I-IT. The overall effects therefore are often variable. 1. CVS Arteries are constri cted (by direct action on vascula r smooth muscle) as well as di lated {th rough EDRF release) by 5-HT. dependi ng on the vascular bed and the basal tone. In addition, 5-HT releases Adr from adrenal medulla, affects gang lionic transmission and evokes cardiova ·cular refl exes. The net effect is complex. Larger arteries and veins are characteristically constricted. In the microcirculation 5-HT di lates arterioles and constricts venu les: capillary pressure rises and fl uid escapes. The direct action to increase capillary permeability is feeble. Isolated heart is stimu lated by 5-HT: both directly and by release of A from nerve endings. In intact animals, bradycard ia is mostly seen due to activation of coronary chemoreflex (Bezold Jarisch reflex) through action on vagal afferent nerve endings in the coronary bed. evoking bradycardia, hypotension and apnoea. BP : a tri phasic response is classicall y seen on i.v. injection of 5-HT in ani mals. • Ea rly sharp fall in BP- due to coronary chemoreflex. • Brie f rise in BP- due to vasoconstriction and increased cardiac output.

5-HT2A 5-HT3 5-HT, 5-HT,P

in the gut

intestinal smooth muscle-contraction. fast depolarizatio n of enteric plexus neurones; release of 5-HT from enterochromaffin cells. lower esophageal sphincter-contraction; enteric plexus-ACh release-enhanced peristalsis; intestinal mucosa-secretion. s low depolarization of enteric plexus neurones.

• Pro lo nged fa ll in BP- du e to a rte rio lar dilatation and cxtravasation of flu id. llowever, 5-HT is not involved in the physiological regulation of BP.

2. Visceral smooth muscles 5-HT is a potent sti mula to r of g.i.t., both by direct action as well as through enteric plex uses. Severa l subtypes of 5-HT receptors are present in the gut (See box). Peristalsis is increased and diarrhoea can occur {also due to increased secretion). 5-HT constricts bronchi, but is less potent than histamine. Action on other smooth muscles in man are feeble and incons istent. 3. Glands 5-HT inhibits gastric secretio n (both ac id and peps in), but increases mucus production. It thus has ulcer protective property. Effect o n other glandular secretions is not significant. 4. Nerve endings and adrenal medulla A ffercnt nerve endin gs are activated by 5-HT

187

188

AUTACOIDS AND RELATED DRU GS causing tin gling and pricking sensation, as well as pa in. Depolarization of v isceral afferents e licits respiratory and cardiovascular reflexes, nausea and vomiting. S-HT is less potent than histamine in releasing CAs from adrenal medul la. 5. Respiration

A brief stimulation of respiration (mostly reflex from bronchial afforcnts) and hyperventilation arc the u ual response, but large doses injected i.v. can cause transient apnoea through coronary chemoretlex.

6. Platelets By acting on S-HT2A receptors 5-HT causes changes in shape of platelets, but is a weak aggregator. lfowever, it does not induce the release reaction. 7. CNS

Injected i.v., 5-HT does not produce central effects because of poor entry across blood-brain barri er. However, it serves as a transm itter, primarily inhibitory. Direct injection in the brain produces sleepiness, changes in body temperature, hunger and a variety of behavioural effects.

PATHOPHYSIOLOGICAL ROLES I. Neurotransmitter 5-HT is an estab lished neurotransmitter in the brain. The brain 5-HT has a fast turnover rate. Cells containing 5-HT are present in the ra phe nuclei of brainstem, substantia nigra and few other s ites. They send axons rostrally (LO li mbic system, cortex and neostriatum) as we ll as caudally to spinal cord. 5- HT appears to be invol ved in s leep, temperature regulation, thought, cogniti ve function, behav iour and mood, appetite, vomi ting and pain perception. Some serotonergic neurones are present in intestines also. Experimental evidence from pharmacological manipulation of5-HT metabolism, genetic models (li ke knock out mice), as well as therapeutic effect of selective serotonin reuptake inhibitors (SSRls) and TCAs, etc. strongly suggest a role of S-HT in the pathogenesis of anx iety, depression, aggression and other behavioural disorders in humans. 2 . Precursor of melatonin 5- HT is the precursor of me latonin in pineal gland. It is believed to regul ate the biological clock and maintain circadian rhythm.

3 . Neuroendocrine function The hypothalamic neurones that control release of anteri or pituitary hormones are probably regulated by serotonergic mechanism. 4 . Nausea and vomiting Especially that evoked by cytotox ic drugs or radiotherapy is mediated by re lease of S-HT and its action on 5-HT3 receptors in the gut, area postrema and nuc leus tractus solita rious. 5 . Migraine 5-HT is said to initiate the vasoconstrictor phase of migraine and to participate in neurogenic inflammation o f the alTected blood vessels. Methysergide (5-HT antagonist) is a n efTective prophylactic and sumatriptan (S-HTIDnB agonist) can control an attack. However, the role of 5-HT in thi s condition is not precisely known. 6 . Haemostasis Platelets re lease 5-HT during aggregation at the site of injury to blood vessel. Acting in concert with col lagen and other mediators, th is 5-HT accelerates platelet aggregation and c lot formation. Thus, it serves to amplify the response. lls contractile action appears to promote retraction of the injured vessel. Both the above actions contribute to haemostasis. 7 . Raynaud's phenomenon Release of5-HT from platelets may tri gger acute vasospastic episodes of larger arteries involved in Raynaud's phenomena. Ketanserin has prophylactic value. 8 . Variant angina Along with thromboxane~, 5-HT re leased from platelets has been implicated in causing coronary spasm and variant angina. However, the inefficacy of anti S-HT drugs in this condition points to the involvement of other med iators. 9 . Hypertension Increased responsiveness to 5-HT, as well as its reduced uptake and c learance by platelets has been demonstrated in hypertensive patients. Ketanserin has antihypertensive property. S-HT has been held responsible fo r preeclamptic rise in BP. 10. Intestinal motility Enterochromaffin cells and 5-HT containing neurones may regulate peristals is and local reflexes in the gut. This

5-HT, ITS ANTAGONISTS AND DRUG THERAPY OF M IGRAINE syste m a ppears to be acti vated by in test inal distension and vagal effe rent acti vity. 11 . Carcino id syndrome T he ca rc inoid tumours produce massive quantities of 5-HT. Bowe l hy permot ility and bronchoco nstrict ion are featu res of carcinoid whic h appear to be due to 5-HT, but flushing and hypotens ion are probabl y caused by other med iators. Pellagra in ca rc inoid may occur d ue to diversi on of tryptophan for synthesizing 5-HT.

Use Due to widespread and variable actions, 5-HT has no therapeutic use.

(ii) Azapirones like buspirone, gcpironc and ipsapirone are a novel class of ant ianxiety drugs which do not produce sedation. They act as partial agonists of 5-1-IT,A receptors in the brain. (iii) Sumatriptan a nd other triptans arc selective 5-HT,D'IB agonists. T hey constrict cerebral blood vessels and have emerged as the most effective treatment of acute migraine attacks. (iv) Cisapride This prokinetic drug, which increases gastrointestinal motility, is a selective 5-HT, agonist. Renzapride is still more selective for 5-l·IT, receptors.

8 . 5-HT receptor antagonists A variety of drugs block serotonergic receptors; many are nonse lective, but some newer ones are highly subtype selective.

5-HT ANTAGONISTS DRUGS AFFECTING 5-HT SYSTEM 1. 5-HT precursor Tryptophan increases brain 5-HT and produces behavioural effects because tryptophan hydroxylase in brain is not saturated by the amount of tryptophan available physiologically. 2. Synthesis inhibitor p-Chloropheny lalanine ( PC PA) selectively inhibits tryptophan hydroxylase (rate limiting step) aad reduces 5-HT level in tissues. It is not used clinically due to high toxicity. 3. Uptake inhibitor Tricyclic antidepressants inh ibit 5- HT uptake along with that of NA. The selective serotonin reuptake inhib itors (SSRl) like ft uoxetine, sertraline, etc. inhibit only 5-HT reuptake and have antidepressamantianxiety property. 4. Storage inhibitor Reserpine blocks 5- HT (as well as NA) uptake into storage vesicles by inhibiting VMAT-2, and causes depletion of a ll monoamines. Fentluramine selectively re leases 5-HT by promoting its reverse transport at serotonergic nerve endings in the brain, followed by its prolonged depletion. It has anorectic property. 5. Degradation inhibitor Nonselective MAO inhibitor (tranylcypromine) and selective MAO-A inhibitor (clorgyline) increase 5-HT content by preventing its degradation. 6. Neuronal degeneration 5, 6 dihydroxytryptami ne selectively destroys 5-HT neurones. 7 . 5-HT receptor agonists A diverse range of compmmds producing a variety of actions have been found to activate one or more subtypes of 5-HT receptors. Notable among these are: (i) O-Lysergic acid diethyl amide (LSO)--Synthesized as an ergot derivative, LSD was found to be an extremely potent hallucinogen. It is a nonselectivc 5-HT agonistactivates many subtypes of 5-HT receptors including 5-HTIA on raphe cell bodies, 5-HT:w,c (probably responsible for the ha llucinogenic effect) and 5-HT5_7 in specific brain areas. However, LSD inh ib its 5-HT,A receptors in the ileum. A number of other hallucinogens also interact with brain 5-HT receptors.

The ab ility to antagonize at least some actions of 5-HT is fou nd in many classes of drngs, e .g. e rgot derivatives (ergotam ine , LSD, 2-b romo LS D, methyserg ide), adrenergic a blockers (phenoxybenzamine), antihistaminics (cyproheptadine, cinnari zine ), ch lorpromazine, morphine, etc., but these are nonse lecti ve and interact with receptors for other signal molecules as well. Many are partial agonists or antago nize ce rtai n actions of 5- HT but mimic others. The salient features of drugs which have been used clinically as 5-HT antagonists and some new e r selecti v e antagoni sts are described below:

1. Cyproheptadine It primari ly blocks 5-HT 2A receptors and has additional H 1 antihistaminic, anticholinergic and sedati ve properties (see C h. 11) . Like othe r a nti histaminics, it has been used in al lergies a nd is a good antiprnritic, but the anti 5-HT act ion has no ro le in these conditions. 11 inc reases appetite and has been used in childre n as wel l as in poor eaters to promote weight gain . The H, antihistaminic action and an acti on on growth horm on e s ecretion has been suggested to account fo r this property. The anti 5-HT acti vity of cyproheptadine has been utilized in controlling intestinal manifestations of carcinoid and postgastrectomy dumping syndromes, as well as in antago nizing priapism/ orgasmi c delay caused by 5-HT uptake inhibitors Iike fluoxetine and trazodone.

189

190

AUTACO IDS AND RELATED DRUGS

Side effects drowsiness, dry mouth, confusion, ataxia, weight gain. 2 . Meth yserg ide 11 is chemically related 10 ergo1 alkaloids; an1agoni1cs ac1ion or 5-1-IT on vascular and visceral srnoo1h muscles, withoul producing 01hcr ergot like effec1s. II docs nol in1erac1 wilh o. adrcncrgic or dopamine receptors. Mc1hyscrgidc is a potent 5-1-IT,,_->< antagonist with some tissue specific agonistic actions as well. Introduced briefly for migraine prophylaxis, carcinoid and postgastrectomy dumping syndrome, it went into disuse because it caused abdominal, pulmonary and endocardial fibrosis. 3. Ketanserln It has selective 5-1IT, receptor blocking property with negligible action on 5-HT;, 5-HT, and 5 HT, receptors and no partial agonjstic activity. 5-1IT induced vasoconstriction. platelet aggregation and contraction of airway smooth muscle are antagoni zed. Weak 11 1 and dopaminergic block ing activities are also present. Ketanserin has sign ificant a , adrenergic blocking activity and was introduced as antihypertensive, hut did not gain popularity.

Ritanserin is a relatively more 5-1 ITz., selecti, e congener of ketanserin.

4. Clozapine In addition to being a weak dopaminerg ic antagonist. this atypical antipsychotic is a 5- HT2AJ2c blocker (see Ch. 32). Clozapine may a ls o exert in ve rse agonist acti vity a t cerebral 5-HT 2Al2c receptors which may account for its efficacy in resistant cases of schizophrenia. 5. Risperidone This atypical antipsychotic is a combined 5-HT2A + dopamine 02 antagonist, s imilar to clozapine. Like the latter, it especially ameliorates negative symptoms of schizophrenia, but produces extrapyramidal side effects. Other atypical antipsychotics like olanzapine and quetiapine are also combined 5- HT and DA antagonists, but interact with other neurotransmitter receptors as well. 6. Ondansetron It is the prototypical selective 5-HT3 antagonist that has s hown remarkable efficacy in controlling na usea and vomiti ng fo llowing administration of hi ghly e metic anticancer drugs and radiotherapy. The 5-HT3 antagonists are described in Ch. 48. ERGOT ALKALOIDS Ergot is a fung us Claviceps purpurea wh ic h grows on rye, mi llet and some other grains.

The grain is replaced by a purple. hard, curved body called 'sclerotium ·. Epidemics o f ergot poisoning (ergotism), due to consumption of contaminated grains, have been recorded from the beginning of history. IL still occurs in epidemic and sporadic forms . Painful dry gangrene of hands and feet which become b lack (as if burnt) occurs due to vasospastic ischemia. Miscarriages occur in women and cattle. Some suffer hallucinations and a convu lsive type is also described. Ergot had been used by midwives to quicken labour since the middle ages. This use recei ved medical sanction in the 19th century, but its dangers were recognized by the beginning of the 20th century and then it was advocated only after de livery. Dale and Barger ( 1906 onwards) isolated the ergot alkalo ids and studied their pharmacology. Ergometrine was isolated in 1935. Ergot contains a host of pharmacologically active substances- alkaloids, LSD, histamine, ACh, tyramine and other ami nes, sterols, etc. Na tural ergot alkaloids These are tetracyclic indo le contai ning compounds whi c h may be considered as deri vatives of lysergic acid. They are div ided into(a) Amine alkaloid Ergometrine (Ergonovine): which is oxytocic. (b) Amino acid alkaloids Ergotamine, Ergotoxine (mixture of ergocris tine + ergocornine + e rgocryptine): they are vasoconstrictor and a adrencrgic blocker/ partial agonist. Semisynthetic derivatives (a) Dihydroergotamine (DI IE), Dihydroergotoxine (Codergocrinc) : are antiadrenergic, cerebroactivc. (b) 2-Bromo-a -ergocryptine ( Bromocriptine): is a dopaminergic D2 agoni st (see Ch. 17). (c) Methysergide: it is mainly anti 5-HT. The ergot alkaloid related compounds have dive rse pharmacological properties. They act as agonists, partia l agonists a nd antagonists on certai n subtypes of a adrenergic, serotonergic and dopaminerg ic receptors in a tissue specific manner.

5-HT, ITS ANTAGONISTS AND DRUG THERAPY OF MIGRAINE Actions

Ergotamine It acts as a partial agon ist and a ntagoni st at a adrene rgic a nd a ll subtypes of 5-HT 1 and 5- HT2 receptors, but does not interact wi th 5-HT 3 or dopamine receptors. Sustained vasoconst ri ction (because of very slow dissociation from the receptors), visceral smooth muscle contraction, vasomotor centre depression are produced, and the action of A and 5-HT on smooth muscles is antagonized. However, the overall effect of oral/rectal doses o r ergotami ne on BP is ins ignificant. It is a potent emetic (through CTZ and vomiting centre) and moderately potent oxytocic. At high doses C S stimulation a nd paresthesias may be experi e nced. On c hronic exposure (ergot poisoning) vasoconstriction is accompan ied by damage to capillary e ndothe lium resulting in thrombosis, vascu lar stasis and gangrene. Dihydroergotamine {DHE) Hydrogenation of ergotamine reduces serotonc rgic a nd a-adrenerg ic agonistic actions, but enhances a-receptor blocking property. Consequently DHE is a less potent vasoconstrictor; primarily constricts capacitance vesse ls and causes less intimal damage. It is a weaker emetic and oxytoc ic, but has some antidopaminergic action as well. Dihydroergotoxine (Codergocrine) This hydrogenated mixture of crgotox ine group of a lka lo ids is a more potent a blocker and a very weak vasoconstrictor. In the brain, a variety o f partial agonistic/ antagonisti c actions on 5-HT receptors, as we ll as metabolic and vascu la r effects are produced. In addition, ACh release in cerebral cortex may be enhanced. Dihydroergotoxin e has been ad vocated fo r treatment of dementia (see Ch. 35). Bromocriptine The 2- bromo deri vative of ergocryptine is a re latively selective dopa mine D2 agonist with prominent action on pitu ita ry lactotropes (inhibits prolactin release), in striatum (antiparkinsonian) and in CTZ (emeti c). The emetic action is weaker than that of ergotamine. In certai n bra in areas weak antidopam incrgic

action has also been shown. IL has very weak anti 5-HT or a blocking actions and is not an oxytocic.

Ergometrine (Ergonovine) This am ine ergot alkaloid has very weak agonistic and practically no antagoni sti c action on a adre nergic receptors: vasoconstriction is not significant. Partial agon is tic action on 5-HT 2 receptors is most probabl y responsible fo r its powerful uterotonic effect. It is a moderately potent 5-HT2 antagonist in g. i. smooth muscle and a wea k dopaminergic agonist on the pituitary lactotropes as well as CTZ; e metic potential is low. The most prominent acti on or ergometrine is contraction of myometrium, because of w hich it is used excl usively in obstetrics (see C h. 23). Pharmacokinetics Oral b ioavaila bility of amino acid ergot a lkaloids and their hydrogenated derivatives is poor (< 1%) due to slow and incom plete absorptio n as well as high firstpass metabolism. B ioavailabi lity is higher a fter sublingual a nd rectal administration, but still ofte n erratic. They are metabolized in liver and excreted primarily in bile. Ergotamine is sequestrated in tissues-produces longer las ting actions compared to its plasma t½ o f 2 hours. Ergot alkaloids effective ly cross bloodbrai n barrier. Adverse effects ausea, vomiting, abdominal pain, muscle cramps, weakness, paresthesias, coronary and other vascular spasm, chest pain (due to coronary vasoconstriction) are the frequent side effects. These drugs are contrai ndicated in presence of sepsis, ischaemic heart disease, peripheral vascular disease, hypertension, pregnancy, li ver and kidney disease. Preparations and dose Ergotamine: For migraine 1-3 mg oral/sublingual, repeal as required (max 6 mg in a day): rarely 0 .25-0.S mg i.m. or s.c.; l:.RGOTAMl' 60 yr ( x 0.6), CHF ( x 0.6), pneumonia (x 0.4). liver fai lure (x 0.2- 0.4). Adverse effects Theophylli ne has a narrow margin of safety. Dose-dependent toxicity starts from the upper part of therapeutic concentration range ( Fig. 16. 1). Ad verse effects are primarily referable to the g.i.t., C S and CVS. Headache, nervousness and nausea are

early symptoms. Children are more liable to develop C S tox icity.

z l~#:it!Mof• 0

j::

Delirium, worsening cardiovascular sta1us Increased muscle lone, extrasystoles, flashes of light seen Agitation, tachypnoea. flushing, hypotension ResUessness. tremors. vomiting, palpitation, diuresis

c( .J

B 0 I

l)

Dyspepsia. headache, nervousness. insomnia

z

0 a::: al

IM!Jtolii'I

Death Convulsions, shock, anilythmias

,.....

Minimal side effects

Fig. 16.1 : Relationship between efficacy and toxicity of theophylline with its plasma concentration. The depicted concentration ranges are approximate

The irritant property oftheophylline is reflected in gastric pai n (w ith oral), rectal inflammation (with suppositories) and pain at site of i.m. injection. Rapid i. v. injection causes precordial pai n, syncope and even sudden death- due to marked fa ll in BP, ventricula r arrhythmias or asystole.

Interactions I. Agents which enhance theophy lline metabolism primarily by inducing CYP I A2 lower its p lasma level: dose has to be increased by the factor given in pare111hesis. Smoking ( 1.6), phcnytoin ( 1.5), rifampicin ( 1.5), phenobarbitone ( 1.2), c ha rcoal broiled meat meal ( 1.3). 2. Drugs which inhibit thcophylline metabolism and increase its plasma level arc--erythromyci n, ciprofl oxacin , cimetidine, oral contraceptives, allopurinol; dose shou ld be reduced to 2/3. 3. Theop hylline enhances the effects offurosem ide, sympathomi metics. d igitalis, oral anticoagulants, hypoglycaemics. 4. Theophylline decreases the efTects of- phenytoin, lithium. 5. Aminophylline injection should not be mixed in the same infusion bonle/syringe with- ascorbic acid. chlorproma7ine, prometha7ine, morphine. peth idine. phcnytoin. phenobarbitone, insulin, penicillinG, tetracyclines, erythromycin.

DRUGS FOR COUGH AND BRONCHIAL ASTHMA

Preparations and dose

(i) Theophylline (Anhydrous) I 00-300 mg TDS (15 mg/kg/ day) oral, THEOLO'\G 100, 200 mg SR cap., DURALYNCR 400 mg continuous release cap, UNICONTIN 400 mg, 600 mg CR tabs, TII EOBID 200 mg, 300 mg S R tabs. Only sustained release (SR) tab.leaps. whi ch produce effective blood levels for ~ 12 hours are used , because fast release tabs. produce high peak and low trough plasma concentrations. Because solubility o f theophyllioc is low, a number of soluble complexes and salts have been produced for oral and for parenteral use.

(ii) Aminophylline (Theophylline-ethylenediamine; 85% theophylline) water soluble, can be injected i. ,. but not i.m. or s.c.- highly irritating. 250-500 mg oral or slow i.v. injection; children 7.5 mg/kg i.v.; AMI NOPHYLLl'-IE 100 mg tab. 250 mg/10 ml inj . (iii) Hydroxyethyl theophyl/ine (Etophylline, 80% theophylline) water soluble; can be injected i. v. and i.m. (but not s.c.), less irritating: 250 mg oral/i.m.li.v.; D ERIPIIY LLI 100 mg tab ., 300 mg S R tab., 220 mg/2 ml inj. (iv) Theophylline ethanolate of piperazine 250-500 mg oral or i.v.; CA DIPIIYLLAT E 80 mg/5 ml elixir, ETOPHYLATE 125 mg/5 ml syrup.

Doxophyl/ine A long-acting oral melhylxanthine that is claimed not to interfere with sleep or stimulate gastric secretion.

Dose: 400 mg OD or BO, children 12 mg/kg/day; DOXOBID, DOXOVENT, DOXORIL 400 mg tab, 100 mg/5 ml syr.

severe asthma as an adjuvant drug, especially in patients with nocturnal asthma. It is more useful in COPD, and often added to othe r drugs. Use of intravenous aminophylline in status asthmaticus is outmoded. 2. Apnoea in premature infant: Theophylli ne reduces the frequency and duration of episodes of apnoea that occur in some preterm infants in the first few weeks of life. Closely monitored oral or i.v. treatment is employed for 1- 3 weeks. Caffeine is equally effective.

ANTICHOLINERGIC DRUGS (see Ch. 8) Atropinic drugs cause bronchodilatation by blocking M 3 receptor mediated choli nergic constri ctor tone. They act primarily in the larger airways (Fig. 16.2) which receive vagal innervation. However, some recent evidence points to presence of M3 receptors on peripheral bronchiolar muscles as well, though these are not vagally innervated.

adrenergic receptor e M3 muscarinic receptor



~2

The double salts/derivatives of theophylline are claimed to be less gastric irritant and better a bsorbed. However, anhydrous tbeophylline is completely absorbed and gastric irritancy of the salts is the same in terms of theophyll ine content.

Uses 1. Bronchial asthma and COPD: Theophylline may benefit by causing bronchodilatation as well as by decreasing release of inflammatory mediato rs, improvin g mucoci liary c learance, stimulating respiratory drive and by augmenting diaphragmatic contractility. However, it has several lim itations, viz, poor tolerabi Iity, narrow safety margin, lower e ffi cacy and wide individual variability, because of which its use has declined. Sustained release theophylline is occasionally employed in mild-to-moderate ly

Anticholinergics Fig. 16.2: Primary sites of bronchodilator action of inhaled adrenergic ~. agonists and inhaled anticholinergics. Salbutamol mainly relaxes bronchio lar smooth muscle; lpratropium blocks b ronchoconstriction mainly in the larger airways

247

248

RESPIRATORY SYSTEM DRUGS Jpratropium bromide is a short acting (duration 4-6 hours) inhaled anticholinergic bronchodilator, while tiotropium bromide is long acting (duration 24 hours). Administered as aerosol, they produce minimal antimuscarinic side effects due to poor systemic absorption. Anticholinergics are less efficacious than inhaled p2 sympathomimetics in bronchial asthma. However, patients of asthmatic bronchitis, COPD and psychogenic asthma respond better to anticholinergics. Reflex cholinergic tone appears to be the major reversible component of airway obstructi on in COPD. Inhaled anticholinergics a re the bronchodilators of choice in COPD. Tiotropium is rated more effective than ipratropium in COPD; more suitable for severe cases (FEVt 600 µg/day. The significant ones are-mood changes, osteoporosis, growth retardat ion in children, early cataract,

251

DRUGS FOR COUGH AND BRONCHIAL ASTHMA bru ising, petechiae, hyperglycaemia and pituitaryadrenal suppression. Several reports of adrena l crisis have appeared, especially in children , during stress (of an infection, etc). Inhaled steroids are safe during pregnancy.

Beclomethasone dipropionate BECOR IDE 50, 100, 250 ~ag per puIT inhaler. AEROC'ORT 11'.HALER 50 µg/mctered dose inhaler ,,ilh salbutamol I 00 11g. ECONASE 0.05% nasal spray.

Intranasal spray (50 µg in each nostril BD- TOS) is effective in perennial rhinitis.

Budesonide It is a nonhalogenated glucocorticoid with high topical: systemic activity ratio (greater first pass metabolism than beclomethasone). Small fraction that is absorbed is rapidly metabolized; less systemic effects are noted. It may be preferred in more severe cases. PULMICORT 100, 200. 400 µg/mctered dose inhaler, 0.25 mg/ml and 0.5 mg/ml respulcs. FORACORT: Formoterol 6 µg t Budcsonidc 100 µg/200 µg rotaeaps. RH INOCORT 50 µg per mclcred dose nasal spray; BUDENASE AO 100 µg metered dose aqueous nasal spray; 200--400 pg/day by intranasal spray for prophylaxis and lTeatment of seasonal and perennial allergic or , asomotor rhinit is. nasal polyposis; initially 2 puffs in each nostnl every morning, maintenance I puff in each nostril in the morning. Nasal irritation, s nee7ing, crusting. itching of throat and dryness may occur, especially in the beginning. ll is contraindicated m presence of infection or nasal ulcers.

F/uticasone propionate This inhaled glucocorticoid has high potency (about double of beclomethasone); longer duration and negligible oral bioavailability. The dose swallowed after inhalation has little propensity to produce systemic

efTects. At high doses, systemic effects may be due to absorption from the lungs. It may be preferred in patients requiring higher doses. FLOMALI:: INHALER 25 µg. 50 11g, 125 µg per actuation. FLOI IALE ROTACAPS 50 µg, 100 µg. 250 µg rotacaps. fLO\11 IST 50 µg per actuation nasal spray.

Flunisohde This topical steroid is available for prophylaxis and treatment or seasonal and perennial rhinitis. SYNTAR IS 25 µg per actuation nasal spray; one spray m each noslTil 2-3 time, daily.

Cic/esonide This ICS utilizes a novel approach to improve topical: systemic activ ity ratio. ll is a prodrug that is cleaved by esterases in the bronchial epithelium to release the active moiety. Though it is absorbed from lhe lungs, oral bioavailability is < I%. In the circ ulation it is extensively bound to plasma proteins, further minimi zing exposure of tissue cells to the free and active drug. C ICLEZ 80 µg and 160 µg per metered dose inhaler wath llFA propellant for once daily use, preferably in !he evening.

The dose of the ICS depends upon the pa rticular compound, its formulation, vi:::. metered dose inhaler (MDI) or dry powder inhaler (DPI), and the severity of asthma. The dai ly doses for each preparation can be categorized into low dose, medium dose or high dose, as presented in Table 16.2. ANTl-lgE ANTIBODY

Oma lizumab It is a humnni,:ed monoclonal antibody against lgE. Administered s.c., it neutralizes free lgE in circulation "ithoul acti, aring mast cells and other inflammatory cells. On antigen challenge. little lgE is available bound to the mast cell surface receptors (F.,.RI) to trigger mediator release (see Fig. 11 .2) and cause bronchoconstriction. In severe extrinsic asthma, omali,rnmab has been found to reduce exacerbations and ~teroid requirement. 'o benefit has been noted in nonallergic asthma. It is very expensive;

Table 16.2: Low. medium and high doses of inhaled cort1costero1ds for adults and adolescents(> 11 years age) Inhaled corlicosteroid

Daily doses (µg/day) Low

Medium

High

Maximum

1. Beclomethasone dipropionate-MDI

200--400

40().-800

> 800

2. Budesonide-MDI

200--400

400--800

> 800

1600 1600

100-250 200--400

250-500 40().-800

> 500 > 800

1000 2000

80-160

160-320

> 320

640

3. Fluticasone propionate-MDI -DPI

4. Ciclesonide-MDI MDI-Metered dose inhaler; DPI- Dry powder inhaler

252

RESPIRATORY SYSTEM DRU GS use is reserved for resis1.a m asthma patients with positive skin tests or raised lgE levels who require frequent hospitali,:ation. It is being tried in other allergic diseases as well.

INHALED ASTHMA MEDICATION Four classes of antiasthma drugs. viz. j3, agonists, anticholinergics, cromoglycate and glucoconicoids are available for inhalational use. They are ai med at delivering the drug to the site of action so t hat lower dose is needed and systemic side effect.s arc minimized. Faster action of bronchodilators can be ach ieved compared to oral administration. Most a thma patients are no" maintained on inhaled medication only. Aerosols are of two types: (i) use drug in solut ion: prcssuri7ed metered dose inhaler (MDI). nebuli,:ers. (ii) use drug as dry powder (DPI) spinhaler. rotahaler. Pressurized metered dose inhalers use hydrofluroalkane ( HFA) propellants and deliver a specified dose of the drug in spray fonn per actuation. Use of chlorofluorocarbon (CFC) propellant, is now banned due 10 their effect on o/onc layer. Device actuation has to be properly coordinated with deep inspiration, which many patients are unable to learn. A large volume 'spacer' (chamber interposed between the inhaler and the pat1c11l 's mouth) can be used to improve drug delivery by obviating the need for precise coordination. Moreover. larger particles settle on the walls of the spacer reducing the fraction that deposits in the throat and is later swallowed. Local complications (candidiasis with inhaled steroids) as well as systemic exposure arc reduced. Jct nebulizcrs produce a mist of the drug solution generated by pressurized air or oxygen which can be inhaled through a mouth piece, face mask or in a tent. Ultrasonic nebuli7_.ers use e lectrically vibrated crystals; pressurized air/oxygen is not needed. Metered do e inhalers are convenient handheld devices wh ich can be carried along, while ncbulizen. are used at patient's bed side. Nebulizers arc preferred for severe episodes of asthma as \\ell as for children and elderly. More than one drug can be nebuli1ed ;,imu ltaneously. Dry powder inhalers (DPls) are also portable device in which Lhc capsule (rotacap) containing the drug is puncnirecV cut across and the powder is aerosoli7ed by the inspiratory air flow of the patient. It requires h igh velocity inspiration which children, elderly and the very sick may not be capable of. The dry powder is also more likely to irritate the air passage producing cough and bronchoconstriction. Efficacy of aerosolized drug depends on the particle si7e: I 5 µm diameter particles deposit on the bronchioles and effectively deli ver the drug. Larger particles cttle on the oropharynx, wh ile , cry fi ne particles do not settle anywhere and are exhaled out. O n an average only - I 0% of the i nha led drug reaches t he site of action. A considerable fraction is S\\ allowed. Therefore. to mi nimize systemic action, the drug should have low

oral bioavai lability. Spacer devices improve inha led 10 drug ratio. Slow and deep inbreathing after device actuation and ho lding the breath after inhalation also enhances efficacy of the inhaler. Greater proportion of smaller particles in a rclativt:ly narrow band width of 1-2 pM can be generated using the newer HFA propellant based MDts. Thi;, improve, delivery of the drug to the smaller bronchioles. llowcvcr, systemic absorption from the peripheral lungs is also more.

S\\ allowed

CHOICE OF TREATMENT The severity of asthma symptoms ranges from transie nt respiratory difficu lty to incapacitating breath lessness, and characteristicall y fl uctuates over time. The goals of drug therapy in asthma are to re lieve the symptoms of breathlessness and wheezing, prevent recurrent exacerbations a nd to mini mise need fo r e mergency hospital izations. A stepwise approach to assist the c lini cal decisio n-making in the treatment of asthma as per needs of the patient has been advocated.The following guide lines summ arise the recommendations of the British Thoracic Society (20 12)* as well as that of the Nati onal Asthma Education and Prevention Progra mme. Expert Panel Report (2007).£ After the asth ma is under control fo r 3- 6 months, an attempt to reduce med ication s hou ld be made in a stepwise manne r. 1. Mild episodic asthma (symptoms less than once dai ly, normal in between allacks): In haled short-acting ~2 agonist at onset of each episode. Since asthm a is intermitte nt, it does not require continuous prophy lactic therapy (Step-1 }.

2 . Seasonal asthma Start regular low-dose ICS 3- 4 weeks befo re a ntic ipated seasona l attacks and continue till 3-4 weeks after the season is over. Alternatively oral montelukast/ zafirl ukast may be tried, especia lly in ch ildren. Treat asthma episodes with inhaled sho1t -acti ng ~ agonist. 2

3 . Mild chronic (persistent) asthma with occasional exacerbations (symptoms once

*British T horacic Socie ty and Scottish Intercollegiate Guidelines etwork (Jan 2012; www.b riuhoracic.org.u k) £National Asthma Education and Prevention Programme. Expert Pane l Report 3: Guidelines for the D iagnosis and M a nagem e nt of Asthma. Natio n a l Institute of Health P ub o . 08-405 1 (2007).

DRUGS FOR COUGH AN D BRONCH IAL ASTHMA

daily or so, subnormal ventilatory performance). Regular low-dose JCS (Step-2). Alternatively, montelukast/zafirlukast as above; but they may not be effective in some patients, or may afford less complete relief. Oral theophylline is a less satisfactory option. Episode treatment with inhaled short-acting p2 agonist. 4 . Moderate asthma with frequent exacerbations (allacks affect activity, occur > I per day or mild baseline symptoms persist). Low doses of ICS + inhaled long-acting p2 agonist (Step-3), or med ium dose ICS alone. In view of the potential risk of prolonged treatment with long-acting P, agonists. attempt should be made to discontin'iie it after maintaining asthma control over few months. Lcukotriene antagonists may be tried in place of long-acting p2 agonists, but their additive effect is less marked. Sustained release theophylline may be used as alternative additional drug to long-acting p, agonists, especially in nocturnal asthma. 5 . Severe asthma (continuous symptoms; activity limitation; frequent exacerbations/ hospitalization) Regular high dose ICS through a large volume spacer device + inhaled longacting P2 agonist (sa lmeterol) twi ce daily. Additional treatment with one or more of the following (Step-4): Leukotriene antagonist/sustained release oral thcophylline/sustained relea e oral P, agonist. Rescue treatment with short-acti~g inhaled p2 agonist. In patients not adequately controlled or those needing frequent emergency care- institute ora l steroid therapy (Step-5) while continuing long acting p2 inhalations and !CS. Efficacy of oral steroids is proven, but should be the last resort. Attempt to withdraw it should be made periodically. The British guidelines recommend continuing high dose !CS along with oral steroids.

6. Status asthmaticus/Refractory asthma Any patient of asthma has the potential to develop acute severe asthma which may be life-threatening. Upper respiratory tract infection is the most common precipitant. • Hydrocortisone hem isucc inate 100 mg (or equivalent dose of another glucocorticoid) i.v. star, followed by 100 200 mg 4-8 hourly infusion: may take upto 6 hours to act. • Nebulized salbutamol (2.5- 5 mg) + ipratropium bromide (0.5 mg) intermittent inhalations driven by 0 ,. • High flow humidified oxygen inhalation. • Salbutamol/terbutaline 0.4 mg i.m./s.c. may be added, since inhaled drug may not reach smaller bronchi due to severe narrowing/ plugging with secretions. • Intubation and mechanical ventilation, if needed. • Treat chest in feet ion wi th intensive antibiotic therapy. • Correct dehydration and acidosis with saline + sod. bicarbonate/lactate infusion. Aminophyllinc 250-500 mg diluted in 20- 50 ml glucose (5%) so lution injected i.,. over 20 30 min had been routinely used, but recent evidence shows that it does not afford additional benefit: may even produce more adverse effects; use is restricted to resistant cases.

Some antlasthma combinations BRO KOPLUS: Salbutamol 2 mg. anh)drous theophylhne I 00 mg tab.. also per 5 ml syrup. BRO, KOTuS: 13rornhe,me 8 mg, salbutamol 2 mg tab., also S)rup-bromhe,ine 4 mg. salbutamol 2 mg per 5 ml. TERPI IY LIN: rcrbutalinc 2.5 mg, ctophyllinc 100 mg tah. and per IO ml syr. TIIEO ASTI IAI I : Salbutamol 2 mg. theophylline anhydrous I00 mg tub. and per IO ml syr. TIIEO ASTHA LI -SR: Salhutamol 4 mg. thcophyllinc 300 mg SR tah. TII EO BRIC: Terbutaline :!.5 mg. thcophylline 100 mg tab. Tl IEOBRIC SR: Tcrbutalinc 5 mg. thcophylline 300 mg SR tab. DURASALY'-l-CK: Salbutamol 4 mg. theophyllme 200 mg CR cap.

253

254

RESPIRATORY SYSTEM DRUGS

er PROBLEM DIRECTED STUDY

16.1 A 30-year-old man presents with complaints of episodic breathlessness, often following exertion. One to three episodes occur daily and are accompanied by wheezing. Every 2-3 days, he wakes up at night with breathlessness. The episodes subside on taking 2 puffs of the inhaler prescribed by his family doctor (salbutamol 100 µg/puff). Lately, the episodes have become more frequent and are limiting his activities to some extent. Spirometry revealed FEV 1 to be 70% of predicted value at baseline, and 88% after 2 puffs of salbutamol (100 µg/puff) inhalation. He is diagnosed to be a case of moderate bronchial asthma. (a) Is this patient receiving adequate treatment for his condition, or some other drug/drugs need to be used? If so, what should be the first line treatment for this patient? (b) In case another drug is prescribed, should this patient continue to use salbutamol inhalations to relieve episodes of breathlessness, when they occur? (c) Which alternative drugs can be used in this patient.

16.2 A 60-year-old male patient of moderately severe chronic obstructive pulmonary disease (COPD) with FEV1 45% of predicted, who has quit smoking for the last 5 years, and is maintained on-lpratropium br. 20 µg/puff metered dose inhaler, 2 puffs 3 times a day, and Theophylline 400 mg SR tab. twice a day, developed sore throat and fever. He was prescribedTab Erythromycin 250 mg, one tab 4 times a day for 5 days Tab Paracetamol 500 mg 3 times a day till fever persists. After 3 days he presented with pain in epigastrium, restlessness, irritability, inability to sleep, palpitation, tremor of fingers and hand, and had vomited twice. His fever had subsided and throat was better. (a) What could be the reason for his recent illness? (b) Could this illness be prevented, if so, how? (see Appendix-1 for solution)

HORMONES AND RELATED DRUGS

Introduction

Hormone (G reek hormaein-10 stir up) is a substance of intense biological acti vity that is produced by specific cells in the body and is trans ported through circulation to act on its target cells. Hormones regulate body functions to bring about a programmed pattern of life events (growth, development. metabolism, reproduction, ageing, etc.) and maintain homeostas is in the face of markedly variable external and internal environment. Body function 1. Availability of fuel 2. Metabolic rate 3. Somatic growth

Major regulator hormone(s)

Thyroid sti mulating hormone (TS H) or Thyrotropi n. Gonadotropins -Follicle stimulating hormone (FS H) and Lutein izing hormone (LH).

(b) Posterior-Oxytocin, Antidiuretic hormone (ADH) or Vasopressin. 2 . Thyroid Thyroxine (TJ, Triiodothyron ine (T 1), Calc itoni n.

3. Parathyroid

Parathyroid hormone (PT H) or Parathormonc.

4 . Pancreas (Islets of Langerhans) Insul in, G lucagon.

Insulin, Glucagon, Growth hormone T riiodothyronine, T hyroxine Growth hormone, Insulin-like growth

factors 4 . Sex and reproduction

Gonadotropins, Androgens, Estrogens, Progestins

5. Circulating volume

Aldosterone, Antidiuretic hormone

6. Adaptation to stress

Glucocorticoids, Adrenaline

7. Calcium balance

Parathormone, Calcitonin, Vitamin D

Hormo nes arc secreted by the endocrine or

ductless glands. These are:

1 . Pituitary (a) Anterior Gro,.,vth hormone (G H), Prolactin ( Prl ), Adrenocorticotropic hormone (ACT H) o r Corticotropin.

5 . Adrenals (a) Cortex Glucocorticoids (hydrocortisone) Mineralocorticoids (aldosterone) Sex steroids (dehydroepiandrosterone) (b) Medulla Adrena line,

oradrenaline

6. Gonads Androgens (testosterone) Estrogens (estradiol) Progestins (progesterone) In addition, hypothalamus, which is a part of the C S and not a gland, produces many releasing and re lease inhib itory ho rmo nes which control the secretion of anterior pituitary hormones. Some impo rtant ones or these a re given in the box. Placenta also secretes many hormones: Chorionic gonado tropin Prolactin Estrogens Progesterone Placental lactogen Chorionic thyrotropin

256

HORMONES AND RELATED DRUGS

Hypothalamic hormone/factor

Chemical nature

1. Thyrotropin releasing hormone (TRH)

T ripeptide

2. Corticotropin releasing hormone (CRH)

Peptide (4 1 M s)

3 . Gonadotropin releasing hormone (GnRH, LH-RH/ FSH-RH) or Gonadorelin

Decapeplide

4 . Prolactin release inhibitory

Dopamine

hormone (PRIH) 5 . Growth hormone releasing hormone (GHRH) 6. Somatostatin (Growth hormone release inhibitory hormone)

Peptide (40, 44 M s) Peptide (14 AA)

fu nction: Ca2+ acting as third messenger in some situations. b. Th rough IP/ DAG generation : release of intrace llul ar Ca 2• and protein kinase C acti vation. c. Direct trans membrane activation o f ty ros ine pro tein k inase • phosphorylation cascade • regulation of various enzym es.

Vasopressin ( V2 receptor) Vasopress in ( V, receptor), Oxytocin

Insulin, Growth honuo ne Prolactin

The hormo nes act o n their spec ifi c receptors located on o r within their target cells. Receptor activation by the hormo nes is translated into respo nse in a variety o f ways.

2. At cytoplasmic receptors Pe netrating cell Steroidal hormones: membrane, ho rmone G lucocorticoids combines w ith a Mine ra locorticoid cyto plasmic receptor Androgens • exposes its DNA Estrogens binding do main • Pro gestins; m igrates to nucleus Calcitriol and binds to specific genes • D A med iated mR A synthesis • synthesis of functional proteins.

I . At cell membrane receptors

3. At nuclear receptor

The natural hor mo nes a nd in m any cases their syntheti c analog ues, wh ich may be mo re s uitable therapeutically, are used as drugs fo r substitution therapy as well as for pham1acotherapy. In addition, hormone antagonists and synthesis/ release inhibitors are o f therapeutic importance.

Sites and mechanisms of hormone action

a. Through alteration of intracellular cA MP concentration alteration o f protein kinase A • reg ulatio n of cell

Adrenaline, Glucagon, TSH, FS H, U T, PTH, Calcitonin. ACTH , so me hypo tha la mic releas ing hormones,

The hormone penetrates the nucleus • combines w ith its receptor • alters D A- RNA med iated protein synthes is.

Thyroid hormones: Thyrox ine, Triiodothyronine

Chapter

17

Anterior Pituitary Hormones

Anterior pituitary (adenohypophysis), the master endocrine gland, elaborates a number of important regulatory hormones. All of these are peptide in nature and act at extracellular receptors located on their target cells. Secretion of p ituitary hormones is controlled by the hypothalamus through releasing a nd releaseinhibitory hormones that a re transported via

hypothalamoh ypophyseal portal system, and is subjected to feedback inhibition by the hormones of their target glands. Each anterior pituitary hormone is produced by a separate group of cells, which according to their staining characteristic are either ac idophilic or basophilic. The p itu itary cells and the hormone they secrete are:

Anterior pituitary hormones

I

I

I

Acldophll cells

Basophll cells

Somatotropes - - Growth hormone (CH) Lactotropes- - - Prolachn

Thyrotropes - - Thyroid stimulating hormone (TSH) Corticotrope,,- lipotropes• Gonadotrope~

Adrenocorticotropic hormone (ACTH)

60 hours) than bromocriptine. It needs to be given only tw ice weekly. Incidence of nausea and vomiting is also lower; some patients not tolerating or not responding to bromocriptine have been successfully treated w ith cabergo line. It is the first choice drug for treatment of hyper-prolactinaemia; serum prolacti n levels fall to the normal range in 2-4 weeks, and many women conceive within one year. Cabergoline should be stopped w hen pregnancy occurs, though no teratogenic effect has been observed. Most micro- and some macroprolactinomas show regression during therapy, and neurological syrn ptoms (visual field defect , etc.) due to pressure on optic chiasma are re lieved. Response is generally mai ntained only till the drug is g iven, with recurrence on stoppage. Some patients who achieve total regression of prolactinoma and normaliz.ation of prolactin levels can stop cabergoline without recurrence.

261

262

HORM ONES AND RELATED DRU GS

Cabergoline is also beneficial in acromegaly due to pituitary adenoma, but efficacy is lower. It may be used to supplement surgery/radiation/ octreotide. Dose: Start after every CABERLI COLETTE

with 0.25 mg twice weekly; if needed increase

4-8 weeks lo ma:-.. of I mg twice weekly. 0.5 mg tab. CAMFORTE 0.5, I mg tabs. 0.25. 0.5 mg tubs.

GONADOTROPlNS (Gns)

The anterior pituitary secretes two Gns viz. FSH and LH. Both are glycoproteins containing 23- 28% sugar and consist of two peptide chains. The a -chain (92AA) is common between FS H and LH, but their ~-chains are different: FS H ( 111 A A), LH ( 12 1 AA ). Paradoxical ly the MW of FS H (- 33 KD) is greater than that of LH (-30 KO), because of the sugar moieties. Physiological functions FSH and LH act in concert to promote gametogenesis and secretion of gonadal hormones. FSH In the female it induces follicular growth, development of ovum and secretion of estrogens. In the male it supports spermatogenesis and has a trophi c in0uence on seminiferous tubules. Ovarian and testicular atrophy occurs in the absence of FSH.

LH It induces preovulatory swelling of the ripe graafian follicle and triggers ovulation fol lowed by luteinization of the ruptured follic le and sustains the corpus luteum till the next menstrual cycle. It is also probably responsible for atresia of the remaining follicles. Progesterone secretion occurs only under the influence of LH. In the male LH stimulates testosterone secretion by the interstitial cells and is designated interstitial cell s timu lating honno ne (ICSH ). Distinct L il and FS H receptors are expressed on the target cells. Bo th a re G protein coupled receptors "hich on activation increase cAMP production. This in tum stimulates gamelogcnesis and conversion of cholesterol lo pregncno lone-the first step in progesterone, testosterone and estrogen synthesis.

Regulation of secretion A single relca ing factor (decapeptidc designated GnRli) is produced by the hypothalamus ,, hich sti mulates synthesis and release of both FSH and LH from pituitary. It is. ihercfore, al o referred

to as FSHIL/1-RH or s imply LHRII or gonadorelin. It has been difficult to explain how hypothalamus achieves a divergent pa11ern of FSH and LH secretion in menstruating women through a single releasing hormone. Since Gn RH is secreted in pulses and the frequency as well as amplitude or ihc pulses differs during foll icular (high frequency, low a mplitude) and luteal (lower freq ue ncy, higher amplitude) phases. it is considered that frequency and amplitude of GnR H pulses determines whether F SH or LH o r hoth wil l be secreted. as well as the amo unt of each. Further, the feedback regulation o f FSH and LI I may be di fTercnt. In general. feedback inhibition or LH is more marked than that of FSH. In add ition there arc other regulatory substances. e.g. lnhibin- a peptide from ovaries and testes, selective ly inhi bits FSH release, but not LH release. Dopamine inhibits only LH release. Testosterone is weaker than estrogens in inhibiting Gn secretion, but has efTcct on both FSH and LH. GnR H acts on gonadotropcs through a G-protein coupled receptor which acts by increasing intracellular Ca'' through PIP, hydrolysis. The Gn secretion increases at puberty and is higher in women than in men. In men, the levels of FSH and LH remain practically constant ( LH > FSl-1) whi le in mens truating women they fluctuate cyclically. During the follicular phase, moderate levels of FSII and lo" levels of LH prevail. There is a midcycle surge of both, but more of LI I. just before ovulation. followed by progressive fall during the lutcal phase. Gn levels are high in menopausal women due to loss of feedback inhibition by sex steroids and inh ibin.

Pathological Involvement

Disturbances ofGn secretion from pituitary may be responsible for delayed puberty or precocious puberty in g irls as well as in boys. Inadequate Gn secretion results in amenorrhoea and sterility in women; oligo.lOospermia, impotence and infertility in men. Excess production of G n in adult women causes polycystic o, arics.

Preparations All earlier gonadotropm preparations were administered by i.111. route. The newer more purified preparations can be given s.c. T hey are panly metaboli£Cd. but mainly excreted unchanged in urine: t½ 2-{i hours.

I. Me11olropi11s (FSII + UI): is a preparation o btained from urine of menopausal women: PREGNORM, PERGONAL. GYNOGEN 75/ 150; 75 IU FSII + 75 IU LH activity per amp, also 150 tU FSH + 150 I U Lil per amp. 2. Urofollitropi11 or Me1101rop111 (pure FSH): METRODI . FOLGEST, FOLICULfN, PUREGON 75 IU and 150 JU per amp. This preparation has been preferred over the combined FSH + LI I preparation for induction of ovulation in women with polycystic ovarian disease. T hese patients have c lc,ated LI-VFSH ratio; use of FSH alone is considered advantageous. This preparation is also claimed 10 improve chances of obtaining good quality ova for in ritro fcrtiliLation.

ANTERIOR PITUITARY HORMONES 3. lluman chorionic gonadotmpin (hCG) · is derived from urme of pregnant women.

CORION, PROfAS I. PUBERGE 1000 JU, 2000 JU, 5000 Iu, I 0,000 I U. all as dry po~ der with ~eparnle solvent for injection. The foetal placenta secretes hCG "hich is absorbed in maternal circulation and ma intains corpus luteum of pregnancy. It is a g lycoprotcin with 33% sugar and 237 amino acids in t" o chains, MW 38000. ll is excreted in urine by the mother from which it is commercially obtained. hCG binds lo LH receptor with equal a, idity; action of hCG is indistinguishable from that o f LH. Recombinant human FSH ( rhFSH o r Follitropin a and P) and recombinant human LH (rhLH or lutropin) as well as recombinant hCG (rhCG or Choriogonadotropin a) have been produced. These are more purified and have venually replaced the urine deri\ ed preparations in the developed countries. They are more expensive. Lutropin (rhll I} has been "ithdrawn.

follitropin

Uses I . Amenorrhoea and infertility Exogenous Gns may he lp w he n amenorrhoea/ in fertil ity is due to deficien t production of Gns by pituitary. Gn s are gene rall y tri ed when attempts to induce ovulation with clomiphe ne have fai led or w he n nonovulati on is due to polycystic ovaries. Several protocols fo r use of Gns have been e mpl oyed, one of which is to g ive I injec ti on of menotropins (75 IU FSH .,. 75 JU LH or 75 IU pure FSH)) i.m . dai ly fo r 10 days fol lowed the next day by 10,000 IU of hCG. Ovulation occurs within th e next 24 48 hours in upto 75% cases and the woman may conceive if insemi nated at this time. Howeve r, rates o f a bortion and multipl e pregnancy are hi g h , bu t n ot o f teratogenesis. To improve the predi ctab ility of time of ovulation (controlled ovarian s1imulation), it is the standard practice now to conc urrently suppress endogenous FS H/ LI I secretion either by continuo us pretreatment with a superactive GnRH agonist or by a GnRH a ntagonist. 2. To aid in vitro fertilization Mcnotropins (FS H + LH or pure FS H) is used a long the GnRH agoni st/ a ntagon ist (for suppress ing endogenous FSH/ LH) to induce simultaneous maturation of several ova and to precisely time

ovulation so as to faci litate their harvesting for assisted reproductive techniques.

3. Male hypogonadotropic hypogonadism It manifests as delayed puberty or defective spermatogenesis produc ing oligozoospermia and male sterility. Generally, sexual maturation is induced by androgens and therapy with hCG is started when fertility is desired. Start with hCG i.m. 2- 3 times a week (to stimulate testosterone secretion), add FSH 75 IU + LH 75 lU after 3-4 months (to stimulate spermatogenesis) and reduce dose o f hCG. Treatment is continued for 6- 12 months for optimum results, whic h neverthe less are not a lways impressive. 4. Cryptorchidism A 2-6 weeks course o f hCG was used to treat undescended testes in early childhood. However, this is not practiced no" due to poor eflieacy. risk of precocious pubeny and other concerns. Surgical correction is the preferred modality.

Adverse effects and precautions Ovarian hyperstimulation syndrome is the most serious complication. Polycystic ovary, pa in in lower abdomen and even ovarian bleeding and shock can occur in females during ovulation induction. Multiple pregnancy 1s another complication. Precocious puberty is a risk w hen g iven to children . Alle rgic reactions have occurred and skin tests are advised. Hormone dependent malignancies (prostate, breast) must be excluded. Other side effec ts are ede ma, hea dache, mood c hanges. GONADOTROPIN RELEASING HORMONE {GnRH) {GONADORELIN) Synthetic hum an GnRH (gonadorcl in) injected i. v. ( I00 µg) induces prompt relea~e of LH and FSH followed by elevation of gonadal steroid levels. ll has a shon plasma t½ (4-8 min) due 10 rapid enzymatic degradation. ll has been used for testing pituitary-gonadal axis in male as well as female hypogonadism. Only pulsatile exposure 10 GnRH induces FSII/LH secretion. while sustained e.\posure desensil17cs the pituitary gonadotropes resulting in loss of Gn rc lea;,e. Therapy with GnRI I is not useful in the treatment of hypogonadism.

263

264

HORMONES AND RELATED DRUGS

Superactive/long-acting GnRH agonists

Many analogues of GnRH , e.g. Goserelin, Leuprolide, afarelin, Triptorelin, have been deve loped which are 15- 150 times more potent than natural Gn RH and longer acting (t ½ 2- 6 hou rs) because of high affinity for GnRH receptor and res istance to enzymatic hydrolysis. Because phys iological release of GnR H is in pulses, whereas these agonists act continuous ly; they on ly initially trigger Gn secretion . After 1-2 weeks they cause desensitization and down regulation of GnRH receptors causing inhibition of FSH and LH secretion followed by suppression of gonadal function. Spem1atogenesis or ovulation cease and testosterone or estradiol levels fall to castration levels. Recovery occurs within 2 months of stopping treatment. The superactive Gn RH agonists are used as nasal spray or injected s.c/i.m. Long-acting preparations for once a month s.c. injection have been produced (triptorel in, gose rel in depot). The resulting reversible pharmacological oophorectomy/orchidectomy is being used in precocious puberty, prostatic carcinoma, endometriosis, premenopausal breast cancer, uterine leiomyoma, polycystic ovarian disease and to assist induced ovulation. They also have potential to be used as contraceptive for both males and females. Menopausal symptoms, viz. headache, sweating, hot flashes, mood changes, vaginal dryness amenorrhoea and loss of libido can occur due to suppression o r ovarian fun ction. Use of GnR H agonists for more than 6 months carries risk of developing osteoporosis. Nafarelin This long-acting GnRH agonist is 150 times more potent than native GnRJ-1. It is used as intranasal spray from which bioavai lability is only 4-5%. ASA R EL 2 mg/ml soln for nasal spray; 200 µg per actuation.

Down regulation of pitui ta ry GnRH receptors occurs in IO days but peak inhibition of Gn release occurs at one month. It is broken down in the body to shorter peptide segments; plasma t½ is 2-3 hou rs. Uses are:

Assisted reproduction: Endogenous LH surge needs to be suppressed when controlled ovarian stimulation is attempted by exogenous FSH and LH injection, so that precisely timed mature oocytes can be harvested . This is ach ieved by 400 µg 8D intranasa l nafarelin, reduced to 200 µg 8D when menstrual bleeding occurs. Uterine fibroid : Nafarel in 200 1.1g 8D intranasal for 3-6 months can reduce the size of leiomyoma and afford symptomatic relief. Endometriosis: 200 µg in alternate nostril BD fo r upto 6 months. It is as elTective as danazol, but second course cannot be given due to risk of osteoporosis. Cemra/ precocious puberty: 800 µ g 8D by nasal spray; breast and genital de velopment is arrested in girls and boys. Treatment is generally carried out till the age of 11 years in gi rls or 12 yea rs in boys. The effect is reversible; pubertal changes resume when therapy is discontinued. Adverse effects: Hot flashes, loss of libido, vagi nal dryness, osteoporosis, emotional (ability. Goserelin Another long-acting GnRH agonist available as a depot s.c./i.111. injection to be used both for endogenous Gn suppression before ovulation induction, as well as for endometriosis, carcinoma prostate, etc. To achieve pitui Lary desensitization before ovulation induction with exogenous Gns; 3.6 mg of depot goserelin injection is given once in the anterior abdominal wall 1- 3 weeks earlier. For endometriosis and carcinoma prostate 3.6 mg is injected in the same way every 4 weeks or I 0.8 mg is injected every 3 months. An androgen antagonist (bicalutamide) is given concurrently for 3-4 weeks when gosereli n is used fo r carcinoma prostate. This is needed to block the effect of androgen secreted under the influence of enhanced Gn production by initial agonistic action of the GnRH agonist on gonadotropes. ZOLADEX 3.6 mg prefilled ~yringe. ZOLDEX L-A 10.8 mg ~ial depot injection.

ANTERIOR PITUITARY HORMONES

Triptorelin : This long acting GnRH agonist is fo rmulated as a regula r re lease dail y s.c. injection for short term indications, such as lemale infertility, and as a de pot i.m. monthly injecti on for long-term Gn suppression in the treatment of carcinoma prostate, endometriosis, precocious p uberty and uterine leiomyoma. For prostate cancer. it is combined w ith an androge n a ntagonist flutamide or bicaluta mide to prevent the initi al flare up of the tumour that occurs due to increase in Gn secretion for the first 1- 2 weeks. Fibroids. cndometriosis, carcinoma prostate: 3.75- 7.5 mg i.111. every 4 weeks. Precocious puberty: 50 ~1g/kg i.111. of depot inj. nery

4 weeks. Ass isled reproduction: 0. 1 mg s.c. daily for IO days from 2nd day of cycle. D ECAP EPTYL D A ILY 0 . 1 mg inj .. D ECA l'I- PTY L DEPOT 3.75 mg inJ.

Leuprolide This long acting GnRH agon ist is injected s.c./i.m. da ily or as a depot injection once a month for palliation of carcinoma prostate alongwith an androgen antagonist, as well as for other conditions needing long term Gn suppression. Daily .c. injections are also used for assisted reproduction. LUPRIDE I mg inj .. 3. 75 mg depot inj ., PROGTASE I mg/ ml inj .

GnRH antagonists Four analogues of GnRH, Ganire/ix, Cetrorelix, Degarelix and Abare/ix which are more extensively substituted act as GnRH receptor antagonists. They inhibit Gn secretion without causing initial stimula tio n. Ganirelix and Cetrorelix a re relatively short ac ting. Administe red daily by s.c. injection, they are approved fo r inhibiti ng LH surges during controlled ovarian stimulation in women undergoing in vitro fertilizati on. The ir advantages over long-acting GnRH agonists include: • They produce quick On suppression by competitive antagonism, and need to be started only from 6th day o f a ttempting ov ar ian stimulation with Gns. • They carry a lower ri sk of ovaria n hyperstimulation syndrome.

• They achieve more complete suppression of endogenous Gn secretion. However, pregnancy rates are sim ilar or may even be lower. Degarelix and Abare/ix are long acting GnRH antagonists, appro, ed for androgen withdrawal therapy of advanced carcinoma prosiatc.

TH YROID STIMULATING HORMONE (TSH, THYROTROPIN) It is a 2 10 amino acid, two chain glycoprote in (22% sugar), MW 30000.

Physiological function TSH stimula tes thyro id to synthesize and sec rete thy roxine (T4 ) and tri iodothy ron ine (T 1 ) (see Fig. 18. 1). Its actions are: • Induces hy perplas ia a nd hype rtrophy of thyroid folli cles and increases blood suppl y to the gland. • Pro motes trapping o f iod ide into thy roid by increasing a ': Iodide symporter (N IS). • Promotes o rganifi cation of trapped iodine and its incorpo rat io n into T 3 and T~ by increasing peroxidase activity. • Enhances endocytotic uptake of thyroid co llo id by th e follicular cells and proteolys is of thyroglobulin to re lease more o f T3 and T 4 • This action starts wi thin minutes of TS H administration. The TS H receptor present on thyroid cells is a GPCR which u1 ilizcs the adenylyl cyclase-cAM P transducer mechanism (by coupling to Gs protein) to produce its eftccts. In human thyroid cells high concentration of TSH a lso induces PIP, hyd rolysis by the linking of TS! I receptor to Gq protei~. The resulting increase in cytosolic Ca' and protein kinascC activa1ion may also mediate TS H action, particularly generation of 11,O, needed for oxidation of iodide and iod ination of tyrosil residues.

Regulation of secretion Symhcsis and release of TSH by piruitary is controlled by hypothalamus primarily through T RH (see Fig. 18.3), whil e somatostatin inh ibits T S H secretion. The T RJ 1 receptor on p11ui1ary thyrotrope cells is a G PC R wh ich is lin ked to Gq protein and activates PLC- IP/ DAG-cytosolic Ca' pathway to e nhance TSH synthesis and release. The negative feedback for inhibiting TSI I secretion is provided by the thyroid hormones which act primarily at the level of the pituitary, but also in the hypothalamus.

265

266

HORMONES AND RELATED DRUG S Pathological Involvement Only few cases of hypoor hypertl1yroidis111 are d ue to inappropriate TS H secretion. In majority o f cases of myxoedema TSH levels are markedly elevated because of deficient feedback inhibition. Graves' disease is due to an immunoglobulin of the lgG class which attaches to the thyroid cells and stimulates them in the same way as TSH. Consequently, TSH levels are low.

through cAMP ): high doses cause hypertrophy a nd hyperplasia. Lack of ACT H results in adrenal atrophy. However, zona g lomerulosa is little affected because angiotensin II also exerts trophi c influ ence o n this layer a nd sustains aldosterone secretion.

Use Thyrotropin has no therapeutic use. Thyrox ine is

Regulation of secretion Hypothala mus regulates ACTH release from pituitary through corticotropi n-releasing hormone (CRJI). The CRJl receptor on corticotropes is also a GPCR which increases ACTH synthesis as well as release by raising cytosolic cAM P. Secretion of ACTH has a circadian rhythm. Peak plasma levels occur in the early morning, decrease during day and are lowest at midnight. Corticosteroids exert inhibitory feedback influence on ACT! I production by acting directly on the pituitary as well as indirectly through hypothalamus. A variety of stressful stimul i, e.g. trauma, surgery, severe pain, anxiety, fear, blood loss, exposure to cold, etc. generate neural impulses w hich converge on median eminence to cause elaboration of CRH. The feedback inhibition appears to be overpowered during stress-rise in ACTH secretion continues despite high plasma level o f cortisol induced by it.

the drug of choice even when hypothyroidism is due to TSH defi ciency. The d iagnostic application is 10 differentiate myxoedema due 10 pituitary dysfunction from primary thyroid disease.

ADRENOCORTICOTROPIC HORMONE (ACTH, CORTICOTROPIN) It is a 39 ami no acid single chain peptide, MW 4 500, derived from a larger pe ptide pro-opio melanocortin (MW 30,000) which also gives rise to endorphins, two lipotropins and two MS Hs.

Physiological function

ACTH promotes steroidogenesis in adrena l cortex by stimulating cAMP formation in corti cal cells (through specific cell sur face GPC Rs). This inturn rapidly increases the availa bility of cholesterol for conversion to pregnenolone w hich is the rate limiting step in the production of g luco, mineralo and weak ly androgenic steroids (see Fig. 20. 1). Induction o f steroidogenic enzymes occ urs after a de lay res ulting in 2nd phase ACTH action. The stores o f adrenal steroids are very limited and rate of synthesis primarily governs the rate of release. ACTH also exerts tro phic influence on adrenal cortex (again

Pathological Involvement

Excess production of ACT H from basophil pituitary tumours is responsible for some cases of C ushing's syndro me. Hy pocorticism occurs in p ituitary insufficiency due to low ACT H production. Iatrogenic suppression of ACTH secretion and pituitary adrenal axis is the most common form of abnormality encountered currently due to the use of pharmaco logical doses of glucoconicoids in noncndoerine diseases.

Use ACT! I is used rarely for the diagnosis of disorders of pituitary adre nal axis. Direct assay o f plasma ACTH level is now preferred. For therapeutic purposes, ACT H does not offer any advantage over cortieosteroids.

Chapter

18

Thyroid Hormones and Thyroid Inhibitors

THYROID HORMONES The thyroid gland sec retes 3 hormones- thyroxine (T), triiodothyronine (T1 ) and calcitonin. The former two are produced by thyroid follicles, have similar biological activity and the term 'thyroid horm one' is restricted to these only. Calcitonin produced by interfollicular 'C' cells is chemically and biologically entirely different. It is considered along with parathormone (see C h. 24) with which it regulates calci um metabolism. The physiolog ical significance of thyro id g land was recognized only after Graves and Basedm, ( 1835, 1840) associated the clinica l features o f the ·Graves' disease' with swelling of thyroid gland and Gull ( 1874) correlated myxoedema with its atrophy. Kendall ( 19 15) obtained crysta lline thyroxine and postulated its chemical formula which was confirmed in 1926. Thyroxine was the first hormone to be synthesized in the laborator) . Since T4 could not account for all the b iological activity o f thyroid extract. search was made and more potent T3 "as discovered in 1952.

Chemistry and Synthesis Both T 4 and T, are iodine containing derivatives of thyronine which is a condensation product of two molecules or the amino acid tyrosine. Thyroxine is 3 , 5, 3 ' , 5 ·- tctraiodothyronine whi le T.1 is 3, 5, 3 · triiodothyronine. The thyroid hormones are synthesized and stored in the thyroid follicles as part of thyroglobulin molecule- which is a glycoprotein synthesized by thyroid cells, MW 660 KDa, contains I0% sugar. The synthesis, storage and release of T4 and T, is summarized in Fig. 18. 1 and involves the following processes.

1 . Iodide up take The total body conte nt of Ir obtai ned from food and water, is 3050 mg, out of which about 1/5 is present in the thyroid. Concentration of iodide in blood is low (0.2- 0.4 µg /dl) but thyroid ce ll s have an active transport process a-=: iodide symporter (N/S) to concentrate this anion; this trapping is stimulated by thyroid stimulating hormone (TS H) to exceed a gradient of more than l 00 fold by induci ng and activating NIS. The 12 content of thyroid g land somehow regulates the uptake mechanism: meagre store activating and large store inhibiting it. The iodide concentrating mecha nism is not peculiar to thyroid. Skin, salivary glands, gastric mucosa, intestine, mammary g lands and placenta also possess it, but uptake in these organs is no t stimulaced by TSH. 2 . Oxidation and iodination Iodide trapped by follicular cells is carried across the apical membrane by another transporter termed 'p endrin' an d oxidized by the membrane bound thyroid peroxidase enzyme to iodinium W) ions or hypoiodous acid (RO I) or enzyme-linked hypoiodate (E-01) with the help of H 2 0 2 · These forms of iodine combine avidly w ith tyrosil residues of thyroglobulin, a pparen tl y without any enzymatic intervention, to form monoiodotyros ine (M IT) and di iodotyrosine ( DIT) while these residues are still attached to the thyroglobulin chains.

3 . Coupling Pairs of iodinated tyrosi l residues couple together (Fig. 18.2) to form T 3 and T,.

HORMONES A ND RELATED DRUGS

268

r

0

0

T3

Deiodination

T4

0 6

T3

NIS Secretion 5 Trapping

010

1

l r

T3 MIT O Deiodination

Fig. 18.1: Synthesis, storage and secretion of thyroid hormone TG-Thyroglobulin; MIT-Monoiodotyrosine; DIT-Diiodotyrosine; T 3- Triiodothyronine; T4 - Thyroxine (Tetraiodothyronine); HOI- Hypoiodous acid; EOI-Enzyme linked hypoiodate; NIS-Na•-iodide symporter; Thyroid-stimulating hormone (TSH) activates steps 1, 2, 3, 4, and 5; Ionic inhibitors block step 1; Excess iodide interferes with steps 1, 2, 3 and 5 with primary action on step 3 and 5; Propylthiouracil inhibits steps 2 and 6; Carbimazole inhibits step 2 only

HO -of;:,7~~~-:-10-0-cH,yHcOOH I MIT 3

NH2 I ,I ___________ J

t

I

3 DIT

Coupl ng (Peroxidase)

NH2

MIT+ DIT , T3 DIT + DIT-+ T4

I 5

HO -Q--o-9-cH"rHCOOH I 3'

I 3

NH2

3,5.3 '-Tmodothyromne (T3 )

Fig. 18.2: Coupling of monoiodotyrosine (MIT) and diiodotyrosine (DIT) to produce triiodothyronine (T3)

T HYROID HORMONES AND THYROID INHIBITORS

Normally much more T4 than T 3 is formed, but during I, deficiency relatively more MIT is available and a greater proportion of T 3 is formed. The result is- more active hormone is generated with lesser amount of 12. Cou pling is an oxidative reaction and is catalysed by the same thyroid peroxidase. Thyroglobulin is the most efficient protein, compared to other si milar prote ins, in supporting coupling by providing favourable spatial configuration to faci li tate the reaction. Oxidation of iodide and coupling are both stimulated by TSH. 4 . Storage and release Thyroglobulin containing iodinated tyrosil and thyronil residues is transported to the interior of the fo llicles and remains stored as thyroid colloid till it is taken back into the cells by endocytosis and broken down by lysosomal proteases. The T 4 and T 1 so released is secreted into circulation while MIT and DIT resid ues are deiodinated and the iodide released is reutilized. The uptake of colloid and proteolysis are stimulated by TSH: the quiscent gland has follic les distended with colloid and cells are flat or cubical, while the TSH stimul ated gland has columnar cells and colloid v irtual ly disappears. Normal human thyroid secretes 60- 90 µg of T 4 and 10-30 ~tg of T 3 daily.

5 . Peripheral conversion of T4 to T3 Peripheral tissues, especiall y liver and kidney, convert T 4 to Tr About 1/3 of T 4 secreted by thyroid undergoes this change and most of the T 3 in plasma is derived from liver. Target tissues take up T 3 from circulation for their metabolic need, except brain and pituitary which take up T 4 and convert it to T 3 within their own cells. Almost equal amounts of 3, 5, 3 · triiodothyron ine (normal T 3 : active) and 3, 3', 5 ' triiodothyroninc (reverse T 3 or rT, : inactive) are produced in the periphery. The T 4 to T 3 conversion is carried out by the enzyme iodothyronine deiodinase which exists in 3 forms (D I, 02, 03). These forms difTer in their organ and cellular localization as well as in th e product formed. Whereas type 2 deiodinase (02) generates T 3 and 0 3 generates

rT3 , the D I form generates both T 3 a nd rTr The antithyroid drug propylthiouracil (but not carbimazole) inh ibits Type I dciodinase and the antiarrhythmic amiodarone inhibits both DI and 02 forms. Propranolol (high dose) and glucocorticoids also inhibit peripheral conversion of T4 to T3 (except in brain and in pi tu itary).

Transport, Metabolism and Excretion Thyroid hormones are avidly bound to plasma proteins-only 0.03- 0.08% of T 4 and 0.2- 0.5% of T 3 are in the free form. A lmost all protein bound iod ine (PB I) in plasma is thyroid hormone. of which 90-95% is T 4 and the rest T 3 • Binding occurs to 3 plasma protei ns in the fol lowing decreasing order of affinity for T4 : (i) Thyroxine binding globulin (TBG) (i i) T hyrox ine binding prealbumin (trans-thyretin) (iii) Albumin The normal concentration of PBI is 4-10 µg/ di; only 0.1-0.2 µg/dl of this is T 1 , rest is T 4• During pregnancy thyroxine binding globulin is increased- PB I levels are e levated, but there is no effect on thyroid status because the concentration of free hormone remains unaltered. Only the free hormone is availab le for action as well as for metabolism and excretion. Metabolic inactivation of T 4 and T 3 occurs by deiodination and g luc uronidc or sulfate conjugation of the horm ones as we ll as that of their deiodinated products. Li ver is the primary site (a lso salivary g lands and kidneys). The conjugates are excreted in bile, of which a significant fraction is deconjugated in intestines and reabsorbed (enterohepatic circulation) to be finally excreted in urine. Plasma t½ of T 4 is 6-7 days. while that of T 1 is I 2 days. The half-lives are shortened in hyperthyroidism and prolonged in hypothyroidism due respectively to faster and slower metabolism.

Regulation of Secretion The secretion of hormones from the thyroid is controlled by anterior pituitary by the elaboration of TSI I, while TSI I secretion itself is

269

270

HORMONES AND RELATED DRUGS

TRH

_ §.S.J_~

e A~

----,\ H.~:~thal

ry~ ' : : ,

0,

(

TSH

''

-

\

I

,,

/

s

/

T3 T. ........

)

---+ Stimulation - - • Inhibition

Fig. 18.3: Regulation of thyroid function TSH- Thyroid stimulating hormone; TAH-Thyrotropin releasing hormone; T3- Triiodothyronine; T•- Thyroxine; SST-Somatostatin

regulated by TRJ I produced in hypothalamus (see p. 265). Somatostatin elaborated by hypothalamus inhibits not only GH and prolactin, but also TSH secretion from p ituitary. The relation between thyroid, anterior pituitary and hypothalamus is depicted in Fig. 18.3. The negative feedback by the th yroid hormones is exercised directly on the pituitary as well as through hypothalamus. The action of TR.I-I on pituitary and that of TSH on thyroid cells is mediated by enhanced cAMP synthes is. High concentration of TSH also acts via IP/ DAG- increased intracellular Ca1- pathway in the thyroid cells. T he blood iodine level and iodine content of thyroid also regulate thyroid function (see p. 277).

Actions The actions of T 4 and T 3 are qualitatively simi lar and are nicely depicted in the features of hypo- and hyperthyroidism. They affect the function of practically every body cell.

1. Growth and development T4 and T 3 are essentia l for normal growth and development. The most remarkable action is metamorphosis of tadpole to frog: the tai l is used-up to build lungs, limbs and other organs. The actions

cannot be broadly labelled as catabolic or anabolic, and are exerted through a critical control of protein synthesis in the translation of the genetic code. Congenital deficiency of thyroid hormone resulting in cretinism emphasizes its importance. The milestones or development are delayed and practically every organ and tissue of the body suffe rs. The greatest su fferer, however, is the nervous system. Retardation and nervous deficit is a consequence of paucity of axonal and dendritic ramification, syna pse formation and impai red myelination. In adult hypothyro id ism also, intelligence is impaired and movements are slow. 2 . Intermediary metabolism Thyroid hormones have marked efTect on lipid, carbohydrate and protein metabolism.

Lipid T4 and T 3 indirectl y enhance lipolysis by potentiating the action of catec holamines and other lipolytic hormones. As a resul t plasma free fatty acid levels are elevated. Lipogenesis is also stimulated. All phases of c holesterol metabolism are accelerated, but its conversion to bile acids dominates. Thus, hyperthyroidism is characterized by hypocholesterolemia. Plasma LDL levels are reduced. Carbohydrate Ca rbohydrate metabolism is also stim ulated. Though utilization of sugar by tiss ues is increased ( mainl y secondary to increased BMR), glycogenolysis and gluconeogenesis in liver as well as faster absorption of glucose from intestines more than compensate it produc ing hyperglycaemia and a diabetic-like state with insulin resistance in hyperthyroidism. Protein Synthesis of certai n proteins is increased, but the overall effect of T, is catabolicincreased amounts of protein being used as energy source. Prolo nged action resu lts in negative nitrogen balance a nd tissue wasting. Weight loss is a feature of hyperthyroidism. T 3, T 4 inhibit mucoprotein synthesis which so characteristically accumulates in myxoedema. 3 . Calorigenesis T , and T 4 increase BMR by stimulation of cellular metabolism and reselling of the energystat. This is important

TH YROID HORMONES AND TH YROID INH IBITORS for rnainta111111g body temperature. However, metabol ic rate in brai n, gonads, uterus, spleen and lymph nodes is not significantly affected. The mechanism o r calorigenesis was believed to be uncoupling of oxidative phosphoryla tion: excess energy being released as heat. However, this occurs only at very high doses and is not involved in mediating the physiological actions or T;, T4 • 4 . CVS T 3 and T 4 cause a hyperdynamic state of c irculation wh ich is partly secondary to increased peri pheral demand and partly due to direct cardiac actions. Heart rate, contractility and output are increased resulting in a fast, bounding pulse. T3 and T 4 stimulate heart by di rect action on contractile e lements (increasing the myosi n fraction having greater Ca2 ATPase activ ity) as we ll as by up regul a ti on of adre nergic receptors. Atrial fi brillation a nd other irregula rities are common in hyperthyroid ism. T hyroid ho1mones can a lso precipitate C HF and angina. BP, specially systo li c, is often raised. Myocardi a l 0 2 consumption can be marked ly reduced by inducti on of hypothyroid ism. 5 . Nervous system T 3 , T 4 have profo und fu nctional effect on C 1S. Menta l retardation is the ha ll mark of creti nism; s luggishness and other behavioral features are seen in myxoedema. Hyperthyroid indi viduals are anx ious, nervous, excitable, exhi bi t tremors and hyperreftexia. 6 . Skeletal muscle Muscles are flabby and weak in myxoedem a, wh ile thy rotox icos is prod uces increased muscle tone, tremor and weakness due to myopathy. 7 . GIT Propulsive activity of gut is increased by T/ f 4 • Hypothyroid patients are often constipated, while dia rrhoea is common in hyperthyroid ism. 8 . Kidney T 3 and T 4 do not cause diuresis in euthyro id indi viduals, but the rate of uri ne flo w is often increased when myxoedematous patients are treated with it. 9 . Haemopoiesis Hypothyroid patients suffer from some degree of a naemia which is restored only by T4 treatment. Thus, T 4 appears to be fac ilitatory to erythropoiesis.

10. Reproduction T hyroid has an ind irect effect on reproduction. Fertility is impa ired in hypothyroidism and wome n suffer from oligome norrhoea. orrnal thyroid fu nction is required fo r maintenance of pregna ncy and lactation. Mechanism of action Both T 1 and T 4 penetrate cel ls by acti ve transport a nd produce majority of their actions by combining with a nuc lear thyroid honnone receptor (TR) which be longs to the teroid and retinoid superfamily or intracellular receptors. Two TR isofom1 families (TRa and TR~) have been identified . Both bind T, and function in similar manner, but their tissue distribution differs, which may account for quantitative diflercnccs in the sensitivity of diflerent tissues to T 1.

ln contrast to the steroid receptor, the TR resides in the nucleus even in the unl iganded inactive state. It is bound to the 'thyroid hormone response element' (TRE) in the enhance r region of the target genes a long with corepressors ( Fig. 18.4). T his keeps gene tran cription suppressed. When T 1 binds to the ligand-binding domain of TR, it heterodimerizes w ith retinoid X receptor (RXR) and undergoes a conformation change releasing the corepressor and binding the coactivator. This induces gene transcription fol lowed by production of specific rnRNA a nd a specific pattern o f protei n syn thesis which produces the va rious metabolic and anatom ic effects. The expressio n of certain genes is repressed by Tr In their case, th e unliganded TR a llows gene transcription, w hile b inding of T3 to T R halts the process. Many of the effects of T 3 e.g. tachycard ia, arrhythmias, raised BP, tremor, hype rglycaemia are med ia ted, at least partly, by sensitization of adre ne rg ic rece ptors to ca tec ho la mines. Induction of adenylyl cyclase, proliferation of adrenoceptors and a better coup ling be tween these two has been demonstrated. Apan from the nuclear T 1 receptor, other sites of thyroid hormone action have been described. It acts on cell membrane to enhance amino acid and glucose entry and on mitochondria to increase oxygen consumption. At these sites T, appears lo be equipotent to T 3, while al lhe nuclear receptor T, has much lower afflllity. Moreover, even on binding to the TR, T, does not promote gene transcription.

271

HORMONES AND RELATED DRUGS

272

Transcription repressed

II mRNA ~

cription

Fig. 18.4: Mechanism of action of thyroid hormone on nuclear thyroid hormone receptor (TR) T - Triiodothyronine; T 4- Thyroxlne; TRE-Thyrold hormone response element; RXR-Retinoid X receptor; 3 mRNA-Messenger ribonucleic acid; 5'Dl-5'Deiodinase (See text for explanation)

Relation between T 4 and T 3

Preparations

• Thyroid secretes more T 4 than T 3 , but in iodine deficient state th is difference is reduced. • T 4 is the maj or circulating hormone because it is 15 times more tightly bound to plasma proteins. • T 3 is 5 times more potent than T4 and acts faster. Peak effect of T 3 comes in 1- 2 days while that of T 4 takes 6-8 days. • T 3 is more avidly bound to the nuclear receptor than T 4 and the T 4-receptor complex is unable to activate/derepress gene transcription. • About l /3 of T 4 is converted to T 3 in the thyroid cells, liver and kidney by type I deiodinase (DI ) a nd released into circulation. ln addition, T 3 is generated within the target cells (skeletal muscle, heart, brain, pituitary) by another type (D2) of deiodinase. Thus , it may be concluded that T 3 is t he active hormone, while T 4 is mainly a tran sport fonn which functions as a prohonnone of Tr However, it may directly produce some nongenomic actions.

I-thyroxine sod. (synthetic levothyroxine sod.): ELTROXIN 25 µg, 50 µg, 100 µg tabs, ROXIN 100 µg tab, THYRO1'0RM 12.5 µg, 25 µg, 50 µg, 62.5 µg, 75 µg, 88 µg, 100 µg, 112 µg, 125 µg. 137 µg. 150 ~1g tabs, TIIYROX 25 µg, 50 µg, 75 µg. I 00 µg tabs. An injectable preparauon for i.v. use is available elsewhere. Triiodothyronine (Liothyronine) is not freely available in India. It is occasionally used i.v. along with I-thyroxine in myxoedema coma.

C linically, I-thyroxine is preferred over liothyronine for routine as well as urgent indications, because of more sustained and uniform action as well as lower risk of cardiac arrhythmias.

Pharmacokinetics and interactions Oral bioavailability of I-thyrox ine is ~ 75%, but severe hypothyroidism can reduce oral absorption. It should be administered in empty stomach to avoid interference by food. Sucralfate, iron, calcium and proton pump inhibitors also reduce I-thyroxine absorption. CYP3A4 inducers like rifampin, phenytoin and carbamazepine

TH YROID HORMONES AN D THY RO ID INHIBITORS accelerate metabolism of T4 ; dose of I-thyroxine may need enhancement.

Uses The most importanr use of thyroid hormone is for replacement therapy in deficiency states. Synthetic I-thyroxine is the preparation of choice:

1. Cretinism

It is e ither due to fa ilure of thyroid development or a defect in hormone synthesis (sporadic creti nism) or due to extreme iodine deficiency (endemic cretinism). Cretinism is usua lly detected during infancy or childhood; but screening of neonates is the best preventive strategy. Treatment with thyroxine (8- 12 µg/kg) dai ly should be started as early as possible, because mental retardation that has already ensued is only partially reversible. Response is dramatic: physical growth and development are restored and further mental retardation is prevented.

2. Adult hypothyroidism (Myxoedema) This is one of the commonest endocrine disorders which develops as a consequence of autoimmune thyro iditis or thyro idectomy. It may accompany s imple goiter if iodine deficiency is severe. Antibodies against thyroid peroxidase or thyroglobul in are responsible for majority of cases of adult hypothyroidism. Im portant drugs that can cause hypothyroidism are 13 '1, iodides, lithium a nd amiodaronc. Treatment with T4 is most gratifying. Though in younger patients, full replacement doses may be started from the beginning, in those >50 years it is wise to start with a lower dose (50 µg/day), wh ile in the elderly or in those with heart disease, the initial dose should be 12.5- 25 µg/day. Doses may be increased every 2- 3 weeks to an optimum of I 00-150 µg/day (adjusted by clinical response as well as serum TSH and free T4 levels). Further dose adjustments are made at 4-6 week intervals needed for reaching steady-state. Individualization of proper dose is critical, aiming at normalization of serum TSH levels. Inc rease in dose is mostly needed during pregnancy.

Subclinical hypothyroidism characterized by euthyroid status and normal free serum thyroxine ( FT4 ) level (~ 9 pmol/L) but raised TSH level (> 10 mU/L) shou ld be treated with T 4 • For TSH level between 6- 10 mU/L, replacement therapy is optional. Replacement is preferable if patient has other cardiovascular risk facto rs .

3. Myxoedema coma It is an em ergency; characterized by progressive mental deterioration due to acute hypothyroidism: carries s ignificant morta lity. Rapid thyroid rep lacement is crucial. Though liothyronine (T3) acts faster, its use is attended by higher risk of cardiac arrhythmias, angina, etc. Drug of choice is I-thyroxi ne (TJ 200-500 µg i.v. followed by 100 µg i.v. OD till oral therapy can be instituted. Some authorities recommend adding low dose i. v. T3 IO µg 8 hourly in younger patients wi th no arrhythmia or ischaemia. Alternatively oral T 4 500 µg loading dose followed by I 00-300 µg dai ly may be used, but in severe h ypothyroidism, ora l absorption is delayed and inconsistent. Other essential measures needed are-warming the patient, i.v. corticosteroids to cover attendant adrena l ins ufficiency, venti la tory and cardiovascu lar support, correction of hyponatrae mia and hypoglycaemia.

4 . Nontoxic goiter This is characteri zed by enlargement of thyroid with euthyroid status. ft may be endem ic or sporadic. Endemic is due to iodine deficiency which may be accentuated by factors present in water (excess calci um), food or mil k (goitrin , thiocyanates). A defect in hormone synthesis may be responsible for sporadic cases. ln both types, deficient production of thyroid hormone leads to excess TSH • thyroid enlarges, more efficient trapping of iodide occurs and probably greater proportion o f T3 is synthesized • enough hormone to meet peripheral demands is produced so that the patient is clinically euthyroid. Thus, treatment with T 4 is in fact replacement therapy in this condition as we ll, despite no overt hypothyroidism. Full maintenance doses must be gi ven. Most cases of recent diffuse enlargement of thyroid regress. Long-standing goiter wi th degenerative

273

274

HORMONES AND RE LATED DRUGS and fi brotic changes and nodular goiter regress little or not at all. Thyroxine therapy may be w ithdrawn after a year or so in some cases if adequate iodine intake is ensured. Others need life-long therapy.

THYROID INHIBITORS These are drugs used to lower the func tional capacity of the hyperactive thyroid gland.

7. Empirical uses T, has been sometimes used in the

Thyrotoxicosis is due to excessive secretion of thyroid hormones. The two main causes are Graves' disease and toxic nodular goiter. Graves' disease is an a utoimmune disorder due to production of an IgG class of antibody called 'thyroid stimulating immunoglobu li n.' It binds to the TSH receptor and causes long lasting stimulation of the thyroid cells. The exophthalmos of Graves' disease appears to be due to stimulation of TSH receptors on the periorbital tissues. Due to feed-back inh ibition, TSH levels are low. Toxic nodular goiter, which produces thyroid hormone independent of TSH, is less common and mostly supervenes on old nontoxic goiters. It is more common in the elderly; ocular changes are generally absent. The th ioamide a ntithyroid drugs and ionic in hibitors are also called goitrogens because, if given in excess, they cause enlargement of thyroid by feedback release of TSH.

followi ng conditions without any rationale; response is unpredictable. Refractory anaemias. Mental depression. Menstrnal disorders, infenility not corrected by usual treatment. Chronic/ non-healing ulcers. Obstinate constipation. Thyroxine is not to be used for obesity and as a hypocholesterolemic agent.

In addition, certain drugs used in high doses for prolonged periods cause hypothyroidism/goiter as a side effect: • Lithium: inhibits thyroid hormone release. • Amiodarone: inhibits peripheral conversion of T, 10 T ,; also interferes with thyroid hormone action. • Sulfonamides, paraaminosalieylic acid: inhibit thyroglobulin iodination and coupling reaction. • Phenobarbitone, phenytoin, carbamazepine, rifampin: induce metabolic degradation of T/f3•

Endemic goiter and cretinism due to iodine deficiency in pregnant mother is preventabl e by ens uring daily ingestion of 150-----200 µg of iodine. This is best achieved by iodi7ing edible salt by adding iodate (preferred over iodide). In India iodization of table salt ( 100 µg iodine/g salt) is required under the ational Programme, but recently mandatory iodization rnle has been withdrawn.

5 . Thyroid nodule Certain benign functioning nodu les regress when TSH is suppressed by T 4 therapy. onfunctional nodules and those non-responsive to TS H (that are associated with low TSH levels) do not respond and should not be treated with levothyroxine. T 4 therapy should be stopped if the nodule does not decrease in size within 6 months, as well as when it stops regressing after the ini tial response.

6. Papillary carcinoma of thyroid T h is type of cancer is often responsive to TSH. In nonresectable cases, full doses of T4 suppress endogenous T SH production and may induce temporary regression.

THYROID INHIBITORS

Inhibit hormone synthesis

Inhibit Iodide trapping

{Thioa mides) P ropylthiouraci l Methimazole Carbirnazole

(Ionic inhibitors) Thlocyanates (-SCN) Perchlorates (-CI04) Nitra tes (-NO 3)

I

Inhibit hormone

release

lodinc Iodides of Na, K Organic iodide 1

Destroy thyroid tissue

Radioactive iodine 1311

THYROID HORMONES AND THYROID INHIBITORS G oitrin- found in plants (ca bbage, turnip, mustard. etc.), is the cause of goite r in can le " ho feed on these plants. Goitnng may contribute to endemic goiter in certain iodine deficient regions.

THIOAMIDES (Antithyroid drugs) By convention. only the thioamide hormone synthesis inhibitors are called ' antithyroid drugs' , though this term has also been applied to all thyroid inhibitors. Thiourca de ri vatives were found to produce goiter and hypothyroid is m in rats in the I 940s. Ope n chain thio urea compounds we re found to be toxic. Subsequently, methyl and propyl thiouracil and thioimidazole derivati, es methima,:ole and carbimazole " e re found to be safe and effecti, e.

Thioamidcs bind to the thyroid peroxidase and prevent oxidation of iodide and iodotyrosyl res idues, thereby; (i) Inhibit iod ination of tyrosine residues in thyroglobu lin (ii) Inh ibit coupling ofiodotyrosine residues to form T 3 and T 4 . Action (ii) has been observed al lower concentration of antithyroid drugs than action (i). Thyroid colloid is depleted over time and blood levels of T/ f 4 are progressively lowered. Thioamides do not interfere w ith trapping of iodide and do not modify the action of T3 and T4 on peripheral ti ssues or on pituitary. Goiter th at occurs with hi gher doses is not the result of potentiation of TSH action on thyroid. but is d ue to increased TSH release as a conseq uence of reduction in feed back inhibition. No goiter occurs if antithyro id drugs are

give n to hypophysectomised anima ls or if T 4 is given along w ith them. Antithyroid drugs do not affect release of T 3 a nd T 4- thei r efTects are not apparent till thyroid is depleted of its hormone content. Propylthiouracil also inhibits peripheral conversio n of T4 to T 1 by DI type of 5-Dl, but not by D2 type. Th is may partly contribute to its antithyroid effects . Methimazole and carbimazo lc do not have thi s action (Table 18.1 ).

Pharmacokinetics The antithyroid drugs are quickly absorbed o rally, wide ly distributed in the body, enter milk and cross placenta; are metabolized in li ver and excreted in urine primarily as metabolites. A ll are concentrated in thyroid ; the intrathyroid t½ is longer: effect of a single dose lasts longer than would be expected from the plasma t ½. Carbimazole acts largely by getting converted to methimazole in the body and is longer acting than propythiouracil. Adverse effects Hypo thyroidism and goiter can occu r due to overtreatment, but this is reversible on stopping the drug. Enlargement of thyroid is an indicatio n of hypothyroidism and is due to excess feedback TS H production . Goiter does 1101 develop with appropriate doses which restore T 4 concentration to normal so that feed back TS H inhibition is maintained. Important side effects are: g. i. intolerance, skin rashes and joint pain. Liver damage can occur, especia lly with propylthiouracil. Loss or graying of hair, loss of taste and fever are infrequent.

Table 18.1 : Differences between propylth1ourac11 and carbimazole Propylthiouracil

Carbimazole

1. Dose to dose less potent

About 5 x more potent

2. Highly plasma protein bound

Less bound

3. Less transferred across placenta and in milk

Larger amounts cross to foetus and in milk

4. Plasma t½ 1-2 hours

6-10 hours

5. Single dose acts for 4-8 hours

12-24 hours

6. No active metabolite

Produces active metabolite-methimazole

7. Multiple (2-3) daily doses needed

Mostly single daily dose

8. Inhibits peripheral conversion of T, to T3

Does not inhibit T 4 to T3 conversion

275

276

HORMONES AND RELATED DRUGS A rare but se riou s adve rse effect is agranulocytosis ( I in 500 to I 000 cases); It is mostly reversible. There is partial cross reactivity between propylthiouracil and carbimazole. Preparations and dose

Propyllhio11racil: 50- 150 mg TDS follo"ed by 25-50 mg BD- TDS for maintenance. PTU 50 mg tab. Me1himazole: 5- 10 mg TDS initia lly, maintenance dose 5- 15 mg daily in 1- 2 doses. Carhimazole: 5- 15 mg TDS init ially, maintenance dose 2.5-10 mg daily in 1- 2 divided doses, Nl::O McRCAZOLE, THYROZOLE, A'\JTITHYROX 5, 10, 20 mg tab.

Carbimazole is used in lndia while methimazole is not marke ted. Since popylthi ouracil has shorter duration of action and carries risk of severe hepatitis, it should be reserved for use in early pregnancy (due to less placental transfer than carbimazole) and in thyroid stonn for its inhibitory action on peripheral conversion of T4 to more active Tr It may be tried in patients developing adverse effects with carbimazole.

Use

Antithyroid drugs control thyrotoxicosis in both Graves' di sease and toxic no dular goiter. Cli nical improveme nt starts after 1- 2 weeks o r more (depending on hormone content of thyroid gland). Iodide loaded patients (who have received iodide containing conn-asl med ia/cough mixtures, a miodarone) are less responsive. Ma intenance doses are titrated on the basis of clinical status of the patient. The fol lowing strategies are adopted.

(i) As definitive therapy (a) Remission may occur in upto half of the patients of Graves' disease after 1- 2 years of treatment: the drug can then be withdrawn. If symptoms recurtreatment is reinstituted. This is preferred in young patients with a short history of Graves' disease and a small goiter. (b) Remissions are rare in toxic nodular goiter; surgery (or 131 1) is preferred. However, in frail elderly patients with multinodular goiter who may be less responsive to 1311, permanent maintenance therapy with antithyro id drugs can be employed. (ii ) Preoperatively Surgery in th yrotoxic patients is risky. Young patien ts w ith florid

hyperthyroidism a nd substant ia l goi te r are rende red eu th yro id wit h carbimazole before performing su btotal thyroidectomy. (iii) Along with 13 1I Initia l control with antithyroid drug- I to 2 weeks gap-radioiodine dosing- resume antithyroid drug after 5- 7 days and gradually withdraw over 3 months as the response to 131 I deve lops. This a pproach is preferred in o lder patients and in those with heart disease who are to be treated wi th 13 1 I. but require prompt control of severe hyperthyroidism. This will a lso prevent in itial disease flare-up following 13 1 I due to release of stored T4 • Ad vantages of antithyroid drugs over surgery and 131 1 are: (a) o surgical risk , scar or chances of injury to parathyroid glands o r recurrent laryngeal nerve. (b) Hypothyroidism , if induced , is reversible. ( c) Can be used even in children and young adults. Disadvantages are: (a) Prolonged (often life- long) treatment is needed because rel apse rate is high. (b) Not practicable in uncooperative/unintelligent patient. (c) Drug toxicity. Thyroidectomy and 131 1 are contra indicated during pregnancy. With anti thyroid drugs risk of foetal hypothyroidism and goiter is there. Howeve r. low doses of propylthiouracil may be used : its greater protein binding a llows less transfer to the foetus. However, methimazole has also now been fo und safe during pregnancy. Propylthiouracil is used in thyroid storm as well (see p. 279). IONIC INHIBITORS Certain mono\'alent anions inhibit iodide trapping by NIS into the thyroid. Consequently, T/ T , cannot be synthesized. Perchlorate is IO times more potent lhan thiocyana1e in blocking IS. while nitrate is very weak. T hese ions are tox ic and no! clinically used .

IODINE AND IODIDES Though iodine is a constituent of thyroid hormones, it is the fastest acting thyroid inhibitor. In G raves' disease the gland, if enlarged, shri nks,

THYROID HORM ON ES AND THY ROID INHI BITORS becomes firm and less vascular. The thyroid stanis starts returning to normal at a rate commensurate with complete stoppage of hormone release from the g land. The thyroid gland involutes and colloid is restored. The response to iodine and iodides is identical, because e lemental iodine is reduced to iodide in the intestines. With daily administration, peak efTects are seen in 10- 15 days, after which 'thyroid escape' occu rs and thyrotoxicosis may return with f,'Teater vengeance. Worsening of hyperthyroidism occurs, especia lly in multinodular goiter. All facets o f thyroid function seem to be affec ted, but th e most impo rt a nt action is inhibition ofhom1one release- ' thyroid constipation' . Endocytosis of colloid and proteolysis of thyroglobulin comes to a ha lt. T he mechanism of action is not clear. Excess iodide inhibits its own transport into thyroid cells by interfering with expression of IS on the cell membrane. ln addition, it attenuates TSH and cAM P induced thyroid stimulation. Excess iodide rapidly and briefly interferes with iodination of tyrosil and thyronil residues of thyroglobulin (probab ly by a ltering redox potential of thyroid cells) resulting in reduced T/ f 4 synthesis (Wolff-Chaikoff effect). However, within a few days, the gland ' escapes' from thi s effect and hormone synthesis resumes. Preparations and dose

Lugol 's solution (5% iodine in 10% Pot. iodide solution): LUGOL'S SOLUTION, COLLOID IODINE 10%· 5 10 drops/dny. COLLOSOL 8 mg iodmc/5 ml liq. Iodide (Sod.I Pot.) 100-300 mg/day before thyroidecto my, 5- 10 mg/day (prophylactic) for endemic goiter.

2. Thyroid storm Lugol 's iodine (6- 10 drops) or iod ine containing radiocontrast media (iopanoic acid/ipodate) orally are used to stop any further release of T/ T4 from the thyroid and to decrease T4 to T 3 conversion.

3 . Prophylaxis of endemic goiter

It is genera lly used as " iodized salt"(see p. 274).

4 . Antiseptic As tincture iodine, povidone iodine, etc. (see Ch. 67).

Adverse effects l. Acute reaction It occurs o nly in rare individuals sensi tive lo iodine, and can be triggered even b y a minute quantity. Manifestat ions are swelling of lips , eyelids. angioedema o f larynx ( may be dangerous), fever, j oint pain, pe1ech1al haemorrhages, thro mbocytopenia, lymphadcnopathy. Funher exposure to iodine should be stopped immediate ly.

2. Chronic overdose (iodism) Inflammatio n of mucous membranes. sali vation, rhinorrhoea, sneezing. lacrimat ion, swe lling of eyelids, burning sensation in mouth. headache, rashes, g.i. symptoms, etc. The symptoms regress on stopping iodide ingestion. Long-term use of hi gh doses ca n caus e hypothyroidism and goiter. Iodide may cause flaring of acne in adolescents. Given to pregnant or nurs ing mothers, it may be responsible for foetal/infantile goiter and hypothyroidi sm. Thyrotoxicosis may be aggravated in multinodular goiter.

RADIOACTIVE IODINE

1. Preoperative preparation for thyroidectomy

The stable isotope of iodine is 127 1. lts radioactive isotope of medicinal importa nce is: 131 I: physical half-life 8 days. The c he mical behaviour of 13 11 is si milar to

in Graves' disease: lodi ne is genera lly given for IO days just preceding surgery. The aim is to make the g land firm, less vascular and easier to operate on. Though iodide itself will lower the thyroid status, it cannot be relied upon to attai n euthyro idism w hich is done by use of carbimazole before sta1iing iodide. Propranolol may be given additionally for rapid control or symptoms.

that of the stable isotope. 111 1 emits y-rays (higher energy X-rays emitted by atomic nuclei), as well as p particles (electrons). The former are useful i11 tracer studies, because they traverse the tissues and can be monitored by a counter, whi le the latter are uti lized for their destructive efTect on thyroid cells. 131 1 is conce ntrated by thyroid and incorporated in the colloid. Thus, it emits radiati on from within the

Uses

277

278

HORMONES AND RELATED DRUGS follicles. The particles penetrate only 0.5- 2 mm of tissue. The thyroid foll icular cell s are affected from within, undergo pyknosis a nd necros is followed by fibros is when a sufficien tly la rge dose has been administered , wit ho ut damage to neighbouring ti ssues. With carefully selected doses, it is possible to achieve partial ablation of thy roid. Radioacti ve iodine is administered as sodium salt of 13 1I di ssol ved in water and taken orally.

Diagnostic 25- 100 µ c urie is g iven; cou nting or scannin g is done at intervals. o damage to thyroid cells occurs at thi s dose. Therapeutic The most common indication is hyperthyroidism due 10 Graves' disease or toxic nodular goiter. The average therapeutic dose is

3- 6 m c urie-calculated on the basis of previous tracer studies and thyroid size. Hi gher doses are genera lly required fo r toxic multinodular goiter than for Graves· disease. T he clinical response is slow- starts after 2 weeks and gradua lly increases, reaching peak at 3 months o r so. Thyroid status is evaluated after 3 months, and a repea t dose, if needed, is given. About 20-40% patients requ ire one or more repeat doses.

Advantages I. Treatment with 13 11 is simple, conveniently given on outpatient basis and inexpensive. 2. o surgical risk, scar or injury to parathyroid glands/ recurrent laryngea l nerves. 3. Once hyperthyroidism is controlled, cure is perma nent.

Disadvantages I. H ypothyroi di sm: About 5- 10% pati en ts of Graves' disease treated with n, , become hypothyroid every yea r (upto 50% or more patients may ultimately require supplementa l thyroxi ne treatment). This probably re flects the natural history of Graves' disease, because only few patients of toxic nodular goiter treated with 13 11 develop hypothyroidism. Moreover, evenn1al hypothyroidism is

a complica tion o f s ubtotal thyroidectomy/ prolonged carbimazole therapy as we ll. 2. Long latent period of response. 3. Contraindicated during pregnancy- foe tal thyroid will a lso be destroyed resulting in cretinism, other abnormalities if given during first trimester. 4. Not suitable for young patients: th ey are more likely to develop hypothyroidism la te r and would then requi re life-long T 4 treatment. 131 l is the treatment of choice after 25 years of age and if C HF, ang ina or any othe r contraindication to surgery is present.

Metastatic carcinoma of thyroid (especially papillary or those cases of follicular carcinoma w hich concentrate iodine), 13 11 may be used as pall iative therapy after thyroidectomy. Much hi gher doses a re required and prio r stimulation with TSH is recommended.

p ADRENERGIC

BLOCKERS

Propranolol (and other nonselective P blockers) have emerged as an important fonn of therapy to rapidly alleviate mani festations of thyrotoxicosis that are due to sympathetic overactivity. viz. palpitation. tremor, nervousness, seve re myopathy, sweating. They ha ve little e ffect on thyroid function and the hypermetabolic state. They are used in hyperthyroidism in the following situations. (a) Whil e awaiting response to carbimazole or

IJ1! . (b) Along with iodide for preoperative preparation before subtotal thyroidectomy. (c) Thyroid storm (thyrotoxic crisis): This is an emergency due to decompensated hyperthyroidism. Vigorous treatment with the fo ll owi ng is indicated: o nse lec ti ve P blockers (e.g. proprano lol) are the most va luable measu re. They afford dramatic sy mptomati c re lie f. I n addit ion , h igher doses reduce periphera l co nve rsion of T 4 to T r Propranolol 1- 2 mg s low i. v. may be fo llowed by 40- 80 mg oral every

THYROID HORMONES AND THYRO ID INHIBITORS 6 hours. It may be withdrawn gradually when T 4 levels normalise. Propylthiouracil 200-300 mg oral 6 hourly: reduces hormone synthesis as well as peripheral T 4 to T 3 conversion. lopanoic acid (0.5- l g OD oral one hour after propylthiouracil) or ipodate are iodine con taining radiocon trast media. T hey are potent inhibitors of thyroid hormone re lease from thyroid, as well as of peripheral T 4 to T 3 conversion.

(j/"

• Corticosteroids (hydrocortisone I 00 mg i. v. 8 hourl y followed by oral prednisol one) : help to tide over crisis, cover any adrenal insufficiency and reduce conversion of T 4 to T 3 in periphery. • Di ltiazem 60-120 mg BO oral may be added if tachycardia is not controlled by propranolol alone, or w hen it is contraindicated. •

Rehydration, anxiolytics. external cool ing and appropriate antibiotics are the other measures.

PROBLEM DIRECTED STUDY

18.1 A 20-year girl was diagnosed as a case of recent onset Graves' disease with mild diffuse pulsatile thyroid enlargement. She was treated with tab. Carbimazole 5 mg 2 tab 3 times a day. Her symptoms started subsiding after 2 weeks and were fully controlled after 3 months. Th e thyroid swelling also subsided and she was maintained on a dose of carbimazole 5 mg twice daily. After one year she noticed that the neck swelling was reappearing and her body weight increased by 2 kg in the last one month, but without recurrence of her earlier symptoms. She rather felt dull, sleepy and depressed. Th e serum TSH was 12 µU/ ml and free thyroxine {FT4 ) was 9 pmol/ L. (a) Why was the initial response to carbimazole delayed? Could any additional medicine be given to her initially to afford more rapid symptomatic relief? (bl What wa s the cause of reappearance of th e neck swelling and her condition after 1 yea r? What measures need to be taken at this stage? (see Appendix-1 for solution)

279

Chapter

Insulin, Oral Antidiabetic Drugs and Glucagon

19

Diabetes mellitus (DM)

It is a metabolic di sorder characterized by hyperglycaemia, (fasting plasma glucose 126 mg/ dL and/ or 200 mg/dL 2 hours after 75 g oral glucose), glycosuria, hyperlipidaem ia, negati ve nitrogen balance and sometimes ketonaemia. A widespread pathological change is th ickening of capillary basement membrane, increase in vessel wal l matrix a nd cellular proliferation resulting in vascular complications li ke lumen narrowing, early atherosclerosis, sclerosis of gl omerular capillaries, retinopathy, neuropathy and peripheral vascular insuffic iency. Enhanced nonenzymatic glycosylation of tissue proteins due 10 persistent exposure to high glucose concentrations and the accumulation o f larger quantities of sorbitol (a reduced product of g lucose) in tissues are be lieved to be causative in th e pathological changes of diabetes. The concentration of g lycosylated haemoglobin ( HbA,J is taken

as an index of protein glycosyla1ion: it reflects the state o f glycaemia o,er the preceding 2- 3 months.

Two major types of diabetes mellitus are:

Type I Insulin-dependent diabetes mellitus (lDDM)/juvenile onset diabetes mellitus: There is cell destruction in pancreatic islets; majority of cases are auto immune ( type I A) antibod ies that destroy cells are detectable in blood, but some are idiopathic (type I 8 ) -no cell antibody is found. In all type I cases circulating insulin levels are low or very low, and patients are more prone to ketos is. This type is less common and has a low degree of genetic predisposition. Type II Noni nsulin -dependen t diabetes mell itus (NlDDM)/maturity onset diabetes mellitus: There is no loss or only moderate reduction in cell mass; insulin in ci rc ulation is low,

Approaches to drug therapy in type 2 DM Improve insulin availability

Overcome insulin resistance

Exogenous insulin

Biguanides (Metformin)

Sulfonylureas

Thiazolidinediones (Pioglitazone)

Meglitinide/phenylalanine analogues

a. glucosidase inhibitors

Dipeptidyl peptidase-4 inhibitors (DPP-41s) GLP-1 receptor agonlsts

MaJOr limitations (except for DPP-4/s and GLP1 agonists)

Ma1or limitations

Multiple daily injections (Insulins) Hypoglycaemic episodes Weight gain Concern about premature atherosclerosis due to hyperinsulinaemia

Inability to achieve normoglycaemia by themselves in many patients, especially moderate-to-severe cases

281

INSULIN, ORAL ANTI DIABETIC DRUGS AND GLUCAGON normal or even high, no anti-P-cell antibody is demonstrable; has a high degree of genetic predisposition; genera lly has a late onset (past middle age). Over 90% cases of diabetes are type 2 OM. Causes may be: • Abnormality in gluco-receptor of p cells so that they respond a t hi g her glucose concentration or relative p cell deficiency. In either way, insulin secretion is impaired; may progress to p cell failure. • Reduced sensitivity (relative resistance); of peripheral tissues to insulin : reduction in number of insulin receptors, i.e. 'down regulation' of insulin receptors. Many hypertensives are hyperinsul inaemic, but normoglycaemic; and have associated dys lipidaemia, hyperuricaemia, abdominal obesity (metabolic syndrome). Thus, there is relative insulin resistance, particularly at the level of liver, muscle and fat. Hyperinsu linaemia per se has been implicated in causing angiopathy. • Excess of hyperglycaemic hormones (g lucagon, etc.)/obesity: cause relative insulin deficiency- the p cells lag behind. In most cases, both relative insulin resistance and p cell insuffic iency are produced by a combination of genetic and environmental factors. Other rare forms o r DM arc those due to specific genetic defects (type-3) like 'maturity onset diabetes of young' (MODY), other endocrine disorders. pancrcatectomy and ·gestational diabetes mellitus· (GDM, typc-4).

INSULIN Insulin was di scovered in 1921 by Banting and Best who demonstrated the hypoglycaemic action o f an extract of pancreas prepared after degeneration of the exocrine part due to ligation of pancreatic duct. It was first obtained in pure crystalline form in 1926 and the chem ical structure was fully worked ou t in 1956 by Sanger. Insulin is a two chain polypeptide having 51 a mino ac ids and MW about 6000. The A-chain has 21 while B-c hain has 30 amino acids. There are minor differences between human, pork and beef ins ulins: Species

A-chain

B-chain

8th AA

10th AA

30th AA

THR THR

Beef

ALA

ILEU ILEU VAL

THR

Pork

Human

ALA ALA

Thus, pork ins ulin is more homologous to human insulin than is beef insul in. The A and B chains are he ld together by two di sulfide bo nds. In su lin is synthes ized in the P cel ls of panc reatic islets as a single chain peptide Preproinsulin ( 110 AA) from which 24 AAs are first removed to produce Proinsulin (Fig. 19.1 ). The connecting or 'C' peptide (35 AA) is spl it ofT by proteolysis in Golgi apparatus; both insulin and C peptide are stored in granules within the cell. The C peptide is secreted in the blood along with insulin. Assay Insulin was bioassayt..-d either by measuring blood sugar depression in rabbits or by its potency 10 induce hypoglycaemic convulsions in mic.:. With the availability of pure preparations, insulin is now assayed chemically and the quantity expressed by weight ( I mg of International standard of insulin = 28 Units). Plasma insulin can be measured by radioimmunoassay or cn7yme immunoassay.

Regulation of insulin secretion

Fig. 19.1: Human proinsulin; simplified structure depicting the main features. The connecting peptide, having 35 amino acids, is split off leaving insulin molecule with two chains joined by two disulfide bridges

Under basal condition - I U insu lin is secreted per hour by human pa ncreas. Much larger quantity is secreted after every meal. Secretion of insulin from p cells is regulated by chemical, hon11onal and neural mechanisms.

Chemical The p cells have a glucose sen ing mechanism depende nt on entry of glucose into

282

HORMONES AND RELATED DRUGS the P cells {through the aegis of a g lucose transporter GLUT I) and its phosphorylation by glucokinase. Glucose entry and metabolism leads to activation of the glucosensor and production of ATP wh ich inhibits the ATP-sensitive Kchannel ( K ' ATP) resulting in pa rt ial depolarization o f the p cells (see Fig. I 9.6). This inc reases intracellular Ca2- availabi lity (due to increased influx, decreased efflux and re lease from intracellular stores) • exocytotic release of insulin storing granules. Other nutrients that can evoke insulin release are- am ino acids, fatty ac ids and ketone bodies, but glucose is the principal regulator and it sti mulates the synthes is of insulin as well. Glucose induces a brief pu lse of insulin output with in 2 min (first phase) followed by a delayed but more sustained second phase of insulin re lease. Glucose and other nutrie nts are 2-4 times more effective in invoki ng insulin release when given orally than i. v. They generate chemical signa ls 'incretins' from the gut which act on p cells in the pancreas to cause ant icipatory release of insulin. The incretins involved are glucagon-like pe ptide- I {GLP-1 ), glucose-dependent insulinotropic polypeptide (G IP), vasoactive intestinal peptide (VfP), pancreozymin-cholecystokinin, etc.; but various incretins may mediate signal from di !Terent nutri ents. Glucagon and some of these peptides enhance insulin release by increasing cAM P fonnation in the p cells.

Hormonal A n umber of hormon es, e.g. g rowth horm o ne, co rticosteroids, thyrox ine modify insulin release in response to glucose. More impo rtant are the intra- is let paracrine interactions between the hormones produced by different types of is let cells. The P ce lls constitute the co re of the islets and are the most abundant cell type. The a cells, comprising 25% of the islet cell mass, surro und the core and secre te glucagon. The 8 cell s (5- 10%) e laborating somatostatin are interspersed between the a cells. T here are some PP (pancreatic po lypeptide containing) cells as wel l. • Somatostatin inhibits release of both insulin and glucagon.

--+

S!Jmulat1on

- -+ lnh1b,tion

+-0 - - - - - - - - - Fig. 19.2: Paracrine modulation of hormone secretion within the pancreatic islets of Langerhans

• G lucagon evokes release of insulin as well as somatostatin. • Insulin inhibits glucagon sec retion. Amylin, another P cell polype ptide released with insulin, inh ibits glucagon sec retion through a central site of action in the brain. The three hormones released from closely s ituated cells influence each other 's secretion and appear to provide fine tuning of their o utput in response to metabolic needs (Fig. 19.2). Neural T he is le ts are richly suppli e d by sympathetic and vagal nerves. • Adrenergic a 2 receptor activation decreases insulin release (predominant) by inh ibiting p cell adenyly l cyclase. • Ad renergic p2 stimulation increases ins ulin release ( less promi nent) by stimulating p cell adenylyl cyclase. • Cholinergic-muscarinic activation by ACh o r vagal stimulation causes insulin secretion thro ugh IP/ DAG-increased intracellular Ca 2' in the P cells. I lowever, neural in fl uences have only modulatory effect on insulin secretion.

ACTIONS OF INSULIN The overall effects of insulin are to dispose meal deri ved gl ucose, am ino acids, fatty acids and favour storage of fuel. It is a major anabol ic hormone: promotes synthesis of gylcogen, lipids and protein. The actions of insu li n and the results of its deficiency can be summarized as:

INSU LI N, ORAL AN TI DI ABETIC DRUGS AND GLUCAGON I . Insulin facilitates glucose transport across cell membrane: skeletal muscle and fat are

hi ghly sensiti ve. The ava ilability of g lucose intrace llularl y is the lim iting factor for its utili zation. However, g lucose e ntry in liver, brain, RBC, WBC and renal medullary cells is largely independent of insuli n. Ketoacidosis interferes with glucose utilization by brain and contributes to diabetic coma. Muscular acti vity induces glucose entry in muscle cells without the need fo r in sulin . As su ch, exerc ise has insulin sparing e ffect. The intracellu lar poo l of ves icles conta ining gl ucose transporter glycoproteins GLUT4 (insulin activated) a nd GLUT ! is in dynamic equilibrium with the GLUT vesicles inserted into the membrane. This equilibrium is regulated by insulin to favour translocation to the membrane. Moreover, on a long-term basis, synthesis of GLUT4 is upregulated by insulin. 2. The first step in intracellular utilization of glucose is its phosphorylation to fom, glucose6-phosphate. T his is enhanced by insulin through increased produclion of glucokinase. Insulin facili tates g lycogen synthesis from glucose in liver, muscle and fat by stimulating the e nzyme g lycogen synthase. It a lso inh ibits glycogen degrading enzyme phosphorylase • decreased glycogenolysis in liver. 3. Insulin inhibils gluconeogenesis (from prote in, FFA and g lycerol) in li ver by gene mediated dec reased synthesis of phosphoenol pyruvate carboxykinase. In insulin deficiency, proteins and amino acids are funneled from peripheral ti ssues

to liver w here these substances are converted to carbohydrate and urea. Thus, in diabetes there is underutil ization and over production of glucose leading to hyperglycaemia and glycosuria. 4. Insulin inhibits lipolysis in adi pose tiss ue and fa vours triglycerid e synthesis. Jn diabetes increased amount of fat is broken down due to unchecked action of lipolytic hormones (glucagon, Adr, thyroxi ne, etc.) • increased FFA and g lycerol in blood • taken up by liver to prod uce acetyl-CoA. Normally acety l-CoA is resynthesized to fatty acids and trig lycerides, but th is process is red uced in diabetics and acetyl CoA is d iverted to produce ketone bod ies (acetone, acetoacetate, P-hydroxy-butyrate). T he ketone bodies are released in blood and partly used up by muscle and heart as energy source, but when their capacity is exceeded, ketonae mia and ketonuria result. 5 . Insu lin e nhances transcripti on of vascu la r endothelial lipoprotein lipase, thereby accelerating clearance of VL DL a nd chylomicrons. 6. Insulin facilitates AA entry into muscles and most other eel Is. Their synthesis into proteins is en hanced, and protein breakdown is inh ibited. Insulin defic ie ncy leads to protein breakdown • AAs are released in blood • taken up by li ver and converted to pyruvate, gl ucose and urea. The excess urea produced is excreted in urin e resulting in negati ve nitrogen balance. Thus, catabo l ism takes the uppe r hand over anabolism in the d iabetic state. Most of the above metabolic actions of insulin are exerted within seconds or minutes and are

Actions of insulin producing hypoglycaemia Uver

Muscle

Adipose tissue

• Increases glucose uptake and glycogen synthesis

• Increases glucose uptake and utilization

..,. Increases glucose uptake and storage as fat and glycogen

• Inhibits glycogenolysis and glucose output ..,. Inhibits gluconeogenesis from protein, pyruvate, FFA and glycerol

..,. Inhibits proteolysis and release of amino acids, pyruvate, lactate into blood which form the substrate for gluconeogenesis in liver

..,. Inhibits lipolysis and release of FFA + glycerol which form substrate for gluconeogenesis in liver

283

284

HORMO NES AND RELATED DRUGS

Insulin receptor Glucose

I

~--GNE proteins

GTP

~--.....

GDP

GLUT4 -

t

~- ~rt'

~ -

GLUT4

::?,:'

,§ 100 time more potent than tolbutamide. Clinically useful biguanide phe11formi11 was produced parallel to sul fonylurcas in 1957. Newer approaches have consiamly been explored and have successively yielded rhiazolidinediones, meglirinide analogues, a -glucosidase inhibilors, dipeptidyl pepridase-4 (DPP-4) inhibirors and sod-glucose co1ra11spor1 (SG LT-2) inhibitors, etc.

293

294

HORMONES AND RELATED DRUGS ORAL ANTIDIABETIC DRUGS

Enhance Insulin secretion

lAMP. activator)

Thiazolidinecf,one (PPARy activetor)

a-Glucosidase inhibitors

Metformin

Pioglitazonc

Acarbose Miglitol Voglibose

Biguanide

I

I K,.w channel blockers

Tolbutamide Clibenclamide Glipizide Glidazide Glimepiride

Meglitinide/ Phenylalanine analoaues

Repaglinide Nateglin ide

-

Dlpeptidyl peptldase-4 (OPP-4) inhibitors

I

I Sulfonylureas

Miscellaneous drugs

Overcome insulin resistance

Sitagliptin Vildagliptin Saxagliptin Teneligliptin Alogliptin Linagliptin

Sod-glucose cotransport-2 (SGLT-2) Inhibitor

Dopamine 02 agonist

Bromocriptine

Dapagliflozin Canagliflozin

I

Sulfonylureas (KATP Channel blockers) The generic fo rmula of su lfony lu reas (SUs) isR1- 0 -so2-NH-CO-NH-

R2

SULFONYLUREA

All SUs have similar pharmacological profi le, their sole significant action being lowe ring of blood glucose level in normal subjects and in type 2 diabetics, but not in type I diabetics. Being more potent and c lin ically superior, only the second generation SUs are employed now. All first generation compounds have been discontinued except to lbutamide which is infrequently used.

Mechanism of action Sulfony lurcas bind to a specific 'sulfo nylurea receptor' (SUR I) located on the pancreatic p cell membrane and provoke a bri sk re lease o f insulin, the mechanism of which is deta iled in Fig. 19.6. The rate of insulin secretion at any glucose concentration is increased, i.e. insulin release is provoked even at low-glucose concentration risking production o f severe and unpredictable hypoglycaemia. In type 2 DM the kinetics of insulin release in

response to glucose or meals is delayed and subdued. T he SUs primarily augment the 2nd phase insulin secretion with little effect on the Ist phase. That they do not cause hypoglycaemi a in pancreatectomised anima ls and in type I diabetics (presence of at lea t 30% functional p cells is essentia l for their action), co nfirms their indirect action through pancreas. A minor action reducing g lucagon secretion. probably by increasing insulin and somatostatin release has been demonstrated. He patic degradation of insu lin is also slowed.

Extrapancreatic action After few months of adm ini stration, the insul inaemic action of SUs decli nes, probably due to down regulation o f sul fonylurea receptors (SUR I) on P cells, but improvement in glucose tolerance is maintained. ln thi s phase, they sensitize the target tissues (especially li ver) to the action of insulin. It is hypothes ized that long-term improvement in carbohydrate t o le ra nce leads to ove ra ll lowering of c ircula ting insuli n co ncentration which reverses the down regulation of insulin receptors. An apparent increase in their number

Table 19.2: Important features of oral antidiabet1cs Drug

Preparations

.. SULFONYLUREAS 1. Tolbutamide RASTINON, 0.5 g tab.

-

Duration of action (hr)

Clearance route·

Daily dose

No. of doses per day

6

6--8

L

0.5-3 g

2-3

2-4

24

L

2.5-15 mg

1-2

-

2. Glibenclamide DAONIL, EUGLUCON, (Glyburide) BETANASE 2.5, 5 mg tab.

3. Glipizide

Plasma t½ (hr)

GLYNASE, GLIDE, MINI DIAB 3-5 5 mg tab

4. Gliclazide

DIAMICRON 80 mg tab. 8--20 DIAZIDE 20, 80 mg tab GLIZID 30, 40, 80 mg tab 5. Glimepiride AMARYL, GLYPRIDE 5-7 GLIMER 1, 2 mg tab MEGLITINIDE I PHENYLALANINE ANALOGUES EUREPA, RAPLIN, 1. Repaglinide .$. 1 NOVONORM, REGAN 0.5, 1, 2 mg tab 2. Nateglinide GLINATE, NATELIDE 1.5 60,120 mg tab DPP-4 INHIBITORS JANUVIA 50, 100 mg tab. 1. Sitagliptin - 12

2. Vildagliptin BIGUANIDE 1. Mettormin

GALVUS, JALRA, ZOMELIS 50 mg cap

GLYCIPHAGE, GLYCOMET 0.5, 0.85 g tab, 0.5 g and 1.0 g SR tabs THIAZOLIDINEDIONE 1. Pioglitazone PIONORM, PIOREST, PIOZONE 15, 30 mg tab

2-4

12

12-24

24

3-5

L

L

L

L

5-20 mg 1-2

40-240 mg

1-2

1-6 mg

1-2

1-8 mg

3-4

Remarks

Weaker, shorter acting, flexible dosage, safer in elderly and in those prone to hypoglycaemia. Potent but slow acting, higher incidence of hypoglycaemia, single daily dose despite short t½ due to active metabolite and sequestration in pcells. Fast and shorter acting, higher daily dose to be devided, hypoglycaemia and weight gain less likely, preferable in elderly. Has antiplatelet action, generates only inactive metabolite, daily dose > 80 mg to be divided.

z

Vl

C

!::

z

0 ::0

:l> r

:l>

z

-I

0

Long acting, only inactive metabolite. Stronger extra-pancreatic action.

:l> OJ

m

-I

Given ½ hr before each meal for limiting post· prandial hyperglycaemia.

n 0 ::0

2-3

L

180-480 3-4 mg

C

Stimulates 1st phase insulin secretion, less likely to cause delayed hypoglycaemia.

G) Vl

:l> 24

12-24

K

L,K

100 mg

50-100 mg

1

1-2

z

Non-covalent binding to DPP-4; excreted unchanged in urine. Low risk of hypoglycaemia. Body weight neutral. Covalent binding to DPP-4; Metabolized in liver. Hepatotoxicity reported.

0

G)

r C

n

:l>

G)

1.5-3

6-8

K

0.5-2.5 g 1-2

No hypoglycaemia. Not metabolized. Lactic acidosis rare and only in kidney disease.

3-5

24

L

15-45 mg

May improve lipid profile. Reverses insulin resislance. No hypoglycaemia, C/1in liver and heart disease.

1

0

z

• L-Metabolized in liver; K-Excreted unchanged by kidney

I

""'

\0

V,

296

HORMO NES AND RELATE D DRUGS occurs, and/or a post receptor action improves translation of receptor activation.

Pharmacokinetics All SUs are well a bsorbed oral ly, and are 90% or more bound to plasma prote ins : ha ve low volumes of di stribution (0.2- 0.4 L/kg). They are primarily metabolizedmay produce active metabo lite. The metabolites (active/ inactive) are excreted in urine. As such , they should be used cautiously in patients with liver or kidney dysfunction. The distinctive features of differe nt SUs are given in Table 19.2. Interactions Drugs that enhance SU action (may precipitate hypoglycaemia) are(a) Displace from protein binding: Phenylbuta -zone, sulfinpyrazone, salicylates, sulfonamides. (b) Inhibit metabolism/excretion: Ci metidine ketoconazolc, sulfonamides, warfarin, c hl oramphenicol, acute a lcohol intake (a lso synergises by causing hypoglycaemia).

(c) Synergise with or prolong pharmacodynamic action: Salicylates, propranolol (cardioselective

p1

bloc kers a re less liable), sy mpathol ytic ant ihypertensivcs, lithium, theophy lline, alcohol (by inhibiting gl uco neogenesis).

Drugs that decrease SU action (vitiate diabetes control) are(a) Induce metabolism: Phenobarbitone, phenytoin , rifampicin, chronic alcoholism.

(b) Opposite actionlsuppres insulin release: Corticosteroids. th iazides, furosem ide, o ral contracepti ves. Adverse effects Incidence of adverse e ffects is quite low (3- 7%). I. Hypoglycaemia It is the commonest problem, may occasionally be severe a nd rarely fatal. It is more common in elderly, liver and kidney disease patients and when potentiating drugs a re added. Tolbutamide carries lowest risk due to its low potency and short duration of action.

Treatme nt of hypogl ycaemic episode is to give glucose. may be for a few days because hypoglycaemia may recur. 2. Nonspecific side effects Majority of diabeti cs started on SUs te nd lo gain 1- 3 kg weight. This may be a conseque nce of insulinaem ic action of SUs. ausea, vom1trng, flatulence, dia rrhoea or con tipation, headache and paresthcsias are generally mild and infrequent. 3. Hypersensitivity Rashes, photosensiti vity, purpura, transient leukopenia, rarely agranulocyto is. Flushing and a disulfiram-like reaction after alcohol is reported to occur in some individuals taking SUs. Tolbutam ide reduces iodide uptake by thyroid but hypothyroidism does not occur. Safety of SUs during preg na ncy is not established. C hange over to insul in is advised. Su lfony lureas are secreted in milk; they should not be given to nursing mothers.

Meglitinide/D-phenylalanine analogues ( K ATP Channel blockers) These are KATP chan nel blockers with a quick and short lasti ng insulinemic action.

Repaglinide This me glitin ide a nal og ue oral hypoglycaem ic is designe d to norma lise meal-ti me gl ucose excursio ns. Though not a sulfony lurea, it acts in an analogous manner by binding to SUR • closure of ATP sensitive K+ channels • depolarisat ion • insulin re lease (see Fig. 19.6). Repaglinide is quickly absorbed and rapidl y metabolized. It induces fast onset short-lasting insuli n release. Because of thi s characteristic its pallc rn of use is different fro m that of SUs . It is adminis1ered before each major meal to control postprandial hyperglycaemia; the dose should be omitted if a meal is missed. Because of sho rt lasting acti on it may have a lower ri sk of serious hypog lycaemia. Side effects are mild headac he, dyspeps ia, arthral g ia and weight ga in .

INSULIN, ORAL AN TI DIABETIC DRUG S AND GLUCAGON

Ca 2+

DPP-4

\ ¥-. ATPi ) Granule Glucose metabolism

Pancreatic

cell

Fig. 19.6: Mechanism of action of insulin secretagogues The sulfonylureas (SU) and meglitinide analogues (Megli) act on the sulfonylurea receptor (SUR1 ) which constitutes a subunit of the inwardly rectifying ATP-sensitive K· channel (KATI') in the membrane of pancreatic 13 cells. The outward flow of K• ions is thereby restricted, intracellular K• concentration rises and the membrane is partially depolarized augmenting Ca 2• channel opening as well as release of Ca2 • from intracellular stores. The Ca2 • ions promote fusion of insulin containing intracellular granules with the plasma membrane and exocytotic release of insulin. lncretins such as glucagon-like peptide 1 (GLP1) and glucose-dependent insulinotropic polypeptide (GIP) act upon their own G-protein coupled receptors on the 13 cell membrane to activate adenylyl cyclase and generate cAMP, which also promotes exocytosis of insulin. Exenatide (Exe) and liraglutide (Lira) are GLP1 receptor agonists-produce the same response as GLP1. The incretins GLP1 and GIP are rapidly inactivated by the capillary endothelial enzyme dipeptidyl peptidase-4 (DPP-4). Their action is enhanced by DPP-4 inhibitors sitagliptin (sita) and vildagliptin (vilda). The DPP-4 inhibitors thus markedly accentuate the insulin response to ingested glucose/meal and attenuate post-prandial glycaemia.

Repaglinide is indicate d on ly in selected type 2 diabetics who suffer pronounced postprandial hyperglycaem ia, or to supplement metformin/ long-acting ins ulin. It sho uld be avoided in liver disease.

Nateglinide

It is a D-phe nylalani ne derivative which principally stimulates the l st phase insulin secretion by closing~ cell KATPchannels res ult ing in fas ter onset and shorter lasting hypog lycaemia than repagl inide. Ingested I 0 min before meal, it limits postprandia l hyperglycae mia in type 2 diabetics without producing late phase hypoglycaemia. There is little effect on fasting blood glucose level. Episodes of hypoglycaemia are less freq uent than with SUs. S ide effects are dizziness, nausea, flu-like symptoms and joint pain. It is used in type 2

OM a long with other antidiabetics, to control postprandial ri se in blood glucose.

Dipeptidyl peptidase-4 (DPP-4) inhibitors Realizing the key role of the enzyme DPP-4 in rapid degradation of endogenous GLP- 1, orall y active inhibitors of thi s enzyme have been deve loped as indirectly acting insulin secretagogues. In the past few years, DPP-4 inhibitors have emerged as important adjun ctive drugs in type 2 DM , but their blood sugar lowering efficacy is moderate compared to that or SUs.

Sitagliptin

This is the first DPP-4 inhibitor introduced in 2006. It is a competiti ve and selecti ve DPP-4 inhibitor which pote ntiates the acti on of G LP- 1 (F ig . I 9.6) and GIP, boosts postprandia l insulin release, decreases glucagon

297

298

HORMONES AN D RELATED DRUGS

secretion and lowers meal-time as well as fasting blood glucose in type 2 diabetics. o effect on gastric emptying and appetite have been noted. It carries low ri sk of hypoglycaemia un less combined with SUs or insulin, because GLP- 1 evokes little insul in release al normal plasma gl ucose levels. The HbA,c lowering achieved by sitagliptin is almost equivalent to that with metformin. Further lowering of HbA ,c by 0.5- 1.2% occurs when it is added to pioglitazone/SUs/ insulin with or without metform in. However, sitagliptin monotherapy is recommended only when metformin cannot be used. Most professional guidelines recommend DPP-4 inhibitors primarily as adjuvanl drugs in type 2 diabetics not well controlled by metformin/SUs/pioglitazone or insulin. Though clinical efficacy of all DPP-4 inhibitors is comparable, one metaanalysis has found sitagliptin to cause greater reduction of fasting blood glucose than vildagliptin. Sitagliptin is well absorbed orally, is little metabolized and is largely excreted unchanged in urine with a t½ averaging 12 hours. Dose reduction is needed in renal impai rment, but not in liver disease. Sitagliptin is well tolerated, though nausea may occur, it is not prominent as in the case of GLP- 1 agonists. It also does not cause we ight loss and is body weight neutral. Side e ffects are nausea, loose stools, headache, rashes, allergic reactions including some serious ones like, angioedema, exfoliative dennatitis and edema. asopharyngitis and cough occurs in some patients. Other DPP-4 inhibitors also produce this side effect, which has been ascribed to prevention of substance P degradation. Pancreatitis is reported. Vildagliptin This DPP-4 inhibitor binds to the enzyme covalently. The complex dissociates very slowly resulting in persistent DPP-4 inhibition even a fter the free drug has been cleared from circulation. Th is explains the longer duration of action ( 12- 24 hours) despite short plasma t½ (2-4 hours). The major route of elimination is by he patic metabolism; only 20- 25% is excreted unchanged in urine. Dose reduction is needed in moderately severe liver and kidney

disease. o significant drug interactions have been reported. Vi ldagliptin is less selective than sitagliptin for DPP-4; causes some inhibition of DPP-8, DPP-9 as well, but the cli nical significance of this feature is not known. The tolerability of vildagliptin is similar to that of sitagliptin, but hepatotoxicity has been reported. Vi ldagliptin may require twice daily dosing; though single daily dose suffices in most cases when combined with another hypoglycaemic. Saxagliptin Like vildagliptin, saxaglipti n binds covalently with DPP-4 and acts for 24 hours despite a plasma t½ of 2-4 hours. Added to metformin/SU ± pioglitazone it causes HbA,c reduction similar to that caused by sitagliplin or vildagliptin. It is metabolized by CYP3A4 and generates an active metabolite that has a t½ of 3-7 hours. Drug interactions with CYP3A4 inhibitor are po sible. Dose: 5 mg OD; reduce by half in moderately severe renal failure, but not in liver disease. ONGLYZA 2.5, 5 mg tabs

Teneligliptin It is a new DPP-4 inh ibitor developed in Japan which exerts long-lasting (>24 hours) DPP-4 inhibition and antiglycaemic effect. Following a single morni ng dose, postpra ndial hyperglycaemia is suppressed at all 3 mea ls of the day. Therapeutic effi cacy both as monotherapy as well as in combination with metformin ± SUs or a thiazolidinedione is com parable to other gliptins. Metabolites of teneligliptin are excreted by both liver and kidney; no dose reduction is needed in patients with renal impairment. Though no cardiovascular adverse effects have been noted, caution is to be exercised in patients prone to QT prolongation. Dose: 20 mg/before breakfast daily, increase to 40 mg/ day if needed. TENGLYN. TENEFIT-20, TENLIMAX 20 mg tab. Alogliptin and Linagliptin are the other DPP-4 inhibitors.

Biguanide (AMPK activator)

Two biguanide antidiabetics, phenformin and metformin were introduced in the 1950s. Because of higher risk of lactic acidosis, phenformin has been banned in India since 2003.

INSULIN, ORAL ANTIDIABETI C DRUGS AND GLUCAGON

H -

N

C -

N -

I

II

I

H

NH

H

C -

I

NH

N -

I

CH3

CH 3

METFORMIN

Metformin

It d iffers markedly fro m SUs: causes little or no hypoglycaemia in nondiabetic subj ects, and even in diabetics, hypoglycaemia is rare. Th us, it is 'euglycaemic', rather than hypoglycaem ic' . 1t does not stimulate pancreatic 13 cells. Metformin is reported to improve lipid profile as well in type 2 diabeti cs.

Mechanism of action Bigu a nides do not cause insulin release, but presence of insulin is essential for the ir actio n. Metfo1min is not effective in pancreatectomized animals and in type I diabetics. Though the deta i Is are not clear, recent studies have recognized activation of AM P-dependent protein kinase (AMPK) to play a crucial role in me diating the actions of metformin, the key features of w hich are: I. S uppresses hepa tic g lu coneoge nesis an d glucose output from liver. Th is is the major action responsible for lowering of blood glucose in diabetics. 2. Enhances insulin-mediated glucose uptake and disposal in ske leta l mu scle a nd fat. Insuli n resistance ex hi bited by type-2 diabetics is thus overcome. This Lrnnslates into- glycogen storage in skeleta l muscle - reduced lipogenesis in ad ipose tissue and enhanced fatty ac id oxidation. 3. Interferes w ith mitochondrial respiratoty chain and promotes peripheral glucose utilization through anaerob ic glycolys is. AMPK activation by metform in appears to be an indirect consequence of interference w ith cellular respiration and lowering of intracellular ATP and other energy sources. Metformin a lso retards intestinal absorption of glucose, othe r hexoses, amino ac ids and Vit B 12 • Pharmacokinetics Metformin is we ll absorbed orall y, not metabolized, but excreted uncha nged by kidney (Tab le 19.2). Clearance of metformin approx imates g.f. r. lt accumulates

in renal failure and increases the risk of lactic acidosis.

Adverse effects Side effects w ith metformin are frequent, but generally not serious. Abdominal pain, anorex ia, bloating, nausea, metal lic taste, mild diarrhoea and tiredness are the usual complaints, which tend to subside with time. Metformin does not cause hypoglycaemia except in overdose. Lactic acidosis Small increase in blood lactate occurs with metformin, but lactic acidosis is rare (< I per 10,000 patient years) because it is poorly concentrated in hepat ic cells. Alcohol ingestion can precipitate lactic ac idosis. Vit B 12 deficiency due to interference with its absorption can occur with h igh dose of metformin.

In additio n Lo genera l restrictions for use o f oral hypoglycaemics (see below), rnetformin is contrai ndicated in rena l insufficie ncy, because risk of lactic acidosis increases. It sho uld also not be given in hypotensive states, heart fa il ure , severe res piratory/hepatic di sease, as well a s in alcoholics. Drugs like ci m e ti dine, furos e mide ma y compete w ith metformin excretion a nd enhance its toxicity.

Uses Metformin is now established as a first choice drng for all type 2 DM patients, except w hen no t tolerated o r contra ind icated. Advantages of metform in are: an tihyperglycaemic, but not hypoglycaemic weight loss promoting has potential to prevent macrovascular as we ll as mi crovascular complications of diabetes no acce leration of 13 cell ex haustion/ fa ilure in type 2 DM. antihyperglycaem ic efficacy ( HbA 1, reduction by 0.8- 1.2%) equ ivalent to other oral drugs. can be co mbined wi th a ny o ther oral or injectable antidiabetic. can prevent new o nset type 2 DM in obese, middle aged subjects w ith impaired glucose tolerance (but not in nonobese o lder subj ects) as shown by the Diabetes Pre vention Programme . The limi t ing feature is g. i. in to lerance, especially al higher doses, but lack of serious toxicity is well established by decades of use.

299

300

HORMON ES AND RELATED DRUGS lnferlility: Metformin has been found to improve ovulation and fenil ity in some infertile women w ith polycystic ovary. This benefit is observed irrespective of the glycaemic status of the woman; may be due to mitigation of insulin resistance and lowering of circu lating insulin levels.

Thiazolidinedione (PPARy agonist) Pioglitazone Only one t hi azo lidinedi o ne Pioglitazone is c urrentl y available in India. Rosiglitazone, the other member, is ban ned in India since 20 I O and has been withdrawn in Europe due to unacceptable increase in ri sk of myocard ial infarcti o n, C HF, stro ke and death. However, a large recent tria l failed to confi rm such ri sk, and restrictions on the use of rosiglitazone have been lifted in the USA. Because of some reports linking piogli tazone with carc inoma of urinary b la dd e r, it was banned in India in June 20 13. However, wi thin 2 months the ban was Ii fted wi th a wa rning label ' not to use it as a first line drug' . This class of ora l antidiabetic drugs are selective agon ists for the nuclear peroxisome proliferator-activated receptor y (PPARy) which is expressed mainly in fat cells, but also in muscle a nd some other ce ll s. Pi ogl itazone e nhances the transcription of severa l insulin responsive genes and tends to reverse insu lin resistance by enhancing G LUT4 ex pression and translocation. Entry of glucose into muscle and fat is improved; the primary action is to enha nce peripheral insulin sensitivity. Hepatic gluconeogenesis is a lso suppressed. Activation of genes regu lating fatty acid metabo lism and Iipogenesis in adipose tissue contributes to the insulin sensitizing action. Lipolysis and plasma fa tty acid levels are reduced. Adipocyte turnover and differentiati on is accelerated by glitazones. Thus, fatty tissue is a major s ite of t he ir action. The magnitude of blood glucose reduction is somewhat less than SUs and metformin. lmproved g lycaemic control results in lowering of circulating HbA,c a nd insulin levels in type 2 DM patients. Pioglitazone, in addition, lowers serum trig lyceride level and raises HDL level without much change in LDL level, probably because it acts on

PPARa as we ll to induce expression of reverse cholesterol transporter and some apoproteins. Pi oglitazone is we ll tolerated; adverse effects are plasma vol ume expans ion, edema, weig ht ga in, headache, mya lg ia an d mild anaemia. Monotherapy with g litazones is not associated with hypoglycaemic episodes. Few cases of hepatic dysfun ction have been reported; CHF may be precipitated or worsened. Monitoring of liver function is advised. Pioglitazone is contraindicated in liver disease and in C l-IF. Glitazones increase the risk of fractures, especially in e lderly women. Piogli tazone is metabolized by both CYP2C8 and CY P3A4. Ketoconazole inhibits and rifampin induces its metabolism. Failure of oral co ntraception may occur during pioglitazone therapy. Piog litazone is indicated in ty pe 2 DM, but not in type I DM. It reduces blood g lucose and HbA,c (by 0.5- 1. 2%) w ithout increasi ng c irculating insu lin . About 25% patients may not respond (nomesponders), probably due to low baseline insulin levels. It s hould be stopped if HbA 1, reduction is < 0.5% at 6 months. Pioglitazone is primaril y used to supplement SUs/metfo rm in a nd in case of insulin resistance. However, it is not likely to be effective when cell fa ilure has set in, wh ich may be responsible for loss of efficacy of combination of SUs + metfom1 in. It may also be used as monotherapy (a long with diet and exercise) in mild cases. Several reports describe greater fluid retention, weight ga in a nd precipitation of C H F after combi ned use of glitazones with insulin. Expert s advise avoid ing such combi nation. Pioglitazone should not be used during pregna ncy. The Diabetes Prevention Programme (2005) has shown that glitazones have the potentia l to delay progression of prediabetics to overt type 2 OM. Pioglitazone may he lp to conserve cell func tion in di abetics.

a. Glucosidase inhibitors Acarbose It is a complex ol igosaccha ride which reversib ly inh ib its a-g lucosidases, the fina l enzymes for the digestion of carbohydrates

INSULIN, ORAL ANTI DIABETIC DR UGS A N D G LUCAGON

in the brush border of small intestine mucosa. It slows down and decreases digestion and absorption of polysaccharides (starch, etc.) and sucrose. In addition, GLP-1 release is promoted which may contribute to the effect. Postprand ial glycaemia is reduced without significant increase in insulin levels. Regular use lowers HbA, c modestly ( by 0.4- 0.8%), but c hange in body weight and lipid levels is minimal. The stop- !DOM trial (2002) has shown that long-term acarbose treatment in prediabetics reduces occunence of type 2 OM as well as hypertension and cardiac disease. In diabetics, it reduces cardiovascular events. Acarbose is a mild antihyperglycaemic and not a hypog lycae mi c; may be used as an adjuvant to diet (w ith or without me tformin/SU) in obese diabeti cs. Acarbose 50- 100 mg TDS is take n at the beginning of each major meal. Only a small fraction of the dose is absorbed. Flatulence, abdo mina l di scomfort and loose stool are produced in about 50% patients due to fermentation of unabsorbed carbohydrates. Patient acceptability of o.-glucosidase inhi bi tors is poor due to uncomfortable g. i. symptoms. Hepatic transaminases may rise, but liver damage is rare. GLUCOBAY 50, 100 mg tabs, ASUCROSE, GLUCAR 50 mg Labs.

Miglitol

It has a smal ler molecule than acarbose, a nd it is a stronger inhibitor of sucrase. Potency for other o.-glucosidases is equivalent to acarbose. Absorption of miglitol is substantial, but variable. The absorbed drug is excreted by th e kidney. o systemic tox1c1ty is known. Dose: 25-100 mg TDS at beginning ofcach meal. M IGTO R, DIAM IG, ELITOX 25, 50 mg tabs.

Voglibose Has properties, use and side effects similar to that of acarbose. Dose: 200- 300 µg TDS jusJ before meals. VOGLITOR, VO LIX , V0Lll30 0 .2, 0.3 mg tabs.

Dopamine 02 agonlst Bromocriptine

Recently (2009) a quick release oral fo rmu lation of bromocriptine has been approved by US-FDA for adjunctive treatment of type 2 OM. Taken early in the morn ing it is though! 10 act on the hypo-

thalamic dopaminergic control of the circadian rhythm of hormone (GH, prolactin , ACTH , etc.) release and reset it to reduce insul in resistance. Bromocriptine can be taken alon e to supp lement diet+exercise or added to metform in or SU or both. Started at 0.8 mg OD and increased upto 4.8 mg OD (as needed) it has been shown to marginally improve glycaemic control and lower HbA ,, by upto 0.5%.

Sodium-glucose co-transport-2 (SGLT-2) Inhibitor Practically all the glucose fi ltered at the glomerulus is reabsorbed in the proximal tubules. The major transporter which accomplishes this is SGLT-2, whose inhibition induces glucosuria and lowers blood glucose in type 2 OM, as well as causes weight loss.

Dapagliflozin, Canagliflozin These SGLT-2 inhibitors are approved for use in type 2 DM patients. A ftcr once daily dosing, they produce round-the-clock glucosuria and lower blood g lucose levels. Used alone or in combination with other antidiabetic drugs, they reduce HbA ,c levels by 0.5-1.0%, but do not cause hypoglycaemia. The concerns which appear inbuilt due 10 the ir mechanism of action are-glycosuria wh ich ca n predispose lo urinary a n d gen ital infections, electrolyte imbalance, kctoacidosis and increased urinary frequency. They are contraindicated in pat ients with renal insufficiency. Other long term safety concerns have also been raised. Bile acid sequestrant Colesevelam

It is a bile acid b inding resin which lowers cholesterol as well as glucose levels in blood. It is approved as add-on drug in type 2 OM patients who are not properly controlled by other antid iabetic drugs. The mechanism of act ion is not clear, but may involve reduction in farnesoid X receptor (FX R) activation . T he HbA,c lowering efficacy of colesevelam is modest, and g.i. upset is frequent.

Status of oral antidiabetics in diabetes mellitus After 8 years of prospective study involving large number of patients, the Univers ity Group Diabetes Programme (UGDP) of USA ( 1970) presented findings that cardiovascular mortality was higher in patients treated with biguanides and SUs than in those treated with diet a nd exercise alone or with insulin. A decline in the use of oral hypoglycaemics followed. Subsequent studies both refuted and supported these conclusions. The controversy was settled by the UK PDS t rial which found that both SUs and metform in did not increase cardiovascular mortality

301

302

HORMONES AND RELATED DRUGS over > IO years observation pe riod. Related to degree of glycae mia control, both insu lin and SUs reduced mi crovascula r compli cations (reti nopathy, neuropathy, nephropathy) in type 2 DM, but did not have significant effect on macrovascular complications (coronary artery di sease, stroke, etc). Metformin, however, co uld red uce both microvascular as well as ma c ro vasc ul ar co mplicati ons. It decreased risk of death and other diabetes related endpoints in overweight patien ts. This may be related to the fact that both SUs and exogenous insulin improve glycaemic control by increasing insulin supply rather than by overcoming ins ulin resistance, while me tformi n can decrease insul in resistance which is a pathogenic factor in type 2 DM. All oral hypoglycaemics do however control symptoms that are due to hyperglycaemia and glycosuria, and are much more conve nient to use than insulin. Oral antidiabetics are in d icated on ly in type 2 diabetes, in addition to diet and exerci se. They are most effecti ve in patients with!. Age above 40 years at onset of disease. 2. Obesity at the lime of presentation. 3. Duration of d isease < 5 years when starting treatment. 4. Fasting blood sugar < 200 mg/di. 5. Insulin requirement < 40 U/day. 6. o ketoacidosis or a history of it, or any other complication. The Diabetes Preventi on Programme (2002) has establi shed that in middle aged, obese prediabetics metformin prevented progression to type 2 DM, but no t in o lder nonobese pred iabetics. G litazones a lso appear to have prophy lactic potential. Long-term acarbose therapy as well can delay type 2 DM . Oral antidiabetics should be used to supplement d ietary management and not to replace it. In view of the prophylactic and outcome benefits, c urrent recommendation is to institute metformi n therapy right at the diag nosis of type 2 DM, along with dietary and other li festyle measures, without waiting to see if the latter alone are sufficient. Mctformin may delay progression of d iabetic severity by

favourably affecting P cell health a nd retarding fail ure. It is especia lly valuable for obese patients, because it may also aid weigh t reduction. Further, it has the potential to reduce the risk of myocardial infarction and s troke. Thus, unless contraindicated or not tolerated, metformin is prescribed to all type 2 diabetics, despite its inferior patient acceptability due to g.i. side effects. Many type 2 DM patie nts do not attain the target leve l of glycaemia control and HbA 1c reduction (to < 7%) with metformin alone, and a second drug is needed. SUs are the most common ly selected 2nd drug. They have good patient acceptab ility, convenient dosing a nd high efficacy, but can cause weight ga in and hy poglycaemia. There is some evidence that SUs given over long-term (2- 10 years) exped ite p cell apoptosis and failure. Recepto r desensitization may also be a cause, and SUs tend to lose efficacy in few years (5-10% per yea r failure rate). There is no difference in the clin ical efficacy of var ious 2 nd generation SUs. Howeve r, thi s docs no t ind icate that choice among them is irrelevant. Differences among them are mainly in dose , onset and duration o f action which govern flexibility of regimens. Some specific features of various SUs are g iven in Table 19.2. If a particular SU proves inadequate in a given patient, a nother one may still work. Patie nts with near norma l fast ing blood g lucose but prominent post-prandial hyperglycaemia, or those experiencing late postmea l hypoglycaemia may do better with premeal repaglinide or nateglinide. Pioglitazone is usually the 3 rd choice drug; may be added to metformin or a combination of metformin + SU. It is particularly useful for patients wi th prom inent insulin res istance. Though it tends to preserve p cell fu nc tion and does not cause hypoglycaemia, it is rarely selected for monotherapy. Its major limitations are- tendency to fl u id retentio n, weight gain, increased ris k of heart fail ure and fractures, and need to monitor liver function. Moreover, it is ineffective in a sig nificant number of patie nts.

p cell

INSULIN, ORAL ANTI DIABETIC DRUGS AN D GLUCAGON

Acarbosc-like drugs are m ild a ntihyperglycaemics, mostly used as suppleme ntary drugs to a hy poglycaem ic regimen. They arc disliked by ma ny patients because of bloating, ind igestion and other abdominal symptoms. T he newer oral antidiabetics that have ga ined popularity now are the DPP-4 inhibitors. The ir favo urable feat ures are: • Insuli n release is glucose dependent (occurs only at high g lucose concentrations) therefore not likely to induce hypoglycaem ia. • Suppress g lucago n re lease, thus lowering fasting blood glucose level as well. • Improve cell hea lth and retard progression of cell failu re. • Body weight neutral. • Can be used in patients with re nal impairment. • Mostly single dai ly dose, well to lerated with few side efTects, no drug interacti ons, except with saxagliptin. However, the DPP-4 inhibitors ca use moderate HbA 1c lowering compared to SUs and some serio us reactions like ang iocde ma, S tevensJo hnson syndrome, a na phy lax is hav e bee n repo rted . Moreover, they a re relative ly new drugs and have not w ithstood the test of time yet. T heir impact on card iovascular morta lity a nd other outcomes is yet to be established. As suc h, most professiona l guide lines place the m as second line/add on antidiabeti c drugs. They are espec ia ll y val uable for patients having body we ight problem and those experi enc ing frequent episodes of hypoglycaem ia. The SGLT-2 inhibitors (e.g. dapagliflozin) are the newest class o f oral drugs that may be considered for addition to a combination oral antidiabetic regime when the latter fai ls lo achie, e the target HbA" level. They predispose to urinary lrncl infections. genital fu ngal infections. hypovolaemia and have other limitations

Upto 50% pati ents of type 2 OM initially treated with ora l antidiabetics ultimately need insulin . Moreover, w he n a di a betic on o ra l antidiabe ti cs presents with infec ti on, severe tra uma o r stress, pregnancy, ketoac idosis or any other complication, or has to be operated upo n- switc hover to insulin (see Flow chart in Fig. 19.7). Metformin alone or in combina-

t ion with SUs or DPP-4 inhibitors is usually continued with insulin, particularly when a single dail y injection of long-act ing (e.g. g larg ine) insulin is used to provide basal control. The oral drug given before mea ls serves to check postprandia l glycaemi a, while metfo rm in ma in ly reduces hepatic glucose output. The GL P- 1 agoni sts (exenatide/ liragl utide injected s.c.) are an alternative to insulin in obese type 2 diabet ics in whom weight gai n is to be avoided. They help weight reduction a nd a re unlike ly to indu ce hypoglycae mi a. Frequent na usea a nd ri sk o f pancrca titi s are the ir drawbacks. Epalrestat Sorbitol is a minor metabolite of glucose generated by the enzyme a/dose reducwse. In diabetics, excess sorbitol is produced and gets deposited in nerves and other tissues. This is im olved in the pathogenesis of diabetic neuropathy and other complications. Epalres1at is an aldose reductase inhibitor de, eloped in Japan "hich has been found to de lay sorbitol accumulation in sciatic nerve/other tissues of d iabetics imparting potential to delay progression of diabetic ncuropat hy. In tria ls it has caused mod est improvement in nerve conduction, neuropat hic pa in and other symptoms. However, magnitude of benefit and safety are yet to be defined. ausea, vom iting and elevation of liver enqmes are the adverse effects. Dose: 50 mg TDS before meals: ALRlSTA 50 mg tab. GLUCAGON A hyperglycaemic principle was demonstrated lo be present in the pancreatic islets j ust two years a fter the discovery o f insulin in 192 1. It was named ·glucagon·. G lucagon is a single chain polypeptide containing 29 amino acids, MW 3500. It is secreted by the n cells of the islets of Langerhans and commercially produced now by recombinant DNA technology.

Regulation of Secretion Like insulin, glucagon is also derived by cleavage of a larger peptide prohormonc. Its secretion is regulated by glucose levels, other nutrients, paracrine hormones and nervous system. Glucose has opposite eflects on insulin and g lucagon re lease, i.e. high g lucose level inh ibits glucagon secretion. T he incretin G LP-1, F FA and ketone bodies a lso in hibit glucagon re lease. Amino acids, however, induce both insulin and g lucagon secretion. Insulin, amylin and somatostati n, elaborated by the neighbouring and 6 cells, inhibit g lucagon secretion. Sympathetic stimulation consistently and parasympathetic stimulation under cenain conditions evokes glucagon release.

303

304

HORMONES AND RELATED DR UGS

New patient of Diabetes Type II

Type I

I

I

Diet+ Exercise + Met (unless C/1)

Diet + Insulin

l

!

I Controlled I



-c -= on ~t-= ro-ll= ed,...,I

'"'"I

Uncomplicated, well nourished / obese

I

Met + One drug (SU / Megli / Pio / DPP-4i / a-Gli)

Controlled

Uncontrolled

Severe, underweight, complications present, pregnancy, infection, surgery

Uncontrolled

I Met+Two drugs (SU / Megli / DPP-4i) + (Pio ± a -Gli)

Controlled

I i

Insulin± Met

I Controlled )

t

Uncontrolled 1--- -+ lnsulin (or GLP-1 Ago)± Met ± (SU/DPP-4i / a -Gli)

Fig. 19.7: Simplified flow chart of management approaches in diabetes mellitus Met-Metformin; SU-Sulfonylurea; Megli-Meglitinide/d-phenylalanine analogue; DPP-4i-Dipeptidyl peptidase-4 inhibitor; a Gli-a Glucosidase inhibitor; Pio-Pioglitazone; GLP-1 Ago-GLP-1 receptor agonist. Note: A meglitinide drug is indicated only in patients with predominant postprandial hyperglycaemia. An a glucosidase inhibitor can be additional add on drug.

Actions Glucagon is hyperglycaemic; most of its actions are opposite to that of insulin. Glucagon causes hyperglycaemia primarily by enhancing glycogenolysis and gluconeogenesis in liver. Suppression of glucose utilization in muscle and fat contributes modestly. Glucagon is considered to be the hormone of fue l mobilization. Its secretion is increased during fasting. and is largely responsible for the high fasting blood glucose levels in rype 2 diabetics. It plays an essential role in the development of diabetic ketoacidosis. Increased secretion of glucagon has been sho,vn 10 anend all forms of severe tissue inj ury. Glucagon increases t he fo rce and rate of cardiac contraction and this is not antagonized by 13 blockers. fl has a relaxant action on the gut and inhibits gastric acid production. Mechanism of action Glucagon, through its own receptor and coupling Gs prote in activates adenylyl cyclase and increases cAM P in liver. fat cells, heart and

other tissues; most of its actions are mediated through this cyclic nucleotide. Glucagon is inactive orally; that released from pancreas is broken down in liver, kidney, plasma and other tissues. Its pla ma t½ is 3-6 min.

Uses I. Hypoglycaemia Use of glucagon to counteract insulin/ oral hypoglycaemic drug induced hypoglycaemia is only an expedient measure for the emergency, and must be folio" ed by oral glucose/sugar given repeatedly till the blood glucose level stabilizes. It may not work if hepatic glycogen is already depleted. Dose: 0.5 1.0 mg i.v. or i.m. GLUCAGON I mg inj. 2. Cardiogenic shock Glucagon may be used to stimulate the heart in 13 adrenergic blocker treated patients. However. action is not very marked.

INSULIN, ORAL ANTI DIABETI C DRUGS AND GLUCAGON

Other hyperglycaemics

D1azoxide Chemically related to thiazides,

11 inhibits insulin release from P cells and causes hyperglycaemia lasting 4- 8 hours. Its action on ATP sensit ive K channels of p cells i~ opposite to that of SUs. Other actions which may contribute lo hyperg lycaemia are decreased peripheral utili?alion of glucose and release of catecholamines. It has been used to prevent hypoglycaemia in

insulinomas. Other action, are , asodilatation. fall in BP and ant idiuresis. Somatostatin It causes hyperglycaemia primari ly by inhibiting insul in release.

Streptozocin It is obtained from Srrepromyces achromogenes. Causes select ive damage to insu lin sec rct111g

p cells.

It has been used to produce experimental diabetes in animals and to treat insulin secreting tumours of pancreas.

r:r PROBLEM DIRECTED STUDY 19.1 During routine medical checkup a SO-year male office executive with sedentary lifestyle was diagnosed to have developed type 2 diabetes mellitus. His fasting and post-meal blood glucose was 130 mg/di and 190 mg/d i respectively, HbA1 c was 7.8%, BP was 130/82 mm Hg and body mass index was 27 kg/ m 2• He was asymptomatic and investigations revealed no end organ damage. He was advised suitable diet, exercise and other lifestyle modifications. (a) Should he be prescribed an antidiabetic medication as well? If so, which drug/ combination of drugs should be selected, and why? (see Appendix-1 for solution)

305

Chapter

20

Corticosteroids

The adrenal cortex secretes steroidal hormones which have glucocorticoid, mineralocorticoid and weak ly androgenic activ ities. Conventionally, the term 'corticosteroid' or 'corticoid' includes natural gluco- and mineralo-co11icoids and their synthetic analogues. By the middle of 19th century it was demonstrated that adrenal g lands were essential for life. Later 11 was appreciated tbat the cortex was more important than the medulla. A number o f steroidal acti,e principles were isolated and thei r chemical structures were elucidated by Kendall and his coworkers in the 1930s. However, the gate to their great therapeutic potential \\as opened by Hench (1949) who obtained striking relief in rheumatoid arthritis by using cortisone. The obel Prize was awarded the very next year to Kendall, Reichstcin and Hench.

BIOSYNTHESIS The co rticoids (both gluco and mineralo) are 21 carbon compounds having a cyclopentanoperhydro-phenanthrene (steroid) nucleus. They are synthesized in the adrenal cortical cells from cholesterol. A simp lified vers ion of the bio ynthetic pathways is presented in Fig. 20.1. Adrenal steroidogenesis takes p lace under the influence ofACTH which makes more cholesterol available for conversion to pregneno lone and induces stero idogenic enzymes. Since adrenal cortical cells store only mi nute quantities of the hormones, rate of release is governed by the rate of biosynthesis. The circulating corticosteroids

Cholesterol

l

Pregnenolone

/

0

6 () i=

a:::

0

()

0

_J

z

0 ::0 0

G)

m

z

(/)

I

307

CORT ICOSTERO IDS inhibi t ACTH re lease from p1tu1ta ry as well as C RH production fro m hypotha lamus (see Ch. 17) and th us provide negative feed back regula tion of the hypoth alamo-pituitary-adrena l (J-1 PA ) ax is . The normal rate o f secretion of the two principal corticoids in man isHydrocortisone I 0- 20 mg daily (nearly ha lf o f this in the tew morning hours). Aldostcrone - 0.1 25 mg daily.

involveme nt of hy potha lamo-pituitary-adrenal axis in stress response is depicted in Fig. 20.2. Corticoids have some direct and some permissive actions. By permissive acti on is meant that while they do not themselves produce an clTect, the ir presence facilitates other hormones to exert that action, e.g. they do not have any eftect on BP but the pressor action of Adr is markedly blunted in their absence. Actions of corticoids are di vided into:

ACTIONS

Glucocorticoid Effects on carbohydrate, protein and fat metabolism, and other actions that are inse parably linked to these.

The cort icoids have wi despread actions. T hey main ta in fl uid-electro lyte. cardiovascular a nd energy substrate homeostasis as well as functional status of skeletal muscles and nervous system. They prepare the body to withstand effects of a ll kinds of nox io us stim ul i and stress. T he

Minera/ocorticoid Effec ts on a . K· a nd fl uid balance. Marked dissociation between these rwo types of actions is seen a mong natural as well as synthetic corticoids. Accordingly. compounds are labe lled as ' glucocorticoid' or ' mineralocorticoid'.

•flii?11•1i1;~JWi1i*ii=-ii•---•~ Higher centre s

i

, - - - -- - HYPOTHALAMUS

-CRF

- -- • Stimulation Inhibition

- - - - -•

--- -- ---

---------------------- --

'-----+

I

ACTH Angiote ns inogen .- - - -RENIN Angiote nsin I

KIDNEY

..

I

I I

re tention

Adr.

GLOMERULOSA -

I I

1

Na+ & water

r:JJ:/JJXjj

I I

Hypovolaemia l Na· deple tion

Angiote ns in II

lN\l!l:J',XJ 36 hr)

6. Dexamethasone 7. Betamethasone

0

8

C)

::,

a

Prednisolone Methylprednisolone Triamcinolone Deflazacort

Mineralo

20mg 4 5 5 3-4

0.8 0.5 0 0

5mg 4mg 4mg 6mg

25 25

0 0

0.75 mg 0.75 mg

' 'O en

.2

r~

C

·-

:::ii:

'C 0 C)

8. Desoxycorticosterone acetate (DOCA) 9. Fludrocortisone 10. Aldosterone

Equiv. dose (antiinflammatory)

0

100

10 0.3

150 3000

Equiv. salt retaining dose 2.5 mg (sublingual) 0.2mg not used clinically

314

HORMONES AND RELATED DRUGS 4-32 mg/day oral , 5--40 mg i.m., intraarticular injection. Also used to picall y. Kl:.NACORT.TRICORT 1,4, 8mgtab.• 10mg/ml,40mg/ml(as acctonide) for 1.m., intraarticular inj .. LEDERCORT 4 mg tab.

5. Dexamethasone

Very potent and highly selective glucocorticoid. It is a lso long-acting, causes marked pituitary-adrenal suppression, but fluid retention and hypertension are not a problem. It is used for inflammatory and allergic conditions 0.5- 5 mg/day oral. For shock, cerebral edema, etc. 4-20 mg/day i.v. infusion o r i.m. injection is preferred. II can also be used topically. DFCADRON. DCXONA 0.5 mg tab, 4 mg/ml (a5 sod. phosphate) for, .~.• i.m. mj., 0 .5 mg/ml oral drops: WYMESO E, DECO/\ \l 0 .5 mg tab, 4 mg/ml inj.

6. Betamethasone Similar to dexamcthasone, 0.5- 5 mg/ day oral, 4-20 mg i.111., i.v. injection or infusion, also topical. OETNESOL. BETACORTR IL, CELESTONE 0 .5 mg, I mg tab, 4 mg/ml (as ~ocl. phosphate) ror i.v.• i.m. inj.. 0.5 mg/ml oral drops. BETNELAN 0.5 mg. I mg tabs.

Dexamethasone or betamethasone a re preferred in cerebral edema and othe r states in which fluid retention must be avoided.

7. Deflazacort

The glucocorticoid potency of thi s newer steroid is somewhat less than of prednisolone, but it lacks mineralocorticoid activity. It is claimed to produce fewer adverse effects, but that may be due to its lower potency. In some trials it caused lesser g rowth retardation in children; has been particularly recommended fo r pediatric patie nts. lt is used m ainly for inflammato ry and immunolog ical disorders. Dose: 60-1 20 mg/day initially. 6- 18 mg/day for maintenance; children 0.25 1.5 mg/kg daily or on alternate days. DEFGLU 6. 30 mg tabs. DEFLAR. DI:.~ /..A , DFZ I. 6. 30 mg tabs. DEFCORT I mg, 6 mg, 12 mg, 18 mg. 24 mg. 30 mg tabs. 6 mg/5 ml S)r.

8. Desoxycortlcosterone acetate (DOCA)

It has

only mineralocorticoid activity. Used occasionally for replacement therapy in Addison's disease: 2- 5 mg sublingual, 10 20 mg i.m. o nce or twice weekly.

9. Fludrocortisone

A potent mineralocorticoid having some g lucocorticoid ac ti vity as well, orally active, used for: Replacement therapy in Addison's disease 50-200 µg daily. Congenital adrenal hyperplasia in patients "ith salt wasting 50- 200 µg/day. Idiopathic postural hypotcnsion 100- 200 µg/day. FLORICORT LOO µg tab.

It is the most potent mineralocorticoid . used clinically because of low oral bioava1lability and difficulties in regulating doses. 10. Aldosterone 01

In addi tion a number of topically ac tive gl ucocorticoids have been developed. Beclomethasone dipropionate budesonide, fluti casone, etc. a re used by inhalation in asthma, as spray in nasal a llergy, as well as for skin and mucous membrane lesions (see Ch. 16). Fluocinolone acetonide, fluocortolone, clobetasol propionate and esters of betamethasone, dexamethasone, triamcinolo ne are described in Ch. 66.

USES

A. Replacement therapy 1 . Acute adrenal insufficiency It is an emergency. Hydrocortisone ( I 00 mg) or dexamethasone (8- 16 mg) are given i.v., first as a bolus injection and then as infusio n, a long w ith isotonic saline in glucose solution . The amount of flu id infused i.v. is guided by monitoring centra l venous pressure, because these patients have reduced capacity to excrete water load. Short-term i.v. in fusion of a vasopressor (dopa mine) may be needed. The cause of adrenal insufficiency should be treated.

2. Chronic adrenal insufficiency (Addison's disease) Hydrocortisone given orally is the most com monly used drng along with adequate salt and water allowance. Some patients who continue to excrete excess Na· need additional mineralocorticoid: fludrocortisone is added.

3. Congenital adrenal hyperplasia (Adrenogenital syndrome) It is a fam ilial disorder due to genetic deficiency of steroidogenic enzymes, mostly 2 1-hydroxylase. As a result the synthes is of hydrocorti sone and aldosterone suffers. There is compensatory increase in ACT! I ecretio n causing adrenocortical hype rtroph y. Enzyme deficie ncy being only partial in most cases, norma l amounts o f gluco- and mineralocorticoids are produced a lo ng with excessive amou nts of wea k androgens which produce viril ization and/or precocious sexual development. If the deficiency is severe, salt wasting a lso occurs.

CORTICOSTEROIDS Treatment is to give hydrocortisone 0.6 mg/ kg daily in di vided doses round the c lock to maintain feed back suppression of pituitary. Jf salt wasting persists- ftudrocortisone 50- 200 µg/day may be added.

B. Pharmacotherapy (for nonendocrine diseases)

SA IDs when distress and d isabi Iity persists despite other m easures, or to s uppress exacerbations, or when there arc systemic manifestations (see C h. 15). (ii) Osteoarthritis: It is treated with analgesics and SA IDs; systemic use or co1tico ids is rare. lntraarticular inj ection of a steroid may be g iven to control an acute exacerbation. Injections may be repeated 2- 3 times a year, but have the potential lo cau e joint destruction. (i ii) Rheumatic fever: Corticoids are used only in severe cases with carditis and CHF w ith the aim or rapid suppression of symptoms, because they ac t fas te r than aspir in, or in patients not responding to aspirin. Aspirin is g iven in add iti on a nd is continued after cort icoids have been withdrawn. (iv) Gout: Corticoids (short course) should only be used in acute gouty arthritis when NSAIDs have failed to afford relief and colchicine is not tolerated. lntraarticular injection of a soluble glucocon icoid is prererable to systemic therapy but is reserved for selec ted cases (see p. 232). Thoug h they are uricosuric- use in c hro ni c gout is not recommended.

Systemic as well as topical corticosteroids have one of the widest spectrum of med icinal uses fo r their antiinflammatory and immunosuppressivc propert ies. Corticosteroid s are powerful drugs. They have the potential to cause dramatic improvement in many severe diseases as well as produce equa lly dramatic adverse effects i r not properly used. The use of corticosteroids in nonendocrine diseases is e mpirical and pa lliative, but may be life saving. The fo llowing general principles must be observed. • A s ingle dose (even excessive) is not harmful: can be used to tide over morta l crisis, even when benefit is not ce rtain. • S hort courses (even high dose) are no t li kely to be harmful in the absence of contraind ications; starting doses can be high in severe illness. • Long-term use is poten ti a ll y hazardous: keep the duration of treatment and d ose to minimum, whic h is found by trial and erTor; even partial relief may have lo be tolerated. • Initi al dose depends on severity of the disease; s tart with a high dose in severe illnessreduce gradually after symptoms subside, while in mild cases start with the lowest dose and titrate upwards to find the correct dose. The dose s hould be reassessed from time-lo-time. • No abrupt w ithdrawal after a cortico id has been given for > 2 to 3 weeks: may precipitate adrenal insufficiency. • Infection, severe trauma, surgery o r any stress during corticoid therapy- increase the dose. • Use local therapy (cutaneous, inhaled, intranasal. etc.) wherever possible.

3 . Severe alfergic reactions Cortico ids may be used for s hort periods in anaphylaxis, angioneurotic edema, urticaria and serum sickness. However, even i.v. injecti on of a glucocorticoid takes 1- 2 hou rs to act and is not a substitute for Adr (which acts immediately) in anaphylactic shock and angioedema of larynx. Topical use is made in allergic conj unctivitis and rhinitis.

1 . Arthritides ( i) Rheumatoid arthritis: Corticosteroids a re indicated only in severe cases as adjuvants to

4 . Autoimmune diseases Autoimmune haemo1ytic anaemia, idi opat hic th rombocytopenic pu rpu ra, active chronic hepatitis respond to

2 . Collagen diseases Most cases of system ic lup us erythematosus, polyarteritis nodosa, dermatomyositis. nephrotic syndrome, g lomcrulo nephr itis and related diseases need corticosteroid therapy. They may be life saving in these diseases. Therapy is generally started with high doses which are tapered to maintenance dose when remission occurs. Later othe r immunosuppressants may be added or substi tuted.

315

316

HORMONES AND RELATED DRUGS corticoids. Prednisolone 1- 2 mg/kg/day is given till remission, followed by gradual withdrawal or low-dose maintenance depending on the response. Remiss ion may also be induced in severe cases of myastheni a gravis, in w hich their use is adj unctive to neostigmine. Patients requirin g long term maintenance therapy are better shifted to other immunosupprcssants. 5 . Bronchial asthma Early institution of inhaled glucocorticoid the rapy is now recommended in most cases needing inhaled ~, agonists almost daily (see Ch. 16). Systemic corticosteroids are used only for: • Status asthmati cus: give i. v. glucocorticoid; withdraw when emergency is over. • Acute asthma exacerbation: short-course of high dose ora l corticoid, fo llowed by gradual withdrawal. • Severe chronic asthma not con trolled by inhaled steroids and bronchodilators: add low dose prednisolone daily or on alternate days. 6 . Other lung diseases Corticosteroids benefit aspiration pneumonia and pulmonary edema from drownin g. Given during late pregnancy, corticoids accelerate lung maturation and surfactant production in the foeta l lung and prevent respiratory distress syndrome al birth. Two doses of betamcthasone or dexamethasone 12 mg i.m. at 24-hour interval may be administered to the mother if premature delivery is con templated. Beta methasone or dexamethasone are preferred because of their higher placental transfer.

chamber, e.g. a llergic conjunctivitis, iritis, iridocyclitis, keratitis, etc. Ordinarily, steroids should not be used in infecti ve co ndit io ns. But if inflammation is severe, they may be applie d in conjunction with an effecti ve antibiotic. Steroids are contraindicated in herpes simplex ke ratitis and in ocular injuries. Posterior segment afflictions like retinitis, optic neuritis, uvcitis require systemic steroid therapy. Retrobulbar injection is occasiona ll y given to avoid systemic side efTccts. 9. Skin diseases (see Ch. 66) Topical corticosteroids are widely e mpl oyed and a re hi g hl y e ffective in ma ny eczematous skin diseases. Systemic therapy is needed (may be life-savi ng) in pemphigus vu lgaris. exfoliative dermati tis, Stevens-Joh nson syndrome and other severe a ffi ictions.

10. Intestinal diseases U lcc rati ve col itis, Crohn 's disease, coeliac disease are inflammatory bowel diseases wit h exacerbations and remissions. Corticoids are indicated during acute phases-may be used ora lly or as retention enema (for colonic involvement). They are particularly valuable for patients with systemic manifestations, and a re given in add iti o n to sulfasalazi ne/mesalaz ine ± other measures (see Ch. 49). Some specia lists advocate small maintenance doses to prevent relapses.

7 . Infective diseases Administered under effecti ve chemotherapeutic cover, corticosteroids arc indicated only in serious infective diseases to tideover crisis o r to prevent complications. They are indicated in conditi ons like severe forms of tuberculosis (miliary. meningea l, renal, etc.), severe lepra reaction, ce rtain fo rm s of bacte rial meningitis and Pneumocystis carinii pneumoni a with hypoxia in AIDS patients.

11 . Neurological conditions Cerebral edema due to tumours, tubercu la r menin gitis, e tc., respond s to corticoids. Dexa- or bctameth aso ne are preferred beca use they donot have Na retaining activity. The ir value in traumatic and poststrokc cerebral edema is questionable. Large doses g iven i. v. soon after spinal injury may reduce the resulling neurological seqt1elae. A shor1 course (2-4 weeks) of oral prednisolone can hasten recovery from Be ll's pa lsy and acute exacerbation of multiple sclerosis. In the latte r, me thy l prednisolonc I g i.v. daily for 2- 3 days may be given in the begiru1 ing .

8 . Eye diseases Corticoids arc used in a large number of in0ammatory ocular diseases. In certain severe conditions, they may prevent blindness. Topical instillation as eye drops or ointment is effective in diseases of the a nterior

Neurocysticercosis: Wh e n albendazole/ praz iquantel is used to kill cysticerci lodged in the brain, prednisolone 40 mg/day or equivalent is given for 2-4 wee ks to suppress the reaction to the dying larvae.

CORTICOSTEROIDS

12. Nausea and vomiting Dexamethasone 8- 20 mg i.v. is frequently used to augment the antiemetic effect of ondansetron or metoclopramide against highly emetogenic cancer chemotherapy. High dose glucocorticoids by themselves afford modest protection from chemotherapy induced as well as general anaesthesia associated nausea and vomiting, probably due to their anti-inflammatory action.

17. To test pituitary-adrenal axis function Dexamethasone suppresses pituitary-adrenal ax is at doses which do not contribute to steroid metabol itcs in urine. Responsiveness of the axis can be tested by measuring daily urinary steroid metabolite excretion after dosing with dexamethasone.

13. Malignancies Corticoids are an essential component of combined chemotherapy of acute lymphatic leukae mia, Hodgk in 's and other lymphomas, because of their marked lympholytic action in these conditions. They have a secondary place in hollllone responsive breast carcinoma- act probably by causing HPA suppression so as to reduce production of adrenal androgens which are converted to estrogens in the body (see Ch. 64). Corticoids also afford symptomatic re lief in other advanced malignancies by im proving appetite and controlling secondary hypercalcaemia. For hypercalcaemia, however, bisphosphonatcs are more effective and have superseded corticosteroids.

These arc extension of the pharmacological action which become prominent with prolonged therapy, and are a great limitation to the use of corticoids in chronic diseases.

14. Organ transplantation and skin al/ograft 1-1 igh dose corti coids are give n a long with

other immunosuppressants to prevent the rejection reaction. Low maintenance doses may be continued over long term ± maintenance doses of companion drugs (see Ch. 65). 15. Septic shock High-dose corticosteroid therapy for septic shock has been abandoned, because it worsens the outcome. However. man y s uch patients have relati ve adrena l insufficiency. Recent studies have documented beneficial effects of low-dose (hydrocort isone 100 mg 8 hourly i.v. infusion for 5-7 days) therapy in patients who are adrenal deficient and who do not respond adequately to fluid replacement and vasopressors. 16. Thyroid storm Many patients in thyroid storm have concomi tant adrenal insufficiency. Moreover, corticosteroids reduce peripheral T4 to T 1 conversion. Hydrocortisone 100 mg i.v. 8 hourly may improve the outcome.

ADVERSE EFFECTS

A. Mineralocorticoid Sodium and water retention, edema, hypokalaemic alkalosis and a progressive rise in BP. These are now rare due to availability of highly selecti ve glucocorticoids. Gradual rise in BP occurs due to excess glucocorticoid action as well. B. Glucocorticoid 1. Cushi ng's habitus: characteristic appearance with rounded face, narrow mouth, supraclavicular hump, obesity of trunk with relati vely thi n limbs. 2. Fragile skin, purple striae-typically on thighs and lower abdomen, easy bruising, telangiectasis. hir utisrn. Cutaneous atrophy localized to the site occurs with topical application as well. 3. Hyperglycaemia, which may lead to glycosuria, precipitation of diabetes. 4. Muscular weakness: proximal (shoulder. arm, pelvis. thigh) muscles are pri marily affected. Myopathy occurs occasionally, warrants withdrawal of the corticoids. To offset the catabolic state, patients receiving corticosteroid therapy longer than 2 weeks should be put on high protein, high potassium diet. 5. Susceptibility to infection: this is nonspecific for all types of pathogenic organisms. Latent tuberculosis may flare; opportunistic infections with low grade pathogens (Candida, etc.) set in.

317

318

HORMONES AND RELATED DRUGS 6. Delayed healing: of wounds and surg ical incisions. 7. Peptic ulceration : risk is doubled; bleed ing and silent perfora tion of ulcers may occur. Dyspeptic symptoms are frequent with high dose therapy. 8. Osteoporosis: especially invol ving vertebrae and other flat spongy bones. Compression fractures of vertebrae and spontaneous fracture oflong bones can occur, especially in the e lderly. Radio logical evidence of osteoporosis is an indication for withdrawal of corticoid therapy. Corticostero id induced osteoporosis can be prevented/arrested to some extent by ca lcium supplements + vi i D, bu t bisphosphonates are the most effective drugs in this regard. Avascular necrosis of head o f femur, humerous, or knee j oi nt is an occasional abrupt onset complication of high dose corticoste ro id therapy. 9. Pos terior subcapsular cataract may develop after several years of use, especially in children. I 0. Glaucoma: may develop in susceptible individuals after prolonged topical therapy. 11. Gro,'11:h retardation: in children occurs even with small doses if g iven for long periods. Large doses do inhibit GIi secretion, but growth retardation may, in addition, be a direct cellular elTect of corticoids. Recom bi nant G H given concurrently can prevent growth retardation , but risk/benefit of such use is not known. l2. Foetal abnonnalities: Cleft palate and o ther defects are produced in animals, but have not been encounte red o n cl inical use in preg nant women. The risk of abo rtion, still-birth or neonata l death is not increased, but intrauterine growth retardation can occur after prolonged therapy, and neurological/behavioral disturbances in the offspri ng are feared. Prednisolone appears safer than dexa/beta methasone, because it is metabolized by placenta, reducing foeta l exposure. There is no evidence of foetal growth retardati on occurring a fter sho rt tenn use in the mother.

Prolonged corticosteroid therapy during pregnancy increases the risk of gestational diabetes, pregnancy induced hypertension and preeclampsia. 13. Psychiatric disturbances: mild euphoria frequently accompanies high dose ste roid treatme nt. This may rarely prog ress to manic psychosis. ervousness, decreased sleep and mood changes occur in some patients. Rare ly a depressive illness may be induced after longterm use. 14. Suppression ofhypothalamo-pituitary-adrenal (HPA) axis: occurs depending both on dose and duration of therapy. In time, the adrenal cortex atrophies. Stoppage of exogenous steroid precipitates withdrawal syndrome consisting of malaise, fever, anorexia, nausea, postural hypotcnsion, electrolyte imbalance, weakness, pain in muscles and joints and reacti vation of the disease for which the steroid was used. ubjected to stress, these patients may go into acute adrenal insuffic iency leading to cardiovascular collapse. Any patient who has received > 20- 25 mg/ day hydrocortisone, or ~ 5 mg prcdni solone/day or equivalent for longer than 2- 3 weeks s hou ld be put on a scheme of gradual wi thdrawal: 20 mg hydroco1tisone/day reducti on every week and then still smaller fractions once th is level has been achi eved . Such patients may need protection w ith a corticosteroid (oral or i.v.) if a stressful situation develo ps up to one year after w ithdrawal. Administratio n of ACTH during withdrawal does not hasten recovery because it has been found that adrenals recover earlier than pitui tary and hypothalamus. I f a pat ient on corti cos te roid the rapy develops an infection- the steroid should not be discontinued despite its propens ity to weaken host defence and de lay healing. Rather, the dose may have to be increased to meet the stress of infection. urgery in such a patient sho uld be cove red by intraoperative and postoperative i.v. hydrocortisone till the condition stabilizes, fo llowed by oral prednisolone.

CORTICOSTEROIDS Measures that mini mise I-IPA ax is suppression are: (a) Use shorter acting steroids (hydrocortisone, prednisolone) at the lowest possible dose. (b) Use steroids for the shortest period o f time possible. (c) Give the entire daily dose at one time in the morn ing. (d) Switch to a lternate-day therapy if the condition does not deteriorate on the 'off' day. h has been found that moderate dose of a short acting steroid (e.g. prednisolonc) gi,en at 48 hr interval did not cause HPA suppression. \\hereas the same total amounl given in 4 divided 12 hourly tloses produced marked HPA s uppression. Al ternate-day therapy also resulted in less immunological suppression ]O\\Cr risk of infection. The longer acting steroids (dcxamethasone, etc.) are not suitable fo r alternate-day therapy. O nly problem with alternate-day therapy is that many steroid dep.:ndent patients are incapacitated on the •ofT day' .

(e) If appropriate, use local (dermal, inhaled. ocular, nasal , recta l, intrasynovial) preparations of a steroid with poor systemic avai labi lity (beclomethasone dipropionate. tria mcinolone acetonide, fl uticasone, etc.).

CONTRAINDICATIONS The fo llowing diseases are aggravated by co rticosteroids. Since corticoste roids may have to be used as a Ii Fe-saving measure, all o f these a re relative contraindications in the presence of which these drugs are to be employed only under compelling circumstances, and with due precautions.

I. 2. 3. 4. 5. 6. 7. 8. 9.

Peptic ulcer Diabetes me ll itus Hypertension Vira l and fungal infections Tuberculosis and other infections Osteoporosis Herpes simplex keratitis Psychosis Epi lepsy

I O. CHF 11 . Renal failure Combination of any othe r drug with corti costeroids in fixed dose fo rmulation for internal use is banned. Metyrapone It inhib its 11-P hyd roxy lase in adrenal cortex and pre, cnts synthesis of hydrocortisone so that its blood level fall s • increased ACTI I release • increased synthesis, release and excretion of 11-desoxycortisol in urine. Thus, it is used to test the responsi, eness of pituitary and its ACTH producing capacity.

Ammoglutethim1de, trilostane and high doses of the antifungal drug Ketoconazole also inhibit steroidogenic enzymes-can be used to treat Cushing's d isease when surgery or other measures are not an option. Kctoconazolc reduces gonada l steroid synthesis as well.

Glucocortlcoid antagonist The antiprogestin m1fepristone (see p. 345~6) acts as a g lucocorticoid receptor antagonist as well. In Cushing·s syndrome, it can suppress the manifestations of corticosteroid excess. but blockade of feedback ACTH inhibition leads to oversccretion o f ACT! I • more hydrocortisone is produced, .-hich tends to annul the GR bloc king act ion o f mi fepristone. It is indicated only for inoperable cases of adrenal carcinoma and in patients with ectopic ACTH secretion.

er PROBLEM DIRECTED STUDY 20.1 A 35-year female patient of inflammatory bowel disease was treated with prednisolone 40 mg/day and mesalazine 800 mg TDS. After 4 weeks, the symptoms subsided and pre dniso lone dose was tape red at the rate of 10 mg every 2 weeks. When she was taking 10 mg prednisolone/ day, she met with a road-side a ccident and suffe red compound fracture o f both bo nes of the right le g. Internal fixatio n of t he fracture a nd s uturing of wounds under genera l anaesthesia is planned. (a) Whethe r any add itiona l measure nee ds t o be take n du ring su rgery in view of he r corticosteroid the rapy? (b) Does the prednisolone therapy need discontinuation or any alteration in the postoperative period? Give reasons. (see Appendix-1 for solution)

319

Chapter

21

Androgens and Related Drugs, Drugs for Erectile Dysfunction

Drngs that have androgenic action or that modify androgen function can be grouped as follows: ANDROGENS AND RELATED DRUGS

Natural

S nthetic

Testosterone Dihydrotestosterone Androstcrone

Methyltestosterone Fluoxymesteronc Mesterolone

Methandienone Nandrolone phenylpropionate androlone dccanoale Oxymetholone Stnnozolol

Cyproterone acetate Flutamide Bicalutamide

Finastcride Dutasteride

ANDROGENS

OH

(Male Sex Hormones) These are substances which cause development of seconda ry sex characters in the castrated ma le. That testes are responsible for the male characters is known since prehistoric times. Its endocrine function was established by Berthold in 1849. Testosterone was isolated as the testicular hormone, its structure was worked out and it was synthetically prepared by the year 1935.

Natural androgens Testes of adult ma le produce 5- 12 mg of testosterone daily. a pa11 of which is conve11ed in extraglandular tissues to the more active dihydrotestosterone (DHT); by the enzyme steroid 5 a-reductase; cholesterol is the starting material and the sa me pathway depic ted in Fig . 20. 1 is utilized. Adrenal cortex produces small quantities of dehydroepiandro-

0 TESTOSTERONE OH

H DIHYDROTESTOSTERONE

sterone and androstenedione which are called · weak androgens' (potency I/20 to I/30), but are infact inactive as such, and derive their weak activi ty from partial conversion to testoste rone

ANDROGENS AND RELATED DRU GS, DRUGS FOR ERECTILE DYSFUNCTION

,._ -------/

I

I

I I

:,,.

' \I

e•

I ..-------

+ -----,

e

+ --

l

,,

I

I

'\

I I I I

I

I

LHIFSH

I

I I I I

I I I

I I I I

I

I lnhibin

I I I I

I

,, I Estradiol

--

Testosterone

Fig. 21 .1 : Production of sex steroids and their regulation in the male In liver and many target cells the Sa-reductase enzyme converts testosterone to the more potent androgen dihydrotestosterone (DHT) which combines more avidly with the androgen receptor (AR). The aromatase enzyme in testes, liver and adipose tissue converts some testosterone into estradiol which exerts certain actions in male target cells by combining with estrogen receptor (ER) and is probably important for feedback inhibition of gonadotropins (LHIFSH) as well as that of gonadotropin releasing hormone (GnRH) from hypothalamus.

in peripheral tissues. Adrenals themse lves do not produce significant quantity o f testosterone. In women ovary produces small quantity of testosterone; this together with that derived indirectly from adrenals amounts to 0.25-0.5 mg/day.

Androsterone It is a metabolite of testosterone which is excreted in urine. IL has 1/10 the activity of testosterone.

Synthetic androgens Me1hyltestosterone and fiuoxymeslerone are 17-alkyl substituted derivatives of testosterone which are orall y active because of res istance to first pass metabolism, but have submaximal androgenic efficacy and potential to cause cholestatic jaundice. Other orally acti ve compounds are lestosterone undecanoate which is a dmini s tered as oi ly solution to be abso rbed through lym phatics bypassing the liver. and mestero/one. A numbe r of lipid-soluble esters of testosterone have been produced, suitable for injection in oily vehic le, from which they are absorbed slowly and exert prolonged action after deesterification in the body. Regulation of secretion Testosterone is secreted by the interstitial (Leydig) cells of the testes under the influence of pulsatile secretion of LH from pituitary. FS H is mainly responsible for promotion of spennatogenesis i.n tubular (Sertoli) cells. The mediator of feedback relationship with pituitary is uncertain . Whi le relatively high concentration of testosterone inhibits LH secretion and in time causes atrophy of interstitial cells. it has only weak inhibitory action on FS H secretion. Estrogens are more potent inhibitors of Gn secretion even in males, and it is believed that the small amount of estradiol produced by testes as well as that generated from conversion of testosterone to estradiol in liver and fat plays a role in the feedback inhibition. Jnhibin (a protein), produced by Scrtoli cells, has strong FS H inhibitory action and may be med iating the feedback inhibition. Testosterone and estradiol act on hypotha lamus to reduce GnRH as well as act directly on pituitary. The plasma level of testosterone in adult males ranges from 0.3 to I µg/d l. In women, small amounts of testosterone arc produced by corpus luteum and adrenal cortex ; blood levels remain low (20-60 ng/dl).

ACTIONS

1. Sex organs and secondary sex characters (Androgen ic) Testosterone is responsible for a ll the changes that occur in a boy at puberty:

321

322

HORMONES AND RELATED DRUGS Growth of genitals- penis. scrotum, seminal vesicles, prostate. Growth of hair- pubic, axillary, beard, moustache. body hair and male pattern of its distribution. Thickening of skin which becomes greasy due to proli feration and increased activ ity of sebaceous glands-especially on the face. The duct often gets blocked and infection occurs resulting in acne. Subcutaneous fat is lost and veins look prominent. Larynx grows and voice deepens. Behav ioural effects a re- increased physical vigour, aggressiveness, penile erections. Male libido appears to be activated by testosterone 1 direct ly, and probably to a greater extent by 1 estradiol produced from testosterone. Testosterone is also importan t for the intrauterine development of the male phenotype. Re latively large amounts of testosterone are produced by the foetal testes during the first half of intrauterine life.

2 . Testes

Modera tely large doses cause testicu lar atrophy by inhibiting Gn secretion fro m pituitary. Stil l larger doses have a direct sustaining effect and atrophy is less marked. Testosterone is needed for normal spermatogenesis and maturation of spermatozoa. H igb concentration of testosterone is a ttained locall y in the spermatogenic tubules by diffusion from the neighbou ring Leydig cells and stimul ates spennatogenesis.

3. Skeleton and skeletal muscles (Anabolic) Testos te rone is respo ns ible for the pubertal spurt of growth in boys and to a sma ller extent in girls. There is rapid bone growth, both in thickness as well as in length. After puberty, the epiphyses fuse a nd linear growth comes to a ha lt. Estradiol produced from testosterone, and not testosterone itself, is responsible for fusion of epiphyses in boys as well as in girls. Moreover, estradiol largely mediates the effect of testosterone on bone mineral ization. Testosterone also promotes muscle building, especially if aided by exercise. T here is accretion of nitrogen, minerals (Na, K, Ca, P, S) and water- body weight inc reases

rapidly, more protoplasm is built. Appetite is improved and a sense of well being prevai ls. The nitrogen retaining and muscle bu ilding effect of exogenous androgens is more marked in women and children than in adult men. Testosterone given to patients prone to salt and water retention may develop edema.

4. Erythropoiesis Testosterone accelerates erythropoiesis by increasing erythropoietin production and probably direct action on haeme synthes is. Men have higher hematocrit than women. Mechanism of action Testosterone can largely be regarded as the circ ulating prohormone. In most target cells, the 4- 5 double bond is reduced producing dihydrotes/oslerone- whieh binds more avid ly with the cytoplasm ic androgen receptor (A R), a nd this complex is more active than testosterone-receptor complex in combining wi th DNA. o subtypes of AR are known; both genital and nongenital (muscle, bone) cells express the same AR. After combining with androgen response elements of the target genes, DNA transcription is enhanced/ repressed with the help of coactivators or corepressors, which may be tissue specific. The effects a re expressed through modification of protein synthesis. The Sa-reduccase en7yme exis ts in two isoforms: Sa-reductase-1 and Sa-reductase-2. The genital skin of both sexes and urogenital tract of male contnins Sareductase-2 "hich is more sensitive 10 inhibition by finasleride. Genetic deficiency of this isoen7yme causes male pseudohermaphroditism because of inability of male genitalia to produce the active hormone dihydrotcstosteronc from circulating testosterone. So..-reductasc- 1

has a wider distribution in the body including nongenital skin and liver; and is inhibited by finasteride 10 a lesser ex1c111.

Testosterone itself appears to be the active hormone at certain sites, such as• foetal genital rudiments hypothalamus/pitui tary site involved 111 feed back regu lation • erythropoietic cells • spermatogenic cells in testes.

L_

ANDROGENS AND RELATED DRUGS, DRUGS FOR ERECTILE DYSFUNCTION

Principal androgen for different target tissue actions Testosterone

Dihydrotestosterone

Pituitary: LH inhibition

External genitalia (scrotum, penis, male urethra) formation in foetus and maturation during puberty

Testes: Spermatogenesis Internal genitalia (epididymis, vas deferens, seminal vesicle)



Pubertal changes. sexual maturity, male behaviour



Prostate growth; hypertrophy in elderly

development in foetus Erythropoiesis



Hair follicles: growth during puberty, scalp hair loss in adults

Bone and skeletal muscle: increase in mass and strength

PHARMACOKINETICS Testosterone i inactive orally due to high first pass metabolism in liver. The duration of action after i.m. injection is also very short. Therefore, slowly absorbed esters of testosterone are used by this route; they arc hydrolysed to the active free form. While testosterone propionate is short acting, and needs daily injections, the enanthate, cypionatc, isocaproate and decanoate esters are long acting; can be injected every 2-4 weeks depending on the dose. The latter are preferred for long term replacement therapy. Testosterone in circulation is 98% bound to sex hormone binding globulin (SHBG) and to a lbumin. The SHBG bound testosterone is unavailable for action due to tight binding. The major metabolic products of testosterone arc androsterone and etiocholanolone which arc excreted in urine, mostly as conj ugates with g lucuronic acid and sulfate. Small quantities of estradiol are also produced from testostero ne by aromatization of A ri ng in extraglandular ti ssues (l iver, fat, hypothalamus). Plasma t½ of te tosterone is 10 20 min. Mcthyltestosterone and fluoxymesterone are metabolized slowly and have a longer duration of action. but a re weaker androgens. Estrogens a re not produced from fluo xy mesteronc a nd dihydrotcstosteronc.

2. Te~tostcronc propionate: 25- 50 mg i.m. daily lO twice weekly: TESTOVIRO, • PARCNDRE~. TESTANON 25, 50 mg/ml inJ. 3. TESTOVI RON DEPOT 100: tc,to. propionate 25 mg+ testn. enanthate I 00 mg in 1 ml amp; I ml i.m. weekly. 4. TESTOV! RO"l DEPOT 250: testo. propionate 250 mg + testo. .:nanthatc 250 mg in I ml amp; i.m. every 2-4 weeks. 5. SUSTANON · 100' : tcsto. propionate 20 mg .,. tcsto. phenyl propionate 40 mg - tcsto. isocaproate 40 mg in 1 ml amp; I ml i.m. every 2-3 weeks. 6 . SUSl ANON ·250': tcsto. propionate 30 mg + 1es10. phcnylprop1onate 60 mg ... tcsto. isocaproate 60 mg + teslo. dccanoatc 100 mg in I ml amp: I ml i.m. every 3-4 \\eeks. 7. Nl,VIR. ANDRIOL; Testosterone undecanoatc 4 0 mg cap, 1-3 cap dai ly for male hypogonadism, ostcoporosi . 8. \ Jesterolonc: Causes less feedback inhibition of Gn secretion and spermatogcnesis, and has been promoted for treatment of male infertility PROVlRO"lUM, RESTORE. MESTILON 25 mg tab; 1- 3 tab daily for androgen deficiency, ohgoLOOspcrmia and male infertility.

Tran sdermal androgen Delivery of androgen across skin has been achieved by developing suitable solvents and absorptio n faci Iita tors. By cutaneous de! ivery, testosterone/ dihydrotestosterone circumvent hepatic first pass metabolism and uniform blood levels arc produced round the clock. A gel fonnu la tion has been marketed fo r once dail y application which has become the preferred method of androgen replacement for hypogonadism and impotence.

Preparations and Dose

Al'.DRACTlM: D1hydro1csto~terone 25 mg/ggcl ( 100 g tube); 5- 10 g gel to be applied over nonscrotal skin once daily.

I. Testosterone (free): 25 mg i.m. daily to twice weekly; AQUAVIRO:s; 25 mg in I ml inj.

Fixed dose combinations of testosterone with yohi mbine, strychnine and vitamins arc banned m India.

323

324

HORMONES AN D RELATED DRUGS

SIDE EFFECTS I. Admin istered to women, androgens can cause virilization, excess body ha ir and menstrua l irregularities. Many effects, e.g . voice change may be pe rmanent after prolonged therapy. 2. Acne: in both males a nd females. 3. Frequent, sustained and often pai nfu l erection s in ma les in the beginning of thera py ; subside spontan eously after sometime. 4. Oligozoospermia can occur with moderate doses g ive n fo r a few weeks to men w ith normal testoste rone levels. Prolonged use may produce testicular atrophy. 5. Precociou s pube rty, p rem a ture sex ual behaviour, and stunting of stature due to early c losure of cpiph ysis -if testosterone is given continuously to young boys for increasing stature. 6. Salt re tention and edema : especially when large doses are g iven to pa tie nts with hea rt or kidney disease. T his is rare with the doses used for hypogonadism. 7. Cholestatic j aundi ce: occurs w ith methyltcstosterone and other 17-alkyl substi tuted derivatives (ftuoxymesterone and some anabolic steroids li ke oxymetho lone, stanozolol) in a dose depende nt manne r, but not with parenterally used esters of testosterone. For this reason, the latter a re preferred. Howe ver, the jaundice is reversib le on discontinuation. 8. Hepa tic carc inoma: inc ide nce is hig her in patie nts who have received long-term methyltestosterone or other oral a ndrogens. 9. G ynaecomastia: may occ ur, es pecia lly in children and in patients with liver disease. This is due to peripheral conversion of testosterone to estrogens. Dihydrotestosterone does not cause gynaccomastia beca use it is no t con verted to estrad iol. I 0. Lowe ring of HDL and ri se in LDL le vels, especially with I 7a-a lkylated ana logues.

Contraindications A ndrogens are contraind icated in carc ino ma of prostate and carc inoma of male breast, as well as in li ver and ki dney disease and during pregnancy (masculinization of fe ma le foetus). They should not be given to men aged >65 years, and to those with coronary

artery disease or C HF. Androgen therapy can worsen sleep apnoea, migraine and epilepsy.

USES

1 . Testicular failure

It may be primary, which is noticed during childhood. and results in delayed pu be1ty. Treatment with parenteral testosterone esters or transdem1a l testosterone/ dihydrotestostcrone in courses of 4-6 months at a time may achieve satisfactory results/normal adul t stature if administered j udiciously to avoid acceleration of epiphyseal closure which occurs if the dose is high. Secondary testicular fai lure (hypogonad ism) occurring later in life manifests ma inl y as loss of libido, muscle mass a nd energy, feminization, m ild anaemia and impotence. T hese are corrected gradually over months by androgen treatm ent. However, impote nce due to psychological and othe r factors, and not testosterone defic iency, does not respond. 2. Hypopituitarism 1-lypogonadism is one of the features of hypopituita rism. Androgens are added at the time o f puberty in boys to othe r hormonal replacement. 3. AIDS related muscle wasting Testosterone therapy has been shown to improve weakness and muscle wasting in A IDS patients with low testosterone levels. 4. Hereditary angioneurotic edema Th is is a genetic disorder. T he attacks can be prevented by I 7a.-alkylated and rogens (methy ltestosterone, stanozolol, danazol) but not by testoste rone. These drugs act by increasing the synthesis of comple me nt (C I) esterase inh ibitor. 5 . Ageing Because 1es1os1ero nc levels dec line in o ld age, it has been administered 10 e lderly ma les with the aim o f impro\ ing bone mincraliz.a1ion and muscle mass. However, sa fety of such therapy in terms of metabolic. cardio, ascular and prostatic complications is not known. Occasionally small amo unt of androgen is added to posunenopausal hormone replacement. 6 . Idiopathic male infertility S ince high in1ra1es1icular level of 1estos1eronc is essential for spennatogcnesis, it is presumed that exogenous androgens will stimulate spenna10gcncsis or improve spenn maturation in epididymis. On the other hand. androgens can ad, ersely affect spcrmatogenesis by suppressing Gn secretion . S ince mesterolone causes less feedback inhibition o f Gn (probably due to restricted entry into brain) it is bel ieved that moderate doses will predominantly stimulate testis directly.

ANDROGENS AND RELATED DRUGS, DRUGS FOR ERECTILE DYSFUNCTION A recent metaanalysi;, of 11 clinical trials has found that oral androgens (mesterolone and testosterone undecanoate) had no crrc,cl on spenn count or sperm motility as well as on subsequent pregnancy rate I\ he n g iven to oligoasthcno-;,pe r m1c , ubfe rt ile men . As , uch, use of these androgen~ for imprm ing male fertility is not backed by objective evidence.

ANABOLIC STEROIDS These are synthetic androgens with suppo edly higher a nabolic and lower androgenic acti vity. Drugs are Nandro/one, Oxymethofone, Stanozo!of and Methandienone. The anabolic: a nd rogen ic activity ratio is dete rmined by injecting the drug in castrated rats and measuring the increase in weight of levator ani muscles to that of ventral prostate. The anabolic: androgenic ratio o f testosterone is considered as I; The anabolic selectivity of these steroids is modest w ith ratios between I to 3 in the rat model, and probably still lower in man. The anabolic effects are s imila r to that of testosterone and are mediated through the same receptor as the androgenic effects. For all practical purposes, they are androgens.

Preparations and dose I. Methandicnonc: 2 5 mg OD- BD oral; children 0.04 mg/ kg/day, 25 mg i.m. weekly; A '\iA BOLEX 2, 5 mg tab, 2 mgiml drops, 25 mg/ ml 111j . 2. Nandrolone phenyl propionate: I0-50 mg: children 10 mg: i.m. once or twice weekly; DURA BO LIN 10, 25 mg,m l mJ. 3. Nandrolonc decanoate: 25 I 00 mg i.m. every 3 weeks. D FC'ADURABOLIN. 25, 100 mg/ml inj. 4. Oxymetholonc: 5 10 mg, chi ldren 0.1 mg/kg, OD: A DRO YD 5 mg tab. 5. Stanozolol: 2- 6 mg/d ay. \1 ENABOL, :-JEURABOL. TANZOL 2 mg lah. Combination of anabolic steroids \\ ith any other drug is banned in India.

Side effects

Anabolic steroids were developed with the idea of avoiding the virili z ing side efTects of androgens while reta ining the anabolic effects. But the same adverse effect profile applies to these compounds. The 17-alkyl substituted compounds oxymetholone, stanozolol, can produce jaundice a nd worsen lipid profi le. Contraindications are same as for testosterone.

Uses I. Catabolic states Ac ute illness, severe trauma, major surge ry. etc. are anended by negative balance. Anabol ic steroids ca n reduce , loss over shor1 periods, but long-term benefits are questionable. T hey may cause a transient response in the e lde rly, unde r-no urished or debilitated indi viduals. but controlled studies have failed Lo demonstrate a difference in the tota l weight ga ined. ll owcver, short-term use may be made du ring conva lescence for the sense of wellbeing and improvement in appetite caused by uch treatment. 2. Osteoporosis In elderly ma les and that occurring due lo prolonged immobilization may respond lo anabolic steroids, but bisphosphonatcs arc more eflectivc and arc lht: preferred drugs. 3. Suboptimal growth in boys Use of anabolic steroids for accele ratmg the growth rate is controversial: somalropin is a better option. Brief spurts in linear growth can be induced by anabolic steroids. bul this probably docs not make a difference in the final stature, ei.cept in hypogonadism. Use for more than 6 months is not recommended-premature closure of e riphyscs and shortening o r uh11nale slaturc may result. 4. Hypoplastic, haemolytic and malignancy associated anaemia Majority of properly selected patients respond to anabulic steroids/androgens by an increase in RBC count and Hb%. Howe~er, crythropoictin therapy is more effective. 5. To enhance physical ability in athletes When administered during the period of trainmg androgens/ anabolic ste ro ids can increase the strength or exercised muscles. Ho" ever, effects are mostly short-lived and the magnitude o r imrro, cmcnt in perfonnance is uncertain except in women who may gain substamially. Howe ver, such u,e is considered illc:gal. and anabolic s teroids are included in the lis t of 'dope test' performed o n athletes before competiti, c games.

ANTIANDROGENS S11peractive GnRH agonim, arc the most potem inhibitors of gonadal ti.met ion. Administered 0\ er a fe,~ days, they markedly inhibit LH and FSH release. resulting in loss of androgen secretion (see Ch. I 7).

Keroco11a:ole al high doses inhibits stcroiclogenic CYP 450 lt:Slostcrone as well as adrenal steroid rroduction is interfered. Plasma rrotein binding of testosterone i~ also reduced. However. toxicity of' high doses precludes its use to suppress androgens.

Cimeridine and spimnolactone have weak antiandrogenic action "hich manifests as side errects. Progesterone has weak androgen receptor blocking action.

325

326

HORMONES AND RELATED DRUGS Drugs that ha ve been c lin ica ll y used as antiandrogens are:

Dose: 250 mg TDS

Cyproterone acetate This relatively weak AR antagonist is chemically related to progesterone. In contrast to flmamide which increases LH release by blocking feedback inhibition, cyprot.erone acetate inhibits LH release by its progestational activity. Lowering of serum testosterone (consequent to LH inhibition) supplements its d irect antiandrogenic action. Given to boys in relatively higher doses, it prevents pubertal changes, while in adult men libido and androgenic anabolism are suppressed. Its clinical indications arc- precocious puberty in boys, inappropriate sexual behaviour in men, acne and hirsutism in women (i n combination with an estrogen). Combined with cthinylestradiol (35 µg) cyclic (3 weeks on- I week ofl) treatment with cyprotcrone has also been used as a contraceptive in women. Eflicacy o f cyproteronc in metastatic prostate carcinoma is inferior to other forms of androgen deprivation. I lepatotox icity limits its use. Do~·e: 2 mg OD; G INCTTE-35. DINAC-35; cyprot~ronc: acctnte :! mg+ cthinyl cstradiol 35 µg tab.

Bicalutamide This more potent and longer acting (t ½ 6 days) congener of Autamid e is suitable for once daily administration in metastatic carc inoma of prostate as a component of CAB therapy. When used along with a GnRH agonist or castration, 50 mg OD affords marked relief in bone pain and other symptoms due to the metastasis. Side effects are hot Rashes, chills, edema a nd loose stools, but it is better tolerated and less hepatotoxic than Autamide. Elevation of hepatic transaminase above twice nonnal is a signal for stopping the drug.

Flutamide

A nonsteroidal A R antagonist with no other hom1onal activity. Its active metabolite 2-hydroxyflutamide competiti vely blocks androgen action on accessory sex organs as well as o n pituitary. Thus, it increases LH secreti on by blocking feedback inhibition . Plasma testosterone levels increase in males which partially o vercome the direct antiandrogenic acti on. This limits utility of monotherapy with antiandrogens in carcino ma prostate. They are now used only in conjunction w ith a GnRH agon ist (to suppress LH and testosterone secretion) o r after castratio n to block the residual action of adrenal androgens as combined androgen blockade (CA B) therapy of metastatic carc inoma prostate (also see p. 264). It is pre ferabl y started 3 days before the Gn RH ago nist to block the initia l Aare up that may occur due to excess re lease of LH and testoste rone in the beginning (be fo re GnRH receptors are desensitized). However, long-term benefit of CAB over GnRll ago nist a lone is not established. Along with oral contraceptives it has been tried in female hirsutism. but its hepatotoxic potential may not j ustify such use. Though gynaecomastia and breast tendern ess occur frequently, libido and potency are largely preserved during Autamide treatment. Reports of liver damage have restricted its use.

PROST.\'\11D. fL l:TIDE, C'YTOMID 250 mg tab.

BIPROSTA. C \ LUTIDE. TA BI 50 mg tab. Nilutamide is another potent and long acting antiandrogen.

5 a -REDUCTASE INHIBITOR Finasteride A competitive inhibito r o f the e nzyme 5 a -reductase wh ic h conve t1s testosterone into the more active DHT responsible for androgen acti on in many tissues inc luding the prostate gland and hair follicles. It is relatively se lective fo r 5 a-re ductase type 2 isoenzyme which predominates in ma le urogenita l tract. C irc ulating and prostatic DHT concentration are lowered, but plasma LH and testosterone levels remain unchanged because testosterone itself mediates feedback pituitary LH inhibition. Treatment with fi naste ride has res ulted in decreased prostate size and increased peak urinary Aow rate in ~ 50% patients wirh sympto matic benign hypertrophy of prostate (BHP). The benefi cial effects arc typically delayed needing ~6 months for maximum symptomatic relief. Patients with large prostate (volume > 40 ml) obtain greater relief than tJ10se with smaller gland. Upto 20% reduction in prostate size may be obtained. Withdrawal of the drug results in regrowth o f prostate, but with continued therapy benefit is maintained for 3 years or more. The re lief of obstructi ve sy mpto ms, however, is less marked compared to surgery and adrenergic a 1 blockers (see p. 157). It primarily reduces the static component of obstruction, wh ile a I b lockers overcome the dy namic compo ne nt.

ANDROGENS AND RELATED DRUGS, DRUGS FOR ERECTILE DYSFU NCTION

Concurrent treatment with both produces greater symptomatic relief. Finastcride has also been found effective in male pattern baldness, though hair follicles have primarily type I enzyme. In such subjects it promotes hair growth and prevents further hair loss. Observable response takes 3 or more months therapy and benefit is reversed withi n I year of treatment cessation. However, 20-30% cases do not improve. Finasteridc is effective orally, extensively metabolized in liver-metaboli tes are excreted in urine and faeces; plasma t½ 4-8 hours {elderly 6 15 hours). It is well tolerated by most patients; side effects are decreased libido, impotence and decreased volume of ejaculate (each in 3-4% patien ts). Gynaecomastia, skin rashes, swelling of lips are rare. Dose for BHP 5 mg OD. re, ie w afler 6 months; for male pancm baldness I mg/day. fll'sCAR, flNA ST, Ftr-.ARA 5 mg tab; Fl1'PECIA, ASTIFINE I mg tab.

Dutasteride This newer congener of finasteride inhibits both type I and type 2 5a-reductase and reduces DHT levels. It is metabolized by CYP3A4 and is very long-acting {t½ is - 9 weeks). It is approved for use in BHP and can benefit male pattern baldness. In clinical trias, both finasteride and dutasteride have been found to reduce the risk of developing carcinoma prostate by upto 25%. Interactions with CYP3A4 inducers and inhibitors are possible. Dose: 0.5 mg OD; DLPROST. DURIZE 0 .5 mg tab.

DRUGS FOR ERECTILE DYSFUNCTION Erectile dysfunction (ED) refers to the inability of men to at1ain and maintain an erect penis with sufficient rigidity to allow sexual intercourse. It occurs mainly past middle-age and is common after the age of 65 years. A variety of vascular. neurogenic, hormonal, pharmacologic or psychogenic causes may underlie the disorder. Sexual arousal increases blood Aow to the penis and relaxes the cavernosal sinusoids so that they fill up with blood, making the penis rigid, elongated and erect. itric oxide ( 0)

released from parasympathetic nonadrenergic noncholinergic ( A C) nerves and vascular endotheli um is the major transmitter causing re laxation of smooth muscle in corpus cavernosum and the blood vessels supplying it; ACh and PGs also play a role. A variety of mechanical/prosthetic dev ices and surgery have been used for ED, but drug therapy has made a big impact now. 1. Androgens Hypogonadism is an infrequenL cause of ED. Parenteral testosterone esters or transdermal testosterone therapy is effective only when androgen deficiency is proven to be responsible for the loss of libido and ED. 2. Phosphodiesterase-5 (PDE-5) inhibitors This class of drugs have become the first li ne therapy for ED. itric oxide causes smooth muscle relaxation by generating cGMP intracellularly which then promotes dephosphorylation of myosin ligh t chain kinase (MLCK) so that myosi n fails to interact with actin (see Fig. 40.3). Inhibition of PDE-5, the cGMP degrad ing isoenzyme in cavernosal and vascular smooth muscle, results in accumulation of cGMP and marked potentiation of O action. Sifdena.fil, Tadala.fil and vardenaji/ are selective PDE-5 inhibitors found effective in a majority of patients with ED. cGM P

I · - -- - - PDE-5



S-GMP

Inhibited by sildenafil

Sildenafil 11 is an orally active drug, marketed in the USA in 1998 and 2 years later in India, for treatment of ED. It became an instant hit, and evoked world wide response. Sildenafi l acts by selectively inhibiting PDE-5 and enhancing NO action in corpus cavernosum. Penile tumescence during sexual arousal is improved, but it has no such effect in the

327

328

HORMON ES AND RELATED DRUGS

absence of sexua l activity. It does not cause priapism in most recipients. Oral bioavailab ility of sildenafil is ~40%, peak blood levels are attained in 1- 2 hr; it is metabolized largely by CY P3A4 and an active metabolite is produced; t½ in men 65 years 25 mg), if not effecti ve the n I 00 mg I hour be fo re intercourse. Duration a nd degree of pe ni le erection is increased in 74-82% men with ED including diabetic neuropathy cases. Over 20 contro lled trials have confirmed its efficacy. However, sildenafi l is ineffective in men who have lost libido or when ED is due to spinal injury or damaged nervi eregantes. Adverse effects Side e ffects are mainly due lo PDE-5 inhi bition related vasodilatation- headache, nasal congestion, dizziness, facia l flushing and fa ll in BP, loose motions. Relaxation of lower esophageal sphincter may cause gastric reflux and dys pepsia. Si ldenafil, in addition, weakly inhibits the isoenzyme PD E-6 which is involved in photoreceptor transduction in the retina. As such, impairment of colour vision, especially blue-green discrimination , occurs in some recipients. Few cases of sudden loss of vision due to nonarteritic ischaemic optic neuropathy (NA IO N) among users of PDE-5 inhibitors ha ve been reported. Sildenafil markedly potentiates the vasodilator action of nitrates; precipitous fall in BP; Ml can occur. After >6 million prescriptions dispensed in USA, the FDA received reports of 130 deaths related to sildenafi l use by the year 2002. Most deaths occurred in patients wi th known risk factors, drug interac tions or contraindications. and were timed either during or within 4-5 hours of sex. Si ldenafil is contraindicated in patients of coronary heart disease and those taking nitrates. Though sildenafi l remains effective for 90% success rate and is the accepted non surgical method of earl y first trimester abortion. In place of oral misoprostol, a I mg gemeprost pessary can be inserted intravagi nally. Mifepristone adm inistered wi thin IO days of a missed period results in a n a ppare nt late heavy period (w ith dislodged blas1ocyst) in upto 90% cases. This procedure is generally safe, but prolonged bleeding and failed abortion are the problems in some cases. A norexia, nausea, tiredness, abdom ina l discomfort. uterine cramps, loose motions are the other side e lTects. 2. Cervical ripening 24 30 hours before attempting surg ica l abortion or induction of labour, mifepristone 600 mg results in softe ning of cervix; the procedure is facil itated. 3. Postcoital contraceptive Mifepristone 600 mg given w ithin 72 hr of intercourse interferes with implantat ion and is a highl y effective method of e me rgency contraception. The subsequent menstrua l cycle, however, is distu rbed. 4 . Once-a-month contraceptive A sing le 200 mg dose or mifepristonc given 2 days after midcyclc each month causes lutcolysis and prevents conception on most occasions. Administering mifcpristone in late luteal phase to

345

346

HORMONES AND RELATED DRUGS dislodge the embryo (if present) and to ensure menstruation irrespective of conception, has also been tried. These alternative methods of contraception, though attractive, may prolong/disrupt the next menstrual cycle, and thus cannot be used continuously. There is linle experience and little justification to use these methods on regular basis. 5 . Induction of labour By blocking the relaxant action of progesterone on uterus of late pregnancy. mifepristonc can promote labour. It may be tried in cases with intrauterine foetal death and to deliver abnormal foetuses. 6 . Cushing's syndrome Mifcpristone may have palliative effect due to glucocorticoid receptor blocking property, but efficacy is low. It may be tried in case the causative tumour is nonrcseclable. O1her proposed uses are--in endomctriosis, uterine fibroid. certain breast cancers and in meningioma. MIFEGCST. MIFEPR I 200 mg tab. T-PILL + MISO. ABORTOM : ,1ifepristone 200 mg (3 tabs) M isoprostol 200 ~1g (2 tabs); for medical terminatio n of pregnancy of upto 49 days: take 3 tablets o f T-PILL on day I, followed on day 3 by 2 tablets of MISO.

Ulipristal It is a recen tly approved ' selective progesterone receptor modulator ' (S PRM) for use as emergency contraceptive. It inhibits ovulation by suppressi ng LH surge as well as by direct effect on foll icular rupture. In addition, its action on endometrium can interfere with implantation. In clinical tria ls the efficacy o f ulipristal (30 mg) as emergency contraceptive has been rated equal to that of levonorgcstrel ( 1.5 mg) whe n tak en within 72 hou rs of unprotected intercourse, and to be more effecti ve when taken between 72- 120 hours. Th us, it may have an advantage, if the woman misses to take the drug within 72 hours. Headache, nausea. vom iting, abdominal pain and menstrual delay are the s ide effects, as they are with levono rgestrel. Few cases of ovarian cysts are reported. The antig lucoco rticoid activity of ulipristal is weaker than that of mifepristone. Onapristone (a pure progesterone amagonis1) and Gestinone (more e ffi cacious in endomctriosis) are the other antiprogestins.

HORMONAL CONTRACEPTIVES These are ho rmo nal preparations used for reversible supp ression of fe rtility. Because of

our alarming population trends, anti fertility drugs are a growing need. In developing countries particularly, the mortality rate has declined and birth rate has increased due to urbanization. ln the earlier part of the 20th century, methods of contraception used (condoms, diaphragms, spermicidal creams, foam tablets. etc.) were intimately related to sexua l intercourse, therefore, despised by most couples. These devices have a hi gher fa ilure rate as well. Rock and Pincus ( 1955) announced the successful use of an oral progestin for contraception, separating fertility control from coirus. It was soo n discovered that addi tion of a small quantity o f an estrogen enhanced thei r effi cacy; combi ned pills have become the most popular method of contraception. particularly because the hormone content of the pills has been reduced, minimizing the potential harm and affording other health benefits. FEMALE CONTRACEPTION Over I 00 million women worldwide are currently usi ng hormona l contraceptives. With these drugs, fertility can be suppressed at will, for as long as desired, with almost 100% confidence and complete return of fertility on discontinuation. The efficacy, convenience, low cost and overa ll safety of oral contraceptives (OCs) has allowed women to decide whether and when they want to become pregnant and to plan their acti vities. A variety of oral and parenteral preparations are now available offering indivi dual choices.

TYPES OF METHODS Oral 1. Combined pill It contains an estrogen a nd a proges t in in fi xed dose for all the days of a treatment cycle (monophasic). With accumu lated experience, it has been possible to reduce the amount of estrogen and progestin in the 'second gene ratio n' OC pills w ithout compromising efficacy, but reducing side effects and complications. 'Third generation'pills containing newer progestins like desogestrel with

ESTROGE NS, PROGESTINS AND CO NTRACEPTIVES

Table 22.2: Oral contraceptive preparations PROGESTIN

ESTROGEN

TRADE NAME

COMBINED PILLS

1. Norgestrel

0.3 mg

2. Norgestrel 3. Levonorgestrel

Ethinylestradiol

30 µg

MALA-D (21 tabs + 7 ferrous sulfate 60 mg tabs.)

0.5mg

Ethinylestradiol

50 µg

OVRAL-G 20 tabs.

0.25 mg

Ethinylestradiol

50 µg

OVRAL, DUOLUTON-L 21 tabs.

4 . Levonorgestrel

0.15 mg

Ethinylestradiol

30 µg

OVRAL-L, OVIPAUZ 21 tabs.

5. Levonorgestrel

0.1 mg

Ethinylestradiol

20 µg

LOETTE, OVILOW, COMBEE 21 tabs

6. Desogestrel

0.15 mg

Ethinylestradiol

30 µg

NOVELON 21 tabs.

7. Desogestrel

0.15 mg

Ethinylestradiol

20 µg

FEMILON 21 tabs.

1. Levonorgestrel

50-75 - 125 µg

Ethinylestradiol

30-40 - 30 µg

TRIQUILAR (6 - 5 + 10 tablets)

2. Norethindrone

0.5--0.75 -1.0 mg

Ethinylestradiol

35-35 - 35 µg

ORTHONOVUM 1nn (7+7+7 tabs)

1. Levonorgestrel

0.25 mg

Ethinylestradiol

50 µg

OVRAL, DUOLUTON-L (2+2 tabs)

2. Levonorgestrel

0.75 mg 1.5 mg

-Nil -Nil-

NORLEVO, ECEE2 (1+1 tab}iPILL, NOFEAR-72, OH GOD (1 tab)

1. Norethindrone

0.35 mg

-Nil-

MICRONOR", NOR-OD'

2. Norgestrel

75 µg

-Nil-

OVRETTE'

PHASED PILL

POSTCOITAL PILL

MINI PILLS

'Not marketed in India.

improved profile of action have been introduced in the 1990s. Ethinylestradiol 30 µg daily is cons idered threshold but can be red uced to 20 ~1g/day ifa progestin with potent antiovulatory action is included. The progestin is a 19-nortestosterone because these compounds have potent antiovulatory action. Used alone the ovulation in hibitory dose (per day) of the currently used progesti ns is estimated to be- levonorgestrel 60 µg, dcsogestrel 60 µg, norgestimate 200 µg. gestodene 40 µg, but the amoun t in the pill is 2- 3 times higher to attain I 00% certainty. Whi le both estrogens and progestins synergise to inhibit ovulation, the progestin ensures prompt bleeding at the end of a cycle and blocks the risk of developing endometrial carcinoma due to the estrogen. One tablet is taken daily for 21 days, starting on the 5th day of menstruation. T he nex t course is started after a gap of 7 days in wh ich bleeding occurs. Thu , a cycle

of 28 days is maintained. Calendar packs of pills are available (Table 22.2). This is the most popular and most efficacious method. 2. Phased pill Triphasic regimens have been introduced to permit reduction in total steroid dose w ithout compromising efficacy by mimicking the normal hormonal pattern in a menstrual cycle. The estrogen dose is kepi constant (or varied slightly between 30-40 µg), while the amount of progestin is low in the first phase and progressively higher in the second and third phases. Phasic pills are particularly recommended for women over 35 years of age and for those with no withdrawal bleeding o r breakthrough bleeding while on monophasic pill, or when o ther risk factors are present. However, evidence of superiority of triphasic pills over monophas ic pills is insufficient.

347

348

HORMONES AND RELATED DRUGS 3 . Progestin-on/y pill (Minipi/1) It has been devi ed to eliminate the estrogen, because many of Lhe long-term risks have been ascribed to this component. A low-dose progestin-only pill is an alternative for women in whom an estrogen is contraindicated. It is taken daily continuously without any gap. The menstrual cycle tends to become irregular and ovul ation occurs in 20-30% women, but other mechanisms (hostile cervical mucus and interference with implantation) contribute to the contracepti ve action. The efficacy is lower (96- 98%) compared to 98- 99.9% with combined pill. Pregnancy should be suspected if amenorrhoea of more than 2 months occurs. This method is not popular. 4 . Emergency (postcoital} pill These are for use in a woman not taking any contraceptive who had a sexual intercourse risking unwanted pregnancy. The most commonly used and standard regimen is• Levonorgestrel 0.75 mg two doses 12 hours apart, or 1.5 mg si ngle dose taken as soon as possible, but before 72 hours of unprotected intercourse. Trials conducted globally by a WHO task force on postovulatory methods of ferti lity control have found this regimen to be 2-3 times more effective and better tolerated than the earlier ' Yuzpe method' which used levonorgestrel 0.5 mg + ethinylestradiol 0.1 mg, two doses at 12 hours interval within 72 hours of exposure. incidence of nausea and vomiting is -6% in the progestin only regimen compared to 20-50% with the estrogen+progcstin regimen. Headache and other side effects arc also milder. However, the next period may be delayed or disrupted with either regimens. Recently (20 I 0) a SPRM ulipristal has been approved fo r emergency conLraception. • U liprista l 30 mg s in gle dose as soon as possible, but within 120 hours of intercourse. It is a n eq uall y effective (fa il ure rate 1- 3% compared to levonorgestrel 2- 4%) and eq ua lly well tolerated alternative method now available with an extended window of protective acti on (see p. 346).

Another antiprogestin that has been used , particularly in Europe and China, with high success rate and few side effects is• Mifepristone 600 mg single dose taken within 72 hours of intercourse. Emergency postcoital contraception should be reserved for unexpected or accidental exposure (rape, condom rupture) only, because all emergency regimens have higher failure rate and side effects than regular low-dose combined pill.

Injectable These contraceptives have been developed to obviate the need fo r daily ingestion of pills. They are given i.m. as oily solution; are highly effective; over 50 mil lion women have used them so far. Their major limitations are: (a) Animal data has indicated carcinogenic potential, but there is no proof from human studies despite >30 years of experience. o increase in overall risk of cervical, ovarian or hepatic cancer has been noted by a WHO sponsored study. Breast cancer risk may be sligh tly increased in younger women (< 35 yr). The logistics of administration and supervision for mass use are considered inadequate in developing countries and use- effectiveness in field conditions is low. In India approval has been granted fo r use on ly under close supervision. but not on mass scale under the ational Programme. (b) Menstrual irregularities, excessive bleeding or amenon-hoea are very common; incidence of amenorrhoea increases with increasing duration of use. Return of fertility may take 6- 30 months after discontinuation; permanent sterility may occur in some women. Weight gain a11d headache occur in >5% subjects. Bone mineral density may decrease after 2- 3 years of use (especially with DMPA) due to low estrogen levels caused by Gn suppression. T his may also produce menopause-l ike symptoms (hot flashe , vaginal dryness, reduced Iibido). Only the long-acting proges1in alone injections arc in use now. They arc injected once in

349

ESTROGENS, PROGESTINS AND CONTRACEPTIVES 2- 3 months depending on the steroid and its dose. Two compounds have been marketed: (a) Depoe medroxyprogesterone acetate (DMPA) 150 mg at 3-month intervals. After i.m. injection peak blood levels are reached in 3 weeks and decline with a t½ or - 50days.

A progestin impregnated intrauterine insert (PROGES· TASER I') has been introduced in some countries. It contains 52 mg of levonorgestrel which primarily acts locally on cndometrium. The device remains effective for 5 years, but efficacy is rated lower. Transdermal contraceptive A transdermal patch con• taining a progestin norelgcstromin and ethinylcstradiol for once \\ eckly applica11on for 3 weeks followed by one week gap is avai lable in some countries.

DEPOT·PROVERA 150 mg in I ml , ial for deep i.m. injection during first 5 days of menstnaal cycle. Repeat every 3 months.

(b)

MECHANISM OF ACTION

orethindrone (Norethisterone) enanthate (N EE) 200 mg at 2-month intervals.

Hormonal contraceptives interfere with fertility in many ways; the re lative importance depends on the type of method. This is summari zed in Table 22.3. I. Inhibitio n of Gn re lease from pituitary by reinforcement of normal feedback inhibiti on. The progestin reduces freq uency of LH secre tory pulses (an optimum pulse frequency is required for tiggering ovulation) whil e the estrogen primarily reduces FSH secretion. Both synergise to inh ibit midcycle Lll surge. When the combined pill is taken both FS H and LH are reduced and the midcycle surge is abolished. As a result, fo llicles fail to deve lop and fai l to ruptu re. Suppression of ov11/arion is the primary mode of action of combined oral pill. The low dose progesti n- only pill and progestin-only injectab le reg imen also atte nuate LH surge but less consistently--ovul ation may occur irregu la rly in ~ 1/ 3 cycles. Postcoital pill when taken before ovulation can dampen LH surge and inhi bit ov ul ati on in some cases. However, pregnancy is still prevented by direct actions on the genital tract.

"lOR ISTERAT 200 mg 111 I ml vial for deep i.m. injcc• tion during first 5 days of menstrual cycle. Repeat every 2 months.

The mos t importa nt drawback is co mp le te disruption of menstrual bleeding pattern or total amenorrhoea (more common with DMPA). It is not suitab le for adolescent girls a nd lactating mothers. Use of DMPA is gene rally restricted to women who are unlikely to use other contraceptives effecti vely. NEE is shot1er acting and fai lure rates have been higher than wi th DMPA. All fixed dose combmation injectable preparations of syn· thetic estrogens and progestins arc not allm,ed in India and discontinued in most countrie;.

Implants These are drug delivery systems implanted under the skin, from which the steroid is released slowly over a period of 1- 5 year,. They consist of eithcr(a) Biodegradable polymeric matrices-do not need to be removed on expiry. (b) Non•biodegradable rubber membranes-have to be removed on expiry. 1'.ORPLA"IT: A set of 6 capsules each comaining 36 mg levonorgestrel (total 216 mg) for subcutaneous implantation is a,ailable in some countries. but has been discontinued in the USA. It works for up to 5 years.

Table 22.3: Effects of different forms of hormonal contraception Injections

Oral pills Combined E + P

Progestin-only P

Postcoital only P

Progestin·only

+

1. FSH inhibition

++

2. LH inhibition

+++

+

+

+++

3. Antiovulatory effect

+++

+

+,-

++

4 . Hostile cervical mucus

+++

+++

5. Endometrium

Hypersecretory

Out of phase

Unfavourable

Atrophic

6. Failure rate (on perfect use)

0 .3%

2-3%

2-4%

35 years of age, diabetics, hypertensives and in those who s moke. The excess ris k is due to the estrogen component of the OC, and norma lizes shortly after stopping the OC. 2. Corona,y and cerebral thrombosis resulting in myocardial infarction or stroke: A 2 to 6-fold increase in ris k was estimated earl ier, but rece111 s tudies have demonstrated only small increase in incidence with the low dose pi lls in the absence of other ri sk factors. Women above 35-year age are particularl y at ris k. T he estrogen component of O C has been mainly held respons ible for venous thrombocmbolism, whi le both estrogen and progestin have been blamed for the arteria l phenomena. The mechan ism s involved may be: • Increase in blood clotting factors (eoagulability is enhanced). • Decreased antithrombin Ill. Decreased plas minogen activator in endothelium. Increased platelet aggregation. 3. Rise in BP: Occurred in 5- 10% women ta kin g the ea rlier pills. This again is less frequent and sm a ller in mag nitude with th e low-dose pills of today. I f the BP rise s, best is to stop OCs: the BP normalizes in the next 3-6 m onths. Both the estrogen and proges tin com po n e n ts are res pons ibl e fo r this effect, probab ly by increas ing plasma angiotensinogen leve l and ren in activ ity which have been fo und to be ra ised during OC use. A ldoste rone secretion is enhanced res ulting in salt and water retention.

4. Estrogen tends to raise plasma I ID L/LDL ratio (beneficial), but the progestin nullifies this benefit. Lipid profile is not sig nificantly a ltered by low dose OCs, except that trig lyceride level may rise marg inally which poses no excess risk. 5. Genital carcinoma: An increased inc idence of vaginal. cerv ical , and breast cancers was feared on the basis of animal data, but extensive epidemiological data over the past 30 years has repeatedly shown that oral as well as injected contraceptives do not increase the occurrence of these cancers in the general population. Howe ve r, ri s k is increased in predisposed indi viduals. Growth o f a lready exist ing hormone dependent tumour may be hastened. Epidemio logical data has record ed minor increase in breast ca ncer incidence amo ng current OC users, but no t a mong past users. Since breast cancer is rare in yo ung women, this finding is considered inconseque111ial. A p ro tective effect against endometri a l carc inoma has been shown for the prog estin component. Prolonged s uppression of gonadotropic stimu lati on of ovary may account for the lower in cidence of ovari an malignancy noted in contraceptive users. 6. Benign hepatomas: which may rupture or tum malignant; incidence of thi s rare tumour appears to be s lightly higher in OC users. 7. Gallstones: Estrogens increase biliary cholesterol excretion; incidence of gallstones is s lig htly higher in women who are taking OCs, or after long-term use.

Contraindications The combined ora l contraceptive pill is absolutely contraindicated in: 1. Thromboernbolic, coronary and cerebrovasc ular disease or a history of it. 2. Moderate-ta-severe hypertens io n; hyperlipidaemia. 3. Acti ve liver disease, hepatoma or h/ o j aundice during past pregnancy. 4. Suspected/overt maligna ncy of gen itals/ breast. 5. Prophy ria. 6. Impending maj or surgery- to avoid excess risk of postoperative thromboembolism. Estrogen

351

HOR MON ES AND RELATED DRUGS

352

containing contracepti ve shou ld be stopped 4 weeks before surgery.

Relative contraindications (requiring avoidance/ cautious use under supervision) I . Diabetes: contro l may be vitiated. 2. Obesity 3. Smoking 4. Und iagnosed vaginal bleeding 5. Uteri ne lciomyoma: may enlarge with estrogenic preparations; progestin only pills can be used. 6. Mentally ill 7. Age above 35 years 8. Mi ld hyperten s ion 9. M igraine I 0. Gall bladder disease.

Interactions

Contraceptive fa ilure may occur if the followi ng drugs are used concurrently: (a) Enzyme inducers: phenytoin, phenobarbitonc, primidone, carbamazepine, ri fampin, ritonavir. Metabol ism of estrogenic as well as progestational component is increased. (b) Suppression ofintestinal micro.flora: tetracyclines, ampicillin, etc. Deconjugation of estroge ns excreted in bile fai ls to occur • their enterohepatic circu lation is interrupted • blood levels fall. With both types of interacting drugs, it is wise to switch over to a preparation contai ning 50 µg of ethinylestradiol or to use alternative method o f contraception. Rifampin is usua lly taken for a long time and is such a potent e nzyme inducer that a lternative contraception should be advised.

Other health benefits Apart from benefits due to prevention of unwanted pregnancy and the risks during delivery, use of oral contraceptives affords certai n other beneficial efTects as a bonus: • Lower risk of developing endomctrial and ovarian carcinoma; probably colorectal cancer as well. • Reduced menstrua l blood loss and associated anaemia; cycles if irregular become reg ular; premenstrual tens ion. dysmenorrhoea and menorrhagia are ameliorated.



Endometriosis and pelvic inflammatory disease tend to subside. Reduced incidence as well as symptomatic relief of fibrocystic breast d isease and ovarian cysts.

Ormeloxifene (Centchroman) It is a nonsteroidal SERM developed at C ORI India as an oral contraceptive. It has p redominant estrogen antagonistic action in uterus and breast with little action o n vagi nal ep ithelium and cervical mucus. Endometrial proIi feration is suppressed by down regulation of endometrial ER. Contraceptive action is probably due to utero-em byon ic asynchrony and failure of implantation. Pituitary, ovarian and ot her endocrine fu nctions remain practically unaffected. Menstrua l cycle is not disrupted, but in so m e women it may be lengthened irregu larly. Excess ive bleeding attending anovu latory cycles (that genera ll y occurs near menopause) is dimini shed; orme lox ifene is a lso app roved for use in dysfunctional uterine bleeding. The plasma t ½ of o rme loxifene is lon g (-1 week). It prevents conception as long as taken with return of ferti lity few months after stoppage. Failure rate is considered acceptable, but it has failed to gain popu larity for use as contraceptive. Side effects are nausea, headache, n uid retenti on, weight ga in , rise in BP and prolongation of menstrual cycles. Dose: For contraception- 30 mg twice a week for 12 weeks followed by once a week.

For dysfunctional uterine blccding- 60 mg twice a week for 12 weeks, then once a week for 12 "eeks. CENTRON 30 mg tab, SAHFLI 60 mg tab.

MALE CONTRACEPTION The only way to suppress male fertili ty by drugs is to inhibit sper matogenesis. T hough considerable effort has been made in this direction and effective drugs have been found, no satisfactory/acceptable solution is yet tangible. Reasons are1. Complete suppression of spcrmatogenesis is relatively difficult without affecting other tissues: millions of spermatozoa are released at

ESTROGENS, PROGESTINS AND CONTRACEPTIVES each ejaculation vs a singl e ovum per month in women. 2. Spermatogenesis takes 64 days. A drug which even completely inhibited spem1atogenesis wi ll take a long latent period to produce infertili ty. Accordingly, return of fertility will be slow. 3. Gonadotropin suppression inhibits testosterone secretion as well, resulting in loss of libido and impotence, which i s unacceptable to all men and to most spouses. 4. Ri sk of adverse efTects. 5. M ost importantly- men don't get pregnant: few would be ready to bear the contingency of regular medication so that their sexual partners do not become pregnant. Drugs and approaches tried arc1. Antiandrogens Depress spermatogcncsis, but raise Gns; cause unacceptable loss of libido. 2. Estrogens and progestins unacceptable femini73tion.

QI-

Act by suppressing Gns-cause

3. A11droge11.1· They inhibit Gns but have poor efficacy. Even combination " ith progestin ts not reliable. 4. Superactive Gn RH unalogues They inhibit Gn release by continuous action: inhibit testosterone secretion as well; produce impotence. loss of libido.

5. Cytotoxic drugs Cadmium, nitrofurans and indoles suppress spcrmatogencsis, b ut arc toxic. 6. Gossypol It is a nonstcroidal compound, obtained from con on seed which has been studied in China. It is efTective orally--causes suppression of spermatogenesis and reduces sperm motility- infcnility develops afier a couple o f months. Fcnili ty is restored several mo nths after discontinuation. However, about 10% men remain o ligozoospermic. During treatment serum LH and testosterone levels do not change: libido and potency arc not afTectcd. The mechanism o f action though not clearly known, appears to invohe direct toxicity on semini ferous epithelium. Most imponant adverse efTec t of gossypol is hypokalacmia (due to renal loss of K·' with its anendant muscula r weakness (even paralysis). Other side efTects arc--edema, d iarrhoea. breathlessness and neuritis. Its use has now been dropped.

PROBLEM DIRECTED STUDY

22.1 A 55-year-old postmenopausal woman developed a cancerous lump in the left breast for which radical mastectomy was performed. The tumour was ER positive and only one of the excised axillary lymph nodes had metastasis. She was put on adjuvant therapy with tamoxifen 20 mg per day. On her checkup visit one year late r, she was found t o be asymptomatic wit h no sign of local recurrence or lymph node enlargement, but ultrasound examination of the uterus revealed thickening of endometrium. (a) What could be the cause and implication of the increase in endometrial thickness? (b) Should the same adjuvant therapy continue, or should it be stopped altogether, or be replaced by another drug? Give reasons. 22.2 A 28-year-old mother with a 9 month baby wants to space out her next child and consults you for taking oral contraceptive. (a) What questions will you ask, what physical examination will you perform and what investigations will you order before advising her whether she should take oral contraceptive or not, as well as for selecting the contraceptive preparation most suitable for her? (see Appendix-1 for solutions)

353

Chapter

23

Oxytocin and Other Drugs Acting on Uterus

Drugs acting on uterus can primarily affect the e ndometrium or the myometrium. The most important drugs affecting endometrium are estrogens, progestins and their antagonists. Myometrium receives both sympathetic and parasympathetic innervation. As such, autonomic dru gs can affect its motil ity. However, the directly acting drugs are more important and have more selective action. The respons iveness of myometrium to drugs is markedly affected by the hom1onal and gestational status.

OXYTOCIN

Oxytoci n is a nonapeptide secreted by the posterior pituitary along with argenine vasopressin (AV P or ADH ). Pitu itary extract was first used in labour in 1909. Contro versy as to whether the antidi urctic and uterine stimulating activities were due to one substance or tw~ separate substances was finally resolved by du Vigncaud in 1953 when he separated Oxy tocin and Vasopressin, determi ned their chemical structure and synthesized them. Both have 9 am ino acids, and differ at positions 3 and 8. Both oxytoci n and AVP are synthesized within the nerve cell bodies in supraoptic and paraventricular nuclei of hypothalamus; are transported down the axon and stored in

UTERINE STIMULANTS (Oxytocics, Abortifacients)

These drugs increase uterine motil ity, especially at term.

UTERINE STIMULANTS

(Oxytocics)

I

I [ Posterior pituitary hormone Oxytocin Desamino-oxytoci.n

I IErgot alkaloids Ergometrinc (Ergonovi.n e) Mcthy lergomctrine

I I Prostaglandins (PGs) PGE2 (Dinoprostone) PGF2a (Dinoprost) 15-methyl PGF211 (Carboprost) Misoprostol (+ Mifepristonc)

I Miscellaneous Ethacridine Qui.nine

OXYTOCIN AND OTHER DR UGS ACTING ON UTERU S the nerve endings within the neurohypophysis. They are stored in separate neurones as complexes with their spec ific binding proteins (neu rophysins) to form g ranules. Both are released by stimuli appropriate for oxytocin, i.e. coitus, parturition, suckling; or for AD H, i.e. hyperto nic saline infusion, water deprivation, haemorrhage, e tc. , or nonspecific. i.e. pain and apprehension. However, the proportion of oxytocin to A DH can vary depending upon the nature of the stimu lus.

ACTIONS 1. Uterus

Oxytoci n increases the force and frequency of uterine contractions. With low doses, fu ll rel axation occurs inbetween contractions; basal tone increases onl y with high doses. Estroge ns sensitize the uterus to oxytocin. They increase oxytocin recepto rs. onpregnant uterus and that d uring early pregnancy is rather resistant to oxytocin; sensitivity increases progressively in the third trimester, and there is a sharp increase near term. A qu ick fa ll in sensitivity occurs during puerperium. Proge tins decrease the sensitivity, but thi s elTect is not marked in vivo. At term the enhancement of contractil ity is restricted to the fundus and body; lower segment is no t contracted; may even be relaxed.

Mechanism of action Action of oxytocin on myometrium is independent of innervation. There are specific G-protein coupled oxytocin receptors o n t he myomet rium w hic h mediate the response mainly by depolarization of muscle fibres and infl ux of Ca2+ ions as we ll as through phosphoinositidc hydrolysis and 1P3 mediated intrace llular release of Ca2+ ions. The number of oxytocin receptors increases markedly during later part of pregnancy. Oxytocin also increases PG synthes is and release by the endometrium which may contribute to the contracti le response. Distinct subtypes of oxytocin receptors have been shown on the myomctrium and the endometrium. 2. Breast

Oxytoci n contracts the myoepitheliu m of mammary alveoli and fo rces milk

into the bigger milk sinusoids. Oxytocin mediates the ' milk ejection reflex ' (milk letdown in cattle) which is initiated by s uckling, so that milk may be easily sucked by the infant. Oxytocin has been used in milch ca tt le to facilitate milking.

3. CVS

Conventional doses of oxytocin used in obstetrics have no effect on BP, but hig her doses cause vasodilatation producing brief fall in BP, renex tachycardia and flushing. This action is most marked in chicken and is used for bioassay of oxytocin. The umbil ical vessels a re markedly constricted; oxytocin may help in their closure at bi rth.

4 . Kidney

Oxytocin in high doses exerts AD Ii- like action- urine o utput is decreased: pulmonary edema can occur if large amounts of i.v. fluid s and oxytocin are infused together. Conventional doses are without any effect.

Physiological role 1 . Labour Oxytocin is released du r ing labour and the uterus is hi g hly se nsi ti ve to it at thi s time. However, it does not appear to be obligatory for initiating parturition. Delivery occ urs even in hy pophysectomized animals and humans, though labour may be prolonged in its absence. A facil itatory ro le is more plausible. PGs and PAF arc complementary to oxytocin.

2 . Milk ejection reflex S uckling re n ex ly induces oxytocin release from pituitary and it contracts the myoepithelia l cells. These cells in breast a re more sensitive than the myomerri um 10 oxytocin. Milk ejection reflex is absent 111 the hypophysectom ized anima l. 3 . Neurotransmission Oxytocin appears to function as a peptide neurotransm itter of oxytoc inergic neurones in the hypo tha lamus and brainstem wh ich serves to regulate autonomic outflow.

PHARMACOKINETICS Being a peptide , oxytoci n is inacti ve orall y and is generall y administered by i.m. or i.v.

355

HORM ONES AND RELATED DR UGS

356

Unitage and preparations I IU of oxytocin = 2 µg of pure hormone. Commercially available oxytocin is produced synthetically. OXYTOClN. SYNTOCINON 2 IU/2 ml and 5 IU/ml inj ., PITOCI 5 lU/0.5 ml inj.

(a) Because of its short t½ and s low i.v. infusion, intensity o f action can be controlled and action can be quickly terminated. (b) Low concentrations a ll ow normal relaxation inbetween contractions-foeta l oxygenation does not suffer. (c) Lower segment is not c ontracted : foetal descent is not compromised. (d) U terine co n tractio ns are consistently augmented.

USE

3. Postpartum haemorrhage, Caesarean section Oxytocin 5 IU may be injected i.m.

routes, rarely by intranasal spray. It is rapidly degraded in liver and kidney; plasma t½ 6- 12 min. This is further shortened at tern,. Pregnant uterus and placenta elaborate a spec ific aminopeptidase called oxytocinase. This enzyme can be detected in maternal plasma.

1 . Induction of labour Labour needs to be induced in case of postmaturity or eve n prematurely in toxaemi a of pregnancy, d iabetic mother, erythroblastosis, ruptured me mbranes or placental insufficiency. For this purpose oxytocin is given by slow i.v. infusion: 5 IU is diluted in 500 ml of glucose or saline solution ( 10 mi Iii IU/ml}-infusion is started at a low rate and progressively accelerated according to response (0.2- 2.0 ml/min). Before starting the infusion. it should be made certain that: • presentation is correct • foetal lungs are adequately mature • there is no cephalopelvic disproportion • no placenta previa • no foetal distress and no uterine scar (due to previous surgery). Uterine contractions are the n closely monitored: the drug is discontinued when they are strong enough . Usuall y a total of 2-4 ru is needed.

or given by i.v. infus io n for an im mediate response, especially in hypertensive women in whom ergomctri ne is contraindicated. It acts by fo rcefully contracting the uterine muscle which compresses the blood vessels passing through its mes h work to arrest haemorThage from the inner surface exposed by placental separation.

2 . Uterine inertia



When uterine contractions are feeble and labour is not progressing satisfactorily-oxytocin can be infused i.v. (as described above) to augment the contractions. It should not be used to hasten normally progressing labour. Before deciding to use an oxytocic fo r strengthening uterine contractions, all the conditions as setout above (for induction of labour) must be fu lfilled. Too strong contraction can be catestrophic: use should only be made in selected cases a nd by experienced people. Oxytocin is the drug of c hoice and is preferred over ergometrine/PGs for the above two purposes:

4 . Breast engorgement It may occur due to inefficient milk ejection reflex. Oxytocin is effective only in such cases but not w hen milk secretion is defic ient. An intranasal spray may be given few minutes before suckling. Oxytocin does not increase milk production. 5 . Oxytocin challenge test 11 is perfonned to determine uteroplaccntal adequacy in high risk pregnancies. Oxytocin is infused 1.v. at ,ery low concentrations till uterine contractions are elicited every 3-4 mins. A marked increase in foetal heart rate indicates uteroplacental inadequacy. The test is risky and is rarely perfonncd.

Adverse effects Injudicious use of oxytocin during labour can produce too strong uterine contractions forcing the presenting part through incompletely di lated birth canal, causing mate rnal and foeta l soft tissue inju ry, rupture of uterus, foetal asphyxia and death. • Water intoxication: T his occurs due to A DH like action of large doses given along with i.v. fluids, especially in toxaemia of pregnancy and renal insuffici ency. It is a serious (may be fatal) complication. Desamino-oxytocin

II has been developed as a buccal formulation. The action is similar to injected oxytocin, but less consistent. Its indications are:

OXYTOCIN AND OTHER DRUGS ACTING ON UTERUS Induction of labour: 50 JU buccal tablet repeated every 30 min, ma., IO tabs. Uterine incnia: 25 IU e, ery 30 min. Promotion or uterine involution 25- 50 IU 5 times daily for 7day&. Breast engorgement 25- 50 IU just before breast feeding. BUCTOCIN 50 IL tab.

Carbetocln It is a long-acting analogue of oxytocin that has been introduced recently for prevention of uterine atony aficr caesarean section and to control PPI I.

ERGOMETRINE, METHYLERGOMETRINE The phannacology of ergot alkaloids is described in Ch. 12. Only the amine ergot alkaloid ergometrine (ergonovine) and its derivative methylergometrine arc used in obstetrics. Both have sim ilar pharmacological property.

1. Uterus These alkaloids increase force, frequency and duration of uterine contractions. At low doses, contractions are phasic w ith nom,al relaxati on in between, but only moderate increase in dose raises the basal tone, contracture occurs with high doses. Grav id uterus is more sensitive. Marked increase in sensitivity occurs at term and in early puerperium. Thei r stimulant action involves the lower segment of uterus as we ll. The ute roton ic action is be lieved to resu lt from partial agonistic action on 5-HT2 and a ad rcnergic receptors. 2 . CVS Ergometri nc and methylergomctrine are much weaker vasoconstrictors than ergotamine and have low propensity to cause e ndothel ial damage. Though they can raise BP, this is not significant at doses used in obstetrics.

3 . CNS

o overt effects occ ur al usua l doses. However, high doses produce complex acti ons- partial agon istic/antagonistic interaction with adrenergic, serotonergic and dopaminergic receptors in the brain have been shown. 4 . GIT High doses can increase peristalsis.

Methylergometrine is I½ times more potent than ergometrinc on the uterus, but other actions are less marked. It has thus replaced ergometrine at many obstetric units.

Pharmacokinetics In contrast to the a mino acid ergot alka loids, ergometrine and methy l

ergometrine are rapid ly and nearly complete ly absorbed from the oral route. The onset of uterine action is : Oral- 15 min; i.m.- 5 min; i.v.- almost immediate. They are partly me tabo li zed in live r and excreted in uri ne. Plasma t½ is 1-2 hours. Effects of a single dose last 3 4 hours.

Adverse effects Ergornetri ne and mcthylergomctrine are less toxic than ergotam ine. When correctly used in obstetrics-hardly any com plications arise. Nausea, vomiting and ri se in BP occur occasionally. They can decrease milk secretion if higher doses are used fo r many days postpartum. This is due to inhibition of prolactin release (dopaminergic action). Ergometrine should be avoided in• patients with vascular disease, hypertension, toxaemia. • presence of sepsis- they may cause gangrene. • liver a nd kidney disease. They are contra indicated during pregnancy and before 3rd stage of labou r. Use I. The primary indication for ergometrine/ methy lergometrinc is to control and prevent postpa rtum haemorrhage ( PPH ): 0.2- 0.3 mg i.m. at delivery of a nterior shoulder redu ces blood loss attending delivery a nd preven ts PPH. However, routi ne use in a ll cases is not justi fied. They should be employed only in patie nts pro ne to bleed more, e.g. grand multipara, uterine inertia. Multiple pregnancy should be excluded before injecting. If PPH is occurri ng- 0.5 mg i.v. is recommended. A combination of 0.5 mg ergometrinc with oxytocin 5 IU i.m./i.v. may be used in severe bleeding. These drugs produce sustained tonic uterine contract ion: perforati ng uterine arte ri es are compressed by the myometria l meshworkbleeding stops. 2. After caesarean section/instrumental delivery these alkaloids serve to prevent uterine atony. 3. To ensu re normal invo lution: A firm and active uterus involutes rapidly. To ensure this:

357

358

HORMONES AND RELATED DRUGS 0. 125 mg of ergometrine or methylergometrine has been g iven TDS orally for 7 days. However, routi ne use in a ll cases is not j ustified because normal in volution is not hastened. Mu ltipara and others in whom slow involut ion is apprehended, these drugs may be given prophylactically. 4. Diagnosis of variant angina: A small dose of ergornetrinc injected i.v. during coronary angiography causes prompt constriction of reactive segments of coronary artery tha1 are responsible for variant angina. ERGOMETRI E 0.25. 0.5 mg tab. 0.5 mg/ml inj. Methylergometrinc: M[TH ERG IN, \ii ETH ERONF, ERGOMET 0.125 mg tab, 0.2 mglml inj .

PROSTAG LANDI NS PGE2, PGF 2a and LS-methyl PGF2... are potent uterine stimulants, especially in the later part of pregnancy. They also promote ripening of cervix. Thei r actions and use in obstetrics is described in C h. 13. Since misoprostol (a PG analogue) produces less side effects, it is being used for obstetric indications as well. Ethacridine Available as 50 mg/50 ml solut ion (E"1CREDIL. VECRI:DIL) for extra-amniotic infusion : 150 ml (containing 150 mg) is injec1cd slowly for medical tennination of pregnancy in the 2nd trimester. T his is an alternative method used occasionally.

UTERINE RELAXANTS (Tocolytics) T hese are drugs which decrease uteri ne motility. They have been used to de lay or postpone labour, arrest threatened a bortion and in dysmenorrhoea. Preventi on of prematu re labour in those at higher risk due to past history or short cervix has been attempted by administration of high dose progesterone in the later half of pregnancy, with some success demonstrated in prospective controlled

tria ls. Suppression of pre mature labour may be needed to allow the foetus to mature, to allow time for initiating glucocorti coid therapy for foetal lung maturation (see p. 3 16) or to transfer the mother in labour to a centre with prope r fac ili ties. However, no clearly satisfactory drug is avai lable si nce none of them has been shown to improve foetal outcome. An attempt to delay premature labour is likely to succeed onl y if cervical d ilatation is < 4 cm and 'taking up' of lower segment is min imal. Measures to delay labou r should not be undertaken i r membranes have ruptured, antepartum haemorrhage is occurri ng, in severe toxaemia of pregnancy, in trauterine in fectio n or foeta l death.

1. Adrenergic agonists (see Ch. 9) Ritodrine, the p2 selec tive agonist having more prom inent uterine relaxant action is approved to suppress p remature labou r a nd to delay de li very in case of some exigency or acute foetal distress. For dependable action it is started as 50 ~tg/m in i.v. infusion, the rate is increased every IO min ti II uterine contractions cease or maternal HR rises to J 20/min. Contractions are kept suppressed by continuing i.v. infusion or by IO mg i. m. 4- 6 hourly fo llowed by IO mg oral 4-6 hourly. However, treatment beyond 48 hours is not recommended, since risk to mother increases and benefit is uncertain. Del ivery can be postponed in about 70% cases by few hours to few weeks. However, cardiovascular (hypotension, tachycard ia, arrhythm ia, pulmona ry ede ma) and metabolic (hyperglycaemi a, hyperinsulinacmia, hypokalaemia) complication and anxiety, restlessness, headac he occur frequently. Use of ritodrine to arrest labour has been found to increase matern al morbidity.

UTERINE RELAXANTS (Tocolytlcs)

Adrenergic agonlsts

Calcium channel blockers

Oxytocin antagonist

Miscellaneous drugs

Ritodrine Salbutamol Terbutaline lsoxsuprine

Nifedipine (Other dihydropyridines)

Atosiban

Magnesium sulfate Proges tero ne

Nitrates Halothane

OXYTOCIN AND OTHER DRUGS ACTI NG ON UTER US Foetal pulmonary edema can develop; vol ume of i.v. infusion should be kept to a minimum to avoid fluid overload. The neonate may develop hypoglycaemia and i leus. It should not be used if mother is diabetic, havi ng heart disease, or receiving blockers or steroids. Ritodrine has been discontinued in the USA, but is still avai la ble in UK and India. YUTOf'AR, RITROD 10 mg/ml inJ (5 ml amp). 10 mg tab. RITODl'I\E 10 mg tab, IO mg m I ml rnj.

Salbutamol and terbutal ine can be used as alternatives to ritodri ne. lsoxsuprine oral/i.m. has been used to stop threatened abortion. but efficacy is uncertain.

2. Calcium channel blockers

Because influx of Ca2 ions plays an important role in uterine contractions, Ca2 channel blockers (see Ch. 40) reduce the tone of myometrium and oppose contractions. T hese drugs, especially nifedipine, which has prominent smooth muscle relaxant action, can postpone labour i f used early enough. Efficacy comparable to p2 adrenergic agonists has been demonstrated and side effects are few er. Oral nifedipine IO mg repeated once or twice after 20 30 min, followed by IO mg 6 hourly has been used. Tachycardia

r:ir

and hypotension are prominent at doses whi ch suppress uterine contractions. Reduced placental perfusion causing foetal hypoxia is apprehended. However, fewer babies delivered after nifedipine needed intensive care. 3. Oxytocin antagonist

Atosiban is a peptide analogue of oxytocin that acts us antagonist at the oxytocin receptors. In clinica l tria ls. i.v. infusion o f atosiban has been fo und to suppress pre mature ute rin e contract ions and postpone pretcrm deli very with fewer cardiovascular and metabolic complications than ~' adrencrgic agonists. In Europe and UK it is a\'ailable to'r inhibition of labour between 24-33 week; o f gestation. and may offe r bette r benefit: risk ratio than other tocolytics. Howe, e r. it is not approved in USA and India. 4. Magnesium sulfate In fused i.v. it i, a first line drug for pre, ention and treatment o f seizures in preeclampsia and cclampsia. It a lso acts as a tocolytic by competing with Ca'· ions for entry into myometrium through both voltage se nsitive as well as ligand gated Ca' · channels. However, its use to delay premature labour is risky. may increase perinata l mortality and is not recommended now.

5. Miscellaneous drugs

Ethyl alcohol. nitrates, progesterone, gene ral anaesthetics and indomethacin (PG synthes is inhibitors) arc the othe r drugs, which can depress ute rine contractions. However, their effect is not depe ndable and they are not used clinically as toeo lytic;. Halothanc is an c flicacious uterine relaxant that has been used as the a nae thet ic w hen external or inte rna l version is atte mpted.

PROBLEM DIRECTED STUDY

23.1 A full term primigravida aged 26 years is brought to the hospital with the complaint of having labour pain s for the past 24 hours without making much progress. Two hours ago she had passed meconium stained liquor. The lady is in dist ress, mildly dehydrated and looks exhausted. The presentati on is vertex and head is engaged, but cervix is incompletely dilated and uterine contractions are relatively weak. Foetal tachycardia is noted with irregularity during contractions. (a) What course of action is appropriat e? (b) Can she be administered a uterin e stimulant to strengthen the contractions? If yes, which drug should be given and how? If no, then w hy? (see Appendix-1 for solution)

359

Chapter

24

Hormones and Drugs Affecting Calcium Balance

CALCIUM After C , 0 , H a nd , calc ium is the most abundant body constitue nt, ma king up abo ut 2% of body weight, or 1- 1.5 kg in an adu lt. About 98% of this is stored in bones, the rest be ing distributed in plasma and all tissues and cells . Calc ium serves important physio logical roles in several cell ular functions.

Physiological roles I. Calcium controls excitabi lity of nerves and muscles a nd regulates permeability o f c e ll membranes. It also maintains integrity of cell membranes and regulates ce ll adhesion. 2. Ca2+ ions are essential for exc itation -contraction coupling in a ll types of muscle and excitationsecretion coupling in exocrine and endocrine g lands, re lease of transmitters from nerve ending and other release reactions. 3. Cai+ is an in trace llu lar messenge r for hormones, autacoids and transmitte rs. 4. Ca 2+ controles impulse ge ne ration in heart; determines le vel of auto maticity and A-V conduction. 5. Ca 2- is essential for coagulati on of blood. 6. Calcium serves structural function in bone and teeth.

Plasma calcium level It is precisely regulated by 3 hormones a lmost exclusivel y devoted to this function, viz. parathormone (PTH), calcitonin and calcitriol (acti ve fo rm of vit D). These regulato rs control its intestinal absorption, exchange with bone a nd renal excretion as summa rized in Fig. 24. 1. ln addition, se veral other hormones, viz.

Influences affecting bone turnover

t

Resorption

Corticosteroids Parathormone

.J, Resorption Androgens/Estrogens Calcitonin

Thyroxine (excess)

Growth hormone

Hypervitaminosis D

Bisphosphonates Fluoride

Prostaglandin E2 Interleukin 1 & 6 Alcoholism Loop diuretics

Gallium nitrate Mithramycin Thiazide diuretics

gl uc ocorticoids, sex stero ids, thyroid honnone, growth hormone affect calcium homeostasis. and can be considered its secondary regu lators. Man y other sig nal mo lec ules a nd drugs a lso influence calcium handling (see box). N o rma l plas ma c a lci um is 9- 11 mg/ d i. Of th is a bout 40% is bound to plasma proteins- chiefly to albumin; I 0% is complexed with citrate, phos phate and c arbonate in an undi ssociable form; the remaining (about 50%) is ionized and physiologically important. For example, in hy poa lbuminemia, tota l plasma calcium may be low but the concentration of Ca2+ ion is usual ly normal. Acidosis fa vours and a lkalosis disfavours ionization of calc ium. As such, hyperventi lation (by rai sing plasma pH) precipitates tetany and laryngospasm in calcium defic iency by reduc ing ion ization. Calcium turnover Major fraction of calcium in the bone is stored as crystalline hydroxyapatite deposited on the organic bone matrix osleoid, while a small labile pool is in dynamic equilibrium with plasma. Even the full y laid down parts of the bone under 0. 7.

No drug

Pancuronium (Competitive block}

(TOF-R= 1 .0) Train of four protocol (TOF-R=0.5) Fade

Succiny1chollne (Depolarizing block: phase I)

(TOF-R=1 .0) No fade

T, T2

T3 ,,,, T4

:

Double-burst stimulation protocol

Fade

No fade

-7.:-

Fig. 25.3: Clinical assessment of neuromuscular block. (A) Train-of-four (TOF) protocol: Contractile responses of adductor pollicis muscle to transcutaneous ulnar nerve stimulation with train-of-four protocol of impulses during recovery of neuromuscular block. TOF-R- Train of four ratio (strength of 4th contraction divided by that of the 1st). (B) Double-burst stimulation (DBS:r ) : Evoked responses to burst o f three 0.2 ms pulses at 50 Hz followed 750 ms later by a second burst of two s imilar pulses. Note 'fade' in the second burst after nondepolarizing block.

On the other hand, classical or phase-I dcpolari7ing block does not exhibit fade; the TOF-ratio remains 1.0. though all the 4 twitches are depressed equally depending on the degree o f block. Fade is again seen when phase II or d~sensiti.G.1tion block occurs with prolonged use or a depolari1ing agent and TOF-ratio is depressed as in the case of compet itive block. However, C h generally requires no monitoring. Rather than measuring each contraction and calculating TOF ratio. in practice. it is easier to simply observe the disappearance (during onset) or reappearance (during reco, ery) of the successive twitches. Reappearance of 2nd t,, itch (T,) corresponds to - I 0% recovery (- 90% residual block) and that of 4th twitch (T4 ) to - 25% recovery. Because fade is more prominent during s usta ined stimulation. an alternate method is ' tetanic st imulation ' protocol, in which 0.2 ms pulses are applied at 50 100 Hz for 4 5 seconds and presence or absence of fade is noted (see fade in Fig. 25.2A). Many anaesthesiologists prefer to use the lc:ss painful , ariant of tetanic stimulation. l'i= 'double-burst stimulation· (DBS). A burst of three 0.2 ms pulses at 50 Hz is fo llm, ed aner a gap of 750 ms by a second burst o f 2 or 3 s unilar pulses (Fig. 25.3 B). The s trength of response during the 2nd burst relative to the first is a measure of the recovery from block. Measurement of ·post-tctanic count (PTC)' is another clinically used method.

2. Autonomic ganglia Because the cholinergic receptors in autonomic ga nglia are nicotinic (though of a different subclass )\), competitive neuromuscular blockers produce some degree of ganglionic blockade; d-TC has the maximum propensity in this regard, while the newer drugs (vecuronium, etc.) are practically devoid of it. SCh may cause ganglionic stimulation by its agonistic action on ganglionic nicotinic receptors. 3. Histamine release d-TC releases histamine from mast cells. This does not involve immune system and is due to the bulky cationic nature of the molecule. Histamine release contributes to the hypotension produced by d-TC. Flushing, bronchospasm and increased respiratory secretions are the other efTccts. lntradermal injection or d-TC produces a wheal similar to that produced by injecting histamine. Atracurium and mivacurium have significant histamine releasing potential. Histamine releasing polential of neuromuscular blockers is graded in Table 25.2. Heparin may also be simultaneously released from mast cells.

377

DRUGS ACTING ON PERIPHERAL (SOMATIC) NERVOUS SYSTEM

378

Table 25.2: Comparative properties of neuromuscular blocking drugs Drug

Dose£ (mg/kg)

Onset· (min)

Duration• (min)

Hist. release

Gang. block

Vagal block

0.2--0.4 0.04--0.1 0.03--0.08 0.05--0.08

4-6 4-6

40--60 60-120 60-120 50-100

+++ ±

++

±

±

+

LONG ACTING

1. 2. 3. 4.

d-Tubocurarine Pancuronium Doxacurium Pipecuronium

4-8 2-4

+ ±

INTERMEDIATE ACTING

5. Vecuronium 6. Atracurium 7. Cisatracurium

8. Rocuronium

0.08--0.1 0.3--0.6 0.15--0.2 0.6--0.9

2-4 2- 4 1- 2

30-50 20-40 25-50 25-40

0.15--0.2 0.5--0.8

2-4 1-1 .5

15-30 5-8

3-6

±

±

+ ±

SHORT ACTING

9. Mivacurium 1o. Succinylcholine

+ ++

St.

St.

Initial paralysing dose for opioid/nitrous oxide+oxygen anaesthesia. In patients anaesthetised with ether/halothane/ isoflurane, the dose may be 1/ 3- 1/ 2 01 the figure given. • Time to maximal block after i.v. injection. • Duration of surgical grade relaxation after usual clinical doses; time to 95% recovery of muscle twitch is nearly double of the figure given (especially for long-acting drugs). Duration is dose dependent as well. St-Stimulation

t

4.

c.v.s.

d-Tubocurarine produces significant fall in BP. This is due to(a) gang lionic blockade (b) hista mine release and (c) reduced venous return- a result of paralysis of limb and respiratory muscles. Heart rate may increase due to vagal ganglionic blockade. Pancuronium tends to cause tachycardia and ri se in BP, whil e at racurium may cause hypotension. All newer nondepolarizing drugs viz. vecuronium, rocurorinium and cisatracurium have negligible effects on BP and HR. Cardiovascular elTects of SCh are variable. Generall y bradycardia occurs initia lly due to activation of vagal gang lia followed by tachycardia and ri se in BP due Lo stim ul a tion of sympathetic ganglia. BP occasionall y fa lls on accou nt of its muscarin ic action causi ng vasodi latation. Prolonged administration of SCh has caused cardiac arrhythmias and even arrest in pa tients w ith burns, soft ti ssue injury and tetanus. Efflux of inlrncellular K occurs in these conditi ons which is augmented by prolonged depolarizat ion of skeletal muscles.

5. G.I.T.

The ganglion blocking act ivi ty of competitive blockers may enhance postoperative paralytic ileus after abdominal operations.

6 . C.N.S.

All neuromuscular blockers are quaternary compound - do not cross blood-brain barrier. T hus, on i.v. adm inistration no central effects fo llow. However, d-TC applied to brain cortex o r injected in th e cerebral ventri c les produces strychn ine li ke effects.

PHARMACOKINETICS All neuromuscular blockers are polar quaternary compounds; therefore not absorbed orally, do not cross cell membra nes, have low volumes of distribution and do not penetrate placenta l or blood-brain barrier. They are practically always given i.v., though i.m. administration is possible. Muscles with higher blood flow receive more drug and are affected earlier. Redistribution to non- muscular tissues plays a significant ro le in the termination of surg ical grade muscle relaxation, but residual block may persist fo r a longer time depending on the elimination t½. The duration of action of competitive blockers is

379

SK ELETAL MUSCLE RELAXANTS directly depende nt on the elim ination t ½. Drugs that are primarily metabolized in the plasma/ liver, e.g. vecuronium, atracu rium , cisatracurium , roc uronium, and especially mi vacurium have re latively shorter t ½ and d ura tio n of acti on (20-40 min), while those largely excreted by the kidney, e.g. pancuronium. d-Tc, doxacurium and pipecuronium have longer t ½ a nd duration of action (>60 min). With repeated adm inistration. redistribution sites are filled up and duration of acti on is prolonged. The unchanged drug is excreted in urine as we ll as in bile. SCh is rapidly hydro lysed by plasma pseudocholinesterase to succinylmonocholine and then succinic acid + choli ne (action lasts 5- 8 min). Some patients have genetically de termined abnormality (low affinity for SCh) or deficiency of pseudocholi nesterase. In su bjects who are ho mozygous fo r the abno rm al e nzyme ( I in > 3000 population), SC h causes prolo nged phase 11 blockade resulting in muscle paralys is and apnoea lasting 4- 6 hour , beca use SCh is a poor substrate for the more specific AC hE fou nd at the motor e nd plate. However, duratio n o f paralysis is increased only by 2- 3 times in s ubjects who are heterozygous for the abno rma l enzyme ( 1 in - 50), or have only relati ve defi c iency. The prolo nged apnoea can be tided over o nly by mechanical ventilation.

NOTES ON IND IVIDUAL COMPOUNDS 1. d-Tubocurarine

Because of its prominent histam ine releasing, ganglion blocking and card iovasc ular actions as wdl as long duration of paralysis needi ng pharmacological reversal, d-TC is not used now.

2. Succinylcholine

Despite its propens ity to cause muscle fascicu lations a nd soreness, changes in BP a nd HR, arrhythmias, histamine release and K efflux from muscles caus ing hypcrkalaemia and its complicatio ns, SCh is the most commonly used muscle relaxant for passing trac heal tube. It induces rapid, complete and pred ictab le paralysis wi th spo ntaneo us recovery in ~5 min. Excellent intubating condition viz. re laxed jaw, vocal cords apart and im mobile with no diaphrag matic movements, is obtained

w ithin 1- 1.5 min. Occasionall y SCh is used by continuous i.v. infusion for producing contro lled muscle relaxati on of longer duration. lt shou ld be avoided in younger children unless absolutel y necessary, because risk of hyperka lacm ia and cardiac arrhythmia is higher. Risk of regurgitation and aspiration of gastric contents is increased by SCh in GERO patients and in the obese, especially if stomach is full. MIDARINE, SCOLIN E, MYORELEX, E 'T U BATE 50 mg/ml inj. 2 ml amp.

3. Pancuronium A synthetic stero ida l compound, ~5 times more potent and longer acting than d-TC, which provides good cardiovascular stability (little ganglio nic blockad e). seldo m induces flushing, bronchospasm or cardi ac arrhythm ias becau e of lower histamine relca ing potential. Rapid i. v. injection of panc uronium ofte n causes rise in BP and tachycardia due to vagal blockade and A release. It is primarily e l iminatcd by renal excretio n. Because of longer d uration of ac tion, needing reversal, its use is now restricted to prolonged operations, especially neurosurgery. PAVULON, PANURON.

EOCURON 2 mg/ml in 2 ml amp.

4. Doxacurium A bisquatemary mu clc relaxant having the least rapid onset and the longest ac tion. h is , uitable for long duration surgeries. ll is primarily eliminated by kidney, though he patic metabolism a lso occurs. Cardiovascular c hanges are less marked.

5. Pipecuronium Another steroidal muscle re laxant w ith a slo w onset and long duration of action: whi ch is recommended for prolonged surgeries. It exerts li ttle cardiovascular action, though trans ient hypo tension and bradycardia can occu r. Elimination occurs throug h both kidney and li ver. ARDUAN 4 mg/2 ml inj.

6. Vecuronium

A close congener of pancuronium w ith a shorter duration of action due to rapid d istribution and conside rable hepati c me tabolis m . It is excre ted mainly in bi le, recovery is generally s po ntaneous, but may need neostigm ine reversal. Cardiovascular stab ii ity is still better due to mild hista min e releasing and ganglion ic action; tachycardi a sometimes occurs. Currently, it is one of the most

380

DRUGS ACTING ON PERIPHERAL (SOMATIC) NERVOUS SYSTEM com monly used muscle re lax ant for rou tin e surgery and in intensive care units. NORCURON 4 mg amp. dissolve in I ml soh·ent supplied. ' EOVEC 4 mg amp, 10 mg vial.

7. Atracurium

A bisquatem ary competitive blocker, 4 times less potent than panc uronium and shorter acting, so that reversal is mostly not required. The unique feature of atracurium is inactivation in plasma by spontaneous nonenzymatic degradation ( Hofmann e limination) in addition to that by cholinesterases. Consequently its duration of action is not a ltered in patients with hepatic/rena l insuffic iency or hypodynamic circulation. Atracurium causes dose dependent histam ine re lease; hypotension can occur. When used for long pe riods (e.g. to faci litate positive pressure ventilation in LC.Us). one of its meta bolites may accumul ate and cause C S toxicity. Avail ability of less toxic cisatracurium has sent racemic atracurium in the background. TRAC RIUM 10 mg/ml inj in 2 ml vial.

8. Cisatracurium

This R-Cis, R-Cis enantiomer o f atracurium is nearly 4 times more potent, slower in onset, but similar in duration of action. Like atracurium it undergoes Hofmann eliminati on, but in contrast to the racemate, it is not hydroly ed by plasma cholinesterase and he patic metabolism is limited, so that less o f the tox ic metabo lite is fom1ed. Most importantly, it does not provoke histamine re lease and side effects are fewer. Currently, cisatracurium is one of the preferred muscle re laxants, especially for li ver/kidney disease patients and for the e lderl y. NIM BEX 2 mg ml mj.

9. Rocuronium

A newer nonde pola ri zing blocker wi th the most rapid onset and intermediate durati on o f action whi ch ca n be used as a lternative to SC h for trachea l intubation without the disadvantages o f depolariz ing block and cardi ovascular changes. T he same drug also serves as maintenance muscle re laxant, seldom needing reversal. The onset of acti on is dosedependenl; intubati ng conditions arc alla ined in 90 sec w ith 0.6 mg/kg, but within 60 sec al 1.0 mg/kg. Within limits, the duration of para lysis is al so dose-de pendent. This neu romuscular blocker is gaining popularity for its versati lity

and more precisely timed onset a nd duration of acti on. Infused i.v. (0.3- 0.6 mg/kg/ ho ur), it is also be ing used to fac ilitate mechanic al venti lation in intensive care units. Though little meta bolized, it is e liminated ma inl y in bile. Mild vagolytic action increases HR somewhat. RO('UNIUM, CUROMID 50 mg/5 ml, 100 mg/10 ml vials.

10. Mivacurium

ll is the shortest acting compet itive blocker; does not need reversal. Dose and speed of injection related transient cutaneous fl ushing can occur due to histamine release. Fall in BP is possible, but change in HR is minimal. It is metaboliad rapidly by plasma cholinestcrascs. Prolonged paralysis can occur in pseudocholinestcrase deficiency. but this can be reversed by neostigmine (unlike paralysis due to SCh which cannot be reversed).

INTERACTIONS I. Thiopentone sod and SCh solutions should not be m ixed in t he same syringe- re a ct chemically. 2 . General anaesthetics po tentiate compe titi ve blockers; ether in particular, fol lowed by fluorinated hydrocarbons of w hi ch isoflurane is most potentiating and halothane the least. Desflurane and sevofluranc a re intermediate. Nitrous oxide potentiates the least. Ketamine also intensifies nondepolarizing block. F luorinated anaesthetics predisp ose to phase II blockade by SCh. Malignant hyperihermi a produced by halothane and isofluranc in rare (gene tica ll y predisposed) indiv iduals is more common in patients receiving SCh as we ll. 3. An1icho/inesterases reverse the action of competi tive blockers. Neosti gmine 0.5 2 mg (30- 50 µg/ kg) i. v. is a lmost routin e ly used a fter pancuronium and other long/intermedi ate acting blockers to hasten recovery at the end of o perati o n. It is importa nt to reve rse the residu a l b lock , beca use re s id ual wea kn ess can ca use hypo ve ntil a ti on a nd hy pox ia in the recovery room . Though neostigmine also reverses gang lion ic blockade to some extent, hypotension and bronchospasm can occur due to muscarinic action or neostigmine; this can be prevented by prior atropinization (atropine or glycopyrrolate 5- 10 µ g/kg i.v.). Pretreatme nt with ll 1 antihista mines reduces hypote nsion due to histami ne re leasing neuromuscular blockers.

SKELETAL MUSCLE RELAXANTS 4. Antibiotics Aminoglycoside antibiotics reduce ACh release from prejunctional nerve endings by competing with Ca 2+ . They interfere with mobilization of ACh containing vesicles from a central location to near the terminal membrane, and have a weak stabili7ing action on the postjunctional membrane. In clinica lly used doses, they do not by themselves produce muscle relaxat ion, but potentiate competitive blockers. The dose of competitive blocker should be reduced in patients receiving high doses of these antibiotics. Application of streptomycin powder locally at the end of bowel surge,y has caused prolonged apnoea if a competitive blocker had been used during the operation. Tetracyclines (by chelating Ca2- ). polypeptide antibiotics, clindamycin and lincomyc in also synergise with competitive blockers. 5. Calcium channel blockers Verapamil and others potcntiate both competitive and depolarizi ng neuromuscular blockers. 6. Diuretics may produce hypokalemia which enhances competitive block. 7. Dia:::epam, propranolol and quinidine intensify competitive block, while high dose of corticostero ids reduces it. Sugamadex This is a novel reversing agent developed for terminating th~ act ion or nondepolarizing muscle relaxants rocuronium and vecuronium. Sugamadex is a modified y-cyclodextrin with hig h affinity for rocuronium and vecuronium. It encapsulates one molecule of the blocker within its molecule fom1ing an inactive chelate which is excrt!led in urine with a t½ of - 2 hours. As the plasma concentration o f free rocuronium falls, it rapidly dissociates from the Nm receptor and neuromuscular trans mission is restored. Thus, the mechanism of re, ersal by sugamade!I. is entirely diflerent from that of the currently used reversing agents ncostigminc and cdrophonium. Sugamadcx 2-4 mg/kg i.v. reverses roc uron ium block within 3 m in. in majority of patients. Its side cflccts arc mild prccordial pain. nausea, alteration of ta,te and rarely allergy. No cardiovascular effects have been noted.

TOX IC ITY I. Respiratory paralys is and prolonged apnoea is the most important complication of neuromuscular blockers. 2. Flushing (due to histamine release) is common with d-TC, can occu r with atracurium and mivacuriu m, rare with others.

3. Fall in BP and cardiovascular collapse can occur, especially in hypovolemic patients. This is less I ikely with the newer drugs. Muscle relaxants should be used with great caution in patients with severe hepatic and renal disease. 4. Cardiac arrhyth mias and even arrest have occu rred, especially wi th SCh, particularl y in digital ized patients, because SCh releases K• from muscles. Intubating dose generally raises sernm K• by 0.5 mEq/ L, but dangerous hyperkalemia can occur, especially in patients with extensive burns, nerve damage and soft tissue injuries. 5. Precipitation of asth ma by histamine releasing neuromuscular blockers. 6. Pos toperative muscle soreness and myalgia may be complained after SCh, particularly when larger dose is used. 7. Malignant hypcrthcrmia can be triggered by SCh in patients anaesthetized with fluorinated anaesthetics.

USES I . The most important use of neuromuscular blockers is as adjuvants to general anaesthesia. By the use or these drugs, adequate muscle relaxation can be achieved at lighter planes. Many surgical procedures are performed more safely and rapidly by employing muscle relaxants. They also reduce reflex muscle contraction in the region undergoing surgery, and faci litate maintenance of controlled ventilation during anaesthesia. They are particularly helpful in abdom inal and thoracic surgery, intubation and endoscopies, orthopedic manipulations, etc. Choice of th e neuromusc ular blocker depends on the nature and duration of the procedure, pharmacokinetics o f the blocker and ca rdiovascu lar stability that it provides. Vecuronium, cisatracurium and rocuronium are the most frequentl y selected nondepolari zing blockers. SCh is em ployed for brief procedures, e.g. endotracheal intubation, laryngoscopy, bronchoscopy, esophagoscopy, reduction of fracrures.

381

382

DRUGS ACTI NG ON PERIPHERAL (SO MATI C) NERVOUS SYSTEM d islocations, and to treat lary ngos pasm . For ocular surgery competitive blockers are preferred, beca use they paralyse ex traocula r m uscles at doses which have little effect on larger muscles. Other fac to rs w hich shou ld be considered in selecting the re laxant are-onse t o f action, d ura tion o r bloc kade requ ired, c ard iovascular effects of the drug as wel l as patien t's hepatic, re nal and haemodynamic status. Advantages of newer neuromuscular blockers over the older ones • No o r minimal gang lionic, cardiac or vascular effects. • No or minimal histamine release. • Many a re s hort a cting: easy reversal. • Some are rapid acting: provide alternative to SCh without the attendant complications.

2. Assisted ve ntil ation: C riti cally ill patients in intensive ca re un its o fte n need ventilatory support. This can be fac ilitated by cont inuous infusion of subpara lys ing doses of a co mpetitive neurom uscular blocker which reduces the c hest wall resistance to inflation. Vec uronium is most commonly used, but after pro longed infus ion it can cause blockade lasti ng 1- 3 days due to accumulation of a n active metabolite and/ or deve lopment o f neu ropathy. 3. Convu ls ions and trauma fro m e lectroconvu lsive therapy can be avoided by the use o f muscle re laxants w ithout decreasing the therapeutic benefit. SCh is most commo nly used for this purpose. The short acting competitive blocker mivacurium is a n alte rnative. 4. Severe cases of tetanus and status epi lepticus, w ho are not contro lled by diazepam or other drugs, may be pa ra lysed by a neuromusc ular blocker (repeated doses of a competi tive blocker) and ma inta ined on intermittent posit ive pressure respirati on til l the disease subsides. DIRECTLY ACTING MUSCLE RELAXANTS

Dantrolene This muscle relaxant is chemically and phamrncologically entirely d ifferent from neuromuscular blockers. Its effects superficially resemble those of centrally acting muscle relaxants. Neuromuscular transmission or MAP arc not affected, but muscle contraction is uncoupled

from depolari.aition of the membrane. Dantrolene acts on the RyR I (Ryanodinc Receptor) calcium channels in the sarco1i lasmic reticulum of skeletal muscles and prevents ca=· induced Ca'' release through these channels. Intracellular release or Ca'· needed for excitation-contract ion coupling is interfered with. Fast contracting ·twitch ' muscles are affected more than slow contracting 'antigrnvi ty' muscles. Since Ca' channels in the sarcoplasmic reticulu m o f cardiac and smooth muscles arc of a d iflcrent subtype ( Ry R2), these muscles are affected lillle by dantrolene. Dantrolene is slowly and incompletely absorbed from the g.i.t. It penetrates brain and produces some sedation. but has no selective effect on polysynaptie reflexes responsible for spasticily. It is metabolized in the liver and excreted by kidney with a t½ of 8- 12 hours. Used ora lly dantrolene (2S- 100 mg Q ID) reduces spasticity in upper motor neurone disorders, hemiplcgia, paraplegia, cerebral pal y and multiple sclerosis. HO\, ever. it also reduces voluntary po" er: the resulting weakness considerably neutrali1cs t he benefit a nd limits use to bedridden patients. Used i.,. ( I mg/kg repeated as required) it is the drug of choice for malignant hyperthermia which is due to persistent release of Ca2' from sarcoplasmic reticulum (induced by fl uorinated anaesthetics and SCh in genetically susceptible indi viduals w ith abnormal RyR I, see p. 406). Reversal has a lso been obtained in neuroleptic malignant syndrome, though this reaction has a di fferent pathogenesis.

Adverse effects Muscu lar weakness is the dose limit ing side effect. Sedation, ma laise, light headedness and other cemral effects occur. but ar~ less pronounced than with centrally acting muscle relaxants. Troublesome diarrhoea is another problem. Long term use causes dose dependent serious liver toxicity in 0.1-0.S% patients. Use in chronic disorders is therefore restricted. Quinine (see Ch. 6 1) I! increases refractory period and decreases cxcnability of motor end plates. Thus. responses to repetitive nerve stimulation arc reduced. It decreases muscle tone in myotonia cong.:nita. Taken at bed time (200-300 mg) it may abolish noclumal leg cramps m some patients.

CENTRALLY ACTING MUSCLE RELAXANTS These arc drugs w hich reduce ske leta l muscle tone a nd lessen muscle spasm by a se lective action in the eerebrospi na l ax is, without altering consciousness. T hey selective ly depress spi nal and supraspinal polysynaptic reflexes involved in the regu lation o f muscle tone without significa ntly affecting monosynaplically mediated stretch reflex. Polysynaptic pathways in the ascending reticular formation which are involved in the ma intenance of wakefullness are a lso depressed, though to a

383

SKE LETAL MUSCLE RELAXANTS CENTRALLY ACTING SKELETAL MUSCLE RELAXANTS

Mephenesin congeners

Carisoprodol Chlorzoxazone Chlormezanone Methocarbamol

Benzodlazeplnes

GABA mimetic

Central a 2 agonlst

Dia.tepam, etc.

Baclofen Thiocolchicoside

Tizanidine

lesser extent. Thus, all centra ll y acting muscle relaxants do have some sedative property. They have no effect on neuromuscular transmission and on muscle fibres, but reduce dccerebrate ri gidity, upper motor neurone spasticity and hyperreflexia. The prominent differences between neuromuscular blockers and centrally act ing muscle relaxants are listed in Table 25.3. It was the first drug found to cause muscle relaxation in animal, wi1hou1 producing unconsciou~ness and was called intermmcial neurone blocking agent because its primary site of action is the spinal intemuncial neurone which modulates reflex es mai ntaining muscle tone. It is 1101 used clinically beca use orally it causes marked gastric irritation. and injected i.v., it causes thrombophlebitis, haemolysis and fa ll in BP. It has been included in coun1erirri1an1 oi111ments (MED/CREME. RELAXYL) "here its irritant rather than muscle relaxant property could be afford ing relief. 1. Mephenesin

2. Carisoprodol

It has a favourable muscle

relaxant: sedative acti vity ratio with weak analges ic, antipyretic and anti choli nergic properties. It is used in tissue injury, mu c lc sprains and disorders associated with local muscle spasm. CARISOMA 350 mg tab; one tab. TD -QID. SOMAFLAM 175 mg + ibuprofen 400 mg tab.

3. Chlorzoxazone

It is pharmacologically similar to mephenesin, but has a longer duration of action and better oral tolerance.

FLEXO -MR 250 mg + ibuprofen 400 mg + paracetamol 325 mg tab; ULTRAZOX 250 mg + diclofcnac 50 mg + paracetamol 325 mg tab: MOOIZOX 500 mg +- diclofenac 50 mg + paracetamol 500 mg tab; PARAFON: 250 mg + pamcetamol 300 mg tab, I 2 tab TDS.

4. Chlormezanone

It has antia nxiety and hypnotic actions as well, and has been used for tens ion states as ociated with increased muscle tone.

DOLOBAK 100 mg .,. paracetamol 450 mg tab, 1- 2 tab TDS.

5. Methocarbamol

It is less sedati ve a nd longer acting than mephenesin. Orally it has been used in refl ex muscle spasms and chronic neurologica l di seases. It ca n be injected i.v. witho ut producing thrombophl eb itis and haemolysis. and has been utilized for orthopedic procedures and tetanus.

ROBI AX 0.5 g tab, I TDS: 100 mg/ml inj. for i.v. or i.m. use. ROBIFLAM 750 mg + ibuprofen 200 mg tab: Et.:ROMOL-MR 400 mg + pamcetamol 500 mg tab.

C linical efficacy o f none of the above drugs as muscle relaxant is well established. Gastric irritation and sedation are the most important side efTects.

6. Diazepam

(see Ch. 29) It is the prototype of be nzodiazepines (BZ Ds) wh ich act in th e brain on specific BZ D recepto rs enhan cing GABAergic transmission. Muscle tone is

Table 25.3: Comparative features of centrally and penpherally acting muscle relaxants Centrally acting

Neuromuscular blockers

1. Decrease muscle tone without reducing voluntary power

Cause muscle paralysis, voluntary movements lost

2. Selectively inhibit polysynaptic reflexes in CNS

Block neuromuscular transmission

3. Cause some CNS depression

No effect on CNS

4. Given orally, sometimes parenterally

Practically always given i.v.

5. Used in chronic spastic conditions, acute muscle spasms, tetanus

Used for short-term purposes (surgical operations)

384

DRU GS ACTING ON PE RIPHERAL (SOMATIC) NERVOUS SYSTEM

reduced by supraspinal rather than spinal action, but muscle relaxant: sedative activity ratio is low. o gastric irritation occurs and it is very well tolerated, though sedation limits the dose which can be used for reducing muscle tone. It is particularly valuable in spinal inj uries and tetan us. Combined with analgesics, it is popular for rheumatic disorders associated with muscle spasm. Dose: 5 mg TDS orally, 10-40 mg i.v. (in tetanus).

7. Baclofen This analogue of the inhibitory transmitter GA BA acts as a selective GABA 8 receptor agonist. The GABA receptors have been divided into:

GABAA receptor It is an intrinsic ion channel receptor which increases CJ- conductance, and is blocked by bicuculline; facilitated by BZDs. GABA8 receptor It is a G-protein coupled receptor, which hyperpolarizes neurones by increasing K+ conductance and altering Ca2 ' flux. It is bicuculline insensitive, but blocked by saclofen. Baclofen does not affect c1- conductance and its actions are not antagonized by bicuculline. The primary site of action of baclofen is considered to be in the spinal cord where it decreases exc itatory transmitter re lease and depresses both polysyna ptic and monosynaptic refl exes. As such, it does produce muscle weakness, but is less sedative than diazepam. Spasticity in many neurological disorders like multiple sclerosis, amyotropic lateral sclerosis {ALS), spinal inj uries and fl exor spasms is reduced. Baclofen is the preferred drug for symptomatic relief in these conditions. However, it is relatively ineffective in stroke, cerebral palsy, rheumatic and traumatic muscle spasms and parkinsonism. Baclofen is wel l absorbed orall y and is primarily excreted unchanged in urine with a t½ of J-4 hours. Side effects are drowsiness, mental confusion, weakness and ataxia; serum transaminases may rise. Sudden withdrawal after chronic use may cause hallucinations, tachycardia and seizures. Dose: IO mg 8D to 2S mg TDS. LIORESAL. LIOFEN 10 mg. 25 mg tab.

8. Thiocolchicoside Chemically related to colchicine, this muscle relaxant is believed to act as a GABA mimetic and glycinergic drug with additional analgesic action. Combined with NSA IDs, it is being used for painful muscle spasms, such as torticolis, sprains, backache, etc. Side effects are gastric upset and photosensitivity reactions Dose: 4 mg TDS-QID; ,ucoX IA-MR : rl11ocolchicosidc 4 mg - ctoricoxib 60 mg tabs.

9. Tizanidine This clonidine congener has minimal cardiovascular effects, but is a central a 2 adrenergic agonist which inhibits release of excitatory amino acids in the spinal interneurones. It may facilitate the inhibitory transminer glycine as well. Polysynaptic reflexes are inhibited resulting in decreased muscle tone and frequency of muscle spasms without reducing muscle strength. Efficacy similar to baclofen or diazepam has been noted in multiple sclerosis, spinal injury and stroke, with fewer side effects. Tizanidine is absorbed orally, undergoes first pass metabolism and is excreted by the kidney; t½ 2- 3 hours. It is indicated in spasticity due to neurological disorders and in painful muscle spasms of spinal origin. Side effects are drymouth, drows iness, night-time insomnia and ha llucinations. Dose-dependent e levation of Iiver enzymes occurs. Though no consistent efTect on BP ha been observed, it should be avoided in patients receiving antihypertensives, especially clonidi ne. Dose: 2 mg TDS; max 24 mg/ day. SIRDA LUD 2, 4, 6 mg rnb. TIZAN 2. 4 mg tab; BRUFENMR, TIZAFE:->: 2 mg + ibuprofen 400 mg tab; T IZANAC 2 mg + drclofenac 50 mg tab. PROX IV0'1-\1R 2 mg nimcsulide 100 mg cap.

Uses of centrally acting muscle relaxants

1. Acute muscle spasms Overstretch ing of a muscle, sprain. tearing of ligaments and tendons, dislocation, fibrosi tis, bursitis, rheumatic disorders, etc. cause pain ful spasm of muscles. The mephenesin-like and BZD muscle relaxants, combined with analgesics, are commonly used, but efficacy is not impressive.

SKELETAL MU SCLE RE LAXANTS

2. Torticollis, lumbago, backache, neuralgias T hese are other conditions in which pai nful spasm of certain muscles is a prominent feature. They respond to centrally acting muscle re laxants in the same way as acute muscle spasms.

3. Anxiety and tension Inc reased to ne of muscles often attends these stales. Diazepam group of drugs and chlormezanone benefit by the ir antianxiety as well as muscle relaxant actions. 4 . Spastic neurological diseases Impairment of descend ing pathways in the cerebrospinal axis and wi thdrawal of inhibitory influence over the stretch reflex causes c hroni c increase in muscle tone or spastici ty. Hem iplegia, paraplegia, spina l injuri es, mu ltiple sclerosis, ALS and cerebra l

palsy fall in this category. These conditions are benefited by baclofen, diazepam, tizanidine and dantrolene but not by mephenesin group of drugs. However, therapy of these disorders is far from satisfactory. 5 . Tetanus Most commonly diazepam is in fused i.v. and the dose is titrated by the response. Methocarbamol is an alternati ve. 6 . Electroconvulsive therapy Oiazepam decreases the intensi ty of convul sions resulting from ECT, without dimin ishing its therapeutic effect. Often SCh is used in addition for total suppression of the muscular component of ECT. 7 . Orthopedic manipulations T hese procedures may be performed under the influence of d iazepam or methocarbamol given i.v.

r:r PROBLEM DIRECTED STUDY

25.1 A 30-ye ar lady brought to the hospital emergency with 40% burn inju ry has to be operated under general anaesthesia. (a) Which muscle relaxant should be preferred for tracheal intubation and a brief su rgical procedure in this patient? Give reasons. (see Appendix-1 for solution)

385

..

Chapter

26

Local Anaesthetics

Local anaesthetics (LAs) a rc drugs which upon topical ap plication or loca l injectio n cause reversible loss of sensory perception, especia ll y of pain, in a restricted area of the body. They block ge neration and conduction of nerve impulse at any part of the neurone with which they come in contact. without causing

any structural damage. Thus, not only sensory but also motor impulses are interrupted when a LA is applied to a mixed nerve, resulting in muscula r para lysis and loss of autonomic contro l as well. Im portant differences between genera l and local anaesthesia are tabulated in Table 26. 1.

LOCAL ANAESTHETICS

I •J~T

• YJL:; •

.... ~.

-

I

I Low potency,

I

short duration

'Intermediate potency and duration

Proca inr Chloroprocaine

Lidocaine {Lignocaine) Prilocaine

--.

I

I High potency, I Soluble long duration

Tetracaine Bupivacaine Ropivacaine Dibucaine

Mep ivacainc. Etidocaine. Artica ine. Dycloninc, arc other local anaesthetics, occasionally used in some countries. Some other drugs, e.g. propranolol, chlorproma7ine, H1 antih1staminics. quinine have significant LA activity, but

I

I insoluble

Cocaine Lidocaine Tetracaine Proparacaine

Bcnzocaine Butylaminobenzoa te Oxethazaine

arc 1101 used for this purpose because of local irritancy or other prominent systemic activity. Local anaesthesia can be produced by cooling as wel l, e.g. application o f ice, CO, snow. ethylchloride spray.

Table 26.1: Comparative features of general and local anaesthesia 1. Site of action 2. Area of body involved 3. Consciousness 4. Care of vital functions

5. Physiological trespass 6. Poor health patient 7. Use in non-cooperative patient 8. Major surgery 9. Minor surgery

General anaesthesia CNS Whole body Lost Essential High Risky Possible Preferred Not preferred

Local anaesthesia Peripheral nerves Restricted area Unaltered Usually not needed Low Safer Not possible Cannot be used Preferred

387

LOCAL ANAESTHETICS CHEMISTRY

+30

T he clinically usefu l LAs are weak bases with amph iphilic property. A hydrophilic secondary or tertiary amine on one side and a lipophilic aromatic residue on the other arc jo ined by an a lkyl cha in thro ugh an ester or amide linkage.

b 0

-30

>

.s -60

PROCAINE (ester)

-90

ft c::J-CH2-~C2Hs CH3

C2Hs

LIOOCAINE (amide)

Ester-linked LAs Cocaine. procaine, chloroprocaine, tetracaine, benzocaine. Amide-linked LAs

Lidocainc, bupivacaine, dibucaine, prilocaine, ropi vacaine. Features of amide LAs (compared to ester LAs) • Produce more intense and longer lasting anaesthesia • Bind to a, acid glycoprotein in plasma • Not hydrolysed by plasma esterases • Rarely cause hypersensitivity reactions: no cross sensitivity with ester LAs

Because of their short duration , less intense analgesia and higher risk of hyper ensitivi ly, the ester-linked LAs are rarely used for infiltration or nerve block, but are still used topically on mucous membranes.

MECHANISM OF ACTION The LAs block nerve conduction by decreasing the entry of a ' ions during upstroke of action potential (A P). As the concentration of the LA is increased, the rate of rise of AP and maximum depolarization decreases (Fig. 26. 1) causing slowing of conduction. Finally, local depolarizat ion fai ls to reach the threshold potential and conduction block ensues.

t Fig. 26.1: Effect of progressively increasing concentrations (b,c,d) of a local anaesthetic on the generation of an action potential in a nerve fibre, (a) Untreated nerve fibre

The LAs interact with a receptor situated within the voltage se nsiti ve a c hannel and raise the thres hold of channel opening: Na permeabil ity fai ls to increase in response to an impulse or stimulus. Impulse conduction is interrupted when the I a' c hanne ls over a critical length of the fibre (2- 3 nodes of Ran vier in case of myelinated fibres ) a rc blocked. The details are ex plained in Fig. 26.2. At physiological pH, the LA molecule is partly ionized. The equ ilibrium between the unioniz ed base fom1 (B) and the ionized cati onic form ( BH ' ) depends on the pKa of the LA. Potency of a LA ge nerall y corresponds to the lipid solubility of its base form (B), because it is this form which penetrates the axon. Howe ver, the predo minant active s pec ies is th e cationic form of the LA w hich is able to approach its receptor easi ly when the cha nne l is open at the inne r face. and it binds more avidly to the activated a nd inactivated states of the channel. than to the resting state. Binding of the LA prolongs the inacti vated state, so that the c hannel ta kes longer to recover. Consequently refractory period of the fibre is increased. A rest ing ne rve is rather resista nt to blockade. Blockade develops rapidly

388

DRU GS ACTING ON PERIPHERAL (SOMATIC) NERVO US SYSTE M

RESTING

ACTIVATED

INACTIVATED

Axoplasm Fig. 26.2: A model of the axonal Na·channel depicting the site and mechanism of action of local anaesthetics. The Na• channel has an activation gate (make or 'm' gate) near its extracellular mouth and an inactivation gate (halt or 'h' gate) at the intracellular mouth. In the resting state the activation gate is closed. Threshold depolarization of the membrane opens the activation gate allowing Na• ions to flow in along the concentration gradient depolarizing the membrane. This depolarization inactivates the Na• channels within a few msec by closing the inactivation gate; K· channels open quickly-outward flow of K• repolarizes the membrane allowing Na• channels to recover to the resting state in a time-dependent manner. The local anaesthetic (LA) receptor is located within the channel in its intracellular half. The LA traverses the membrane in its unionized lipophilic form (B), reionizes in the axoplasm and approaches the LA receptor through the intracellular mouth of the channel. It is the cationic form (BH· ) of the LA which primarily binds to the receptor. The receptor has higher affinity, or is more accessible to the LA in the activated as well as inactivated states compared to the resting state. Binding of LA to its receptor stabilizes the channel in the inactivated state and thus reduces the probability of channel opening. The neuronal Na· channel is a 300 KO glycoprotein composed of a large (a) and two small (~,. ~2) subunits. The a subunit encloses the Na• selective pore within its 4 homologous domains (I to IV), each domain has 6 membrane spanning helical segments (S 1 to S6) connected alternately by intracellular and extracellular loops. The wall of the pore is formed by all four S5-S6 segments, while the short nonhelical loops connecting SS-S6 on the extracellular surface fold into the pore and serve as the activation gate. Voltage sensors located in the S4 segments move verti· cally on depolarization and open the activation gate by allosteric conformational change. A few msec later, the short intracellular loop connecting domains Ill and IV folds into the inner mouth of the pore inactivating the channel. The LA receptor is located in the S6 segment of domain IV. Channel activation either transforms the LA receptor to a higher affinity conformation or exposes it on the wall of the pore, and this persists during the subsequent inactivation phase.

w hen the nerve is stimu lated re peatedly. The degree of blockade is frequency depe ndent, i.e. greater blockade occ urs at highe r freq uency of sti mulat io n. Mo reover, e x posure of the fi bre to h igher concentrati o n of Ca2• reduces inacti vati on of a· c hannels and lesse ns the degree of block. Blockade of cond uct io n by LA is not due to hyp erpolarizat ion ; in fact, resting membra ne pote ntial is unaltered, because K• cha nne ls are blocked o nly a t higher concentrations of the LA. The onset time of blockade is related primarily to the pKa of the LA . Those w ith lower pKa (7.6- 7.8), e.g. lidocaine, mepivacaine are fast

acting, beca use 30-40% LA is in the undi sso• ciated base form at p H 7.4 and it is this form w hic h pe ne trates the axon. Procaine, tetracaine, bup ivacaine have higher p Ka (8. 1- 8.9). only 15% or less is unionized at pH 7.4; these LAs are slow acting. Chloroprocaine is an excep• tion, hav ing rapid onset despite high pKa (9. 1).

LOCAL ACTIONS The clinica lly used LAs have no/minim al local irrita nt action and block sensory nerve end ings, nerve trunks, neuromuscul ar j uncti on, ganglionic synapse an d receptors (no n-selectively), i.e. those structures which function thro ugh increased

LOCAL ANAESTHETICS Na permeability. They also reduce re lease of acety lcholine from motor nerve endings. Injected around a mixed nerve they cause anaesthes ia of skin and paralysis of the voluntary muscle supplied by that nerve. Se nso ry and moto r fibres a re inhere ntl y equall y sensi tive, but some LAs do ex hi bi t unequa l abi lity to block the m. e.g. e pidu ra l bupi vacaine produces sensory block at muc h lower concentration than that needed fo r motor block. The di fTerential se nsory blockade is greatly advantageo us for pa in re lief in normal labour by enabling the mother to acti vely push the foetus down during uterine contractions. Simi larly, in continuous e pidural ana lgesia for postoperative pain relief (e.g. after knee replacement) absence of motor b lock is he lpful in early ambu lation and performance of exerc ises. The sensi tivity to LA is dete rmin e d by diame ter o f the fibres as well as by fi bre ty pe. Diameter re maining the same, mye linated nerves a re blocked earlier than nonmyelinated. ln genera l, sma ller fibres a re more sensiti ve tha n larger fi bres; but th is is not always true. Fibres difTer in the critical length of the axon tha t must be exposed to the LA for effecti ve blockade. In myelinated fibres the critical length usually covers 2- 3 nodes of Ranvier. Smaller fibres te nd to have s horter cri t ical le ngths, because in them vo ltage c hanges p ropaga te passively for shorter distances. Further, more slender axons have shorter intemodal d istances, and LAs easily en ter the axon at the nodes of Ranvier. The density of Na channe ls is much hi ghe r a t these nodes. Moreove r, frequ e ncy dependence of blockade makes sma ller sensory fibres more vulnerable since they generate high frequency longer lasting action potentials than the motor fibres. T hus, fibre diameter itself may not govern sen itivi ty to LA. Autonomic fibres are generally more susceptible than somatic fibres. Among the somatic afferents order o f blockade is: pain- temperature sense-touch-deep pressure sense. Since pain is genera lly carried by sma ller diame ter fibres tha n those carrying other sensations or motor impulses, pain in the first moda lity to

be afTccted. Applied to the tongue, bitter taste is lost Arst followed by sweet and sour, and salty taste last of all. In general, fibres that are more susceptible to LA arc the first to be blocked and the last to recover. Also, locati on o f the fibre with in a nerve trunk detennincs the latency, duration and often the depth of local anaesthesia. e rve sheaths restrict diffusion o f the LA into the nerve trunk so that fib res in the oute r layers are blocked ea rl ier than the inner or core fibres. As a result, the more prox imal areas supplied by a nerve arc a fTected earlier because axons supply ing them are located more peripherally in the nerve than those suppl ying distal areas. The di fferential arrangement of vari ous types of sensory and motor fibres in a m ixed nerve may partly account for the di fferentia l bloc kade. Motor fibres are usually present c ircumferentially. As such, they may be blocked earlier than the sensory fibres in the core of the nerve. The LA often fa ils to afford adequate pain control in inflamed tissues (like infected tooth). The like ly reasons arc: a. Infla mmation lowe rs pl! of the tissuc--greater fractio n of the LA is in the ionized form hindering diffusion into the axolemma. b. Bloo d now to the innamcd area is increased- the LA is removed more rapidly from the site. c. Effectiveness of Adr injected with the LA is reduced at the inflamed site. d. Inflammato ry products ma y oppose LA action. Addition of a vasoconstrictm; e.g. adrenaline ( I :50,000 to I :200,000): • Prolo ngs durati o n o f acti o n o f LAs by decreasing their rate of removal from the local site in to the c irculation: contact time of the LA with the nerve fibre is prolonged. • Enhances the intensity of nerve block. • Reduces systemic toxic ity of LAs: rate of absorpti on is reduced and metabolism keeps the plasma concentration lower. • Prov ides a more bloodless field for surgery. • Increases the chan ces of subsequent local tissue edema a nd necrosis as well as delays

389

390

DRUGS ACTING ON PERIPHERAL (SOMATIC) NERVOUS SYSTE M wound heali ng by red ucing oxygen supply and enhanc ing oxygen consumption in the affected a rea. • May raise BP and promote arrhyth mia in susceptible individ ual s.

SYSTEMIC ACTIONS Any LA injected or applied locally is ul tim ately absorbed and ca n produce syste mic effects depending on the concentration attained in the plasma and tissues.

C.N.S. All LAs are capable of produci ng a sequence o f stimulation fo ll owed by depression. Cocaine is a powerful CNS stimu lant causing in sequence euphoria---excitemcnt- mental con fusion- restlessness- tremor a nd twitching of musclesco nvu Is ions- un conscious ness- respirato ry depression-death, in a dose-dependent manner. The synthetic LAs are much less potent in th is regard. At safe clinical doses, they produce litt le apparent CNS effects. H ighcr dose or accidental i.v. injection produces C S stimulation followed by depression. The early neurological symptoms of overdose w ith lidocaine and other c linically used LAs are--circumoral num bness, abnormal sensation in the tongue, dizz iness, blurred vision, tinnitus fo llowed by drowsiness, dysphoria and lethargy. Sti ll higher doses produce excitation, restlessnc s, agitation, muscle twitching, seizures and finall y un consciousness. T he basic actio n of all LAs is ne uro na l inhibition; the apparent st im ulation seen initially is due to inhibition of inh ibitory neurones. At high doses, all neurones are inhibited and flatteni ng of waves in the EEG is seen.

c.v.s. Heart LAs arc cardiac depressants, but no significant effects are observed at conventional doses. At hi gh doses (2- 3 times the doses producing C S effects) or on inadvertent i. v. injection, they decrease automaticity, excitability, contractility, conductivity and prolong effective

refractory period (E RP). They have a quin idineli ke antiarrhythmic action. While procaine is not used c linically as antiarrhythmic because of short duration of action and propensity to produce CNS effects, its amide derivative procainamide is a class IA antiarrhythmic (see Ch. 39). Electrophysiological prope1ties of heart may be markedly altered at high plasma concentrations of LAs : QTc interva l is prolonged a nd LAs ca n themselves induce ca rdiac a rrh ythmi as. Bupivacaine is relatively more cardiotoxic a nd has produced ventricular tachycardia, fibrillation and arrest. Lidocaine has little effect on cono·actility and conductivity; it abbreviates ERP and has minimal proarrhythmic potential. It is used as an antia1Thythmic (see C h. 39).

Blood vessels LAs tend to produce fall in BP. This is primarily due to sympathetic blockade, but high concentrati ons, as obtained locally at the site of injection, do cause direct relaxation of a1teriolar smooth muscle. Bupivacai ne is more vasodi la tory than lidocaine, w hile pril ocai ne is the least vasodilatory. Toxic doses of LAs produce cardiovascular collapse. Cocaine has sympathomimetic property; increases sympathetic tone, causes local vasocon triction, marked rise in BP and tachycard ia. Procaine and related drugs have weak anticholinergic, antihistaminic, ganglion blocking, neuromuscular bloc king and smooth muscle relaxant properties, but these are cl inically insignificant.

PHARMACOKINETICS Because LAs act near their site of adm in istration, pharmacok inetic characte ri sti cs are not important determinants of the ir e fficacy, but ma rkedly inOuencc their systemic effects and toxicity. So luble surface anaesthetics (lidocaine, tetracaine) are rapidly abso rbed from m ucous membranes and abraded areas, but absorption from intact ski n is min ima l. Procaine does not sign ificantly penetrate mucous membranes. Rate of absorption depends on the blood flow to the area of application or injection, e.g. absorption and blood levels arc higher after interco tal block than after sciatic block. The absorbed LA being lipophilic, is wide ly d istributed; rapidly

LOCAL ANAESTHETICS enters highly perfused brain, heart, li ver, and kid ney, fo llowed by muscle and other viscera. Procaine is negligibly bound to plasma proteins, but amide LAs are bound to plasma a 1 acid glycoprotein. LAs are rapidly but temporarily bound 10 tissues, especially nerves, at the site of injection. Ester-linked LAs (procaine, etc.) are rapidly hydrolysed by plasma pseudocholinesterase and the remaining by este rases in the li ver. Am ide-linked LAs ( lidocai ne, e tc.) are degraded only in the liver microsomes by dca Iky lat ion and hydrolys is . M etabol ism of lidocaine is hepatic blood-fl ow dependent. The maxi ma l safe dose of LAs is lower in patients with hepatic disease and in the e lderly who have decreased li ver function. Afte r oral ingestion both procaine a nd lidocaine have high first pass metabol ism in th e liver. Thus, they are not active oral ly fo r anti arrhythm ic purposes.

ADVERSE EFFECTS Systemi c toxicity on rapid i. v. injection is related to the intrinsic anaesthetic potency of the LA. However, toxicity after topical application or regional injection is influenced by the relati ve rates of absorption and metabolism. Those rapidly absorbed but slowly metabolized are more toxic. • CNS effects are light-headedness, dizziness, audi tory an d visua l dis turbances, mental confusion, disorientation, shivering, twitchings, invo luntary movements, finally convulsions a nd respiratory arrest. This can be prevented and treated by diazepam/ midazolam. • Cardiovascular toxicity of LAs is manifested as bradycardia, hypotension, cardi ac arrhythmias, asystole a nd vascu lar collapse. • Inj ection of LAs may be painful, but local t iss ue tox ic ity of LAs is low. However, wou nd healing may be sometimes delayed. Addition of vasoconstrictors enhances the local tissue damage; rarely necrosis results. Vasoconstrictors should not be added fo r ring block of ha nds, feet, fingers, toes, penis and in pinna. Bupivacaine has the highest local tissue irrilancy.

• Hypcrsen itivity reactions li ke rashes, angioedema, dem,atiti s, contact sensitization, asthma and rare ly anaphylaxis occur. A llergic reactions are more common with ester-linked LAs, but rare with lidocaine or its congeners. Cross reactiv ity is freq uent among ester compounds, but not with ami de-linked LAs. Often methy lparaben added as preservative in certain LA solutions is respons ible for the a llergic reaction.

Precautions and interactions I. Before injecting the LA, aspi rate lightly to avoid intravascular injection. 2. Inject the LA s lowly and take care not lo exceed the maxi mum safe dose, especia ll y in children. 3. Propranolol (probably other p blockers a lso) may reduce metabolism of lidocai nc and other amide LAs by reducing hepatic blood flow. 4. Vasoconstrictor (adrenaline) conta ining LA should be avoided for patients . wi th ischaem ic heart disease, ca rdiac arrhythmia, thyrotoxicosis, uncontrolled hype rtension, and those receiving P blockers (rise in BP can occur due to unopposed a action) or tri cyclic antidepressants (upta ke blockade and potentiation of Adr).

INDIVIDUAL COMPOUNDS Important properties of local a naestheti cs are compared in Table 26.2. Cocaine

It is a natural a lkaloid from leaves of Erythroxy-

lon coca, a south American plant growing on the foothills of the Andes. The natives of Peru and Bolivia habitually chew these leaves. Cocaine is a good surface anaesthetic and is rapidly absorbed from buceal mucous membrane. II was first used for ocular anaesthesia in 1884. Cocaine should never be injected; it is a protoplasmic poison and causes tissue necrosis. Cocaine produces prominent CNS stimulation with marked effect on mood and behaviour. It induces a sense of wellbeing, delays fatigue and increases power of endurance. In susceptible individuals it produces a state referred 10 as ' high · lead ting to strong psychological but liule physical dependence. Cocaine is unique among drugs of abuse in not producing signifi cant tolerance on repeated use; sometimes reverse to lerance is seen (behavioural effects are experienced at lower doses). Cocaine also s timulates vagal ccntrc• bradycardia; vasomotor centre • rise in BP: vomiting centre• nausea

391

392

DRUGS ACTING ON PERIPHERAL (SOMATIC) NERVOUS SYSTE M

Table 26.2: Comparative features of commonly used local anaesthetics Drug

Surface anaesthesia

Nerve block

Max.

Relative potency

Cone. used (%)

dose

Onset

Duration (Min)

Cardiotoxicity

Lidocaine

+

1

0 .5-2.0

300mg

Fast

60-120

+

Bupivacaine

-

4-5

0.25---0.5

150 mg

lnterm.

120-360

+++

3-4

0.25---0.75

200 mg

Slow

120-300

++

Ropivacaine

and vomiting; temperature regulating ccntrc• pyrexia (also due to increased heat production as a result of enhanced muscular activity). In the periphery, it blocks uptake of A and Adr into adrenergic nerve endings, (see Fig. 9.4) resulting in higher concentration o f the transmiller around the receptors • sympathom imetic e ffect, potcntiarion of d irectly acting sympathomimetics and suppression of indirectly acting sympathornirnctics. Local vasoconstriction , tachycard ia, rise in BP and myd riasis arc the manifestations o f its sympathomimctic action. The only indicatio n for cocaine is in ocular anaesthesia. However, it causes constriction of conjunctiva! vessels, c louding and rarely sloughing of cornea (due to drying and local tissue toxicity). Its use, therefore, is not warranted.

Procaine It is t he first synt hetic local a naest hetic introduced in 1905. Its popularity dec lined after the introduction of lidocainc, and it is not used no\\. It is not a surface anaesthetic. Procaine fom1s poorly soluble salt with bcn7yl penicillin; procaine pe11icilli11 injected i.m. acts for 24 hours due 10 slow absorption from the site o f injection.

Lidocaine (Lignocaine)

Introduced in 1948, it is c urrentl y the most w ide ly used LA. It is a versatile LA, good both fo r surface application as well as injection and is ava ilable in a variety of form s. Inj ected around a nerve it blocks condu ction wi thin 3 min, w hereas p roca ine ma y ta ke 15 m in ; a ls o a naesthes ia is mo re intense and longer lasting. Vasod ila ta tion occurs in the injected area. Tt is used for surface applicatio n, infi ltration, ne rve block. epidural, spinal a nd intravenous regio nal block anaesthesia. Lidocaine 2% with or without adrenaline is the most popular dental anaesthe tic. Cross-sensitivi ty with ester LAs is not seen. T he earl y centra l effects of lidocaine arc depressant, i.e. drowsiness, mental clouding, dysphoria, a lte red taste and ti nn itus. Overdose causes muscle twitching, convuls ions. cardiac a rrhythmias, fa ll in BP, coma a nd respiratory

a rrest li ke o ther LAs. Lidoca ine is a popular ant iarrhythmic (see Ch. 39). XYLOCAl'IE, GESICA I -t%, topical solution. 2°0 3clly, 2% ,i~cous, 5% ointment. 1•-• and 2% injection (with or without adrenaline), 5% heavy (for spinal anaesthesia): l00 mg/ml spray (IO mg per actuation). XYLOCA INE 2% with adrenaline 1:80,000 in 1.5 ml cartridge for dental anaesthesia. A transdermal patch of lidoeaine has been produced for application over the affected skin for relief of burning pain clue to postherpetic neuralgia.

Prilocaine It is similar to lidocainc but docs 110 1 cause vasodilatation at the site of infiltration and has lower C S toxicity clue 10 larger vo lume o f distribution and faster clearance. One of its metabolites has the potential to cause methacmoglobinaemia. Prilocainc has been used for infiltration. nerve block, spinal and intravenous regional anaesthesia.

Eutectic lidocaine/prilocaine Also called eutectic mixture of local anaesthetics (EM LA), th is is a un ique preparation which can anaesthe tise intact skin afte r s urface application. E11tectic mixture refers to lowering of me lting poi nt o f two solids when they are mixed. Th is happens when lidocai ne and prilocaine are mixed in equal proportion at 25°C. The resulti ng oil is emulsified into water to fo nn a cream that is applied under occlusive dressing for 1 hr before i.v. cannula ti on, s plit skin graft harvesting and other superficial procedures. umbness up to a depth o f 5 mm lasts for 1-2 hr a fter removal. It can be used as an altern at ive to lidocaine infi ltration. PRILOX 5% cream.

Bupivacaine

It is a more lipophi li c a nd more potent lo ng-actin g a mide -li nked LA, whi ch is used for infilt ra ti o n, ner ve b lock. epidu ral and spinal anaesthesia of long duration. Because of slow onset of action, it is not preferred for periphe ral nerve block. A 0.25% solution injected epidurally produces adequate

393

LOCAL ANAESTHETICS anal gesia without significant motor blockade. As a result, it has become very popular in obstetrics (mother can actively cooperate by ' bea ri ng down' during vaginal de livery) and for postope rative pain relief by continuous epidural infusion. Due to high lipid-solubility it distributes more in tissues than in blood after spinal/e pidural injection. T herefore. it is less like ly to reach the foetus (when used during labour) Lo produce neonatal depression. Bupivacaine is more prone to prolong QTc interval and induce ventricular tachycardia or cause cardiac depression. Epidural anac thesia with 0. 75% bupivacaine during labour has caused few fatalities due to cardiac arrest. As such, use of this concentration is contraindicated. Cardiac toxicity is a maj or concern which limits the total dose of bapivacaine. Recently, i.v. infusion of lipid emulsion has been found to reverse bupivacaine cardiotoxicity, probably by extracting lipophil ic bupivacaine from plasma and cardiac tissues. The strength of bupivacaine solution for different types of blocks is: Peripheral ner,e block: 0.25 0.5% Spinal anaesthesia: 0.5% (hyperbaric) Epidural anaesthesia: 0.25 0.5% Continuous epidural analgesia: 0. 125%. \1ARCAI 0.50., inj . for ncnc block, o.5% hyperban c ror spinal anae,thcsia. SENSORCA INE 0.25%, 0.5% inj, 0.5% heavy inj . Levobupivacalne The S(-) cnamiomcr ofbupivacai ne is equally potent but less cardiotoxic and less prone to cause sci7ures (after inadvertent intra vascular injection) than racemic bupivacaine. It is being used in some countries as a single enantiomer preparation.

Ropivacaine A newer bupivacaine congener, nearly as

long act ing but less card iotox ic.

It blocks A8 and C fibres ( involved in pain transmission) more completely than AP fib res wh ic h control motor fu nction . Lower lipid solubility of ropivacaine makes it less like ly to penetrate large myelinated motor fibres than the sensory fibres. The same may account for its lower C S and cardiac toxicity. Used as the S(- ) e nantiomer, equieffective concentrations of ropivacaine are higher tha n those of bupivacaine, but a greater degree of separation between sensory and motor block has been

obtai ned wi th epidural rop ivaca ine. Continuous epidura l ropivacaine has become popular for relief of postoperative and labour pain. It can also be employed for nerve blocks. ROPIN 0 .2° 0 inJ .

Tetracaine (Amethocaine) A highly lipid-soluble PABA ester, more potent and more toxic due to slow hydrolysis by plasma pseudochol inesterasc. It is both surface and conduct1011 block anaesthetic, but its use is restrictt:d to topical application 10 the eye, nose and throat. Though n is slow acting, absorption from tracheobroncbial spray is , ery fast and blood concentrations approach those attamed aller 1.v. injection. Therefore, its use for bronchoscopy is contraindicated. ANCTI 1/\NE po"der for solution. 1° o omtmcnt

Proparacaine (proxymetacai ne) This newer surface anaesthetic is the most commonly used ophthalmic anaesthetic now. It is a metaamino ben zoic acid derivative which is equally effective but faster acting than tetracaine. Tonometry can be performed 30 sec. after instilling one drop 0.5% proparacaine in the eye. Corneal anaesthesia lasts for I 0-20 min. Deeper anaesthesia needed for cataract extraction can be obtai ned by applying 1- 2 drops eve1y 5- 10 min. for a maximum of 5 applications, whi le for foreign body/suture removal 2- 3 applications may be enough. Proparacaine causes minimal ocular irritation. A llergic reactions are infreq uent compared to tetracaine. However, severe immediate corneal reac tion may occu r rarely. PARACAl'\E. PROPCA I E 0.5°0 eye drops. 5 ml bottle.

Dlbucaine (Cinchocaine)

It is the most potent, most toxic and longest acting LA. It is used as a surface anaesthetic on less delicate mucous membranes (anal canal). Use for spinal anaesthesia of long duration has been abandoned. NUPERCA l"IA L 1% ointment, in OTOG ESIC I% ear drops.

Benzocaine

and

Butylaminobenzoate

(Butam-

ben) Because of very low aqueous solubility, these LAs are not significantly absorbed from mucous membranes or abraded skin. They produce long-lasting numbness without systemic tox icity. When absorbed . benzocaine can cause methaemoglobinemia. These LAs arc used as lozenges for stomatitis, sore throat; as dusting powder/ointment on wounds/ ulcerated surfaces and as suppository for anorcctal lesions. Both arc PABA derivative, therefore can antagonize sulfonamides locally. PROCTOSEDYL- M: Butylammobcn,wate I% oint "ith fram)Ct:t in and hydrocon isonc acetate: for piles. PROCTOQUINOL 5% ointment of bcn7ocamc. ZOKE:-. 20°0 gd.

394

DRUGS ACTING ON PER IPHE RAL (SOMATIC) NERVO US SYSTEM Oxethazaine A potent topical anaesthetic, unique in ion• izing lo a very small extent even at low pH values. ll is, therefore, able to anaesthetise gastric mucosa despite acidity of the medium. Swallowed along with antacids it affords symptomatic relief in gastritis, drug induced gastric irritation, gastroesophageal reflux and heartburn of pregnancy. Doses exceeding I 00 mg/day may produce di.a.iness and drowsiness. MUCAINE Oxetha?ainc 0.2% in alumina gel + magnesium hydroxide suspension; 5 IO ml orally. TRICAINE-M PS: OxethaMtine 10 mg with methyl polys il oxane 125 mg, alum. hydroxide gel 300 mg, mag. hydroxide 150 mg per 5 ml gel.

USES ANO TECHNIQUES OF LOCAL ANAESTHESIA 1. Surface anaesthesia This is produced by topical application of a surface anaesthetic to mucous membranes or abraded skin. Only the superficial layer is anaesthetised and there is no loss of motor function. Onset and duration depends on the site, the drug, its concentration and form, e.g. lidocaine ( 10%) sprayed in the throat acts in 2- 5 min and produces anaesthesia for 30-45 m in. Addition of Adr does not affect duration of topical anaesthesia, but phenylephrine can cause mucosal vasoconstr iction and

prolong topical anaesthesia. Absorption of soluble LAs from mucous membranes is rapid; blood concentrations of tetracaine sprayed in throat/ tracheobronchial tree approach those attai ned o n i. v. injec tion- toxicity can occur. Except for eutectic lidocaine/prilocaine, no other LA is capable of anaesthetizing intact skin. The sites and purposes fo r which surface anaesthes ia is used a re given in Table 26.3.

2. Infiltration anaesthesia Dilute solution of LA is infiltrated under the skin in the area of operation so that sensory nerve endings are blocked. O nset of act ion is almost immediate and durati on is shorter than that afte r nerve block, e.g . lidocaine 30-60 min, bupi vacaine 90-1 80 min. Infiltration is used for minor operations. e.g. incis ions, excisions, hydrocele, herniorrha phy, etc. when the area to be anaesthetised is small. Relatively larger amount of LA is required compared to the area a naesthetised, but motor function is not affected. 3. Conduction block The LA is inj ected aro und nerve trunks so that the area d istal to injection is anaesthetised and paralysed. Choice of

Table 26.3: Sites and uses of surface anaesthesia Site

Drugs

Form

Purpose

1. Eye

Proparacaine Tetracaine

0.5% 1-2%

drops ointment, drops

tonometry, cataract and other ocular surgery

2. Nose, ear

Lidocaine Tetracaine

2--4% 1-2%

drops

painful lesions, polyps

3. Mouth, throat

Benzocaine Lidocaine

2%

lozenges rinse solution

stomatitis, sore throat painful ulcers

4- 10% 2%

spray jelly

tonsillectomy, endotracheal intubation, endoscopies

4 . Pharynx, larynx, trachea, bronchi

Lidocaine

5. Esophagus, stomach

Oxethazaine

0.2%

suspension

gastritis, esophagitis, heartburn

6. Abraded skin

Tetracaine Benzocaine Butamben

1% 1- 2% 1-2%

cream, ointment, dusting powder

ulcers, bums, itching dermatoses

7. Intact skin

Eutectic lidocaine/ prilocaine

5%

cream under occlusion

i.v. cannulation, skin surgery

8. Urethra

Lidocaine

2%

jelly

for dilatation, catheterisation

9. Anal canal, rectum

Lidocaine Dibucaine Benzocaine

2--4% 1% 5%

ointment, cream, suppository

fissure, painful piles, surgery, proctoscopy

LOCAL ANAESTHETICS the LA and its concentrarion is mainly dic tated by the required duration of action; lidocaine ( 1- 2%) wi th shorter duration of action is most commonly used. but for longe r lasting anaesthesia bupivacaine may be se lected. (a) Field block l t is produced by injecting the LA subcutaneously in a manner that a l I nerves comi ng to a partic ula r field a re blocked-as is don e for he rni orrhaphy, appe ndi cectomy, dental procedures, scalp stitching, operations on forearms and legs. etc. Larger area beginning 2- 3 cm distal to the line of inj ection can be anaestheti sed with lesser drug compared to infi ltraiion. The same concentrat ion o f LA as for infi ltration is used for field block. (b) Nerve block It is produced by injecting the LA around the appropriate nerve trunks or plexuses. T he area of resulting anaesthes ia is larger compared to the amount of drug used. Muscles suppl ied by the injected nerve/plexus are paralysed. The latency of anaesthesia depends on the drug a nd the area to be cove red by diffusion , e.g. lidocaine anaesthetises intercostal nerves within 3 min, but brachia! plexus block may take 15 min. For plexus block a ' flooding' technique is used and larger volumes are needed. erve block lasts longer than field block or infiltration anaesthesia. Frequently performed nerve blocks are-lingual, intercosta l, ulnar, sciatic. femoral, brachia! plexus, trigeminal, facial, phrenic, etc.- uscd for tooth extracti on, operations on eye, limbs, abdominal wall, fracture setting, trauma to ribs, neuralgias, persistent hiccup, etc. The primary purpose of nerve block anaesthesia is 10 abolish pain and other sensations. The accompanying motor paralys is may be advantageo us by providing muscle relaxation during surgery, as well as disadvantageous ifit interferes with breathing, ability 10 walk afte r the operation. or pa11ic ipation of the patient in labour or produci;,s postural hypote nsion.

4. Spinal anaesthesia

The LA is injected in the subarac hno id space between L2- 3 or L3--4, i.e. below the lower end of spi nal cord. The primary site of action is the nerve roots in the cauda equina rather than the spi nal cord. Lower abdomen and hind limbs are anaesthetised and paralysed. The level of anaesthesia depends on the volu me and speed of injection, spec ific

gravity of drug solution a nd posture of the patient. The drug solution could be hyperbaric (in 10% glucose) or isobaric with CS F. The nerve roots rapidly take up and retain the LA, therefore, its concentration in CSF falls quick ly after injection. T he level of anaesthesia does not change with change o f posture (becomes fixed) after IO min. Also, h ighe r segments a re exposed to progressively lower concentrations of the LA. ince autonomic preganglionic fibres are more sensitive and somatic motor fibres less sensitive than somatic sensory fibres, the level of sympathetic block is about 2 segments higher and the level o f motor para lysis about 2 segments lower than the level of cutaneous ana lgesia. The duration of spina l a naesthesia depends on the drug used and its concentration (Table 26.4). Addit ion o f 0.2-0.4 mg of adrenaline to the LA prolongs spinal anaesthesia by about I/3rd when measured by the time taken for the level of ensory block to recede to LI. Ad r may be enhancing spinal anaesthesia by reduci ng spina l cord blood flow or by its own analgesic effect exerted through spina l a , adrenocepto rs (intrathccal clonidine, an a , agonist, produces spinal ana lgesia by itself). • Women during late pregnancy req ui re less drug for spinal anaesthesia, because inferior vena cava compression by the e nlarged uterus leads to engorgement of the vettebral system and a decrease in the capacity of suba rac hnoid space. Spina l anaesthesia is used fo r operations on the lower limbs. pelvis, lower abdomen, e.g. prostatectomy, fracrure setti ng, obstetric procedures, caesarean section. etc. Choice of the LA fo r spinal anaesthesia primarily depends on the nature and duration of the operative procedure. The LAs employed with their doses and duration of a naesthesia are given in Table 26.4. Advantages of spinal anaesthesia over general anaesthesia are: • It is safer. • Produces good analgesia and muscle relaxati on without loss o f consciousness. • Cardiac, pulmonary. renal disease and diabetes pose less problem .

395

396

DRUGS ACTING ON PER IPHERAL (SOMATIC) NERVOUS SYSTE M

Table 26.4: Local anaesthetics used for spinal and epidural anaesthesia and continuous epidural analgesia Cont. epidural

Epidural (lumbar)

Spinal

Drug

Concn. (max. dose)

Concn.

Tota/dose

Duration

Concn.

Tota/dose

Lidocaine

1.5-5% (Heavy)

25-75 mg

60-90 min

1- 2%

200-300 mg

Bupivacaine

0.5% (Heavy)

10-20 mg

90-150 min

0.25-0.5%

75-150 mg

0.125% (400 mg/day)

Ropivacaine

0.75%

25mg

90-120 min

0.75%

100-200 mg

0.2% (12-24 mg/hour)

Complications of spinal anaesthesia 1. Respiratmy para(vsis With proper care, this is rare. Even when intercostal musc les a re paralysed, the diaphragm (supplied by phrenic nerve) mainta ins breathing. Hypotension and ischaemi a o r respiratory centre is more frequently the cause of respiratory fai lure than diffusion or the anaesthetic to higher centres. Due to paralysis of external abdominal and intercostal muscles, coughing and expectoration becomes less effective. This may lead to pulmonary compli cations. 2. llypotension It is due to blockade of sy mpatheti c vasocons tri c tor o utfl ow to the blood vessels; venous pooling and decreased return to the heart contributes more to the fall in BP than arteriolar dilatation. Paralysis of skeletal muscles of lower limb is another factor reducing venous return. Decreased sympathetic flow to heart and low venous return produce bradycardia. Rais ing the foot end overcomes the hypotension by pro moting venous d rainage. Sympathomimetics. especial ly those with prominent constrictor effect on veins (ephedrine, mephcntcrmine) effectively prevent and counteract the hypotension. 3. Neurologicul ~ymptoms Pain an d/ or paraeslhesias in the back and lower limbs lasting short periods are fe lt by some patients after recovery from spina l anaesthesia, especially when lidocai ne is used. This is infrequent w ith bupivacai ne. However, no neurological deficit attends the symptoms. 4. Headache It may occur due to seepage of CSF, which can be minim ised by us ing smaller bore needle.

5 . Cauda equina syndrome It is a very rare neurological complicati on resulting in pro longed loss of control over b ladder and bowel sph incters. T he cause is uncertain, but may be due to traumatic damage to nerve roots or chron ic a rach n oid itis caused by inadvertent introduction of the antiseptic in the su barachnoid space. eurotoxicity of the LA has also been blamed, especially when high/ repeated doses are given. 6. Nausea and vomiting This is more common after abdom inal operations, and is due to refl exes triggered by traction on abdominal v iscera. Premedica tion with ondansetron or metoclopramide prevents it.

5. Epidural anaesthesia

The spinal dural space is filled with semiliquid fat through which ne rve roots travel. The LA injected in this space-acts primarily on the nerve roots (in the epidural as well as subarachnoid spaces to whi ch it diffuses) and small amount permeates through intervertebral foramina to produce multiple paravertebral blocks. Epidural anaesthesia can be di vided into 3 categori es depending on the site of injection. (i) Thoracic The LA is injected in the midthorac ic region. The epidural space in thi s region is relative ly narrow; therefore smaller vol ume of the drug is needed and a wide segme ntal band of analgesia involving the midd le and lower thoracic derrnatomes is produced. It is used generally for pain relief following thoracic/upper abdominal surgery. Specially des igned catheters are avai lable which can be placed for repeated injections or continuous infusion of the LA to achieve epidural analgesia lasting few days.

LO CAL ANAESTHETICS

Contraindications to spinal anaesthesia • Hypotension and hypovolemia. • Uncooperative or mentally ill patients. • Infants and children-control of level is difficult. • Bleeding diathesis. • Raised intracranial pressure. • Vertebral abnormalities e.g. kyphosis, lordosis, etc. • Sepsis at injection site.

(ii) Lumbar Relatively large volume of drug is needed because epid ura l space is wide. It produces anaesthesia of lower abdo men, pelvis and hind limbs. Use of lumbar epidural anaesthesia is similar to that of spina l anaesthesia. (iii ) Caudal Inj ection is g iven in the sacral cana l through the sacra l hiatus. T his produces anaesthesia of pel vic and perinea[ region. 1t is used mostly fo r vagina l de livery, ano recta l and genito urinary o peratio ns. Lidocaine ( 1- 2%) and bup ivacaine (0.250.5%) are popular drugs for epidural anaesthesia. Onset is slower a nd duration of anaesthesia is longer w ith bupivacaine and actio n of bo th the d rugs is prolo nged by addition of adrena line. Technica lly epidura l a naesthesia is more difficult than s pi nal ana esthesia and re latively larger vo lumes of drug are needed. Consequ ently, blood co ncentrat ions o f the L A are hig he r. Ca rd iovascul ar co mp lication s a re s im ila r to th ose after s pina l anaesthe sia , but headac he and neurological c omplications are less Iike ly, because intrathecal space is not entered and the LA is not restri cted to a small area for lo ng

periods. Spread of the LA in the epidural s pace is governed by the vo lume injected: larger volume anaesthetizes mo re exte nsive area. Zone of d ifferentia l sympathetic blockade is no t evident after epidura l injecti on but motor para lysis is 4- 5 segme nts c auda l, especia lly w ith lower concentrati ons of the LA. Greatest sepa ratio n between sensory and mo to r b lock is o bta ined by use of 0.25% bupivacaine and ropi vaca ine. T his is especiall y valuable for obstetric purposes (mother can partici pate in labo ur w ithout fee ling pain) a nd for postoperative pa in re lief. 6. Intravenous regional anaesthesia (lntravascular infiltration anaesthesia, or Bier's block) It consists o r inject ion of the LA solution in a large vein of a tourn iquet occluded lim b such that the drug diffuses retrograde from the peripheral vascu lar bed to nonvascular tissues incl uding nerve endings. The limb is fi rst elevated to ensure venous drainage by gravity and the n tightly wrapped in an elastic bandage for max imal exsanguinat ion. Tourniquet is then applied proximally and inflated to above arterial BP. Elastic bandage is now removed and 20-40 ml o f 0.5% lidocaine is injected i.v. under pressure dista l to the tourniquet. Regional ana lgesia is produced within 2- 5 min and lasts ti ll 5-10 min after deflating the tourniquet which is kept inflated for not more than 15-60 min to avoid ischaemic injury. Deflation in < 15 min may allow toxic amounts of the LA to enter systemic circulation. The safety of the procedure depends on the rapid uptake of LA by peripheral tissues; only 1/4 of the injected drug enters systemic circulation when the tourniquet is released. Bradycard ia can occur. It is mainly used for the upper limb and for orthopaed ic procedures. Obstruct ing the blood supply o f lower limb is more difficult and larger volume of the anaesthetic is needed. Therefore, it is rarely used for lower limb. except the foot. Bupivacaine should not be employed because of its higher cardiotoxicity.

er PROBLEM DIRECTED STUDY

26.1 A healthy full-term primigravida aged 26 ye ars w ho has gone into la bour presents for del ivery. There is no ce phalopelvic dis pro portion o r a ny oth e r cont ra ind icatio n t o no rm a l vagina l delive ry. However, she de mands rel ief of pain associated with la bo ur and de live ry. (a ) Can some form of regional anaesthesia be use d t o re lieve he r pa in? If so, w hich type of regiona l a naesthe sia with whi ch drug wo uld be most s uita ble fo r he r? (see Appe ndix-1 for so lution)

397

DRUGS ACTING ON CENTRAL NERVOUS SYSTEM

Chapter

27

General Anaesthetics

General a naesthetics (GAs) are drugs w hi ch produce reversib le loss of all sensation and consciousness. The cardinal features of general anaesthesia are: • Loss of all sensation, espec ially pain • Sleep (unconsciousness) and amnes ia • Immobility and muscle relaxatio n • Abolition of somatic and autonomic reflexes. In the modem practice of balanced anaesthesia, these modalities are achieved by us ing combination of inhaled and i. v. drugs, each drug for a s pecific purpose. Anaesthesia has developed as a hig hly specialized science in itself. History Before the middle of I 9th century a number of agents like alcohol, opium, cannabis, or even concussion and asphyxia were used to obtund surgical pain, but operat ions were horrible ordeals. Horace Wells, a dentist, picked up the idea of using 11itrot1l' oxide (N,O) from a demonstration of laughing gas in 1844. 1lowever, he often fai led to relieve dental pain completely and the use of N,O had to wait till other advances were made. Morton. a dentist and medical student at Boston, after experimenting on animals. gave a demonstration of ether anaesthesia in 1846, and it soon became very popular. Chloroform was used by Simpson in Britain for obstetrical purpose in 1847. and despite its toxic potential. it became a , ery popular surgical anaesthetic. Cyclopropane was introduced in 1929, but the new generation of anaesthetics was heralded by ha/01ha11e in 1956. The first i.v. anaesthetic thiopentone was introduced in 1935.

MECHANISM OF GENERAL ANAESTHESIA The mechanism of ac tion of GAs is not precisely known. A wide variety of chemical agents produce genera l anaesthesia. Therefore, GA actio n had been related to so me common physicoc hemi cal property of the drugs. Mayer and Overton (190 1) pointed out a direct parallelism between lipid/water partition coefficient of the GAs and their anaesthetic potency.

Minimal alveolar concentration (MAC)

It is the lowe t concentrat ion o f the anaesthetic in the pulmonary alveoli needed to produce immobil ity in response to a painful stimulus (surgical inc is io n) in 50% individuals. It is accepted as a valid meas ure of potency of inhalational GAs, because it remains fairly constant for most young adults. The MAC of a ll inhalational anaesthetics declines progressively as age advances beyond 50 years. The MAC of a number of GAs shows excellent correlation wi th their oil/gas partition coefficient. However, th is only reflects capacity of the anaesthe tic to e nter into C S and attain sufficient concentrati on in the neurona l membrane, but no t the mechanism by which

400

DRUGS ACTING ON CENTRAL NERVOUS SYSTEM anaesthesia is produced. The 'unita,y hypothesis' that some single common molecular mechanism (like membrane expansion or membrane perturbation or membrane fluid ization) is responsible for the actio n of all inha lational anaesthetics has now been replaced by the 'agent specific theory' according to which different GAs produce anaesthesia by dilTerent mechanisms. Recent evidence favours a di rect interaction of the GA mo lec ules with the hydrophobi c do mains of me mbra ne prote ins or the lipidprotein interface. ot only different anaesthetics appear to act by different mo lecular mec hanisms, they also may exhibit stereospecific effects, and that various components of the anaesthetic state may involve action at discrete loci in the cerebrospinal axis. The principal locus of causation o f unconsciousness appears to be in the thalamus or reticular activating system, amnesia may result from action in cerebral cortex and hippocampus, while spinal cord is the likely seat of immobility on surgical stimulation. Recent findings show that ligand gated ion channels (but not voltage sensiti ve ion channels) are the major targets of a naesthetic action. The c hannel is the most GABAA receptor gated important of these. Many inhalational a naesthetics, barbiturates, benzodiazepines and propofol potentiate the action of inhibitory transmitter GABA to open C l- channe ls. Each of the above a naesthetics appears to interact with its own specific binding site o n the GABAA receptor-C l channe l complex, but none binds to the GABA binding site as such; though some inhaled anaesthetics and barbiturates (but not benzodiazepines) can d irectly activate Cl channels. Action of glycine (another inhibitory transmitter which also activates Cf channels) in the spinal cord and medulla is augmented by barbiturates, propofol and many inhalational anaesthetics. This action may block responsiveness to painfu l stimuli resulting in immobility of the anaestheti c state. Certain fluorinated anaesthetics and barbiturates, in addition, inhibit the neuronal cation channel gated by nicotinic cholinergic receptor which may mediate anal gesia and amnesia.

er

p

On the other hand, a nd ketamine do not affect GA BA or g lycine gated C f channels. Rather they selectively inhibit the excitatory MDA type of glutamate receptor. This receptor gates mainly Ca2 selective cation channels in the neurones, inhibition of which appears to be the primary mechanism of anaesthetic action of keta mine as well as N 20. The volatile anaesthetics have little acti on on this receptor. Neuronal hyperpolarization caused by GAs has been ascribed to activation of a specific type of K+ cha nne ls called ' two-pore domain' cha nnels. This may cause inhibition of presynaptic transmitter release as well as postsynaptic activation. Inhibition of transmitter re lease from presynaptic neurones has also been related to interaction with certain critical synaptic proteins. Thus, dilTerent facets of anaesthetic action may have distinct neuronal basis, as opposed to the earlier belief of a g lobal neuronal depression. Unl ike local anaesthetics which act primarily by blocking axonal conduction, the GAs appear to act by depressing synaptic transmission.

STAGES OF ANAESTHESIA GAs cause an irregularly descending depression of the CNS, i.e. the higher functions are lost first and progressively lower areas of the brain are in volved , but in the spinal cord lower segments are a ffected some what earli er than the higher segments. The vital ce ntres located in the medu lla are paralysed the last as the depth of anaesthesia increases. Guedel ( 1920) described four stages with ether anaesthesia, di viding the 111 stage into 4 planes. These clear-cut stages are not seen now-a-days with the use of faster acting GAs, premedicatio n and employment o f many drugs together. The prec ise sequence of even ts d iffers somewhat with anaesthetics other than ether. However, description of these stages still serves to define the effects of light and deep anaesthesia. Important features of different stages arc depicted in Fig. 27.1.

I. Stage of analgesia Stans from beginning of anaesthetic inhalation and lasts upto the loss of consciousness. Pain is progressively abolished. Patient remains conscious, can hear and see, and feels a dream like state; amnesia develops by the end of thi s stage. Reflexes and respiration remain nonnal.

401

GENERAL ANAESTHETICS

Respiration

STAGE

Thor.

Ocular movem.

Reflexes

SK.mus. tone

B. P.

H. R.

0

.J

,(

..I

w II w z w •w •0: n:z JC, 0 J

ANALGESIA

II

" ,(

DELIRIUM

J: 0..

Labour, Incisions and Minor ops. NIL

CJ

1

u5 w

USES

Abd.

I

2 MAC. • Decrease in cerebral metabolic rate which indirectly reduces cerebral blood flow (CBF). However, at higher concentrations, their direct vasodilatory action predominates and CBF may actually increase. These higher concentrations should be avoided in patients w ith raised intracranial pressure. • A ll fluorinated anaesthetics depress card iac contractility a nd lower BP. • Respiratory depression, bronchodilatation and reduced airway mucociliary function occurs; so that prolonged anaesthesia predisposes to respiratory complications. • Limited and reversible lowering of GFR and urine fl ow. • Reduced gastrointestinal motility.

PHARMACOKINETICS OF INHALATIONAL ANAESTHETICS lnhalational anaesthetics a re gases or vapour that diffuse rapidly across pulmonary alveoli and tissue barriers. The depth of anaesthesia depends on the potency of the agent ( MAC is an index of potency) and its partial pressure ( PP) in the brain, while induction and recovery depend on the rate of change of PP in the brain. Transfer of the anaesthetic between lung and brain depends on a series of tension gradients wh ich may be summarized asAlveoli _.- Blood ..,....... Brain Factors affecti ng the PP of anaesthetic attained in the brain are-

! . PP of anaesthetic in the inspired gas This is proportional to its concentratio n in the inspi red gas mixnire. Higher the inspired tension more anaestheti c wil l be transferred to the blood. Thus, induction can be hastened by adm inistering the GA at high concentration in the beginning. 2. Pulmonary ventilation It governs delivery of the GA to the alveoli. Hy perventilation will bring in more anaesthetic per minute and respiratory depression will have the opposite effect. Influence of minute volume on the rate of induction is greatest in the case of agents which have high blood solubility because their PP in blood takes a long time to approach the PP in alveoli. However, it does not affect the terminal depth of anaesthesia attained at any given concentration of a GA. 3. Alveolar exchange T he GAs diffuse freely across a lveoli, but if a lveolar ventilation and pe rfusion are mismatched (as occurs in emphysema a nd other lung diseases) the attainment of equilibrium between alveoli and blood is delayed: we ll perfused alveol i may not be well ventilated- blood draining these alveoli carries less anaesthetic a nd di lutes the blood coming from well ventilated alveoli. Induction and recovery both are slowed.

GENERAL ANAESTHETICS

4. Solubility of anaesthetic in blood This is the most important property determining induction and recovery. Large amou nt of an anaesthetic that is high ly soluble in blood (ether) must dis olve before its PP is raised. The rise as well as fall of PP in blood and consequently induction as well as recovery are slow. Drugs with low blood solubility, e.g. Np, sevoflurane, desfl urane induce quickly. Blood: gas partition coefficient p,.) given by the ratio of the concentration of the anaesthetic in blood to that in the gas phase at equi librium is the index of solubility of the GA in blood. 5. Solubility of anaesthetic in tissues Relative solubili ty of the anaesthetic in blood and a tissue determines its concentration in that tissue at equil ibrium. Most of the GAs are equally soluble in lean tissues as in blood, but more soluble in fauy tissue. Anaesthetics with higher lipid solubility (halothane) continue to enter adipose tissue for hours and also leave it slowly. The concentration of these agents is much higher in white matter than in grey matter. 6. Cerebral blood flow Brain is a highly perfused organ; as such GAs are quickly delivered to it. This can be hastened by CO2 inhalation which causes cerebral vasodilatation- induction and recovery are accelerated. Carbon dioxide stimulates respiration and this also speeds up the transport. Elimination When inhalation of the anaesthetic is discon tinued, gradients are reversed and the channel of absorption (p ulmonary epithelium) becomes the channel of elimination. All inhaled anaesthetics are eliminated mainly through lungs. The same factors which govern induction also govern recovery. Anaesthetics, in general, continue to enter and persist for long periods in adipose tissue because of their high lipid solubility and low blood flow to fatty tissues. Muscles occupy an intermediate position between brain and adipose tissue. Most GAs are eliminated unchanged. Metabolism is significant only for halothane which is >20% metabolized in liver. Others are practically not metabolized. Recovery may be delayed after

prolonged anaesthesia, especially in case of more lipid-soluble anaesthetics (halothane, isoflurane), because large quantities of the anaesthetic have entered the muscle and fat, from which it is released slowly into blood. Second gas effect and diffusion hypoxia In the initial part of induction, diffusion gradient from alveoli to blood is high and larger quantity of anaesthetic is entering blood. If the inhaled concentration of anaesthetic is high, substantial loss of alveolar gas volume will occur and the gas mixture will be sucked in, independent of ventilatory exchange- gas Row will be higher than tidal volume. This is significant only with Np, since it is given at 70- 80% concentration. Though it has low solubility in blood, about I litre/ min of ,0 enters blood in the first few minutes. As such, gas flow is I litre/min higher than minute volume. If another potent anaesthetic, e.g. halothane ( 1- 2%) is being given at the same time, it will also be delivered to blood at a rate I litre/min higher than minute volume and induction will be faster. Th is is called 'second gas effect '. The reverse occurs when 20 is discontinued after prolonged anaesthesia: ,0 having low blood solubility rapidly diffuses into alveoli and dilutes the alveolar air, and PP of oxygen in alveoli is reduced. The resulting hypoxia, called diffusion hypoxia, is not of much consequence if cardiopulmonary reserve is normal. but may be dangerous if it is low. Diffusion hypoxia can be prevented by continuing I00% 0 2 inhalation for a few minutes after discontinuing 20 , instead of straight away switching over to air. Di ffusion hypoxia is not significant with other anaesthetics, because they are administered at low concentrations (0.2-4%) and cannot dilute alveolar air by more than 1- 2% in any case. TECHNIQUES OF INHALATION OF ANAESTHETICS

Diffore111 1echniques are used according to facility available, agen 1 used. condilion of 1hc patient, type and dura1ion of operation. 1. Open drop method Liquid anaesthetic is poured over a mask "ith gauze and its Yapour is inhaled with air. A lot of anaeslhelic , apour escapes in the surroundings and the

403

404

DRUGS ACTI NG ON CENTRAL NE RVOUS SYSTEM concentration of anaesthetic breathed by the patient cannot be determined. IL is wastefiil-can be used only for a cheap anaesthetic. I lowever. it is simple and requires no special apparatus. Use now is limited to peripheral areas. Ether is the only agent administered by this method. especially in children.

2. Through anaesthetic machines Use is made of gas cylinders. spec ialized graduated vaporizers, flow meters, unidirectional valves, corrugated rubber tubing and reservoir bag. The gases are delivered to the patient through a tightly fitting face mask or endotracheal tube. Administration of the anaesthetic can be more precisely controlled and in many situations its concentration estimated. Respiration can be controlled and assisted by the anaesthetist. (a) Open system The exhaled gases are allowed to escape through a valve and fresh anaesthet ic mixture is drawn in each time. No rcbreathing is allow.:d now rates are high- more drug is consumed. However, prcdctcrmined 0 2 and anaesthetic concentration can be accurately delivered. (b) Closed system The patient rebreaths the exhaled gas mixture after it has c ircu lated through sodalime which absorbs CO,. Only as much O, and anaesthetic as have been taken up by the patient are added to the circuit. Flow rates are low. This is especially useful for expensive and explosive agents (litt le anaesthetic escapes in the s urrounding air). Halothane, isonurane, desfluranc can be used through closed system. However, control of inhaled anaesthetic concentration is imprecise. (c) Semiclosed system Partial rcbrcathing is allowed through a partially closed valve. Cond itions are intennediate with moderate flow rates.

Properties of an ideal anaesthetic

A. For the patient lt should be pleasant, nonirritating, should not cause nausea or vomiting. Induction and recovery should be fast with no after elTects. 8 . For the surgeon It should provide adequate analgesia, immobility and muscle relaxation. It should be non inflammable and nonexplosive so that cautery may be used. C. For the anaesthetist Its admi nistration should be easy, controllable and versatile. • Margin of safety should be wid no fall in BP. • Heart, li ver and other organs should not be affected. • It should be potent so that low concentrations are needed and oxygenation of the patient does not suffer. • Rapid adj ustments in depth of anaesthesia should be possible. • It should be cheap, stable and easily stored. • It should not react with rubber tubing or soda lime.

The important physical and anaesthetic properties of inhalational anaesthetics are presented in Table 27.1.

GENERAL ANAESTHETICS

Gas Nitrous oxide

Fast acting drugs Ether

Halothanc Isoflurane Desflurane Sevoflurane

Slower acting drugs

Thiopentonc sod. Methohexitone sod. Propofol

Etomidate

Benzodlazepines Diazepam Lorazcpam Midazolam

Opioid analgesic

Fentanyl Remifentanil Dissociative anaesthetic

Kctamine

Cyclopropane, trichloroethylene, methoxyflurane and enflurane are no longer used.

405

GENERAL ANAESTHETICS

Table 27.1 : Physical and anaesthetic properties of inhalat1onal anaesthetics Anaesthetic Botling lnflamma lrritancy Oil: Gas Blood: MAC point (°C) bility

1. Ether

35

2. Halothane

50

Intl.+ Explo. Noninfl.

(odour)

Induction

Muscle relaxation

Slow

V. good

partition Gas parti- (%) coefficient· lion coefficient·

+++ (Pungent)

65

12.1

1.9

224

2.3

0.75

lnterm.

Fair

(Pleasant)

3. lsoflurane

48

Noninfl.

± (Unpleasant)

99

1.4

1.2

lnterm.

Good

4. Desflurane

24

Noninfl.

+ (Unpleasant)

19

0.42

6.0

Fast

Good

5. Sevoflurane

59

Noninfl.

50

0.68

2.0

Fast

Good

1.4

0.47

105

Fast

Poor

(Pleasant)

6. Nitrous oxide

Gas

Noninll.

•At 37°C; Oil: gas and blood: gas partition coefficients are measures of solubility of the anaesthetic in lipid and blood respectively. MAC-Minimal alveolar concentration ; lnfl. -lnflammable; Explo.- Explosive; lnterm.-lntermediate

INHALATIONAL ANAESTHETICS

1. Nitrous oxide (N2O) It is a colourless, odourless, heavier than a ir, noninftammable gas supplied under pressure in steel cylinders. It is nonirritating, but low potency anaesthetic; unconsciousness can not be produced in all individuals without concomitant hypoxia; MAC is I OS% imply ing that even pure cannot produce adeq uate anaesthesia at I atmosphere pressure. Patients maintai ned on 70% 2 0 + 30% 0 2 a long with muscle re laxa nts often reca ll the events during anaesthesia, but some lose awareness completely. itrous oxide is a good analgesic; even 20% produces analgesia equivalent to that produced by convent ional doses of morphine. Muscle relaxation is min ima l. euromuscular blockers a re mostly requi red. Onset of 20 action is quick and smooth (but thiopentone is o ften used for induc tion), and recovery is rapid, because of its low blood solubility. Second gas effect and diffus ion hypoxia occur with only. Post-anaesthetic nausea is not marked. It tends to increase sympathetic tone which inc reases cerebral blood flow and counteracts the weak direct de pressant action on heart and circulation. itrous oxide is generally used as a carrier and adjuvant to other anaesthetics. A mixture

p

p

p

of 70% + 2S- 30% 0 2 + 0.2- 2% another potent anaesthetic is employed for most surgical procedures. In this way concentration of the other anaesthetic can be reduced to 1/3 for the same level of anaesthesia. Becau e Np has little effect on respiration, heart and BP: breathi ng and circulation are better maintained with the mixture than with the potent anaesthetic given alone in full doses. However, N,O can expand pneumothorax and other abnom1a-l air pockets in the body. It increases cerebral blood flow and tends to e levate intracranial pressure. Thus, N2O a lone is not suitable for patients with raised intracranial pressure. As the sole agent, Np (50%) has been used with 0 , for dental and obstetric analgesia. It is nontoxic to liver, kidney and brain. However, prolonged anaesthes ia has th e potential to depress bone marrow and cause peripheral neuropathy, probably by depressing methionin e synthase acti vity. Metabolism of Np does not occ ur; it is qu ickly removed from the body by lungs. It is cheap and commonly used.

p

2. Ether (Diethyl ether) ll is a highly , olati le liquid, produces irritating vapours which arc innammable and explosive.

(C2H 5 -

0 - C,H ,)

Ether is a potent anaeslhelic, produces good analgesia and marked muscle relaxation by reducing ACh oulpuL from motor nerve endings. The dose o f compelilive neuromuscular blockers should be reduced to about 1/3.

406

DRUGS ACTING ON CENTRAL NERVOUS SYSTEM II is highly soluble in blood. Induction is prolonged and unpleasant with struggling, breath-holding. sali, at ion and marked respiratory secretions (atropine must be given as premedica1ion to prevent the patient from drowning in his own secretions). Recovery is slow: post-anaesthetic nausea, vomiting and retching are marked. Respiration a nd BP are generally \\ ell maintained because o f reflex stimulation and high sympathetic tone. It does not sensitize the hcan to Adr, and is not hcpatotoxie. Ether is not used now. except in periphe ral and resource-poor areas, because of its unpleasant and inflammable properties. However. ii is cheap, can be given by open drop method ( though congestion of eye. soreness of trachea and ether b ums o n face can occur) without the need for any equipment, and is relatively safe even in inexperienced hands.

3. Halothane (FLUOTHANE)

It is a vo lat ile liquid with sweet odour, nonirritant and noninflam mable. Solubility in blood is modera teinduction is reasonably qu ick and pleasant. F'-... / H F-C-C- Br

F/

"-c1

HALOTHANE

Ha lothane is a pote nt anaesthetic; therefore precise control of admin istered concentration is essential. For induction 2-4% and for maintenance 0.5- 1% is delivered by the use of a special vapourizer. It is not a good analgesic or muscle relaxant, but it po tentiates competitive neuromuscular blockers. Halothane cau es direct depression of myocardial contractility by reducing intracellular Ca1 concentration. Moreover, sympathetic activity fails to increase reflexly. Cardiac o utput is reduced with deepening anaesthesia. BP starts fa lli ng early a nd parallels the depth. A 20-30 mm Hg drop in BP is common. Many vascular beds dilate but total peripheral resistance is not significantly reduced. Heart rate is reduced by vagal stimulation, direct depression of SA noda l automatici ty and absence of baroreceptor activation even w hen BP fa lls. It te nds to sensitize the heart to the aIThythmogenic action of Adr. The electrophysiological efTects are conducive to reentry-tachyarrhythmias occur occasionally, particularly if sympathetic stimulation occurs d ue to inadequate anaesthesia.

Halothane causes relatively greate r dep ression of respiration; breath ing is shal low and rapid- PP of CO2 in blood rises if respiratio n is not assisted. Ce rebral blood flow increases. Ventilatory support w ith added oxygen is freque ntly required. It tends to accentuate perfusion-ventilation mismatch in the lungs by causing vasodi latation in hypoxic alveoli. Pharyngea l and laryngeal reflexes are abolished ea rly and cough ing is s uppressed w hile bronchi dilate. As such, ha lothane is the preferred anaesthetic for asthmatics. It inhibits intestinal and uterine contractions. This property is util ized for faci litating external or internal version during la te pregnancy. However, its use during labour can prolong de li very and inc rease postparta l blood loss. Urine formation is decreased during ha lothane anaesthesia- primarily due to low g.f.r. as a result of fall in BP. Hepati ti s occurs in rare susceptible ind ividuals ( I in 35000 to I in I0,000) especially a fter repeated use and in those w ith fami lial predi sposition. A metaboliteofhalothane is probably involved which is believed to cause chemical or immunological injury. Halothane toxicity is less frequent in children. A genetically determi ned reaction malignant hyperthermia occu rs rare ly. Many s usce pti ble s ubjec ts have a n a b normal Ry RI ( Rya nodine receptor) calcium channel at th e sa rcoplasmic reticulum of skeletal muscles. This c hannel is triggered by halothane to release massive amounts of Ca2• intracell ularly causing persistent muscle contraction and increased heat production. Succ iny lcho li ne accentuates the condition (see Ch. 25). Ra pid external cooling, bicarbonate infusion, 100% 0 2 inhalation and i.v. dantrolene (seep. 382) are used to treat maligna nt hyperthermia. About 20% of halothane that enters blood is metabo li zed in the liver, the rest is exhaled out. El iminati on may continue for 24-48 hours after prolonged adm inistration d ue to accum ulation in fauy and other tissues. Recovery from halothane anaesthesia is smooth and reasonably quick; shivering may occur but nausea and vomiti ng are rare. Psychomotor performa nce and mental

407

GENERAL ANAESTHETICS

ability remai n depressed for everal hours after regaining consciousness. Halothane is a frequently used anaesthetic in developing countries, because it is relatively cheap and nonin-itanl, noninflammable, pleasant with relatively rapid action. It is particularly suita ble for use in children, both fo r induction as well as maintenance. In adul ts, it is mainly used as a maintenance anaesthetic a fter i.v. induction. However, in affluent countries it has been largely replaced by the newer agents which are costlier. Its deficiencies in terms of poor analgesia and muscle relaxation are compensated by concomitant use of p or opioids and neuromuscular blockers. 4. lsoflurane (SOFANE, FORANE, ISORANE) This fl uorinated anaesthetic introduced in 198 1 is currently the routinely used anaesthetic. It has totally replaced its earl ier introduced isomer enflurane. Isoflurane is somewhat less potent and less soluble in blood as well as in fat than halothane, but equally volatile. Compared to halothane, it produces relatively rapid induction and recovery, and is adm inistered through a special vapourizer; 1.5- 3% induces anaesthesia in 7- 10 min, and 1- 2% is used for maintenance. F

H

F

I

I

I

F

Cl

F

F-C- C-0-C-H I I I ISOFLURANE

Magnitude of fall in BP is similar to that with halothane, but unlike halothane, this is pri marily due to vasodilatation, while cardiac output is well mai ntained. As such, isofl urane is a suitable anaesthetic for patients with latent or overt myocardial insufficiency. Heart rate is increased. These cardiovascular effects probably resu lt from stimulation of adrenergic receptors, but isoflurane does not sensitize the heart to adrenergic arrhythmias. Jt dilates coronaries. Though not encountered clinically, possibility of ' coronary steal' has been apprehended in coronary artery disease patients on theoretical grounds. Respiratory depression is prominent and assistance is usually needed to avoid hypercarbia. Secretions are slightly increased.

Uterine and skeletal musc le re laxation is similar to that with halothane, but potentiation of neuromuscular blockers is greater. Pupils do noLdilate and light reflex is not lost even at deeper levels. Metabolism of isofl urane is negligible. Renal and hepatic toxiciLy has not been encountered. Postanaesthetic nausea and vomiting is mild. Though mildly pungent, isoflurane has many advantages, i.e. better adj ustment of depth of anaesthesia and low toxicity. It is a good maintenance anaestheLic, but not preferred for induction because of ether-like odour which is not liked by conscious patients, especially children. In contTast to enflurane, it does not provoke seizures and is particularly suitable for neurosurgery. 5. Desflurane It is a newer all fluorinated congener of isoflurane which has gained popularity as an anaesthetic for out patient surgery. Though it is highly volatile, a thermostatically healed special vapourizer is used to deliver a precise concentration of pure desfl urane vapour in the carrier gas CNP + 0 2) mi xture. Its distinctive properties are lower lipid sol ubility as well as very low solubility in blood and tissues, because of which induction and recovery are very fast. Depth of anaesthesia changes rapidly with change in inhaled concentration giving the anaesthetist better control. Postanaesthetic cognitive and motor impairment is shortlived, so that patient can be discharged a few hours after surgery. F

H

F

F

F

F

I I I F-C- C-0-C-H I I I DESFLURANE

Desflurane is 5 times less potent than isoflurane; higher concentration has to be used for induction, which in-itates the air passage and may induce coughing, breath-holding and laryngospasm. A somewhat pungent odour also makes it unsuitable for induction. Rapid induction sometimes causes brief sympathetic stimulation and tachycardia which may be risky in those with

408

DRUGS ACTING ON CENTRAL NERVOUS SYSTEM cardiovascular disease. The degree of respiratory depression. muscle relaxation, vasodilatation and fall in BP are similar to that with isoflurane. Cardiac contractility and coronary blood flow are maintained. Lack of seizure provoking potential arrhythmogenicity and absence of liver as well as kidney toxicity are also similar lo isoflurane. Desflw·ane is rapidly exhaled unchanged. As such, it can serve as a good alternative to isoflurane for routine surgery as well, especially prolonged operations. If closed circuit is used, the soda lime should be fresh and well hydrated to prevent formation of carbon monoxide.

6. Sevoflurane (SEVORANCJ This new polyfluorinated a naesthetic has properties intermediate between isoflura ne and desflurane. Solubility in blood and tissues as well as potency are less than isoflurane but more than desflurane.

F CF3 I

I

H I

F-C - C- 0-C-F I

F

I

H

I

H

SEVOFLURANE

Induction and emergence from anaesthesia are fast so that rapid changes in depth can be achieved. Absence of pungency makes it pleasant and administrable through a face mask. Unlike desflurane, it poses no problem in induction and is frequently selected for thi s purpose. Acceptabili ty is good even by pediatric patients. Recovery is smooth; orientation, cognitive and motor functions are regained almost as quickly as with desflurane. Sevoflurane is suitable both for outpatient as well as inpa tient surgery, induction as well as maintenance, but its high cost and need for high-flow open or semiclosed system makes it very expensive to use. In India, only high-end hospitals are using it. Sevofl urane does not cause sympatheti c stimulation and airway irritation even during rapid induction; can be used in patients prone to bronchospasm. Fall in BP is due to vasodilatation as well as modest cardiac depression. Respiratory depression, and absence of seizure or arrhythmia precipitating propensity are similar to isofl urane. However, occasional epileptiform patterns

have been seen in the EEG during sevoflurane anaesthesia. About 3% of absorbed sevoflurane is metabolized , but the amount of fluoride liberated is safe for the kidney and liver. However, it reac ts with sodalime, and is not recommended for use in full y closed ci rcui t.

INTRAVENOUS ANAESTHETICS

FAST ACTING DRUGS These are drugs which on i.v. injection produce loss of consciousness in one arm-brain circulation time (~ 11 sec). They are generally used for induction because of rapidity of onse t of action. Except in children, they have largely replaced inhalational anaesthetics for induction. Anaesthesia is then usually maintai ned by an inhalational agent. They also serve to reduce the amount of maintenance anaesthetic. Supplemented with analgesics and muscle relaxants, they can also be used as the sole anaesthetic.

1. Thiopentone sod. It is an u ltrashort acting thiobarbiturate, highly solu ble in water yielding a very alkaline solution, which must be prepared freshly before inj ection. Extravasation of the solution or inadvertent intraarterial injection produces intense pain; necrosis and ga ngrene can occur. Injected i.v. (3- 5 mg/kg) as a 2.5% sol ution, thiope ntone sod. produces unconsciousness in 15- 20 sec. Its undissociated form has high lipid solubility; therefore it enters brain almost instantaneously. Initial distribution depends on organ blood flow- brain gets large amounts. However, as other less vascular tissues (muscle, fat) gradua lly take up the drug, blood concentration falls and it back diffuses from the brain: consciousness is regained in 6- 10 min (t½ of distribution phase is 3 min). On repeated injection, the extracerebral si tes are gradually tilled up, so that lower doses produce anaesthesia which lasts longer. Its ultimate disposal occurs mainly by hepatic metabolism (eli mination t½ is 8-12 hr), but this is irrelevan t for termination of acti on of a s ing le dose. Residual C S depression may

409

GENERAL ANAESTHETICS persist for > 12 hr. The patient should not be allowed to leave the hospital without an attendant before this time. Thiopen tone is a poor ana lgesic. Painful procedures should not be carried out under its influence unless a n opioid or 20 has been given; otherwise, the patient may strugg le, shout and show reflex changes in BP and respiration. It is a weak muscle relaxant; does not i1Titate air passages. Respiratory depression w ith inducing doses of thiopentone is generally marked; transient apnoea can occur. The BP falls immediately after injection, mainly due to va odi latation, but recovers rapidly. However. cardiovascular collapse may occur if hypovolemia, shock or sepsis are present. Cardiac contractility is reduced, but reflex tachycardia occurs. However, thiopentone does not sensitize the heart to Adr, arrhythmias are rare. Cerebral blood Aow is reduced, both due to fall in BP as wel l as constric tion of cerebral vesse ls. However, cerebral oxygenati on does not suffer, because there is greater decrease in cerebral metabolic rate and 0 2 consu mption, whi le cerebral perfus ion is maintained. A comparative summary of effects of i. v. anaesthetics is presented in Table 27.2. Thiopentone was a commonly used inducing agent, but has now been largely replaced by propofol.

Adverse effects

Laryngospasm occurs generally when respiratory secretions or other irritants are present in the airways, or when intubation

is attempted while anae thesia is light. This can be prevented by atropi ne premedication and administration of succinylcholine immed iately after thiopentone. Succinylcholine and thiopentone react chemically-should not be mixed in the same syringe. Shivering and delirium may occur during recovery. Pain in the postoperative period is likely to induce restless ness; adequate analgesia should be provided. Postanaesthetic nausea and vomiting are uncommon. Thiopentone can precipitate acute intermittent porphyria in susceptible individuals, and is contraindicated in such subjects.

Other uses Thiopentone is occasionally used for rapid control of convulsions. Gradual i.v. infus ion of subanaesthetic doses can be used Lo fac ilitate verbal communication with psychiatric patients and fo r 'narcoa nalysis' of crim ina ls. IL acts by knocking off guard ing. PENTOTHAL, INTRAVAL SODIUM 0.5, 1 g powder for making fresh injectable solution. 2. Methohexitone sod. It is s imilar to thiopcntone, 3 times more potent. has a 4uicker and briefer (S-8 min) action. Excitement during inductio n and recovery is more common. It is more rapidly metabolized (t½ 4 hr) than thiopentone: patient may be roadworthy more quickly.

3. Propofol

Currently, propofol has superseded thiopentone as an i. v. anaesthetic, both fo r induction as we ll as maintenance. It is an oily liquid employed as a I% emulsion. Sensit ive patien ts may develop allergic reaction to the emulsifier. Unconsciousness after propofol injection occu rs in 15-45 sec and lasts 5- 10

Table 27.2 : Effects of intravenous anaesthetics on vital funclions CBF Resp. BP Anaesthetic HR drug

1. Thiopentone 2. Propofol

i -. J,

3. Etomidate 4. Diazepan 5. Ketamine 6. Fentanyl

-. t ti -1-

u

u

U-1-1-1t

-1-U

.!-

-1-1-,J. J.

U-lU-1-1- -1-1--1At A -1-

HR- Heart rate; BP-Systemic arterial blood pressure; Resp.-Respiratory drive; CBF---Cerebral blood flow. (Changes in intracranial pressure parallel CBF).

410

DRUGS ACTI NG ON CENTRA L NERVO US SYSTE M min. Propofo l di stributes rap idly (d istributi on t ½ 2-4 min). Elimi nati o n t ½ ( 100 min) is much shorter than that of thiopcntone due to rapid metabolism. Propofol is believed to cau e CNS depression by enhancing GA BAAreceptor mediated neurona l inhibition. lntenn ittent injection or continuous infusion of propofol is also used for total i. v. a naesthesia when suppleme nted by fentany l. It lac ks a irway irritancy and is not like ly to induce bronchospasm, the refore preferred in asthmat ics. It is partic ularl y suited fo r outpatient su rgery, because residual impairment is less marked and shorter-lasting. Incidence of postoperative nausea and vom iting is low; patient acceptability is very good. Excitatory effects and involuntary movements are noted in few patients. Induction apnoea lasting - I min is common. Fall in BP i more profound than that w ith thiopentone. Th is is due primari ly to vasodilatation with less marked cardiac depression. However, hypote nsion is short lasting. Ba roreflex is suppressed; hea rt rate rema ins unchanged or may decrease. Ma intenance anaesthesia w ith propofol produces dose-dependent respiratory depression which is more marked than with th iopentone. Effect o n cerebral blood flow and 0 , consumption is si milar to that of thiopentone. Pain during injection is frequent; but can be minimized by com bining with lidocaine. Dose: 2 mg/kg bolus i.v. for induction; 100-200 µg/kg/ min for maintenance. PROPOVAN 10 mg/ml and 20 mg/ml in 10, 20 ml vials.

In s ubanaesthetic doses (25- 50 µg/kg/min) it is the drug o f choice for sedating intubated patients in intensive care units. However, it is not approved for such u e in child ren. Prolonged sedation with hig her doses has caused severe metabolic acidosis, lipae mia and heart failure even in adults. It is another induction anaesthetic (0.2- 0.5 mg/kg)_ which has a briefer duration of action (4-8 min) than th1opentone; produces lirtle cardiovascular and res piratory de pression, but motor rest lessness and rigidity is more prom inent as are pain on injection or nausea and vomiting on recovery. It is a poor analgesic and has not found much favour except for patients with co mpromized cardioH1sculur status.

4. Etomldate

SLOWER ACTING DRUGS 1. Benzodiazepines (BZDs) In addi tion to preanaestheti c medication, BZDs are now frequently used for induc ing, maintaining and supplementing anaesthesia as well as for 'conscious sedation'. Relatively large doses (diazcpam 0.2-0.3 mg/kg o r equivalent) injected i.v. produce sedation, amnesia and then unconscio usness in 5- 10 min. If no other anaesthetic or opioid is g iven , th e patient becomes responsive in I hr or so due to redistributi o n o f the drug (d istribution t ½ of diazepam is 15 m in), but amnesia persists fo r 2- 3 hr and sedation for 6 hr o r mo re. Recovery is further delayed if la rger doses are g iven. BZ Ds are poor ana lgesics: an opioid or is usually added if the procedure is painful. By themselves, BZDs donot markedly depress respiration, cardiac contractility or BP, but whe n opioids are a lso given these functions are considerably com promised. BZ Ds dec rease muscle tone by central action, but requ ire neuro muscular blocking drugs for muscle re laxatio n of s urgical grade. They do not provoke postoperative nausea or vomiting. Involuntary movements arc not stimulated. BZ Ds are now the preferred drugs fo r endoscopies, cardiac catheterizatio n, angiographics, conscious sedation during local/regional anaesthesia, fracture setting, ECT, etc. They are a fre que nt com po ne nt of balanced anaesthes ia e mpl oying several drugs. Anothe r adva ntage of BZ Ds is that thei r anaesthetic action can be rapidly reversed by fluma zen il 0.5- 2 mg i.v.

p

Diazepam

0.2- 0.5 mg/kg by slow undiluted injection in a running i.v. drip: thi techniq ue reduces the burning sensation in the vein and incidence o f thrombophlebitis. Because of the irritant property of i. v. diazepam fo rmulation (an organic solvent is included due to its poor water solubi li ty) it has become less popular in anaesthetic practice. VA LI U M, CAL\4POSE JO mg/2 m l inj .

Lorazepam Three times more potent. slower acting and less irritating than diaz cpam. Tt

GEN ERAL ANAESTHETICS distributes more gradual ly-awakening may be delayed. Amnesia is more profound. Dose: 2-4 mg (0.04 mg/kg) i.,. CALMESE 4 mg/2 ml mj.

Midazolam This BZ D is water soluble , nonirritating to veins, faster and s horter acting (t½ 2 hours) and 3 times more potent than d iazepam. Fall in BP is omewhat greater than with diazepam. It is being preferred over diazepam for a naesthetic use: 1- 2.5 mg i. v. followed by I/4th supplemental doses. Because of its short duration of action. midazola m is the o nly BZD suitable for sedation of intubated and mechanica lly ventilated patients. It is being commonly used in critical care anaesthesia as 0.02-0.1 mg/kg/hr continuous i.v. infusion. FULSED. MCZOLAM. SIIORTAL I mg/ml, 5 mg/ml mj.

2. Ketamine

This uniqu e a naes th e ti c is pharmacologicall y related lo the hallucinogen phencycl idine. It induces the so called 'dissociative anaesthesia' characterized by profound ana lgesia, immobility, amnes ia with only light sleep. T he patient appears to be conscious, i.e. opens his eyes, makes swall owing movements a nd his muscles a re rigid, but he is un able to process sensory stimuli and does not react lo them. Thus, the patient appears to be dissociated from his body and surro undi ngs. T he primary s ite of action of ketamine is in the cortex and s ubco rtical areas, but not in the reticu lar activating system, which is the si te o f action of barbiturates. Ketam ine acts mainly by inhibit ing the NM DA type of excitatory amino acid receptors in the brain. and not by interacting wi th GABAA receptor. Respiration is not depressed, bronchi di late, airway re n cxes are mai ntai ned, muscle tone increases. Non-pu rposive limb movements occur. Heart rate , cardiac output and BP are elevated due to sympathetic stimulation. Ketamine is hig hly lipid soluble a nd acts rapidly. A dose of 1- 2 (average 1.5) mg/kg i.v. or 3- 5 mg/kg i.m. produces the above effects within a minute, and recovery starts a fter I0-15 min. but the patient remains amnesic fo r 1- 2 hr. Emergence delirium, halluc inations, fear reactions and invo lunta ry movemen ts occur in u pto 50% patients

during recovery. However. Ketamine injection is not painful. Children tolerate the drug bette r and emergence reactions are milder. Ketamine is rapidly metabolized in the liver and has an elimination t½ of 2-4 hr. Ketamine has been used for operations o n the head and neck, in patients who have bled, in asthmatics (relieves bronchospasm), in those who do not want to lose conscio usness and for short operations. It is good fo r repeated use; particularly suitable for bum dressing. Combined w ith diazepam/midazolam it has found use in an giograp hi es, ca rdi ac catheteriz ation and trauma surgery. However, Ketamine can be dangerous for hypertensives, in ischaemic heart disease ( increase cardiac work), in congestive heart fa ilure a nd in those w ith ra ised intracra nial pressure (ketamine increases cerebral blood flow and 0 2 consumption), but is good for hypovolemic patients. KETM l"J. KETAMAX. ANEKET 50 mg/ml in 2 ml amp, 10 ml vial.

C landestine ly mixed in drinks. ketamine has been misused as rape drug.

3. Fentanyl Th is highly lipophilic, short acti ng (30-50 min) potent opioid analgesic related to pethidine (see C h. 34) is generally given i.v. at the beginn ing of painfu l surgica l procedure . Reflex effects of pa inful stimuli are abolished. It is frequently used to supplement anaesthetics in ba la nced anaesthesia. This perm its use of lower a naesthetic concentrations w ith better haemodynamic stability. Combined w ith BZDs, it can obviate the need fo r inhaled anaesthetics for d iagnostic, e ndoscopic, angiographi c and other minor procedures in poor risk patients, as well as for burn dressing. A naesthetic awareness with dreadful recall is a risk. After i.v. fe ntany l (2-4 ~1g/kg) the patient remains drow sy but conscio us a nd hi s coope ration can be com manded. Respirato ry depression is marked. but predictable; the patient may be encouraged to breathe and ventilatory assistance may be provided. Tone of c hest muscles and masseters may inc rease w ith rapid fe nta nyl injection: a muscle relaxant is then required

411

412

DRUGS ACTIN G ON CE NTRA L NERVOUS SYSTEM to facilitate mechanical ventilation. Hea rt rate decreases, because fen tany l stimulates vagus. Fall in BP is slight and heart is not sensitized to Adr. Cerebral blood flow and 0 , consumption are slightly decreased. Supplen{ental doses of fentanyl are needed every 30 min or so, but recovery is prolonged afler repeated doses. Nausea, vomiting and itc hing often occurs during recovery. The opioid antagonist naloxone can be used to counteract persisting respiratory depression and me nta l c louding. Fentanyl is also employed as adjunct to spinal and nerve block anaesthesia, and to re lieve postoperative pain. TROFE TYL, FENDOP, FE T 50 µg/ml in 2 ml amp, 10 ml vial. A/fenlanil and Sufe111a11il arc other shoner acting analogues which can be used in place of fentanyl.

Remifentanil It is a faster act ing congener of fentanyl with a shorter and more predictable duration of action. It is similar to fenta nyl in ana lgesic potency, respiratory depressant action a nd other properties, includ ing potential to cause spasm of c hest muscles. After i. v. injection it produces a nalgesia within 1- 2 min which fades in I 0-15 min. The duration of action is not changed even afier prolonged i.v. infusion. The very brief action is due to rapid metabolism by plasma and tissue estera es to yield a plasma t½ of 10- 15 min. The dose and duration of action are not altered by li ver or kidney disease, because remifentanil e limination is not dependent on these organs. Administered by bolus i. v. injection followed by conti nuous infusion remifentanil is mai nly used to provide strong a nalgesia to cover short and pai n ful procedures. It is particul arly suitable when ti tra table action a nd qu ick recovery are desired. Dose: 0.5 mg/kg/min i.v, infusion. followed by 0.250.5 mg/kg/min. ULTIVA I mg/3 ml. 2 mg/5 ml and 5 mg/10 ml inj.

4. Dexmedetomidine

Activation of central a 2 adrenergic receptors has been known to cause sedation and ana lgesia. Clonidine (a selective a 2 agonist antihyperten ive) g iven before surgery reduces anaesthetic requirement. Dexmedetomidine, the acti ve -enantiomer of medetomidine, is a centrall y acting selective a 2A agonist that has been introduced for sedating

critically il l/ventilated patients in intensive care units. It is also being used to sedate patients during endoscop ies, spinal/epidural/regional anaesthesia and as adjunct to GA by lowering the dose of the inhalationa l anaesthetic. On i.v. infusion of 0.5- 1 µg/kg/hour, it produces sedation and a na lgesia with little respiratory depression , amnesia o r anaesthes ia. Sympathetic response to stress and noxious stimulus is blun ted. Dex medetomid ine is metabo lized rapid ly by the liver w ith a t½ of 2- 3 hour and excreted in urine as we ll as in bile. Side effects are s imilar to those w ith clonidine, viz. hypotension, bradycardia and dry mouth. DEXDI E, DEXEM. XAMDCX 100 µg/ml inJ, I ml and 2 ml amp.

CONSCIOUS SEDATION 'Consciou!. !.edation· is a monit0red state of altered conscious ness that can be employed (supplemented with local/ regiona l anaesthesia), to carryout diagnostic/short therapeutic/dental procedures in apprehensive subjects or medically compromised patients, in place of general anaesthesia. It allows the operative procedure to be performed with minimal physiologic and psycholog ic stress. In conscious sedation. drugs arc used to produce a state of CNS depression (but not unconsciousness). sufficient to withstand the trespass of the procedure, while ma intaining communication with the patient, who at the same time responds to commands and is able to maintain a patent airway. The di !Terence between conscious sedation and anaesthesia is one of degree. The protective airway and other reflexes are not lost, making it safer. Drugs used for consciou sedation are: diazepam, midazolam, nitrous oxide. dexmedetomidine and fcntanyl.

COMPLICATION S OF GENERA ANAESTHES IA A. During anaesthesia I. Respiratory depression and hypercarbia. This is more prominent with isofl urane; may be countered by providing ventilatory assistance by the anaesthetist. 2. Salivation, respiratory secretions and depressed mucociliary fu nction in the airway. This is less problematic with modem anaesthetics. 3. Cardiac arrhythmias, asysto le. 4. Fa ll in BP. 5. Asp iration o f acidic gastric contents. 6. Laryngospasm a nd asphyxia.

GENERAL ANAESTHETICS 7. Awareness: dreadful perception and recall of events during surgery. This may occur due to use of light anaesthesia. 8. Deli ri um , convulsions and other excitatory effects: generally seen with i.v. anaesthetics.

B. After anaesthesia I. Nausea and vom iting. 2. Persisting sedation: impaired psychomotor function. 3. Mucus plugging or airways, ate lectasis an d pneumonia, especially after prolonged anaesthesia. 4. Organ toxici ties: li ver, kidney damage. 5. erve palsies-due to faulty posi tioning. 6. Emergence delirium. 7. Cogni tive defects: prolonged excess cognitive decline has been observed in some patients, especially the elderly, who have undergone general anaesthesia, particularly of long duration.

DRUG INTERACTIONS 1. Patients on anti hypertensives given general anaesthetics- BP may fa ll markedly. 2. Neuroleptics, opioids and clonidine potentiate anaesthetics. 3. Halothane sensitizes the heart to Adr. 4 . Ir a patient on corticosteroids is to be anaesthetized, give 100 mg hydrocortisone intraoperatively because anaesthe ia is a stressful state--can precipitate adrenal insuffic iency and cardiovascular collapse. 5. Insulin need of a d iabetic is increased during GA: switch over to plain insu lin even if the patient is on oral hypoglycaemics.

PREANAESTHETIC MEDICATION Preanaesthetic medication refers to the use of drugs before anaesth esia to make it safe and less unpleasant. T he aims are: I. Relief of anxiety and apprehension preoperatively and to facilitate smooth induction. 2. Amnesia for perioperative events. 3. Supplement analgesic action of the anaesthetics and potentiate it so that les anaesthetic is needed.

4. Decrease secretions a nd vagal stimulation that may be caused by the anaesthetic. 5. A ntiemetic effect extend ing into the postoperative period. 6. Decrease ac id ity and vol ume of gastric juice so that it is less damaging if aspirated. Differe nt d rugs ach ieve different purposes. One or more drugs may be used in a patient depending on the needs.

1. Sedative-antianxiety drugs Benzodiazepines like diazepam (5- 10 mg oral) or lorazepam (2 mg oral or 0.05 mg/kg i.m. I hour before) have become popular drugs for preanaesthetic medication because they produce tranquility and smoothen induction. There is loss of recall of perioperative events (especial ly with lorazepam) with little respiratory depression or accentuation of postoperative vomiting. They counteract C S toxicity of local anaesthetics an d are being used along with fentanyl fo r a variety of m inor surgical and endoscopic procedures. Midazolam is a good a mnesic with potent an d shorter lasting act ion. It is a lso better suited for i.v. injection, due to water solubility. Promethazine (50 mg i.m.) is an antihistaminic wi th sedative, antiemetic and anticholinergic properties. It causes little respiratory depression, except in young children. 2. Opioids

Morphine ( 10 mg) or pethidine (50- 100 mg), i.m. allay anxiety and apprehcn ion of the operation, produce pre- and postoperative analgesia. smoothen induction, reduce the dose of anaesthetic required and s upplement poor analgesics (thiopentone. halothane) or weak anaesthetics (N,O). Postoperative restlessness is also reduced.

Disadvantages

Opioids depress res piration, interfere with pupi llary signs of anaesthesia. may cause fall in BP during anaesthesia, ca n precipitate asthma and lend LO delay recovery. Other disadva111agcs arc flushing. delayed gasrric emptying and biliary spasm. Morphine particularly contributes 10 1>0s1opcrntive cons1ipa1ion, vomiting and urinary retention.

Use of opioids is now mostly restricted to patients havi ng preoperative pain. When indicated, fentanyl is mostly injected i. v. just before induction.

3. Anticholinergics

(see Ch. 8) Atropine or hyoscine (0.6 mg or I 0-20 µg/kg i.m./i.v.) or g lycopyrrolate (0.2-0.3 mg or 5- 10 µg/kg

413

414

DRUGS ACTING ON CENTRAL NERVOUS SYSTEM i.m./i.v.) have been used, primaril y to reduce salivary and bronchia l secre tions. This need is infrequent now d ue to use of non- irrita nt anaesthetics. The main a im of using antic holinerg ics now is to prevent vaga l bradycardia and hypotension (which occur reflexly due to certai n surgical procedures), and prophylaxis of laryngospasm which is precipitated by resp irato ry secretions. Hyoscine, in a ddition, produces amnesia and antie metic effect, but tends to delay recovery. Some patients get di so rien te d ; emergence del irium is mo re comm on. T herefore, it is infrequently used.

G/ycopyrrolate is twice as potent and longer acting quaternary ant imuscarinic w hich does not produce central effects. Antisecretory action is mo re marked than atropine and tachycardia is less marked. It acts rapidly when given i.v. and is the preferred a ntimuscarinic in a naestheti c practice. Action

Atropine

G/ycopyrrolate

1. Antisecretory

++

+++

2. Tachycardia

+++

++

3. CNS effects

+

4. Bronchodilatation

++

++

Antimuscarinics fac ilitate assisted ventilation by reducing airway res istance, but tend to increase the anatomic dead space. T hey dilate pupils, abolish the pupi llary sig ns and increase chances of gastri c reflu x by decreasing tone of lower esophageal sphincter (LES). They should not be used in febrile patients. Dryness of mouth in the pre- a nd postoperative period may be distress ing. As suc h, they are now mostly used i.v. intraoperatively when need arises. 4. Neuroleptics Ch lorpromazine (25 mg), 1riflupromazine (10 mg) or halopcridol (2-4 mg) i.m. are occasionally used

in prcmcdication. They a llay anxiety, s moothen induction and have an1iemetic action. However, they accentuate respi ra10ry depression and hypotcnsion caused by the anaesthet ics and delay recovery. Involuntary movements and muscle dystonias can occur, especially in childre n.

5. H2 blockers/proton pump inhibitors Pat ie nts undergoi ng prolonged o p erations, caesari an section a nd obese patients are at inc reased ri sk of gastr ic reg urgitatio n and aspi rat io n pn eumonia. Ranitidine ( 150 mg)/ fa motidine (20 mg) o r omep razo le (20 mg)/ pantoprazole (40 mg) given nig ht before and in the morning benefit by ra ising the pH of gastri c juice. They may a lso red uce its vo lume, thereby decreasing the c hances of regurgitation. The ri sk of refl ux and damage to lungs on aspiration is minimal if the volume of gastri c juice is 3.5. Prevention of stress ulcers is another advantage. H 2 blockers/PPis are now ro utine ly used before prolonged surgery.

6. Antiemetics

Metoclopramide I 0-20 mg i.m. preoperatively is effective in reducing postoperative vomiting. By enhancing gastric emptyi ng a nd to ne of LES, it reduces the chances of reflux and aspiration. Extrapyramidal side effects and motor restlessness can occur. Combined use of metoclopramide and H2 blockers is more effective. Domperidone is nearly as effective as metoc lopramide and does not produce extrapyramidal side effects. Ondansetron (4-8 mg i.v.) the selective 5-HT 3 blocker has been found hig hly effective in reducing the incidence of post-anaesthetic nausea and vom iting (see Ch. 48). It is practical ly devoid of side effects and has beco me the antiemetic of choice in anaesthetic practice.

Chapter

28

Ethyl and Methyl Alcohols

ETHYL ALCOHOL (Ethanol) A lco hols are hydroxy derivatives of aliphatic hydrocarbons. When unqualified, 'alcohol' refers to e1hyl alcohol or ethanol. Pharmacology of a lco ho l is important fo r its presence in beverages (which have been used since recorded history), a lcoholism and for a lcohol intoxicati on, rather than as a med ici nal substance. A Icoho I is man u factured by ferme ntation of sugars: Zymase

C6 H120 6 - - - -• 2C02 + 2C2 H5 0H Glucose

(in yeast)

Ethanol

Fermentation proceeds ti ll alcohol conten t reaches - 15%. Then the reaction is inhibited by a lcohol itself. Starc hy cereals, e.g. barley, when soa ked produce malt:

light "'ines

C laret, C ider; alcohol content 9- 12%, cannot exceed 15%.

Fortified wines

Pon, Sherry (alcohol 16-22%): distilled beverages are added from outside to the wine.

Effervescent wines Champagne ( 12-16% alcohol): these are bottled before fermentation is complete: the C01 produced after bottcling remains dissolved under pressure. Wines are called 'dry· when all sugar presen t has been fermented and 'sweet · when some is left.

C. Spirits

These are produced by distillat ion of the fennented broth; e.g. Rum, Gin, Whiskey, Brandy, Vodka, etc. Though the alcohol content of these beverages can vary from 40-55%, in India (and almost internationally) for all licenced brands it is standardized to 42.8% v/ v or 37% w/ w. The taste. fl avour and va lue of various alcohol ic beverages depends not on ly on alcohol conten t but on the presence of higher ethers, higher a lcohols, aldehydes, esters, polymers, and volatile oi ls; many of which are formed during 'maturation' of the beverage.

Convertase STARCH

MALTOSE

which can the n be fermented by yeas t to produce a lcohol. The major source o f commercial alcohol is mo/lases, the leftover byproduct of sugar industty. ALCOHOLIC BEVERAGES There are a large variety of alcoholic beverages. A. Malted liquors Obtained by fermentation of germinating cereals; these are undistilled- alcohol content is low (3-6%) e.g. Beers, Stout. Now strong beers (upto I 0%) are also available. B. Wines Produced by fermentation of natural sugars as present in grapes and other fruits. These are a lso undistilled.

Other forms of alcohol I. Absolute alcohol alcohol).

99% w/ w ethanol ( dehydrated

1. Rectified spirit 90% w/w ethyl a lcohol produced from fermented mollases, by distillation. 3. Proof spirit It is an old term. If whiskey is poured on gun powder and ignited and it explodes, then it was labelled to be of ·proof strength'. If water is mixed to it, the gun powder will not ignite. 100% proof spirit is 49.29% w/ w or 57. I% v/ v alcohol. 4. Methylated spirit (industrial) Also called 'denatured spirit' is produced by adding 5 pans of wood naphtha (methyl a lcohol) to 95 pans of rectified spirit so as LO render it unfit for drinking. It is tinted blue by methylene blue dye for distinction. It can be applied on the skin for antiseptic, cleaning and astringent purposes.

416

DRUGS ACTING ON CENTR AL NERVOUS SYSTEM

PHARMACOLOGICAL ACTIONS

1. Local actions

Ethano l is a mild rubefac ient and counte rirritant when rubbed on the skin. By evaporation it produces cooling. Applied to delicate skin (scrotum) or mucous membranes it produces irritation and burning sensation. Concentrated a lcohol (spirit) should not be appl ied in the mouth, nose, etc. Injected s.c. it causes in tense pain, inflammation and necrosis followed by fibrosi s. Injected around a nerve it produces permanent nerve damage. Applied to the surface, alcohol is an astri ngent- precipitates surface proteins and hardens the skin. By precipitating bacterial proteins it acts as an antiseptic. The antiseptic action increases with concentration from 20 to 70%, remains constant from 70 to 90% and decreases above that. That I 00% ethanol is more deh ydrating but poorer antiseptic than 90% ethanol, shows that antibacteria l acti on is not due to dehydration of bacterial protoplasm. Alcohol does not kill bacterial spores.

2. CNS A lcohol is a neurona l de pressant. S ince the highest areas are most easily deranged an d th ese are primarily inhibitory- apparent excitation and euphoria are produced at lower plasma concentrations (30-60 mg/di). Hesitation, caution, self-critic ism and restraint are lost first. Mood and feelin gs are a ltered; anxiety may be allayed. A t 50- 100 mg/di some ind ividua ls may experience what is labelled as ' high'. With increasing concentration ( I00- 150 mg/di) me nta l c loud ing, disorgani zation of thought, impai rment of a ttention, memory and oth er faculties, alteration of gait and perception, and drowsiness superve ne. At 150-200 mg/di the person is sloppy, atax ic and drunk, ' black-outs' occur; 200-300 mg/di result in srupor and above th is unconsciousness prevails, medullary centres are paralysed and death may occur. T hough,

a lcohol can prod uce anaesthesia, margin of safety is narrow. Any measurable conce ntration of alcohol produces a measurab le s lowing of reflexes: driving is dangerous. Performance is impaired, fine discrimination and precise movements are obliterated; errors increase, except if fear of punishment and anxiety of failure has already impaired it. Under such situation performance may be improved by allaying of anxiety and fear. At any given blood a.lcohol level, central effects are more marked when the concentration is rising than whe n it is fa lling. T his is considered to be a manifestation of acute tolerance. Alcoho l ca n induce s leep but is not a dependable hypnotic. Some individuals report poor quality of sleep and repeated or early morning awaken ing. S leep a rchitecture may be disorgani zed and s leep apnoea aggravated. 'Hangover' ( headache, dry mouth, laziness, di sturbed mood, impaired performance) may occ ur the next morn ing. A lcohol ra ises pain threshold and a lso alters reaction to it, but is not a dependab le analgesic- severe pain can precipitate confusion and convulsions. During the time alcohol is actin g on the brain, it exerts anticonvulsant action, but th is is followed by lowering of seizure threshold: seizures may be precipitated in epileptics. Chronic alcohol abuse damages brain neurones, causes shrinkage of brain. The cortex and the reticular activating system are the most sensiti ve to alcohol; other areas get depressed as concentration rises. Mechanism of action Alcohol was believed to produce CNS depression by a generalized membrane action altering the state of membrane lipids. However, lately specific effect on multiple receptor operated and voltage gated ion channels as well as other critical prote ins has been demons trated al concentrations attained during moderate drinking. Thus, several neurohumo ral systems are concurrently affected producing a complex pattern o f action quite different from

Alcohol

: conscious - - - - -- - - - - --

Ether

: conscious - - - --

anaesthesia- death.

anaesthesia - - - - - - - --+ death.

ETHYL AND M ETHYL ALCOHO LS that produced by other depressants like barbiturates and benzodiazepines, which predominantly facilitate GABAA receptor mediated Cl- channel opening. Alcohol has been shown to enhance GABA release at GABA. sites in the brain. It also inhibits N 1 DA type of excitatory amino acid receptors (operating through cation channels) which has been implicated in memory impairment caused by alcohol. Action of 5-HT on 5-HT 1 inhibitory autoreceptor (havi ng an intrinsic ion channel) is augmented. Some studies suggest that cerebral nicotinic cholinergic receptor (operating through a· channel) may also be one of the targets of alcohol action. Ethano l can indirectly reduce neurotransmitter release by inhibiting voltage sensitive neuronal Ca2• channels. It also activates specific type of K channels m certain brain areas. Release and turnover of DA in brain is enhanced through endorphin release in nucleus accumbens and an op ioid receptor dependent mechanism. This is probably important in the pleasurable reinforcing effects of alcohol and in the genesis of alcohol dependence. Activ ity of membrane bound enzymes like a· K ATPase and adenylyl cyclase is also altered. The activity and translocation of channel/enzyme proteins in the membrane could be affected by alcohol through protein kinase C ( PKC) and protein kinase A (PKA) mediated alteration in the state of their phosphorylation.

3. CVS The effects of alcohol are dependent on the dose.

Small doses: produce only cutaneous(especially on the face) and gastric vasodilatation. Skin is warm and fl ushed and there may be conjunctiva) injection; BP is not affected.

Moderate doses: cause tachycardia and a mild rise in BP due to increased muscular activity and sympathetic stimulation. Large doses: cause direct myocardial as well as vasomotor centre de pression a nd there is fall in BP. Epidemiologica l sn1dies have confirmed that chronic alcoholism contributes to hypertension and can lead to cardiomyopathy. Atria l fibrillation and other cardiac arrhythmias may occur due to conduction defects and Q-T prolongation.

4. Blood Reg ular intake of smal l to moderate a mounts of alcohol ( 1- 2 drinks) has been found to raise H DL-c ho lesterol leve ls and decrease LDL ox idation . T his may be responsible for the 15-35% lower incidence of coronary artery disease in suc h individuals. Ri sk reduction is g reatest in high ri sk subjects and protection is lost if 2: 3 drinks arc consu med daily.

However, it is considered inappropriate to advise nondrinkers to start drinking on this account, si nce other adverse consequences may more than nullify this benefit. Mild anaemia is common in chronic alcoholics. Megaloblastic anaemia occ urring in chronic alcoholism is due to interference with folate metabolism.

5. Body temperature A lcohol is reputed to combat cold. lt does produce a sense of wam1th due to cutaneous and gastric vasodilatation, but heat loss is actually increased in cold surro undings. Drinking before going out in the cold is inad visable. High doses depress tem perature regulati ng centre.

6 . Resp iration

Bran d y or w hi s key are reputed as respiratory stimula nts in collapse. They irritate bucca l and phary ngeal m ucosa which may transiently stimulate res piration reflexly. H owever, it is better not to depend on thi s, because the di rect action of alcohol on respiratory centre is only a depressant one.

7. GIT Alcoho lic beverages have variable effect on gastric secretion depending on the beverage itself and whether the individual likes it. However, dilute a lcohol (optim um I 0%) put in the stomach by Ryle 's tube is a strong stimulant of gastric secretion (espec ia lly of acid). It acts directly as well as reflexly. Higher concentrations (above 20%) inhibit gastric secretio n, cause vomi ting, mucosa! congestion and gastritis. Al coholism is an important cause of chronic gastritis. Lower esophageal sphincter (LES) tone is reduced by alcohol. Drinking may accentuate gastric refl ux . Bowel movements may be altered in e ithe r direction. Acute pancreatitis is a complication of heavy drink ing.

8. Liver either brief alcohol intoxication nor c hroni c intake o f small-to-moderate a mounts cause significant liver damage, provided adequate nutriti on is maintained. However, it docs mobilize peripheral fat and increases fa t synthesis in live r in a dose-dependent manner producing 'fatty li ver '. This is reversible, but may progress to c irrhosis if prolonged or excessive. Proteins may also accumulate in liver beca use their secretion is decreased. Chronic alcoholism

417

418

DRUGS ACTING ON CENTRAL NERVOU S SYSTEM

exposes liver to oxidative stress and causes cell ular necrosis fo llowed by fibrosis. Acetaldehyde produced during metabolism of alcohol appears to damage the hepatocytes and induce inflammation, especia lly on chronic ingestion of large amounts. Increased lipid peroxidation and glutathione deple tion occurs. These combined with vitamin and other nutritional deficiencies may be responsible for the so called alcoholic cirrhosis. Regular alcohol intake induces microsomal enzymes. 9. Skeletal muscle Alcohol produces little direct efTcct. Fatigue is allayed by smal l doses, but muscle work is increased or decreased depending on the predominating central e fTect. Weakness and myopat hy occurs in c hronic alcoholism. 10. Kidney Diures is is often noticed after alcohol intake. This is due to water ingested along with drinks as well as alcohol induced inhi bition of ADH secretion. A lcohol does not impair renal fu nction. 11 . Sex Alcohol is reputed as an aphrodisiac. Though, loss of restra int and inhi bition may provoke sexual behaviour in some individuals or on some occasions, the re is no uni fo rm effect of alcohol on sexua l desire. Moreover, performance of the sex ual act is often impai red. Chronic alcoholism can produce impotence, testicular atrophy, gynaecomastia and infert ility in both men and women.

12. Endocrine effects Moderate amounts of alcohol increase Adr release which can cause hyperglycaemia and other sympathetic effects. However, acute intoxication is often associated with hypoglycaemia and dep letion of hepatic glycogen, because gl uconeogenesis is inhibited. Glucago n, therefo re fails to reve rse alcohol ind uced hypoglycaemia, and glucose must be given to counteract it.

13. Ute rine contractions are s uppressed at moderate blood levels.

PHARMACOKINETICS

Rate of alcohol absorption from the stomach is dependent on its concentration, presence of food, and other factors, but is generally qu ite slow. Absorption from intestines is very fast; peak levels after ingestion in the fasting state are attained after ~30 min. Thus, gastric emptying determines the rate of absorption. Limited first pass metabolism occurs in stomach and li ver. Absorption of alcohol from sk in of adults is minimal but may be significant in infants given alcohol sponges. Alcohol gets distributed widely in the body (vo l of dist ribution 0.7 L/kg, i.e. equal to volume of total body water), crosses blood-brain barrier efficiently: concentration in the brain is very near blood concentration. It also crosses placenta freely. Alcohol is oxidized in liver 10 the extent of 98%. Even with high doses, not more than I 0% escapes metabolism. The primary pathway sequentially utilizes alcohol dehydrogenase and aldehyde dehydrogenase. Alcohol Aldehyde Ethyl - -- • Acetaldehy d e - - -• Acetate_..co,+ alcohol dehydrodehydroH,O

genase

genase

Alcohol dehydrogenase exhibits considerable genetic variability which is reflected in the rate at which individuals metabolize alcohol, and probably governs their susceptibility to alcohol. In add ition to alcohol dehydrogenase, small amounts of alcohol are ox idized by hepatic mi c rosomal enzymes ( ma inly CY P2E l ) as well. Metabolism of alcohol follows zero order kinetics, i.e. a constant amo unt (8- 12 ml of absolute alcohol/ hour) is degraded in unit time, irrespective of blood concentration. Thus, the rate of consuming drinks governs whether a person will get drunk. Excretion of alcohol occurs through kidney and lungs, but neither is quantitatively signifi cant. Concentration in exhaled air is about 0.05% of blood concentration. This is utilized for medicolegal determination of drunken state. The s ubject blows in a portab le hand-he ld

ETHYL AND METHYL ALCOHOLS breath-analyser which measures the alcohol content of exhaled air and displays the resuh in terms of coJTcsponding blood a lcohol concentration (mg/di).

INTERACTIONS I. Alcoho l synerg ises wi th a nx iol ytics, antidepressants. antihistaminics, hypnotics, opioids, so that marked C S depression with motor impairment can occ ur. Chances of accide nts increase. 2. Some indi v iduals taking a sulfonylurca, cefoperazone, or metronidazole have experi e nced bizarre, so mewhat disul firam-1 ike reactions when they consume a lcohol. This reaction occurs only in few individuals and its basis is unclea r. It passes off with time as a lcohol is metabolized. Only reassurance a nd supportive treatment is needed. 3. Ac ute alcoho l ingestion inhibits. whi le chronic in take induces CY P enzymes (especially CYP2E I). Formation of toxic metabolite o f paracetamol (NA BQI) is increased in chronic alcohol ics (see p. 224). Safe dose limit of paracetamol is lower in them. Meta bolism of tolbutamide, phenytoin and some other drugs is similarly affected by acute and chronic alcohol intake. 4. Hypoglycaemic action of insuli n and sulfonylureas is enhanced by alcohol ingestion. 5. Aspirin and othe r NSA IOs cause more gastric bleeding when taken with alcohol.

Food value Al cohol requires no digestion and is metabolized rapidly. It is an energy yielding substrate: 7 Cal/g, but these cannot be stored. However, it spares carbohydrates and fats as energy source, so that regular intake can contribute 10 obesity. Alcohol docs not supply body building and other essential constituents of food. Those who consume substantial pan or their caloric intake as alcohol, often suffer from nutritional deficiencies. Thus, alcohol is an imperfect and expensive food.

CONTRAINDICATIONS Alcohol is seldom prescribed medically. However, it is rampently consum ed. In take of alcohol should be avoided byI. Patients with peptic ulcer, hyperacidity and gastroesophageal reflux disease (alcoho l increases gastric secretion and relaxes LES). 2. Epileptics: seizures may be precipitated. 3. Severe li ver disease patients. 4. Unstable pe rsona lities: they a re likely to abuse it and become excessive drinkers. 5. Pregna nt women: Even moderate dri nking during pregnancy can produce foetal alcohol syndrome resulting in intraute rine and postnatal growth retardation, low IQ offspring, microcepha ly, cranio-facial (flat face) and other abnormal ities, and imm uno logical impairment causi ng increased susceptibility lo infections. Heavy drinking durin g pregnancy, in addition, increases the incidence of miscarriage, stillbirths and low birth-weight babies.

Guidelines for safe drinking

Alcoholic beverages are wide ly consumed, and in many cultures they a re served as a part of social courtesy. Moderate drinking is generally considered socially acceptab le. Physicians are often asked 10 advi e on safe ways of drinking. Various official agenc ies, physician organizat ions and alcoholism ex perts have putlorth guidel ines in this regard , but they arc not uniform. T he following may be concluded: • On an average 1- 2 drinks per day is usua lly safe. 1ot more than 3 drinks on any one occasion. • Consumption of >3 drinks per day is associated with documented adverse health effects. • Do not drive or engage in hazardous activities after drinking. • Do not drink if an inte racting drug has been taken. • Subjects wi th any contraindi cation should not drink. • Safe limits are somewhat lower for women than for men, because metabolism of a lcohol

419

420

DRUGS ACTING ON CENTRAL NERVOUS SYSTEM is slower and its bioava ilabili ty higher (due to less first pass metabolism in stomach) in wome n than in men. (Note: I drin k = 50 m l of spirits = 150 ml of w ines = 400 ml of bee r; a ll have ro ughly 16 g a lcohol, wh ich taken in empty stomach produces a peak alcohol blood level of ~ 30 mg/di in an adult ma le of average built.]

TOXICITY A. Side effects of moderate drinking ausea, vomi ting, Aushing, hangover, traffic accidents. B. Acute alcoholic intoxication Unawareness, unresponsiveness, stupor, hypote nsion, gastriti s, hypoglycaemia, res piratory depression, collapse, coma and death.

Treatment: Gastric lavage is he lpful o nly w hen the pa tie nt is bro ug ht soon after inges ting a lcohol, which is rare. Since most patie nts are disoriented o r comatose, the first priority is to maintai n patent a irway a nd prevent aspiration o f vomitus. Tracheal intubat ion and positi ve press ure res piratio n may be neede d if it is markedly depressed . Ana leptics shou ld not be given. They may precipitate convulsions. Most patients w ill recover with supportive treatment, mainte nance of fl uid and electrolyte ba la nce a nd correctio n of hypoglycaem ia by gl ucose infusio n till a lco ho l is meta bolized. T hiamine ( 100 mg in 500 ml glucose solution infused i.v.) should be added. Reco very can be hastened by haemodialysis. Insulin + fructose drip has been fo und to accelerate a lcohol metabolism. However, its c linical impact is not remarkable. C . Chronic alcoholism On chroni c inta ke, tole rance develops to s ubjective and behavioural effects o f alcohol, but is generally of a low degree. It is both pharmacokine tic (reduced rate of abso rption du e to gastritis and faster metabolism due to e nzyme induc tion) and cellular tolerance. Mild addiction occurs in some subjects even w ith moderate drinking. 1t depends a lo t on the indi vidua l's likings and attitudes. A ddi ct ion is manifes ted in alcoho l-seeking be havi o ur, and the priority th at the s ubj ect

accords to o btai n ing and consu ming alcohol over o ther needs, o r the extent to w hic h he wil l go fo r mainta ining a lcoho l intake. Recent studies have confirmed that a genetic basis contributes to progression from social drinking to alcoholism in about 50% individuals. Alcoholism is often a familial trait. Some differences in sensitivity of various neuronal systems to alcohol among ' predisposed· and · not predisposed' individuals have been demonstrated. There is no single explanation for why people drink. Diverse feelings and behaviours are provoked by alcohol in different individuals and in the same individual on different occasions. Alcohol can make people happy as well as sad, cunious as well as mean, talkative as well as silent, friendly as well as hostile. All this cannot be explained on the basis of pharmacological action of alcohol alone. Attitudes, beliefs, peer groups. social setting and learned experiences a ll have a bearing. Alcohol is said to produce good mood, sense of wellbeing. self confidence, sociability, etc. But these infact are learned behaviours. In some societies. alco holic beverages have become an acceptable form of extending counesy and of entenainmcnt. Drinking is ot1cn related to ·celebration· and · high living' . T here is ' wine snobbery' in high social groups. To some individuals. excess drinking provides the exc itement of ris k taking. People often boast of t he ir capacity to drink. To the young, d rinking may be a symbol of rebellion against the oppressive older generation and rejection of the values of tbe establishment. · Bing~ drinking ' is a specific behavioural pattern o f bouts of excessive drinking. Alcohol is o t1cn an e:-.cuse for bad behaviour. Society's view that intoxicated person is unaware of his actions (therefore not responsible) makes intoxication an anractive state, because there is increased freedom of what one can say or do at1er drinking. T hus, there are a variety o f motivations for drinking.

A lcohol depe ndence occurs only on heavy and round- the-c lock d rinking (if a lcohol is present in the body continuously). Heavy d rin king is often associa ted w ith nutritio nal deficienc ies, because food is neglected and ma labsorpt ion may occur. In addition to impai red menta l and phys ical performance, neurological a fflictions are common- polyneuritis, pellagra, tremors, seizures, loss o f brain mass, Wernic ke 's encepha lopathy, Korsakoff's psychosis and megaloblastic anaem ia. Thia mine deficiency is very common in chroni c a lcoholism; and h ig h dose thiamine injection partly rever es many neuro logica l sym ptoms. Alcoholic c irrhosis of liver, hype1tension, cardiomyopathy, CH F, arrhythmias , stroke, acute pancreatitis, impotence, gynaecomastia, in fctt ility

ETH YL AND METHY L A LCOHOLS and ske letal myopathy are the othe r complications. Incidence of oropharyngeal, esophageal and hepatic malignancy and respiratory infections is high; immune function is depressed .

Withdrawal syndrome

Wh e n an a lc oho l depende nt subj ect stops drinking, withdrawa l syndrome appears within a day. Its seve rity depe nds on the duration and quanti ty of alcohol consumed by the subject. It cons ists o f anxiety, sweating. tachycardia, tre mor, impai m1ent o f sleep, c onfus ion, ha lluc inat ions, delirium tremens, convulsions and collapse.

Treatment Psycho logical and medical supporti ve measures are needed during wi thdrawal. Many CNS depressants like barbitura te , phenot hiazi nes, ch lo ral hy d rate have b een u sed as substitution therapy in the past (to suppress withd rawa l synd rome) but benzod iaze pines (chordiazepox ide, diazepam) are the pre fe rred drugs now. These drugs have a long duration of acti on and can be gradually withdrawn later. Naltrexone: Several studies have demonstrated in volvement of opio id system in the pleasurable re inforcing e ffects of alcoho l through dopam ine mediated reward function. Post-add ict subjects treated with the long-acti ng opioid antagonist na ltrexone (see Ch. 34) do not experience the sa me p leasurab le e ffects on ta king a lc oho l; reinforcement is weakened. Trials ha ve shown tha t it he lps preve nt re la pse o f alcohol ism . Naltrexone reduced alcoho l craving, number of d rinking days and chances of resumed heavy drinking. It is approved fo r use as adjuvant in comprehensive treatment programmes fo r a lcohol dependent subjects and is be ing used in Ind ia at most deaddiction centres, after the individual has undergone w ithd rawa l and is motivated. Acamprosate It is a "eak M DA- receptor antagonist with modest GA B A A recepto r agonistic activity tha t is being used in USA, UK and Eu rope for maintenanci, therapy of alcohol abstinence. In conjunction with social and mo tivationa l therapy. it has been found to reduce relapse o f the drinking behaviour. The efficacy of acamprosatc in this regard is rated comparable 10 naltrexonc. It should be started soon after withdrawing alcohol and then given cominuously al a dose of 666 mg 2- 3 times a day. Loose motion is a common side effect. Others arc nausea, ahdomina l pain and itching.

The 5-HT, antagonist 011dame11v11 and the antiepileptic topiramate have also shown some promise in treating alcoholism.

CLINICAL USES Medici na l uses o f e thanol are primari ly restricted to exte rna l app lic ation and as a vehic le fo r liquid preparations used internally. I. As antiseptic (see C h. 67). 2. Rubefacie nt and counteri rri tant for sprains. joint pains, etc. Spir it is genera lly used as veh icle fo r other ingredie nts. 3. Ru bbed into the skin to prevent bedsores; works by ha rd en ing the ski n du e to its astringent property. However, a lcohol should not be applied on a lready fo nned sores. T he astringent action of alcohol is a lso utilized in antiperspirant and a ftershave lotions. 4. Alcoho lic sponges have been used to reduce body temperatu re in fever. However, co ld water/ice may be bette r. 5. Intractable neura lgias (trigemina l and others), se ve re c ance r pa in: Inj ect ion of alcoho l round the nerve causes permanent loss o f transmission. 6. To ward o ff co ld. Alcoho l in the form o f whiskey or brandy may benefi t by causing vasodilatat io n o f b lanc hed mucosae ; but further ex posure after taki ng alcohol may be deleterious because al coho l increases heat loss due to c utaneous vasod ilata rion. 7. As a ppe tite s ti mu lan t and c a rmi nat ive : 30-50 ml o f 7-10% a lcoho l may be taken as beve rages or as t inctures (of g inger/ cardemom, etc.) before mea l. 8. To treat methanol poisoning (see below).

Aldehyde dehydrogenase inhibitor Disulfiram It inhibits the enzyme a lde hyde dehydrogenase (Fig. 28. 1) probably after conversion into active metabolites. Whe n alcohol is ingested after taking disulfiram , the concentration of acetaldehyde in tissues and blood rises and a number of highly distressing symptoms (a ldehyde syndrome) are produced promptly. These are-flushing, burning sensation, tlu·obbing headache, perspiration, uneasiness, tightness in chest, dizziness, vomiting, visua l disturbances,

421

422

DRUG S ACTING ON CENTRAL NERVO US SYSTEM and several cytochrome P450 i oen zymes. T hus, it prolongs t ½ of many drugs. Fomep1zolc

;.>( ALCOHOL DEHYDROGENASE

ESPI-RAL.ANTADICT. DEADICT 250 mg tab. (internationally marl..etcd asANTABUSI: )

METHYL ALCOHOL (Methanol, Wood alcohol)

Fig. 28.1: Shared metabolic pathway of ethyl and methyl alcohols

mental confusion, postural fai nting and circulatory collapse. Duration of the syndrome ( 1--4 hours) depends on the amount of alcoho l consumed. Because of risk of seve re reaction, disulfi ram is to be used with great caution, only in wellmoti vated subjects. Di ulfiram avers ion therapy is indicated in abstinen t subjects who sincerely des ire to leave the habit. After making sure that the subject has not taken alcoho l in the past I2 hours, di sufiram is given at a dose o f 500 mg/day for o ne week fo llowed by 250 mg daily. Sensiti zation to a lcohol develo ps a fte r 2- 3 hours of first dose, reaches its peak at ~ 12 hours and lasts for 7- 14 days after stopping it, because in hibition of aldehyde dehydrogenase with d isulfiram is irreversible: synthesis of fresh enzyme is required for return of activity. The subject's resolve not to drink is re inforced by the distressing symptoms that a ppea r if he drinks a little bit. The s ubject shou ld be cautioned to avoid al cohol a ltogether. Disu lfi ram s hould not be given to patients who are physica lly dependent on a lcoho l. S ide effects o f disul firam (as suc h) are infrequent, include rashes, metall ic taste, nervousness, malaise and abdominal upset. It inhibits a number of o ther enzymes a well including alcohol dehydrogenase, dopamine~ hydroxylase

Methyl a lcoho l is a common ly used industrial solvent and constitue nt of c leaning products. It is a lso added to industrial rectified spirit to render it unfit for drinking. lt is on ly of tox icological importa nce. Mixing of methylated spirit w ith alcoho li c beverages by bootlegers o r its inadvertent ingestion results in methano l poisoning. Methanol is metabolized to formaldehyde and fom1ic acid by a lcohol and aldehyde dehydrogenases res pectively ( Fig. 28.1 ), but the rate is 1/7th that of ethanol. Like ethano l, metabol is m of methano l also follows zero order kinetics and t ½ of 20- 60 hours has been measured. Methanol a lso is a C S depressa nt, but less inebriating than ethano l. Tox ic effects of methanol are largely due to fom,ic acid, si nce its further metabolis m is slow and fo late dependent. A blood level of >50 mg/d i methanol is associated w ith severe poiso ning . Even 15 ml of methano l has caused blindness and 30 ml has caused death; fatal dose is regarded to be 75- 100 ml. Ma ni fes ta ti o ns of met ha nol po iso nin g appear afte r a delay of few hours when its tox ic me tabolites accumul ate. T hese arc vomiting, headache, epigastric pa in, uneas iness, drunkenness, disorientation, tachypnoea, dyspnoea, brad yca rdia and hy po te ns ion. Delirium and seizures may occur and the patient may s uddenly pass in to c oma. Acidosis is pro minent and entirel y due to product io n of fonn ic acid. T he spec ific tox ic ity of form ic acid is retinal damage. Blurri ng of vision, congestion o f optic disc followed by blindness a lways precedes death w hich is due to respiratory fai lure.

Treatment I. Keep the patient in a quiet, dark room ; protect the eyes from lig ht.

ETHYL AND METHYL ALCOHOLS 2. Gastric lavage with sod. bicarbonate if the patient is brought withi n 2 hours of ingesting methanol. Supportive measures to maintain ventilation and BP should be instituted. 3. Comba t acidosis by i.v. Sod. bicarbonate infusion. This is the most important measure, which prevents retinal da mage and othe r symptoms; large quantities may be needed. 4. Pot. chloride in fusion is needed only when hypokalcmia occurs due to alkali therapy. 5. Ethanol is preferentiall y metabo li zed by a lcohol dehydrogenase over me tha no l. At a concentratio n of I 00 mg/ di in blood it saturates alcohol dehydrogenase and retards methanol meta bolism . This helps by reducing the rate of generati on of formaldehyde and fonnic acid. Ethanol ( I0% in water) is administered through a nasogastric tube; loading dose of 0. 7 ml/kg is fo llowed by 0. I5 ml/ kg/hour. Because pharmacokinetics of alcohol c hanges over time and no i.v. fonnulati on is available, mai nte nance of uni form blood alcoho l concentration is difficult. Al coho l blood level needs to be repeatedly measured. Moreover, the enzyme saturating concentration of ethanol itself produces intoxication and can cause hypoglycaemi a. Use of ethanol for this purpose is tricky. Treatment has to be continued for several days because th e sojourn of methanol in body is long. 6. Haemod ialys i : clea rs methanol as well as its tox ic metabo lite- formate and haste ns recovery.

7 . Fomepizole (4-methylpyrazole) is a spec ific inh ibitor of alcohol dehydrogenase and the drug of choice for meth a nol po isoning , which acts by retarding its metabolism. A loading dose of 15 mg/kg infused i.v. over 30 min is followed by IO mg/kg every 12 hours till serum methanol falls below 20 mg/ di. This has been found e ffective and safe, and has several advantages over ethanol, viz. longer t½, lack of inebriating action , and i. v. do age form fo r regulated administration. Marketed elsewhere as A TIZOL, it is not available commerc ially in India. Adverse e ffects of fomepizole are burning pain in infused limb, headache, nausea, taste disturbance and hypotension. 8. Folate th e rapy: Ca lc ium le ucovo rin 50 mg injected 6 hourl y has been shown to reduce blood formate levels by enhanc ing its oxidation. This is a promising adjuvant approach. Ethylene glycol poisoning Ethylene glycol poisoning ha occurred sporadically, especially among children. It is an industrial solvent, coolant and antifreeze. Ethylene glycol is oxidized in tl1c body by alcohol dehydrogcnase to glyeoaldehyde and then to glycolic acid-glyoxylic acid-oxalic acid in steps. Ethylene glycol it elf can cause into;,.ication similar lo ethanol. but generation of metabolites results in acidosis, cardiopulmonary complicat ions an d renal tub ular necrosis. FomepiLole used in the same manner as for methanol poisoning is the drug o f choice. It is appro~ed by U -FDA for this indication and has ·orphan drug status'. Ethanol is employed as an ahemative.

r:r PROBLEM DIRECTED STUDY

28.1 A school boy aged 16 years developed tonic-clon ic epilepsy a nd was maintained on carbamazepine 200 mg 3 times a day. He was seizure free for the last one year, but reported back one afternoon with the complaint of recurrence of two seizu re episodes since morning. On questioning, he revea led that last evening he attended a party with his friends and consumed 4 drinks of whiskey, and was awake til l late night. This was the first time that he had taken an alcoholic drink. (a) Could the recurrence of seizures be related to th e intake of alcohol previous night? If so, what could be th e mechanism? (b) Does his antiepi leptic therapy need any change or adjustment of doses due to this recurrence of seizures. What further advise will you give to this patient? (see Appendix-1 for solution)

423

Chapter

29

Sedative-Hypnotics

Sedative

A drug tha t subdues exc itement and calms the subject without inducing sleep, though drowsiness may be produced. Sedation refers lo decreased responsiveness to stimulation; is associated with some decrease in alertness, ideation and motor activ ity.

Hypnotic A drug that induces and/or maintains sleep, s imilar to normal arousable s leep. This is not to be confused with ' hypnosis' meaning a trans-like state in which the subject becomes passive and highly suggestibl e. The sedatives a nd hypnotics are CNS depressants with somewhat di fferi ng time-action and dose-acti on relations hips. Those wi th quicker onset, shorter duration and steeper dose-response c urves are preferred as hypnotics while more slowl y acti ng drugs with fla tter dose- response cu rves are em ployed as sedatives. However, there is cons iderable overla p; a hy pnoti c at lower dose may act as sedative. Thus, sedation- hypnosis- ge neral anae thes ia may be regarded as increasing grades of C S depression. Hypnotics given in high doses can produce general anaesthesia. However, benzodiazepines (BZDs) ca nnot be cons idered nonselective or global C S depressants like barbiturates a nd othe r older hypnotics. Treatment of insomnia is the most important use of this class of drugs. A lcohol and o pium have been the oldest hypnotics and continue 10 be u eel for this purpose as se lf-medication by peo ple. Bro mides introduced in 1857 became obsolete long back, as did chloral hydrate ( 1869) a nd paraldchyde ( 1882). Fischer and vo n Mering introduced barbitonc in 1903 and phcnobarbitone in 1912. Barbiturates reigned supreme till

I 960s whe n benLocl ia zcpines stan ed eroding their position a nd have now totally re placed them. In the mean time, a number of other sedative-hypnotics (glutcth imide, mcthyprylon. methaqualonc) were introduced but none was significantly di 1Tcrent from barbiturates; a ll are redundant now. Few non-BZD hypnotics ha,e also become popular over the past two decades, and ome novel hypno tics, viz ramelteon (me latonin receptor agonist) and s111•ore.xant (orexin receptor antagonist) ha,e been recently introduced.

Sleep The durat ion a nd pa u e rn of sleep varies cons ide ra bly among individuals. Age has an impon ant effect on q ua ntity and depth of sleep. It has been recogni,ed that s leep is an architecturcd cyclic process (Fig. 29. 1). The d ifTerc nt phases or sleep a nd their characteristics arc-

Stage O (awake) From lying down to fa lling asleep as well as the occasional nocturnal awakenings: it constitutes 1- 2% of s leep time. The EEG shows a acti, ity when eyes are closed and J3 activity whe n eyes are open. Eye mo,emcnts arc irregular or s lowly rolling. Stage 1 (dozing) a act ivity is inte rspersed w ith 0 waves. Eye. movements are red uced but the re may be bursts of rolling. N ec k muscles relax. It occupies 3- 6% of s leep time.

Stage 2 (unequivocal sleep)

e

waves with interspersed spindles, K complexes can be evoked on sensory stimulation: li1tle eye moveme nt; subjects a re easily arousablc. This comprises 40-50% of s leep time.

Stage 3 (deep sleep transition) The EEG shows 8, Ii and spindle acti, ity. K comple xes can be evoked with strong stimuli only. Eye movements arc few; s ubjects a rc no1 easily arousa blc. It comprise, 5- 8% of sleep time. The EEG waves have been div ided into-a: high amplitude, 8 14 c.p.s. (cycles per second) J3: low amp Iillldc, 15 35 c.p.s. 0: lo" amplitude, 4-7 c.p.s. 6: high amplitude, 0.5-3 c.p.s. K complex: deep negative wa ve fo llowed by positive wave and a few spind les.

425

SEDATI VE-HYPNOTICS

HOUR 23

24

2

4

3

5

6

Fig. 29.1: A normal sleep cycle

Stage 4 (cerebral sleep) 1> activity predominates in the EEG, K complexes cannot he ~voked. Eyes arc practically

are marked, irregular and daning eye mo, cments; dreams and nightmares occur, which may be recalled if the subject is aroused. Hean rate and BP fluctuate: respiration is irregular. Muscles are fully relaxed, but irregular body movements occur occasionally. Erection occurs in males. About 20-30% of sleep time is spent in REM. onnally s1agcs O 10 4 and RE~ occur in succession over a period of 80-100 min. Then stages 1- 4-REM arc repeated cyclically.

fixed; subj ects are dillicult to arouse. ' igh1 terror may occur a1 this time. II comprises I 0-20% of sleep time. During stage 2. 3 and 4 hean rate, BP and respiration are steady and muscles are relaxed. Stages 3 and 4 together are called slo" wave sleep (SWS). REM sleep (paradoxical sleep) The EEG has waves of all frequency, K complexes cannot be elicited. There

SEDATIVE-HYPNOTIC DRUGS

Hypnotic

Antianxiety

Anticonvulsant

Dinzepam Flurazcpam Ni trazepam Alprazolam Lorazepam Temazcpam Triazolam

Diazepam Chlordiazepoxide Oxazepam Lorazepam Alprazolam Clonazepam

Clonazepam Clobazam Diazepam Lorazepam

Zopiclone Eszopiclone Zolpidem Zaleplon Etizolam

Melatonin Rameltcon Suvorexant

Barbiturates

I ( Long acting

IPhenobarbitonc

I Short acting

Butobarbitone Pentobarbitone

I lUltra-short acting Thiopentone 1Methohexitone

426

DRUG S ACTING ON CENTRA L NERVOUS SYSTE M In addition some antihistaminics (promethazine, diphenhydramine), some ncuroleptic/antidepressants (chlorpromazine. amitriptyline), some anticholi11ergic (hyoscine) and opioids (morphine, pethidine) have significant sedative action, but are not reliable for treatment of insomnia.

BARBITURATES Barbilllrates have been popular hypnotics and sedatives of the last century upto 1960s, but are not used now to promote sleep or to calm patients. However. they are briefly described first because they are the prototype of more or less global CNS depressants. Barbiturates are substituted derivatives of barbituric acid (malonyl urea). Barbituric acid as such is not a hypnotic but compounds with alkyl or aryl substitution on CS have this propeny (see pheno barbitone s tructure on p. 443). Replacement of O with S at C2 yields 1hiobarbitura1es which are more lipid-soluble and more potent.

Barbiturates are general depressants for all excitable cells, more so for the CNS. 1. CNS Barbiturates produce dose-dependent elTects: sleep •

Cl:'.

-0.75

L

Non REM

!HYPNOTiq

0

10 Days

Fig. 29.2: Effect of hypnotic use for 6 consecutive nights on the ratio of REM / Non-REM sleep duration

PHARMACOLOGICAL ACTIONS

sedation

w

anaesthesia •

coma.

Hypnotic dose shonens the time taken to fall asleep and increases sleep duration. N ight awakenings are reduced. REM and stage 3, 4 sleep are decreased; REM- REM sleep cycle is disrupted. The elTects on sleep become progressively less marked if the drug is taken every night consecutively. A rebound increase in REM sleep and nightmares is often noted when the drug is discontinued after a few nights of use and it takes several nights for normal pattern to be restored (Fig. 29.2). Hangover (headache, dizziness, distortions of mood, irritability and lethargy) may occur in the morning after a nightly dose. Sedative dose (smaller dose of a longer acting barbiturate) g iven at daytime can produce drowsiness, reduction in anxiety and excitabi lity. However, barbiturates do not have selective antianxiety action. They can impair learning, short-term memory and judgement. Euphoria may be experienced by addicts. Barbiturates have anticonvulsant property. Phenobarbitone has higher anticonvulsant : sedative activity ratio, i.e. it has specific anticonvulsant action independent of genera l C S depression (see Ch. 30).

Mechanism of action Barbiturates appear 10 act primarily at the GABA : BZD receplor-CI- channel complex (see Fig. 29.3) and potentiate GA BAergic inhibition by increasing the lifetime of c1· channel o pening induced by GABA (contrast BZDs which enhance frequency of C l channel opening). They do not bind to the BZD receptor, but bind lo another site on the same macromolecular complex to exert the GA BA-facilitatory act ion. The barbiturate site appears to be located on the a or 13 subunit. At high concentrations, barbiturates directly increase conductance (GABA-mimetic action). This is in contrast to BZD action

er

which have only GABA-facilitatory property. Moreover, at high doses barbiturates inhibit Ca 2• dependent release of neurotransmitters, depress glutamate induced neuronal depolarization through AM PA receptors (a type of excitatory amino acid receptors) and depress voltage sensitive Na· and K" channels. A dose-dependent effect on multiple neuronal targets appears to confer barbiturates the ability to produce any grade of C S depression .

2. Other actions At relatively higher doses, barbiturates depress respiration, lower BP, decrease cardiac contractility and heart rate, but reflex tachycardia can occur due to fall in BP. Muscle tone, bowel motility and urine output are also reduced. Toxic does cause respiratory failure and cardiovascular collapse. PHARMACOKINETICS Barbiturates are well absorbed fro m the g.i. tract. They are widely distributed in the body. The rate of entry into CNS is dependent on lipid solubil ity. Highly-lipid soluble thiopentone has practically instantaneous entry, while less lipid-so luble ones (pe ntobarb itone) take longer; phenobarbitone enters very slowly. Barbiturates cross placenta and are secreted in milk; can produce elTects on the foetus and suckling infant. Three processes viz. redistribution, metabolism and excretion, are involved in termination of action o f barbiturates: the relative importance of each varies with the compound. On i.v. injection, the highly lipid soluble compounds (e.g. thiopentone) enter C S rapidly • produce anaesthesia in < I min , and then diffuse back quickly to redistr ibute to less vascular tissues (muscle, fat). Consciousness is regained in 6-10 min, but ultimate disposal occurs by metabolis m (t½ 9 hours).

427

SEDATIVE-HYPNOTICS Intermediate lipid solubility barbiturates are primaril) metaboli1ed in the liver by oxidation. dealkylation and conjugation. Their plasma t½ ranges between 12-36 hours. Barbiturates "ith lo,, lipid-solubility. in addition to being metaboliled. arc significantly e,-creted unchanged in urine. The t½ of phenobarbitone is 80-120 hours. Alkalini78tion of urine increases ionitation and excretion or phenobarbitone. Barbiturates induce several hepatic microsomal en.lymes (CYP3A4/5, CYP2D6, CYP2C8/9. CYP2B6, UGTs) and increase the rate of their own metabolism as well as that of many other drug . USES Except for phcnobarbitone in epi lepsy (Ch. 30) and thiopcntone/methohc,-itone in anaesthesia (Ch. 27) no other barbiturate is commercially available or used no,,. As hypnotic and anx1olytic the) ha,e been superseded by BZDs. Phenobarbitone 30--60 mg oral OD TDS; I 00--200 mg i.mJ i. v. G ARDE'JA L 30, 60 mg tab. 20 mg/5 ml S)T; LUMf,AL 30 mg tab; PHENOBARBITO'-.iF SOD 200 mg /ml inj.

ADVERSE EFFECTS Hangover was common after the use of barbiturates as hypnotic. Mental confusion, impaired performance and traffic accidents may occur (also see Ch. 30).

Tolerance and dependence Both cellular and phannacokinetic (due to enzyme induction) tolerance de, elops on repeated use. However, fatal dose is not markedly increased: addicts may present with acute barbiturate into,-ication. There is panial cross tolerance ,, ith other C S depressants. Addiction and dependence occurs, and barbiturates ha, e considerable abuse liabi lity. This is one of the major disadvantages. Withdrawal symptoms are xcitement. ha llucinations, delirium, convulsions: deaths have occurred.

Acute barbiturate poisoning 'vlostly suicidal. sometimes accidental. It is infrequently encountered now due to ina,a ilability of barbiturates. However, the principles of treatment apply to any CNS depressant poisoning. Manifestations are due to excessive CNS depressionpatient is flabby and comatose with shallow and failmg respiration, fall in BP and cardiovascular collapse, renal shut down, pu lmonary complications, bullous eruptions.

-1. Haemodialysis and haemoperfusion (through a column of activated charcoal or other adsorbants) i, highly etTective in removing long-acting as well as shon-acting barbiturates. Then: is no specific antidote for barbiturates. INTER ACTIONS I. Barbiturates induce several CY P iso-enzymes, including g lucuronyl transferase, and increase the metabolism of many drugs, thereby reducing their effectiveness-warfarin. steroids (including contraceptives), tolbutamide. gri co fulvin. ch loramphenicol, thcophylline. 2. Additi\ e action with other CN depressants - alcohol. antihistamines, opioids. etc.

BENZODIAZEPINES (BZDs) Be nzodiazepines ( BZ Ds) are selective C S depressants w hi c h produce sedation, relieve anxiety, facili tate sleep, suppress seiz ure and reduce muscle tone. Chlordiazepoxide a nd diazepam, the first BZ Ds, were introduced aro und 1960 as antianxiety drugs. S ince then th is c lass has proliferated and has replaced barbitu rates and other similar drugs as hypnot ic a nd sedati ve as well. because1. BZDs produce a lower ceiling C S depression than barbiturates. They have a high therapeutic index. In ges tion o f even 20 hy pnotic doses does not usually endanger life: there is no loss of consciousness (though amnesia occurs) and patient can be aroused ; respiration is mostly not so depressed as to need assistance. CH3 II

,:::;-0

N ->

Treatment I. Gastric lavage: leave a suspension of activated charcoal in the s tomach to prevent absorption of the drug from intestines. 2. Support i, e measures: such as. patent airway. assisted respiration, o,-ygen, maintenance of blood volume by fluid infusion and use of vasopressors--dopamine may be preferred for its renal \ asodilating action. 3. Alkaline diuresis: with sodium bicarbonate I mEq/kg i.v. with or without mannitol is helpful only in the case of long-acting barbiturates which are elimmated primarily by renal excretion.

DIAZEPAM

2. Hypnotic doses do not affect respiration or cardiovascular functions. Higher doses produce mi Id respiratory depression and hypotension which is problematic only in patients with respiratory insufficiency or cardiac/haemodynamic abnormality.

428

DRU GS ACT ING ON CENTRAL NERVOUS SYSTEM 3. BZDs have practically no action on other body systems. O nly on i.v. injection the BP falls (fall in BP may be marked in an occasional patient) and cardiac contractili ty decreases. Fall in BP in case of diazepam and lorazepam is due to reduction in cardiac output while that ca used by midazo lam is due to decrease in peripheral resi lance. The coro nary arteries di late o n i.v. injection of diaze pam. 4. BZDs cause less d istortion of sleep architecture; rebound phe nomena on di scontinu a tion afler regular use are less marked. 5. BZ Ds do not alter disposition of other drugs by microsomal enzyme induction. 6. They have lower abuse liability than barbiturates and s imilar drugs; tolerance is mild, psychological and physical dependence, drug see king and w ithdrawal sy ndrome are less marked. 7. A specific BZD antagon ist jlumazenil is available which can be used in case of poisoning.

CNS actions The overall action of all BZDs is qualitatively similar, but there are prominent differences in selecti vity for different facets of act ion, and in their time-cou rse of act ion. Different members are used for different purposes. In contrast to barb iturates, they are not general depressants, but exert relatively selecti ve anxio lytic, hypnotic, muscle relaxant and anticonvulsant effects in different measures. Even w hen apparentl y a naesthetic dose o f d iazepam is administered i. v., some degree of awareness is maintained, though because of anterograde amnesia (i nterference w ith establishment of memory trace) the pat ie nt doe s not clearl y recollect the events on recovery. A111ianxie1y: Some BZDs exert relati ve ly selective antianxiery action (see Ch. 33) wh ich is probably not dependent on their seda tive prope rty. With chronic administration relief of anxiety is maintained, but drows iness wanes off due to development of tolerance.

Sleep: Wh ile there are significant differences among d ifferent BZDs, in general , they hasten onset of leep. reduce intermittent awakening and increase total sleep time (s pecially in those

who have a short sleep span). Time spent in stage 2 is increased whi le that in stage 3 and 4 is decreased. They tend 10 shorten REM phase, but more REM cycles ma y occur, so that effect on total REM sleep is less marked th an wit h barbiturates. itrazepam has been show n to actually increase REM sleep. ight terrors and body movements during s leep are reduced and stage shi fts to stage I and O are lessened. Most subjects wake up with a feel ing of refreshing sleep. Some degree of tolerance develops to the sleep promoting action of BZDs after repeated nightly use.

Muscle relaxant: BZDs produce centrally med iated skeletal mu scle re laxation with out impairi ng vol untary acti vity (see Ch. 25). Clonazepam and diazepam have more marked muscle relaxant property. Very high doses depress neuromuscular transmission. Anticonvulsant: C lonazepam, diazepam, nitrazepam, lorazepam and Aurazepam have more prominent anti co nvu lsa nt acti vity than other BZDs. Diazepan and lorazepam are highly effecti ve for short-term use in status-epilcpticus, but their util ity in long-term treatment of epilepsy is limited by development of tolerance to the anticonvulsanl action. Given i. v., diazepam (but not others) causes ana lgesia. In contrast to barbiturates, BZDs do not produce hyperalgesia.

Other actions Diazepam decreases nocturnal gastric secretion a nd prevents stress ulcers. BZDs do not significantly afTcct bowel movement. Short-lasting coronary dilatat ion is produced by i.v. d iazepam. Site and mechanism of action Benzodiazepi nes act preferentially on midbrain ascending reticular formation (which maintains wakefu lne s) a nd on limbic system (thought and mental fu nctions). Muscle relaxation is prod uced by a primary medu llary site of action and ataxia is due to action on cerebellum. BZDs act by enhanc ing presynaptic/postsynaptic inhibition through a speci fie BZD receptor which is an integral part of the GABAA

SEDATIVE- HYPNOTICS

Fig. 29.3: Schematic depiction of GABAA-benzodiazepine receptor-chloride channel complex The chloride channel is gated by the primary ligand GABA acting on GABAA receptor located on the p subunit. The benzodiazepine (BZD) receptor located on the interface of o and y subunits modulates GABAA receptor in either - channel direction: agonists like diazepam facilitate, while inverse agonists like DMCM hinder GABA mediated c1 opening, and the BZD antagonist flumazenil blocks the action of both. The barbiturate receptor, located either on o or p subunit also facilitates GABA and is capable of opening c1- channel directly as well. Bicuculline blocks GABAA - channel directly receptor, while picrotoxin blocks the c1

receptor- Cl channel complex. The subunits of this complex form a pentameric transmembrane anion channel (Fig. 29.3) gated by the primary ligand (GABA), and modulated by secondary ligands which incl ude BZ Ds. Only the a and p subunits arc required for GABA action, and most likely the binding site fo r GA BA is located on the p subunit, while the a/y subunit interface carrys the BZD binding site. The modulatory BZD receptor increases the frequency of C( channel opening induced by submaximal concentrations of GA BA. The BZDs also enhance GA BA binding to GABA ~ receptor. The GABAA antagonist bicuculline antagonizes BZD action in a noncompetitive manner. It is noteworthy that the BZDs do not themselves increase Clconductance; have only GABA facilitatory but no GABA mimetic action. This probably explains the lower ceiling CNS depressant effect of BZDs.

The BZD receptor exhibits a considerable degree of constitutive ac tivation. As such, it is ca pab le of fine tun ing GA BA ac tion in either direction. Wh ile the BZD-agonists e nhan ce GA BA ind uced hype rpolarizat ion (due to infl ux of Cl- ions), and decrease firing rate of ne uro nes, other compoun ds called BZD-inverse agonists like dimethoxyethyl-carbo me thoxy-P-carbo l ine ( DM CM) inh ibit GA BA ac ti on and are convulsants. The competitive BZD-antagon ist flumazenil blocks the sedative act ion of BZDs as well as the convulsa nt action of DMCM. The GA BA .,-BZD receptor-Cl channel complex is composed of five a, p, y, and in some cases 6, t, 9 or ,r subunits as "ell. Severn! i~oforms of a, p and y subunits have been cloned. The subunit composition of the complex differs at different sites in the brain. Accordmgly, there arc mullipk subtypes of the BZ D receptor. The (a,2 P,2 y2) pcntarner appears 10 be the most commonly expressed BZD receptor isofonn.

429

430

DRUGS ACTI NG ON CENTRAL NERVOU S SYSTEM

Drugs affecting GABA -receptor gated chloride channel • GABA • Muscimol • Bicuculline • Picrotoxin • Barbiturates • Alcohol, lnhalational anaesthetics, Propofol • Benzodiazepines • P-Carboline (DMCM) • Flumazenil

- influx Endogenous agonist at GABA. receptor • promotes c1 Agonist at GABA, site Competitive antagonist at GABA. receptor Blocks Cl channel noncompetitively; acts on picrotoxin sensitive site Agonist at an allosteric site; prolong GABA action; and open Cl channel

l

Open Cl channel directly; allosteric facilitation of GABA

Agonist at an allosteric BZD site • facilitate GABA action Inverse agonist at BZD site • impede GABA action Competitive antagonist at BZD site

Based on studies conducted in genetically mutated mice. it has been suggested that BZD receptor isoforms containing the a , subunit are i nvolved in mediating sedative, hypnotic. and amnesic actions of BZDs, while those containing a , subunits mediate anxiolytic and muscle relaxant actions. Diazcpam has similar affinity for BZD receptor containing di!Terenl (a, or a, , or a 3 or a s) subunits, and has broad spectrum action. Receptor inhomogeneity may pro, ide an explanation for the pharmacological di,ersity of other BZDs. The newer non-BZD hypnotics 7alcplon, Zolpidem, etc. have high affinity for a , subunit isofom1 of BZD receptor and exert selective hypnotic-amnesic c!Tcct, but have l iule antiscizurc or muscle relaxant property. A t high concentrations BZDs also potentiate the depressant action of adenosine by blocking its uptake. Certain actions of BZDs are cou111ered by the adenosine antagonist 1heophyll ine. Thus, BZDs could be acti ng through other mechanisms as well.

PHARMACOKINETICS T here are marked pharmacokinetic differences a mon g B Z Ds, because they d iffer in lipidsolubility by > SO fold. These differences are important facto rs governing their choice for difTerent uses. Oral absorption of some is rapid w hile that of others is slow. Absorption from i.m. s ites is irregu la r except for lo razcpam. Plasma prote in binding also varies ma rkedl y ( flura z epam I 0 % to diazepam 99%). BZDs are widely di stri buted in the body. The more lipid soluble members enter brain rapidly and have a two phase plas ma concentration decay c urve; first due to d istribution to other tissues and later due to e limination. A relative ly short duratio n o r acti o n is o btained with si ngle dose o f a drug that is rapidl y redi s tri buted,

even though it may have a long elimination t½. Using the elimination t½ a lo ne to predi ct duration of action may be misleading. However, eliminatio n t½ detem, ines durati on of acti on in case or drugs whose e limination is by far the dominant feature or w he n the drug is given repeatedl y. Benzodiazepines are metabo lized in li ver mainly by CY P3A4 and CY P2C 19 to dealkylated a nd hydroxylated metabolites, some o f which may be active. The biologica l effect halflife of drugs w ith active metabolites may be much longer than the plas ma t½ of the adm ini stered compound. T he phase I metabolites and certain BZDs themselves are conjugated with glucuronic acid. Some BZ Ds (e.g. diazepam) undergo enterohepati c c irculation. BZ Ds and their phase I metabolites are excreted in urine as g lucuronide conjugates. BZD s cross placenta and are secreted in milk. Drugs w ith a long t½ or those w hic h generate active metabolites cumulate on nightly use; their action may then extend into the nex t day. Some features of BZDs used as hy pnotic are g iven in Table 29. 1.

HYPNOTIC BENZODIAZEPINES The salient featu res of BZDs that are used in the treatment of insomn ia are summarized he re.

1. Flurazepam It produces an acti ve metabolite which has a long t½ . Res idual effects arc likely nex t morning; c umul ati on occurs

431

SEDATIVE-HYPNOT ICS

TABLE 29.1: Some pharmacokinet,c and clinical features of benzod1azep1nes used as hypnotics Drug

t½ (hr)'

Redistribution5

50-100 30--60 30 10-20

+

12 8-12 1-3

+ +

Hypnotic dose (mg)

Clinical indications

15-30 5-10 5-10 1-2

Chronic insomnia, short-term insomnia with anxiety; Frequen t nocturnal awakening; Night before surgery

0.25-0.5 10-20 0.125-0.25

Individuals who reac t unfavourably to unfamiliar surroundings or unusual timings of sleep. Sleep onset difficulties.

/. LONG ACTING Flurazepam Diazepam Nitrazepam Lorazepam

±

II. SHORT ACTING Alprazolam Temazepam T riazolam

±

• t~ of elimination phase, including that of active metabolite s + indicates that redistribution contributes to termination of action of single dose

on daily inges tion peaking after 3-5 days. It is suitable for patients who have frequent nocturnal awaken ings, but not for the elderly due to risk of nex t day sedation, ataxia a nd mental confusion. INDRAL. FLURAZ 15 mg cap.

2. Diazepam

It is the oldest and all purpose BZ D, used as anxiolytic. hypnotic, muscle relaxa nt, premedicant, anaes thetic and for emergency control of seizures due to its broad spectru m activity. Beca use of rapid oral absorption, it can be used for sleep onset diffi culty as well as for sleep maintenance. It generates active metabolites (desmcthy l-diazepam, oxazepam). On occasional use it is free of residual effects. With regular use accumulation occurs and prolonged anxiolytic effec t may be obtained. It is less likel y to cause rebound insomnia on discontinuation of chronic use. Withdrawal phenomena are mild. VALIUM 2, 5, 10 mg tab., 10 mg/2 ml inj., CALNIPOSC 2.5. 5. IO mg tab. 2 mg/ 5 ml syr, IO mg/ 2 ml inJ. PLACIDOX 2, 5, 10 mg tab, 10 mg/2 ml inj.

3. Nitrazepam Dose to dose equipotent as diazepam. Accumul ation and residual effects can be avoided only if ingestion is occasional. Good for patients wi th frequent noctu rnal awakenings, when some day time sedation is acceptable. SEDAMON, HYPNOTEX, "\ilTRAVET 5 mg tab .. 5. 10 mg cap.

4 . Lorazepam

Th is BZD is absorbed s lowly and has prominent sedative-anx io lytic

action, because of which it is mainly used to treat anxiety (see p. 494). However, it is al so employed to promote s leep in anxious and tense patients. Since no active metabolite is produced, c umulative residual drowsiness is not a problem. ATIVAN, LARPOSE, CALMESE I. 2 mg tabs.

5. Alprazolam

T he primary indication of this potent and intermediate duration of action BZD is anxiety disorder (see Ch. 33), but it is also being employed as night-time hypnotic due to rap id oral absorption. Its active metabo lite has a short half-li fe. As such, there are few res idual effects the next day. Discontinuation after regular use has produced relatively marked withdrawa l phenomena. 6. Temazepam It is an intermediate duration 13ZD. Absorption is slow when taken as tablet but fast when taken in soft gelatin capsule. Good for sleep onset dimeuhy. free of residual effects because active metabolite has short t½. 7. Triazolam

A very potent BZD who's peak effect occurs in < I hour; good for sleep induction but poor for maintaining it due Lo short duration o f action. Patient may wake up early in the morning and feel anxious. TI1is may be a withdrawal phenomenon. Rebound insomnia may occur ,, hen it is discontinued after a few nights of use. It does not accumulate on repeated nightly use and no residual effects are noted in the morning. 1lowever. higher doses can alter sleep architecture, produce anterogradc amnesia anc.l anxiety the following day. Some cases of paranoia and ot/11:r psychiatric disturbances have been noted. For this reason, it has been withdrawn from U.K., but is employed in other countries for elderly patients. shill workers, tra,ellers, etc.

8. Midazolam Extremely rapid absorption- peak in 20 min. It can cause problems in the elderly (ataxia, blackouts), and is more liable for abuse. Therefore. it is not

432

DRUGS ACTING ON CENTRAL NERVOUS SYSTEM available now for oral use as a hypnotic. It is mainly used as an i.m. premedicant or an i. v. anaesthetic (see p. 411 ).

ADVERSE EFFECTS Benzodiazepines are relatively safe drugs. Side effects of hypnotic doses are dizziness, vertigo, ataxia, disorientation, amnesia, pro longation of reaction time-impairment ofpsychomotor skills (should not drive). Sleep walking, sleep driving and other abnormal sleep behaviours with no memory of these events has been recognized as a possible hazard. Hangover is less common, but may be noted if larger doses are taken, especially of longer acting drugs. Weakness, b lurri ng of vision, dry mouth and urinary incontinence are sometimes complained. Older individuals are more susceptible to psychomotor s ide effects. Like any hypnoti c, BZDs can aggravate sleep apnoea. Paradoxical stimulation, irritabi lity and sweating may occur in an occasional patient, especially w ith flurazepam. Some patients, particularly those taking nitrazepam or flurazepam, may experience increase in nightmares and behavioural alterati ons. Tolerance to the sedative effects develops gradually, but there is little tendency to increase the dose. Cross tolerance to alcohol and other C S depressants occurs. The dependence producing liability of BZDs is low. They are weak reinforcers (less pleasurable) and seldom abused alone. Drug abusers find them rather bland and prefer other C S depressants. Withdrawal syndrome is generally mild; may be more intense in case of ultrarapid elimination drugs. Anxiety, insomnia, restlessness, ma laise, loss of appetite, bad dreams are all that occurs in most cases. Agitation, panic reaction, tremors and delirium are occasional; convulsions are rare. Drug seeking behaviour is not prominent. An earlier report of increased birth defects on use of diazepam during pregnancy has been disputed. Administration during labour may cause flaccidity and respiratory depression in the neonate.

INTERACTIONS BZDs synergise with alcohol and other C S depressants leading to excessive impairment. Concurrent use with sod. valproate has provoked psychotic symptoms. Drug interactions due to displacement from protein binding or microsomal enzyme induction are not significant. Since CYP 3A4 isoenzyme plays important role in metabolism of severa l BZDs, their action can be prolonged by CYP 3A4 inhibitors like ketoconazole, erythromycin and others. Cimetidine, isoniazid and oral contraceptives also retard BZD metabolism.

NON-BENZODIAZEPINE HYPNOTICS This lately developed group of hypnotics are chemically different from BZDs, but act as agonists on a specific subset of BZD receptors. Their action is competitively antagonized by the BZD antagonist flumazenil, which can be used to treat their overdose toxicity. The non-BZD hypnotics act selectively on a 1 subunit containing BZD receptors and produce hypnotic-amnesic action with only weak antianxiety, muscle relaxant and anti convulsant effects. They have lower abuse potential than hypnotic BZDs. Given their shorter duration of action, they are being preferred over BZDs for the treatment of insomnia.

Zopiclone

This is the first of the nonBZD hypnotics, w hich acts as an agonist at a subtype of BZD receptor invol ved in the hypnotic action. The effect on sleep resemble those of BZDs, but it does not alter REM sleep and tends to prolong stages 3 and 4. 1t is reported not to disturb sleep archi tecture, but some degree of next mornin g impairment can occur. Zopiclone has been used to wean off insomniacs taking regular BZD medication. It is metabolized mainly by CYP3A4 with a t½ of 5- 6 hours. Dose reduction is needed in the elderly, hepatic impairment and in patients taking CYP3A4 inhibitors. Zopiclone is indicated fo r short term (< 2 weeks) treatment of insomnia. Side effects are

S EDATI VE-HYPNOT ICS metallic or bitter a fter-taste, impaired j udgement and alertness, psychological disturbances, dry mo uth and milder dependence. Safety in overdose is similar to BZDs. 70PITRAN. ZOPICO'l, ZOLILJ\J , 7.5 mg tab. one tab at bedtime for not more than 2-4 weeks (elderly 3. 75 mg).

Eszopiclone The acti ve (S) enantiomer of zop iclone has recentl y been marketed. It increases total sleep time mainl y by prolonging stage 2 with minimal erTect on sleep architecture. o acti ve metabo lite is produced , and t½ is 6 hours. It produces li ttle tole ra nce and dependence, and is co nsidered suitable for treatment of shortterm as well as chronic insomnia. Dose: 2- 3 mg at bed time, elderly and liver disease patient I mg: FUL:--.lTE, ZODIITC I mg, 2 mg tab,.

Zolpidem This struc turally non-BZD, but selective BZD receptor agonist has pronounced hypnotic effect. Sleep latency is shortened. sleep duration is prolonged in insomniacs, but anticonvulsant, muscle relaxant and antianx iety effects are not evident. The relatively selecti ve hypno tic efTect of zolpidem has been ascribed to its preferential action on a 1 subuni t containing BZD receptor. Its advantages are: relative lack of effect on slow wave s leep; minimal residual day t ime sedalion or fading of hypnoti c action on re peated nightly use; no/ little rebound insomnia on discontinuation; low abuse potential combined with safety in overdose like BZDs. Zolpidem is nearly completely metabolized in liver (t½ 2 hr), a nd has short duration of action. It is indicated for short-term ( 1-2 weeks) use in sleep onset insom nia as well as fo r intermittent awakenings. Becau e the plas ma t½ is short, next day sedation is min ima l, but mornin g sedation or prolongation of reaction-time can occur if it is taken late at night. Side effects a re few. Even large doses do not markedl y depress respiration. Currently, it is one of the most commonly prescribed hypnotics. Dose: 5- 10 mg (max 20 mg) at bedtime; ½ dose in elderly and liver disease patients. NlTREST, ZOLDl:M. DEM 5, 10 mg tabs.

Zaleplon

This is the shortest acting o f the newer non-BZ D hypnotics that se lective ly act

on a subset of BZD receptors contai ning the a 1 subunit which appear to mediate the hypnotic action. It is rapid ly absorbed; ora l bioavailability is ~30% due to first pass metabolism; is rapidl y cleared by hepatic metabolism with a t½ of I hour. No active metabolite is produced. As such, it is effecti ve only in sleep-onset insomni a; docs not prolong total sleep time or alter REM -REM cycles, or reduce the number of awakenings. Because of brev ity of action. it can be taken late a t night (> 4 hour before wa king time) without causing morning sedation. Surprisingly, des pite ve ry short action, no day-time anxiety or rebound insomnia has bee n observed with low doses, an d hypnotic effect does not fade on nightly use. However, its use should be limited to 1- 2 weeks. The hypnotic efficacy o f zalep lon is rated similar to zolpidem. Like the latter, effect o n sleep stages and REM sleep are less than that of BZDs. Tolerance and dependence is unusual. Dose: 5- 10 mg (max 20 mg) at bed time. /.APLO:-J. ZALEP, ZASO 5, IO mg tabs. Etizolam This BZD analogue is a thienorriamlodiazepinc derivat ive, and a BZD receptor agonist which produces the fu ll range of BZD actions. Thus, it has anxiolytic, hypnotic. anticonvulsant and muscle re laxant properties. Actions of etiLolam arc antagoniLed by fl umEUenil which can be used to counteract ovcrdosage. It is absorbed rapidly, peak level is produced in 0.5- 2 hours. and it is metaboli7cd by oxidation with a t½ of 3.5 hours. Chronic intake of eti701am produces tolerance and dependence, and it carries abuse potential. because of which it is a controlled drug in many coumries. EtiLolam is indicated fo r the short-term management of anxiety, panic and sleep disorder, but does not appear to offer any specific advantage o,er the established BZDs. Dose: For anxiety 0.5- 1 mg BD; for insomnia 1-2 mg at bed time. ETIZOLA, ETILA \4, ETIREST 0.5, 1.0 mg tabs.

USES Currently, BZDs are one o f the most frequently prescribed drugs. They have also been combined with many othe r categories o f drugs w ith a view to improve efficacy by relieving attendant a nxiety.

1. As hypnotic A hypnotic should not be casually prescri bed for every case of insomnia.

433

434

DRUGS ACTING ON CENTRAL NERVOUS SYSTEM

Unde rstanding the pattern and cause of insomnia in the patient is important, and use of a variety of other measures can avoid unnecessary hypnotic medication. When indicated, BZDs or the newer non-BZDs like eszopiclone, zolpidem, zaleplon are the hypnotic of choice. A wide range of compounds are available to suit specific requirements. Some important points are outlined below: • A hypnotic may be used to shorten sleep latency, to reduce nocturnal awakenings, or to provide anxiolytic effect the next day when insomnia is accompanied with marked element of anxiety. • ln the use of hypnotics, consideration must be given to onset and duration of action of the drug. The most suitable pharmacokinetic profile drug should be chosen for a given case. • Next morning impairment is largely re lated to the dose and pharmacokinetic profile of the drug. The next day effects are either due to prolonged sedation (longer acting drugs) or rebound anxiety (shorter acti ng drugs). • Any hypnotic (probably except zolpidem-like drugs) becomes ineffective after regular use for a few days; may actually be harmful. • Though effect of the drug on EEG stages of sleep, including REM sleep, could be physiologically re levant, most important is the subject's own assessment of having slept restfully and waking up feeling fresh with no impairment the following day. The subjective impression that quality of sleep was poor is the major criterion of insomnia. • Insomnia arises under a variety of circumstances. It could be a long-term (monthsyears), short-term (weeks) or transient (a day or two, mostly situational) problem. Chronic insomnia (> 3 weeks) Uncertainty exists about the va lue of hypnotics in this situation. The patient may have a personality disorder, but often there is no specific stress factor. He may have used hypnotics for long periods or may be alcoholic or ha ve some somatic disease, e.g. gastroesophageal reflux,

pai n, COPD, etc. which interfe re with sleep. Measures like aerobic exercise, training at mental relaxation, avoiding anxiety about past/ future performance whi le in bed, attempting sleep when slee piness is maximum, avoiding napping at day-time, mai ntaining regular sleep-wake timings and other sleep-hygiene measures, coffee/alcohol restriction, treatment of concurrent somatic illness, psychotherapy and controlled sleep curtailment may succeed. Good nightly sleep improves the quality o f day-time wakefulness. Patients of obstructi ve sleep apnoea have poor sleep and feel sleepy during the day. All hypnotics aggravate sleep apnoea and are contraindicated. In selected cases of chronic insomnia intem1ittcnt use of a hypnotic, say once every J days, may be tried. Risk of tolerance and abuse are maxi mum among chronic insomniacs. A slowly eliminated drug is preferable because rebound insomnia and withdrawal symptoms are least marked with such drugs. Short-term insomnia (3-21 days) Emotional prob le m ( occupational stress, bereaveme nt) and physical illness a re the usual causes. Patient may have induction difficulty or may be waking up early. Cautious use of low doses of an appropriate drug fo r the type of sleep disturbance may be made. Generall y a hypnotic, free of residual effects should be selected, but when anxiety is a dominant feature, a BZD whose action extends into the next day may be better. Short acti ng drugs are preferable in the elderly. lntermittent hypnotic use should be limited to 2- 3 weeks. Transient insomnia (1 - 3 days) Due to alterations in the circumstances of sleep, e.g. unusual noise, on an overn ight train, new place, unusual pattern of work, shift workers, intercontinental travel- jetlag, etc. A rapidly eliminated hypnotic or one with marked distribution is to be preferred to avoid residual elTects the next morn ing. Popularity of the newer non-B ZD hypnotics has increased due to their rapid onset of action, minimal next-day impairment and absence of cumulation on repeated nightly

SEDATIVE-H YPNOTICS usc as well as rebound insomnia on stopping. However, night before surgery- a long acting drug is better.

2. Other uses • As anxio lytic and for day-ti me sedation (see Ch. 33). • As anti convulsa nt, especially emergency control of statu s epilepticus, febrile convulsions, tetanus, etc. (see Ch. 30). • As centrall y acting muscle re laxant for muscle spasms and spastic ity (see Ch. 25). • For preanaesthetic med ication, i.v. anaesthesia and conscious sedation (see Ch. 27). • Before ECT. e lectrical cardioversion of arrhythmias, cardiac catheterization, endoscopies, in obstetrics and many minor p rocedurcsdiazc pam i.v. has gained popularity because of its calming-amnesic-analgesic and muscle relaxant properties and relati ve safety. • Alcohol withd rawa l in depen dent subjects. • Along with analgesics. SA IDs, spasmolytics, antiulcer and as adjuvants to treat 'gas ' or nonspecific dyspeptic symptoms. Fixed dose combinations o f sedati ve/ hypnolic/anxiolytic drngs with ana lgesic-antipyretics has been banned in India.

BENZODIAZEPINE ANTAGONIST

Flumazenil

It is a BZD analog ue which has little in trinsic act ivity (practicall y no e ffect on normal subjects), but competes with BZD agonists as well as inverse agonists for th e BZD receptor and reverses the ir depressant or stimula nt elTects respectively. Flumazenil abolishes the hypnogenic, psychomotor, cognitive and EEG effects of BZDs. Flumazenil is absorbed orally, but it is not used orall y. Injected i. v., its act ion sta rts in seconds a nd lasts fo r 1-2 hours; e li minatio n t½ is I hr, due to rapid metabo lism.

Uses 1 . To reverse BZD anaesthesia Pa tients anaesthetized/ sedated wi th a BZD wake up, get

oriented and regai n motor control within I m in of an i.v. injection of 0.3-1 mg of flu mazen il. Resedation generally occurs within I hour (more wi th diazepam than with midazolam): supplemental doses of flu mazenil may be given. This may al low early di scharge o f patients after diagnostic procedures and facilitates postanaesthetic management. 2. BZD overdose Majority of patients of BZD overdose require only supportive measures like patent airway, maintenance of BP, cardiac and rena l function (by fluid transfus ion. etc.). In ad ditio n, tlumaze nil 0.2 mg/ min may be injected i. v. till the patient rega ins consciousness. Practicall y a ll patients intoxicated with a BZD a lone respond within 5 min (maxi mum total dose I mg). I lowever, reversal of respiratory de pression is incomplete. Flumazenil blocks the hy pnotic effect o f zo lpidem- li ke non- BZDs as well. In mi xed C S depressant poisoning, w hatever sedation is not abolished by 5 mg of flumazeni l should be taken to be due to a non-BZD/non-Zolp idem-like depressant. However. administration o f flumazen il may be risky in m ixed overdosage w ith a BZ D + a tricyc lic an tidepressant. It can a lso be hazardous in BZ D-dependent subjects by prec ipitating severe withdrawal. Thus, use of flumazeni l in BZD overdosage requires great caution.

Adverse effects Flumazcn il is sa fe and well tolerated. Side effects noted duri ng use of tlumazenil as a BZD-antagonist are-agitation, disco mfo rt, fea rfulness, anxiety, coldness and withdrawal seizures. ANEXATE 0.5 mg in 5 ml amp.

Triclofos

It is an old CNS depressant very si milar lo chloral hydrate but does not have the acrid odour and burning taste of the latter. Gastric irritation and abdominal side effects are milder. Like ch loral hydrate. it is rapidly hydrolysed to trichloroethanol, which is the active metabolite of both. Triclofos is a fast-acting hypnotic; acts in 30 mm, action lasts 6-8 hours. Though obsolete, il is some times used lo sedate children in dis tress, and rarely to induce sleep in adult .

435

436

DRUGS ACTING ON CENTRAL NERVOUS SYSTEM Dose: Adult 1- 2 g at bed time: child aged 5-10 years 250-500 mg or 15-25 mg/kg. T RICLORYL, PED ICLORYL 500 mg/5 ml ora l so ln., 30, 60 ml bottle.

Melatonin

It is the principal hormone of the pineal gland secreted at night and bas been fou nd to play an imponant role in entraining (synchronizing) the s leep-wakefulness cycle with the circadian rhythm. Two subtypes of melatonin receptor MT, and MT, have been identified in the brain. Both are GPCRs and are believed to carry out the function of facil itating sleep onset and fixing its timing in relation to the circadian c lock. Though high doses (80 mg) of melatonin administered orally can induce sleep, lo" doses (2- 10 mg) do not depress the CNS, but probably increase the propensity of fall ing asleep. Started before the flight it has been shown to reduce jet-lag symptoms and to hasten reentrainmcnt with day-night cycle of the new place in intercontinental travellers. Beneficial effects in shift workers and in individuals with delayed sleep phase syndrome have also been reported. Elderly insomniacs have reported subjective improvement in sleep quality. However. melatonin is not a dependable hypnotic; especially in non-elderly insomniacs. Use may there fore be restricted to treatment of jct-lag. shi ft workers and elderly insomniacs. Since me laton in secretion dec lines with age, it has been argued that melatonin supplementation might retard ageing. Though there is no proof of benefit, melato nin (2-5 mg/day) is being consumed as a health food. In India it is marketed as a remedy for disturbed biorhythms and sleep disorders. MELOSET 3 mg tab, ZYTONIN, ETERNEX mclatonm 3 mg • pyridoxinc IO mg tab; one tab at evening daily.

Ramelteon

It is a MT 1, MT2 melato nin receptor agoni st introduced as a new class of hypnotic for sleep o nset insomn ia, that does not alter GA BAergic neurotransmission or produce the usua l BZD-like side effects. Administered in a dose of 8 mg ½ ho ur before going to bed, it is shown to hasten s leep onset as well as increase sleep duration, w ithout causing next morn ing sedation or impairment. In clinical trial on chronic insomnia patients, continuous nightl y treatment with rameltcon maintained its effect to shorten sleep latency and was fou nd to be free of rebound phenomena/ withdrawal syndrome on stoppage. No dependence producing potential has been noted so far. It is rapidly absorbed orally, undergoes extensive first

pass metabolism in liver by CYP isoenzymes, so that bioavailabi lity is low and el imi nation t½ is 1- 3 hours. Fluvoxamine, ciprofloxacin and fluconazole can inhibit rame lteon metabolism and raise its blood levels, wh ile ri fampici n lowers it by induc ing its deg radati on. Side effects are fatigue, sleepiness, and dizziness. Ramelteon appears to be a promising novel hypnotic, provided its efficacy is established. ROZEREM. RAM ITAX 8 mg tab: I tab 1/2 hour before going to bed.

Suvorexant It is the first member of a novel c lass of insomnia drugs- ·dual orexin receptor antagonists' ( DORAs). Orcxins are neuropcptides found in lateral hypothalamus which promote wakefulness by acting on OX IR and OX2R subtypes of o rexin receptors. Observations that orexin It:, els arc high during day time and low at night, and that narcolepsy (episodes o f sudden sleep) is associated with loss of orexin neurones, support this role of orexin. Suvorexant blocks both OX IR and OX2R. In clinical trials, suvorexant has been found to hasten sleep on et. help slee p ma intenance and increase total sleep time. With low dose ( 10 mg) the next-day residual elTccts were not detected. but higher doses reduced subjective alertness and prolonged reactio n time. The dose-related safety concerns expressed by the US-FDA are day-time somno lence, impai red driving, unconscious night-ti me bcha, iours (talking in sleep, dream-enacting e pisodes, sleep walking, weakness in bed and feeling immobilized), narcolcpsy-like symptoms and suicidal ideation. However, compared to zolp idem, suvorcxant (IO mg) produ ced minima l neurophysiological disturbances. Suvorexant is absorbed in 1-2 bours, me tabolized mainly by CY P3A4 with t½ of 12 hours. and excreted in faeces. Inhibitors of CYP3A4 may increase its levels producing next-day effects. In the absence of head-to-head comparative studies, the status of suvorcxant in relation 10 other hypnotics is not yet clear. I lowever, it appears not to cause amnesia or ataxia and 10 have low potential for produc ing addiction or dependence. It is considered suitable for continuous long-term use with no rebound insomnia on stopping. Thus. it may o fTer sustained benefit for patients with chronic insomnia. I IO\\ ever. elderly pa tients arc more prone 10 sulTer adverse eflects which arc somnolence, mu cle weakness, abnormal dreams and headache. Patients with depression may worsen. Dose: 10 mg half hour before bed time, higher dose only when lower dose is well to lerated. Internationally marketed as BELSO:\1RA 5, 10, 15, 20 mg tabs.

SEDATIVE-HYPNOTICS

r:r PROBLEM DIRECTED STUDY 29.1 A 70-year-old man consults his family physician for the problem of failing to fall asleep occasionally (3-4 times in a month) for the past few months. He usually sleeps well and has a 6-7 hour sleep duration. However, on certain nights he keeps lying in bed for 2-3 hours before getting sleep. Such episodes are unpredictable, and he cannot relate them to any disturbance, anxiety, worry or physical illness. He has tried relaxing, getting up and walking around or readi ng, but nothing helps. As a result, next day he feels lethargic, impaired, unable to concentrate and has poor creativity. He requests a sleeping pill t hat he can take occasionally when he fails to fa ll asleep. (a) Can he be prescribed a hypnotic for occasional use? If so, which drug would be suitable for late night intake without next morning sedation? (see Appendix-1 for solution)

437

Chapter

30

Antiepileptic Drugs

Epilepsies These are a group of disorders of th e C S cha racte ri zed by paroxys mal ce reb ral dysrhythmia, ma ni festi ng as bri ef episodes (seizures) of loss or disturbance of consciousness, with or wi thout cha racteristic body movements (convulsions). sensory or p syc hiat ric phe nome na. These episodes are unpredictable and their frequency is highly va ri a ble. Epi lepsy has a focal origin in the brain, manifestations depe nd on the si te of the focus, regions into which the discharges spread a nd post ictal dep ression of these regions. Recognised from the dawn of history as 'disease o f lightening', it was correctly described by JH Jackson little over a century ago. Epileps ies have been classified variously; major types are described below. I. Generalised seizures They have a diffuse origin involving both hemispheres of the brain; manifestations and EEG abnormalities arc bilateral.

1. Generalised tonic-clonic seizures (GTCS, major epilepsy, grand mal): lasts 1-2 min. The usual sequence is aura-cry- unconsciousness and patient falls- tonic spasm of all body musclcs-clonic jerking f'ollo"ed by prolonged sleep and depression of all C S functions. 2 . Absence seizures (minor cpilep y. pctit mal): prevalent in chi ldren. lasts about 1/2 min. No or o nly momentary loss of consciousness, no fall, patie111 apparently freezes and stares in one direction. no muscular component or minimal bi lateral jerking or blinking of eyes, EEG shows characteristic 3 cycles per second spike and wave pattern. Multiple episodes may occur each day. Seizures may remit spontaneously in adolescence.

3. Aton,c seizures (Akinetic epilepsy): Brief loss of consciousness with relaxation of all muscles due to excessive inhibitory discharges. Patient may fall. 4. Myoclonic seizures Shock- like momentar) contraction of muscles of a limb o r the whole body. Myoc lonic jerking may accompany any type of genera lised or partial seizures. 5 . Infantile spasms (Hypsarrhythmia) Seen in infants. Probably not a form of epilepsy. lntern1incnt mu,cle spasm and progressive men tal deterioration. Diffuse changes in the imerseiLure EEG are noted.

II. Partial seizures They have a unilateral locali7cd origin in the bra in, but may spread to small or large area, or to the whole brain. Focal origin may be evident c linica lly, or may be detected in the EEG.

1. Simple partial seizures (SPS): There i sudden onset unilateral clonic jerking of a group of muscles or a limb last ing 30-90 sec, or localized sensory di turbances such as pin pricks. visual/auditory hallucinations, etc. depending on the area of the cortex involved. The patient remains conscious and aware of the attack. 2. Complex partial seizures (CPS, temporal lobe epilepsy. psychomotor): attacks of bizarre and confused behaviour. dream- like state and purposeless movements, or even walking unaware. emotional changes lasting 1-2 min along "ith impairment of consciousness. The patient has no recollection o r the attack. An aura o llcn precedes. The sci711re focus is located in the temporal lobe. 3. Simple partial or complex partial seizures secondarily generalized The partial sei7Ure occurs first and evolves into genera lized tonic-clonic sei?ures with loss of consciousness.

Most o f the cases of epilepsy are prima ry (idiopathi c), some may be secondary to trauma/ surgery o n th e head . in trae ranial tum our.

439

ANTIEPILEPTIC DR UGS Phenytoin (Diphenylhydantoin)

tuberculoma, cysticercosis, cerebral ischaemia, etc. Treatment is symptomatic, depends mostly on the seizure type, but not on the etiology of seizure or whether it is primary or secondary.

h was synthesized in 1908 as a barbiturate ana logue, but shelved due to poor sedative property. Its anticonvulsant activity was specifically tested in 1938 in the newly developed electroshock seizure model, and till recently it was a major antiepileptic drug.

Experimental models These models for testing anticpileptic drugs ha, e also shed light on the etiopathogenesis of epilepsy.

I. Maximal electroshock seizures Brief high intensity shock is applied to the head of a rodent Gust as in ECT): produces tonic flexion- tonic cxtension~lonic convulsions. The tonic phase (especially extensor) is selectively abolished by drugs effective in GTCS and complex panial seizures. Activity in this mode l represents action on spread of seizure discharge.

1

HN 3

2. Pentylenetetrazol (PTZ) clonic seizures Injection of PTZ in rats or mice produces clonic convulsions which are prevented by drugs effective in absence and myoclonic seizures. Activity in this model represents action on sei1ure focus itself.

11

NH

0

PHENYTOIN

3. Chronic focal seizures Produced by application of alumina cream on the motor conex of monkey.

Phenytoin is not a global CNS depressant; only mild sedation occurs at therapeutic doses. The most promin ent action is abolition of toni c phase of maximal electroshock seizures, with no effect on or prolongation of clon ic phase. It limits spread of seizure acti vity. T hreshold for PTZ convulsions is not raised.

4. Kindled seizures Brief bursts of weak electrical impulses are applied to the brain (especially amygdala) intermittently over days. A ner-discharges increase progressively and tonic-clonic seizures are produced after I 0-15 shocks. With time spontaneous seizures set in, usually after > I 00 shocks. This indicates that seizures ha ve a self perpetuating and reinforcing effect: more neuronal circuits are facilitated and recru ited in the seizure process. Kindling is probably involved in the genesis of complex partial se izures and GTCS.

Mechanism of action

Phenyto in prevents repetitive detonation of normal brain cel ls. Thjs

ANTIEPILEPTIC DRUGS

I

I

I_

IBarbiturate Phenobarbitone

Hydantoin

Pheny toin Fosphenytoin

I [Succinlmide

Ethosuximide

1

I

fDeoxybarbiturate Primidone I

hminostilbene

Ca rbamazepinc Oxcarbazepine Eslicarbazepine

I

rAliphatic carboxvlic acid Valproate sod (Valproic acid) Divalproex

I

IBenzodlazepines Clonazepam Diazepa m Lorazepam Clobazam

IPhenyltriazine Lamo trigine

I

I !Newer drugs

Topiramate Levctiracetam Zonisamide Vigabatrin ITiagabinc Lacosamide I CyclicGABA analogues Gabapentin I Pregabalin

Perampanel, retigabine, stiripentol, rufinamide and few other newer amiseizure drugs ha ve been introduced countries as second line/add-on drugs for refractory partial seizures.

111

some

440

DR UGS ACTIN G ON CE NTR A L NERVOUS SYSTEM

Prolongation of Na•channel inactivation Phenytoin Carbamazepine Valproate Lamotrigine Topiramate Zonisamide Lacosamide

Inhibition of excitatory glutamatergic synapse Phenobarbitone Phenytoin Lamotrigine Topiramate Valproate Levetiracetam Lacosamide

Facilitation of Gaba mediated Cl- channel opening Barbiturate (Barb.) Benzodiazepine (Bzd.} Vigabatrin (Viga.} Valproate (Valpr.) Gabapentin (Gabp.} Tiagabine (Tiag. )

Inhibition of 't' type ca 2• current Ethosuximide Valproate Zonisamide

Fig. 30.1: Major mechanisms of anticonvulsant action m-Activation gate; h- lnactivation gate; GABA-T-GABA transaminase; SSA-Succinic semialdehyde; GAT-1- GABA transporter; GLU-Glutamate; SV~ -Synaptic vesicular protein 2A; CRMP2-- 30 mild, > 60 severe

Phenytoin

12- 36

10-20

> 20 mild, > 35 severe

Carbamazepine

10--40

5-10

> 12

Ethosuxim ide

30-50

50-100'

>200

Valproate

10-15

40-1 oo·

Clonazepam

20--40

0.0HJ.1·

• Poorly correlated with response.

Febrile convulsions Some children, especially unde r 5 years age, deve lop convu lsions during fever. Seizures may recur every time w ith fever and few may become chron ic epilepti cs. Every attempt s hould be made to see that they do not develo p fever, but when they do, temperature s hould not be a llo wed to rise by using paracetamo l and external cooling. T he best treatment of febri le convulsio ns is recta l d iazepam 0.5 mg/kg g iven at the onset of convulsio ns. TI1e i.v. preparation can be used where the recta l fo rmu latio n is no t available. A rectal solution (5 mg in 2.5 m l) in tubes is available in the UK and some other cou111ries. Seizures genera lly stop in 5 min; if not, another dose may be g iven. The d rug is repeated 12 ho urly fo r 4 doses. If fe ve r is prolo nged a gap of 24-48 hr is given before starting next series of doses. In recun-ent cases or those at particula r risk of developing epilepsy- i111ern1i ttent prophylaxis w ith d iazepa m (ora l o r rectal) s ta rted at the o nset of fever is recomme nded. Chronic prophylax i w ith phenobarbitone ad vocated ea rlie r has been a bandoned, because of poor efficacy and behavioural side effect .

Infantile

spasms

(hypsarrhythmia)

T he ra py of this co nd ition is unsatisfactory, a n ti e pi le ptic drugs a re ge ne ra ll y useless. Corticos tero ids a ffo rd sym ptomatic re lief but canno t be used fo r lo ng-term due to adverse e ffects. C lonazepam, valp roate and viga batrin may afford some relie f.

Status epilepticus W he n seizure act1 v1ty occurs for >30 min, or two o r mo re seizures occur w ithout recovery of conscio usness, the cond iti on is call ed status epilepticus. Recurrent tonic-clonic convul sions without recovery of consciousness in between is an emergency; fi ts have to be controlled as quic kly as possible to prevent death and permanent bra in damage. • T he firs t pri or ity is to ma in tai n pa ti e nt ai r way. T hi s is fo l lowed by 20- 50 ml of 50% dex trose injected i. v. to co rrect hy poglycaemia, in case that is responsi ble for the seizures. • Lorazepam 4 mg (0.1 mg/ kg in ch ildren) injected i.v. at the rate of 2 mg/mi n, repeated once after IO min if required, is the fi rst choice drug now. It is e ffective in 75- 90% cases and produces a mo re sustai ned anticonvulsant effect ( lasting 6- 12 ho urs) than d iazepam , because of lower lipid olubility and slower redi stribution. Moreover, thrombophlebitis o f injected ve in is less likely w ith lo razepam. • Diazepam IO mg (0.2- 0.3 mg/kg) injected i.v. at 2 mg/min, repeated once after 10 min if required, has been the standard therapy t ill rece ntly. H owever, its a nti con vul sa nt effect sta rts fad ing after 20 m in, and many supplementa l doses may be req uired. It is also more damaging to the inj ected vein. • Fospheny loin I 00-150 mg/mi n i.v. infusion to a max im um of I 000 mg ( 15- 20 mg/ kg) under contin uous ECG mo nitoring is a slower acting drug which should be g iven

ANTIEP ILE PTIC DRUGS subsequen tly irrespective o f response to lorazepam. Thi s may suppress seizures that have not responded to lorazepam, and pave the way for long-term seizure treatment. • Phenytoin sod. It should be used only when fosphenytoin is not available, because it can be injected only at the rate of 25- 50 mg/ min and causes more marked local vascular comp I ications. • Phenobarbitone sod. 50- 100 mg/ min i .v. injection to a max i mum of I O mg/kg is another slower ac ting drug w hich can be used a alternative to fospheny toi n or when

the latter is ineffecti ve. It is also employed to maintain seizure free stale over short term before defi niti ve oral th erapy is instituted. • Refractory cases who fail to respond to lorazepam and fosphenytoin within 40 min of seizure onset may be treated w ith i. v. midazolam/ propofol/thiopentone anaesthesia, with or w ithout curarization and full intensive care. • General measures, includi ng maintenance of airway (intubation i f requi red), oxygenation, flu id and electroly te bal ance, BP, normal cardiac rhy thm, euglycaemia and care of the unconscious must be taken.

r:r PROBLEM DIRECTED STUDY 30.1 A young lady aged 25 years comes for consultation along with her husband for having suffe red two episodes of fits lasting 2- 3 min each over the past one week. Just before each fit, she experienced flickering in her right arm. Descri ption of the fit given by the husband corresponds to tonic-clonic seizures. She gave th e history of having met a car accident about one year back in which she received head inj ury. There is no family history of epilepsy. General physical and neurological examination revealed no abnormality. Investigations, including EEG and MRI scan of the brain, were ordered. (a) W hat instructio ns should be given to t he husband regarding care to be taken, if and when, the next fit occurs? (b) Should antiepileptic drug/drugs be started right away, or therapy be delayed till fin dings of t he investigations become available or till more fits occur? (c) In case antiseizure therapy has to be started right away, should a single drug or a combination of drugs be given? Which drug(s) would be the most appropriate for this patient ? (see Appendix-1 for solution)

451

Chapter

31

Antiparkinsonian Drugs

T hese are drugs that have a therapeutic effect in parkinsonism.

Parkinsonism lt is an extrapyramidal motor disorder characteri zed by rigidity, tremor and hypokinesia w ith secondary manifestations like defective posture and ga it, mask-like face and s ialorrhoea; dementia may accompany. If untreated the symptoms progress over several years to end-stage disease in whi ch the patient is rigid, unab le to move, unable to breathe properly; succumbs mostly to chest in fections/embolism. Parkin on's disease ( PD) is a progressive degenerative disorder, mostly affecting o lder people, first described by James Parkinson in 18 17. Majority of the cases are idiopathic, some arc arteriosclerotic while postencephalitic are now rare. Wilson's disease (hepatolenticular degeneration) due to chronic copper poisoning. is a rare ca use. The most consistent lesion in PD is degeneration of neurones in the substantia nigra pars compacts (S - PC) and the nigrostriatal (dopaminergic) tract. Th is results in deficiency of dopamine (DA) in the striatum which controls muscle tone and coordinates movements. An imbalance between dopaminergic (inhibitory) and cholinergic (excitatory) system in the striatum occurs giving rise to the motor defect. T hough the cholinergic system is 1101 primari ly a ffected. its suppression (by anticholincrgics) tends to restore balance. The cause o f selective degeneration o f nigrostriatal ne urones is no t prec isely known. but appears to be muhifactorial. Ageing, genetic predisposition. oxidative generation of free radicals, N-methy l-4-phenyl tetrahydropyridine (M PT P)-like environmental t0xins and excitotoxic neuronal death due to NMDA-receptor (exc itatory glutamate receptor) mediated Ca'' overload have all been held re ponsible. Drug-induced reversible 1Jarkinsonism due to ncuroleptics, metoclopramide (dopaminergic blockers) is now fairly common, while that due to reserpine (DA depleter) is h istorical.

Belladonna alkaloids had been empirically used in Parkinson's disease ( PD). A breakthrou gh was made tn 1967 when levodopa was found to produce dramatic improveme nt. Its use was based on sound sc ientific investigations made m the preceding IO years that: • DA is present in the brain; • it (along with other monoamines) is depleted by reserpi ne; • reserpine induced motor defect is reversed by DOPA (the precursor of D A); • strialum of patients dy in g of PD was deficient in DA. Thus, parkinsonism was characterized as a DA deficiency state and levodopa was used to make good this defic iency, because DA itsel f does not cross the blood-brain barrier. In the subsequent years, a number of levodopa potentiators and DA agonists have been deve loped as adjuvants/ alternatives. one of the drugs alter course of the disease in PD, but improve quality of Ii fc for a few years.

LEVODOPA Levodopa has a spe c ifi c sa lutary effect in PD: efficacy exceeding that of any other drug used a lone. It is inacti ve by itself, but is the immediate prec ursor of the transmitter DA. More than 95% of an oral dose is deca rboxylated in the peripheral tissues (mainly gut and liver). DA thus fonned is further metabolized, and

453

ANTIPARKINSONIAN DRUGS

ANTIPARKINSONIAN DRUGS

Dopamine precursor

Levodopa

Bromocriptine Ropinirole Pramipexole

Peripheral decarboxylase inhibitors

Carbidopa Benserazide

Central antlcholine~lcs

Antlhistamlnlcs

Trihexyphenidyl Procydidine Biperiden

Orphenad rine Promethazine

MAO· B Inhibitors

Glutamate (NMDA receptor) agonlst (Dopamine facllltator)

Selegiline Rasagiline

Amantadine

the remaining acts on heart, blood vessels, other peri pheral organs and on CTZ (though located in the brain, i.e. floor of IV ventricle, it is not bound by blood-brai n barrier). About 1-2% of administered levodopa crosses to the brain, is taken up by the surviving dopaminergic neurones, conve11cd to DA which is stored a nd released as a transmitter. This is supported by the finding that brains of parkinsonian patients treated with levodopa till death had higher DA levels than those not so treated. Further, those patients who had responded well had higher DA levels than those who had responded poorly.

ACTIONS

1. CNS Levodopa hardly produces any effect in normal individuals or in patients with other neurol ogical di seases. Marked sym ptoma tic improvement occurs in parkinso nian patie nts. Hypokinesia and rigidity resolve fi rst, later tre mor as well. Secondary symptoms of posture, gait, handwriting, speech, facial expression, mood, selr care and interest in li fe are gradually normalized. Therapeutic benefit is nearly complete in early di sease, but declines as the di sease adva nces. The effect of levodopa on behaviour has been descri bed as a 'general alerting response'. In some patie nts this progres es to excitement-

fra nk psychos is may occ ur. Embarrassing ly disproportionate increase in sexual activity has also been noted. Dementia, if present, does not improve; rather it predisposes to emergence of psychiatric symptoms. Levodopa has been used to produce a nonspecific 'awakening' effect in hepatic coma. Two subtypes of DA receptors ( DI. D2) were originally described. Three more ( D3, D4, D5) have now been identified and cloned. All are G protein coupled receptors and are grouped into two families: DI /iAe (DI, D5) Are excitatory: act by increasing cAMP formation and Pl P2 hydrolysis thereby mobilizing intracellular Ca1• and activating protein kinase C through IP1 and DAG. D1 like (D2, DJ. D4) Are inhibitory: act by inhibiting adcnylyl cyclase/opening K· channels/depressing voltage sensitive Ca'' channels.

The various subtypes of DA receptors are differentially expressed in di fferenl areas of the brain. and appear 10 play distinct roles. Both D l and D2 receptors are present in the striatum and are involved in the therapeutic response to levodopa. They respectively regulate the activity of two pathways having opposite effects on the thalamic input LO Lhc motor cortex ( Fig. 31. l ). Thus, stimulation of excitatory DI as well as inhibitory D2 receptors in the striatum achieves the same net cfTecL of smoothening movements and reducing muscle tone. Dopamine receptor in SN-PC and in pitu itary is also of D2 type. The D3 receptors predominate in nucleus accumbans and hypothalamus, but are sparse in caudate and putamen. while D4 and D5 are mostly distributed in neocortex, midbrain, medulla and hippocampus.

454

DRUGS ACTIN G ON CE NTRA L NE RVO US SYSTEM

2. CVS The peripherally formed DA can cause tachycardia by acting on p adrenergic receptors. Though DA can stimulate vascular adrenergic receptors as we ll, rise in BP is not seen. Instead, postural hypotension is quite common. This may be a central action. Excess DA and A formed in the brain decrease sympathetic outflow; also DA formed in autonomic ganglia can impede ganglionic transmission.

Gradual tolerance develops to both cardiac stimulant and hypotensivc actions.

3. CTZ Dopaminergic receptors are present in this area and DA acts as an excitatory transmitter. The DA formed peripherally gains access to the CTZ without hindrance-elicits nausea and vomiting. Tolerance develops gradually to this action. 4. Endocrine DA acts on pituitary mammotropes to inhibit prolac tin re lease and on

To spinal motor neurone

Glu @ DA STRIATUM Direct pathway GABA8

Glu

GABA 8

,,;,.ci23 ..

pathway i Gt

Thalamus

I

GABA 0

© GP-I + SN-PR Fig. 31 .1: Simplified scheme of side loop circuits in the basal ganglia that provide modulatory input to the motor cortex. The striatal GABAergic neurones receive side-loop excitatory glutamatergic (Glu) input from the motor cortex and modulatory dopaminergic (DA) projections from the substantia nigra pars compacta (SN-PC). There are also balancing cholinergic (ACh) interneurones. The striatal neurones express both excitatory 01 and inhibitory 0 2 receptors. The output from the striatum to substantia nigra pars reticulata (SN-PR) and internal globus pallidus (GP-I) follows a direct and an indirect pathway. The direct pathway modulated by DI receptors releases inhibitory transmitter GABA, while the dominant indirect pathway modulated by D2 receptors has two inhibitory (GABAergic) relays and an excitatory (glutamatergic) terminal. Due to this arrangement, dopaminergic action in the striatum exerts inhibitory influence on SN-PR and GP-I via both the pathways. The output neurones from SN-PR and GP-I feedback on the motor cortex through the thalamus using an inhibitory GABAergic link and an excitatory glutamatergic terminal. The basal ganglia modulatory loop serves to smoothen output to the spinal motor neurone and reduce basal tone. The degenerative lesion (in SN-PC) of Parkinson's disease (PD) decreases dopaminergic input lo the striatum, producing an imbalance between DA and ACh, resulting in hypokinesia, rigidity and tremor.

455

ANTIPARKINSONIAN DRUGS

-...

PERIPHERY

Tolcapone Entacapone

l l

... C

·ro...

..._

-

Tolcapone

l .. l

····--···-r-•

~.> Levodopa

-0

DOC

0 0

i:o

Dopamine

..___

•• • - - - .. Inhibition (-)

CO 'T

co I

Carb1dopa Benseraz1dr--····c·) DOC Dopamine

3-0MD

co

(..:'\ C">MT Levodopa

BRAIN

Q)

·c ro

3-0MD

~·········:

3-MT MAO·B

0/t AC : COMT :

Seleg1line

··T··c·.\ l

AO

8

Tolcapone ······~·C?.,~~ (-)

HVA

' · · ·· · ·· · - - - - · · - - - ·

i :

....... f

Fig. 31.2: Metabolic pathways of levodopa in the periphery and the brain. 3·0MD-3·0-methyldopa; COMT-Catechol•O•methyl transferase; MAO--monoamine oxidase; 3·MT3•methoxytyramine; DOPAC-3,4 dihydroxy phenylacetic acid; HVA- Homovanillic acid (3•methoxy4-hydroxy phenylacetic acid), DOC-Dopa decarboxylase

somatotropes to increase G H re lease. Thoug h prolactin levels in blood fall during levodopa therapy, increased GH leve ls are not noted in parki nson ian patients. Probably the mechanisms regulatin g GH secretion are a ltered in these patients.

active transport across bra in capi llaries, a lso undergoes the same transformation. The plasma t½ of levodopa is 1- 2 hours. Pyridoxal is a cofactor for the enzyme dopa-decarboxylase. The metabolites are excreted in urine mostly after conjugation.

PHARMACOKINETICS

ADVERSE EFFECTS

Levodopa is rapidly absorbed from the smal l intestines by utilizing the active transport process meant for aromatic amino acids. Bioavailabi liry of levodopa is affected by: (i) Gastric empty ing: if slow, levodopa is exposed to degrading enzymes present in gut wall a nd liver fo r a longe r time- less is available to penetrate blood-brain ba rrier. (ii) Am ino acids present in food compete for the same carrier for absorption: blood level are lower when taken with meals. Levodopa undergoes high first pass metabolis m in g.i. mucosa and liver. The peripheral and centra l pathway of metabolism of levodopa is depicted in Fig. 31.2. About I % of administered levodopa that enters brain, aided by am ino acid carrier mediated

Side effects of levodopa therapy are frequent and often troublesome. Most are dose-related and limit the dose that ca n be administered, but are usually reversible. Some are prominent in the beg innin g of th erapy while o thers appear late.

At the initiation of therapy These side effects can be minimized by starting with a low dose. I. Nausea and vomiting It occurs in almost every patient. Tolerance gradually develops and then the dose can be progressively increased. 2. Postural hypotension It occurs in about 1/ 3 of patients, but is mostly asymptomat ic; some patients experience dizziness, few have fainting attacks. IL is more common in patients receiving antihypertensives. Tolerance develops w ith continued trea tment and BP norma lizes.

456

DRUGS ACTING ON CENT RAL NERVOUS SYSTEM

} Due to P adrenergic action arrhythmi~s of peri_pheral ly formed DA; 4. Exacerbation more in patients with preof angina existing heart disease. 5. Alteration in taste sensation 3. Cardiac

After prolonged therapy I. Abnormal movements (dyskinesias) Facial tics, grimacing , tongue thrusting, c horeoathetoid move ments of limbs start appeari ng after a few mo nths of use of levodopa at optimum therapeutic dose. These dyskinesias worsen with time and practica lly all patients get involved after few years. Their intensity correspond with levodopa levels. o tolera nce deve lops to this adverse effect, but dose reduction decreases severity. Abnormal movements may become as di sabling as the origina l disease itself. and are the most important dose-limiting side etTects. 2. Behavioural effects They range from mild an xiety, nightmares, etc. to severe depression, mania, halluci natio ns, menta l confusion o r frank psychosis. Excessive DA action in the limbic system is probably respons ible (a ntidopaminergic drugs a re a ntipsycho tic). Lcvodopa is contraindi cated in patients with psychotic illness. 3 . Fluctuation in motor performance After 2- 5 years of therapy, the level of control of parkinsonian sympto mato logy starts showing fluc tuation. 'E nd of dose' deterioration (wearing off) w hic h is in itially gradual, develops into rapid 'switc hes' or 'on-ofT' effect. With time 'all o r none' response deve lops, i.e. the patien t is alternate ly wel l and disabled. Abnormal movements may jeopardise even the 'on' phase. This is probably a reflection of progression of the disorder. With progres ive degeneration of DA neurones the abi li ty to regulate storage and release of DA may be largely lost: DA is then synthesized in the stri atum o n a mo ment- tomoment basis resulting in rapid and unpredictable fluctuations in motor control. Dose fractio nation a nd more frequen t ad mi nistration tends to diminish these fluctuations for a time.

Cautious use of levodopa is needed in the elderly; patients with ischaemic heart disease;

cerebrovascular, p ychiatric, hepatic and renal ?i sease; pe pti c ulcer ( ri sk of bleedin g is mcrcased); glaucoma and gout. Dose: Stan" ith 0.25 g BO after meals. gradually increase till adequate response is obtained. Usual dose 1s 2- 3 g/day. LEVOPA. BIOOPAL 0.5 g tab.

Interactions I. Pyridoxine: Abolishes the therapeutic efTcct of levodo pa (if it is not combi ned with carbidopa) by en hancing it peripheral decarboxylation so that less of it remains ava ilable to cross to the brain. 2. Phenothiaz ines, butyrophenone , metoclopramide reverse the therapeutic effect of levodopa by blocking DA receptors. The a ntidopaminergic domperidone blocks levodopa induced nausea and vomiting without abolishing its antiparkinsonian effect, because domperidone does not cross blood-brain ba rrier, bu t reache s CTZ. Domperidone can be used to al lay vom iting due to levodopa. Reserpine abolishes levodopa action by preventing entry of DA into synaptic vesicles. 3. Nonselective MAO inh ibitors: prevent degradation of DA a nd A that is synthesized in excess from th e administered levodopa at peripheral si tes. This may cause hypertensive crisis. 4. A ntihypertensivc drugs: postura l hypote nsion caused by levodopa is accentuated in patients receiving antihypertensive drugs; reduce their dose if levodopa is started. 5. Atropine, and antiparkinsonian anticholinergic drugs have additive therapeutic action w ith low doses of levodopa, but retard its absorption- more ti me is avai lable for peripheral degradatio nefficacy of lcvodopa may be reduced (no t w hen combined with carbidopa).

PERIPHERAL DECARBOXYLASE INHIBITORS Carbidopa and benserazide are extracerebral dopa decarboxylasc inhibitors ; they do nol penetrate blood-brain barrier and do not inhibit conversion of levodopa lo DA in the brain.

ANTIPARKIN SONIA N DRUGS Administered along with levodopa, they increase its t½ in the periphery and make more of it available to cross blood-brain barrier and reach its site of action.

Beneji1s of the combination arel . The plasma t½ of lcvodopa is prolonged and its dose is reduced to approximately I/4th. 2. Systemic concentration of DA is reduced, na usea and vo miting are m inimizedtherapeutic doses of levodopa can be attained more quickly. 3. Cardiac complications are reduced. 4. Pyridoxine reversal of levodopa efTcct does not occur. 5. 'On-off' elTcct is minimized since cerebral DA levels are more sustained. 6. Degree of impro vement may be hi gher ; some patients, not responding adequately to levodopa alone, also improve.

Problems not resolved or accentuated are-

may e, en be more

I. Involuntary movements d d . . . pronounce an 2. Behavioural abnonnaltt1es } appear earlier.

3. Excessive day time s leepiness in some patients. 4. Postural hypotension. Currently, levodopa is practically always used along with a decarboxylase inhibitor, except in patients who develop marked involuntary movements with the combination. Combination of levodopa with carbidopa has been given the name 'Co-careldopa'.

Preparations and dose Carbidopa l...eiocbpa (per /ab/cap) 10 mg + 100 mg T IDO~ET-LS, SY!' 24 hr

24 hr

> 24 hr

>24 hr

8 . Daily dose (mg)

25-150

2.5-40

5-40

10-40

2-8

1.25-10

1. Chemical nature

• I½ including that of active metabolite

DRUGS AFFECT IN G RENIN-ANGIOTENSIN SYSTEM to other inactive products occurs. Efficacy and tolerance of perindopri l are s im ilar lo other ACE inhibitors. COVERSYL 2, 4 mg tab.

Fosinopril

This ACE inhibi to r is un ique in bei ng a phosphinate compound that is glucuronide conj ugated a nd eli minated both by li ver a nd kidney. T he t½ is no t al te red by renal impa irment a nd the dose re ma ins th e same. However, like most ot hers, it is a prod ru g suitab le for once dai ly administration. First dose hypotension is more likely. Dose: In itially IO mg (elde rly 5 mg) OD; maximum 40 mg/day.

FOS l'\ACE, FOVAS 10, 20 mg tabs.

Ramipril

The distinctive feature of this longacting ACE inh ibito r is its extensive ti ssue distribution. Greater inh ibition of local RAS has been claimed. However, whether this confers any therapeutic ad vantage is not known. The plasma t½ of its active metabo li te rami prilat is 8- 18 hours, but termi na l t½ is longer d ue to slow release of tissue bound drug. C'ARDACE, RAM IRI L, CORPR IL, R.PRIL 1.25, 2.5, 5 mg caps.

Quinapril A prodrug carboxyl ACE inh ibi tor that is rapid ly and completely converted in the liver lo the active form Quinapri lat. Like ramiprilat, it is highly bound to the tissue ACE and exhibits a biphasic plasma t½ of 2 hours and 24 hours. Elimination occurs in urine and bile in a ratio of 2: I. Dose: I 0-40 mg/day ACCU PRIL-11: Quinapnl 20 mg + hydrochlorothta7ide 12.5 mg tab.

Trandolapril It is a carboxyl prodrug that is 40- 60% bioava ilable in th e active form. Absorption is delayed but not decreased by food. The peak effect occurs at 4 6 hours. It is partly metabolized and eliminated both in urine and faeces. The plasma t½ o f active metabolite is biphasic I 0- 24 hours, suitable for once da ily dosing. Dose: 2-4 mg (max 8 mg) OD; 7ETPRIL I, 2 mg wbs.

lmidapril

The oral bioavailabil ity of this longacting prodrug ACE inhibitor is 40%, which is reduced by tak ing the drug w ith meals. T he

peak effect occurs at 6- 8 hours and plasma t½ is >24 hours. Dose: Initially 5 mg OD taken I hour before food: usual maintenance dose 10 mg OD. TANATRIL 5, 10 mg tabs.

Benazepril Another nonsu lfhydry l prodrug ACE in hibitor; has a bioavailability of 37% a nd is excreted by kidney with a t½ of 10- 12 hr. Dose: 10 mg initially. max 20-40 mg/day; BENAC'F.. 5, 10, 20 mg tab.

USES

1. Hypertension The ACE inhibitors a re first line drugs in all g rades of hypertension, but the angiotensin receptor blockers (A RBs) have now surpassed them in popularity. A bout 50% patients of essential hypertension respo nd to monotherapy with ACE inhibitors and many of the rest to their combination with diuretics or blockers. The ant ihyperte nsive effect o f lower doses develops gradually over 2- 3 weeks. T hey offer the fo ll owing advantages: • Free o f pos tura l hypotens ion, e lectroly te disturba nces, feeling of weakness and CNS effects. • Can be used in a sthmatics, diabetics and peripheral vascular disease patients. • Long-term ACE inhibitor therapy has the potential to reduce incidence of type 2 diabetes in high risk subjects. • Secondary hyperaldosteronism and K+ loss d ue to diu retics is prevented. • Renal blood fl ow is well maintained. • Left ven tricular hypertrophy and increased wa ll-to-l umen ratio of blood vessels that occurs in hy pe rtensive patients is reversed. o hyperuricacmia, no deleterious effect on plasma lipid profile. • No rebound hypertension on withdrawal. • Minimum wo rsening of quality o f life parameters like general well being, work performance, sleep, sexual performance, etc. Large multicentric trials have co11fim1ed that ACE inhibitors reduce cardiovascular morbi dity and increase life expectancy of hypertens ive

533

534

CARDIOVASCULA R DRU GS patients. It appears that by their specific effect on myocardial and vascular cel l growth/ remode ling, they have grea ter protec ti ve pote ntia l than other classes of antihypertensive drugs. ACE inhibitors are elTcctive and first choice drugs in renovascular and resistant hypertension. T hey are particularly suitable for diabetic hypertensives in whom th ey reduce card iovascular complications more than other antihype rte ns ive drugs, probably by improving endothelial fu nction.

2. CHF ACE inhi bitors cause both arteriolar and venodilatation in Cl IF patients; reduce afterload as well as preload on heart and improve a ll haemodynam ic parameters. T hough they have no direct action on myocardium, stroke volume and cardiac output are inc reased , wh ile heart rate is reduced. Diu resis occurs initially and the accumulated salt and water a re lost due to improved renal perfus ion and abolition of mineralocorticoid mediated a+ retention. Cardiac work as measured by heart rate x pressure product is reduced; thereby, exercise capacity of CHF patients is enhanced. Beneficial effects are wel l sustained with chronic therapy and the NY HA fu nctional class of most patients is improved. Robust multicentric trials have shown that ACE inhibi tors retard the progression of left ve ntricular systo lic dys function and prolong survival of CHF patients of a ll grades (I to IV). Morta li ty is reduced by ~ 20% in symptomatic C H F patients. Un less contraindicated , ACE inhibitors are now advocated by several professional bodies, including American Heart Association a nd Ameiican College of Card io logy, as fi rst line drugs in all patients with symptomatic as well as asymptomatic left ventricular inadequacy because even in asymptomatic cases, they prevent progression to overt heart failu re. A di uretic, p blocker with or w ithout digital is may be added according to need. ACE inhibitors reduce episodes of decompcnsation. myoca rd ial infarction and sudden death. In addition to improved haemodynamics, longterm benefits of ACE inhibitors acc rue fro m

withdrawal of Ang II m ediated ventricu lar hypertrophy, remodeling, accelerated myocyte apoptosis and fi brosis. Ind irect benefits occur due to reduction in sympathetic activation and aldosterone levels. The Assessmem of Treatment with Lisinopril and Survival (ATLAS) trial on 3 164 heart failure patients (NYHA class II 10 IV) has shown that high close lisinopril (32.5- 3S mg/day) g iven for 39-58 months was more efT~cti vc in reducing all cause mortality. hospitalization for heart fa ilure and risk of MI than lower dose (2.5- 5 mg/day). To afTord maximum protection against progression o f heart fai lure. the dose of ACE inhibitors needs to be titrated to nearly the upper lim it of recommended do e range, as shown in other mega trials like G ISSl-3, SOLVD, AIRE, etc. as well. ACE inhibitors are efTec1ive in reducing developme nt of ventricular dysfunction, heart failure and related monal ity in post-Ml patients also (SAV E, TRACE, AIRE trials).

3. Myocardial infarction (Ml)

Several megatrials have es tablished that oral ACE inh ibitors adm inistered while Ml is evolving (within 24 hr o f an attack) and continued for 6 weeks reduce e arly as well as long-term morta lity, irrespective of presence or absence of systolic dysfunction, provided hypotcnsion is avoided. In high risk patients and those with latent or overt ventricula r dysfunc tion (C HF) extens ion of therapy continues to afford survival benefit over years. In unstable angi na/non-ST segment e levation Ml ( ST EM l), long-term ACE inhibitor therapy reduces recurrent MI and need for coronary angioplasty (SAVE and SOLVD tria ls), though no benefit was apparent in the short- te rm ( ISJS-4 study). C urren t evide nce shows that if there are no contraind ications, all Ml patients stand to gain from ACE inhibitor therapy, though magnitude of benefit is greatest in those hav ing associated hyp ertension a nd/ or diabetes.

4. Prophylaxis in high cardiovascular risk subjects The results of Heart Outcomes Prevention Evaluation (HOPE) study in 9297 post-M I and other high risk subjects, but having no left ventricular dysfu nction or heart failure have shown that ramipril reduced cardiac death a nd MI or stroke by 2 2% over a period of 4.5 years. Risk of developing heart failure or d iabetes was a lso reduced. These results we re

DRUGS AFFECTING RENIN-A NGI OTENSI N SYSTEM

confi nned by the EUROPA trial and appear to hold true even for patients who have undergone coronary revasculari zation (APRES trial). Thus, ACE inhibitors a re protective in high cardi ovascular risk subjects even when there is no associa ted hypertension or left ventricular dysfunction. Protective effect is exerted both on myocardi um as well as vasc ul ature; may involve improved endothelial function, and is independent of hypotensive action.

5. Diabetic nephropathy

Prolonged ACE inhibitor therapy has been found to prevent or delay end-stage renal disease in type I as well as type II diabetics. Albumin uria (an index of glomeru lopathy) remai ns stable in those treated with ACE inhibitor, but aggravates in untreated diabetics. Treated patients have higher creatinine clearance, require less dialysis and have longer life expectancy. Benefits appear co be due to haemodynamic (systemic and intrarenal) as well as abnorma l mesangial cell growth allenuating effects of ACE inhi bitors. T hey red uce intrag lome rular pressure and hypertiltratio n. ACE inhibitors arrest/partly reverse any degree o f a lbuminuria, but benefits in type 2 diabetics are rather limited once macroalbuminuria has set in. The RAS seems to accentuate micro- and maerovascu lar complications in diabetics, and ACE inhib itors have specific organ protective effect by attenuating the same. A ll patients with diabetic nephropathy, whether hypertensive or normotensive, deserve ACE inhibitor therapy. Deterioration of re tin opathy in diabetics also appears to be retarded by ACE inhibitors.

Nondiabetic nephropathy T here is evidence now that chronic rena l failu re due to nondiabetic causes may a lso be improved by ACE inhibitors. These drugs reduce proteinuria by decreasing the pressure gradient across glomerular capilla ries as well as by altering membrane permeability. T his retards disease progression. Among hypertens ive nephropathy patien ts the inc idence of doubli ng of serum creatinine or end stage renal failure is significantly lower in those treated wi th ACE inhi bitors than those treated with other antihypenensive drugs .

6. Scleroderma crisis

T he mark ed rise in BP and deterioration of rena l funct ion in scleroderma cri sis is mediated by Ang II. ACE inhibitors produce dramatic improvement and are life saving in thi s cond ition. Captopril test This test has been devised to obviate the need for renal angiography for diagnosis of renovascular hypertension. The basis of the 1cs1 is-acute b lockade of Ang II lonnation by captopril results in a reactive increase in PRA which is much higher in renovascular compared 10 essential hypertension. llowcver, this test is on ly of adjunctive value.

ANGIOTENSIN ANTAGONISTS (Angiotensin receptor blockers or ARBs) Ove r th e past 25 years, several nonpeptide orally active AT1 receptor blockers (ARBs) ha ve been developed as alternati ves to ACE inhibitors. These include losarran, candesartan, valsartan, telmisartan, olmesartan and irbesartan. Selective antagon ists of AT2 receptors as well as combined AT 1 + AT 2 antagon ists have also been produced, but are not used c lin ically.

Losartan

It is a competitive a ntagonist a nd inverse agonist, I 0,000 times more selective for AT 1 than for AT 2 receptor; does not block a ny other receptor or ion channe l, except thromboxane A 2 receptor (has some platelet antiaggregatory pro perty). A II overt actions of A ng 11 , viz. vasoconstriction, centra l and peripheral sympathetic sti mulation, release of aldosterone and Adr from adrenals, renal actions promoting salt and water reabsorption, central actions like thirst, vasopressin release and growth-promoting actions on heart and blood vesse ls are blocked. No inhibition of ACE has been noted. Pharmacologically, AR.Bs differ from ACE inhibitors in the following ways: • They do not interfere wi th degradation of bradykinin and other ACE substrates: no rise in level or potentiation of bradykinin , s ub sta nce P occurs. Co nsequently, ACE inhibitor related cough is rare. • They result in more complete inhibition of AT 1 receptor activation, because responses to Ang II generated via a lternative pathways

535

536

CARDIOVASCULAR DRUGS and con sequent AT 1 receptor activation (which rema in intact with ACE inh ibitors) are also blocked. • They result in indirect AT 2 receptor acti vatio n. Due to blockade of AT I receptor mediated feedback inhibition- more Ang 11 is produced w hi ch acts on AT 2 receptors that remai n unb locked. • ARBs cause little increase in the level of Ang ( 1-7) which is raised by ACE inhibitors, since Ang ( 1-7) is also partly degraded by ACE. The impact of these differences on clinical efficacy and therapeuti c value of the two classes of RAS inh ibitors is not known. Losartan causes fall in BP in hypertensive patients wh ic h lasts for 24 hours, while HR remains unchanged and cardiovascular reflexes a re not inte rfe red. No s ignificant effect on plasma lipid profile, carbohydrate tolerance or insulin sensitivity has been noted. A mild probenecid-like uricosuric action is produced.

Pharmacokinetics Oral absorption of losartan is not affected by food, but bioavailability is o nly 33% due to first pass metabolism. It is partially carboxylated in liver to an active metabo li te (E3 l 74) which is a I 0- 30 times more po te nt no ncompetiti ve AT 1 receptor antagonist. After oral ingestion peak plasma levels are attai ned at I h r fo r losartan and at 3-4 hours for E3 174. Both compounds are 98% plasma protein bound, do not enter brain and are excreted by the kidney. The plasma t½ of losartan is 2 hr, but that of E3 I 74 is 6- 9 h r. o dose adj ustment is required in re nal insufficiency, but dose should be reduced in presence of hepatic dysfunction. Adverse effects Losartan is well to lerated; has s ide effect profi le simi lar to placebo. Like ACE inhibitors it can cause hypotension a nd hype rka le mia, but first dose hypotens ion is uncomm on. Tho ug h, a few reports of dry cough have appeared, losartan is cons idered to be free of cough and dysgeusia inducing potential. Patients with a history of ACE inhibitor related cough have taken losarta n witho ut recurrence. Angioedema is reported in fewer

ca es. Headache, dizzines , weakness and upper g.i. side effects are mi ld and occasional. However, losartan has fetopa thic potential like ACE inhibitors; it should not be adminis tered during pregnancy. Dose: 50 mg OD, rarely BD; in liver disease or volume depleted patients 25 mg OD: addition of hydrochlorothiazide 12.5 25 mg enhances its effectiveness, LOSA CAR. LOSAR, TOZAAR, ALSARTAN 25, 50 mg tabs.

Candesa rtan

It has th e highest affinity for the AT 1 receptor and produces largely un s urmountable antagonism, probably due to slow dissociation from the receptors or receptor desensiti zation. Elimination occurs by both hepatic metabolis m and renal excreti on with a t½ of 8- 12 hours: action lasts 24 hours. Dose: 8 mg OD ( max 8 mg BD), liver/kidney impaim1cnt

4 mg OD.

CANDESAR 4, 8, 10 mg tab., CANDILONG, CANOESTA N 4, 8 mg tabs,

lrbesartan

The oral bioavailability of this ARB is relatively high. It is partly metabolized and exc reted main ly in bile . The t½ is ~ 12 hours. Dose: 150-300 mg OD. lROVEL. IRBEST 150, 300 mg tab, .

Valsartan The AT 1 receptor affinity of valsartan is simi Jar to that of Iosa rtan . Its oral bioavailabi li ty averages 23% a nd food interferes with its absorption. Elimination occurs mainly by the li ver in unchanged form with a t½ of 6- 9 hours; acti on lasts 24 hours. Dose: 80-160 mg OD I hour before meal (initia l dose in li ver disease 40 mg), DI OVAN, STARVAL. VALZAAR 40, 80. 160 mg tahs.

Olmesartan

Another potent ARB with high affinity for AT1 receptor. It is available as an ester prodrug which is complete ly hydrolysed during absorption fro m the gut. It is e li minated in urine as well as in bile with a t½ of - 12 hours. o dose adjustment is needed in liver or kidney disease, unless it is severe. Abdominal pain and sprue-lik e symptoms have been reported in few recip ients. Dose: 20-40 mg OD; OLMAT. OLSAR 20, 40 mg tabs.

Telmisartan

The AT 1 receptor blocking action of telm isartan is similar to losartan, but it does no t produce any active metabolite. After an oral dose, peak action occurs in 3 hours and

DRUGS AFFECTING RE NIN-ANG IOTEN SIN SYSTEM action lasts > 24 hours. It is largely excreted unchanged in bile; dose reduction is needed in liver disease. Dose: 20-80 mg OD. TELMA. TELSAR. TELVAS 20, 40, 80 mg tab;.

Eprosartan It is a newer AT 1 receptor a ntagonist which is believed to lower BP by blocking vasoconstrictor AT 1 receptors as well as by reducing NA release. Though, blockade of prejunctional AT 1 receptors on sympathetic neurones and resultant decrease in sympathetic outAow is a class effect of A RBs, thi s action is claimed to be more prominent in eprosartan, so as to be clinically meaningful. Oral bioavai labi lity of eprosartan is low (~ 13%) and it is mainly excreted unchanged in faeces. Renal excretion is about I 0% and the terminal elimination t ½ is 20 hours. Dose: 600 mg OD (range 400-ROO mg OD). TEVET EN, EPROZAR 600 mg tab.

CHF The ARBs afford c lear-cut symptomatic relief as we ll as survival benefit in CHF. However. thei r relative value compared to ACE inhibitors, especially in long-term morbidity and mortality reduction. is still uncertain. A number of large randomi,:ed endpoint trials like Eval uation of losartan in the elderly ( ELITE, 1997). ELITE-II (2000), OPTIMAAL (2002), Valsanan in acute Ml (VALIA T, 2003) have produced inconsistent results. This may be due to within g roup differences among specific ARBs. Valsanan and candesarlan have produced superior benefit, than losanan and irbcsartan. Thus, for CHF, it may he prudent to use ACE inhibitors as the first choice drugs and torcscr.c six,>Cific ARBs for those who fail to respond "ell or "ho develop cough/angioedema/other intolerance to ACE inhibitors.

Myocardial infarction

The evidence so far indicates that utility of ARBs in M l, including long-term survival, is comparable to ACE inhibitors. llowever, th e latter are genera lly used first, si nce there is greater experie nce with them.

Uses of ARBs

Diabetic nephropathy Several studies have

The ARBs have the same overall range of clinical utility as ACE inhibitors, but the suitability and efficacy of o ne over the other is not c learly defined; may depend on the condition being treated and/or specific features of the patient.

confirmed that ARBs are renoprotectivc in type2 diabetes mel litus, independent o f BP lowering. The magnitude of benefit is comparable to ACE inhibitors, but because of better tolerability profile, many cons ider ARBs to be the first choice now.

Hypertension

Losartan and o ther ARBs are now first line drugs, comparable in efficacy and desirable features to ACE inhibitors, with the advantage of not inducing cough and a lower inc idence of angioedema, rashes and dysgeusia. As such, they are more commonly prescribed now than ACE inhibito rs, thou gh superiority of one over the other is not established. Like ACE inhibitors, the maximum antihypertensive efTect is reached in 2 4 weeks and ventricular/ vascu lar hypertrophy/remodeling is arres ted/ reversed simi larly. o additional benefit has been obtained by combining an ACE inhibitor w ith an ARB in the treatment of hypertens ion. The Losartan inte rvention for endpoint red uction in hypertension (LIFE. 2002) study has found losartan 10 be more effective than J3-blockcrs in reducing stroke among > 9000 hypcncnsive patient, with lefl ventricular hypertrophy. and is approved for stroke prevention. In cirrhotics, losanan has been found 10 control ponal hypertension.

Combination of ACE inhibitors with ARBs T here are theoretical reasons to combine an ACE inhib itor with an ARB to obtain more complete suppression of RAS and achieve added cardioprotection in CH F or reno protcction in diabeti c ncphropathy. These are: •





Ang II is generated in severa l tissues (especially heart and kidney) by non-ACE mechani sms. whose effect can be blocked by ARBs. ACE inhibi1ors produce bradykinin and Ang (1-7) related vasodilatation and other effects that arc 1101 produced by AR Bs. ARDs cause compensatory increase in Ang II production that can be checked hy ACE inhibitors.

Additional haemodynamic and symptomatic improvement over short-term has been obtained in CHF with addition of an ARB to existing ACE inhibitor therapy. However, risk of hyperkalcmia, hypotension and renal impairment is a lso increased. Several large randomized tria ls

537

538

CARDIOVASCULAR DRUGS including Randomized evaluation of strategies for left ventricular dys function (RESOLVD , 1999), Va lsartan heart fai lure trial (VAL-He FT, 200 1), C HARM-additive tria l (2003), VA LI ANT (2003) trial , Ongoing Telmisa rtan alone and in combination with ramipri l g lobal endpoi nt tria l (0 TARGET, 2008) of combinations of A RBs and ACE inhibitors Vs their monotherapy in affording mortality and o ther endpoint benefits in CHF have yielded discordant res ults. Thus, it is not certa in that long-term use or ARB + ACE inhibitor combination is uniformly beneficial. However, selected cases, not optimally benefited by mo notherapy, may be offered com b ined therapy, especially by includ ing candesartan or valsartan as the ARB. In non-diabetic renal disease, the Combination treatment of ARB and ACE inhibito r randomized trial (COOPERATE, 2003) has concluded that ARB + ACE inhibitor combination therapy retards progression of non-d iabetic renal disease to a greater extent compared w ith their monotherapy.

DIRECT RENlN INHIBITOR Aliskiren Direct renin inhibitors (D Rl s) arc the latest class or RA S inhibitory drugs, of which only o ne membe r Aliskiren has become avai lable for the treatment of card iovascular a nd renal di seases in wh ich ACE inhibitors and A RHs are curre ntl y used . Al is ki ren is a no npeptide wh ich binds selectively to the catalytic site of renin and competiti vely blocks the access or a ng iotensinogen to this site. As a resu lt A ng 1 is not produced a nd the effector chain of RAS is interrupted. While the concentration of renin in plasma is increased by feed back, the plas ma renin activ ity (PRA) is decreased. Ang I and Ang 11 levels fal l in a dose-dependent manner. A lis k iren causes dose related fa ll in BP which is mo re marked in the a depleted subject wi th high basal PRA. Simi lar to ACE inhibitors, plasma aldosterone levels are

lowered accompanied by mild natriuresis and a te ndency to K• retention. The antihypertensive efficacy of a liski ren is nearly equ iva lent to that of ACE inhibitors o r ARBs. Combination of these drugs with a liskiren pro duces greater fall in BP, at least in the short term. This is due to blockade of rise in PRA caused by ACE inhibitors/ARBs. The pattern of haemodynamic effect of ali kiren resembles ACE inhibitors; postural hypotension is not a problem. Trials so far have shown that al iskircn reduces hypertensive left ventricular hypertrophy and benefits CH F patients, but its va lue compared to AC E inhibitors/ARBs as monothe rapy and as additional drug remains to be establis hed o n the basis of outcome data. Alis kiren reduces albuminuria and has renoprotecti ve effect in hypertension and diabetes mellitus. At present, aliskiren is recommended as an alternative antihypertensive drug (for those w ho do not respond or do not tolerate I st line drugs) and in combi nation w ith ACE inh ibito r/ AR B to achieve more complete RAS blockade and g reater BP lowering/cardioprotcction. However, addition o f al iskiren to ACE inhi bitor/ARB in type 2 diabetics has not produced any benefic ial effect; may even worsen outcome and is not ad vised.

Pharmacokinetics A liskiren is administered o rally, but bioavailability is very low due to active ex trus ion o f the absorbed drug by Pg lycoprotein. The drug is mainly e liminated in faeces; small amount in urine. The plasma t½ is > 24 hours, and its BP lowering effect persists for days after regular intake.

Adverse effects Aliskiren produces few and mild side e!Tccts- mainly dyspepsia, abdomi nal pain, loose motions, headache and dizziness. Acute hypotension, hyperka lemia, cough, angiocdema and rashes are less frequent than with ACE inhi bi tors. Aliskiren is co ntraindicated during preg nancy. D ose: 150-300 mg OD; RASILEZ 150 mg tab;

RASI LLL-HC: along "ith hydrochloroth,a7idc.

DRUGS AFFECTING RENIN-ANG IOTENSIN SYSTEM

er PROBLEM DIRECTED STUDY 36.1 A 65-year-male was diagnosed to be suffering from congestive heart fai lure (CHF). He had pitting edema of feet, dyspnoea and cough on mi ld exertion, fatigue, engorged neck veins, soft enlargement of liver, pulmonary congestion and mild cardiac dilatation. The pulse was 100/min, respiration 20/min and BP 130/86 mm of Hg. He was prescribedTab furosemide 40 mg once daily in the morning. Tab captopril 25 mg twice daily, morning and evening. After 2 hours of taking the medicines, he started passing increased quantity of urine and in the next few hours he gradually started feeling weakness, nausea, sweating and fainted while walking to the toilet. The pulse was recorded as 110/min and BP 80/40 mm Hg. (a) What could be the cause of sudden onset symptoms and the marked fall in BP? (b) Is t he choice of drugs incorrect? (c) How could such adverse event be prevented? (d) What immediate management is requ ired? (see Appendix-1 for solution)

539

Chapter

37

Nitric Oxide and Vasoactive Peptide Signal Molecules

NITRIC OXIDE

(Fig. 37. 1); flavin mononucleotide ( FMN ), flavin adenine dinucleotide (FAD), haeme and tetrahyd robiopterin {H4 8) act as coenzyme to OS. The rate of O production depends on the activity of NOS, because the substrate is normally present in excess. Three isoforms of NOS, viz. neuronal OS (nNOS or OS- I ), induc ible OS ( iNOS or NOS-2) and endothelial OS (e OS o r OS-3) have been characteri zed. Though all

It is one of the oxides of nitrogen with molecular formula o· indicating that it is a free radical with an unpai red electron. Nitric oxide is the smallest endogenous signal molecule and the only one which is a gas at normal temperature. Since its discovery as a biological mediator in 1980, it is now known to perform a wide array of functions. Furchgott and Zwadzki ( 1980) demonstrated that ACh caused vasc ula r s mooth mu scle relaxation only when the endothelial lining was intact, but not when it was denuded by rubbing. They proposed that on activation by ACh, the endothelial cells released a soluble 'endothelium deri ved relaxi ng factor' (EDRF) which diffused to the underl y ing s mooth muscle cell causing relaxation. A little earlier Farid Murad ( 1977) had already shown that GTN caused vasodilatation by releasi ng 0. lgnarro in 1987 confirmed that EDRF infact was NO. All three, Furchgott, Murad and lgnarro received obcl Prize in 1998 for these discoveries. Soon a number of other vasorelaxants (hi stamine, 5- HT, bradykinin, etc.) were also shown to act through EDR_F release. It is now established that NO is a key messenger in the cardiovascular and nervous systems, as well as in inflammatory and some other types of cells.

L

Generation and fate of NO: Nitric oxide is produced in the cells from the amino acid L-argeni ne by the enzyme nitric ox ide synthase ( OS) using molecular oxygen and NADPH

Fig. 37.1: Generation and metabolism of nitric oxide {NO) FMN- Flavin mononucleotide; FAD- Flavin adenine dinucleotide; NADP-Nicotinamide adenine dinucleotide phosphate; NADPH-Reduced form of NADP; H4 8- Tetrahydrobiopterin; NOS-Nitric oxide synthase

NH 2 I

NH 2 I

+

C=NH2 I

~H

NADPH+H' "'--

yH2 FMN CH2 Haeme I 02 CH 2 I + HCNH3

NADP'

C=O I NH

FAD H,B

CH 2 I CH2

I

/ NOS

I

cool •Arginlne

I

CH2 I • HCN H3 02

boo-

OxyHb) : / No·~ L-Citrulline Hb N02 NO;- (Nitrite) (Nitrate)

ci;+ No· -+ OONo· {Superox1de) (Peroxynitrite)

*

Lipid peroxidation-+ Cell injury

NITRIC OX IDE AND VASOACTIVE PEPTIDE SIGNAL MOLECU LES 3 isoforms are expressed by severa l types of ce lls, they are designated on t he basis of their primary localization. Wh ile eNOS and nNOS are constitutive enzymes, i.e. are synthesized at uniform rate, which is relatively low; the i OS found in macrophages, monocytes, neutrophils, hepatocytes, etc. is induced by bacterial endotoxins, cytokines and TNFa, etc. Inflammation induces production of large quantities of iNOS in macrophages a nd other cells. This results in generation of huge amount of NO which can continue over hours or even days. Activati on of eNOS and nNOS is Ca2+-calmodulin dependent. Agents and mediators activate eNOS/nNOS by raising cytosolic Ca2+. In contrast, i OS is active as such, and not Ca2+ dependent. After induction by inflammation, macrophages and other eel Is start generating NO without needing any iNOS activator. After generation, NO exists in tissues only for 4-10 seconds due to its rapid reaction with 0 2 to fonn nitrites (N02) , and with metalloproteins, especially with Hb, to produce nitrates ( 0 ) , both of which are excreted in urine. Ex posed to reactive oxygen species superoxide, NO reacts very avidly to form peroxynitrite, a highly reactive oxidant. Produced in excess, peroxynitrite leads to lipid peroxidation, DNA damage and other cytotoxic effects. Superoxide dismutase, a superox ide scavenger, enhances and prolongs NO action.

Actions and roles of NO 1. Vascular system: N itric oxide is a potent vasod ilator and serves to regulate vascular tone. Produced in the endothelium it freely diffuses to the nearby smooth muscle cell and activates a soluble guany lyl cyclase (sGC) raising cytosolic cGM P concentration (Fig. 37.2). This in turn activates a cGMP-de pendent protein kinase (PKG) w hi c h dephosphorylates the myosin ligh t chai n kinase (MLCK) -+ reduced availability of phosphorylated (active) MLCK so that actin-myosin interaction is interfered resulting in muscle relaxation. The PKG a lso

phos phorylates membrane Ca2• c hannels reducing influx of Ca2+-+ decreased availabil ity of Ca2+-calmodulin complex further interferes with MLCK-mediated actin- myosin interaction, promoting muscle relaxation. As already described, 0 mediates the vasodilator action o f endogenous vasodi lators like ACh, bradykinin, etc. There is some evidence that deficient NO production plays a role in the developme nt of hypertension. Bronchi al, gastrointestinal and other viscera l smooth muscles are also relaxed by NO, but the effect is less marked than in blood vessels. 2 . Platelets: Just as in smooth muscle, NO acti vates the cGMP pathway in plate lets to inhibit aggregation and thrombus formation. Defective endothelial and platelet OS-NO functio n increases the risk of thrombosis. NO also has antiatherogenic property. Adhes ion of leucocytes to the endothelium and cellular pro Ii feration/migration are suppressed by NO, conferring it a protective effect on plaque for mat ion. Defective NO generation is implicated in the development of atherosclerosis.

3 . Infection, inflammation and immune function: Nor ma lly the i OS acti vity in macrophages, etc. is very low. Bacterial endotoxins, T Fa, interferon and interle ukins induce iNOS in macrophages and other inAammatory cells wh ich then generate large quantities of NO. Activated macrophages also produce superox ides that combine with O to fo rm peroxynitri te which decomposes to release free redica ls that have broad antim icrobia I acti on on bacteria, viruses, fungi a nd protozoa. Thus, 0 helps in combating infection. 0 has many proinAammatory actio ns, viz. vasod ilatation, increased vascular permeability and en hanced PG production . On the other hand, excess and prolonged NO production has detrimenta l conseq ue nces by promotin g tissue inj ury. Raised iNOS and NO levels have been found in bronchial epithelium of asthma patients, in mucosa! lesions of inflammatory

541

542

CA RDIOVASC ULA R DR UGS

Ach / Hist / Bradykinin

~

l

PIP2

'___,/- r--~

DAG IP - - - - - 3 CaM

,I

CaM•Ca2• Argenine

. __ _ _

Ca2+

NO' Endothelial cell

-

-~-.........

e In h'b·t· ·············• 1 1 10n Fig. 37.2: Schematic depiction of major steps in endothelium•dependent vasodilatation and role of nitric oxide (NO) GPCR-G·protein coupled receptor (M/ H/ 8 2); PLC-Phospholipase C; PIP2- Phosphatidyl inositol bisphosphate; DAG- Diacylglycerol; IP3- lnositol trisphosphate; eNOS-endothelial nitric oxide synthase; sGC- soluble guanylyl cyclase; PKG-Protein kinase G; MLCK- Myosin light chain kinase.

bowel di sease (IBD) a nd in many other c hronic inflam matory conditions. Excessive produc t ion of O has been impli ca ted in causing severe hypote nsio n auending septic shock. Sepsis induces iNOS in inflammatory cells as wel l as in many other types of cells which produce mas ive amount of O over long periods, contributing to the sustained and refractory fall in BP (see box). However, use of drugs which check O pro• duction has not yet yie lded survi val benefit in septic shock. 4 . Nervous system: Nitric oxide serves as ncurotransmitter/ ne uromodu lato r in ce ntral

Major actions of Nitric Oxide

• • • • •

Relaxes vascular and visceral smooth muscle Prevents platelet aggregation Serves as neurotransmitter in brain and periphery Mediates microbicidal action of macrophages Plays a role in inflammatory diseases, septic shock, pulmonary arterial hypertension, etc.

as wel l as in perip heral ne rvo us sys tem. Both n O and e OS are expressed in the centra l neurones, but unlike other tra nsmitters, there are no preformed sto res of 0. It is synthesized on demand and re leased by diffu s io n ( not exocytosis). NO is released

NITRIC OXIDE AND VASOACTIVE PEPTIDE SIGNAL MOLECULES at many neuronal sites in the brai n, the most prominent of which is the excitatory g lu tamate neurones. Activation of NMDA g lutamate receptors causes long-lasting Ca2influx tri ggering O release. Several functions are ascribed to NO in the brain, viz. nociception, increased cerebral blood flow, learning and memory. NO is believed to mediate 'syna ptic plasticity ' a process by which junctional transmission is strengthened in a use depe ndent manner. This is achi eved by retrograde diffusion of post-junctional NO to enhance prejunctional transmitter release. Excessive production of NO has been implicated in promoting 'excitotox icity' due to over- stimulation of g lutamate neurones. Aberrant NO generation has been ascri bed a causative role in degenerative neurolog ical diseases like Huntington 's disease, Park insoris disease, amyotrophi c lateral scleros is, etc. In the genitalia and g .i . tract, certain nonadrenergic nonchol inergic ( ANC) neurones function by releasing NO. It is now es tab l ished that pelvic nerve stimula tion causes penile erection by releasing O from NANC neurones in the corpora cavernosa. NO relaxes cave rnosal smooth muscle, fi lling the s inusoids with blood and causing penile tumescence (see p. 327). Accord ingly, sildenafil and other PDE-5 in hibitors are effective in the treatment of erectile dysfunct ion, because they inhi bit degradation of cGM P which mediates NO action. NO appears to p lay a role in relaxing pyloric sphincter and faci litatin g gastric emptying. Defective NO signaling is bel ieved to be invo lved in py loric stenosis a nd gastric dilatation.

5 . Lungs: NO appears to play a promi nent role in the regulation of pulmonary vascular resi stance and the deve lopment of pulmonary arterial hypertension (PAH). Sildenafil, the PD E-5 inhibitor w hich indirectl y potentiates the vasodi lator action of NO by preventing inactivation of cGM P, causes marked reduction

in pu lmonary artery pressure, and is the drug of choice for PAH (see p. 328). Inhalation of diluted O gas also lowers pulmonary a rtery pressure in PAH , and is the I st line therapy in newborns with se vere respiratory distress and raised pulmonary artery pressure. Inhaled NO re laxes bronchial smooth musc le as we ll.

Nitric oxide donors These are compounds which contain a O or similar moiety in their structure, that is released in the body, mostly enzymatically, to elicit smooth muscle relaxation. Organic nitrates, nitrites and sodium nitroprusside are the clinically usefu l NO donors. They are also called ' nitro-dilators' . However, the pattern of vasod ilatation produced by different O donors d iffers accord ing to the NO species that is released and the mechanisms involved in the re lease process.

Glyceryl trinitrate (GTN): It is a rapid ly acting antiangi nal drug (see Ch. 40) whic h main ly dilates ve ins and coronary arteries. Mitochondrial aldehyde dehydrogenase is the most important enzyme wh ich releases NO from GTN. This enzyme is abundantly expressed in veins, offering an explanation for the pre ferential venodi lator action of GTN. Jsosorbide dinitrate a nd lsosorbide mononitra1e generate a related NO species by process/enzy mes that a re not we ll character ized. They are s lower acting, but the pattern of action is simi lar to GT . W hen ad ministe red continuous ly, organi c nitra te s produce rapidly developing tolerance (w ithin few hours), which a lso weans off rapidly. Though the processes in vo lved in rap id ni trate tolerance a re incompletely understood, it is believed that the reactive oxygen species generated during metabolism of organic nitrates inhibit mitochondrial a ldehyde de hydrogenase a nd other enzymes whic h produce the NO species, limiting further action. Platelets do not have the enzymes to denitrate GTN and other nitrates. Therefore, organic nitrates

543

544

CARDIOVASCULAR DRUGS do not significantly inhibit platelet aggregation. This is in contrast to N O, which has potent antiaggregatory action. Amyl nitrite: It is the only organic nitrite that was used for ang ina pectoris. Inhaled amyl nitrite rapid ly d ilated both veins and arteries, including coronaries, and produced throbbing headache. Like nitrates, it a lso needs convers ion to a 1 0 species, but the enzymatic mechanism has not been elucidated. N itrate-like rapid tolerance has not been observed with amyl nitrite.

Sodium nitroprusside: Admi niste red by i.v. infusion, nitroprusside equally dilates arterio les and vei ns by releas ing O as we ll as by directly activating guanylyl cyclase. It rapid ly lowers BP and the action is titratabl e. The breakdown of nitroprusside into NO and CN moieties is both enzymatic and non-enzymatic; th e latte r by reaction with g lutathione (see Ch. 4 1). Not only endothelial cells, but RBCs and some other eel Is can spli t nitroprusside. However, the enzymes involved have not been identified. N itrate- like to lerance is absent even after continu ous infusion. The released CN moiety is detoxified by forming thiocyanate, w hi c h is excreted slowly. Toxic amount of thiocyanate may accum ulate if nitroprusside infus ion is prolonged. Nitic oxide as a therapeutic agent: Apart from being an important signal mo lecule. exogenously administered 0 gas has limited therapeutic application as well. Inhaled NO is indicated in severe respiratory distress in neonates with elevated pulmonary arterial pressure. It dilates pulmonary vessels, part icularly in areas o f the lung that are be::tter ventilated, reduc ing the ventilat io n-pe rfusion mi smatch and improving gaseous excha nge. T he elevated pulmonary artery pressure is lowered and cardiopulmonary function is improved. Inhaled NO improves cardiopulmonary function in adult patients as well with severe PAH, but this can only be a short-term m easure.

VASOACTIVE PEPTIDES Over the past 50 years it has been reali zed that a large number of peptides are utilized by the body as signa l molecules in the nervous system as well as for othe r cell-to-cell communicati ons. Some of these have pronounced d irect effects on the vasc ulature, and may be termed ' Vasoactive pe ptides'. These may be grouped into those

which constrict and those whi ch dilate blood vessels. Vasoconstrictor peptides

Vasodilator peptides

1 . Angiotensin II (Ang II)

1. Bradykinin and Kallidin (Kinins)

2. Vasopressin (AVP,

2. Natriuretic peptides

ADH)

(AN P, BNP)

3 . Endothelins (ET)

3. Vasoactive intestinal peptide (VIP)

4. Neuropeptide Y (NPY)

4. Substance P

5. Urotensin

5 . Calcitonin generelated peptide (cGRP) 6. Adrenomedullin

Angiotensin II (Ch. 36) a nd Vasopressin (Ch. 43) are considered in other chapters.

VASOCONSTRICTOR PEPTIDES

Endothelins (ETs) Endotheli ns are three (ET- I, ET-2, ET-3) close ly re lated 2 1 am ino acid peptides having potent vasoconstricto r action. Vasc ular endothelium primarily generates ET-I which is functionally the most re levant ET. It is a lso produced by nervous tissue, lungs, kidney, breast, e ndometrium and some other cells, w hile ET-2 and ET-3 have been located in kidney, brain and intestines. ET- I is synthesized in the e ndothelial (and other) cells as a larger peptide called prepro- endothelin-1 (prepro ET-I), w hic h is broken down first to a 38 amin o acid peptide named 'Big ET- I ' and then by an endothelin converting enzyme (£CE) to ET- I (see Fig. 37.3). Control of ET- I synthesis is exercised at the level of expression of prepro ET- I gene; synthesis is enhanced by tran s fo rm in g g rowth factor (TG F ~), cyto kin es (mainly IL- I) a nd vasoconstrictors like angiotens in II, vasopressin (AVP). On the other hand, vasodilators like NO, prostacyclin ( PG 12 ) and atria l natriure tic peptide (AN P) suppress production of prepro ET- I. Synthesis of prepro ET- 1 is immed iately transla ted into ET- I production and release from the eel I. Though ET- I is released into the circulation, it primarily acts locally to serve as paracrine

NITRIC OXIDE AND VASOACTIVE PEPTIDE SIGNAL MO LECU LES

TGFP, IL-1 , Ang.11,AVP, Sheer stress

NO, PGI, ANP

~/ __

+

PreproET-1

l

Big ET-1

L-Arg NO eNOS

ECEl ET-1

l

Ca2•

Endothelial cell

e ·············•

Inh.b. , 1tion

Fig . 37.3: Schematic depiction of endothelin-1 (ET-1} production in the endothelium, and its direct contractile (through ETA + ET5 receptors), and indirect relaxant (through ET8 receptors) effects on vascular smooth muscle ECE-Endothelin converting enzyme; Ang II-Angiotensin 11; TGFp-Transforming growth factorp; AVP-Argenine vasopressin; PGl2- Prostacyclin; IP- prostacyclin receptor; ANP- Atrial natriuretic peptide; L-Arg- L-argenine; NO-Nitric oxide; sGC-soluble guanylyl cyclase; cGMP-cyclic GMP; cAMP-cyclic AMP ; IL-1- lnterleukin; PLA2Phospholipase ~; eNOS-Endothelial nitric oxide synthase; IP3- Inositol trisphosphate; DAG- Diacylglycerol.

or even autocrine s ignal molecule. ETs are rapidl y c leared from blood (t ½ 4-7 m inutes) by peptidases in lungs and kidneys as well as by receptor binding. Actions and roles of endothelins The most promi nent action of ET- I is pronounced and re latively long lasting vasoconstriction in most vascular beds, including pulmonary, renal and coronary arteries. However, i.v. injection of

ET-I initially causes brief fa ll in BP (due to release of O and P GI, from the endothelium) followed by more marked and sustained rise in BP due to direct action on arterial smooth muscle. ET- I also increases heart rate and cardiac contractil ity which contributes to the rise in BP. Constriction of re nal vesse ls reduces GFR and urine output. Acting on adrenals, ET- I promotes aldosterone release which contributes to the salt and water retaining action. Action

545

546

CARDIOVASCULAR DRUGS of ET- I in the lungs causes bronchoconstriction a nd ri se in pulmonary artery pressure. Mitogenic action of ET- I promotes proli feration of vascular smooth muscle, cardiac myocyte and peri vascular fibrob lasts as we ll as other connective tissue ce lls. There is now evidence that £ Ts play a ro le in the regulation of BP in humans. Sustained overproduction of £Ts has been impli cated in the genesis of pulmona1y arteria l hypertension ( PA H), syste mic hyperte ns ion, cardi ac and vascular hypertrophy, atherosclerosis, coronary artery di sease, asthma and chronic renal disease. ET- I levels are e levated in chronic heart fa ilure, and this may contribute to the impa irment of card iac contractility.

Endothelin receptors: Two types of ET receptors have been identified, ETA and ETw Both are GPCRs. The ETA receptor is distinguished by its hi gher affinity fo r ET- 1 co mpared to ET-2 or ET-3, whereas all 3 ETs have simi la r affin ity for the ET O receptor. Vascular and other smooth muscles express mainly ETA and few ET8 receptors. Activatio n of both results in PLc - IP./D , AG- Ca 2+ re lease me di ate d contractile response (Fig. 37.3). On the other ha nd, vascular endothe liu m ex presses ET8 receptor, activati on of which indirectly e lic its vasodilatation by releasing O and PGl 2 .The transient initial vasodepressor response to i. v. ET- I is due to this action. Susta ined activation of both ETA and ETO receptors is m itogenic and resul ts in vascular/cardiac myocyte, peri vascular fibroblast and other connective tissue cell hyperplas ia. Severa l organs, viz adrenals, kidney, in testines and brain express more ET 8 than ETA receptors.

Endothelin receptor antagonists (ERAs): Characterizati on of ETA and ET8 receptors and appreciation of the etiological role of ETs in several cardiovascular and pulmonary disorders prom pted e ffo rts to develop ERAs, and have resulted in the approval of 3 ERAs for c linical use over the past 15 years. Though these ERAs are being tried in several conditions, the only a pproved indication so far is pulmonary arterial hypertension (PAH).

PAH is an uncommon condition characte rized by severe remodeling of sma ll pulmonary arteries causing rise in pulmonary artery pressure above 25 nun I lg, right heart fa ilure and early death. Etiologically, it may be idiopathic, heritable. or PAH associated wi th connect ive tissue disease or HIV infection or that due to congenita l heart disease, etc. ETA receptors are richly expressed in pulmonary artery smooth muscle, airway muscle and lung fibroblasts, more so in l'AH patients. ET- I is a potent pulmonary vasoconstrictor and smooth muscle mitogen. ERAs and drugs that augment O (sildenafil) or PG I, (epoprostenol) action are the ma instay in the treatment PAH. However, therapy

of

with these drugs is only palliati ve.

ERAs can be divided into:

Dual E~-ET8 antagonist: Bosentan, Macitenta n Selective ET,. antagonist: Ambrisentan S itaxsentan another selective ETA blocker has been wi thdrawn due to fa tal hepatotoxicity. Both dual ERAs and selective ETA antagon ist produce simi la r pharmacodynamic effects. In PAH patients they lower pulmonary arte1y pressu re, improve haemodynami c parameters and increase exerc ise capacity. Follow up of patients in c linic a l trials has shown that risk of worseni n g of c linical conditio n in PAH patients is decreased and deterioration delayed. Signifi cant reduction in morta lity compared to placebo is also reported. Efficacy of all 3 ERAs is comparable, but they differ in pharmacokinetics, drug interaction potenti al and tolerabi lity. All ERAs are nonpeptide and orally effective.

Bosentan: It is the first ERA to be approved for clinical use. Chemically it is a sulfam ide and dual ETA- ET 8 an tagonist. Both ini tial depressor and subsequent pressor response to i. v. ET- I is blocked. O ra l bioavailability of bosentan is - 50%. It is metabolized in li ver by CYP2C9 and CY P3A4 , and excre ted in bile; plasma t½ is 6 hours. One of the metabo lites is active. No dose adj ustme nt is needed in renal disease, but it is contrai ndicated in liver disease, because it causes dose-dependent e levation of hepatic am inotransferases and carries ri sk of hepat ic inj ury. Monthl y monitoring of liver fu nction test is mandatory. Bosentan ind uces microsomal CYP2C9 a nd CY P3A4 isoenzymes, a nd can interact with many drugs, including warfari n, oral contraceptives, glibenc lam ide and si mvasta tin.

NITRIC OXIDE AND VASOACTIV E PEPTIDE SIGNAL MO LECULES It a uto-indu ces own metabolism; the initial dose of 62.5 mg BO is mostly doubled to 125 mg BO after 4 weeks. Ketoconazole doubles blood levels of boscntan, and rifampin lowers its blood level. Due to the above problems, the other ERAs have overtaken bosen tan. BOSENTAS. Ll.,PIBOSE, BOZTAN 62.5, 125 mg tabs.

Ambrisentan: Developed after bosenran, it is the only selective ETA receptor antagon ist currently available, and has the theoretical advantage of retain ing the vasodi lator funct ion through the ET 8 receptor. However, no en hancement of clinical efficacy has been demonstrated. It is not a sulfamide derivative, and carrys a low risk of hepatic aminotransferase e levation. Ambrisentan is rapidly absorbed with oral bioavai labil ity of90%; metabolized by CYP3A4 and glucuronidation. It is excreted mainl y in bile; only 20% in urine. The t ½ is 12- 15 hours, permitting once dai ly dosing. lt is not an enzyme inducer, has low potential for drug interactions (contrast bosentan); has been safely given with warfarin, s ildenafil a nd o ral contraceptives without dose modification. Ambrisentan is genera lly we ll tolerated; peripheral edema is the most common s ide effect. In clinical trials liver enzymes were raised only in 2% patients. This was reversible and comparable to that with placebo. Ambrisentan has been combined with tadalafil (a PDE-5 inhibitor), a nd th is has improved efficacy as well as reduced the ri k of clinical failure (AM BITIO trial) in PAH patients. Dose: 5-10 mg OD.

PULMONEXT. AMBR ICAN, ENDOIJLOC. ZAMBR I 5 mg, 10 mg tabs.

Macitentan : It is a new dua l E RA with hig he r affinity for ETA than for ET 8 receptors. Macitentan has been found to bind tightly to the ETa receptor producing unsurm ountable blockade, which probably contributes to its enhanced activ ity. Ora l abso rption of ma eitentan is slow, takes 8- 10 hours. It is oxidized by CYP3A4 to an act ive metabolite which has a much longer t ½ than the 17 ho urs t½ of macitentan itscl f. Excretion is

largely urin ary and the rema111111g in b ile. o clinically relevant drug interactions have been no ted with macitentan despite its enzyme inducing potential. o modificatio n of warfari n and s ildenafil (wh ich are also given in PAH) dose was needed. In clinical trials it has reduced mortality in PAH patients by 45%, and is a pproved both for monotherapy as we ll in combi nation with a PD E-5 inhibi to r or i.v. cpoprostenol ( PG IJ Macitentan has a good tole rability profile; frequent side e ffects are headache, nasopharyngitis and anaemia. Peripheral e dema and aminotra nsferase elevation were similar to placebo group. Dose: 10 mg OD.

Adverse effects of ERAs The general side effects of ERAs are due to their vasod ilator property, produc ing headache, nasal congestion , fl ushing and pa lpitation. Peripheral edema is common , especially w ith ambrisentan.

Elevation ofaminotransf erase levels is the major adverse effect, m ore common with bosentan, but is reversible. With ambrisentan and macitentan, the incidence is similar to that with placebo.

Anaemia: A mild (0.5- 0.8% ) lowering of Hbo/o and haematocrit occurs with a ll ERAs. It is probably due to fluid retention and haemodilution.

Teratogenicity: All ERAs are teratogenic in an imals. In clinical trial s, ERAs have not been given to pregnant women, a nd t here is no human data, but all E RAs are contraindicated in pregnancy. Orally admi niste red ERAs are now a first line we ll to lera ted o ption for the treatment of PAH of various etiologies. They compare favourably with oral PDE-5 inhibitors (sildenafi l, tadalafil) and can be combined with them for enhanced efficacy. Conti nuous long term i.v. infusion of epoproste nol (PG 12 analogue) has been shown to afford rel ief as well as reduce mortality in PAH cases nonresponsive to oral

547

548

CARDIOVASCULAR DRUGS medication or those with se vere (functional c lass Ill or IV) disease. ERAs arc also bei ng eva luated for use in refrac tory chronic heart fai lure.

Neuropeptide Y (NPY) europeptide Y (NPY) first isolated from pig brain in 1982, is one of the mo t abundant neuropeptides in the brain and is also present in peripheral sympathetic ne rves. It is the leadi ng member of a family of three 36 amino ac id peptides, the other two are pancreatic polypeptide (PP) and peptide YY (PYY) . Wh ile PY is a neurotransmitter, PP and PYY are hormones. PP is secreted by pancreatic islets in response to food ingestion and acts in the C S as well a on gut to suppress appetite, delay gastric emptying and promote fuel uti Iization. Hormonal ce ll s in the ileum secrete PYY according to the quantity of food, and it serves to regu late appetite. ln the periphery NPY is mainly localized in the sympathetic nerves, especial ly those innervating blood vessels. It is stored in separate synaptic vesicles than those storing A (see Fig. 11.38 ). e rvc impulse releases PY along with A, and it appears to serve both as transmitter as well as cotransm itter (seep. 108). N PY exerts postjunctional as well as prejunctional actions. The postj unctional acti on in the vasculature is a direct slow but longlasting contractile effect, as we ll as potentiation of A-induced vasoconstriction. The prejunctional action is inhibition of nerve impulse induced transmitter release. ot only A release from sympathetic nerve endings, but also ACh release from parasympathetic nerves is attenuated. NPY ca uses vasoconstriction in nearly a ll vascular beds, especiall y ren al and cerebral vessels. It has a di rect positi ve inotropic and chronotropic effect on heart and increases a+ and water excretion by the kidney. despite renal vasoconstriction. In the brain, PY is present in several cortical and subconical areas and plays a key role in regulating food ima ke (N PY

strongly promotes feeding), energy homeostasis, stress response, autonomic out fl ow, H PA ax is acti vation and severa I othe r functions. The presynaptic action of N PY in the brain is to inhibi t re lease o f various ne urotransm itters, viz. NA, DA, 5-HT, gl utamate, GABA, as well as that of hypothalamic GnRH, AVP and oxytocin. NPY receptors: In humans, four types of Y receptors (YI' Y1• Y,. Y 1) have been characterized for PY and related peptides, of which PY has high affinity for three (Y ,. Y, , Y,). while Y, has preferential affinity for PP; so that IL may be regarded as PP receptor. The original Y3 receptor is now considered to be CXC chcmokinc receptor and not a Y receptor. A II Y receptors are G PC Rs, which tra nsducc their response by raising cytoso lic Cal• or by inhibiting adenylyl cyclase. The vasoconstrictor and other perir,hcra l post j unctional actions of PY are mediated mainly by Y, receptor, while Y2 is the prcjunctiona l receptor for PY in the periphery as well as in the brain. Both Y I and Y2 receptors are expressed in many areas of the brain. The Y, receptor is restricted to the brain, in areas "hich control food intake or hypothalamo-piluitary function and in limbic areas. Several selective nonpcptidc Y, receptor antagoni sts have been produced which block vasoconstrictor action of NPY. T heir use has indicated that N PY is probably not involved in the regulation o f normal BP or cardiac function, but that it may play a role in the development o f hypertension or heart failure. Selective Y, and Y, antagonists have also become available for investigation. Some of the Y, and Y5 antagonists arc ora lly active and have undergone limited clinical trial as antiobesiry agents, but none has been approved for clinical use.

Urotensin II (UII) Uro1e11si11 II (U/1) is a 11 amino acid cyclic vasoconstrictor ncuropcptidc identified in 1980 from the urophysis of tdeo,t fish and later demonstrated to be present in mammals including humans. It has been located in the cholinergic neurones of the brainstem and spinal cord as well as in kidney, heart and lungs. UII strongly contracts arterial smoot h muscle with less effect on , eins, and depresses the heart. A variety of other actions on the CNS as well as on peripheral organs have been shown, but their physiological relevance is uncertain. T hough UII circulates in blood, it docs not appear LO play significant ro le in the regulation of BP. The receptor for UII has been idenli fied as a cell surface GPCR and named UT recep!or. It is coupled lo Gq protein and produces vasoconstriction (as well as oth er effects) through the IP/ DAG intracellular Ca'· release patll\\ ay. The UT receptor is widely expressed in the CNS, heart, blood , essels. lung. kidney and adrenal medulla. Though UII appears lo be physiologically unimportant in man. it may play a role in certain cardiovascular

NITRIC OXIDE AND VASOACTIVE PEPTIDE SIG NAL MOLECULES and other diseases. Raised UII le, e ls have been noted in hypenension, advanced heart failure. diabetic renal failure. etc. Many pt!ptide and nonpcptidc UT receptor antagonists have been produced, and some ha, c been clinically tested, part icula rly in diabetic ncphropathy and in asthma, but none has been developed as a drug.

VASODILATOR PEPTIDES Bradykinin and Kallidin (Plasma kinins)

Plasma kin ins are polypeptides split ofT from a plasma globul in Kininogen by the action of s pecific enzymes Kal/ikreins . T he tw o important plasma kinins, Ka//idin (decapeptide) and Bradykinin (nonapcptide) were discovered around 1950 by two independent lines of inves tigation into the hypotensive activity of urine and certain snake venoms. These and some o ther biological nuids were fo und to act indirectly: they contained enzymes which generated vasodilator substances in the plasma. Kinins are generated by proteolytic reactions triggered by tissue injury, inflammation, allergic reaction, etc., and play important mediator roles. Generation and metabolism Kininogcns arc a , globulins present in plasma "hich also contains inacti\e kininogcnasc prekal/1kre111. Prekallikrein is activated by llageman factor (factor XII) which itself is activated by tissue injury and contact with surfaces having negative charge. e.g. collagen, basement membrane, bacterial lipo,accharides, urate crystals, etc. Plasmin faci litates contact activation of Hageman factor

(Fig. 37.4). Kinins are also generated by trypsin, proteolytic en1ymcs in snake and wasp venoms and by kallikrein present in kidney, pancreas and other tissues. Bradykinin is generated from high molecular "eight ( HMW) kminogcn by the action of plasma kallikrein, because IIMWkininogcn does nol cross Lhc capillaries. On Lhe other hand, kallidin can be produced from both low molecular weight (LMW) kininogcn as well as HMW-k ininogen by the action of tissue kallikreins. Bradykinin can also be generated from kallidin on the removal of lysine residue by an aminopeptidase. Plasma and tissues also contain kininogenase inhibitory factors of which co111pleme111 (C l) esrerase inhibitor (CI lnh) is the most impo11an1. Moreover. kallikrci ns arc normally present in their inactive 7ymogcn fonn s. Thus, physiologically only small amounts of kinins arc generated in plasma and tissues. Kinins are very rapidly degraded, primarily in lungs. but also in other tissues and have a t½ o f < l min. The princi pal degrading enzyme is Ki11inase II , a lso known as 'angiotcnsin-11 converting en.tyme' (ACE) which splits off 2 amino acids from the carboxyterminal o f the peptide chain. Another carboxypcptidase Ki11i11ase I removes on ly one amino acid (argin ine) producing selec tive B receptor agonistic metabolites (dcsA rg bradykinin and desArg kallidin) which arc fu rther degraded by other peptidase,.

Actions

Bradyki nin and kallidin have similar actions. 1. Blood vessels Kin ins are more potent vasodilators than ACh and histamine. The dilatation is mediated through endothelial NO and PG I, generation, and involves mainly the arterioles~ In contrast, large arteries. most ve ins and

Hageman factor

Fig. 37.4: Generation

and degradation of bradykinin and kallidin HMW- high molecular weight; LMW-low molecular weight.

549

550

CARD IOVASCULAR DRUGS

vessels with damaged endothelium are constricted by direct action of kinins on the smooth muscle. ln addition, they can release histamine and other mediators from mast cells. lnjected i.v. kinins cause flushing, throbbing headache and fa ll in BP. They markedly increase capi llary permeability due to separation of endothelial cells • exudation and inflammation occurs if they are injected in a tissue. Intradermal injection produces wheal and flare (similar to histamine). Kinins have no direct action on hea11; reflex stimulation occurs due to fall in BP. 2 . Smooth muscle Kinins induce contraction of intestine whi ch is characteristica ll y slow (bradys-slow, kinein- to move). They cause marked bronchoconstriction in guineapigs and in asthmatic patients. Action on other smooth muscles is not prominent. Some smooth muscles may be relaxed. 3. Neurones Kinins strongly stimulate nociceptive afferents and produce a burning sensation. Applied to blister base or injected intraperitoneally or in the brachia( artery, bradykinin produces intense, transient pain and has been used in analgesic testing. Kinins release CAs from adrenal medulla. Injected directly in brain they produce a variety of efTects including enhanced sympathetic discharge. They increase permeability of the blood-brain barrier. 4 . Kidney Kinins increase renal blood flow as well as faci litate salt and water excretion by action on tubules. The di uretic effect of furosemide is reduced by kinin 8 2 receptor antagonists, indicating participation of locally generated kinins in this response. Klnln receptors Existancc of lWO types o f kinin receptors ( B,, 8 2) has been es tablished. Both arc GPCRs. Bradyk.inin and Kallidin arc selective agonis ts of B, receptors, while lhcir dcs-Arg me1aboli1es generated by lhc action of kininase I are selective agonists of B 1 receptor. Most of the k.inin actions in noninHamed tissues arc mediated by 8 2 receplors which arc constituti, ely present on: • • •

Visceral smooth mus c l ~ntraction of intestine, uterus, airway. 0 release, vasodilatation, Vascu lar e ndotheliumincreased permeability. Sensory nerves- acute pain.

The B2 receptor is coupled to Gq and Gi prote ins which utilizes the phospholipase C-IP/ DAG-intracellular Ca'' mobilization tran ducer mechanism. Endothelial NO synthase (eNOS) is activated causing vasodilatation. Certain responses to ki nins, e.g. bronchoco nstriction , renal vasodilatation and activation ol' pain nerve endings appear to be mediated by stimulation of phospholipasc A and generation o f PGs at the local site. The activated Gq protein a lso mediates production of NF,cB which triggers MA P kinase pathway leading to generation of proinHammatory mediators. The 8 1 kin in receptors arc minimally expressed in normal tissues, but arc indu ced by inflammation and colocalize with kininase I enzyme, so that the B, agon istic des-Arg k.inins are produced at the same site. Activated B, receptor also transduces through Gq and Gi proteins and cause contraction of ve ins, large vessels, enhanced PG synthesis, leukocyte recruitment, generation of inducible 1OS (iNOS) and chronic pain in inHammed tissues.

Pathophyslologlcal roles I. Mediation of inflammation Kinins produce all the signs of inHammation-redness, exudation. pain and leukocyte mobili1.ation. Tissue injury can cause local kinin production which then sets in motion the above defensive and reparative processes. Activation of B, receptors on macrophages induces production o f I L-1, TNF-et and other inHammatory mediators. 2. Mediation of pain By directly stimulating pain nerve endings and by increasing PG production, kinins appear to serve as mediators of pain. The B, antagonists block acute pain produced by bradykinin, b~t induced B, receptors appear to mediate pain of chronic inHammation. 3. Functional hyperemia (in g lands during secret ion) and ..eg11/a1io11 of microcirculation-cspeciall y in kidney may be occurring through local kinin production. 4. Hereditary angioedema This rare hereditary disorder is due to quantitative or functional deficiency o f C-1 esterase inhibitor which is the most important natural ki• ninogenesis inhibitor. As a result plasma bradykinin levels rise manifesting as unpredictable episodes of swelling of lips, tongue, face, respiratory and g.i. traels, hands, feet, etc. Inhibitors of kin in product ion and 8 2 kinin receptor antagonists affo.rd, symptomatic relief. 5. Kinins appear to play no significant role in the rcgu la• Lio n of normal BP. However, they may serve to oppose overactive RAS and exert antiproliferative influence on cardiac and vascular muscle in hypertensive states. lschaemic preconditioning which limits tissue damage during myocardial infarction appears LO involve kinins.

6. Kinins cause closure of ductus a11eriosus, dilatation of foetal pulmonary artery and constriction of umbilical ves• sets. Thus, they may be involved in adjusting from foeta l Lo neonatal circulation.

NITRIC OXIDE AND VASOACTIVE PEPTIDE SIGNAL MO LE CU LES 7. Kinins play a major role in the development of angioedema. They also appear to be involved in shock, rhinitis. asthma, ACE inhibitor induced cough. carcinoid, postgastrectomy dumping syndrome, fluid secretion in diarrhoea, acute pancreatitis and certain imm,1110/ogical reactions.

Becaus e of evanesce nt and unpleasant actions, kinins have no clinical use.

Bradykinin antagonists After characterization of 8 1 and 8 2 kinin receptors, and in view of prominent pathophysiological roles of kin ins, several peptide and nonpeptide kinin antagonists have been produced and tested for clinical use. However, only one (lcatibant) has been marked so far in UK and Europe. lcatibant: It is a synthetic decapeptide bradykinin B, receptor antagonist which is resistant lo kinin degrading enrymcs, and has a duration of action uplo 6 hour.; after s.c. injection. It effectively reverses the swelling and other symptoms in hereditary angioedema, and is currently the best treatment option for this condition. Its utility in treating drug induced angioedcma. bums, pancrealitis and other conditions involving kinins, is being explored.

A recent controlled trial has reported icatibam injcc1cd s.c. to be more effective in resolving ACE inhibitor induced angioederna than the standard therapy with i.v. pre.dnisolone and an antihistaminic. (lcatibant is marketed in UK, Europe as I IRAZYR 30 mg/3 ml prcfilled syringe for ~.c. mjcction in abdominal urea). Compound 173657, comp. SSR 24061 2 and few other ora lly active nonpeptide B, antagonists have undergone trials and arc being developel as analgesics. antiinflammatory dnigs, and for treating pancreatilis, head injury. etc.

Natriuretic peptides (ANP, BNP, CNP) atriuretic peptides (NPs) are a family of 3 cyclic peptides, namely atrial P (A P), brain P (B P) and c-type P (CN P), al l having natriuretic, diuretic and vasodilator property. They have a com mon I 7 amino acid ring with differing straight chains.

ANP: It is a 28 amino acid peptide produced, mainly by atrial myocytes; also by ventric les, certain neurones a nd lungs. A P is the major circulating NP. Atrial and venrricular diste ntion percei ved by stretch receptors is the primary stimulus for ANP (a lso BNP) release, wh ic h is triggered by blood volume expansion, sympathetic stimulation (o. 1 receptor med iated).

A ngll, endothelin, vasopressin and aldosterone as well. Plasma A P level is raised in CHF, renal fail ure, primary aldosteron ism and in inappropriate AD H secretion syndrome. A P is synthesized in cardiac myocytes as a large peptide prepro-ANP, which is then split to pro ANP and finally to ANP wh ich is released.

BNP: Produced main ly by ventricles, also by brain and some other organs, B P ex ists in 2 forms-either 32 o r 26 amino ac ids. It is also first synthesized as prepro B P, and then broken dow n to B P in 2 steps. Releas ing stimuli for BN P are the same as for AN P, and the actions of the two peptides are identical. However, circulating levels of B P are lower than those of ANP. CNP: This 22 amino acid P is produced main ly by vascular endothelium and neurones in the CNS; also by kidneys and intestines. Circulating levels of CNP arc very low. The primary action of CNP is vasodilatation, while natriuresis and diuresis is mild. The physiological role of CNP is not clear.

Clearance of NPs: Occurs both by enzymatic degradation as well as by receptor mediated internalization followed by intracellular breakdown (see below). A neutral endopeptidase (NEP; also called Neprilysin) present in kidney, lungs and liver rapidly degrades Ps . The plasma t½ of A P is short (2- 5 min in humans), BN P has a longer t½, (4-20 min) probably due to less efficient receptor mediated removal, while that of C P is the shortest due to more marked receptor mediated removal.

Actions and roles: The N Ps produce two major actions, viz vasodilatation and natriuresisdiuresis (see box), and are involved in the regulation of a• and water balance, blood vo lume and arterial press ure. By directly dilating ve ins, Ps lower central venous pressure and preload on the heart, which tends to reduce cardiac output in normal subjects, but may improve it in decompensated heart. Ps a lso dilate arteries reduc ing systemic vascular resistance, and BP.

551

552

CA RDIOVASCULAR DRU GS

Vascular and renal actions of ANP/BNP • Systemic arterial and venous dilatation • !Sympathetic flow Venous pressure and preload on heart Systemic arterial pressure • Natriuresis a nd diuresis • t Renal blood flo w • Glomerular filtration rate and filtration fraction • ! Proximal tubular Na• reabsorption • ! Renin release from kidney ,I.Plasma angiotensin II ! Plasma aldosterone In additi on to direct vasodilato r actio n, N Ps decrease sympathe tic flo w by acting in the CNS as we ll as by reduc ing N A release fro m sympathetic ner ve termina ls. Ac ting on the kid neys, N Ps increase rena l blood flow, GF R an d filtration fraction. T hey also decrease proxima l tu bular a+ reabsorption resulting in natriuresis a nd d iuresis. M oreover, they inh ibit renin re lease from the kidney w hich

translates into lo we red plasma Ang ll reduc ing a ldosterone p rod ucti o n. T his a lso contributes to the natriures is (F ig. 37.5). In contras t to furosemide and thiaz ide induced natri uresis, the A P indu ced natriuresis is not accom pani ed by K· loss, becau se it a lso s uppresse s the re nin-Ang ll-aldostcrone mec ha nis m. Lowe red circulating Ang II contributes to the hypotensive actio n as well. O verall, the Ps seem to act as a counter regu la tory system to the re nin-A ng II-aldostero ne system to ma inta in blood volume and blood pressure ho meostas is, as we ll as a ' K• ba lance.

NP receptors: T he Ps act by combining w ith specifi c cell surface receptors, three s ubtypes o r w hich have been identified, namely AN PA (or PR,), AN P8 (or NP~) and AN Pc (or PR3) . ANPA and ANP8 receptors: Both these receptors are transmembrane enzyme-linked receptors

(see p. 58- 59) and Fig. 4.8) which have guany lyl cyclase activity at the cytoso lic domain and are

Sympathetic stimulation G Angiotensin 11 - - - -------'=-- - -- - - - - Renin!

ej

e 8"----! Renin •~- - ....::.

l

-=== i

-

! BP- - -- - - ! Blood volume - - -- - - Natriuresis Diuresis Fig. 37.5 : Counter-regulatory operation of natriuretic peptides against the renin-angiotensin 11-aldosterone system to maintain blood volume, blood pressure, salt and water homeostasis ANP- atrial natriuretic peptide; BNP- brain natriuretic peptide.

NITRIC OXIDE AND VASOACTIVE PEPTIDE SIGNAL MOLECULES located on the surface of target cells. Binding of ANP or BNP a t the ex tracellu lar domain promotes receptor dimerization and activation of the guanyly l cyclase. The rai sed cytosolic cGM P activates PK(, which brings about vascular smooth musc le re laxation, natriuresis and other responses by lowering intracellular Ca 2• and dephosphorylating MLCK (see Fig. 40.3). Whi le ANPA receptor has equal affi nity for ANP and BNP, the ANP8 subtype has high affinity for CNP and is involved mainly in causing vasodilatation.

ANPc receptor: Located mostly on vascular endothelium, it primarily serves as a c learance receptor, a nd has equal affin ity for AN P, B P and CN P. On binding the P, the receptor dimcrizes and is internalized. The peptide is broken down intracell ularly and the receptor is returned to the surface. The A Pc receptor is also linked to adenylyl cyclase inhibition and PLc activati on , but the physiological consequences of these are not well understood. Clinical application: A recombinant fonn of B ' P called Nesiritide is used clinical ly to a limited extent because of need for i.v. admm1stration and brief life-s pan in the body. Continuous i.v. infusion of nesiritide is occasionally employed to treat severely dccompensated patients of CHF. It benefits by causing natriurcsis-diurcsis, vasodilatationrcduction in cardiac pre- and afier-load and lowering of renin-Angll-aldosteronc levels. However. hypotension and renal injury are the risks, and cost is prohibitive. Seria l measurements of plasma BNP levels have been used to monitor progression of heart failure, and to assess response to therapy: levels fall with improvement.

Neutral endopeptidase (NEP; neprilysin) inhibitors: Thi s is a new class of drugs whic h inhi bit NEP enzymes that degrade Ps, bradyki nin, substance P and adrenomedullin. By raising levels of natriurctic and vasodilator peptides they have potential util ity in C HF. A nonpeptide EP inhibitor Sacubitril has been approved (in 20 15) for the treatment of class lI to IV CHF when combined with the AR B valsartan.

Sacubitril: It is an orally active NEP inhibitor w hich increases the blood levels of NPs and other vasodilator peptides resulting in natriuresis, diuresis, vascu lar relaxation, lowering of Angll

a we ll as aldosterone levels. Combined w ith valsartan, it afford s symptomatic improvement in Cl IF patients. Sacubitri l is a prodrug that is absorbed after oral ingestion and rapidly activated by esterases to sacubilrilat. The abso lute oral bioavailability is 60%, and food does not alter absorption. Passage of sacubitrilat across blood-brain barrier is limited and it is not significantly metabolized further. Excreted in both urine and faeces, the plasma t½ of sacubitrilat is - 12 hours. No ignificant drug interactions with drugs generally used in CHF have been observed. In a large multicentric randomi7cd clinical trial (PARADIGM-HF, 2014), a 1: 1 combination of sacubitril + valsartan (200 mg BD) was compared with cnalapri l ( IO mg BD) in 8442 patients of class II to class IV CIIF already receiving CHF therapy with one or more of diuretics, 13 blockers and aldosterone antagonist. The sacubitril-valsartan combination caused 20% g reater reduct ion in death or hospitalization for heart failure than enalapril. T he most common side effects "ere hypotension, hyperkalcmia and renal impairment. Angioedema was infrequent, and its tolerability profile was similar to cnalapril.

T hus, sacubitri 1-valsa rtan a ppears to be a more effective and wel l tolerated alternative to ACE inhibitors for reduci ng morbidity and mortality in patients with advanced heart fai lure. However, it is expensive. ENTRESTO, VY'v!ADA 50, 100, 200 mg tabs. (eontam1ng 26 mg or 49 T 51 mg or 97 + I 03 mg -.acubitril 24 + ,al-.:irtan.

Vasoactive intestinal peptide (VIP) First isolated from intestines this 28 amino acid ncuropeptide is broadly expressed in the C S, peripheral cholinergic and peptidergic neurones m the A S, e,pecially those innervating blood vessels, heart, intestines, pancreas, urogenital tract and in some immunological cells. VIP is also a cotransmitter with ACh in many parasympathetic nerves (see Fig. 11.3. p. I 08). It produces a slm,, but long lasting respon,e. Though present in c1rculation, the t½ of VIP in blood is very s hort (- 2 min), and it appears to function mainly as a paracrine transmill~r rather than as a hormone. V I P is a potent vasodilntor; coronaries and most vessels arc dilated; BP falls on i.v. injection of VIP. Cardiac contractil ity and heart rate are increased. Intestinal. biliary, bronchial and genitourinary smooth muscle is relaxed. Secretion of salt and water across intestina l mucosa is increased; overactivation may cause watery diarrhoea. VIP promotes glycogenolysis in liver and appears 10 regulate certain me1abolic processes. A role in modulating immune function has also been ascribed to VIP.

553

554

CARDIOVASCULAR DRUGS Two subtypes, Vl'AC , and VPAC 2 of VIP receptors have been designated. Both are G PCRs and are richly expressed in the CNS, heart, blood vessels, intestines, pancreas and immune cells. These GPCRs are linked to the Gs protein; vasodilatation, cardiac stimulation. smooth muscle relaxation and many other responses arc mediated by adenylyl cyclase activation and generation of cAMP. Some other responses are produced via generation of 11'3 and mobil ization of cytosolic Ca2 • Because of very shon lifespan, VIP as such is not suitable for clinical use. Some stable analogues have been produced and their c linical application in cardiovascular, ne urological, gastrointestinal and inflammatory diseases, as well as in COPD and diabetes is being explored, but none has been found satisfactory yet. VIP antagonists are also being investigated.

Substance P {SP) A substance that contracted intestinal smooth muscle was detected in certain tissue extracts by Von Eu ler and Gaddum in 1931 and named substance P. Later it was found to be a 11 amin o acid neuropeptide distributed widely in the brain, spinal cord, peripheral sensory nerves, enteric nervous system, blood vesse ls and skin. Two other related decapeptides Neurokinin A (NKA) a nd Neurokinin B (NKB) have also been found, and together they constitute the Tachykinin (contrast bradykinin which produces slow contraction) family. All three have similar actions, but SP is the dominant member. SP, N KA and KB are produced by the cleavage of a large polypeptide preprotachykinin and all three serve as neurotransmitter or neuromodulator in the CNS as well as in the periphery, where S P is primarily ass ociate d with pain and inflammation.

Tachykinin receptors: SP a nd othe rs ac t by binding to three subtypes of tac hyk inin receptors NK,, and K3 ; all of which are GPCRs coupled to Gq prote in. T he responses are transduced by generation o f IP3 and mobilization of intracellu lar Ca2 tri ggeri ng smooth muscle contraction, glandular secretion, release of O and other messengers, cell proliferation, production of afferent impulse, etc. The NK, receptor (NK,R) has the highest affinity for SP and serves as its major receptor. Binding of SP to NK,R a lso leads to receptor inte rnali zation

into e ndosomes where SP is degraded and the K 1R is returned to the surface.

Actions and role of substance P: SP is a potent arteriolar di lator; acts on endothe lial K 1 receptors to release O which than relaxes the underlying smooth muscle. Rapid fall in BP occurs when SP is injected i.v. Subcutan eous injection produces red spot, w heal and flare by local vasod ilalation, exudation and axon re flex . Howe ver, veins are often contracted due to direct action on their smooth muscle. Intestinal, bronchial and other smooth muscles are rapidl y and strong ly contracted. ecretion from sali vary g lands and intestina l mucosa is increased. Acting on the kidney. SP promotes natriures is and diuresis. In the brai n, SP and K 1R are particularly localized in the hypothalamus, amygdala, periaqueductal gray, nucle us tractus solitari us (NTS), area postrema, a nd appear to be involved in moo d, be hav iou r, anxiety, stress res po nse, depression, rei nforce ment, nociception, e tc. The c hemorece ptor tri gger zone (CTZ) a nd NTS ex press K 1R a long with 5-HT3 and other receptors which respond to noxious sti muli and coordinate the act of vomiting. It has been fou nd that SP is re leased along w ith other mediators in response to ti ssue injury, especially that due to cancer chemotherapy and radiotherapy. The SP reaches CTZ and TS w hich are not protected by blood-brain ban·ier and participates in triggering nausea a nd vomiting. Accordi ngly, the NK,R antagonists have antiemetic properly. SP is involved in pain perception at vario us levels. It is e laborated by pain nerve endings in response to injury, and can trigger neurogenic inflammation. SP is released from primary pain afferents in the dorsal horn of spinal cord, and together with glutamate serves to tran smit pain impulses. SP is one of the first responders to noxious stimuli. It init iates expression of cytokines which in turn induce SP and K 1R, thus sustaining the reaction and inducing cellular proliferation. As such, SP has a ro le in inflammatory response. Recent ly, it has been found that SP and NK,R, are richl y

NITRIC OXIDE A ND VASOACTIVE PEPTI DE SIGNAL MOLECULES expressed o n certain tumour ce lls, and that SP along w ith cytokines induced by it s ti m ula te growth, proliferation and angiogenesis in these rumours. Thus, K 1R antagon ists may have potential uti Iiry in certa in cancers.

SP (NK,R) antagonists: Reali zi ng the pathophys io log ica l impo rtance of P, several nonpeptide orally active K 1R antagonists have bee n produced and a re being tested as potentia l drugs for anxiety, mood di sorders, depression, inflammatory bowel d isease, a rthritis, eczema and cancer. However, the only drug that has succeeded clin ically so fa r is Aprepitant (a nd its pa re nte ra l prod rug Fosaprepilant) whic h is being used as an adj uvant antiemetic for cancer chemotherapy induced vom iting and for postoperative nausea a nd vomiting (see C h. 48). Calcltonin gene-related peptide (CGRP) Disco, cred -30 years ago in medullary thyroid carcinoma, it is a 37 amino acid ncuropcp1idc produced by alternative processing of the calcitonin gene. CGRP is co-located with calcitonin in the C ce lls of thyroid, but is primarily produced by neurones in the central and peripheral nen·ous system; also in enteric neurones. a ro und blood , essels and in lungs. There arc two (a and ~) forms of CGRP which have similar biological activity, bm are products of separate genes. CGRP acts on an unus ual type of receptor which has two pans. a calci1011i11 rr?ceptor-like receptor (CLR) and a receptor actil'il)' modifying pro1ei11 (RAMP) needed for the receptor lo function. CGRP is a very potent vasodi lator which acts via the adenylyl cyclase cAMP mechanism in the vascular smooth muscle, and not through endothelial 0 release. Injected i.,. CGRP causes fall in BP and tachycardia by dilating aneries, especially coronaries. CGRP is released mainly from the sensory C and Ao fibres (where it is mostly colocated with SP). The periarlerial nerves are rich in CGRP, and it appears to serve both afferent (nociccptivi:) as "ell as efferent (neurogenic inflammation) function. CG RP is also found in the dorsal horn of spinal cord and may be an important link in pain tram,mission. Injected in the brain CGRP produces a variety of effects. Severa l lines of evidences have pointed 10 a role of CGRP in migraine. It appears th at sensory neurones of the trigeminal ganglion re lease CGRP around meningeal vessels- ca use dilatation. p lasma cxtravasation and mas t cell dcg ranulation->neurogenic in n a mmu ti on is triggered and a migraine allack is

precipitated. C rania l CGRP level is increased during migraine, and sumatriplan treatment normali7cs it. CG RP injected i. v. has been found 10 produce headache in individuals susceptible 10 migraine. Some peptide as well as nonpeptide CGRP antagonists have been produced. a nd two agents olcegepant and telcagepalll have been tested clinically in migraine, and found 10 be effective. However, olcegepanl has very low bioavailability, and telcagepant was "ithdrawn after phase Ill trial due to hcpatotoxicity.

Erenumab It is a fully human monoclonal antibody which blocks CG RP receptor, and has been approved by US-FDA (May 20 I 8) for prophylaxis of migraine. Administered as 70 mg once a month s.c. injection through a single- use prefilled auto injector, erenumab (marketed as A IMOVIG inj.) significantly reduced the number of migraine days per month in those suffering 4- 14 migra ine days per month . In some patients the reduction in migraine days was > 50%, and few patients required a double dose ( 140 mg/ month). Erenumab has been well to lerated; injection site reaction was the most common ad, erse effect. A protective role of CG RP against development of hypenension has been proposed. Due to presence in enteric nervous system, CGRP has been implicated in v iscera l painful disorders like irritable bowel syndrome. Adrenomedullin (AM) Adrenomedullin (AM) is a 52-am ino acid peptide belongi ng to the calcitonin family that "as first o btained from human pheochromocytoma. It is now known to be present in normal adrenal medulla, hypothalamus, anterior pituitary, heart, lungs, blood vesse ls, k id neys and intes tines, as well as c irculates in blood. AM produces long- lasting vasod ilatation in majority of , ascular beds resu lting in hypotension and renex tachycardia. Direct cardiac s timulation and central ly mediated increased sympathetic out fl ow a lso occurs. Other actions are natri uresis and renin release. The AM receptor is structurally and functionall y very similar to the CGRP receptor, i.e. it is a GPCR consisting of a C LR pan coupled to a subtype or RAM P. On activation by AM, the receptor interacts with Gs protein to increase cAM P production in target cells. including vascular smooth muscle. and releases O from endothelium. Though the physiologica l significance of AM is st ill not defined. it ha;, been hypothesized 10 be playing a counterba lancing role 10 the RAS and endothel in system. Raised AM levels have been found in systemic and pulmonary hypcnension, CHF. kidney failure and in acute myocardial infarction. where AM may be serving as a protective mechanism. No AM based clinically useful drug has been developed so far.

555

Chapter

38

Cardiac Glycosides and Drugs for Heart Failure

CARDIAC GLYCOSIDES These a re g lycos idi c drugs having cardiac inolropic property. They increase myocardial co ntrac til ity and o utput in a hypod ynamic heart without a proportionate increase in 0 2 co ns umpti on. Though other drugs are now more important in the treatme nt of congestive heart failure (C H F), the cardiac glycosides were the primary drugs ti ll the 1970s, and are still va luable for ma ny patients, when co mb ined wi th others. Wil lia m Wit hering, a Dirmingham physician, tried an extract o f 'foxglove' (Digitalis) and found it to be remarkably effective in ome cases of dropsy (edema). He published his classic monograph in 1785 and ascribed the beneficial effect to an action on the kidney. Cushney and Mackenzie, in lhe beginning of 20th century, established it action on the heart and its use in congestive hea rt failure (CHF).

Cardiac glycosides are found in several plants and in toad skin (Bufotoxin). Digi1alis lanata is the source o r Digoxin, the only glycoside that is currently in use. Others like Digitoxin (from Digilalis p111purea) and Ouabain (from Stmphanthus gratus), etc. are no longer clinically used or marketed. By convention the term, ' Digitalis ' has come to mean 'a card iac g lycoside ' . Chemistry The cardiac glycosides consist of an oglycone (genin) to which are attached one or more sugar (glucose or digiloxose) moieties.

OH

0

3 Sugars-O

L___J

Steroid

Unsaturated lactone

DIGOXIN The aglycone consists of a cyclopcntanoperhydrophenanthrcne (steroid) ring to which is attached a 5 or 6 membered unsaturated lactone ring.

Pharmacological actions All digital is glycosides have qualitatively similar action. Digoxi n is described as prototype.

1. Heart Digox in has direct e fTccts on myoca rdia l contracti Iity and e lectro ph ys io logic a l properties. In ad dition, it has vagomime ti c action, reflex effects due to a lteration in haemodynamics and direct CNS effects altering sympathetic activ ity.

Force of contraction

Digoxin causes a dose depe ndent increase in fo rce of co ntrac tion of heart-a posi tive inotropic action. This is especiall y seen in the fai ling hea rt w hich is exqu is itel y se nsitive. Systole is shortened , diastole is prolonged. When a normal heart is subjected to increased impedance Lo outflow, it generates increased tens ion so that stroke volume is maintained upto considerably higher values of

557

CARDIAC GLYCOSIDES AND DRUGS FOR HEART FAILURE impedance (Fig. 38. 1), while the fai ling hean is not able to do so and the stroke volume progressively decreases. The digita lized fai ling heart regains some of its capacity to contract more forcefu ll y when su bjected to increased resistance to ejection. There is more complete empty ing o f fa iling a nd dilate d ventri c lescard iac o utput is increased a nd end-d iasto lic volume is reduced.

Rate Heart rare is decreased by di gox in. Bradycardia is more marked in C H F patients because improved c irc ulati o n (due to pos itive inotropic actio n) restores the diminished vagal tone and abolishes sympathet ic overactivity. In addition, digitalis slows the heart by vaga l (increase in vaga l tone) a nd extravagal (d irect depression of SA and AV nodes) act ions.

+20 0 -20

>E

-40 -60 -80 -100

- - - - - r - - ~- - - r - - - - r- - - i r - -

0

100

200

300

400

m Sec

Fig. 38.2: Effect of digoxin on Purkinje fibre action potential

Electrophysiological properties (a) Action potential (A P ): The effects a rc illustrated diagrammatically in Fig. 38.2. • T he resting membrane potential (RMP) is progressively decreased (to less negative values) wi th increasing doses. • The rate of O phase depolari7ation is reduced resulting in slowing of conduction. TI1is action is most marked in A- V node and bundle of ll is. • The slope of phasc-4 depolari.a11ion is increased in the PFs-ectopic automaticity is enhanced-latent pacemakers become overt al high doses producing extrasystoles. High doses of digoxin produce coupled beats by anotl1er mechanism: the Rl\llP shows oscillations during phase-4: when their magnitude is sufficient enough, delayed afler-depolari?ations result (see Fig. 39.1 ). The SA and A-V node automatieity is reduced at therapeutic

Untreated fibre



concentrations by vagal action which hyperpolari.1:es these cells and reduces their phase-4 slope. The action potential duration (APD) is reduced (primarily at phase-2) and amplitude of A P is diminished.

(b) Effective refractory period (ERP):

.

Atruon

decreased by) Vagal .a~tion normal ly . predominates, cau es vaga I ac t ,on . . mhomogcne1ty: allows . the atria to respond at increased by a higher rate and in an direct action asynchronous manner.




E

::,

g

II> .:,c

g

en

Arterial impedance - - + Fig. 38.1 : Relationship between peripheral resistance and stroke output in normal and failing heart, and the action of digoxin on failing heart

(c) Conduction: A-V cond uction is de m o nstra bly s lowed by therapeutic doses. At high doses, intraventricular conduction in PFs is a lso depressed by uncoupling of gap junctions. (d) ECG : Therapeutic doses of d igox in produce cha nges in th e ECG. These arc accentua ted at high dos es which ma y also produce arrhythmias. • Decreased amplitude or inversion of T wave. • Increased P-R interval (due to s low ing of A-V conductio n), A -V block at toxi c doses.

558

CARDIOVASCU LAR DRUGS • Shortening of Q -T interval (reflecti ng shortening of systole). • Depression of ST segment (at hig h doses-due to interference w ith repo larization).

Mechanism of action

Digita lis increases force o f cardiac contraction by a direct action independent of innervation. It selectively binds to extracellular face of the me mbrane associated a' K+ ATPase of myocardial fibres and inhibitis this enzyme (Fig. 38.3). Inhibitio n of this cation pump results in progress ive accu mulation of a intracellula rly. This indirectly resul ts in intracellul ar Ca2 accumulati on d ue to dim inis hed Ca2- extrusion. During de polariza tion Ca2 ' ions enter the cell dri ven by the s teep Ca2+ gradient (> I mM extracellular to < I00 nM cytosolic during diasto le) through voltage sensiti ve L type Ca2+ channe ls. T his triggers re lease of larger amount of Ca2stored in sarcoplasmic reticulum (S R) through a calcium-acti vated calcium channel known as

Ryanodine receptor calcium channel 2 (RYR2) • cytosolic Ca2 ' in creases transiently to about 500 nM (calcium trans ients) triggering contraction by activati ng tropo nin C on myofibril s. The sarcoplasmic-endoplasmic reticular Cal. ATPase 2 (SERCA2) is then activated which pumps Ca2+ back into t he SR, where it is stored loosely bound to calsequestrin (Ca lS). In the normal state SERCA remains inhibited by a cytosolic protein phospho lamba n ( PL N). Phosphory la ti on of P LN by PKA removes this inhibi tion and SERCA becomes active. A fraction of Cai+ (equal to that w hi ch entered from o utside during depolarization) is extruded mainl y by 3 a+/1 Ca2+ exc hange trans po rter (NCX-anriporrer) a nd to a lesser ex tent by sarcolemmal Ca2' pump (Ca2+ ATPase). During phase 3 of AP, membrane a+K-ATPase moves 3 intrace ll ular a+ ions for 2 extracellular K' ions. The s light ( 1- 1. 5 mM) inc rease in eytosolie a+ over normal (8- 10 mM) due to

(1mM)

ca2•

L-type Ca2 +channel

,, ., +

I

3Na K (10 mM) (150 mM ) +

'.1

SR

CONTRACTION Fig. 38.3: Mechanism of positive inotropic action of digoxin

SR-Sarcoplasmic reticulum; TnC-Troponin C; NCX-Na·-Ca2• exchanger; RyR2-Ryanodine receptor calcium channel 2; PLN-Phospholamban; SERCA2-Sarcoplasmic-endoplasmic reticulum calcium ATPase 2, CalS-Calsequestrin.

559

CARDIAC GLYCOSIDES AND DRUGS FOR HEART FAILURE partial inhibition o f a+K+ATPase by digoxin reduces transmembrane gradient of Na' wh ich drives the extrnsion of Ca2 • The excess Ca 2+ remaining in cytosol is taken up into SR which progressive ly get loaded with more Ca2+ so that subseque nt calcium transients a re augmented. The relationship of cytosolic [Na ' ] and [Ca'·] is such that a small perecmagc increase in a concentration leads to a large percentage increase in Ca'- concentration. Moreover, raised cytosolic Ca'' induces greater .:ntry of Ca'- through voltage sensitive Ca1 • channels during the plateau p hase. It has been shown that I mM rise in cytosolic [Na ) results in 20-30% increase in the tension developed by , entricular fibres.

Binding of glycoside to a•K ATPasc is slow. Moreover, after Na+K+ATPase in hibition, Ca 2 + loading occurs g radually. As such , ino tropic effect of dig italis takes ho urs to develop, e ven after i. v. administratio n. Inh ibition of Na+K• ATPase is clearly invol ved in the toxic acti ons of dig italis. At high doses. the re is depletion o f intracellular K+; an d digitalis toxic ity is partially reversed by infusing K +, becau se K' decreases binding of glycos ide to a•K· ATPa se. Excessive Ca 2• loading o f SR results in s pontaneous cycles of Ca2• re lease and uptake producing oscillatory delayed after-depolari za tions and aftercontractions. Since both therapeutic and toxic effects o f digoxin are due to myocardial Ca2• loading, these are insepara ble and therapeutic index of digoxin is low.

2. Blood vessels

Digoxin has weak direct vasoconstrictor action- peripheral resistance is increased in norma l individuals. However, in CH F patients this is more than compensated by the indirect effect o f improvement in c irculation, i.e. reflex sympathetic overactivity is withdrawn and a net decrease in periphe ral resistance occurs. Digox in has no promine nt effect on BP: systolic BP may inc rease a nd diasto lic may fall in C HF patients- pulse pressure increases. Hypertension is no contraind ication to the use of digox in. T herapeutic doses of digoxin have no significant effect on coronary c irculati on-coronary insuffi.ciency is no contraindication to its use.

3. Kidney

Digoxin pro mptly causes diuresis in CHF patients, secondary to improvement in circulation and re nal perfusion. The retained salt and water is g raduall y excreted. o diures is occurs in normal ind iv idua ls or in patients with edema due to other causes.

4. CN S Digoxin has little apparent C S effect in therapeutic dose. Higher doses cause CTZ activatio n induc ing nausea and vomiting. Still higher doses produce hyperapnoea, central sympatheti c stimulation. menta l confusion, disorientation and visual disturbances. Effects on the g. i.t. are o nly of toxicological nature (see adverse effects).

Pharmacokinetics The pharmacokine tic features of digoxin are listed in Table 38. 1. Bioavailability of digox in tablets from diffe rent manufacturers may di ffcr. Presence o f food in stomach de lays absorption of digox in. The vol ume of d istribution is large (6- 8 L/ Kg). It is concentrated in the heart (- 20 times than plasma), skeletal muscle. liver and kidney.

Table 38.1: Pharmacok1net1c features of d1goxin DIGOX/N

1.

Oral absorption

60-80%

2.

Plasma protein binding

25%

3.

Time course of action· -Onset

15-30 min

- Peak

2-5 hr

-Duration

2-6 days

4.

Plasma 11'2

40 hr

5.

Therapeutic concn .

0 .5-1.4 ng/ml

6.

Toxic concn.

> 2 ng/ml

7.

Daily maintenance dose

0 .125-0.5 mg

8.

Daily elimination ..

35%

9.

Route of elimination (predominant)

Renal excretion

Route of administration

Oral, i.v.

10.

• Of full digitalizing dose given i.v.; ·• fraction of total amount present in the body.

560

CARDIOVASCULAR DRUGS D igoxin is primarily excreted unchanged by th e kidn ey: ma inly by glome ru la r filtra tion ; rate of excreti on is altered paralle l to creatinine clearance. Its t½ is prolo nged in elderly patients and in those with renal insufficiency: dose has to be reduced. Digoxin is a cumulative drug. When maintenance doses are g iven from the beg inn ing, steady state leve ls and full therapeutic effect are atta ined a fter 4 x t½, i.e. 6-7 days. Dig mcin: DIGOXIN 0.25 mg tab., 0.05 mg/ ml pediatric elixir, 0.5 mg/ 2 ml inj. LANOXIN, SANGOXfN 0.25 mg tab, CARDIOXJN 0.25 mg tab, 0.5 rng/ 2 ml inj .

Adverse effects Toxicity of di gitalis is high, margin of safety is low (therapeutic index 1.5- 3). A bout 25% patients receivi ng digoxin therapy develop one or other tox ic symptom. The mani festations are:

Extracardiac Anorexia, nausea, vomiting and abdo minal pain are usually repo rted fi rst: are due to gastric irritation, mesenteric vasoconstriction and CT Z stimulation. Diarrhoea can also occur. Fatig ue, ma laise, headache, mental confusion, restlessness, hyperapnoea, djsorientation, psychosis and visual disturbances are the other complaints. Skin rashes and gynaecomastia are rare. Cardiac A lmost every type of arrhythmia can be produced by digoxin: pulsus bigeminus is common; others are nodal and ventri cular extrasystoles, ventricular tachycard ia a nd terminally ventricular fibrillation. Partial to complete A-V block may be the sole cardiac toxicity, o r it may accompany other arrhythmias. Severe bradycardia, atrial extrasysto les, AF or AFI have also been noted. In abo ut 2/3 patients showing toxic ity, extracardiac symptoms precede cardiac tox icity.

Treatment Further doses of digoxin must be stopped at the earliest sign o f toxicity; nothing more needs to be done in many patients, espec ially if the mani festations are only extracardiac.

glycosides to a+K+ATPase by favouring a conformatio n of the enzyme that has lo wer a ffi nity for the glycoside, and K+ tends to antagonize digitalis induced enhanced a utomaticity. When toxicity is due to acute ingestion of large doses o f digox in, plasma K+ may be high; it should not be given from o utside . In any case, it is desi ra ble to meas ure serum K· to guide K Cl the rapy. K+ is contraindicated if higher degree of A-V block is present, because complete A-V block and ventricular asystole may be precipitated.

(b} For ventricular arrhy thmias

Lidocaine i. v. repeated as required is the drug of cho ice. It suppresses the excess ive automaticity, but does not accentuate A-V block. Quinidine, procainamide and pro pafenone are contraindicated.

(c) For supraventricular arrhythmias Propranolol may be given i. v. or o ra lly depending on the urgency.

(d) For A-V block and bradycardia

T he treatme nt o f c hoice is cardiac pac ing throug h a temporarily inserted pacema ker. Atro pine 0.6- 1.2 mg i.m . may help. Cardioversion by D C shock is contra indicated because severe co nduc tio n defects may be unmasked in the digitalis intoxicated heart. Digoxln antibody Digoxin specific antibody is highly effective in reversing cardiotoxicity due to digoxin as well as many other related glycosidcs. The Fab fragme nt has been marketed in Europe as DIG IBIND (38 mg vial), D IG JFAB (40 mg/vial). fl is nonimmunogenic because it lacks the Fe fragme nt. Given by i.v. infusion it has markedly improved the survival o f seriously digitalis intoxicated patients. Precautions and contraindications





• •

(a) For tachy arrhy thmias

Wh e n caused by chro nic use of digoxin and diuretics (both induce K+ depletion)--infuse KCI 20 m.mol/hour (max. 100 m.mol) i.v. o r give orally in milder cases. High extracellular K. decreases binding of the



Hypokalemia: enhances digitalis toxicity. Elderly, renal or severe hepatic d frease: patients are more susceptible lo d igoxin toxicity. Thyrotoxicosis : patients arc more prone to deve lop digitalis arrhythmias, while digox in e limination is s lowed in hypothyroid patients. Venlricular /achycardia: digita lis is contraindicate d beca use it may prec ipitate ventric ular fibrillation. Panial A-V block: may be converted to complete A- V block by digoxin. Wolff-Parkinson- While syndrome: D igoxin is contraindicated because it decreases the EllP o f bypass tract in 1/3 pat ie nts. In t he m ra pid atrial impulses V F may occur. may be transmitted to ventricles Digoxin can increase the chances o f reentry by s lowing

CARDIAC GLYCOSIDES AND DRUGS FOR HEART FAILURE conduction in the nonnal A-V bundle and accelerating it in the aberrant pathway.

Interactions I. Di11re1ics: cause hypokalemia which increases the risk of digitalis arrhythmias; potassium supplements should be given prophylactically. 2. Calcium: syncrgises with digitalis • precipitates toxicity. 3. Quinidine: reduces binding of digoxin to tissue proteins as well as its renal and biliary clearance by inhibiting elllux transporter P-glycoprotcin • plasma concentration o f digoxin is doubled • toxicity can occur. Verapamil. diltiazem, cap1opril, propafenone and amiodarone also increase plasma concentration of digoxin to variable extents. 4. Adrenergic drngs: can induce arrhythmias in digitalized patients; both increase ectopic automatic ity. 5. Digoxin absorption may be reduced by me1oclopramide, sucra/fa1e, an/acids, neomycin, sulfasalazine. Absorption of digoxin is increased by atropinic drugs, including tricyclic antidepressants. 6. Propranolol, verapamtl, diiliazem and disopyramide: may additively depress A-V conduction and oppose positive inotropic action.

Uses The two main indications of digox in are heart failure and control of ventricular rate in atrial fibrillation/flutter. Its use in heart fa il ure is described in the next section on treatment of heart failure.

Cardiac arrhythmias

Atrial fibrillation (AF), atrial flutter (AF/) Digoxin can be used for controlling ventricu lar rate in AF and AFl, whether associated with CHF or not, but it is incapable of curing the arrhythmia, i.e. does not revert it to s inus rhythm. Often, AFI may be converted into AF because di gox in abbreviates atria l ERP and makes it more inhomogeneous. Digoxin reduces ventricular rate in AF and AFI by decreasing the number of impu lses that are able to pass down the A-V node and bundle of His. It increases ERP of A-V node by direct, vagomimetic and antiadrenergic actions: the minimum interval between consecuti ve impulses that can successfully traverse the conducting tissue is prolonged.

Digoxin may be used for long-tem1 ventricular rate control in persistent or pem,anent AF/AF!, but is not the preferred drug for rapid control of heart rate. Oral or i.v. blocker (e.g. metoprolol) or a non-DHP calcium channel blocker (CCB) diltiazem/verapamil given orally or i.v. (depending on the urgency) are the preferred dntgs. The short-acting i.v. blocker esmolol is preferred for haemodynamically unstable patients. Digoxin is not only slow to act (takes 1-6 hours even after i.v. injection), but also less effective. However, it is preferred when AF/AF! is attended by heart fai lure. Digoxin decreases ventricular rate in a dose dependent manner and abolishes the pulse deficit in AF. The maintenance dose in persistent or permanent AF/AFI can be determined by titrating with heart rate, which 60 per should not be allowed to decrease to min at rest. Digoxin can also be combined with the blocker/CCB, if a single drug fails to lower the heart rate to the desired levels. Paroxysmal supraventricular tachycardia (PSVT) It is a common arrhythmia with a rate 150- 200/min and I : I A-V conduction. It is mostly due to reentry involving the A-V node. Digoxin may be injected i.v. It increases vagal tone and depresses the path through the A-V node, or the ectopic focus, and terminates the arrhythmia (success in 1/3 cases). Adenosine, verapamil or esmolol injected i. v. are more effecti ve, less toxic and act faster, therefore preferred.

TREATMENT OF HEART FAILURE Heart fai lure occ urs when cardiac output is insufficient to meet the demands of tissue perfusion, or is able to do so only by elevating filli ng pressure. It is a progressive disease with graduall y deteriorating cardiac performance. Heart fai lure may primarily be due to systol ic dys function or diastolic dysfunction.

Systolic dysfunction This refers to deficient pumping action of ventricles, i.e. ventricles are unable to develop sufficient wall tension to eject adequate quantity of blood and get progressively dilated. This makes the ventricle further inefficient according to the Laplace equation: Wall tension = lntraventricular pressure x ventricular radius

561

562

CARD IOVASCU LAR DRUGS

i.e. Lo generate the same ejection pressure a dilated ven tricle has to develop higher wall tension. Thus. a self-perpetuating vicious cycle is set up and ejection fraction (EF) is progressively reduced (usuall y to 20 mm [ lg. It decreases cardiac pre load. (b) Vasodifators: Ve nodilato r (GTN s lo w i. v. infus ion) is mainly used. It m ay be supplemented by i.v. csmo lo l or labeto lo l. (c) Jnotropic agents: dopamine or dobuta mine i.v. infusion may be needed to a ug m ent the pumping action of heart and tide over the cris is. 7. Prevention of thrombus extension, embolism, venous thrombosis A spirin ( 162325 m g) s ho uld be g iven fo r c hew ing and swallowing as soon as M I is s us pected (if not a lready bei ng taken on a regular bas is). This is continued at 80- 160 mg/day. C lopidogrel/ pras ug rc l/t icagrelo r may be g iven as a lternative add-on antiplatc let drugs. Anticoagulants (hepa rin/ fo nda parinux fo llowed by oral anti coagulants) are used primarily to prevent deep vein thrombos is (risk is in creased due to bed rest) and pulmonary/systemic arterial em bolism . The value of anticoagu lants in checking corona ry arte ry thrombus ex te nsion is u ncertai n . An y be ne fit is sho rt-term ; anticoagul a nts are not prescribed o n lo ng -term bas is now (see Ch. 45).

8 . Thrombolysis and reperfusion Fibrino lytic agents, i.e. plasm inogen activato rs- altepl ase/ tenecteplase to achieve reperfus ion o f rhe infarcted area (see C h. 45) are indicated only in STEMI o r high risk STEM !. Unless thrombolysis can be s tarted withi n 1- 2 hours o f M l sy mptom o nset, primary percutaneous coronaty intervention (PC I) with stenti ng is now the preferred revascularization procedure, wherever available. 9 . Prevention of remodeling and subsequent CHF A C E inhibito rs/ARBs are o f proven efficacy and affo rd lo ng-term surv iva l benefit (see C h. 36). T hey s hould be started as soon as the patient is hacmodynam ically stable.

I O. Prevention of future attacks (a) Plate let inhibito rs aspirin or cl o pidogrel o r s imilar drug g iven o n lo ng- term bas is is routinely prescribed (see Ch. 45). (b) blockers- These drugs red uce risk o f re in fa rctio n, C HF and m ortality. A ll pati e nts not hav ing any contraindicatio n are put on a blocker for at least 2 years. s uita ble (c) Contro l of hyperlipidaemia-dietary s ubstitu tion wi th unsa tu rated fa ts, hy pol ipi demi c drugs especially statins are routinely prescribed, irrespective of bl ood cho lesterol level. T hey have bee n s ho wn to reduce ris k o f cardi ovasc ular events (see C h. 46).

PROBLEM DIRECTED STUDY 40.1 A 55-year-old man presented with complaints of tightness and discomfort over m iddle part of chest fe lt episodically, particularly after walking briskly o r climb ing stairs or d uring sex. This is relieved within 5-10 minutes of rest. One or two episodes occur practically every day. He is a past smoker who quit smoking 5 years back when he was diagnosed to have chronic obstructive pulmonary disease {COPD), for which he regularly takes 2 inhalations of lpratropium Br. 3 times a day and 2 puffs of salbutamol inhalation whenever he fee ls out of breath. The pulse was 90/min and BP 124/82 mm Hg. The resting ECG was normal, but stress test was positive. A diagnosis of exertio na l angina was made and he was p rescribed-Tab glyceryl trin itrat e 0.5 mg to be put under the tongue as soon a s he begins to feel the chest discomfort, as well as before undertaking any phys ica l exertion. Should he be p rescribed another drug to be taken on a regular basis to prevent (a) episodes of angina? If so, which drugs can be given to him and which cannot be given? (b ) Should add itiona l med ication be given to prevent long-term complications a nd improve survival? (see Appendix-! for solution)

603

Chapter

41

Antihypertensive Drugs

These are drugs used to lower BP in hypertension. Hype rtens ion (HT) is a very common diso rder, particularly past middle age. It is not a disease in itself, but is an important ri sk factor for cardiovasc ular mortality and morbidity. The cutoffmanometric reading between normotensives and hype rtensives is arbitrary. For practical purposes ' hyperte ns ion' could be that level of BP at or above whic h long-term antihypertensive treatment wil l reduce cardiova cular mortality. A lmost a ll HT management guidelines inc luding N ICE (20 11 ). JNC8 (20 14), WHO-IS H (2003). European Society of llypertension (2007, 2013) defi ne the cut-off level to be 140 mm I lg systo lic and 90 mm Hg diastol ic. However, the JNC8 have raised the defining level to 150/90 mm Hg fo r indi viduals above 60 years of age. Epidemiological studie s have confirm ed that higher the pressure (systolic or diastolic or both) greater is the risk of cardiovascular disease. Majority of cases are of essential (primary) hypertension, i.e. the cause is not known. Sym patheti c and re nin-angioten si n systems ( RAS) may or may not be ove ractive, bu t they do contribute to the tone of blood vessels an d c.o. in hypertens ives. as they do in nom1otensives. Many antihypertensive drugs interfere with these

regulatory systems at one level or the other. Antihypertensive drugs, by chro nically lowering BP, may reset the barosta t to functi on at a lower level of BP. A ntihy perten sive d rug t he rapy has bee n n:markably improved in the last 60 years. DifTercnt classes o r drugs ha,e received prominence with passage of time in this period. Before 1950 hardly any effective and tolerated antihypertensive was avai lable. Veratrum and Sod. 1hio9•unale could lower 13P. but were toxic and difficult to use. The ganglion blockers developed in the I 950s were efTecti\t:. but produced a variety o f side effects. Reserpine was a breakthrough. but produced menta l depression. T he therapeutic potemial o f i,ydralazine could not be tapped fully because o f marked side effects when it was used alone. G11ane1h1dine introduced in 1961 was an improvement on ganglion blockers. The aatihypertensives blockers, rhiazide o f the 1960 70s were methy ldopa. nnd high ceiling diuretics and cfonidine. The s tatus of 13 blockers and d iuretics was consolidated in the 1970s and select ive a, blocker pra=osin broke new g round . The ant ihypcrtensives introduced in the 1980- 90s were angiotensi n 11 convert ing enzyme (ACE) inhibilors and calcium channel blockers. Angiotensin receptor blockers ((osartan. etc.) were added soon after, and the d ircc1 renin inhibitor ofiskire11 is the latest drug. With the development of many types o f drugs, dclinea1ion o f 1heir long-term benefits and complications. and understanding of the principles on w hic h to combine them, hypertens ion can now be controlled in mos t cases wi th minimum discom fort . Evidence-based guidelines for selection o f antihypertcnsive drugs for different ca1cgories of patients have no\\ been developed.

JNC 7-The seventh report of JoinL National Committee (of USA) on prevention. detection. evaluation and treatment of high blood pressure. • J C 8- Evidence-based guide line for the management o f hgih blood pressure in adu lts; report from the panel members appointed to the J C-8 (Jo int ational Commiuee (USA). IC E (201 1}--National Institute for Health and Care Excellence (UK): Hypertension in adults: diagnosis and managemenL (CG 127). WHO-IS i I- World Health Organization and International Society o f Hy pertension.

605

ANTIHYPERTENSIVE DRUGS

ANTIHVPERTENSIVE DRUGS (1)

Aldo. antag.

Hydrochlorothia,:idc Chlortha lidonc lndapamide

ACE inhibitors

Captopril Enalapril Lisinopril Perindopril Ramipril Fosinopril Quinapril Trand olapril

Furosemide (others)

Spironolacton Eplcrcnone

Angiotensin (AT1) receptor blockers

Losartan Candesartan Valsartan Telmi~artan lrbesartan Olrnesartan

Direct renin Inhibitor

Aliskiren

P-adrenergic blockers

Proprano lol Mctoprolol Atenolol (otheri.)

O'.-adrenerglc blockers

Pra,:o in Terazosin Doxazosin Phcntolaminc

a+P adrenergic blockers

Labeta lo l Carvedilol

Central sympatholytics

Clonidine Methyl dopa

ANTIHVPERTENSIVE DRUGS (2)

Phenylalkylamine

Veraparnil

Benzothiazepine

Dihydropyridines

Arteriolar dilator

Diltiazcm

ifed ipine Felodipinc Amlodipine Ci lnidipine Lacidipine Lercanidipine Benidipine Nicardipine

Hydralazine Minoxidil

DIURETICS Diuretics have been the standard a ntihyperte nsive drugs over the past nea rly 5 decades, though they do not lower BP in normotensives. Their pharmacology is desc ribed in C h. 42.

Thiazides Hydroc hlorothia zide (HCZ) a nd chlorrhalidone are the d iuretic of c hoice for uncompl icated hy pertension; have similar efficac y and are dose to dose equ ivalent. Al l

itroprussidc sod.

mcga trial s have been carried o ut with these two o nl y. C hlorth a lidone is lo nger acting (~ 48 hours) than HCZ (< 24 hou rs) and may have better round-the-clock action; favoured by the ICE g uidel ines. lndapamide (see later) is a lso ma inly used as antihypcrtensive, a nd is equall y effecti ve. Other membe rs of the th iazide class sho uld not be considered interchangea ble w ith these as a ntihypertensive. The proposed mechanism of antihy pertensivc action is:

606

CARDIOVASCULAR DRUGS I. Initially, the diures is reduces plasma and e.c.f. volume by 5-15%, and this decreases c.o. 2 . Subsequently, co mpensatory m echani s ms operate to almost regain a+ balance and plasma volume; c.o. is restored, but the fall in BP is maintained by a slowly developing reduction in t.p.r. 3. The reduction in t.p.r. is most probably a n ind irect consequence of a sma ll (- 5%) persisti ng Na+ and volume deficit. Decrease in intracellu lar a+ concentration in th e vascu lar smooth muscle may reduce tiffness of vessel wal l, increase their compliance and dampen responsiveness to constr ictor stimuli (NA, Ang 11). Similar effects are produced by salt restri ction; antihypertensive action or di re ties is lost when salt intake is high. A mild slowly developing vasodi lator action or thiazides due to opening of smooth muscle K• ATP c hannels and hyperpolarization has been proposed, but does not appear to be real. The fall in BP develops gradually over 2-4 weeks. During long-term treatment with thiazides, the heart rate and c.o. remain unarfected , while t.p.r. is reduced despite compensatory increase in plasma renin acti vity, which confirms persisti ng Na+ deficit. Thiazides have no effect on ca pacitance vesse ls, sympathetic reflexe are not impaired: postural hypotension is rare. Thiazides are mild a ntihypertensives, average fa ll in mean arte rial pressure is - IO mm Hg. They are effective by themselves in only - 30% cases (mostly low grade hypertension ) but they potentiate all other antihypertensives (excep t DHPs) a nd prevent development or tolerance to these drugs by not allowing expansion of plasma volume. T hus, in combination, t hey a re useful in any grade of hypertension. They are more effective in the e lderly. tfaxima l antihypcrtcnsive efficacy is reached at 25 mg/day HCZ, though higher doses produce greater diuresis. The ir antihypertens ivc action is attenuated by SAIDs.

High ceiling diuretics

Furosem ide is a strong diuretic, but a weaker antihypertensive than thiazides. The fall in BP is entire ly dependent on reduction in plasma volume and c.o. The explanation to thi s paradox may lie in its brief duration of action. The natriuretic acti on lasting only 4-6 hr after the conventi onal morning dose is fol lowed by compensatory increase in proximal tubular reabsorption of a· . The a- deficient state in vascular smooth muscle may not be maintained round-the-clock. The t.p.r. and vascular respon iveness are not reduced. Moreover, the high ceiling diuretics a re more liable to cause flu id and electrolyte imbalance, weakness and other s ide effects. They are indicated in hypertension only when it is complicated by chronic renal failure or coexisting refractory C HF, or when fluid retai ning potent vasod ilators are used. Desirable properties of thiazide diuretics as antihypertensives are:

1. Once a day dosing and flat dose-response curve permitting simple standardized regimens. 2. o fluid retention, no tolerance. 3. Low incide nce of postural hypotension and relative freedom from side effects, especially C S, compared to sympatholytics. 4 . More effective in elderly patients and in those with isolated systolic hype rtension ( ISH). 5. Thiazides decrease Ca2+ excretion; may lower risk of osteoporosis in older women. 6. Low cost. Current status of diuretics as antihypertensives

In the 1960- 70s thiazide diuretics were almost routinely prescri bed alo ne or in combination, to nearly a ll hype11ensive patients. The usual dose used was HCZ/chlorthalidonc 50 mg/day. Soon a number of drawbacks were highligh ted: • Hypok a laemia- muscle pain, fa ti gue and loss of energy. • Erectile dysfunction in males. • Carbohydrate intolerance, precipita tion of diabetes due to inhibition or insulin release (probably secondary to K• depletion).

ANTIHYPERTENSIVE DR UGS • Dyslipidemia: rise in total and LDL cholesterol and triglycerides with lowering of HDL which cou ld increase atherogenic risk. • Hyperuricaemia: by inhibiting urate excretion- increased incidence of gout. • Increased incidence of sudden cardiac death: attributed LO episodes of torsades de pointes and ischaemic ventricula r fibrillation precipitated by hypokalaemia. Consequently, prescribing of diuretics decreased. Subsequently, several d ose- ranging s tudies and intervcntional trials demonstrated that the adverse consequences of thiazide use were dose-dependent, and that 25 mg/day HCZ dose yielded the best benefit-risk ratio. Favourable outcomes obtained at '.:,25 mg/day HCZ in studies, including ALLHAT (2002) and a meta-ana lysis (2003) have reinstated thiazide diuretics as the first choice antihypertensive. Findings with low dose ( 12.5- 25 mg/ day) thiazide therapy are: • Though serum K+ falls a little, significant hypokalaemia does not occur: K + sparing diuretics are usuall y not needed. • Continuou ECG recording studies failed to document increased incidence of arrhythmias. • Impairment of glucose tolerance or increase in serum cholesterol or hyperuricaemia over long-tenn are minimal. Benefits of low-dose thiazides outweigh its potential to worsen diabetes, and are not contraindicated in diabetics. • A nalysis of recent trial has found thiazides to reduce fatal and nonfatal Ml by 27-44%. The incidence of stroke is reduced by 31-49%. Overall mortality and morbidity is reduced in long-term tria ls. • Some recent trials in mild to moderate hypertension have found thiazides to reduce left ventricu lar hypertrophy. The JNC 8, NICE (20 11 ) and other guidelines recomme nd instituting 12.5 25 mg/day thiazide therapy, with or without added K· sparing diuretic, as a first choice treatment of essential hypertension, especially in the elderly. If the low dose (25 mg/day) fails to reduce BP to desired level,

another antihypertensive should be added, rather than increasing dose of the diuretic. Thiazides are ineffective in patients with chronic kidney d isease (C KD), and are not recom mended. High-ceiling diuretes are effective in patients with C ler 30 m g/ml or less, and may be used in place of HCZ.

lndapamide

It is a mild diuretic, chemically related to chlorthalidone; reduces BP at doses which cause little diuresis. Electrolyte di sturbances, K· loss and metabolic effects are minimal at antihyper1ensive doses. The ICE guidelines (20 11 ) favour ch lorthalidone and indapamide over hydrochlorthiazide. lndapamide is well absorbed orally, has an elimination t½ of 16 hr. It is well tolerated: side effects are minor g.i. symptoms and fatigue. Hypokalaemia is infrequent. LORVAS. ATRJLIX 2.5mgtab. '\ATRILIX-SR 1.SmgSR tab

Aldosterone antagonists Spironolactone and eplerenone themselves lower BP slightly. Used in conjunction with a thia z ide diuretic they prevent K+ loss and augment the antihypertensive action. Spironolactone has hormonal side effects (gynaecomastia, impotence, menstrual irregularities). This problem is offset in the newer aldostcronc antagon ist cplerenone, which is increas ingly used. With the realization of the role of aldosterone in promoting hypertension related ventricular and vascular hypertrophy and renal fibrosis, it is considered that a ldosterone antagoni sts wi ll attenuate these complications. As such, there is resurgence in their use, especially in refractory hypertens ion. The ICE and J C 8 guidelines recommend addi ng an aldosterone antagonist to the ACE inhibitor/A RB + CCB + thiazide regimen, if the target level BP is not attained. I Iyperkalemia should be watched when aldosterone antagon ists arc used, particularly in combination w ith ACE

inhibitors/ARBs.

ANGIOTENSIN CONVERTING ENZYME (ACE) INHIBITORS The ACE inhibitors are one of the first c hoice drugs in all grades of essentia l as well as

607

608

CARDIOVASCU LAR DRUGS renovascular hypertension (except those w ith bilateral renal artery stenosis). Most patients require relatively lower doses (ena lapril 2.5-10 mg/day or equi valent) which are well tolerated. Used alone they control hypertension in ~50% patients, and addition of a diuretic/ ~ blocker extends efficacy to ~90%. Because o f supraadditi ve synergism, onl y a low dose o f diuretic ( 12.5 mg of HCZ, ra re ly 25 mg) needs to be added . The pha rmacology and use of ACE inhibitors in hypertens ion are desc ribed in C h. 36. Of parti cular ment io n is their renal blood flow improvi ng action, their potential to retard diabetic nephropathy and their capacity to regress left ventricula r/vascular hypertrophy. They are the most appropriate antihypertensives in patients w ith diabetes, nephropathy (even nondi abetic CKD), le ft ventricular hypertrophy, C HF, a ngina and post M l cases. Several large prospective studies including AI R E ( 1993), HOPE (2000), ALLHAT (2002) have confirmed the antihypertens ive and ca rdioprotective e ffects of ACE inhibi tors. They appear to be more e lTective in younger (< 55 year) hypertensives tha n in the e lderly, as well as in while races than in blacks. Dry persistent cough is the mos t common side effect requiri ng discontinuation of ACE inhib itors.

ANGIOTENSIN RECEPTOR BLOCKERS T he pharmacology of losartan and other ARBs is described on p. 535-37. In a dose of 50 mg/day losarta n is an effective antihypertensive. Action man ifests early and progresses lo peak at 2-4 weeks. Addition of 12.5 mg/day HCZ further en hances the fa ll in BP. The newer ARBsva lsarta n, candcsartan, irbesartan and te lm isartan have been shown to be as e ffective antihypertensives as ACE inhibitors, while losartan may be somewhat weaker than high doses of ACE inhibito rs. ARBs arc remarkab ly free o f side effects. Beca use they do no t increase ki nin levels, the ACE inhibitor related cough is not encountered. Ang ioedema, urticari a and taste disturbance are a lso rare. Though effects of ACE inhibi tors and ARBs are not identical.

the latter have all the metabolic and prognostic advantages of ACE inhibitors. Several in1ervcntional endpoinl reduction trials like LI FE (2002). VALUE (outcomes in hypertensive patients with valsan an or amlodipinc, 2004), SCOPE (study on cognition and prognosis in the elderly; stroke prevention with candesartan in elderly with isolated systolic hypertension, 2004), J LIG HT (Japanese losanan therapy intended for globa l renal protection in hypertensive patients, 2004) have auested to the favourable effects of ARBs on morbidity and mortality in hypertensive patienls.

As antihypertensivc, use of ARBs has outstripped that of ACE inhibitors. The N ICE (201 1) guide lines consider ARBs to be preferable over ACE inhibitors for black races. Both JNC8 and N ICE recommend not to comb ine ACE inhibitors with ARB s fo r hypertension. DIRECT RENIN INHIBITOR Alis kiren the only available member of 1he latest class of RAS inhibitors which act by blocking catalytic activity of renin and inhibiting production of Ang I and Ang 11. It ,s described in Ch. 36 . Aliskircn is an equally e ffective antihypertensive as ACE inhibitors and ARBs, but experience w ith it so far is limited. However, no remarkable features have emerged and presently it is to be employed only when the more established ACE inhibitors or ARBs cannot be used.

CALCIUM CHANNEL BLOCKERS Calcium channel blockers (CCBs) are another c lass of first line antihyperlensive drugs. T heir p harmacology is desc ribed in C h. 40. All 3 subgroups of CCBs, viz. dihydropyri dines ( DHPs , e.g. amlodi pine), pheny la lky lamine (verapami l) and ben zothiazepine (d iltiazem) are equally e fficacious antihypertensives, but O H Ps are mainly used. They lower BP by decreasing peripheral resista nce without compromis ing c.o. Despite vasodilatation, flui d re tention is insignificant. Ankle edema that occurs in some patients is due to increased hydrostatic pressure across capillaries o f the dependent parts as a res uh of reflex constriction of post capillary vessels in these vascular beds.

The onset ofantihypertcnsive action is quick. With the availability of long acting preparations, most agents can be admini stered once a day. Short acting CCBs/ fo rmula tio ns (nifcdipine reg ul ar

AN TIH YPERTENSIVE DRUGS form ulation) are not used to treat hypertension. Monotherapy with CCBs is e ffective in ~ 60% hypertensives, and they may improve arterial compliance. Other advantages of CCBs are: I. Do not co mprom ise haemodynam ics : no impairment o f phys ical work capacity. 2. o sedation or other C S effects; cerebral perfus ion is maintained. 3. Not co nt ra indi cat ed in asthma, a ng ina (especially variant) and PVD patients: may benefit these conditions. 4 . Are particula rly e ffective in elderly patients, black races and low renin hyperte nsives. 5. Do not affect male sexual functi on. 6. o de leterious effect on plasma li pid profi le, uric acid leve l and electrolyte balance. 7. Shown to have no/m inimal effect on quality of life. 8. No ad ve rse foe tal e ffec t ; can be used during pregna ncy (but can weaken uterine contractions during labour). The initial large controlled trials (IIINT, TR E T, SPRI T I, II ) and metaanalysis has consistently indicated increased monality/reinfarction in patients treated with standard nifedipine (or other short-acting DHP) formulations. The increased mortality among coronary heart disease patients has been attributed to repeated surges of adrenergic discharge and marked swings o f BP attending each dose o f rapidly acting DH P. However, this risk cannot be extrapolated to verapam il/dilllazcm as broughtout by DAV IT I, II and other controlled studies, as well as to slow acting DHPs (amlodipine type). The Systo lic hypertension in Europe (Syst-EUR) trial has shown that nitrend ipine (long-acting DII P) reduces cardiovascular morbidity and morta lity in elderly hypertensives. The Hypertension optimal treatment (IIOT), and Sweedish trial in old patients with hypertension-2 (STOP-2) studies have also found CCBs equi-cffcctive as d iuretics/13 blockers/ ACE inhibitors in reducing maj or cardiovascular events as well as cardiovascular/total mortality. o excess mortality "ith the use of amlodipine in post 'vi I and acute coronary syndrome patients has been noted in the ALLHAT (2002) study. On the other hand, CCB~ do not afford sun ival benefit in post Ml patients as B blockers, ACE inhihitors do. CC Bs are also not as effective or low dose in suppressing left vent ricular hypertrophy (a major risk factor in ischacm ic hean disease) as ACE inhibitors.

Some large controlled trials including ASCOTBPLA (2005) and ACCOMPLIS H (2008) have testified to superi or effi cacy o f am lodipi nc both as monotherapy and when combined with an

ACE inhibitor for reducing cardiovascular events in high risk hy pe rtensive patients. Thus, CCBs co nti nue to be used as one of the first line monotherapy options (JNC8, 201 4, 1CE-20 11 guidelines) beca use of the ir high efficacy and excellent to lera bility. Also there is convinci ng ev idence of their stroke pre venting pote ntial (syst EUR, A LLH AT stud ies). The long-acting DH Ps a re next to AC E inhibitors in reducing albuminuria and s lowing disease progression in hypertensive/diabetic nephropathy. They are the most useful antihypertensives in cyclosporine induced hyperte nsion in renal tra nspla nt recipients. However, the negative inotropic and dromotropic action of verapamil and di ltiazcm ma y wo rsen CH F a nd ca rd iac cond uc ti o n de fects, but DH Ps are safe.

13-ADRENERGIC BLOCKERS The pharmacology and mechanism of antihypertensive action o f b lockers is described in Ch. 10. They are mi ld antihypcrtensives; do not significantly lower BP in normotensives. Used alone they suffice in 30-40% patients- mostly stage! cases. Additional BP lowering may be obtained when combined with other drugs. The hypote ns i ve respo nse to bloc kers de velops ove r 1- 3 weeks and is the n well sustained . Despite short and differing plasma half lives, the antihypertensive action of most blockers is ma intained over 24 hr w ith a single daily dose. All blocke rs, irrespec tive o f associated prope rties, exe rt eq u ivalent anti hyperten s ive effect. Drugs with intr ins ic sympathomimeti c activity (!SA) ca use less/no reduction o f HR and c.o. but decrease vascu lar resistance by [32 agonism. ebi volol lowers t.p.r. by enhancing endothe lial O production. and carved ilol does so by bloc ki ng cx. 1 rece ptors as we ll . These vasodilating 13 blockers arc preferred by many ex perts on theoretical grounds. T he re a re several contrai nd ica tions to 13 blocke rs, incl udin g card iac, pulmonary and peri pheral vascular disease. The nonse lective f3 blockers have an unfavourabl e effect on lipid

609

610

CARDIOVASCULAR DRUGS profile (raise triglyceride level and LDL/HDL ratio). They have also fared less well on quality of life parameters like decreased work capacity, fatigue, loss of libido and subtle cogni tive effects (forgetfulness. low dri ve), nightmares and increased incidence of antidepressant use. Many o f these drawbacks are minimized in the B1 selective agents and in those which penetrate brain poorly. However, some recent studies have pointed out that atenolol monothcrapy may be less effective in preventing hypertension related stroke and coronary artery disease. B blockers and ACE inh ibitors arc the most effective drugs for preventin g sudden cardiac death in postinfarction patients. However, blockers are less effective for primary prophylax is of Ml and for preventing lefl ventricular hypertrophy, but all-cause mortality has been decreased in long-term trials. Hypertens ives with stable heart fa ilure should be treated with one of the selected B blockers (metoprolol/ bisoprolol/carvedilol/ ncbivolol) along with an ACE inhibitor/ ARB (CI BIS, 1999; MERIT-HF, 1999, COPERN ICUS, 2002 studies). Barring the above subsets of patients with com pelling indications and suitability c riteria, Bblockers are not selected now as the initial antihypenensive. blockers are considered less effective and less suitable for the older hypertensive. The LIFE (2002) and ALLHAT (2002) trials have found B blockers to be inferior to low-dose thiazide or ACE inhibitor/ARB or a combination of these in preventing stroke, as well as in diabetic patients. As monotherapy, ACE inhibitors/ARBs and CCBs appear to comprom ise quality of life less than B blockers. Rebound hypertension has occurred on sudden discontinuation of~ blockers; myocardial ischacmia may be aggravated and a ngina or MI may be precipitated. Ma inly due to in ferior efficacy in primary prevention of Ml and stroke, as we ll as other drawbacks pointed out above, f3 blockers are

no longer considered first line antihypertensive drugs for monotherapy, except in patiems with oth er compelling indications (J C 8, ICE gu idelines 201 1).

l3+a ADRENERGIC BLOCKERS Labetalol (see Ch. 10). 11 is a combined a and 13 blocker; reduces l.p.r. and acls foster 1han pure 13 blockers. 11 has been used i.v. for rapid BP reduction in hype radrcncrgic stales. cheese reaction, c lonidine withdrawal. eclampsia, etc. (~·ee p. 6 I9). Oral labetalo l therapy is restricted to moderately se,ere hypertension not responding to a pure 13 blocker, because side eOects of both a blocker and f3 blocker occur with il. Labetalol is a preferred an1ihyper1ensive for rise in BP due to preeclampsia. Carvedilol This nonselective f3 + weak ,elect ive a , blocker produces vasodil.italion and has additional antioxidant/free radical scaveng111g properties. Whether these ancillary properties confer any superiority is not known. Canedilol is a frcquentl) selected drug for long-term treatment of CHF, and is approved as an antihypcrtcn~1vc as \\CII. Side effects arc similar to labetalol; liver enLymes may rise in some.

cx.-ADRENERGIC BLOCKERS

Prazosin (see Ch. 10) This prototype selective a 1 antagonist dilates both resistance and capacitance vessels; effect on the former predominating. The haemodynamic effects, viz reduction in t.p.r. and mean BP accompanied by min or dec rease in venous return and c.o. are similar to that produced by a direct acti ng vasodilator hydralazine. However, unlike hydralazine, there is little reflex cardiac stimulation and renin release during long-term therapy. Tachycardia does not compensate for the fall in BP, because re lease inhibitory a 2 (presynaptic) receptors are not blocked: autoregulation of NA re lease remains intact. Prazosin probably decreases central sympathetic tone as well. Renal blood flow and g.f.r. are maintained but fl uid retention may attend fall in B.P. Cardiovasc ular reflexes are not appreciably impaired during chronic therapy. but postural hypotension and fainting may occur in the beginning-called 'first dose effect ', and wi th dose increments. This disappears with continued therapy, but may persist in the elderly. For th is reason, prazosin is always started at low dose (0.5 mg) given at bedtime and gradua lly increased with twice daily administration till an adequate response is produced ( max. dose IO mg BD). An oral dose produces peak fall in BP after 4-5 hours

ANTIH YPERTENSIVE DRUGS

and the effect lasts for nearly 12 hours, though plasma t ½ is only 3 hours. This may be due to generation of active metabolites. Pra7osin does no t impair carbohydrate tolerance, and may have a s ma ll favourable effect on lipid profile. Symptomatic improvement may occur in males w ith urinary symptoms due to prostal ic hypenrophy. \I I IPRESS XL: Pra70sm GITS 2.5 mg, 5 mg tabs.: PRAZOPRESS 1, 2 mg tabs.

A dverse effects

Prazosin is genera lly well tolerated at low doses. Apart from postural hypotension related symptoms ( particularly in the beginning), other side clTects are headache, drowsi ness, d ry mouth, weakness, palpitat ion, nasal blockade, bl urred vision and rash. Ejaculation may be impaired in males: especially with hig he r doses. Fluid retention attending prazosin monotherapy may precipitate C H F.

Use Prazosin is a moderate ly potent a ntihype rtensive, but is not used as a first line drug because fluid retention and tolerance gradually develops with monotherapy necessitating dose increase-more s ide effects an d risk of CHF. lt may be added to a diuretic + blocker in those not achieving target BP. Terazosin, Doxazosin These arc long-acting congeners or prazosin with s imilar properties but suitable for once daily dosing (see p. 155-56). In the AL LH AT (2002) study doxazosin monotherapy has doubled the incidence of CHF: but thi~ can occur "ith any a, blocker. A higher incidence of stroke relati\ e to patients receiving a thiazidc diuretic was a lso noted. Nonselective a blockers (Phentolamine, Phenoxybenzam ine) The nonselecti, e a blockers have been disappoint mg for routine treatment of hypertension. because fa ll in t.p.r. is compensated by increased HR and c.o. They hlock both a, and receptors- NA release is accentuated. TI1ey are n:ser,ed for special situations like pheochromocytoma. clonidine wuhdrawul. cheese react ion. etc., "here circulating CAs are responsible for the rise in BP.

CENTRAL SYMPATHOLYTICS Clonidine Ir is an im idazoline derivative having complex actions. C lonidine is a partial agonist with high affini ty and high intrinsic activ ity at

receptors, especially a 2A subtype in brai nstem. The major haemodynamic efTccts result fro m sti mu lat ion of a 2A receptors present main ly postjunctionally in medulla (vasomotor centre). This decreases sympathetic out flow result ing in fall in BP and bradycardia. Enhanced vagal tone contributes to bradycardia. Plasma 'A declines. Though clonidinc is capable of reduci ng NA release from peripheral adrencrg ic nerve endings (release inhibitory prej unctional a, action), this is not manifest at clinically used d;ses. Clonidine is a moderately potent antihypertensive. Pharmacokinetics C lonidinc is well absorbed o rally; pea k occurs in 2-4 hours; 1/2 to 2/3 of an oral dose is excreted unchanged in urine, the rest as metabolites. Plasma L½ is 8-12 hours. Effec t of a single dose lasts for 6-24 hours. Dose: Stan w ith 100 µg OD or BO. max. 300 µg TDS, orally or i.m . CATAPRES 150 µg tab, ARK.AMIN 100 µg Lab.

Adverse effects Side effects with c lo nidine are rela1i,·cly common. Sedat ion, menial tkpression, d isturbed sleep; d ryness of mouth. nose and eyes (secrelion is decreased by central action). constipation (an1isecret0ry efTccl on the intestines). Impotence. salt and water retention, bradycardrn. Postural hypotension occurs. but is mostly asym pto matic. Alarming rise in BP, in excess or pretreatment level, with tachycardia, restlessness, anxiety, sweating, headache, nausea and vomiting occur in some patients when doses of c lonidine are missed for 1- 2 days. The syndrome is very similar to that seen in pheocbromocytoma: plasma catecholamme (CA) concentration is increa ed. This is due to: (a) Sudden removal of central sympathetic inhibition resulting in release of large quantities o f stored CAs. (b) Supcrsensiti,ity of pcnphcral adrenergic structures to CAs that develops due to c hronic reduction of sympathetic tone during clonidine therapy. A combination of a blocker with a l:l blocker, or a potent , asodi lator (nilrOprusside) or clonidine itself can be used to treat the syndrome.

Interactions Tricyclic antidepressants and chlorpromazine abolish the antihypertensive action of clonidine. probably by block ing a receptors on which clonidi ne acts. Use Clonidinc "as a popular antihypenensive in the late 1960s and 1970s, but frequent side elTects. ris k of withdrawal hype11cns ion and deve lopment of tolerance have

611

612

CARDIOVASCULAR DRUGS re legated it to a 3rd or 4th choice drug. There is no data o n prognostic hcncfits, if any, of clonidine. At present, it is occasionally used in combination with a diuretic.

Other indications I . Opioid "ithdrawal: Opioid and a, adrenergic systems converge on the same elTectors in many systems; both activate the Gi regulatory protein. Clonidinc suppresses sympathetic ovcractivity of opioid withdra\\ al ,yndromc and reduces era, ing to some extent. C lonidine has also faci litated alcohol withdr.mal and smoking cessation. 2. C lonidinc has analgesic activity. It has been used to subs titute morphine for intrathccal/epid ural surgical and postoperative analgesia. 3. Clonid ine attenuates vasomotor ymptoms of menopausal syndrome. 4. C lonidine has been used to control loose motions due 10 diabetic neuropathy. It may be act ing by a , receptor mediated enhancement of salt abwrpt1on in gut mucosa.

Methyldopa This a -methyl analogue of dopa is convened in d1e body to a methyl A, which is a elective a.., agonist. In the brain, the a methyl A generated from methyldopa acts on central a 2 receptors to decrease clferent sympathetic activity. However, in contrast to clonidinc. methy ldopa decreases t.p.r. more than HR or c.o. In large do cs, methyldopa inhibits the e nzyme dopa decarboxylasc. It was initially believed that methyldopa acted as antihyperten ive by depressing synthesis of A in peripheral sympametic neurone , and by generating a methy l A which acted as a false (weak) transmiller. Sub equently a methyl A was found to be as potent vasoconstrictor as A. Many other findings have later confirmed the primary central site of action o r mcthyldopa. Mc th y ldopa is a moderate efficacy antihypcrtcnsive . Ci rculating levels of A a nd renin tend to fa ll due to reduction in sympat hetic to ne. Inhibi t ion of p ostural re!lex e s is mild. Pharmacokmet1cs Though meth) ldopa is trnn~ported acll\Cly by intestinal amino acid carrier, less than 1/3 of an oral dose is absor bed. It is partly metaboli£ed and partly excreted unchanged in urine. Antihypertensiv e c m:ct de, elops over 4-{j hours and lasts for 12- 24 hours.

Dose: 0.25-0.5 g BD- QID oral. EM IX)PA, ALPIIADOPA 250 mg tab. Adverse effects Sedation, lethargy and reduced mental capacity are common side effect;,. Cognitive impairment may develop. Dryness of mouth. nasal stuffiness. headache,

fluid retention. weight gain and impotence are the other side effects. Postural hypotension is generally mild. Positive Coomb', test occurs in 1/6 patients, few devdop haemolytic anaemia. Fc,er, rash. hepatitis, ' flu· like illness, thrornbocytopen ia and rarely lupus syndrome occur. Rebound hypertension on sudden withdrawal of methyldopa is mild and less common.

/nterac/lons Tricyclic antidepressants re,erse its action by blocking 11s active transport into the adrencrg ic neurones.

Use Methy ldopa was a w ide ly used an tihypertensive in the I 960s and 1970s, especia lly in co mbination with a diuretic. I lowevcr, it is rare ly used now, except to treat hypertension during pregnancy wherein it has a long track record of safety, both for the mother as well as the foetus. VASODILATORS Hydralazine /Dihydralaz ine Introduced in the 1950s, it is a directly acting arteriolar vasodilator with little action on venous capac itance vessels. Hydra la,line reduces l.p.r. and causes greater decrease in diastolic than in systolic BP. Reflex compensator y mechani sms a re e voked which cause tachycardia, increase in c.o. and ren in re lea e -t increased aldosterone -t a a nd water retention. Thus, a hyperdynam ic circulatory state is induced- angi na may be precipitated due to increased cardiac work. There is no reduction in rena l blood fl ow despite fall in BP. l lowcver, fluid re te nti on a nd edema may occur by the above mechanism . Tolerance to the hypotensive action o r hydrala zine deve lops unl ess a diuretic or a blocker or both are g iven together to block the compensatory mechan isms. The mechanism o f vascul ar smooth mu c le relaxant acti on of hydralazine is not clearly known. Interference with Cai+ re lease, opening of certa in K c hannels and/or O generation may be invol ved. Pharmacokme llcs llydrala7ine is well absorhcd orally, and is subjected to fi rst pass metabolism 111 liver. The chief metabolic pathway is acetylation which ex hibits a bimodal d1~tribution in the populat ion: there arc ~lmv and fast acetylators. llydrala71ne is completely mctaboli.lcd both m ll\cr and plasma: the metabolites are excreted in urine. t½ is I 2

ANTIHYPERTE NSIVE DRUG S hours. llowc, er. hypotcnsive effect lasts longer ( 12 hours). probably because of its persistence in the vessel wall.

Dose: 25- 50 mg OD TDS; '-lEPRESOL 25 mg tab. Adverse effec ts

arc frequent and mainly due to vasodilaiation. • Facial flushing. conjunctiva) injection, throbbing headache. diz7iness, palpitation. nasal st uffiness, flu id retent ion, edema. C HF. • Angma and Ml may be precipitated in patients with coronary artery disease. • Postural hypotension is not prominent because of linle acuon on veins. • Parcsthcsias, tremor. muscle cramps, rarely peripheral neurit is. Gastrointestinal disturbances arc frequent. • Lu pus crythcmatos us or rheumatoid arthritis like symptoms develop on prolonged use of dose above I 00 mg/day. This is mo re common in women and i11 slow acctylators.

Use

I lydralazine is now rarely used as a second line alternative only in combination with a diuretic and/or f:3 blocker for patients not achieving target BP with first line drugs. It is one of the antihypertensives that has been u ed during pregnancy, especia lly for preeclampsia. Injected hydralazine is occasionally employed in hypertensive emergencies. It is contraind icated in o lde r patients and in those with ischacmic hea1t disease. The arteriolar d ilator actio n of hydra lazine can be employed in the management of C HF parti c ularly in co mb inatio n wi th isosorb ide dinitrate (see p. 566). Minoxidil It is a powerfu l vasodilator, the pallern of action resembling hydralazine. Vasodilator side effects are more marked; risk of cardiac ischaemia and heart failure is high. It is not used orally now. The active metabolite of mi noxidil is an opener of ATP sensitive K channels; causes vasodilatation by hyperpolarizing s mooth muscle.

Use in a/opecia Hirsutism was observed as a side e ffect of oral minoxidi l. A pplied topica lly (2% twice da ily) it promotes ha ir growth in male pallern baldness and in alopecia areata. The res ponse is s low (takes 2- 6 month s) and incomple te, but upto 60% subj ects derive some benefi t, a lbeit for short periods. Ba ldness recurs w hen therapy is discontinued. T he mechanism of increased hair growth is not known; may involve: (a) Ope nin g of K • c ha nn e ls and e nh a nced microcircul ati on around ha ir fo llic les.

(b) Direct stimu lati on o f resting hair fo llicles. (c) A lterati on of and rogen effect on genetically progra mmed hair fo llic les. Local irritation, itching and burning sensa tio n are frequent. De rmatolog ical reactio n and systemic side e ffects (headache, dizz iness, palpitation) occur in 1- 3% cases. MINTOP. GR0\1A. E 2°,. scalp louon, MULTIGAIJ\ 2o/o topical solution and metered spray, MANEXIL So/, gel; apply I\\ ice a day.

Sodium nitroprusside It is a rapidly (w ithi n seconds) and consistently acting vasodilator w ith brief du ration o f actio n (2- 5 min) so that vascular tone can be titrated with the rate of i.v. infusion. itroprusside re laxes both res ista nce and capacitance vesse ls: reduces t.p.r. as we ll as c.o. (by decreasing ve nous return). Myocardia l wo rk is reduced, but ischaemi a may be accentuated due lo coronary stea l. Only mild reflex tachycardi a is produced in s upine posture. Plasma renin is increased . In patients wi th heart fa ilure and ventricular dil atat ion, n itroprusside improves ventri c ular fun ctio n and c .o. ma inly by reduc ing aortic impedance (afterload), but a lso by lowering atria l filling pressure (preload). Endothelia l cells, RBCs (and may be other cells) split nitroprusside to generate NO wh ich relaxes vascular smooth muscle (also see p. 544). This occurs both enzymatically and none nzymatically. The enzymes involved are different from those that produce NO fro m g lyceryl trinitrate. onenzymatically it is converted to O (and CN) by glutathione. This may be respo nsible for the different pattern of vasodilator action compared to nitrates, as well as for the fact that no nitrate like to lerance develops to nitroprusside action. N itro prusside is now a seco nd line d rug fo r ce rta in hypert e ns ive eme rge nc ies (see p. 6 19). 50 mg is added to a 500 m I boul e of saline/ glucose soluti on. T he infusion is started at 0.02 mg/min and titrated upwa rd with the respo nse: 0. 1-0.3 mg/m in is o ften needed. It deco mposes at a lkaline pH a nd o n ex posure to lig ht: the infusio n bottle should be cove red with blac k pape r.

613

614

CARDIOVASCULAR DRUGS itroprusside is sp lit to re lease cyanide. T he latter is converted in liver to thiocya nate wh ich is excreted s lowly. lf larger doses are infused, excess thiocyanate may accumulate and produce toxicity, including psychosis and other C S effects. lntracranial pressure may ri se. Side efTects ma inly due to vasodilatat ion arepalpitation, nervou sness, vomiting, perspiration, pain in a bdomen , weakness, disorientation, and lactic ac idosis (caused by the re lea ed cyanide). itroprusside has also been used to produce controlled hypotension, in re fractory CHF (see p. 566) and in acute mitral regurgitation. SONIDE, PR USIDE. NIPRESS 50 mg m 5 ml 111j.



Hypertensive hea rt disease-left ventricular hypertrophy, heart fai lure. • Coronary artery disease (CAD), angina, myoca rdia l infarction (M 1), sudden cardiac death. • Arteriosclerotic peripheral vascular disease, retinopathy. • Dissecting aneurysm of aorta. • Glomerulopathy, rena l failure. Patie nts who have a lready suffered some TOD have greater risk of further organ damage and death at a ny level of ra ised BP, than those without TOD. T he current ICE guidelines (2011) ha ve graded hypertension as :

ADRENERGIC NEURONE BLOCKERS Reserpine It is an alkaloid from the roots of Rammlfia s erpemi11a (sarpgandha) indigenous to India which has been used in · Ayurvedic ' medicine for centuries. The pure alkaloid was isolated in 1955 and later foun d to act by causing CA and 5- HT depletion. It wa~ a popular antihypertensive of the late 1950s and early 1960s. but is now used only as a pharmacological tool. Reserpine acts at the membrane of intraneuronal vesicles which store monoamines (NA, DA. 5-1 IT) and irreversibly inhibits the vesicular monoamine trans porter (VMAT2}. The monoamines are gradually depleted and degraded by MAO. The efTects last long after the drug is eliminated (hit and run dnig) because tissue CA stores are restored only g-radua lly. Higher doses dcrlcte CAs and 5-1IT in the brain as well; cause sedation and mental depression. Antipsychotic e fTect ( mild) and extrapyramidal symptoms are produced due to DA depletion. Guanethidine It is a polar guani d inc compound which is taken up into the adrenergic nerve endings by active amine transport, where it inhibits nerve impulsecoupled release of NA. Guanethid ine has gone out of use now due 10 marked side eflects.

TREATMENT OF HYPERTENSION The aim of a nt ih ypertens ive th e rapy is to prevent morbidity and mortality associated with pe rsistently raised BP by lowering it to the target level, with mi nimum inconvenience to the patie nt. Both systolic and diastol ic BP predict the likelihood of target organ damage (TO D) and complications such as: • Cerebrovascul ar disease, transient ischaemic attacks, stroke, encephalopathy.

Hypertension

Systolic

Diastolic

Stage I

140-159

90-99

Stage II

160-179

100-109

Severe

55 years (men), > 65 years (women) Family h/o premature CV disease Smoking

4. 5.

Dyslipidemia {TLDL, ! HDL, TTG) Diabetes mellitus

6. 7. 8. 9.

Hypertension Obesity (BMI 30) CKD (Microalbuminuria or g.f.r. < 60 ml/min) Sedentary life style

CKD-Chronic kidney disease.

T he J C7 ha ve a lso identifie d compelling indications (see box) wh ich may mandate use of specific ant ihypertensive drugs even in patients w ith BP va lues lower than 140/90 mm Hg . Moreover, presence of compelling indications may suggest fi x ing a lower target BP value to be attai ned by drug thera py.

ANTIHYPERTENS IVE DRUGS Compelling indications for specific antihypertensive drug classes Diuretics Heart failure 1. High coronary artery disease risk 2. 3. Recurrent stroke prevention ACE lnhibitors/ARBs 1. Heart failure 2. Post-myocardial infarction 3. High coronary artery disease risk Diabetes 4. Chronic kidney disease 5. Recurrent stroke prevention 6. Calcium channel blockers 1. Diabetes 2. Stroke prevention P-Adrenergic blockers 1. Stable heart failure Post-myocardial infarction 2. High coronary artery disease risk 3.

Beneficial cfTects o f lowering BP has bee n established in a ll pati ents havi ng BP a bove 140/90 mm Hg, a nd even in the 120- 139 (systolic) or 80 89 mm Hg (diasto lic) ra nge in those with compelling indicatio ns or cardiovascu lar risk facto rs; e.g. in diabetics, lo wering diastolic BP to 80 mm Hg was found to reduce cardiovascular events to a greater extent than on red uci ng it upto 90 mm Hg. Ho we ve r, fo r patie n ts aged ~60 years t he JN C 8 (2014) has s uggested thresho ld systolic BP value o f 150 mm Hg for initiating treatment, as well as to be the treatment goal (< 150 mm Hg). The threshold and goal distolic BP value of 90 mm l lg is the same as for patients -

K•-

+ I + +

• - - Ca2• • - - Mg2•

Na• Aldosterone

+--

t-

(,)

::>

+

0

SITE II +

('.)

z

j:::

(,)

w

~

U

..J ..J

Active transport

0

u

+- - - , dSSJVe diffusion

'

' -• H 0 2

_ ......

HYPERTONIC

H 0

2 ::: ::: - --

-

__ _

Urea

Fig. IX.1: Diagrammatic representation of nephron showing the tour sites of solute reabsorption. The thick ascending limb of loop of Henle (TAL) is impermeable to water Glu.-Glucose; AA-Amino acid; Org. An.-Organic anions.

622

DRUGS ACT I NG O N KIDN EY

in 24 hours. The diuretics act primaril y by inhibiting tubular reabsorption: just I% decrease in tubular reabsorption would more tha n doub le urine output. The mechanisms that ca rryout ion movement across tubular cells a re complex and involve a variety of energy dependent transmembrane pumps as well as channels in between the loose fitti ng cell of the proximal tubu le ( PT). All the a+ that e nters tubular eel ls through the luminal membrane is pumped out of it into the rena l interstitium at the basolatera l membrane by a+K•ATPase e nergised Na+-K+ anti porte.r (see F igs 42.1 and 42.2). Because there is a large intracellular to extracellular grad ient fo r K+, it di ffuses o ut th rough K' c hannels to be rec irc ulated by th e a+- K+ an tiporter. For simplifi cation, tubular reabsorption can be divided into fou r sites (Fig. IX. I).

Site I: Proximal tubule

Four mechanisms of Na+ transport have been defined in this segment. (a) Direc t en try of Na a lo ng a favourab le electrochemical gradient. This is electrogenic. (b) T ranspori of a• and K· coupled to active reabsorption of glucose, amino acids, other organic anions and Po;- th rough spec ific symporters. Only the glucose coupled Na+ reabsorption is electrogenic. (c) Exchange with W: The PT cells secrete W with the help of a a•-w antiporter (Na -W exchanger-3) located at the luminal membrane. This exchange moves Na• from tubular fluid to inside the cell. The secreted W combines with HCO: in the tubular fluid to form carbonic ac id (Fig. IX.2). This H2 C03 is broken into Hp + CO2 by membrane bound brush border CAse (Type IV enzyme), because potaneous dissociation of H2C03 is very slow. Both CO2 and Hp diffuse inside the cell and recombine to fonn H2C03 ( intracellular soluble type II CAse catalysed reaction) which is the source o f W. The dissociated Hc o ; in the cell is transporied to cortical e.c.f. by basolatera l membrane a' HC0 3 symporter resulti ng in net reabsorption of NaHCOJ" Practically a ll HC03- ion present in glomerular filtrate is reabsorbed in PT by 0

! Na•-H • Antrporter j

Na4iC03 Symporter

P.T. Cell

NaHC03

.....,.....,___ Na·- - -- -,.+ 3 hours a fter methotrexate. 5. To enhance anticancer efficacy of 5-fluorouracil (5-FU) Fo linic acid is now routine ly infused i.v. along with 5-F U (see Ch. 64), because TH FA is required for inhibition of thymidylate synthase by 5-FU.

Adverse effects Oral fo lic acid is e ntire ly nontox ic. Inj ecti o ns rare ly ca use sensi ti vi ty reacti ons. ERYTHROPOIETIN Erythropoietin (E PO) is a s ia loglycoprotein hormone (M W 34000) produced by peritubular cells of the kidney that is essential for norma l erythropoiesis. Anaemia and hypoxia are sensed by kidney cells and induce rapid secreti on of EPO, which acts on erythro id marrow and: (a) Stimul ates proliferation of colony form ing cells o f the erythroid series. (b) induces haemoglobi n formation and erythroblast maturation. (c) Releases reticulocytes in the circulation. EPO binds to specific receptors on the surface of its target cells. T he EPO receptor is a JAKSTAT-bind ing receptor that alters phosphoryla tion o f intracellu lar protei ns a nd activates transcription factors to regulate gene ex pression.

manner, but has no effect on RBC li fespa n. It is recombi nant hum an erythropoieti n (Epoetin a , f3) is admi nistered by i.v. or s.c. injection and has a plasma t½ of 6-10 hr, but acti on lasts se veral days.

Use T he prima ry indication for epoetin is anaemia of chronic renal failure which is due to low levels of EPO. Only sm ptomatic patients w ith I-l b 8 g/dl should be cons ide red for EPO therapy. Epoetin 25- 100 U/kg s.c. or i.v. 3 ti mes a week (max. 600 U/kg/week) raises haematocrit and haemoglobin, reduces need for tran sfusions and improves qua lity o f life. It is therefore increasingly used now in e lig ible patients. It is prudent to start w ith a low dose and titrate upwards to keep haematocrit between 30- 36%, a nd Hb 10- 11 g (max 12 g) per di. Trials have fo und higher mortality if Hb level was raised to normal ( 13.5 g/d l). As such, lowest dose of epoetin sufficient to avoid blood transfusion or that to maintain Hb between LO- I I g/d l should be used. Some recent studies have indicated that dose reduction by about 30% is possible when epoetin is given s.c. compared lo i. v. Exe rc ise capacity and overall wellbeing of the patients is improved. Most patients have low iron stores; req uire concurrent parenteral/o ra l iron therapy for an optimum response. Othe r uses are: I. Anaemia in AIDS pa ti e nts t rea te d w ith z idovudine. 2. Cancer chemotherapy induced anae mia. 3. Preoperative increased b lood prod uction for autologous transfusion during surgery. Response to epoetin is poor in case of primary bone marrow disease, and in nutritional anaemias, because the endogenous c irculati ng levels of EPO are a lready high. Adverse effects Epoeti n is non immunogenic. Adverse effects are related to sudden increase in haematocrit, blood viscosity and periphera l vascul ar res istance (due to correction of anaemia). T hese ar increased c lot formation in the A-V shunts ( m ost p atients are o n di a lysis), hy pe rte nsive e pi sodes or persistentl y ra ised

657

658

DR UGS AFFECTING BLOOD AND BLOOD FORMATI ON

BP, serious thromboembolic events, occasionally seizures. Flu-li ke symptoms lasting 2-4 hr occur in some patients. ll EMA X 2000 IU/ml and 4000 JU/ml vials; l:.PR EX 2000 JU. 4000 IV and 10000 JU in J ml prcfi llcd syringes; ZYROP (epoetin ~) 2000 IV and 4000 IU , iab.

Darbepoetin a

This is a hyperglycosylated m od ified preparati o n of EPO t hat has a

t ½ of 24- 36 ho urs and i s l onger ac tin g th an epoetin . A ccordin g ly, it needs to be injected once a week and is more convenient. Indications, effi cacy and ad verse effects are similar to epoetin. Dom: Initially 0.45 µg/kg s.c./i.v. once a week; dose be adjusted later according 10 rise in Hb level. CRESP 40 µgv1al inj .

PROBLEM DIRECTED STUDY 44.1 A lady aged 40 years consult s you for treatment of her anaemia that is not improvi ng with medici ne prescribed by a local doctor. She told t hat she is suffering from weakness, fatigue and occasio nal giddiness for the last 4-5 months. She went to a local doctor 2 months ago who got her blood tested, which showed Hb was 7.5 g/dl. A liquid medicine was prescribed, that she has been taking 1 tablespoonful daily without any benefit. The medicine was found to be syrup Ferric ammonium citrate 160 mg/ 15 ml along with fol ic acid 0.5 mg and vit B12 7.5 µg. She also revealed that she suffers from heart burn, and has been taking a tablet (Rabeprazole 20 mg) once daily for the last 2-3 years. Repeat blood testing showed Hb to be 7.6 g/dl, haematocrit was 27%, RBCs were microcytic-hypochromic, and other values were consistent with iron deficiency anaemia. Her periods were normal and det ailed examination showed no evidence of bleeding from any site. (a) What could be the reason for her failure to improve with oral iron therapy that she has been taking? (b) Can she still be t rea ted with oral iron, or does she require parenteral iron therapy? What treatment would be appropriate for her? (see Appendix-1 for solution)

10

Chapter

45

Drugs Affecting Coagulation, Bleeding and Thrombosis

Haemostasis (arrest of blood loss) and blood coagulation involve complex interactions between the injured vessel wall, platelets and coagulation facto rs. For blood to clot, a cascading series of proteolytic reactions (Fig. 45.1 ) is started by: (i) Contact acti va tion of Hage man ractor: intrinsic system, in which all factors needed for coagulation arc present in the plasma. This pathway is responsible for clotting when blood is kept in a glass tube, and for amplification of the common pathway. This is slow and takes several minutes to activate factor X. (ii) Ti ss ue thromboplastin : extrins ic system , needs a tissue facto r. but activates facto r X in seconds. In the normal course. coagulati on after injury to vessel wall occurs by this pathway. The subsequent events are common in the t\l\lO systems and result in polymerization of fibrinogen to form fibrin strands. Blood cells are trapped in the meshwork of fi brin strands producing clot. Two in l'ilro tests ·activated partial thromboplastin time· (aPTT) and · prothrombin time· (PT) are employed for testing integrity o r the intrinsic, extrinsic and common patlrn ays of the coagulation cm,cade. The results are interpreted as:

Intrinsic pathway interfered fatrinsic pathway interfered Common pathway interfered

PT

aPTT

Normal

Prolonged

In addition to its critical role in cleaving and polymerizing fibrinogen, thrombin activates many upstream factors (especially f. XI, VIII and V) of the intrinsic and common pathwaysamplifyi ng its own generation and continuati on of clot forma tion. It is also a potent activator of platelets. On the other hand, factors like antithrombin, protein C, prolein S, antilhromboplastin and the fibrinolysin system tend to oppose coagulation and lyse formed clot. Thus, a check and balance system operates to maintain blood in a fluid state while in circulation and allows rapid haemostasis following inj ury. COAGULANTS These are substances which promote coagulation. and are indicated in haemorrhagic states. Fresh whole blood or fresh frozen plasma provide all the factors needed for coagulation and are the best therapy for deficiency of any clotting factor. Moreover. they act immediately. Other drugs u ed to restore haemostasis are:

COAGULANTS

I

{12- 14S) Prolonged

1orrnal

(26-32S) Prolonged

Prolonged

Most clotting factors are proteins present in plas ma in the inactive (zymogen) rorm. By partial proteolysis they themselves become an active protease and activate the next factor.

I Vitamin K

I

--------K1: Phytonad ione (Phylloquinone) K3: Menadione Acetomenaphthone Menadione sod. bisulfitc

I I Miscellaneous Fibrinogen (human) Antihaemophilic factor Desmoprcssin Adrenochrome monosemkarbazone Ethamsylate Rulin

660

DRUGS A FFECTING BLOOD AND BLOOD FORMATION

INTRINSIC SYSTEM

EXTRINSIC SYSTEM

Xlla .,__ Activated by contact

CJ_.7 1HMW-Kininogen xr

o. ..J (/)

~

Tissue damage

1

~ la

m-? @~

VJJa

+ +---------@,

ca2+

xa

Xlla Xla Kallikrein

TF(III)

::0

• ~ 0

v mal PI.Ph

ca2+

0 1- -~

·

0

.,---CJ

~

Ca~r 1.r11~.,_) !PROTHROMBIN (ll~

~-

THROMBIN FIBOGEN

_ ____,_ +_ _ ., r-lBRIN (soluble)

_Xlllal .__ Thrombin C)_.7FIBRIN(insoluble) Fig. 45.1: The coagulation cascade. The vii. K dependent factors (factor 11, VII, IX and X) have been encircled, Factors inactivated by heparin (H) in red; the more important inhibited steps are highlighted by thick arrow. a- activated form; PI.Ph. -Platelet phospholipid; HMW- High molecular weight; TF-Tissue factor (factor Ill)

VITAMIN K It is a fat-soluble d ie tary princ iple required for the synthesis o f c lotting factors. Dam ( 1929) produced bleedi ng disorder in chicken by feeding deficient diet. T his "as later found to be due lo decreased conccnLration of proth rombin in blood and thai it could be cured by a fa t soluble fraction of hog liver. T his factor was called Koag11/a1io11s l'ita111i11 (vit K) and soon its s tructure was worked out. A similar vitamin ,, as isolated in 1939 from alfalfa grass and labelled vit K,, wh ile that from sardine (sea fish) meal \\US labelled K, . In nature. vil K, (menaquinone) is produced only by baeLeria, and that ·found in humans is dcri,·cd from intestinal flora. Synthetic compounds have been produced and labelled K r

o¢=:"' 0

V IT AMIN K

Chemistry and source Vit K has a basic naphthoquinone structure, with or without a side chain (R) at position 3. The side chain in K, is phylyl, in K, pre11yl, whi le in K, there is no side chain. · Dietary sources are-green leafy vegetables, such as cabbage. spinach; and liver, cheese, etc.

Datly requ,rement It is uncertain. because a variable amount o f menaquinone (vit K,) produced hy colonic bacteria

DRUGS AFFECTING COAG ULATION, BLEE DING AND THROMBOSIS becomes available systemically. Even 3-10 µg/day external source may be sufficienl. 1lowever, the total requirement of an adult has been estimated to be 50-100 µg/day.

STYPTOCID 10 mg "1th adrcnochrome monoscrnicarbaL.one 0.5 mg, rulln 50 mg, , it C 37.5 mg, , it D 200 i.u., Cal. pho~phate 260 mg per tab.

Action Vit K acts as a cofactor at a late stage in the synthesis by liver of coagulation protcins-prothrombin, factors VII , IX and X. The vii K dependent change (y carboxylation of glutamate residues of these zymogen proteins (Fig. 45.2) confers on them the capacity to bind Ca2+ and to get bound to phosphol ipid surfaces- properties essenti al for pa rti cipation in the coagulation cascade.

Use The only use of vit K is in prophylaxis and treatment of bleeding d ue to defi ciency of cloning factors in the fo llowing situations: I. Dietwy deficiency : of vit K is very rare in adults. However. when it occurs 5- 10 mg/ day oral or parentera l vit K rapidly corrects the de fects. 2. Prolonged antimicrobial therapy: treat in the same way as d ieta,y deficiency of vit K. 3. Obstructive jaundice or ma/absorption syndromes (sprue, regional ileitis, stea torrhoea, etc.): vii K 10 mg i.m ./day, or orally along with bile salts. 4. Liver disease (cirrh osis, vira l hepati tis) : associated bleeding responds poorly to vit K. Because or hepatocellular damage, synthesis of clotting facto rs is inadequate despite the presence of vit K. However, vit K may be of some use if its absorption had been affected due to lack of bile sails. 5. Newborns: All newborns have low levels of prothrombin and other clotting factors. Further decrease occurs in the next few days. The cause is both lower capacity to synthesize clotting factors as we ll as deficiency or vit K. The defect is exaggerated in the premature infa nt. Vil K I mg i.m. soon after birth has been recommended routinely. Some prefer administering 5- 10 mg i.m. to the mother 4 12 hours before deli very. Haemorrhagic disease o r the newborn can be e!Tectively prevented/treated by such medication. Mcnadione (K,) should not be used for this purpose (see below). 6. Overdose of oral (coumarin) anticoagulants: This is the most important indication of vit K. Phytonadi one (K 1) is the preparation of choice, because it acts most rap id ly; vit K, and its water soluble sails are re latively in~ffective. The dose depends on the severity or hy poprothrombinaemia (mea ured I R) and bleeding. Unnecessarily high dose is to be avoided because it will render the patient unresponsive to oral anticoagulants fo r severa l days.

Utilization Fat-soluble forms of vit K are absorbed from the in testine via lym ph and require bile salts for absorption, while watersoluble forms are abso rbed d irectl y into portal blood. An active transport process in th e jejunum has been demonstrated for K., whi le K2 and K3 are absorbed by simple diffu ion. Vit K is only briefly concentrated in liver, but there arc no significant stores in the body. It is metabol ized in li ver by side chain cleavage and glucuron ide conjugation; metabolites are excreted in bile and urine. Deficiency Deficiency of vii K occurs due to liver disease, obstructive jaundice, malabsorption and long-term antimicrobial therapy which alters intestinal flora , but deficient diet is rarely responsible. The most important man ifestation is bleeding tendency due to lowering o r the levels or prothrombin and other cloning factors in blood. llaematuria is usually first to occur; other common sites o r bleeding are g. i.t., nose and under the skin~ cchymoses. Preparations Phytonadione: VITA~11',- K. KVI, K-\\, I 10 mg,ml for i.m. injecuon. Mcnadione: 0.66 mg in GYNAE CVP with vii C 75 mg, ferrous gluconate 67 mg, Cal. lactate 300 mg and citras biofla, ono,d 150 mg per cap: Ace1omcnaph1hone: ACETO\1 E!\ADIO:-S.E 5, 10 mg tab; KAPIL.IN 10 mg tab. Menadione sod. b isulfite: 20 mg, m CAD I PER-C ,, 1th ~it C 100 mg. adrenochromc monosemicnrbuone, I mg, rutin 60 mg, rnelhylhcspcridin 40 mg, Cal. pho~phate I 00 mg per tab.

661

662

DRUGS AFFECTING BLOOD A N D BLOOD FORMATION

Severe deficiency: IO mg i.111. fol lowed by 5 mg 4 hourly; bleeding generally stops in 6-12 hours, but normal levels of coagulation factors are restored only after 24 hr. This dose of vit K will block anticoagulant acti on for 7- 10 days. Moderate deficiency: IO mg i.m. fol lowed by 5 mg once or twice according to response. Mild deficiency: .lust omit a few doses of the anticoagula nt. Adverse effects Phytonadione injected i.m. or given orally hardly prod uces any adverse effect; allergic reactions are rare. Severe anaphylactoid reactions can occur on i.v. injection of emulsified formulation; this route should not be used. Menadionc and its water-soluble derivati ves can cause haemolysis in a dose-de pendent manner. Pat ients with G-6-PD deficiency and neonates are especially susceptible. In the newborn mcnadione or its salts can precipitate kemicterus: (a) by inducing hacmolysis and increasing bilirubin load. (b) by competiti,ely inhibiting glucuronidation of bilirubin. Glucuronide conjugation is, as such, inadequate in neonates. Because of lower efficacy and higher toxicity, there is little j ustification to use menadione and its water soluble salts for any indication. Fibrinogen The fibri11ogen fraction of human plasma is employed to control bleeding in haemophilia. antihaemophilic globulin (AHG) deficiency and acute afibrinogenemic states: 0.5 g is infused i.v.

rIBRINAL 0.5 g/bottle for i., . infusion. Antihaemophilic factor It is concentrated human AHG (factor VIII) prepared from pooled human plasma. It is indicated (along with human fibri uogen) in haemophilia which is due to AHG deficiency. It is highly effective in controlling bleeding episodes, but action is short-lasting ( I to 2 days). Dose: 5-10 U/kg by i.v. infusion. repeated 6-12 hourly. rIBRL AL-H.ANTII IAE\1OPHILIC FACTOR: 150 U or 200 U fibrinogen 0.5 g,bottle for i.~. infusion. Oesmopressin It releases factor VIII and von Willcbrand's factor from vascular endothelium and checks bleeding in haemophil ia and von Willebrand·s disease (see p. 643). Adrenochrome monosemicarbazone It is believed to reduce capillary fragility, control ooLing from raw surfaces and prevent microvessel bleeding, e.g. cpistaxis, haematuria, secondary haemorrhage from wounds. etc. Its efficacy is unccnain. Dose: I 5 mg oral, i.m. STYPTOCHROME 3 mg/2 ml mj., STYPTOCIO: 2 mg/ 2 ml inj. Autin It is a plant glycoside claimed to reduce capillary bleeding. It has been used in a dose of 60 mg oral

BD- TDS along with vit C which is believed to fac ilitate its action. Its efficacy is uncertain. In CAD! PER-C 60 mg tab. Ethamsylate It reduces capillary bleeding when platelets arc adequate; probably corrects abnormal ities of platelet adhesion, but does not Mabili7c fibrin (not an antifibrinolyiic). Ethamsylate has been used in the prevention and treatmen t of capillary bleeding in mcnon-hagia, after abortion, cpistaxis. malena, hematuria and afier tooth extraction, but efficacy is unsubstantiated, therefore not recommended no\\. Side effects arc nausea. rash. headache, and foll in DP (only after i.,. injection). Dose: 250- S00 mgTDSoralli.,.; FTHAM YL, DICY ENE. HEI\ISYL, ALSTAT 250,500 mg tabs; 250 mg/2 ml inj.

LOCAL HAEMOSTATIC$ (STYPTICS)

After injury to arterioles and other smaller blood vessels, normal haemostasis occurs successively by contraction of injured vcs el wall (lasting few minutes). adhesion and aggregation of platelets to form a plug (primary haemostasis), activation of the coagulation cascade and formation of a blood clot (secondary haemostasis). Finally in due course dissolution o f the clot occurs by fibrino lysis. External bleeding is usually stopped by man ual pressure, cotton-gauze pressure pack or by suturing. Control of bleeding may be aided by local haemostatics (styptics) that are substances used to stop bleeding from a local and ap proachable site. T hey are particularly effective on oozing surfaces, e.g. tooth socket, abras ions, etc. Absorbable materials li ke fibrin (prepared from human plasma and dryed as sheet or foam). gelatin foam, oxidized cellulose (as stri ps which can be cut and placed in the wo und ). It provide a mes hwork which acti vates the clotting mechanism and checks bleeding. Left in s itu these materi als arc absorbed in I.-4 weeks and generally cause no foreign body reaction . Thrombin obtained from bovine plasma may be applied as dry powder or fre hly prepared solution to the bleeding surface in haemophiliacs. Vasoconstrictors like 0. 1% Ad r solution may be soaked in sterile cotton-gauze and packed in the bleeding tooth socket or nose in case of epistaxis to check bleeding when spontaneous vasoconstricti on

DRUGS AFFECTING COAGULATION, BLEEDING A N D THROMBOSIS

is inadequate. Astringents such as tannic acid or metallic salts are occasional ly a pplied for bleeding gums, bleeding piles, etc. SCLEROSING AGENTS These are irritants. ca use inflammation. coagulation and ultimately fibrosis, when injected into haemonhoids (piles} or varicose vein mass. They are used only for local injection. 1. Sod. tetradecyl sulfate (3% with benzyl alcohol 2%): 0.5-2 ml at each site. SETROL 2 m l inj. 2. Polidocanol (3% inj): 2 ml; ASKLEROL 2 ml inj.

ANTICOAGULANTS These are dn1gs used to reduce the coagulabil ity of blood. They are classified in the c hart.

HEPARIN McLean, a medical student, discovered in 19 I6 that liver contains a powerful anticoagulant. Howell and Holl ( 1918) named it ·heparin' because il was obtained from liver. However, it could be used clinically only in 1937 when su fficient degree of purification was achieved.

Chemistry and occurrence

Heparin is a non-uniform mixture of straight chain rnucopolysaccharides with MW I0,000 to 20,000. It contains polymers of two sulfated disaccharide units: D-glucosamine-L- } chain length and proportion iduronic acid of the two disaccharide units D-glucosamine-D- varies. Some glucosamine · 'd residues are -acetylated. g Iucuromc ac1

Heparin carries strong electronegative charges and is the strongest organic ac id present in the body. It occurs in mast cells as a much bigger molecule (MW ~75,000) loosely bound to the granular protein. Thus, heparin is present in all tissues containing mast cells; richest sources are lung, liver and intestinal mucosa. Commerc ially it is produced from ox lung and pig intestinal mucosa.

Actions

1. Anticoagulant Heparin is a powerful and instamaneously acting anticoagulanc, effecti ve both in vivo and in vitro. It acts ind irectly by activating plasma antithrombin lll (AT 11.l, a serine proteinase inhibitor). The heparin-AT Ill complex then binds to clotting factors of the intrinsic a nd common pathways (Xa, Ila, IXa, Xia, Xlla and X IJla) and inactivates them but not factor V Iia operative in the ex trinsic pathway. At low concentrations o f heparin, factor Xa mediated conversion o f prothrombin to thrombin is selectively affected. The anticoagulant action is exerted mainly by inhibition of factor Xa as well as thrombin (Ila) mediated conversion of fibrinogen to fibrin. Low concentrations of hepa rin prolong aPTT without sign ificantly prolonging PT. High concentrations prolong both. Thus, low concentrations interfere selectively with the intrinsic pathway,

ANTICOAGULANTS

Indirect thrombln Inhibitors

Heparin (unfractionated) Low m olecula r weight

heparins (Enoxaparin, Reviparin, Nadroparin, Dalteparin, Pamaparin, Ardeparin) F'ondaparinux Danaparoid

Direct thrombln Inhibitors

Bivalirudin Argatroban

Vit K antagonists

Bishydroxycoumarin (dicumarol) Warfarin sod. Acenocoumarol ( icoumalone) Ethyl-biscoumacetate

Direct factor Xa inhibitor

Oral direct thrombln Inhibitor

Rivaroxaban Apixaban

Dabigatranetexilate

663

664

DRUGS AFFECTING BLOOD A ND BLOOD FORMATI ON

a ffecting amplification a nd continuation of c lotting, whil e hi gh concen trations affect the common pathway as well. Antithrombin-11 l is itself a s ubstrate for th e protease c lotting fac tors; binds with the protease to form a stable co mpl ex (s uic ide inhibitor). However, in the absence or heparin, the two interact very slowly. Heparin enhances the action of AT-111 in two ways: (a) The lo ng heparin mo lec ul e pro v ides a scaffolding for the clotting factors (mainly Xa and Jla) on one ha nd and AT-Ill on the OLher, so that they inte ract wi th each other. (b) Heparin induces conformational c hange in AT-Ill to expose its interactive sites. A specific pentasaccharide sequence, which is present in only some of the hepa rin molecules, binds to AT Il l with high affinity to induce the conformational change needed for rapid interacti on with clotting factors. This has been synthesized and named

fondaparinux. Inhibition ofTTa requires both the mechanisms, but Xa inhibition can occur by mechanism ' b' alone. T his probably explains why low molecular weight heparin, which is insuffic ient to provide a long scaffolding, selecti vely inhibits factor Xa. Hi g her doses or heparin g iven fo r some time cause reduction in AT-111 levels, probably a compensatory phenomenon. Sudden stoppage of conventional-dose therapy may resu lt in rebound increase in coagulabi lity for few days.

2. Antiplatelet Heparin in higher doses inhibits platelet aggregation and prolongs bleeding ti me.

3. Lipaemia clearing

Injection of heparin clears turbid post-prandial Iipaemic plasma by re leasi ng a lipoprotcin lipase from the vessel wall and tissues, which hydrolyses triglycerides of chy lomicra and very low density lipoproteins to free fatty ac ids (FFAs). The FFAs then pass into tissues and the plasma looks clear. Th is action requires lower concentration of heparin than that needed for anticoagulation .

Facilitation of fatty ac id trnnspon may be the physiological funct ion of heparin; but since, ii is not found in circulating blood and its storage form in tissues is much less active, this seems only conjectural.

Pharmacokinetics I leparin is a large, highly ionized molecu le; therefore not a bsorbed ora lly. Injected i. v. it acts instantaneo usly, but after s.c. injection an ti coagula nt effect develops o ver - 60 m in. Bioavailability of s.c. heparin is inconsistent. He parin does not cross blood-brain barrie r or placenta (it is the anticoagulant of choice during pregnancy). Jt is metabolized in liver by heparinase and frag ments are excreted in urine. Heparin released from mast cells is degraded by tissue macrophages, and it is not a physiologically circulating anticoagulant. After i.v. injection of doses < 100 U/kg, the t½ averages I hr. Beyond this, dose-dependent inac ti vati on is seen and pharmaco kineti cs is in co ns is te nt; t ½ is pro lo n ged to 1- 4 hrs. The t½ is longer in c irrho tics and kid ney failure patients, a nd shorter in patients with pulmonary embolism . However UFH is safer than LMW heparins or fondaparinux in kidney failure patients. Unitage and administration Because of variable molecular size of unfractionated heparin (UHi), it is standardized only by bioassay: I U is the amount of heparin that will prevent I ml of citrated sheep plasma from clotting for I hour after the addition of 0.2 ml of I% CaCl2 solut ion. 1leparin sod. I mg has 120-140 U of activity. Hl:.PARl'\I SOD., BEPARJ E, NUPARIN l000 and 5000 U/ ml in 5 ml vials for injection. A quick penetrat ing so lution (QPS) form u latio n o f' heparin l000 u/ml has been recently developed for topical application to speed up healing o f post-infusion superficial thrombophebitis. PHLEBOTROY QPS 1000 wml topical solu110n; 5 ml bollle; apply on the lesion 3 times a day. Heparin should not be mixed with penici llin, tetracyclines, hydrocorti sone or A in the same syringe or infusio n bottle. Heparinized blood is not s uitable for blood counts (alters the shape of R.BCs and WBCs), fragility testing and complement fixation tests. Dosage Heparin is conventionally g iven i.v. in a bolus dose of 5,000- 10,000 U (children 50-100 U/ kg), followed by continuous infusion of 750 1000 U/hr. ln1ermi11en1 i.v. bolus doses o f UFH are no longer recommended. The rate of in fusion is controlled by a PTT measurement which is kept at 50-80 sec. or 1.5-2.5 times the patient's pretreatment value. Alternatively ant iXa activity or protamine titration may be used. If these tests are not available, whole blood clo11ing time should be measured and kepi at - 2 times the normal value.

DRUGS AF FECTIN G COAGULATION, BLEEDING AND THROMBOSIS Haematomas are more common with i.m. injection- this route s hou Id not be used.

Low dose (s.c.) regimen 5000 U of UFH is injected s.c. every 8 12 hours, started before surgery and continued for 7-10 day, or till the patient starts moving about. This regimen has been found to prevent postoperative deep vein thrombosis "ithout increasing surgical bleed ing. IL also does not prolong aPTI or clotting time. However, it should not he used in case of neurosurgery or when spinal anaesthes ia is to be given. The patie nts should not be receiving aspirin or oral anticoagulants. This regimen is ineffective in high-risk s ituations, e.g. hip joint or pelvic s urgery.

Adverse effects I. Bleedi ng due to overdose is the m ost serious complication of heparin therapy. Since heparin (and other a nticoagulants) interfere with secondary haemostasis, bleeding from deeper organs is more common. Haematuria is generally th e first sign; other sites arc g. i. tract, brain, joit~ts, muscl:s, etc. With proper monitoring, serious bleedtng occurs only in 1- 3% patients. 2. Heparin-induced thrombocytopenia {HIT) is another common problem. Genera lly it is mild and tran sie nt; occurs due to aggregat ion of platelets. Occasionally serious thromboembolic events res ult. In some patients antibodies a re formed to the heparin-p latelet complex and marked depletion of plate lets occurs- heparin should be di scontinued in suc h cases. Even low mo lec ular weight (LM W) heparins are not safe in such patients, and they should be treated with a direct thrombin inhibitor. 3. Transient and reversible alopecia is infrequent. Serum transaminase leve ls may rise. 4. Osteoporosis may develop on long-term use of relatively high doses. 5. Hyperse ns itivity reactions are rare; manifestations are urticaria, rigor, fever and anaphylaxis. Patients with allerg ic diathesis are more liable.

Contraindications I. Bleeding disorders, history of HIT. 2. Severe hypertens ion (risk of cerebral haemorrhage), threatened abortion, piles, g.i. ulcers (ri sk of aggravated bleeding). 3. Subacute bacterial e ndoca rditi s ( ri sk of embolism), large malignancies (risk of bleeding

in the central necrosed area of the tumour), tuberculosis (ri sk of haemoptysis). 4. Ocular and neurosurgery, lum bar puncture. 5. Chronic alcoholics, cirrhosis, renal fai lure. 6. Aspirin and other antiplatelet drugs should be used very cautiously during heparin therapy.

Low molecular weight (LMW) heparins He parin has been frac t ionated into LMW forms (MW 3000- 7000) by d ifferent techniques. LMW heparins have a diffe rent anti coagulant profile; i.e. they selecti vely inhibit factor Xa with liule effect on Ila. They act onl y by inducing conformational chan ge in AT Ill and not by providing a scaffold ing for interaction of AT Il l with thrombin. As a result, LMW heparins have smaller efTect on aPTT and whole blood c lotting time than unfractionated hepari n (U FH ) re lative to antifactor Xa activ ity. Also, they have lesser antiplatelet action- less interference with haemostas is. Thrombocytopenia is less frequent. A lower incidence of haemorrhagic co mplications co mpared to UFH has been re po rted in so me studies, but not in others. However, major bleeding may be less frequent. The LMW heparins are e liminated primarily by renal excretion; are not to be used in patients with renal fa ilure. The more important advantages of LMW hepari ns are pharmacokinetic: • Better subcutaneous bioava ilability (70-90%) compared to UFH (20- 30%): Variability in response is minim ized. • Longer and more consistent monoex ponential t½ (4- 6 hours); making possible once dail y s.c. administration. • Since aPTT/clotting times are not prolonged, laboratory monitoring is not needed; dose is calculated on body weight basis. • Ris k of o teoporosis after lo ng term use is muc h less with LMW heparin compared with UF I I. Most studie s have found LMW heparins to be equally efficacious to UFH except during cardiopulmonary bypass surgery, in which hi gh dose UF H is sti ll the prefe rred anticoagulant, beca use LMW heparin a nd fondaparinux are

665

666

DRUGS AFFECTING BLOOD AND BLOOD FOR M ATION less effective in preventing catheter thrombosis. Moreover, effects arc not full y reversed by protamine. Indications of LMW heparins are: I. Pro phy laxis of deep vein throm bosis ( DVT) a nd pulmonary embolism (PE) in high-risk patients undergoing surgery; stroke or other immobil ized patients. 2. Treatment of established DVT. 3. Unstab le angina ( UA) and M l: they have largely replaced continuous infusion o f UFH. 4 . To maintain patency of cann ulae and shunts in dialysis patients. A number of LM\V heparins have been marketed. They differ in composition, phannacokinctics and dosage. Enoxaparin: CLEXANC 20 mg (0.2 ml) and 40 mg (0.4 ml) prcfilled syringes; 20-40 mg OD, s.c. (start 2 hour before surgery). Reviparin: CLI VA RI NE 13.8 mg (eq. lO 1432 anti Xa IU) in 0.25 ml prctillcd s:i,nngc; 0.25 ml s.c. once daily for 5-10 days. Nadroparin: FRAXIPARI I' 3075 IU (0.3 ml) and 4100 IU (0.4 ml) inj .. CARDIOPARfN 4000 anti Xa IU10A ml, 6000 anti Xa IU/0.6 ml, 100,000 anti Xa llJ/10 ml mj. Dalteparin: 2500 IU OD for prophylaxis: 100 U/ Kg 12 hourly or 200 U/Kg 24 hourly for treatment of deep vein chrombosis. FRAGMIN 2500, 5000 IU prclilled S) ringes. Parnaparin: 0.6 ml s.c. OD for unstable angina and prophylaxis of DVT; fL UXUM 3200 11.J {0.3 ml), 6400 IL, (0.6 ml) inj. Ardeparin: 2500-5000 IU OD; INDEPARI N 2500 IL,, 5000 I U prcfi Iled syringcs.

Fondaparinux The pentasaccharide w ith the s pecific sequ ence that b inds to AT- Ill with high affinity, induces an irreversible conformmational change to selecti vely inactivate factor Xa without binding Ihrombin (factor Ila), has been produced syntheti call y, given the name fondaparinu.x, and is be ing increasingly used now. The bioavailability of fondaparinux injected s.c. is I00%, peak effect is produced in 2 hours and it is longer acting (t½ 17 hours). Metabol ism is minimal, and it is largely excreted unchanged by the kidney. As such, it is not to be used in renal failure patie nts. Fondapari nux is less likely to cause thrombocytopenia compared to even LMW heparins. Risk of bleeding is also less and incidence of osteoporosis after prolonged use is minima l. Fondaparinux does not require laboratory monitoring of a PTT, and is

a longer acti ng alternative to LMW heparins with the above advantages. It is indicated for prophylaxis and treatment of DVT and PE, as well as in acute corona ry syndromes (ACS) when immediate intervention is not p lanned. Dose: Prophylaxis of DVT/PE in onhopedic surgery and medical patients 2.5 mg s.c. OD, Treatmcni of DVT/PE 5-10 mg s.c. OD. FO'\JDA PARINUX, ARIXTRA 5 mg/0.4 ml, 7.5 mg/0.6 ml and 10 mg/0.8 ml prefillcd smgle dose syringe. Danaparoid is a preparation con taining mainly heparan sul fate which is a heparin-like substance found in many ti~sucs. ha\ ing less potent anticoagulant action than heparin. Danaparoid is obtained from pig gut mucosa. and is used in cases \\ ith heparin-induced thrombocytopenia.

HEPARIN ANTAGONIST

Protamine sulfate

It is a strongly basic, low molecular we ight protein obtained from the sperm of certain fish. G iven i.v. it neutralises heparin mg for mg. i.e. I mg is needed fo r every I 00 U of heparin. For the treatment of heparin induced bleeding, due conside rat io n must be given to the amount of heparin that may have been degrade d by the patient's body in the mean time. However, it is needed infrequently because the action of hepari n disappears by itself in a few hours, and whole blood transfusion is needed to repleni sh the loss when bleeding occurs. Protamine is more commonl y used w he n heparin action needs to be terminated rapidly, e.g. after cardiac or vascu lar surgery. Protamine does not neutralize fondaparinux, and it only partially reverses the anticoagulant effect o f LMW hepari ns. In the absence o f heparin, protamine itself acts as a weak anticoagulant by interacting with platelets and fibrinogen. Bei ng basic in nature it can re lease histamine in the body. Hypersensiti vity reactions have occurred. Protamine should be injected slowly (5 mg/min i. v.). Rapid i.v. injection causes flushing and breathing diffi culty. PROTA, PROTAM INE SULFATE 50 mg in 5 ml inj .

DIRECT THROMBIN INHIBITORS (DT ls) U nlike heparin. these recently deve loped anticoagulants bind directly to thrombin a nd

DRUG S AF FECTIN G COAGULATION, BLEED ING AND TH ROM BOSIS inactivate it without the need to combine with and activate AT-ll I.

Bivalirudin It is a synth eti cally prepa red 20 amino acid peptide conge ner of the larger polypeptide anticoag ulant hirudin sec reted by the sali vary glands of leech. Biva li rudin b inds firml y to the catalytic as we ll as the s ubstrate recognition s ites of thrombin and inhibits it directl y. Thus, it does not need AT- 111 to exert its action. Circulating as well as clot-bound thrombin is inhibi ted. However, it is a subs trate as well for thrombin , which s lowl y cleaves its peptide bonds, and t he actio n reverses gra dua ll y. Bi va lirudin is cleared from the plasma by both proteolysis as we ll as renal excretion, confcring a short biological t½ of 25 min in subjects wi th no rmal renal function. Thus, it is a s pecific a nd reversible DT I wi th quick onset a nd s hort las ting anticoagulant action, w hi ch is more predictable and consis tent than that of UF H. How ever, protamine does not antagoni ze its action and th ere is no s peci fie antidote o f bivalirudin. Th rombin mediated platelet aggregation is also inhibited. There is no risk o f HIT w ith biva lirudin, and this probobly is its m ost significant feature. Biva lirudin is indicated as an anticoagulant in patients undergoi ng percutaneous coronary intervention (PC!) for STEMI. It is to be used along with an antiplatclet drug viz. a pirin and/ or c lopidogrel or a glycoprotein llb/ 11 la inhibitor (G Pl). It can also be used in unstable angina (UA) and NSTEMI w hen urge nt o r early PC ! is planned. It is specificall y o f va lue in patie nts at risk of HIT. Dose: 0. 75 mg/kg bolus 1.v. injection followed by 1. 75 mg/kg/hour i.~. inrusion for upto 24 hours. Continuation of i.,. infusion 4 hours after the PCI is optional. BIVAFLO, BIVASTAT. BIVASAVL 250 mg\ial inj.

Apart from ri s k of bleeding. the adverse e!Tects are headache, back pain and hypotension. Bivalirudin has been evaluated in multiccntnc clinical trials involving >25000 patients undergoing PC I. and has demonsuated similar angiographic, and procedural outcomes as with UFJ I + a GPI. Rates or stcnl thrombosis have also been similar.

667

Argatroban This is a synthetic non peptide compound which binds reversibly 10 the catalytic site ofthrombin. but nol Lo the substrate recognition site, and acts as a DTI. Administered by i.,. bolus (350 µg/kg over 3-5 min) followed by 25 µg/kg/min infusion. it produces a rapid onset short-lasting antithrombin action. Argalroban prolongs aPTT. and its dose is regulated b) measunng I R. As such. care needs to be exercised "hile sh fiing patients 10 "arfarin prophylax is. II is indicated for prophylaxis and treatment of thrombosis and for PCI in patients with HIT or in those who arc al risk ofdeveloping I IIT. Argatroban is cleared by liver with a biological t½ of 45 min. Thus, it can be given 10 patients with renal disease.

ORAL ANTICOAGULANTS VITAMIN K ANTAGONISTS A haemorrhagic disease was described in canlc in 1924 which was due 10 !ceding them on spoi led sweet elm er hay. T he disorder Wa!> found to be due lo prothrombin deficiency and the toxic prmciple \\al, identified as hishyd1vxycou111ari11 in I 939. II "as cured hy feeding a lfalfa grass. First clinical use of bishydroxycoumann was made in 1941 and many congeners were added later. Wa,farin was initially used as rat poison; demonstration of its safety led to clinical trial; ii is no\\ the principal coumarin oral anticoagulant.

By convention, the term 'Oral anticoagulant' re fe rs to the warfarin -like drugs which act by blocking synthesis of clotting factors. I lowever late ly, severa l orally acti ve direct factor Xa inhibitors and direct thrombin inhibitors have a lso become avai lable, and are increasingly used.

Action and mechanism Warfarin and its congeners act as anticoagulants only in vivo. not in vitro. This is so because they act indirectl y by interfering with the synthesis of vit K dependent clotting factors in li ver. They apparently behave as competiti ve antagonists of vit K and lower the plas ma leve ls of fun ctional c lotting factors in a dose-dependent ma nne r. ln fact, they inhibit the enzyme vit K epox ide reductasc (Y KOR) and interfere with regeneration of the acti ve hydroquino ne form of vit K (Fig. 45.2) which acts as a cofactor for the enzyme y-glutamyl carboxylase that carries out the final step of y carboxy lating glutamate res idues of prothrombin and factors VII, IX and X. This carboxylation is essential for the ability o f the c lotting factor to bind Ca1+ and to get bound to phos pho l ipid s urface s, necessary for the coagulation sequence to proceed.

668

DRUGS AFFECTING BLOOD AND BLOOD FO RM ATI ON

Descarboxyprothrombin (or f. VII, IX. X)

The therapeutic effect occurs when synthesis of clotting factors is reduced by 40-50%.

Prothrombin (or f. VII, IX, X) y-glutamyl carboxylase

Vit K hydroquinone

Vit K epoxide

NAO

t

NADH

Blocked by coumarin anticoagulants

Fig. 45.2: Mechanism of action of coumarin anticoagulants NAD-Nicotinamide adenine dinucleotide; NADH-its reduced form

Factor V I I has the s hortest plasma t ½ (6 hr), its level fa lls first when warfarin is given, followed by factor IX (t½ 24 hr), factor X (t½ 40 hr) and prothrombin (t½ 60 hr). Though the synthesis of clolling factors diminishes within 2- 4 hours of warfa rin admin istration, anticoagulant effect develops grad ually over the next 1-3 days as the levels of the clotting factors already present in pla ma decline according to their half li ves. Thus, there is always a delay betv,een administration of coumarin drugs and the anticoagulant effect. Larger initial doses hasten the effect only slightly.

Protein C. protcm S (both having anticoagulant property) and osteocalcin contain glu1amate residues that require vit. K dependent y carhoxyla1ion. These are also inhibited by coumarin anticoagulants. I lowever, effect or decline in levels or protein C and protein S is overshadowed by the decline in the le vels or cloning factors. Whi le osteocalcin defi cit may depress new bone formation, dcnsily of adult bone is little affected.

The diJTerences betwee n di ffe rent coumarin vit. K antagonists are primari ly pharmacokinetic and in the adverse side effects produced by them. These are summarized in Table 45. 1.

Racemic Warfarin sod. It is the most popu lar coumarin anticoagulant. The commercial preparation of warfa rin is a mixture of R (dextrorotatory) and S (levorotatory) enantiomers. The S fo rm is more potent and is metabolized relatively faster by ring oxidation carried out by CYP2C9, while R fo rm is less potent and degraded by side chain reduction carried out by CYP I A and CY P3A4 . Both are partial ly conj ugated with glucuronic acid and undergo some enterohepatic circulation; finally excreted in urine. Warfarin is rapidly and completel y absorbed fro m intestines and is 99% plasma protein bound. It crosses placenta and is secreted in milk: however, quantity of acti ve form is generally insufficient to affect the suckling infant.

Table 45.1: Pharmacokinet1c and adverse effect profile of v1t K antagonist oral anticoagulants Drug

1.

Bishydroxycoumarin

t½ (hour)

25-100

Duration of action (days)

Dose (mg)

Adverse side effects (other than bleeding)

Loading

Maintenance·

4-7

200 for 2 days

50-100

Frequent g.i.t. disturbances

(dose dependent)

2.

Warfarin sod.

36-48

3-6

5-10

2-1or

Alopecia, dermatitis, diarrhoea

3.

Acenocoumarol (Nicoumalone)

18-24

2- 3

8-12

2-8

Oral ulceration, g.i.t. disturbances, dermatitis, urticaria, alopecia

4.

Ethylbiscoumacetate

2

1-3

900

30o-600

Alopecia, bad taste

• Daily maintenance dose: lo be adjusted by measurement of prothrombin time (INR). r To be taken in a single dose at the same hour (usually bed time) each day.

669

DRUGS A FF ECTING COAGULATION, BLEE DI N G A N D THRO MBOSIS

U'IIWARFIN I, 2, 5 mg tabs; WARF-5: 5 mg tab.

Bishydroxycoumarin (Dicumarol)

It is slowly and unpredictably absorbed orally. Its metabol ism is dose dcpendent- t½ 1s prolonged at higher doses. I las poor g .i. tolerance; not pre ferred now.

Acenocoumarol (Nicouma/one)

The t½ of acenocou ma ro l as s uc h is 8 hours, but a n active metabolite is produced so that overall t½ is a bout 24 hours. Acts more rapidly.

testing laboratory) has been defi ned fo r vari ous ind icati ons. But this va lue di fTe rs depending on the source of the thromboplastin (T p) that has been used for the tes t. A standa rd ized syste m called the Interna tio nal ormalized Ratio ( I R) based on the use of human brai n Tp has been deve loped by W HO and adopted in a ll countries. Recommended INR for various indications of oral anticoagulants

ACITRO\1 0.5, I, 2. 4 mg tabs.

Ethyl b1scoumacetate

It has a rapid and brief action; occasionally used to initiate therapy. but difl'icult. to maintain.

Adverse effects

Bleed ing as a res ult o f ex tension o f th e d es ired pha rmaco logic al action is the most important problem causi ng ecchymosis, epistaxis, hematuria, bleedi ng in the g. i.t. lntracranial or other internal haemo rrhages may even be fa tal. Bleeding is more li ke ly if therapy is not properly monitored, or whe n the Inte rnationa l normalized ratio (I R) exceeds 4, or interacting drugs/contraindications are present.

Treatment: of bleeding due to vit K antagonists consists o f: • Withho ld the a nticoagula nt. • Give fres h blood tran fusion; this su pplies ready made c lotting facto rs and replenishes lost blood. Alternatively fres h frozen plasma may be used as a source o f c lotting factors. • Gi ve v it Kt w hic h is the specific antidote (see p. 66 1-62). but it takes 6--24 hours for the c lotting factors to be resynthesized and re leased in the blood after v it K admi nistration. Ad verse e ffects unre lated to anticoagulat ion are given in Table 45.1. Cutaneous necrosis is a rare complication that can occur with any coumarin an ticoagula nt. Wa rfarin and acenoco umarol are considered to be the most sui table and better tolerated drugs.

Dose regulation The dose ofvit K antagonist oral ant icoagul a nt must be ind ividua lised by repeated measure ment of prothrombin time; the a im is to achieve a thera peutic e ffect without unduly increasing the chances of bleeding. The optimum ratio of PT duri ng treatment with the ora l anticoagulant to the normal val ue (of the

INR

1. Prophylaxis of deep vein thrombosis and similar indicalions

2-2.5

2. Treatment of deep vein thrombosis, pulmonary embolism, TIAs, hip surgery

2-3

3.

3-3.5

Recurrent thromboembolism, arterial disease (Ml), prosthetic heart valves

Factors enhancing effect of coumarin a nticoagulants arc: • De bil ity, ma ln utr ition , ma la bsorpt ion and prolonged antibiotic therapy: the supply of vit K to the liver is reduced in these conditions. • Liver disease, chronic alcoholism: synthesis of clotting factors may be defi c ie nt. • Hy perthyro idi sm: the c lott ing fac to rs a re degraded fas ter. ewborns: have low levels of vit K a nd clotting factors (there should be no need of these drugs in neonates anyway). Factors decreasing effect o f co urn a ri n anticoagulants are: • Pregnancy: plasma level of clotti ng factors is higher. • Nephrotic syndrome: drug bound to plasma prote in is lost in urine. • Genetic warfarin resistance: the affi nity o f warfari n (as well as that of vit K epoxidc) to bind to the reductase (Y K OR) enzyme, whic h generates the active vit K hydroqu inone, is low. Dose of ora l anti coagulant is 4-5 times highe r. Contraindications

Al l contraind ications to heparin (see p. 665) apply to these drugs

670

DRUGS AFFE CTIN G BLOOD AND BLOOD FORMATION

as we ll. Factors whi ch enhance the effect of coumari n anticoagulants (see above) should a lso be taken into consideration. Coumarin oral anticoagulants should not be used during pregna ncy. Warfari n given in early pregna ncy inc reases birth defects, especia lly ske letal abnorma liti es. It c an produce foetal warfarin syndro111e- hy po plasia of nose, eye socke t, ha nd bones, and growth retardation. Given late r in p regnancy, it can cause C S defects, foetal hae mo rrh age, foetal death and accenlllatcs neonata l hypoprothrombinem ia.

Drug interactions

A large nu mber of d rugs interact with these oral anticoagulants at pharmacokineti c or pharmacodynami c level, and e ither e nhance or decrease the ir e ITect. T hese interactions are c linically important (may be fata l if bleeding occurs) and may involve more than one mechanism ; the exact mechanism of an interaction is not always defi nable.

A. Enhanced anticoagulant action I. Broad-spectrum antibiotics: inh ibit gut flora and reduce vit K production. 2. Some cepha lospori ns (ceftriaxone, cefoperazone) cause hypoprothrombinaemia by the sa m e mechanis m as warfarin p roduce additi ve acti on. 3. Aspi rin: inh ibits plate let aggrega ti on and causes g. i. bleeding- this may be hazardous in a nti coagulated patie nts. High doses of sal icylates have synergistic hypoprothrom bincmic actio n and a lso displace warfarin fro m protei n binding site. 4 . Long acting sulfona m ides, ind ometh ac in, phenytoin and probenecid: di splace warfarin from plasma protein binding. 5. Ch loram phenicol. erythromycin. cclecoxib, c imetid ine, a llopurin o l, amiodaro ne a nd metronidaz ole: inhibit warfarin metabolism. 6 . Tolbula mi de and phenytoin: inh ibit wa rfarin metabolism and vice versa. 7. Liqu id para ffin (hab itua l use): reduces vii K absorption.

B. Reduced anticoagulant action I. Barbi turates ( b ut not be nzo diazcpi nes), ca rba mazepinc, rifa mpi n and griseofulvin

induce the metabo lism of warfarin. T he dose of anticoagula nt dete rm ined during the rapy wi th these drugs would be hi ghe r: if the sa me is continued a fter w ithd raw ing the ind ucer- marked hy poprothrombine mia can occur- fatal bleeding is on record. 2. O ra l contraceptives: increase blood levels of clott ing facto rs. T he important features of heparin and warfa rin arc compared in Table 45.2.

DIRECT FACTOR Xa INHIBITORS Warfa rin has been the standard prophy lactic maintcnace oral a nticoagu la nt for the past 5 decades. However, there are several problems in its use, viz. delayed onset of action, s low recovery on s toppage, narrow therapeuti c range, need for repeated laboratory moni toring and dose a dj ustm en t as well as n umerous drug interactions. To overcome these, some oral di rect fac to r Xa inhi bitors a nd ora l di rect thrombin inhi bitor (oDT I) have been late ly introduced. These drugs act rapid ly without a lag ti me. and have short-lasti ng action, but there is no antidote.

Rivaroxaban It is an ora lly ac ti ve d irect inh ibi tor of activated facto r Xa which has become avai lable fo r prophy laxis a nd treatme n t of DVT. Its ant icoagulant actio n develops ra pid ly with in 3 4 hours of ingestion and lasts for ~24 hours. It is large ly me ta bo lized, but also excreted unchanged in urine; plasma t½ is 7- 11 hours. Another advantage is that it requ ires no laboratory monitoring of PT or a PTT. Rivaroxa ban is now routinely used in a fixed dose of IO mg o nce dai ly starting 6- 10 hours a fte r surgery for prophy lax is of veno us throm boembo lis m following total knee/ hip replacement. Duration of prophy lax is is 12 14 days after knee replacement and 35 days after hip replacement. In comparative tria ls, its efficacy has been found simi lar to a regimen of LMW hepari n followed by warfa rin. Ri va roxaban has a lso be en found eq ua lly effective as ·warfarin for preventing stroke in

DRUGS AFF ECTI N G CO AG ULATION, BLE ED I N G A ND T HROM BOSIS

Table 45.2: Some comparative aspects of heparin and warfann Heparin

Warlarin Coumarin derivative

1.

Chemistry

Mucopolysaccharide

2.

Source

Hog lung, pig intestine

Synthetic Oral

3.

Route of admin.

Parenteral (i.v., s.c.)

4.

Onset of action

Immediate

Delayed (1- 3 days)

5.

Duration of action

4---6 hrs

~days

6.

Activity

In vitro and in vivo

7.

Mechanism

Blocks action of factor X and thrombin

Inhibits synthesis of clotting factors

8.

Antagonist

Protamine sulphate

Vil K

9.

In vivo only

Variability in response

Limited

Marked

10.

Lab. control

aPTT/clotting time (desirable)

Prothrombin time/l NR (essential)

11 .

Drug interactions

Few and not significant

Many and significant

12.

Use

To initiate therapy

For maintenance

patients w ith atria l fi brillation. For the treatment of DVT and PE, it is recommended in a dose of 15 mg twice dail y for 3 weeks fol lowed by 20 mg/day. It is now also approved in a dose of 2.5 mg/day for coadministration with aspirin and clopidogrel for prophylaxis of ACS. Side effects re porte d are b leeding, nausea. hypotension, tachycardia and edema.

major bleeding was a lso noted. Apixaban is approved for: • Proph y lax is of VTE fo llowi ng knee/ hi p replacement (dose: 2.5 mg BO) • Prophylaxis of stroke in AF patients (dose: 5 mg 80). • Treatment of D VT a nd PE (dose: IO mg BO for 7 days followed by 5 mg 8D).

XARELfO 2.5 mg. 10 mg. 15 mg, 20 mg tab,.

ELIQUIS 2.5 mg. 5 mg tabs.

Apixaban This is another oral facto r Xa inhibitor having properties and indications similar to rivaroxaban. The oral bioava ilabili ty of apixaban is 85% and peak effects a re produced in 3 hours. It is partl y metabolized by C YP3A4 and el im inated in both faeces as we ll as uri ne. The plasma t ½ is 12 hours. Drng interactions w ith inducers and inhibito rs o f CYP3A4 are possible, and it should not be used in patients with hepati c or renal impa irment. Efficacy of apixaban in the prevention and treatment of venous thromboembolism (VT E) as well as risk of major bleeding appears to be sim ilar to that of rivaroxaban and dabigatran. A lower rate of stroke and systemic embolism has been reported w ith apixaban compared to warfarin by the A RI STOTLE tria l conducted on > 18000 patients of AF. A lower risk of

Advantages of newer oral anticoagulants (direct factor Xa/thrombin Inhibitors) over warlarin • Rapid onset and offset of therapeutic effect • Short half life • No laboratory monitoring required • Fixed dosage guidelines depending on the indication • Antithrombotic efficacy equal to/better than warfarin. • Lower risk of bleeding compared to warfarin • Fewer drug interactions.

ORAL DIRECT THROMBIN INHIBITOR Dabigatran etexilate It is a prodrug which after oral admin istratio n is rapidl y hyd rolysed to dabigatran. This direct thrombin inhib ito r.

671

672

DRUGS AFF ECTI NG BLOOD AND BLOOD FORMATION reversibly blocks the catalytic site o f thrombin and produces a rapid (within 2 hours) anticoagulant effect. Though oral bioavai labil ity is low, the anticoagulan t effect is consistent, and no laboratory monitoring is required. The bioava ilabil ity is increased by P-gp (efflu x transporter) inhibitors (verapamil, amiodarone, etc.) and dose reduction is needed in patients taking th ese d ru gs. D abi gatran i s excreted by the k idney. No interaction w i th C YP3A4 inhibitors/inducers occurs. Plasma t½ is 14 hours and duration of action 24 hours. It is approved for preventi on of VTE following hip/k nee joint replacement surgery. Admi nistered in a dose of 110 mg (75 mg for elderl y > 75 years) once daily, it has been found comparable to war farin. In another large trial dabigatran etexilate 150 mg tw ice daily has y ielded superi or results to warfarin for prevention o f embolism and stroke in patients of atrial fi bri l lation, and has been approved for this indication as wel l. The incidence of major and mi nor bleeding was also less w ith

dabigatran compared to warfarin. However, like any anticoagulant, bleeding is the most important adverse effect . Dyspepsia is a fraquent side effect; hepatobiliary disorder is occasional. PRADAXA 75, 11 0 and 150 mg caps. A monoclo na l antibody idarucizumab has been developed as a reversal agent for dabigatran.

Notes on clinical use of anticoagulants Use of anticoagulants can be l ife saving. They have proven prophy lactic value, but can be risky (may provoke fatal bleeding). T heir use requires grea t caution and expertise. Cond itions in w hich th ey are indicated are summarized i n the box. Some notable points about use of anticoagulants are highlighted below. I . T he aim o f usi ng anti coagulants is to prevent th rombus extension and embolic comp I ications by reducing rate of fibrin formation. They do not d issolve already formed clot, but prevent recurrences.

Choice of anticoagulants for various clinical indications Clinical indications

Drugs utilized

1. Prophylaxis of VTE after knee/hip replacement

Rivaroxaban/apixaban/dabigatran/LMW heparin/ Fonda./low dose s.c. UFH

2. Prophylaxis of DVT and PE in high risk, bed-ridden medicaVsurgical patients

Fonda./LMW heparin/rivaroxaban/UFH followed by warfarin

3.

Treatment of DVT and/or PE

LMW heparins/fonda./UFH (kidney disease patients)/bivalirudin (if risk of HIT) followed by warfarin or rivaroxaban

4.

Prophylaxis of stroke and systemic embolism in nonvalvular AF

Rivaroxaban/apixaban/dabigatran/warfarin

5 . Primary PCI in patients of STEMI or in high risk cases of NSTEMI/UA

LMW heparins/UFH/bivalirudin (if risk of HIT)

6. Along with aspirin ± clopidogrel to prevent reocclusion following fibrinoly1ic therapy of STEMI/ high risk NSTEMI

UFH followed by warfarin

7.

To maintain patency of intravascular cannulae/ catheters

8. To preserve clotting factors in defibrination syndrome (disseminated intravascular coagulation)

Heparin flushes every 4-8 hours UFH

VIE- Venous thrombocmbolism; DVT-Dccp vein thrombosis; PE-Pulmonary embolism; LMW- Low molecular weight; UFH-Unfractionated heparin; Fonda- Fondaparinux; AF-Atrial fibrillation; PClPercutaneous coronary intervention; STEMI-5T segment elevation myocardial infarction; NSTEM1-Non5TEMJ; UA- Unstable angina; HIT- Heparin induced thrombocytopeni a

DRUGS AF FECTI NG COAG ULATION, BLEEDING AND THROM BOSIS 2. Parentera l a nticoagulants, viz. UF H, LM W heparins, fondapa rinux are util ized fo r rapid and short-term action, while oral anticoagulants warfarin, factor Xa inh ibitors (f.Xals) and oral DTls a re suitable for ma intenance the rapy. 3. Anticoag ula nts a re m ost e ffec t ive for preve ntion and treatment of VT E, beca use venous thrombi arc main ly fi brin thrombi. 4. Ora l f. Xa ls and DTls have surpassed low dose s.c. heparin and LMW heparins fo r prevention of VT E in post-kneen1ip replacement surgery patients. 5. LMW he parins a nd fo nd a pa rinu x have a lmost replaced U FH, except in kidney disease patients, because clearance of UFl-1 is a ffected to a lesser ex tent by renal fai lure. U FH is a lso preferred as prophylactic in major surgery and high risk cases, because its action can be rapid ly reversed by protam ine, which ne utra lizes LMW hepar ins to a lesser extent, and does not neutral ise fondaparinux . 6. A nt icoagulants arc of limited and inconsistent benefit in arterial thrombosis, because arteria l thrombi are mainly composed o f plate lets. The ir use in Ml and UA is short term and secondary lo that of antiplatelel drugs. 7. O ra l anticoagula nts (warfarin , f. Xa ls, DTI) a re more effect ive than antiplatelet drugs fo r prevent ing stroke in nonva lvular AF cases. 8. Anticoagulants are o f little value in thrombotic stroke. Neurological seque lac are s im ila r w hether

T he se a re d rugs used to lyse thromb i/ c lot to recanali ze occluded blood vessels (mainly coronary artery). T hey are therape utic rathe r than prophylacti c and work by activating the natura l fibri no lytic system (Fig. 45.3). Haemostatic plug of platelets formed at the s ite of injury to blood vessels is reinforced by fibri n deposition to form a t hrombus. Once repai r is over, the fi brinolytic system is activated to remove the fibrin. T he ~nzyme responsible fo r digesting fibrin is a serine protease Pla,min generated from plawninogen by tissue pla~minogen activator (t- PA), which is produced primarily by vascular endothelium. Plasminogcn circulates in plasma as well as remain s bound to fibrin. The t- PA selectively activates fibrin-bound pla~minogen within the thrombus, and any plasmin that leaks is inactivated by circulating antiplasmins. Fibrin bound plasmin is not inactivated by antiplasmins becau e of common binding s ite for both fibrin and amiplasmin.The t-PA itself is inactivated by plasrninogcn activator inhibitor- I and -2 (PA I- I, PAl-2).

l

Factor Xlla Kallikrein t-PA ~ P A l-1

r - - ACTIVATORS~ Alteplase (rt-PA) Tenecteplase

___i___ _

EACA . L _ INHIBITORS _ Tranexaem1c acid, EXTRINSlC

FIBRINOLYTIC DRUGS (Thrombolytics)

NIRINSlC

EXTRINSIC

PLASMINOGEN (Profibrinolysin)

they are given or not. Moreover, in the initia l stages it is difficu lt to ru le out haemorrhagic stroke. However, they may be given in cerebra l embolism because showers o f e mbo li are often recurrent, and can be prevented by anticoagulants. Anti plate let drugs are superior to anticoagulants in transient ischaemic attacks (TIAs). 9. T he parentera l DTI bivalirudin is specifica lly indicated in patients with HIT, or those w ho are at risk of developing HIT.

__j

u 2 Ant,plasmm u 2 Macroglobuhn

FIBRIN (Insoluble)

l

FIBRIN FRAGMENTS (Soluble)

INTRINSIC

Fig. 45.3: The plasminogen-plasmin system I-PA-Tissue plasminogen activator; rt-PA- Recombinant t-PA; PAl-1-Plasminogen activator inhibitor-1

673

674

DRUGS AFFECTING BLOOD AND BLOOD FORMATIO N When excessive amounts of plasminogen are activated ( by adm ini ste red fibrinolytic s), the o , antiplasmin is exhausted and active plasmin persists in piasma. Plasmin is a rather nonspecific protease: degrades coagulation factors (including fibrinogen) and some other plasma proteins as well. T hus, activation of circulating plasminogen induces a lytic s tate whose major complicat io n is haemorrhage. Even sdectivc activation of thrombus bound plasmin can cause bleeding by dissolving physiological thrombi.

In general, venous thrombi are lysed more easily by fibrinol yti cs than arteria l, and recent thromb i respond better. Fibrinolytics have little effect on thro mbi > 3 days old. The fi brinolytic drugs are: Strcptokinase Alteplase (rt- PA) Uroki nase Reteplase Tenecteplase Streptokinase (Stk) Obtained from 13 haemolytic S11·eptococc1 group C, it is the first fibrinolytic drug to be used clinically, but is not employed no" because it is non-fibrin specific, i.e. acti,ates both circulating as "ell as fibrin bound plasminogen. Therefore, 11 depletes circulating fibrinogen and predisposes to bleeding. Compared to newer more fibrin-specific tissue plasminogcn activators (altcplasc, etc.) it is less cflective in opening occluded coronary ancrics, and causes less reduction in Ml related mortality. Other disadvantages are need for a loading dose due to presence of anti-streptococcal antibodies. Strcptokinase is antigenic ·an cause hypersensitivity reactions; anaphylaxis occurs in 1-2% patients. It canno1 be used second time due to neutralization by antibodies generated in response to the earlier dose. Fever. hypotension and arrhythmias arc reported.

Urokinase It is an enqmc isolated from human urine: but commercially prepared from cultured human kidney cell s. Due to avai labili ty of better librinolytics, it has gone out of use.

Alteplase (recombinant tissue plasminogen activator (rt-PA) Produced by recombinant D A technology from human tissue culture, it catalyses the c leavage of endogenous plasminogcn to generate plasmin, and is moderately specific for fi brinbound plasminogen, so that c irculating fibrinogen is lowered only by ~ 50%. It is rapidly cleared by liver and inactivated by plasminogen activator inhibitor-I (PA I- I). The plasma t½ is 4-8 min. Because of the short l½, it needs to be given by slow i. v. infusion and often requires heparin coadmini tration. Aspirin is also routinely given after establishing reperfusion. Risk of bleeding

(0.5- 5%) is the most important concern. Alteplase is nonantigenic, but nausea, mi ld hypotension and fever may occur. It is expensive. ACTILYSE 50 mg, ial "ith 50 ml solvent "ater. For .\11: (accelerated regimen) 15 mg i.,. bolus injection followed by 50 mg over 30 min, then 35 mg over the next I hr. (total 100 mg in 90 min). For pulmonary embolism: I 00 mg i. v. infused over 2 hr. For iscliaemic ~troke: 0.9 mg/kg by i.v. infusion over 60 min. "ith I 0% of the dose injected in the first minute.

Reteplase

It is a nonglycosy lated deletion muta nt o f r t. PA produced by recomb inan t tec hnology in wh ic h some a mino ac ids of nat ive t- PA are missing. This c hange makes it longer acting, but somewhat less selective for fibrin bound plasminogen. It is cleared by both liver and kidney with an e ffective t½ of 13- 16 min. The longer action permits a double bolus i.v. administration of 18 mg (IOU) over 2 min each 30 min apart in cases of ST EM I and in PE. Dose adjustment is needed in liver and kidney damage. RETl:.LEX 18 mg ( IOU)/vial injection kit.

Tenectepla.se

T hi s genetically engineered substitution mutant of nati ve t- PA has hi gher fibrin selectivity, slower plasma clearance (longer duration of action) and resistance to inh ibition by PA I- I. It is the only fibrinolytic agent tha t can be injected i.v. as a single bolus dose over IO sec, while alteplase requires 90 min infusion and reteplase needs 2 injections at 30 min interval. T his feature makes it possible to institute fibrinolytic th era py immediately on diagnosis of STEM I, even during transport o f the pa tient to the hospital. Several randomized multicentric trials have assessed its efficacy in STEM ! and found it to be at least equally efficacious to alteplasc. Ri sk of noncercbral bleeding wa found to be lower w ith tenectcplase in the ASSENT-2 trial. but cran ial bleeding inc idence was similar. Dose: 0.5 mg/kg single i.v. bolus injection. ELAXIM 30 mg, 50 mg per vial inj.

Uses of fibrinolytics I. Acute myocardial infarction The c hi ef indi cation for fibr inolytic therapy is STEM I. They are not indicated, or may even be harmfu l

DRUGS AFFECTING COAGULATION, BLEEDING AND THROMBOSIS in UA and in low ri sk STEM!. In STEM! fibrinolytics arc a n a lternative first line a pproach to emergency percutaneous coronary intervention (PC I) with stent placement. Recanali zation of throm bosed corona ry artery has been ac hi eved in 50-90% ca es. Time lag in starting the infusion is critical for reduc ing area of necrosis, preserving ventric ular function and reduc ing mortality. The benefits of i. v. thrombo lytic therapy have been established by large randomised studies. Aspirin with or witho ut heparin is generally started concurrently or soon after thrombolysis to prevent reocc lus ion. Recanali zatio n efficacy is similar with alteplase, reteplase and tenecteplase, but tcnecteplase has the simplest regimen. Ri sk o f hypotension and allergic reactio ns is also comparable among the three. 2. Deep vein thrombosis (OVT) in leg, pelvis, shoulder etc.; up to 60% patients of DVT ca n be successfully treated by fibrinolytics. They can decrease subsequent pain and swelling. but the main advantage is preservati on o f venous val ves and may be a reduced ri sk of PE, th ough at the risk of haemo rrhage. Compa rabl e results have been o btained with Stk , urokinase and al teplasc. 3. Pulmonary embolism (PE) Fibrinolytic therapy is indicated in large , life-threate ning PE. Lung functio n may be he lle r preserved, but reduction in mortality is not established. 4. Peripheral arterial occlusion Fibrinolytics n:canalise - 40% l imb artery occl u,ions, especially those treated " ithin 72 hr. However, it is indicated only when surgical thrombectomy is not possible. Regional intraarterial fibrinolytics ha,c been used for limb arteries with greater success. Peripheral art~rial thrombolysis is follo"ed by shon-tcrm heparin and long-term aspirin therapy. Fibrinolytics have no role i n chronic peripheral vascular diseases.

5 . Stroke: Thrombo lytic therapy or ischaemic stroke is dependent on time-lapse since symptom onset; earlier, the better. Possibility of improved neurologica l outcome is to be ba lanced w ith risk o f intracranial haemorrhage. Alteplase is ap proved for use in ischae mi c stro ke. a nd

current op1111on strongl y recomme nds use of i.v. alteplase in carefully selected patients who can be treated within 3-4.5 hours of onset, and in w hom intracranial haemorrhage is ruled out along w ith a ll risk factors for bleeding (see contrai ndications in the box). After 4.5 hours o f sy mptom onset, the ri sk o f intracra ni a l haemorrhage outweighs the like lihood of benefit. Contraindications to thrombolytic therapy 1. H/o intracranial haemorrhage 2 . H/o ischaemic stroke in past 3 months 3. H/o head injury in past 3 months 4. lntracranial tumour/vascular abnormality/ aneurysms 5. Active bleeding/bleeding disorders 6. Patients receiving anticoagulants

7. 8.

Peptic ulcer, esophageal varices Any wound or recent fracture or tooth extraction H/o major surgery within 3 weeks

9. 10. Uncontrolled hypertension 11. Pregnancy

Evaluation All patients w ith ST EM! are candidates for reperfusion therapy. No consistent benefit of fi brinolytics has been demonstrated in nonSTEM I cases. while possibility of haemorrhage is increased. Only high risk cases o f STEM! may be treated w ith fibrino lytics. Both sho11tem1 and long-te1111 outcome is determ ined by early restoration of flow in the occluded artery, regardless of whether it is achieved by thrombolysis or by PC I. Best results arc obtained if perfusion can be restored w ithin the fi rst ho ur (the golden hour). While the e fficacy of fibrinolytics in dissolving the thrombus dim inishes with passage of ti me (little benefit afte r 6 hours of Ml onset), reperfusion by PCI is affected to a lesser extent by the time lapse. Thrombolysis may be favo ured ir iL can be started within I 2 ho urs of onset. After 3 ho urs, PCI is favoured. Moreover, PC I has the advantage of lower bleeding risk, hig her grade of flow in the canalized artery and reduction in the rate of non fa tal recurrent Ml compared to thrombolys is. As such, PC I has

675

676

DRUGS AFFECTING BLOOD AND BLOOD FORMATION

yielded superior resu lts compared to fibrinolytics and is being preferred at centres where it can be performed swiftly with requisite expertise. Primary PCI is the procedure of c ho ice for patients with contra indicatio ns to thrombolytics (see box). Fibrinolytic therapy requ ires careful patient selection, but often it can be instituted w ith less delay, and even at centres not well equipped for PC!. Another approach is 'facilitated PC I' wherei n full or reduced dose fibrinoytic therapy is followed at the earli est by PCI. The res ults of this approach are comparable to those of primary PCI. The European as well as American (ACC, A HA) gu idelines provide that STEM I patients should be treated with primary PC! or with fibrinolytic drugs followed by im media te rescue PCJ, if reperfu sio n fai ls with the fibr ino lyt ic . As pirin and heparin are continued after thro mbo lysis.

ANTIFIBRINOLYTIC DRUGS These are drugs wh ic h inhibit plasmin ogen activation a nd di ssol ution of clot, and are used to check fibrino lysis associated bleed ing.

Epsilon amino-caproic acid (EACA)

It is a lys ine analogue whi c h combines wi th the lysine-binding sites of plasminoge n and plasmin so that the latter is not able lo bind to fibrin and lyse it. It is a specific antidote for fibrinolytic agents and has been used in many hyperplasminaemic states associated with excessive intravascular fibrin o lysis result ing in bleeding. The primary indi cati on of EACA is to co unteract the effect of fibrinolytic drugs a nd bleeding due to their use. In haemophiliacs, it has adjun cti ve value for co ntro llin g bleeding due to tooth extraction, prostatectomy, trauma, etc. EACA is acti ve orally a nd can be infused i.v. as well. It is excreted by the kidney. The major concern in the use of EACA is that it can cause intravascular thrombosis. Rapid i.v. injection res ults in hy po tens ion, bradycardia and may be arrhythm ias . Ureteric obstruction may occur due to un lysed clo ts, and it should

be used cautio us ly when re nal function is impa ired. Myopathy occurs rarely. The large dose needed is a lim itation , a nd tranexamic acid is mostly preferred. Initial priming dose is 5 g oral/i.v., followed by I g hourly till bleeding stops (max. 30 g in 24 hrs). AMICAR, HEMOCID, IIAMOSTAT 0.5 g cab., 1.25 g/5 ml syr., 5 g/20 ml inj.

Tranexamic acid

Like EACA, it binds to the lysine b inding site on p lasminogen and prevents its combi nat ion with fibrin leading to fibrinolysis . It is 7 times mo re potent than EACA, and is preferred for prevention/ control of excessive bleeding due to: • Fibrinolytic drngs. • Cardio-pulmo nary bypass s urgery. • Tonsillectomy, prostatic surgery, tooth extraction in haemophi liacs. • Menorrhag ia, espec ially due lo I UCO. • Recurrent ep istaxis, hyphema due Lo oc ular trauma, peptic u leer. Main si de effects are nausea and diarrhoea. Thromboembolic events, dis turbed colour vision and allergic reactions are infrequent. Thrombophlebitis of injected vein can occur. Dose: 1- 1.5 g TDS oral. 0.5- 1 g TDS by slow i.v. infusion. DUBATRAN, r>AUSE, TRANAREST 500 mg tab, 500 mg/5 ml iuj.

ANTIPLATELET DRUGS (Antithrombotic drugs) T hese are drugs which interfere w ith platelet function and are useful in the prophylaxis of thromboembolic diso rders. Plate lets express several glycoprotc in (GP) int.:grin receptors on the ir surface. React ive prote ins like collagen and von Willcbrand factor (v\VF) are exposed when there is damage to vascular endothelium, and they react respectively with plate let GPI, and GPI• receptors. This results in platelet activation and re lease of proaggregatory and vasoconstrictor mediators like TXA2, ADP and 5-HT. The platelet GPII/ 111, receptor undergoes a confom1alional change favouring bind ing of librinogen and vWF that cross link platelets inducing aggregation and anchorage to vessel wall/other surfaces. Thus, a ·platelet plug' is formed . In veins, due to sluggish blood flow, a fi brinous tail is formed which tmps RDCs ' the red ta il '. In arteries, plate let mass is the main constituent of the thrombus. Antiplale lel drugs arc, therefore, more useful in a rterial thrombosis. while anticoagulanu. are more elTecti,c in venous thrombosis.

DRUGS AFFECTING CO AGULATION, BLEEDI N G AND THROMBOSI S

ANTIPLATELET DRUGS (Antithrombotic drugs)

I

l

Thromboxane synthesis inhibitor

Platelet cAMP enhancer

As pirin

Dipy ridamole

l'rostacyclin (PGI,), synthesized in the intima of blood vessels. is a strong ·inhibitor of platelet aggregat ion. A balance bc1wccn TXA, released from platelets and PGI, released from vessel ";all appears to control intravascula; thrombus formation. Platelets also play a role in alherogenesis.

In the above scheme, various drugs act o n different targets to interfere with platelet function. Therefo re, g iven together, th e ir actions a re synergistic. The c linically important antiplatelet drugs are classified in the cha rt:

Aspirin IL acetylates the enzyme COX I and TX-sy nthase- inacti vat ing them irreversibly. TXA2 activates plate lets to change shape and release mediator rich granules which promote aggregation. Because TXA 2 is the major arac hidonic acid product gene rated by platelets. a nd that plate lets arc exposed to aspirin in th e portal circulation before it is deacetylated durin g first pass in the li ver, and because platelets cannot synthesize fresh enzyme (have no nuclei), TXA 2 formation is suppressed at very low doses a nd ti ll fresh platele ts are form ed. Thus, as pirin induced prolongati on of bleedi ng ti me last for 5- 7 days. EITcct of da ily doses cumulates and it has been shown that doses as low as 40 mg/day have some effect on platelet aggregation. Max ima l inhibition o f platelet func tion occurs at 75- 150 mg aspiri n per day. The American (ACC/ AHA)* guidelines recommend a dose of 75- 162 mg/ day for long-term aspirin prophylaxis. However, asp irin may not effect ive ly inhibit platelet aggregation in certain individua ls, who are labe lled as 'aspirin resista nt'.

I

I

I

P2Y 12 receptor blockers Ticlopidin e Clopidogrel Prasugrel Ticagreior

l

IGP ui,1111.

antagonists

Abc iximab Eplifibatide Tirofiban

Inhib ition of CO X- I by aspmn in vesse l wall decreases PG l2 synthesis as well. However, since intimal cells can synthesize fresh enzyme, COX acti vity returns rapidl y. It is possible that at low doses (75- 150 mg/day or 300 mg tw ice weekly), TXA, formation by platelets is selectively suppressed, wi1ereas higher doses (> 900 mg/day) may decrease both T XA 2 and PGl2 production. Aspirin in hibits the re lease of A DP from platelets and their sticking to each other, but has no effect on platelet survival time and thei r adhesion to da maged vessel wall . ASA 50 mg tab., COLSPRIN, DISPRIN CV-100: aspirin 100 mg soluble tab, LOPRIN 75 mg tab. ASPICOT 80 mg tab, ECOSPRIN 75, 150 mg tab.

Other NSA/Ds arc reversible inhibitors of COX, produce shor1-las1ing inhibition of platelet function-are nol clinically useful.

Dipyridamole

lt is a vasodilator that was introduced fo r a ngina pecloris (see C h. 40). It inhibi ts phosphodiesterase as well as blocks uptake of adenosine to increase plate let cAM P which in turn p otenti ates PG I2 and interferes with aggregation . Levels of TXA2 or PGl2, are not a ltered, but platelet survival time reduced by disease is normalized. Dipy rid a mo le a lone has little c li nically significant e ffect. but im proves the res ponse to warfarin, a lo ng with which it is used to decrease the incidence of th romboembolism in patients with prosthetic heart valves. Dipyridamole has a lso been used lo enhance the anti platelet action of aspirin. This combination may add itionally lower the risk o f stroke in patients with tra nsient ischaemic attacks (T IAs),

• ACC/AHA- American College of Cardiology/American Heart Association

677

678

D RUGS AFFECTING BLOOD AND BLOOD FORMATION

but tria ls have failed to de monstrate addit iona l benefit in prophylaxis of MI. Dose: 150-300 mg/day. PERSANTIJ\: 25. 100 mg tabs. 1 HRO\.1BO\/IL 75, I00mgtabs, DYNASPRIN: d1pyridamolc 75 mg + aspirin 60 mg e.c. tab., CARDIWELL PLUS: dipyridamole 75 mg + aspirin 40 mg tab.

It is the first thienopyridine drug that acts by blocking the P2Y 11 type of purincrgic receptors on the surface of platelets and inhibits ADP-induc~-cl aggregation. Ticlopidine produced beneficial effects in TIAs, stroke prevention, UA, econdary prophylax is of Ml. and synergi7ed with aspirin to lower the incidence of restenosis afler PCI and stent thrombosis. HO\, ever, it produced serious adverse cflects like neutropenia, thrombocytopenia. haemolysis. jaundice, and has been superseded by other P2Y 12 inhibitors (clopidogrcl, etc.). Dose: 250 mg BD with meals; efTect persists several days aflerdiscontinuation: TYKLID, TICLOVAS. 250 mg tab: ASTIC ticlopidinc 250 mg~ a;,pirm I00 mg tab. Ticlopidine

Clopidogrel This is a more potent congener of t iclopidine w h ich has s imilar mechanism of action. i.e. irrevers ibly b locks the P2Y12 type of purinergic recep tor on th e s urface of platelets. This Gi-coupled GPCR med iates AD P- induced platelet aggregation by inhibiting adenylyl cyclase and decreasing cAMP. C lopidogrel resemb les ticlopidine in inhibiting platelet function irreversibly, and has the same range of therapeutic efficacy, but is safer and better tolerated (CLASS ICS study). Whi le neutropenia occurs in about I% patients taking ticlopidine, it is rare with clopidogrel. The 'clopidogrel vs aspirin in patients at risk of ischaemic events' (CA PRI E) tria l has fo und clopidogrel recipients to have a slightly lower annua l risk of primary ischaemic events than aspirin recipients. Combination of clopidogrel and aspirin is synergistic in preventing ischaemic epi sodes. The CURE trial has reported 20% reduction in MI, stroke and cardiovascular death by addition of clopidogrcl to aspirin in cases with UA and NSTEMl. This ' dual antiplatelet the rapy ' (D APT) is a lso utili zed for checking restenosis of stented coronaries. Like ticlopidine, c lopidogrel is also a prodrug. About 50% of the ingested dose is absorbed, and only a fracti on of this is slowly activated in liver by t\vo step metabol ism into an active thiol metabolite invol ving CYP2C I 9. while the

rest is inactivated by other enzymes. Therefore, it is a slow acting drug; antiplatelet action takes about 4 hours to start and develop over days. Since CY P2C 19, exhi bits genetic polymorphism, the activation of c lopidogrel and consequently it antiplatelet action shows high interindivi dual variability. Some patients a re nonresponsi ve. Omeprazole, an inh ibitor of CY P2C 19, reduces meta bo lic acti vat ion of clopidogrc l a nd its a nti platelet action. The action of c lopidogrel lasts fo r 5 days due to irreversible blockade of platelet P2Y 12 receptor . The most important adverse effect is bleeding. Addition of aspirin to c lopidogrel has been found to doub le the incidence of serious b leeding among high risk stro ke patients (MATC H study). However, neutropenia, thrombocytopeni a and other bone marrow toxicity is rare. Side effects are diarrhoea, epigastric pai n and rashes. Do~e: 300 mg loading. then 75 mg OD; CLODREL. CLOPILH, DEPLATT 75 mg tab.

Prasugrel This is a newer mo re potent a nd faster acting P2Y 12 purincrgic receptor blocker, that is being increasingly used in ACS as well as when strong antiplatclet action is required. Like its predecessors, it is a lso a prodrug, but is more rapidly absorbed. It is also more rapidly and more completely activated. resulting in faster and more consistent platelet inhibition. Though CY P2C I 9 is invo lved in activation of prasugrel as well, it is a faster one step process. and geneti c polymorphism re lated decrease in response, or interference by omeprazole treatment has not been prom ine nt. Recovery of platelets from prasugrel action takes longer (7 days) than with clopidogrel (5 days). Because of rapid action, pra ugrel is particularly suitable for use in STEM!. lt is the preferred thienopyridine in ACS to cover angioplasty with or wi thout stent placement. The TRlTO trial compared prasugrel with clopidogrel in STEM] and STEMl. There was 19% greater reduction in death from card iovasc ul a r causes in the prasugrel group. Superior c lin ical outcomes and reduction in Stent thrombosis have been obtained with prasugrel. Bleeding complications are also

DRUGS AFFECTI NG COAGULATION, BLEEDI NG AN D THROMBOSIS more frequent and more serious. Patients with history of ischaemic stroke and TIAs are at greater risk of intracranial haemorrhage. Prasugrel is contraindicated in such patients. Dose: Loading dose 60 mg, followed by I O mg OD: elderly or those 18000 patients with ACS and reported lower cardiovascular mortality as well as all cause mortality with ticagrelor. The risk of imracranial bleeding was higher with ticagrelor, but that of al l major bleeds was similar. It has been observed that high dose aspirin may interfere with the action of ticagrelor, but not low dose aspirin. The European guidelines now recommend that all patients al high risk of ACS be given prophylactic 1icagrelor. Side effects are dizziness, nausea, shortness of breath, tightness in chest and irregular pulse. Th us, ticagrelor is a faster, more potent and more consistent acti ng P2Y 12 inhibitor anti platelet drug. Dose: For ACS requiring urgent PCI 180 mg loading dose followed by 90 mg BD; may be continued for upto 12 months DRI Lr.-:TA 90 mg tah

Glycoprotein (GP) II/ Illa receptor antagonists GP 11 / 111. antagonists are a newer class of potent platelet aggregation inhibitors which act by blocki ng the key receptor in vo lved in platelet aggregation. The G Pl 1/ 111 is an adhesive receptor (i ntegrin) on platelet surface for fibrinogen and vWF through which agonists like collagen, thrombin. TXA2, ADP, etc. finally 0

induce platelet aggregation. Thus, GP Il/ IJI. antagon ists block aggregation induced by all platelet agon ists. They are used only in patients with ACS and to cover PCT or coronary artery bypass grafting (CABG). Abciximab It is the Fab fragment o f a chimeric monoclonal antibody against GP 11/ 111 0 _ protein, but is relatively nonspecific and binds to some other surface proteins as well. Given alo ng with aspirin ..- heparin during PCI it has markedly reduced the incidence of restenosis, subsequent Ml and death. In the ISAR-REACT2 trial addition of abc iximab to clopidogrel (600 mg oral loading do e) for PCI in high-risk ACS patients, reduced isc haemic events by 25%. After a bolus dose, platelet aggregati on remains inhibited fo r 12 24 hr, while the remaining antibody is cleared from blood wi th a t½ of I 0- 30 min. Dose: 0.25 mg/kg i.,. 10 60 min before PCI, followed by 10 r1g/min for 12 hr. REOPRO. FAXI VI A Fl 2 mg/ml in 5 ml ,ial.

Abcix imab is nonant igenic. The main risk is haemorrhage, incidence of which can be reduced by carefull y managi ng the concomitant heparin therapy. Thrombocytopenia is another compl ication. It shou ld not be used second time, since risk of thrombocytopenia increases. Constipation, ileus and arrhythmias can occur. It is expensive. but is being used in unstable angina and as adju vant to coronary thrombolysis/ PCI with stent placement. Eptifibatide It is a synthetic cyclic peptide that selectively binds to platelet surface GPll/ 111 1 receptor and inhibits platelet aggregation. Though its plasma t½ (2.5 hours) is longer than that of abciximab, platelet inhi bition reverses in a shorter time (within 6- 10 hours) because it quickly dissociates from the receptor. Infused i.v., eptifibatide is indicated in: Um,lable angina: 180 µg/kg i.v. bolus, fo llowed by 2 µg/kg/ min infusion upto 72 hours. Coronary cmgioplasry: 180 r1g/kg i. v. bolus immediately before procedure: follow with 2 (lg/kg/min for 12-24 hours. CLOT IDE, COROMAX. UNIG RILII\. CPT IFAB 20 mg/10 ml and 75 mg JOO ml , iai,. Aspirin and heparin arc generally given concurrently.

679

680

DRU GS AFFECTING BLOOD A ND BLOOD FORMATION

B leeding and tluom bocytopenia are the major ad verse effects. Rashes and ana phylaxis are rare .

Tirofiban

T his is a no npeptide bu t specific G Plli llI. antagon ist that is simi la r in properties to e ptifibatide . Its plas ma t½ is 2 ho urs, and it d issociates rapid ly fro m the receptors. The ind ications and adverse effects are a lso si milar to eptifibatidc.

Acute coronary syndromes undergoing PC I: 0.4 ~•g/kg/min for 30 min followed by 0. I µg/kg/min for upto 48 hours. If angioplasty is performed. infusion 10 continue till 12 24 ho urs thereafter. AGGRAMED, AGGRITOR. AGGR IBLOC 5 mg/ JOO ml infusion. TIROBAN 12.5 mg/50 ml and 5 mg/100 ml inj.

Uses of antiplatelet drugs T he a im of us ing anti platelet drugs is to prevent intravascular throm bos is and e mbo lizatio n, with minima l ri sk o f haemo rrhage. The inte ns ity o f a ntip latelet therapy is selected accord ing to th e t h ro m bo ti c in fl ue nces present in a patie nt. Those wi th CA D or r isk fac tors for s tro ke a re g ene ra ll y g ive n a s in g le drug (as p ir in/ c lo p id og rel ). Fo r indic ation s l ike AC S, ma in tenance of vascula r recana li zation. stent placeme nt, vessel g raft ing, etc. po tent inhib iti o n o f pla te le t fu ncti on is requi red. T hi s is prov ided by co m bining two a ntipl ate let drugs w hic h a c t by d i ffe re nt mec ha ni s m s (d ua l ant iplatelet therapy).

1. Coronary artery disease O n the basis of trials in post- MI patients as well as in those wit h no s uc h his tory. it is recom me nded that as pirin 75- 150 mg/day be g iven to a ll indi vidua ls w ith evidence of coro nary a rtery d isease and in those with ri sk fac to rs fo r the same, but routi ne use in the who le po pulatio n is no t wa rranted . Prim ary preventi o n o f ischaemia w ith as pirin is o f no proven benefit. It red uces the inc ide nce o r fa ta l as well as no nfatal M I, but increases the ri sk of cerebral haemorrhage. C lo pidogre l is an a lternative to as pirin in symptomatic patients of ischaemia. Co ntinued as piri n/ c lop id og re l prophy lax is in post- M l patients c lea rly prevents reinfarction and reduces morta lity.

2. Acute coronary syndromes (ACSs) These comprise of a range of acute cardiac ischaem ic states fro m unstable a ng ina (UA) to non-ST elevation m yoca rd ial infarc tion (NSTEM I) to ST EM I (see p. 602). The coronary obstruction in UA and NSTEM I is partial. while that in TEMI is total. UA and 1STEMI arc differentiated on the basis of absence or presence of laborntory markers of cardiac myocyte necrosis (myoglobin, CK-MB, troponin I, etc.). The ischacmic status is oflcn dynamic and the patient may rapidly shi fi from one category to the next.

Solu ble asp irin (32 5 mg o ra l) a nd a LMW he parin (s.c.) are g iven at presentation to all patients w ith ACS . Uns tabl e a ngina (UA) Aspirin reduces the risk of prog ress ion to MI and s udden death. C lo pidog re l is generally co mbined with as pirin, or may be used as a lte rnat ive if as piri n canno t be g iven. For max imum protecti on the a ntip late let d rugs are supplemented w ith LMW heparin fo llowed by warfa rin .

NSTEMI Patients o f ST EM ! w ho are managed w ithout PCI/thrombo lysis are generally put o n a combinatio n o f aspirin + c lopidogre l or ticagrelor, which is continued for upto o ne yea r. STEMI

Primary PC I w ith or witho ut stent placement is the procedure of cho ice for a ll STEM I as we ll as h igh ris k NST EM I patients w ho present within 12 ho urs. Pras ug rel o r ticagre lor + aspirin is the antiplatelct regimen most commo nly selected for patients w ho are to undergo PC I. Pras ug re l a nd ti cag rc lo r act rapidl y a nd mo re pred ictably tha n clopidogrcl. Prasugre l is also perfe rred over clopidogrel in diabetics. The GP ll/ 1111 antagonists are the mos t powerfu l ant iplatelet drugs. They are combined w ith as pirin fo r hig h risk patients undergoing PC I. Abcixi mab/e ptifi bat ide/tiroftban infused i.v. along w ith ora l as p iri n and s.c. heparin marked ly reduce inc idence o f res tcnosis a nd subsequent M l after corona1y angioplasty. T he G Pll/ lll. antagonis ts arc infused for a max imum of 72 ho urs. As p irin a nd/ o r c lo pidog rel/t icagrc lor are routine ly g iven to ACS patients treated w ith thrombolysi . Coronary artery bypass surgery is

681

DRUGS AFFECTING COAGULATION , BLEE DIN G AND THROMBOS IS

a lso covered by intensive antipla telet regimen i11c luding aspirin + GPU/ 111, antagonists/prasugrel. The patency of reca nal ized coronary artery or im planted vessel is improved and incidence of reocclus ion is reduced by continu ing aspirin + c lopidogrel/prasug rel/ ticagre lo r fo r upto I 2 months. Dual anti platelet therapy (DA PT) is recommended after stent placement. Prasugre l is used when stent thrombos is occurs during c lopidogre l treatment. 3. Cerebrovascular disease A nti p la telet drugs do no t alter the course o f s troke du e to cerebral thrombosis. However, asp iri n has reduced the inc idence of T IAs a nd of s troke in pati ents wi th T lAs. Occurrence o f s tro ke is a lso reduced in patients w ith pe rs is tent at ria l fi brillatio n and in those with hi story of stroke in the past. As pirin or c lo pidogrel is g ive n to all patie nt s of TrAs w ho a rc no t to be treated with anticoagul ants. T hough short-term use of aspirin + c lopidogre l DA PT may be benefic ia l, long-te rm use of the combina tion increases the risk of haemorrhage. The European stroke prevention s tudy-2 (ES PS) has found combinati o n o f dipyr ida mo le w ith low

Cir

dose as pmn to be yne rg istic in prevention o f stroke.

econdary

4 . Prosthetic heart valves and arteriovenous shunts A nti platelet drugs, used w ith warfarin reduce fonnation of microthrombi on artifi cial heart va lves and the incide nce of em bolism. Aspirin is clearly effective but increases risk of bleeding due to warfarin. Dipyridamo le does not increase bleeding risk, but incidence of thromboembol ism is reduced w hen it is com bined w ith an oral a nti coagulant. A nti platelet drugs a lso prolong the patency of c hronic a rteriovenous shunts im planted fo r haemodialysis and of vascu lar grafts. 5. Venous thromboembolism Anticoagulan ts arc routinely used in D VT a nd PE. Tria ls have shown a ntiplatelet drugs a lso to have a prophylacti c e ffect, but the ir rela tive va lue in compa rison to, or in additio n to anti coagul ants is not established ; they are in freq uently used. 6. Peripheral vascular disease As pir in/ c lopidogrel may produce some improveme nt in in te rm itte nt c la ud ica tio n and reduce the in cidence of thro mboembo lism.

PROBLEM DIRECTED STUDY

45.1 A 35-yea r-old woman was on mainte nance therapy with warfarin for leg vein thrombosis that she had deve loped during a complicated delivery 2 month s back. The dose was adjusted by re peated me asurement of INR, and for the last one month it was maintained between 2.4- 2.8 with 4 mg taken daily at bed time. She develope d a pelvic infection for which she was admitted to the hospital and g iven lnj. Ceftriaxone 1 g i.v. 8 hourly. On the 3rd day she started ble e ding per-vaginum a nd reported passing dark urine. The haemoglobin level fe ll to 9.0 g/dl, while on admission 3 days back, it was 11.0 g/ dl. The INR was measured to be 5.4. (a) What could be the cause of bleeding per-vaginum, passing dark urine; fall in Hb leve l and rise in INR value ? Could this complication be prevented? (b) How s hould this patient be managed? (see Appendix-! for solution)

Chapter

46

Hypolipidaem·ic Drugs

Hypo li pidaemic drugs are those which lower the levels of lipids and lipoproteins in blood. The hypolipidaemic drugs ha ve attracted considerable attention because of their potential to prevent cardiovascular disease by retarding the accelerated atherosclerosis in hyperlipidaemic ind ividuals.

Lipid transport Lipids are carried in plasma in lipoproteins after getting associated with several apoproteins; plas ma lip id co ncentrations are depe nde nt on t he concent rat io n of li poproteins. The hydrophobic core of lipoprotein g lobules consists of triglycerides (TGs) or cholesteryl esters (CHEs) while the outer po lar layer has phospholi pids, free cholesterol (C H) a nd apoproteins. The lipoproteins have been di vided into 6 classes

on the basis of their particle size and density. They also d iffer in the natu re of apoproteins, the ratio of TG and C HE, tissue of origin and fate. These a re given in Table 46.1. Dietary lipids are absorbed in the intestine with the help of bile acids. Chylomicrons (Chy) are formed and passed into lacteals- reach blood stream via thoracic duct. During their passage through capi llaries, the endothelium bound lipoprotein lipase hydrolyses the TGs into fatty acids which pass into muscle cells to be utilized as energy source and in fat cells to be reconverted into TGs and stored. The rema ining part~hylomicron remnant (Chy. rem .) containing main ly CHE and linle TG is engulfed by liver cells, which have receptors for the surface apoprotcins of Chy. rem., and digested. Free C H that is liberated is either stored in liver cells after reesterification or incorporated into a difTcrcnt lipoprotein and released in blood or excreted in bile as CH or bile acids. Liver secretes very low density lipoproleins (VLDL) containing mainly TG and some CHE into blood. VLDL is acted upon by endothelial lipoprotein lipase in the same

Table 46.1: Characteristics and function of plasma hpoprote1ns Lipoprotein

Diameter (nm)

Lipid contained

Source of lipid

Function

TG »CHE

Diet

Dietary TG transport

class 1.

Chy.

100-500

2.

Chy. rem.

30-50

CHE » TG

Diet, Chy.

Dietary CH transport

3.

VLDL

40-SO

TG >> CHE

Liver

Endogenous TG transport

4.

IDL

30-35

CHE 2, TG

VLDL

Transport CHE and TG to liver, source of LDL

5.

LDL

20-25

CHE

IDL

Transport CH to tissues and liver

6.

HDL

5-10

Phospholipid, CHE

Tissues, cell memb.

Removal of CH from tissues

Chy-Chylomicrons; Chy. rem.-Chylomicron remnant; VLDL-Very low density lipoprotein; IDL- lntermediate density lipoprotein; LDL- Low density lipoprotein; HDL- High density lipoprotein; CHE-Cholesteryl esters; TG-Triglyceride; CH- Cholesterol

683

HYPOLI PIDA EMIC DRUGS HYPOLIPIDAEMIC DRUGS

I

I

I

HMG-CoA reductase inhibitors (Statins)

Lovas ta tin S imvastatin Pravastatin A torvastatin Rosuvastatin Pitavastatin

Lipoproteln lipase activators (PPARa agonlsts: Fibratesl

I

I

Sterol absorbtion inhibitor Ezetimibe

Gemfibrozil Beza fib rate Fenofibratc

IBile acid sequestrants Cholestyram ine Colestipol Colesevelam way as on Chy and the fauy acids pass into adipose tissue and muscle; the remnant called 1111ermedia1e densi/\• lipoprotein (IDL) now contains more Cl IE than TG. About half of the IDL is taken back by the li,er cells by auachment to another receptor (LDL receptor), "hile the rest loses the remaining TGs gradually and becomes low density: lipoprutein (LDL) containing only C l IE. The LDL circulates in plas ma for a long time; its uptake into liver and other tissues is dependent on the need for CH . The rate of LDL uptake is regulated by the rate of LDL receptor synthesis in a particular tissue. Cholesterol is also synthes i.ted in the body by reduction of 3-1lydroxy-3 methyl glutaryl coen.tyme A (1IMG-CoA) to mcvalonate (rate limiting ste p). The C HE of LDL is deesterified and used mainly for cell me mbrane formatio n. The C l I released 11110 blood from degradation of mem branes is rapidly incorporated in high density lipoproreins (H DL), esterified with the help of an e11Lyme lec ith in: cholesterol acyltnrns ferase ( LC AT) and transferred back to VLDL or IDL. completing the cycle. The excess lipoproteins in plasma are phagocytosed by macrophages for disposal. When too much of lipoproteins have to be degraded in this manner, C H is depos ited in atheromas ( in arterial walls) and xa111homas (in skin and tendons). Raised levels of VLDL. IDL and LDL (rarely Chy and Chy. rem. also) arc athcrogenic, "hile I-IDL may be protecti,e. because HDL facil itates removal of CH from tissues.

Hyperlipoproteinaemias can be: (i) Seconda,y: associated with diabetes, myxoedema, nephrotic syndrome, chronic a lcoholism. drugs (corticosteroids, oral contraceptives, 13 blockers) etc. (ii) Primary: due to: (a) A si ngle gene defect: is familial and called ' monogcnic' or genetic.

Lipolysis and triglyceride synthesis Inhibitor Nicotinic acid

(b) Multiple gene tic, dietary a nd physical activity related causes: ' pol ygenic' or multi.factorial. On the whole, LDL is the primary carrier o f plas ma C H E, a nd VL DL that of TGs. The important features of major types of hyperlipoprote inaemias are given in Table 46.2. T he mechanism of acti on and profile of lip id lowering effect of important hypolipidaemic drugs is summarized in Table 46.3.

HMG-COA REDUCTASE INHIBITORS (Statins) 1ntroduced in the 1980s, this class of compounds are the most efficacious, most commonly used and best tolerated hypolipidaemic drugs. They competitively inhibit conversion of 3-Hydroxy3-methyl glutaryl coenzyme A ( HMG-CoA) to mcvalonate (rate limiting step in C H synthesis) by the e nzy me I IMG-CoA rcductase. T herapeutic doses reduce CI I synthesis by 20-50%. This results in compensatory increase in LDL receptor expression on liver cells and causes incrca ed receptor mediated uptake and catabolism of IDL and LDL. Over long-term, feedback induction of l lM G-CoA reductase tends to increase C H synthesis, but a steady-state is finall y attained with a dose-dependent lowering of LDL-C H levels. DifTerent statins differ in the ir potency and max imal efficacy in reducing LDL-CH. T he dose dependent effect of various statins on pla ma

684

DRUGS AFFECTING BLOOD A ND BLOOD FO RMATION

Table 46.2: Types of primary hyperllpoproteinaem,as Type

Ila

Disorder

Cause

Occurrence

Elevated plasma lipoprotein

CH

TG

Familial lipoprotein lipase deficiency

G

Very rare

Chylomicron

ii

m

Familial hypercholesterolaemia

G

Less common

LDL

i1

N N

Plasma lipids

llb

Polygenic hypercholesterolaemia

MF

Commonest

LDL

T

Ill

Familial dysbetalipoproteinaemia

G

Rare

IDL, Chy. rem.

IV

Hypertriglyceridaemia

MF, G

Common

VLDL

T N

V

Familial combined hyperlipidaemia

G

Less common

VLDL, LDL

IT T

CH- Cholesterol; TG- Triglycerides; G- Genetic ; MF- Multifactorial; Chy. rem. -Chylomicron remnants; VLDL-Very low density lipoprotein; IDL-lntermediate density lipoprotein; LDL-Low density lipoprotein. The genetic defect in some of the monogenic disorders are: Type I: absence of lipoprotein lipase-TG in Chy cannot be utilized. Type Ila: deficiency of LDL receptor- LDL and IDL are taken up very slowly by liver and tissues. Type Ill: the apoprotein in IDL and Chy. rem. (apoE) is abnormal, these pa rticles are cleared at a lower rate. Type IV: this type of hypertriglyceridaemia is both multifactorial and monogenic, the former is more prevalent than the latter.

LDL-CH level is presented in Table 46.4. Whi le lovastatin, pravastatin and simvastatin are lowto-moderate efficacy (20-45% LD L reduction) and potency statins, atorvastati n and rosuvastatin are high efficacy (upto 55% LDL-CH reduction) compounds. Though dose-to-dose most potent, efficacy of pitavastatin is limi ted by its ceiling dose of 4 mg/day. All stat ins produce peak LDLCH lowering after 1- 2 weeks therapy. Hepatic synthesis o f VLDL is concurrentl y reduced and its removal fro m plasma is enhanced. A concurrent fall by 10-30% in plasma TG level, probably due to reduction of YLD L, occurs. A modest rise in HDL-CH by 5- 15% is also noted. Statins a re effecti ve in secondary hypercholesterolaemias as we ll. T he more elTicacious stati n (atorvastatin, rosuvastatin) give n at their higher doses effectively reduce TGs (by 25% to 35%) when TG levels are mode rately raised, but not when they are ma rkedly raised. Because HMG-CoA rcductase activity is maximum at midnight, al l stati ns are administered at bed time to obta in maximum effectiveness. However, this is not necessary for atorvastati n and rosuvastatin, which have long pla ma t½.

A ll statins, except rosuvasta tin a re metabolized prima rily by CYP3A4. Inhibitors and inducers of this isoenzyme respective ly increase and decrease statin blood levels.

Lovastatin This is the fi rs t c lin icall y used statin, w hi c h is lipoph ilic a nd g ive n orally in the prec ursor lacto nc form . Absorption is incomp lete a nd first pass metabo lism is extensive. Metabo lites are excreted ma inl y in bile. The t½ is s hort (2- 4 hours). The LDL-CH lowering efficacy is low-to-moderate. Dose: 10-40 mg/day (max. 80 mg). ROVACOR, AL.TATIN, LOVAMEG IO, 20 mg tabs.

Simvastatin

It is nea rl y twice as potent as lovastatin; a lso more efficac io us. A g reater rise in HDL-CH (when low) has been noted with simvastati n tha n lovastatin or pravastatin. Like lovastatin, it is lipophi lic and given in the lactone precursor fo rm. Oral absorption is belier and fi rst pass metabolism extens ive; t½ is 2- 3 hr. Dose: 10-40 mg/day (max. 80 mg) Sl\.fVOTIN. SIMCARD, ZOSTA 5, 10. 20 mg tabs.

Higher risk of myopathy has been noted w ith 80 mg/day dose of simvastati n. T his dose should

685

HYPOLIPI DAEM IC DRUGS

Table 46.3: Mechanism of action and pattern of hp1d lowering effect of hypohp1daem1c drugs Drug (daily dose)

Mechanism of action

Effect on plasma lipids (%)

J. CH synthesis by inhibition of rate limiting HMG-CoA reductase

LDL J, 20-55 HDL t 5- 15

HMG-CoA reductase inhibitors Lovastatin (2D-80 mg) Simvastatin (10-40 mg) Atorvastatin {10--80 mg) Rosuvastatin (5--40 mg) Pravastatin (1D-80 mg) Pitavastatin (1-4 mg)

TG J, 10-35

Bile acid sequestrants Cholestyramine (4-16 g) Colestipol (5-30 g) Colesevelam (3-4 g)

J. bile acid absorption, t hepatic conversion of CH to bile acids, t LDL receptors on hepatocytes

LDL J. 15-25 HDL t 3-5 TG not affected, may some

t

LDL J. 5-15' HDL t 10-20 T G J. 20-50

t in

Fibric acid derivatives Gemfibrozil (1200 mg) Bezafibrate (600 mg) Fenofibrate (200 mg)

Activity of lipoprotein lipase,

J, release of fatty acids from adipose tissue

Nicotinic acid (2--4 g)

J, Production of VLDL,J. lipolysis in adipocytes

LDL J, 15-25 HDL t 20-35 TG J. 20-50

Ezetimibe ( 10 mg)

intestinal absorption of CH and phytosterols

LDL 15-20% HDL t 5% TG-

• Gemfibrozil may t LDL-CH when TG levels are high ; bezafibrate and fenofibrate not likely to raise LDL-CH

be gi ven only when there is no a lternati ve, and expected benefits outwe igh risks.

Pravastatin It is hydrophilic and given in the active form. The LDL-CH lowering potency and e fficacy is almost similar to lovastatin. A n additional action of decrease in plasma fibri nogen level has been observed. T he t½ is 1- 3 hours. PRAVATOR 10, 20 mg tabs.

Atorvastatin This is a high efficacy and one of the most commonly used statin capable of lowering LDL-CH by 55-o0o/o at the 80 mg/ day ceiling dose. Al this dose a greater reduction in TGs is noted if the same was ra ised at baseline. Atorvastatin has a much longer plasma t½ of 14- 18 hr, and has additional antioxidant property. Dose: 10-40 mg/day (max. 80 mg) ALTOR, ATORVA, ATORLIP 5, 10, 20 mg tabs.

Rosuvastatin Thi s is anothe r newer co mmon ly used hi g h pote ncy a nd h igh e ffi cacy

statin ( IO mg rosuvas talin ::: 20 mg ato rvastatin). with a p las ma t½ of 18 24 hours. Greater LD L-CH redu ction can be obta ine d in severe hy pe rcholestero laem ia; pa rtly due to its lon ge r pers iste nce in th e plas ma. In patients w ith raised TG levels , ros uvastatin ra ises HDL-CH by I 0- 15% (greater ri se tha n othe r statins). Dose: 5-20 mg OD. (max 40 mg/day) ROSUVAS, ROSY"J. ROZUTI 5. tO. 20 mg tabs.

Pitavastatin This is the latest and dose-to-dose the most potent statin. Howeve r, no spec ific advantage compa red to other statins has been demonstrated, and experience with its use is li mited. A ceiling response of 40% LDL-C H reduction w ith the maximum recomme nded daily dose of 4 mg is noted. The plasma t½ is 12 hours. Use o f pitavastatin in combi nation with ge rn fibrozil should be avoi ded, as the latte r dec reases its c learance. Dose: 1-4 mg per day; FLOVAS 1.0, 2.0 mg tabs.

686

DRUGS AFFECTIN G BLOOD AND BLOOD FORMAT ION

Table 46.4: Dose-dependent lowering of plasma low-density ilpoprotein cholesterol (LDL-CH) by statins" Dose mg/day

Reduction in plasma LDL·CH Lovastatin

Pravastatin

Simvastatin

Atorvastatin

5

Rosuvastatin 38%

10

20%

27%

37%

43%

20

25%

24%

32%

43%

48%

40

32%

29%

37%

49%

53%

80

40%

42%'

55%

• High risk of myopathy; this dose of simvastatin not advised. •· Based on NICE (National Institute of Health Care Excellence-UK.) guidelines {2014); nice.org.uklguidancelcg181

Adverse effects All statins arc remarkably well tolerated; ove rall incidence of side effects not d iITering from placebo. otable side e ffects are: Gastro intestina l compla ints and headac he are usually mi ld. Rashes and sleep disturbances are uncommon. Rise in scrum transaminase can occur, but liver damage is in frequent. Monitoring of li ver functi on is recommended. Muscle aches are the commonest (10%) side effect. Rise in CPK levels occurs infrequentl y. Myopathy is the only serious reaction, but is rare (< I per I 000). Few fatalities d ue to rhabdomyolysis are on record. Myopa thy and liver injury is more common when nicotin ic acid/ gemfi brozil or CY P3A4 inhibitor- kctoconazole/ erythromyci n/ cyclosporine/H IV protease inhibi tor is given concurrently. Gemfibroz il inhibits the hepatic uptake of stalins by the organic an ion transporter OATP2 and should not be given with them. Fenofi brate interferes the least with stati n uptake/meta bolism and shou ld be preferred for combining with them. A lower dose of statin is advisable when a fibrate is given concurrently. Stalins should not be given to pregnant women, since there is no data o n their safety. A I0% increase in the risk of diabetes has been noted with the use of statins. Use Statins arc the first choice drugs for primary hypcrlipidaemias with raised LDL-CH and total C H levels, with or wi thout raised TG levels (Type Ila, I lb, V), as well as for secondary (diabetes, nephrotic syndrome) hypercholesterolacm ia. Evide nce for efficacy of stati ns in

reducing rai sed LDL-CH associated mortality and morbidity is robust. Incidence of all types of atherosclerotic cardiovascular disease (ASCV D) includ ing new onset a ngi na, myocardial infarction (M I) or other acute coronary syndromes (ACS), thrombotic stroke, peipheral arterial disease is reduced. Stalins also afford secondary prophylaxis of MI in patients who already had a n attack. Benefic ia l effects in s ubjects w ho have ra ised CH levels but no evide nce of CA D may relate to improved coronary artery comp Iiance and atheromatous p laque stabi lization due to suppression of macrophage mediated inflammation , reducing chances of plaque rup ture and thrombus forma tion. Improvement in endothe lia l function due to increased NO production and reduction in LDL oxidation are other putative mechanisms by which stat ins may exert antiatherosclerotic act ion. Lately, a reduction in venous thromboembolism, especially followi ng knee surgery, has also bee n obse rved w ith rosuvastatin, and it is being used in post-knee/ hip replacement patie nts. On the basis o f these re ult as well as the excellent patient acceptabili ty, the statins are being increasingly used for primary and secondary hypercholcstcrolaemia wi th or without raised TG levels. They are the first choice drugs for dyslipidaemia in diabetics. Stalin therapy is continued indefinitely, unless adverse effects occur. BILE ACID SEQUESTRANTS (Resins) Cholestyramine, Colestipol and Colesevelam These are bas ic ion exchange res ins supplied in the c hloride fonn.

HYPOLIPIDAEM IC DRUGS They arc neither digested nor absorbed in the gut: bind bile acids in the intestine interrupting their enterohepatic circulation. Faecal excretion of bile salts and C H (which is absorbed with the help of bile salts) is increased. This indirectly leads to enhanced hepatic metabolism of CH to bile acids. More LDL receptors arc expressed on liver cells: clearance of plasma I DL, LDL and indirectly that of VLDL is increa ed. Resins have been shown 10 retard atherosclerosis and coronary events, but no significant effect on total mortality. They arc not popular cl inically because they arc unpalatable, inconvenient, have to be taken in large doses. cause flatulence and other g.i. symptoms. interfere with absorption of many drugs and ha,e poor patient acccptabil11y.

LIPOPROTEIN-LIPASE ACTIVATORS (Fibrates) The fibrates (isob utyric acid derivati ves) primarily activate lipoprotein lipase which is a key enzyme in the degradation of V LD L resulting in lowering of c irculating TGs. This effect is exerted through nuclear peroxisome proliferator-activated receptor a (PPARa) that is a gene tran scription regulating receptor expressed in liver, fat and muscles. Activation of PPARa enhances lipoprolein lipase synthesis and fatty acid oxidation. PPARa may also mediate enhanced LDL receptor expression in liver seen particularly with second generation fibrates like beza fibrale, fenofibrate. Fibrates decrease hepatic TG synthesis as well. Drugs in thi s class primaril y lower TG levels by 20-50%, with g reater effect when baseline level is hi gh e r. T hi s is ge nera lly accompanied by I 0- 15% decrease in LDL-C H and a I 0-15% increase in HDL-CH. Jn some patients w ith hypertriglyccridaemia LDL-CH may rise, partly because of inability of LDL receptor to c lear the excess LDL pa rtic les generated by enhanced VLDL catabolism. The increase in HDL-CH is at least in part due to transfer of s urface lipid components from catabolized VLDL to HDL, and partly due to increased producti on of H DL apoproteins (apo A-I, apo A-II) by liver. Gcmfibrozil also appears to reduce VL DL secretion by liver.

Gemfibrozil

This fibric acid derivative effectively lowers plasma TG level by enhancing breakdown and suppressing hepatic synthesis

of TGs. In the ' He ls inki ll ea rt Study' men without known CA D treated with gemfibrozil had a 34% reduction in fatal and nonfatal M I. tho ugh overall mortality was not affected. The beneficial effect was restricted to subjects w ho had high TG levels and lower HDL-CH. That these benefits extend to secondary prevention of coronary events in men wi th existi ng CAD and low HDL-C H, has been demonstrated in another trial.

Pharmacokinetics Gemfibroz il is completely absorbed orally, metabolized by glucuronidati on and undergoes some enterohepatic ci rculation. It is excreted in urine; elimination t ½ is 1- 2 hr. Adverse effects

Common side effects are epigastric distress, loose motions. Skin rashes, body ache, eosinoph ilia, impotence, headache and blurred vision have been reported. Myopathy and hepatitis are uncommon. Gemfibrozil + statin increases risk of myopathy and they shou ld not be combined. Risk of C H-gallstones is slightly increased. Gemfibrozil is contrai ndicated during pregnancy. GE:'Y11'AR, I\OR:'Y10LIP 300 mg cap. LOPID 300 mg cap, 600 mg tabs.

Use In a dose of 600 mg BO taken before meals, gemfib rozil is a first line drug for patients with markedly raised TG levels, whether or not C H leve ls are a lso raised. Episodes of acute pancreatitis are preve nted in patients with chylomicronaem ia and severe hypertriglyceridaemia. It is most efTective in type Ill hyperlipoproteinaem ia; also beneficial in type rv and type V d isease. Patients with ra ised TG a nd low H DL-C H levels (as is the case with metabolic syndrome, type 2 diabetes) are the most suitable 10 be treated with fibrates. Bezafibrate Thi s second generation fibric ac id derivative is an alternative to gemfibrozi l in mixed byperlipidaemias (type lll, IV a nd V). Bezafi brate has not shown propensity to increase LDL-CH in hypertriglyceridaemic patients and a ppears to have greate r LDL-C H lowering action than gemfibroz il. Circulating fibrinogen

687

688

DRUGS AFFECTING BLOOD AND BLOOD FORMATION and gl ucose levels may decrease. The 5 year ' Bezafibrate Coronary Atherosclerosis In tervention Trial ' (BECA lT) in young male post-M l subjects showed an atheroscleros is s lowi ng effect and reduction in coronary events. The Bezafibrate Infarction Prevention (BIP) registry has also noted reduction in coronary events in subjects with high TG and low HDL-CH levels. Adverse effects and contraindications are similar to other fibrates. Main side effects are g. i. upset, myalgia, rashes. Dose reduction is needed in elderly and in renal insufficiency. Action of oral anticoagulants may be enhanced. Dose: 200 mg BD or TDS with meals.

observational study on diabetic dyslipidemia patients. Plasma TG and total C H was lowered, while HDL-CH increased marginally. Fasting plasma glucose as well as I lbA ,c also dropped. Side e ffects were weakness, fever and gastritis. IL did no t appear to cause edema or weight gain , but the whole adverse effect profile of saroglitazar is still to unfold, particularly because PPAR agonists arc known Lo cause many off-target toxicities. Saroglitazar is currently approved on ly in Ind ia for diabetic dyslipidacmia and hypcr-triglyceridaemia not controlled by a stalin. Dose: 4 mg OD LIPAGLY 4 mg lab.

LIPOLYSIS AND TRIGLYCERIDE SYNTHESIS INHIBITOR

BEZALIP 200 mg tab, 400 mg SR tab.

Nicotinic Acid (Niacin)

Fenofibrate Another 2nd generation prodrng fibric acid derivative which has greater HDL-CH raising and greater LDL-CH lowering action than other fibrates: may be more appropriate as an adjunctive drug in subjects with ra ised LDL-CH levels in addition to raised TG levels. No rise in LDL-C H has been observed in patients with high TG levels. Its t½ is 20 hr. Adverse effects are myalgia, he patitis, rashes. Choleli thiasis and rhabdomyolysis are rare. Fenofibrate appears to be the most suitable fibrate for combining with stati ns, because statin metabolism is minimally affected and enhancement of statin myopatby risk is lower. Ind ications of fe no fi brate a re simi lar to that of gemfibrozil.

It is a B group vita min (see Ch. 69) which in much higher doses redu ces plasma lipids. This action is unrelated to its vitamin activity and not present in nicotinamide. When nicoti n ic acid is given, TGs and VLDL decrease rapidl y, fol lowed by a modest fa ll in LDL-C H and total CH. A 20-50% reduction in plasma TGs and 15- 25% reduction in C H levels have been recorded. icotinic ac id is the most effective drug to raise HDL-C H, probably by decreasing rate of HDL destruction; a 20- 35% increase is ge nerally obtained. Relative ly lower dose ( I g/day) suffices to ra ise HDL-CH. Niac in also reduces lipoprotein Lp (a), which is considered more atherogenic. Nicotinic acid inhibits intracellular lipolysis by lipases in adipocytes and decreases Aow of free fatty acids (FFAs) from adipose tissue to liver. This reduces availability of FFAs for production of TGs and VLDL in the li ver. A direct inhibition of hepatocyte TG synthesis is also possible. Indirectly, the VLDL degradation products IDL and LDL are also reduced, but no direct effect on CH and bile acid metabol ism has been found. Enhanced clearance of TGs by lipoprotein lipase contributes to lowering of plasma TGs.

Dose: 200 mg OD with meals. FENOLIP, LI PICARD 200 mg cap.

Fibrate therapy should be discontinued after 2 months if an adequate lowering of TG level is not obtai ned . The Nation al Institute for Health a nd Care Excellence (NICE) of UK guidelines (20 14) di scou rage routine use of fi brates for primary or secondary prevention of CVD, as well as their prophylactic use in patients with chronic kidney disease (CKD) or diabetes. Combination of a fibrate with a statin fo r primary or secondary prevention of CVD is also not recommended. Saroglitazar It is a novel plasma glucose as well as lipid lowering drug tbat is at present marketed only in lndia. It is a dual PPARa and PPARy agonist, but chemically d ilTerenL from thiazolidinediones and fibrates. Saroglitazar has been tested in 3 small clinical trials and one 9 month

A cell surface Gi-protcin coupled receptor which negatively regulates adipoc)'le adenylyl cyclase has been found to selectively bind nicotinic acid, and has been called 'niacin receptor' (also termed GPR 109A). icotinic acid appears lO inhibit lipolysis in adipose lissue by decreasing hormone stimulated intracellular cAMP forrnalion through this receptor.

HYPOLI PIDAEM IC DRUGS

Adverse effects

The large doses needed for hypo lipidaemic action are poorly to le rated. Only a bo ut half of the pa tients a re ab le to take fu ll doses. icotin ic acid is a cutaneous vasodilator: marked flushing, heat and itching (especially in the blus h a rea) occur afte r every dose. This is associated with release of PG D, in the skin, and ca n be m inimized by start ing with a low dose taken with meals and gradual ly increasing as tolerance develops. Use of sustained release (SR/ER) tablet also subdues flushing. Aspirin taken before ni acin su bstantially attenuates flushing by in hibiting PG synthesis. Laropiprant, a prostanoid receptor inhib itor, i a specific antifl ushing drug w ith no hypolipidaemic action of its own, that has been combined with nicoti nic acid to minimize fl ushing. An ER ta ble t containing 1.0 g n icotinic acid and 20 mg laropiprant is used in UK and Europe. Dyspeps ia is very common; vomiting and diarrhoea occ ur when ful l doses a re g iven. Peptic ulcer may be activated . Dryness and hyperpigmentation of skin can be troublesome. Other long-term effects are: Liver dysfunction and jaundice. Serious liver damage is the most important risk. Hype rglycaemia, preci pitation of d iabetes (should not be used in diabetics). Hyperuricaemia and gout, atria l arrhythmias. iac in is contraindicated during pregnancy and in chi ldren.

Interaction

Postural hypotension may occur in patients on antihypertensives when they take nicotinic ac id. R isk o f myopathy due to statins is increased. Dose: Start with I 00 mg TDS, gradually increase to 2~ g per day in divided doses. It s hould be taken just after food to minimi7,c flushing and itching. NIALIP, NEASYN-SR 375, 500 mg tabs.

Use Nicotin ic acid is hi gh ly e fficacious in hypertriglyceridaemia (type III, IV, V) whether associated wit h raised CH level or no t. It is m ost ly used to lower V LD L and raise HDL levels, but outcome benefits in terms of ASCVD reduction are uncertain (see p. 693).

The ICE guidelines (20 14) do not recommend u e of nicoti nic aci d alone or in combination with a stati n for primary as well as secondary prevention of CVD. Jt is also not advised for prophylaxis of CVD in patients with diabetes or CKD. The most important indicati on of niacin is to contro l pancreatitis associated with severe hypertriglyceridaemia, mainly in genetic type IV and type V disorders. Continued use prevents further attacks of pa ncreatitis. Doses above 2 g/day are poorly tolerated. This dose should seldom be exceeded fo r maintenance purposes. Because of potential toxicity, use of nicotinic acid is restricted to high-risk cases only. Acipimox It is a nicotinic ac id derivative that is better to lerated, but has "eaker effect on plasma lipids.

STEROL ABSORPTION INHIBITOR Ezetimibe It is a novel drug that acts by inhibiti ng intestinal absorption of dietary as well as biliary C H and phytosterols by in terfering with a specific C H transporter protein PC IL I in the intesti nal m ucosa. There is compensatory increase in hepatic CH synthesis, but LDL-CH level is lowered by 15- 20%. The enhanced CH synthesis can be blocked by statins, and the two drngs have synergistic LDL-CH lowering effect. Due to very poor aqueous solubility, ezetimibe is not absorbed as such. A fraction is absorbed a~er getting conjugated with g lucuronic acid in the intestinal mucosa. This is secreted in bile and undergoes enterohepatic circulation to be ma inly excreted in faeces. A plasma L1/2 of 22 hours has been calc ul ated. Used alone, ezetimibe is a weak hypocholesterolaemi c drug; LDL-C H loweri ng beyond 15- 20% is not obtained by increasing the dose. Its main val ue is to supplement statins w ithout increasing thei r dose. The combination of ezetimibe + low dose of a statin is as effective in lowering LDL-CH as high dose of statin alone. Upto 60% decrease in LDL-CH level has been obtained with a combi natio n of s imvastatin + ezetimibe. However, in clinical

689

690

DRUGS AFFECTING BLOOD AND BLOOD FORMATION

trials no cardi ovascular ou tcome benefit has y et been dem o nst rated eit her by eze tim ibe mo noth e rapy or by its c ombinatio n with a statin. Onl y in o ne stud y ( IMPROVE-IT) a s ma ll (2%) addition al cardiovascular even t reduction by addin g ezetimibe to simvastatin was no ted. Ezetim ibe is primarily indicated as an adjunct to statin treatme nt of primary hypercholestrolemia, o r to supplement dietary measures when statins are not tolerated. o s pecific adverse effect, except reversible hepatic dysfunction and rarely myositis has been noted with ezetimibe. Dose: 10 mg OD; ZETICA, EZEDOC 10 mg tab. BITORVA, LIP IVAS-EZ, LI PONORM-EZ: Atorvastatin 10 mg ezetimibe 10 mg tab; SJMVAS-EZ, STARSTATEZ: Simvastatin 10 mg + ezetimibe JO mg tabs.

Summary guidelines on the use of hypolipidaemic drugs Raised plasma C H is a major risk factor for coronary artery disease (CAD) and stroke. Higher the CH level, greater is the risk. Abundant data has confinn ed that lowering the level of LDLCH, when the same is hig h, results in lowering of cardiovascu lar mortality a nd m o rbidity. Subsequent evidence (HPS, 2002; ASCOT-LLA, 2003 studies) 1.2 has shown that prophylactic use of a statin in CAD/hypertensive patients even with average or lower than average CH levels decreases coronary and stroke events. With the

avai lability of statins which are e fTecti ve, well to lerated and safe hypolipidaemic drugs, it has become a sta ndard practi ce to prescribe statin therapy after an acute coronary event irrespective of lipid leve ls. Evidence that e levated plasma TG level or low plasma HD L-C H level poses independent high ri sk of CAD and stroke is a lso quite strong now. Whereas ra ised LD L-CH is atherogenic, a hig her HDL-CH level is either itself protective or indicates a low atherogen ic state. The US atio nal C ho lesterol Education Programme (NCEP) in its third report (200 I )3 delineated the optima l levels of plasma li pids and various grades o f hyperlipidaemias (Table 46.5). Lifestyle modification, such as low fat, low c ho lesterol diet, limitation of saturated and trans-fats, regular exerc ise, body weight co ntrol, smoki ng cessation, restriction of alcohol are the prim ary approach, w hether drugs are used or not. All patie nts who a re at risk of CA D or thrombotic stroke should be put on low dose aspirin prophylaxis unless it is contraind icated. The decision to prescribe hypolipidaem ic drugs depends not only on the LDL-C H level and t he type of lipid abnormality, but a lso on associated CA D risk factor(s) or ex isting CAD o r its equ iva le nt like diabetes, peripheral/cerebral vascular disease, etc. in an indiv idual patient (see box).

Table 46.5: Interpretation of plasma lipid levels' Plasma lipid levels (mg/di)

Tota/CH

LDL-CH

HDL-CH

TGs

1. OptimaVdesirable

< 200

< 100 (< 70 for CAD pis)

> 40 (men) > 50 (women)

< 150

2. Borderline high

200-239

130-159

240

160-189

3. High 4. Very high

2: 190

150-199 > 60

20o-499 .!: 500

• Adopted from NCEP (2001)

I. HP (2002, 2004): Heart protection study. Lancet (2002) 360, 7-22, and Lancet (2004) 363, 757-767. 2. ASCOT-LLA (2003): Anglo-Scandinavian cardiac outcomes trial- Lipid lowering arm. lancet (2003) 361, 1149-1 158. 3. CEP-ATP Ill (2004 revision). Grundy SM' et al. Circulation (2004) 110. 227-239.

691

HYPOLIPIDAEMIC DR UGS

Treatment of high plasma LDL-CH The earl ier CEP (200 I ) as well as current American (ACC/ A H A -20 I 3)* and British (N ICE-20 I4) guidelines recommend that all subjects w ho requ ire LDL-C H lowering therapy should be treated with a statin. Several studies compared mortality as well as CA D and stroke prevention benefits of standard vs intensive C H lowering regimens. Metaanalysis by Cholestero l Treatment Trial ists collaborator s and others concluded that standard statin therapy lowering LOH -CH by 3~0% reduces cardiovascu lar events by 30-35%, while i ntensive LDL-CH lowering by ~ 50% curtails cardiovascular even ts by nearly 50% . The JU PITER tri al (2008) demonstrated a 44% reduction in combined end-point o f stroke, M I, unstable angina and cardiovascu lar death by using high potency rosuvastatin. Since statins cause a dose dependent reduction in LDL-CH , the current guidelines divide statin therapy into 3 categories, viz. ' high intensity', ' moderate i ntensity' and ' low intensit y'. The LDL-CH reduction achieved by these 3 ca tegories of statin therapy is compil ed in Table 46.6. Not withstanding minor differences i n cri teria** both guidelines recogn ize atorvastatin (20-80 mg/day) and rosuvastatin ( I 0-40 mg/day) to be the drugs for high intensity statin therapy.

Risk factors for coronary artery disease•

• Men > 45 years, women > 55 years • Family history of Ml/sudden cardiac death before 55 year (men), 65 year (women) age in first degree relative • Smoking • Hypertension (BP > 140/90 or use of antihypertensive medication) • Diabetes mellitust • Low HDL-CH (< 40 mg/di in men,< 50 mg/di in women) • High LDL-CH ( 2?: 160 mg/di) or total CH 2?: 240 mg/di • Obesity (BMI > 25 Kg/m2) 1 or waist> 40' (men), > 35' (women) • Adopted from the NCEP-ATP 111 (2001) r Diabetes is considered equivalent to existing CAD 1 Nol included in NCEP guideline (2001} Lower doses of these two statins as well as all others (in their theraputi c dose range) may be used for low-to-moderate intensity statin therapy. O n the basis of random ized clinical trials (RCTs) and metaanalysis the ACC/A H A guidelines (20 13) have identified 4 major groups of subjects who are most likely to obtain a net benefit in term s of ASCVD risk reduction vs potential for adverse effects. T hese groups have

Table 46.6:Oaily doses of statIns for high. medium and low IntensIty stalln therapy Stalin therapy

High intensity

Moderate intensity

Low intensity

Guideline source

ACC/AHA'

NICE2

ACC/AHA

NICE

ACC/AHA

NICE

LDL-CH reduction

2?:

50%

>40%

30-50%

31-40%

75 yr: Moderate intensity statin • High intensity statin (moderate intensity statin if high intensity statin not tolerated or C/1) • Moderate intensity statin or • High intensity statin if estimated 10 yr ASCVD risk 7.5% • Moderate-to-high intensity statin therapy

2. No clinical ASCVD, but plasma LDL-CH 190 mg/dl (primary hypercholesterolemia) 3. Diabetics aged 40-75 yr with LDL-CH 70-189 mg/dl and no clinical ASCVD 4. Non-diabetics aged 40-75 yr without clinical ASCVD, with LDL-CH 70-189 mg/dl and estimated 10 yr ASCVD risk 7.5%

• Based on ACC/AHA guidelines 2013 ASCVD-Atherosclerotic cardiovascular disease; C/1-contraindicated been as igned to high intensity and moderate intensity stati n therapy which is expected to afford optimum benefit (see box). Low intensity statin therapy has not been recommended, but may be appropriate on the basis of to lerability or other criteria in individual patients. In summary, current indications for statin therapy are: I. All subjects with ASCV D irrespective of plasma CH level. 2. All subjects with LDL-CH above 190 mg/dL. 3. All diabetics aged 40 yea rs or more. 4. Subjects aged 40 years or more with estimated 10 yr ASCVD risk more than 7.5%, irrespective of plasma CH level. lntensive lipid lowering by adequate dose of statin is considered to imporove endothelial function and stabilize atherosclerotic plaques in addition to the antiatherosclerotic effect; all of these leading to reduction in CA D, sn·okc and death.

Treatment of low HDL-CH level Epidemiological data has shown that most patients with premature CAD have low HDL-CH level. Recent trials have shown that statin therapy reduces CAD endpoints in subjects with low HDL-CH even though LDL-CH may be in the normal ra nge. Most low HDL-CH subjects have metabolic syndrome (obesity, hypertriglyceridaemia, insulin resistance/diabetes. hypertension). Therapy

directed towards components of this syndrome often helps to normalise HDL-CH. In addition to these measures, the primary approach of therapy in ubjects with low HDL-CH is to reduce LDL-C H by intens ive statin therapy. one of the currently available lipid modifying drugs has a marked effect to raise HDL-CH, but nicotinic acid has the highest efficacy followed by fibrates. However, addition of these drugs to statin therapy has not been unequivocall y demonstrated to fu rther reduce ASCV D. In two recent randomized studies AIM-HIGH (201 I) and HPS-2 THRIVE (20 14) addition of nicotinic acid to statin therapy failed to improve outcome end points, while adverse effects increased. Patients in these trials already had low LDL-CH levels (due to statin therapy). Therefore, it has been suggested that niaci n may benefit if the LDLCH remains high despite statin therapy. Treatment of raised TG level: Though elevated plasma TG has been recognized as an independent risk factor for CAD, it remains to be established that specific drug therapy to lower plasma TGs alo ne improves CAD outcomes. Treatment strategy for hype rt rigl yceridaemia depends on its severity and upon its cause (obesity. physical inactivity, high carbohydrate- high calorie diet, saturated and trans fats, alcohol, smoking, diabetes, kidney disease, drugs like

HYPOLIPIDAEM IC DRUGS corticosteroids, estrogens, high dose blockers, and genetic disorders). Reversible causes (from the above list) must be addressed first. The initial drug treatment is to reduce LDL-CH by statin therapy. This may itsel f decrease plasma TG level. Pa ti e nts w hose fas t ing plas ma T G rema ins above 500 mg/dL ins pite of th e above measures run the risk o f developing acute pa nc reatitis, especia lly a fter a fa tty meal or a lco ho l intake. Treatment w ith a specific TG lowe ring drug , viz. ni cotinic ac id or fibrate (pre ferably fe nofibrate) is ind icated in addi-

tion to the statin w ith the a im of preventing acute pancreati tis . The CAD and stroke preventing benefit o r specific TG lowering therapy (by fi bra le or niacin) for subj ects w hose plasma TG remains high (between 200-499 mg/d L) despite statin therapy is uncertain and discretionary. The N IC E (20 14 ) gu idelines advi se against rout ine a ddition of fi brates or ni cotin ic acid to a statin for preventing A CV D. Ex tra vig ilance is req u ired to g uard against ri sk o r myo pathy whe n n iac in or fi bra te is added to a statin.

r:r PROBLEM DIRECTED STUDY 46.1 The routine medical checkup of a SO-year-old male, asymptomatic, non-smoker business executive with sendentary job and no past history of coronary artery disease or stroke, as well as no family history of premature cardiac death has yielded t he following findings: Body mass index- 27, waist circumferance-92 cm (38" ), BP-130/86 mm Hg, fasti ng blood glucose-98 mg/di, total plasma cholesterol (CH) 268 mg/ di, LDL-CH 198 mg/ di, HDL-CH 38 mg/ di, serum triglyceride 160 mg/di. Liver, kidney and thyroid function test values and ECG are within normal limits. There are no remarkable find ings on physical examination. (a) Apart from counselling on life-style modification, does this person require any medication? (b) In case he needs medication, which drug and dose would be appropriate? What should be th e goal of drug therapy? (see Appendix-1 for solution)

693

GASTROINTESTINAL DRUGS

Chapter

47

Drugs for Peptic Ulcer and Gastroesophageal Reflux Disease

PEPTIC ULCER Peptic ulce r occurs in that part of the gastrointestina l tract (g.i.t.) which is exposed to gastric acid and peps in, i.e. the stomach and duodenum. The e tiology o f pe ptic u lce r is not c lea rl y known. It results probably due to an imbalance betwee n the aggressive (ac id, pepsin , bile and H. pylori) and the defensive (gastric mucus and bicarbonate secretion, prostaglandins, nitric oxide, high mucosa I blood flow, innate resista nce o f the mucosa( cells) factors. A variety of psychosomatic, humoral and vascular derangements have been impli cated and the causati ve role of Helicobacter pylori infection in majority of duodenal ulcers and many gastric ulcers (which are not due to SAlDs) has been realized. In gastric ulcer, generally acid secretion is normal or low, whi le deficient mucosa( defence (mostly impaired mucus and bicarbonate secreti on) plays a greater role. In duodenal ulcer, acid secreti on is hi gh in about ha lf o f the patie nts but normal in the rest. 1 otwithstanding whether production o f acid is norma l or high, it does contri bute to ulce ration as an agg ressive factor, redu cti on of w hi c h is the ma in approach to ulcer treatment. An understa ndi ng of the mechani sm and control of gastric acid secretion will eluc idate the targets of antisecretory drug action.

Regulation of gastric acid secretion The mechanisms operating at the gastric parietal ce lls are summari.ted in Fig. 47. 1. The tcnninal enzyme 1-1 K ATPase (proton pump) which secretes II ' ions in the apical canahcuii of parietal cells can be activated by hi~tamine. ACh and gastrin acting via their own receptors located on the basolaterai membrane of these cells. Out of the three physiologica l secrctagogues, histamine. acting through H 2 receptors, plays the dominant role, because the other two, gastrin and ACh act paniy directly and to a greater extent ind irectly by releasing histamine from paracrine enterochromaffin-iike (ECL) cells called "histaminocytes·• located in the oxyntic glands. While H.. receptors activate WK"ATPase by generating cAMP, mus~arinic and gastrin/ cholccystokinin (CCK,) receptors appear to function through the phospho lipasc C • IP,-DAG pathway that mobilizes intracellular Ca 2' . TI1e cAM P mediated proton pump activation also involves Ca'· . The secretomotor response to gastrin and cholinergic agonists is expressed fully only in the presence of cAMP generated by H1 activation. As such. histamine pan icipates in the acid response to gastrin and ACh at more than one levels, and H1 antagonists suppress not only histamine, but also ACh, pentagastrin and in fact any gastric acid secretory sti mulus. Gastrin is secreted from the antrum in response to rise in antral pH, food constituents, especial ly peptides, and vagally mediated reflexes involving ganglion cells of the enteric nervous system (ENS). The postgangiionic ENS neurones elicit gastrin release from gastrin secreting 'G' cells by elaborating ACh as well as gaslrin releasing peptide (G RP). The dominant mu~carinic receptor mediating vagai responses is of the M 1 subtype. Its locauon on the ganglion cells of the intramural plexuses has been confim1ed. The parietal cell and ECL cell musearinic receptor 11 years w ithout any problem, it may appear prudent to be apprehensive of prolonged achlorhydria, and if possible, a,oid long-tenn use of PPls.

DRUGS FOR PEPTI C ULCER AND GASTROESOP HAGEAL REFLU X DISEASE Lately, few reports of gynaecomastia and erectile dysfunction (possibly due to reduced testosterone level) afler prolonged use of omeprazolc have appeared.

lansoprazole: risk of drug interactions is mi nimal. DoJe: 40 mg OIJ; PANTOCID, PANTODAC 20, 40 mg entcric coated tab; PANTIUM , PANT IN 40 mg tab, 40 mg

Interactions Omeprazole inh ibits oxidati on of

inj for i.v. use.

certain drugs: diazepam, phenytoin and warfarin levels may be increased. lt interferes with activation of clopidogrel by inhi biting CYP2C 19 (see p. 678). Reduced gastric acid ity decreases absorption of ketoconazole and iron salts. Clarithromyc in inhibi ts omeprazolc metabolism and increases its plasma conce ntrat ion.

S-Pantoprazole It is the active single enantiomer, twice as potent as the race mate.

OMIZAC, NILSEC 20 mg cap. Ol\1EZ, OCID, OMEZOL 10. 20 mg caps, PROTOLOC 20, 40 mg caps containing enteric coated granules. Capsules must not be opened or chewed; lo be taken in Lhe morning before meab. OCID, OMEZ 40 mg per amp inj.

Esomeprazole

It is the S-enan tiomcr of

omeprazole; c la imed to have higher oral bioa vai labi lity a nd to produce better contro l of intragas tric p H tha n omeprazole in GE RO patients because of slower elimination and lo nger t ½. Higher heal ing rates of e rosive esophagi tis and bette r G ERD symptom re lie f have been reported in comparative trials with omeprazole. Side e!Tect and drug interaction profile is similar to the racemic drug. Dose: 20-40 mg OD; NEX PRO, RAC IPER, IZRA 20, 40 mg tabs. RACIPER- IV 40 mg/5 ml inj.

Lansoprazole

Somewhat more pote nt than omeprazole but simi lar in properties. Inh ibition of W K' AT Pase by lan soprazo le is pa rtl y reversib le. It has higher ora l bioavailab ility, faster onset of action and slightly longer t½ tha n omeprazole. Dose should be reduced in liver d isease. S ide e ffects are s im ilar, but drug interactions appear to be less signifi cant; diazepam and phenytoi n metabo lism may be reduced. Ulcer healing dose: 15- 30 mg OD; LANZOL. LANZAP. LEVANT, LANPRO 15, 30 mg caps.

Pantoprazole It is similar in potency and cli n ical efficacy to omeprazole. but is more ac id stable and has higher oral bioavailabil ity. Injected i. v., pantoprazole has been particularly e mpl oyed in b leed ing pept ic ulcer and for prophylaxis of acute stress ulcers. Affinity for cytochrome P450 is lower than omeprazole or

PANPURE, ZOSECTA 20 mg tab.

Rabeprazole

This PPI is claimed to cause fas ter ac id suppression. Due to higher pKa, it is more rapidly converted to the acti ve species. However, potency and efficacy a re sim ilar to omeprazole. o sign ificant drug interactions have been observed . Dose: 20 mg OD; ZE syndrome - 60 mg/day. RA0LET, RABELOC, RABICIP, RAZO, HAPP! 10, 20 mg tab, 20 mg/vial inj.

Dexrabeprazole It is the active dextro-isomer of rabeprazole; produces similar acid suppression at half the dose, i.e. 10- 20 mg da ily. DEXPURE 5, 10 mg tabs.

llaprazole

T his PPI developed in Ko rea, has been approved fo r use in India fo r peptic ul cer, GE RO and dyspe ps ia. Ulcer heal in g efficacy and tolerability is similar to omeprazole, but dose-to-dose it is twice as potent. It is excreted in uri ne, both as unchnaged drug and as sulfone metabolite. Dose: 5 20 mg OD before breakfast or before dinner. ILATOP, ILACOOL. ILAGATE, ADIZA 5 mg, 10 mg tab.

Uses of PPls 1. Peptic ulcer: PPls in thei r recomme nded doses, e.g. omeprazole 20-40 mg O D a re more effective than H2 b lockers. Rel ief of pain is rapid (within 1- 2 days) and excellent. Faster healing has been de monstrated with 40 mg/day than wit h 20 mg/day. Some duodenal ulcers heal even at 2 week and the remai ning (over 90%) at 4 weeks. Gastric ulcer generally requi res 4-8 weeks. PPls have achieved healing of ulcers in many patients w ho did not respond to H1 blockers. Maintenance therapy with lower doses (e.g. omeprazole 20 mg dai ly or thrice weekly) can prevent ulcer relapse. PPls are an integral compone nt of anti- H. pylori therapy (see p. 705- 06). PP ls are the drugs o f choice for NSA ID induced gas tric/ duodenal ulcers.

701

702

GASTROINTESTINAL DRUGS Healing may occur despite continued use of the SAID. Howe ver, higher dose or twice da ily treatment given for longer periods are generally required. Whenever possible, the NSA ID should be d iscontinued. lf that is not possible, it is advisable to switch over to a COX-2 selective NSA ID. Mainte na nce PPI treatment reduces recurren ce of SAID associated ulcer. Ulce r comp lications like perforation, a nd bleeding are also reduced. 2 . Bleeding peptic ulcer: Acid enhances clot dissolution promoting ulcer bleed. Suppression or gastric ac id has bee n found to faci li tate clot formation reduci ng blood loss and rcblccd. High dose i. v. PPI therapy (pantoprazole or esomcprazole or rabeprazo le 80 mg bolus injection, followed by 8 mg/hr infusion for 3 days) profoundly inhibits gastric acid, and has been shown to reduce rebleeding aller therapeutic endoscopy. Even in cases w here the bleeding vessel could not be visuali zed , i.v. fo llowed by oral PPI reduces recurrence of bleeding and need for surgery.

3 . Stress ulcers: Intravenous pantoprazole/ rabeprazole are at least as effective prophylactic ( if not more) for stress ulcers as i.v. H2 blocke rs (see p. 699). Though one metaanalysis has shown PPls to be more effective. both classes of drugs continue to be used.

4. Gastroesophageal reflux disease (GERO): PP ls are th e drugs of choice (see p . 707). Hi gher doses than that for peptic ulcer, or twice daily dosing is generally needed. 5. Zollinger-Ellison syndrome: It is a gastric hypersecretory state due to 'gastrinoma', a rare n1mour secreting gastrin , which presents with difficult to bcal gastric/duodenal ulcer(s). High dose oral PPI (60-120 mg,'day in 2 doses) is the drug of choice for controlling sy mptoms. Complete symptomatic re lief and healing of ulcers ca n be achieved by a dose of the PPI which lowers the daily acid output to within normal limits. Inoperable/ metastatic cases have been treated fo r >6 yea rs with susta ine d benefi t and no se ri ous toxic ity. Definiti ve treatment is resection o f the tumour, if possible. Other gastric

hypersecretory states like syste mic mastocytosis endocrine adenomas, etc. also respond well. 6 . Aspiration pneumonia: PPis are an alternative to H2 blockers (see p. 699) fo r prophylax is ofaspiration pneumonia due to prolonged anaesthesia. ANTICHOLINERGICS (SEE CH. 8)

Alropinic drugs reduce the volume of gastric juice withoul raising its pl I un less there is food in stomach to dilute the secreted acid. Stimu lated gastric secretion is less completely inhibited. Effective doses (for ulcer healing) of antimuscarinic drugs (atropine, propantheline, oxyphcnonium) invariably produce intolerable side effects. Introduction of I12 blockers and PP ls has sent them into oblivion.

Pirenzepine (see p. 129) II is a selecuvc M anticholinergic that "as used in Europe for peptic ulct:r. Though antimuscarinic side effects were le,s marked. more effective H2 blockers and PPls have rendered it redundant. PROSTAGLANDIN ANALOGUE

PGE2 and PG l1 are produced in the gastric mucosa and appear to serve a protective role by inhibiting acid secretion and pro11101ing mucus as well as I-ICO3 secretion (see Ch. 13). In addition. PGs inhibit gastrin release. increase mucosa I blood flo\\ and probably hm e an illdefined "cytoprotecti, e" action. However, their abil ity to reinforce the mucus layer covering gastric and duodenal mucosa which is buffered by I ICO3- secreted into thi s layer by the underlying epithelial cells appears to be the 1110,1 important. Natural PGs have very short t½. Misoprostol (methyl-POE , ester) is a longer acting synthetic PGE1 derivative which inhibits acid output. However, reduction in 24 hour acid production is less marked than H, blockers because of shorter duration of action (- 3 hr.) Ulcer.healing rates comparable to cimctidine have been obtained in 4- 8 weeks, but misoprostol is poorer in relieving ulcer pain. ome patients may even complain of increased pain during the first week of therapy. Dose: 200 /lg QID; CYTOLOG 200 µg tab; MISOPROST 100 ftg, 200 µg tabs. Major problems in the use of misoprostol are- 4 evoke reflex gastrin release • more acid is secreted. especially in patients wi th hyperacidity and duodenal ulcer; "acid rebound'. occurs and gastric motili ty is increased. The potency of an antacid is genera lly expressed in terms of its acid 11e111ralizi11g capacity (A , C). "hich is defined as number of mEq of IN I ICI that are brought to pH 3.5 in 15 min (or 60 min in some tests) by a unit dose of the antacid preparation. This takes into consideration the rate at which the antacid dissolves and reacts with I ICI. This is important because a single dose of any antacid uikcn in empty stomach acts fo r 30-{;0 min only before it is passed into duodenum. Taken with meals antacids may act for at the most 2 3 hr.

Systemic Antacids Sodium bicarbonate It is water soluble. acts instantaneously. but the duration of action is short. It is a potent neutraliLer ( I g • 12 mEq HCI), pH may rise above 7. However, it has several demerits: (a) Absnrbed systemically: large dm,es "ill induce alkalosis. (b) Produces CO, in stomach • d istention, discomfort, belching, risk-of ulcer perforation. (c) Acid rebound occurs, but is usually short lasting. (d) Increases Na load: may worsen edema and CHF. Use of sod. bicarbonate is restricted to casual treatment of hcanbum. It provides quick symptomatic relief. Other uses are to a lkalinize urine and to treat acidosis. Sodium citrate Properties similar to sod. bicarbonate; I g neutraliLes IO mEq IICI; C01 is not evolved. Nonsystemic Antacids These are insoluble and poorly absorbed basic compounds; react in stomach to form the corresponding chlo ride salL The chloride salt again reacts with the intestinal bicarbonate so that I ICO3 is not spared for absorption- no acidbase diMurbance occurs. I lowever. small amounts that arc absorbed have the same alkaliniLing effect as NaHC'O,. Mag. hydroxide has low water solubility: its aqueous suspension (milk of magnesia) has low concentration of O H ions and thu; low alkalinity. However, it reac ts with I ICI promptly and is an efficacious antacid ( I g • 30 mEq HCI). MILK 0 1 MAGNES IA 0.4 g/5 ml suspension: 5 ml ncutrali7cs 12 ml::.q acid.

Magnesium trisilicate has low solubility and reactivity: I g can react with IO mEq acid, but in clinical use only about I mEq is neutralized.

Abo ut 5% of administered Mg is absorbed systemically-may cause problem if rena l functiun is inadequate. All Mg salts have a laxative action by generating osmotically acll\c MgCI, in the stomach and through Mg'' ion induced cholccystokinin relea,e.

Aluminium hydroxide gel It is a bland, weak and slowly reacting antacid. On keeping it slowly polymerizes lo \ ariablc extents into still less reactive forms. Thus. the ANC of a preparation gradually declines on storage. Accordingly, 5 ml of its suspension may neutraliLe j ust I mEq HCI. As such, little ,, orthwhile acid neutralization is obtained at conventional doses. The AP ' ions relax smooth muscle. Thus. it delays gastric emptying. Alum. hydrox. frequently. causes constipation due to its smooth muscle relaxant and mucosa! astringent action. Small amount of Al'· that is absorbed is excreted by kidney. This is impaired in renal failure- aluminium toXcicity (encephalopathy. osteoporosis) can occur. ALL, DROX 0.84 g tab. 0.6 g, 10 ml susp.

Magaldrate It is a hydrated complex of hydroxymagnesium aluminate that initia lly reacts rapidly with acid and releases alum. hydrox. which then reacts more slowly. TI1e fre hly relea5cd alum. hydrox. is in the unpolymerizcd more rcacti, e form. Thus, magaldrate cannot be equated to a physical mixturt: of mag. and alum. hydroxides. It is a good ant.acid with prompt and sustained neutralizing action. Its ANC is e timated to be 28 mEq I ICl/g. STAC II) 400 mg tab, 400 mg/5 ml susp.; UL.GEL 400 mg -.uh 20 mg s1meth 1cone per tab or 5 ml susp. Calcium carbonate It is a potent and rapidly acting acid neutrali7er ( I g • 20 mEq I ICI), but ANC of commercial preparations is less and \ ariable due to di fTering particle size and crystal structure. Though it liberates CO. in the stomach at a slo\\ er rate than aHCO1 , it can cause distention and d iscomfort. The Ca' ions are pa11ly absorbed. The greatest drawback of CaCO1 as an antacid is that C'a'· 1ons diffuse into the gastric mucosa- increase HCI production directly by paricuil cells as well as by releasing gastrin. Acid rebound occurs. Milk alkali syndrome In the past, large quantity of milk was prescribed with CaCO1 (or Nal ICO,) for peptic ulcer. Such regimen often produced a syndrome characterized by headache, anorexia, weakness. abdo minal discomfort, ahnom1al Ca deposits and renal stones due to concurrent hypcrcalcaemia and alkalosis. It is rare now.

Antacid combinations A com binati on of two or more antacids is frequently used. These may be superior to a ny s ingle agent on the fol lowing accounts: (a) Fast (Mag. hy d rox.) a nd s low (A lu m . hydrox.) a cting components y ie ld prompt as well as sustained effect.

703

704

GASTROI NTESTINAL DRUGS (b) Mag. salts are laxative, whi le alum. salts are constipating: combination may annu l each oth er's action and bowel movement may be least affected. (c) Gastric emptying is least afTected; wh ile alum. sa lts tend to delay it, mag./cal. sa lts tend to hasten it. (d) Dose of individual components is reduced; sys temi c toxicity (dep ende nt on fractional absorption) is minimized. Some available antacid combinations a re: ACIDIN: Mag. carb. 165 mg, dried alum. hydro:\. gd 232 mg, cal. carb. 165 mg. sod. bicurb. 82 mg, with kaolin 105 mg and belladonna herb 30 µg per tab. AL\1ACARB: Dried alum. hydrox . gd 325 mg. mag. carb. 50 mg, methyl polysilox. 40 mg, deglycyrrh17mntcd liquorice 380 mg per tab. ALLUJEL-DF: Dried alum. h;drox. gel 400 mg, mug. hydrox. 400 mg , methyl polysilox. 30 mg per 10 ml susp. DIGE, E: Dncd alum. hydrox. gel 300 mg, mag. alum. s1hcate 50 mg, mag. hydrox. 25 mg, methylpolysilo,. 10 mg per tab. DIGENE GEL: Mag. hydrox. 185 mg, alum. hydro;,.. gd 830 mg, sod. carboxymethyl cellulose 100 mg, methylpolysilox. 25 mg per IO ml susp. GELUSIL: Dried alum. hydro:\. gel 250 mg, mag. trisilicate 500 mg per tab. GELUSIL LIQUID: Mag. trisilicatc 625 mg, alum. hydrox. gel 3 I 2 mg per 5 ml su,p. MUCAIN E: Alum. hydrox. 290 mg, mag. hydro:\. 98 mg, oxetha7ame 10 mg per 5 ml susp. TRICA INE-MPS: Alum. hydrox. gel 300 mg, mag. hydrox. 150 mg. oxethazaine IO mg. simeth1cone IO mg per 5 ml gel. MAYLOX : Dried alum. hydro:\. gel 225 mg, mag. hydrox. 200 mg, dimethicone 50 mg per tab and 5 ml susp. POLYC ROL FORTE GEi .: Mag. hydrox. 100 mg, dried alum. hydrox. gel 425 mg, methylpolysilo:\. 125 mg per 5 ml s usp.

Drug interactions By raising gastric p H and by forming complexes, t he non-abso rbable antacids reduce the absorption of many drugs, especially tetracyc lines, iron salts, fluoroq uinolones, kc toconazole, I-1 2 blockers , diazcpam, phenothiazines, indomethacin, phenytoin, isoniazid, ethambutol and nitrofurantoin. Stagger their administration by 2 hours. The efficacy of nitrofurantoi n is reduced due to alkalinization of urine by the antacid. Uses Antacids are no longer used for healing peptic ulcer, because they are needed in

large and frequent doses, a re inconvenie nt, can ca use acid rebound a nd bowel upset, afford little nocturna l protection and have poor patient acceptability. They are now employed only for intercurrent pain relief, dyspepsia and ac idi ty, mostly self-prescribed by the patie nts as over-the-counte r preparations. Antacids conti nue to be used for nonulcer dyspepsia and minor episodes of heartburn.

Gastroesophagea/ reflux Antacids afford faster symptom re lief than drugs whic h inhibit acid secretion, but do not provide sustained benefit. They may be used off-and-on for acid eructation and heartburn. ULCER PROTECTIVES Sucralfate It is a basic aluminium salt o f sulfated sucrose, which is a drug of its own kind. Sucralfotc polymerizes at pH < 4 by cross linking of molecules. assuming a sticky gel-like consistency. It preferentially and strongly adheres 10 the ulcer base, especially duodenal ulcer, and has been ,een endoscopically to remai n there for - 6 hours. Surfoce proteins at the ulcer base are precipitated, together with which it acts as a physical barrier preventing acid, pepsin and bile from coming in contact with the ulcer base. Dietary proteins get deposited on this coat, forming another layer. Sucralfate has no acid neutralizing action, but delays gastric emptying. so that its own stay in s1omach is prolonged. Augmented gastric mucosa! PG synthesis may supplement physical protective action of sucralla te. Sucralfate is minimally absorbed aficr oral adm inistration. Its action is entirely local. It promotes healing of both duodenal and gastric ulcers. Healing efficacy has been found similar to cimctidine at 4 weeks, and may be superior in patients who continue to smoke. However, s ucralfate is infrequently used now because of need for 4 large welltimed daily doses and the availability of simpler and more effective H, blockers/PPls. Dose: The- ulcer healing dose o f s ucralfate is I g taken in empty stomach I hour before the 3 major meals and at bed time for 4 8 "eeks. Antacids should not be taken with sucralfate because its polymerization is dependent on acidic pH. ULCERFATE, 5UCRAC E. RECULFAT[, I g tab.

Side effects are fe"; constipation is reported by 2% paticn1s. It has potential for inducing hypophosphatemia by binding phosphate ions in the intestine. Dry mouth and nausea are infrequent. Other uses Bile renux, gastritis and prophylaxis of stress ulcers. In intensive care units. acid suppressant prophylaxis of tress ulcers (" ith i.v./intragastric H, blocker or PPI) is

DRUGS FO R PEPTIC U LCER AND GASTROESOPHAGEAL REFLUX DISEASE almost routinely used now. This practice is considered to contribute to occurrence or pneumonia due to overgrowth of bacteria in the stomach. lntragastric sucralfate administered as a thick suspension provides efTectivc prophylaxis of stress ulcers without acid suppression, and is an alternative to i.v. H, blocker/ PPL As a suspension in glycerol. it has been tried in stomatitis. A topical fom1Ulation ofsucralfate PEPS IGARD LIGHT GEL is available as a protective for application on bums. bedsores, diabetic/radiation ulcers, excoriated skin. etc.

Interactions Sucralfatc adsorbs many drugs. It interferes with the absorption of tetracyclines, nuoroquinolones, cimetidine. phenytoin and digoxin. Antacids given concurrently reduce the efficacy of sucralfate.

Colloidal bismuth subcltrate (CBS; Tripotassium dicitratoblsmuthate) It is a colloidal bismuth compound; water soluble but precipitates al pH < 5. It is not an antacid but heals 60% ulcers at 4 weeks and 80-90% at 8 weeks. The mechanism of action of CBS is not clear; probabilities are: • May increase gastric mucosa! PGE,, mucus and Hco; production. • May precipitate mucus glycoproteins and coal the ulcer base. • May detach and inhibit II. pylon directly. Gastrit is and nonulcer dyspepsia associated with H. pylori are also improved by CBS. The regimen for CBS is 120 mg (as Bi,O 3) taken ½ hr before 3 major meals and al bedtime for 4-& weeks. 1'vlilk and antacids should not be taken concomitantly. TRYMO, DENOL 120 mg tab. Most of the ingested CBS passes in the faeces. Small amounts absorbed are excreted in urine. Side efTects are diarrhoea, headache and dizziness. Patient acceptance of CBS is compromised by blackening or tongue, dentures and stools; and by inconvenience o f the dosing schedule. Presently. it is used occasionally as a component of triple drug anti-If. pylori regimen.

ANTI-HELICOBACTER PYLORI DRUGS H. pylori is a gram negative bacillus uniquely adapted to survival in the hostile environment of stomach. It attac hes to the surface epithelium beneath the mucus, possesses high urease activity- produces ammon ia which maintains a neutral microenvironment around the bacteria, and promotes back diffusion of H+ ions. It has been found as a commensal in 20-70% nonnal individuals, and is now accepted as an important contributor lo the causation of chronic gastritis,

dyspepsia, peptic ulcer, gastric lymphoma and gast ri c carcino ma. H. pylori in fec tion starts with a neutrophilic gastritis lasting 7- 10 days which is usually asymptomatic. Once established, H. pylori generally persists for the life of the host. Up to 90% patients of duodenal and gastric ulcer have tested pos itive for If. pylori. Eradication of H. pylori concu rrentl y with PPI therapy of peptic ulcer has been associated with faster ulcer healing a nd largely prevents ulcer relapse. All H. pylori positive ulcer patients should receive H. pylori eradication therapy. In the absence of H. pylori testing, all cases with failed conventional ulcer the rapy and relapse cases must be given the be nefit of H. pylori eradication. Antimicrobials that are used clinically against H. pylori are: amoxicillin, clarithromyc in, tetracycline and metronidazole/ tinidazole. However, any single antibiotic is ineffective. Resistance deve lops rapidly, especially to metronidazole/ tinidazole and to clarithromycin, but amoxicillin resistance is infrequent. In tropical countries, metronidazole resistance is more common than clarithromycin resistance. Since bismuth (C BS) is active against II. pylori and res istance does not deve lop to it, co mbination regimens including bismuth may be used in case o f metronidazole and clarithromycin d ouble resistance. R outin e use of CBS is precluded by poor patient acceptability. Ac id suppression by PPJs enhances effecti veness of anti -H. pylori antibiotics, and optimum benefits are obtained when gastric pH is ke pt >5 for at least 16-18 hours per day. This is a higher degree of round-the-clock acid suppress ion th an is needed for duodenal ul ce r healing or for reflu x esophagitis. Only twice daily PPI dosing can achieve this degree of acid suppression. The PPis benefit by altering the acid environment for H. pylori as well as by direct inh ibitory effect. One of th e PPls is an integral component of a ll anti-H. pylori regimens along with 2 (triple drug) or 3 (quadruple drug) antimicrobial s.

705

706

GASTROI NTESTI NAL DRUGS

Anti-H. pylori Regimens Proton pump inhibitor

Amoxicillin

Clarithromycin

MetronidazoleJ

One week regimens*

Omeprazole (20 mg BO) or

1.0 g BO

500 mg BO

Esomeprazole (20 mg BO) or Lansoprazole (30 mg BO) or

250 mg BO

400 mg BO

1.0 g BO

400 mg BO

500 mg TOS

400 mg TOS

Pantoprazole (40 mg BO) or Rabeprazole (20 mg BO) Omeprazole {20 mg BO)

Two weeks-twice daily regimens

Omeprazole (20 mg) or

750 mg

Lansoprazole (30 mg) or Pantoprazole (40 mg)

400 mg 250 mg

750mg

400mg

500 mg

• One week regimens adopted from British National Formulary (BNF) Sept. 2016 s Tinidazole 500 mg BO may be used alternatively.

A number of 3 drug regi mens of I or 2 weeks are being used (see box). For majority of cases, especia lly initially, any one of the tripl e drug one week therapy is recommended. On a n average this achieves H.pylori eradication in upto 85% cases. T he two week triple d rug regimens do offer the possi bility of higher eradication rate (upto 96%) but they produce more adverse e lTects, are more ex pensive and complia nce is poor. They should be reserved for patients not ach ieving complete e radication with I week regi me ns. The US- FDA approved regimen is: Lansoprazole 30 mg + Amoxicillin I 000 mg + clarithromyc in 500 mg, a ll given twice daily for 2 weeks. This has achi eved 86- 92% eradication rate. Beller to lerabili ty of regi mens wh ic h exclude a nitroimidazole favour using amoxicillin I- clarit hromycin + PP I, pa1ti cula rly in India where metronidazole resistance is more prevalent. It is advised that patients who in the near past have received a nitroimidazole fo r anothe r infection should be treated with PP I + amoxici ll in + c larithromycin. Similarl y, if a patient has recei ved a macro lide antibiotic, the regi men sho uld consis t of PP! + metronidazole/tinidazo lc + amoxicillin.

For large ulcers (> IO mm in d iameter) or those complicated by bleeding/perforation, the PPI shou ld be continued beyond the 2 weekstriple drug regimen till complete healing occurs.

Quadruple therapy with CBS 120 mg QJD + tetracycline 500 mg Q ID + metron idazole 400 mg TDS + omeprazole 20 mg 8D is advocated for eradication failure cases. Another quadruple therapy regi men with PPI + amoxic ll in + clarithromycin + metronidazole a ll twice daily fo r 2 weeks has been tested in Europe with >90% eradication rate. All regimens a re com plex and expensive, side effects are frequent and compliance is poor. Higher fail ure rates (20-40%) have been reported from India. Also, 5 year recurrence rate of H. pylori infection is higher. Three week treatment is bei ng advocated by some. Neve1theless. long-tcm1 benefits of anti-/-1. pylori therapy include lowering of ulcer disease prevalence and prevent ion o f gastric carci no ma/lymphoma; but benefits in nonulcer dyspepsia are equivocal. Some available anti-II. pylori kits (one kit lo be taken daily in 2 doses) HP- KJT, HELIBACT. OMXIT I'\: Omcpnuole 20 mg 2 cap + Amox1c11lin 750 mg 2 tab ~ Tiniua,wlc 500 mg 2 tab. PYLO\'1OX: Lansoprazole 15 mg 2 cap + Amo,icillin 750 mg 2 tab + Tinidu,,olc 500 mg 2 tab.

DR UGS FO R PEPTIC ULCER AND GASTRO ESOP HAGEAL REFLUX DISEASE LA'\ISi KIT: Lanancreatm 100 mg, bile ext. 40 mg. dry stomach ext. 110 mg tab. UNICNZYME: Fungal diastase 20 mg, papain 30 mg, simeth1cone 50 mg. nicotmamide 25 mg. activated charcoal 75 mg tab. VITAZYME: fungal diastase 40 mg. c innamon otl 0.25 mg, caraway oil 0.5 mg, cardamom o il 0.5 mg p~r 10 ml liq. Enzyme preparat i~ns containing an antispasmodic or a laxative and fixed dose combination~ of pancrcatine or pancrelipasc containing amylase, protease and lipase with any other enLyme arc banned in Ind ia.

Methyl polysiloxane (Dimethyl poly iloxane, Simeth icone, Dimethicone) It is a silicone polymer, a visco us amphiphilic liquid which reduces surface tension and collapses froth, ' anti foaming agent'. It is not absorbed from g.i.t. and is pharmacologically inert. Added to antacid, digestant and antirefl ux preparations (see above). it is briskly promoted as a remedy for 'gas', a very common gastric complaint. It is also cla imed to coat and protect ulcer surface, to aid dispersion of antacids in gastric contents, and to prevent gastroesophageal reflux. However clinica l effi cacy is equi vocal. Dnse: 40- 120 mg 3 lo 4 ti mes a day. DIMOL 40 mg tab. (single ingredient).

2. Papain It is a proteolytic enqme obtained from raw papaya. Its efficacy afier oral ingestion is doubtful. 3. Pancreatin It is a mixture of pancreatic enqmes obtained from hog and pig pancreas. It contains amylase. trypsin and lipase. and is indicated in chronic pancreatitis or other exocrine pancreatic deficiency states. Fat and nitrogen content of , tools may be reduced and diarrhoea/steatorrhoca may be prevented. It has to be used as emeric coated tablets or capsules to protect the enqrnes from being themselves digested in the stomach by pepsin. Nausea, diarrhoea and hyperuricaemia are the occasional side effects. 4. Diastase and Takadiastase These are amylolytic enzymes obtained from the fungus Aspergillus 01yzae. They have been used in pancreatic ins ufficiency, but efficacy is equivocal.

GALLSTONE DISSOLVING DRUGS Laparoscopic cholecystectomy is the treatment of choice for gallstones, but medical therapy is an option in patients with s mall cholesterol (CH) stones who are unw illing or unfit for surgery. C H remains dissolved in bile with the help of bile salts (salts of cholic acid and chenodeoxycholic acid conjugated with g lycine and tauri ne) because bile sa lts are highly amphiphilic. A high C l I : b ile salt ratio favours crysta lli7ation o f C H in bile: these crystals act as nidi for stone formation. Chenodeoxycholic acid (Chcnodiol) and Ursodeoxycholic acid (Ur.;odio l) decrease C H content o f bile, enabling solubilization of C H from the stone surface.

719

720

GASTROINTESTINAL DRUGS Chenod iol is not used now, because it is less effective, frequently causes diarrhoea, can raise plasma LDL-CH and carries risk of liver damage.

Ursodiol It is the hydroxy epirner of chenodiol, wbich is more effective and needs to be g iven at lower doses. Administered orally, ursodiol is well absorbed, gets conjugated in liver with g lycine and taurine, and is excreted in bile. Thereafter, it largely undergoes enterohepatic c irculation and has a long plasma t½ of - 4 days. It promptly reduces C H secretion into bile and promotes solubilization of biliary CH by liquid crystal forma tion. Ursodiol also inhibits intestinal C H absorption. Plasma LDL- CH level is not raised . It can achieve complete dissolution of biliary C H stones in upto 50% cases when given for 6- 24 months. It is more liket ly to succeed in pat ients

rJr

who have a functioning gallbladder with multiple s mall floating gall stones. Diarrhoea and rise in hepatic transaminase level is infrequent. and ursodiol is much better tolerated than clenod iol. However_ gatric mucosa I resistance may be impaired predisposing to ulceration.

Dose: 450- 600 mg dai ly in 2- 3 d ivided doses after meals; UDIH EP 150 mg tab, URSOCOL 150, 300 mg tabs, 450 mg SR cap. Dissolut ion of gallstones is a very slow process: patient compliance is often poor. O nce treatment is discontinued after stone dissolution, recurrences are common, because bile returns to its CH supersaturated state. Repeat courses may have to be given.

PROBLEM DIRECTED STUDY

48.1 A 4-year-old girl is brought to the hospital emergency. The parents are very alarmed by her condition that has developed over the past one hour, when she started making bizarre faces . The neck has become rigid and head has tilted to one side. The teeth are clinched and she is not speaking. The eyes are staring in one direction and there are intermittent purposeless movements of the upper limbs. The parents inform that she had vomited twice in the morning and was taken to a local doctor, who had given her an injection. The vomiting had stopped, but after about 2 hours of the injection she developed the above symptoms. (a) What is the most likely cause of her symptoms? Could it be due to the injection given to her? If so, which drug could have caused it? (b) How should this patient be treated? (see Appendix-1 for solution)

Chapter

49

Drugs for Constipation and Diarrhoea

LAXATIVES (Aperients, Purgatives, Cathartics)

MECHANISM OF ACTION

These are drugs that promote evacuation of bowels. A di stin c tion is so m etimes made according to the intensity of action. (a) Laxative or aperient: milder actio n, elimination of soft but formed stools. (b) Purgative or cathartic: stronger act ion resulting in more fluid and forceful evacuati on. Many drugs in low doses act as laxati ve and in larger doses as purgative. The distinction, thus is vague.

All purgati ves increase the water content of the faeces by: (a) A hydrophi lic or osmotic action, retaining wate r and electrolytes in the intestinal lumen- increase volume of colonic content and make it easily prope lled. (b) Acting on intest inal mucosa, decrease net absorption of water and e lectrolyte; intestinal transi t is enhanced ind irectly by the fluid bulk.

LAXATIVES (Purgatives, Cathartics)

I

I

I

Bulk forming agents

I

Dietary fibre: bran Psyllium (plantago) lspaghula Methyl cellulose

I

I

Stool softener

Osmotic purgatives

Docusates (DOSS) Liquid paraffin

Mag. sulfate Mag. hydroxide Sod . sulfate Sod. phosphate Sod. pot. tartrate Lactulose Lactitol

I

IStimulant purgatives I I

I Diphenylmethanes Bisacodyl Sod. picosulfatc

I

I

Anthraqulnones (Emodlns)

Senna Cascara sagrada

I

I

I 5-HT4 agonlst

PG analogue

Prucalopride

Lubiprostone

722

GASTROINTESTINAL DR UGS (c) lncreasing propulsive acttv1ty as primary action- a llow ing less time for absorption of salt and water as a secondary effect. For some of the drugs, controversy continues as to whether they increase water content of stools as the primary action or it is a conseq ue nce of increased motility, because the amount of water absorbed la rgely depends on transit time. However, certain purgatives do increase motility through an action on the mycnteric p lex uses. Laxatives modify the fluid d ynamics o f the mucosa! cell and may cause fl uid accumulatio n in gut lu men by one or more of fo llowing mechanisms: (a) Inhibiting a K•ATPase of villous cellsimpairing electrolyte and water absorption. (b) Stim ulating adeny ly l cyclase in crypt cellsincreasing water and e lectrolyte secretion. (c) Enhancing PG synthesis in mucosa which increases secretion. (d) Increasing O synthesis wh ich enhances secretio n and inhib its non-propulsive contractions in colon. (e) Structural injury to the absorbing intestinal mucosa! cells.

BULK PURGATIVES Dietary fibre: bran Dietary fibre consists of unabsorbable cell wall and other constituen ts of vegetable food-ce llu lose, ligni ns, gums, pectins, g lycoprote ins and o ther po lysaccharides. Bran is the res idua l product of fl our industry which consists of ~40% d ietary fi bre. It absorbs water in the intestines, swe lls, increases water con tent of faeces- softens it a nd facilitates coloni c transit Osmotically active products may be formed in the colon by bacterial degradation of pectins, gums, etc. which act to retain water. Dietary fibre supports bacterial growth in colo n which contribute to the faecal mass. Certain dietary fib res (gums, lignins, pec tins) bind bile acids and promote their excre tion in faeces • degradation of cho lesterol in liver is enhanced • p lasma LDL-cholesterol may be somewhat lowered.

Increased intake of dietary fibres is the most appropriate method for prevention of funct io nal constipation, particularly if the diet is deficient in fibre. It is the firs t line approach fo r most patients of imple constipation. Dietary fibre is a lso useful when straining at stools has to be avoided.

Drawbacks: Bra n is ge nerally safe, bu t it is unpalatable , large quan tity (20--40 g/day) needs to be ingested. It has been included in some breakfast cereals, biscuits, etc. Full effect req u ires dai ly inta ke for at least 3-4 days. lt does not soften faeces already present in colon or rectum. As such , bran is u eful for prevention of constipation, but not for treating already constipated subjects. Flatulence may occur. Bran should not be used in pat ients with gut ulcerations, adhesions, stenosis and when faecal impaction is a possibility. Symptoms of i1Titable bowel syndrome (JBS) may be worsened by bran and other insoluble fibres.

Psyllium (Plantago) and lspaghula They con tain natural colloidal mucilage which forms a bulky gelatinous mass by absorbi ng water. It is largey fem,ented in colon: increases bacterial mass and so ftens the faeces. Refined ispaghu la husk 3- 8 g is freshly mixed with cold milk, fruit juice or water and taken o nce or twice daily. It acts in 1- 3 days. It should not be swall owed d ry (may cause esophageal impac tion). Prolonged intake of ispaghula and other soluble fibres reduces rectosigmoid intraluminal pressure and helps to rel ieve symptoms of I BS, including pain, constipation as well as d iarrhoea. Symptoms of chronic diverticulosis may a lso be re li eved. lspaghula husk (refined): ISOGEL (27 gt 30 g), NATURE CARE (49 g/100 g), FYBOGEL (3.5 g/5.4 g) po\\,dcr FIBRIL (3.4 g/11 g) powder; Psyllium hydrophilic rnucilloid: I OVAC (65 g/100 g) granules.

Methylcellulose A semi-synthetic, colloidal, hyd rophilic derivative of cellu lose that remains largely unfermented in colon. A dose of 4-6 g/day is satisfactory in most individuals.

DRUGS FOR CONSTIPATION AND DIARRHOEA Bulk forming agents can improve stool consistency in patients w ith colostomy. Generous amounts or water must be taken wi th all bulk fonning agents. The choice a mong different bulking agents is a maller or personal pre ferences.

STOOL SOFTENER Docusates (Dioctyl sodium sulfosuccinate: DOSS) It is an a nionic detergent, so ~ens the stools by net water accumulation in the lumen by an action on the intestinal mucosa. It emulsifies the colonic contents and increases penetration of water into the faeces. By a detergent action, it can dis rupt the mucosa! barrier and enhance absorption of many nonabsorbable drugs, e.g. liquid paraffin- s hould no t be combined with it. It is a mi ld laxative: especially indicated when straining at stools must be avoided. Dose: I00-400 mg/day: acts in 1- 3 days. CELLUBRIL 100 mg cap; LAXICON 100 mg tab, DOSLAX 150 mg cap. As enema 50 150 mg in 50--100 ml: LAXICON 125 mg in 50 ml enema.

Cramps and abdominal pain can occur. It is bitter; Iiquid preparations may cause nausea. I lepatotoxici ty is fea red on prolonged use. Liquid paraffin

It is a , iscous liquid: a mixture of petroleum hydrocarbons. that "as introduced as a laxative at the tum of 19th cc111ury. Million, of gallons have passed through the intes t inal pipeline si nce then . It is pharmacologically inert. Taken for 2-3 days. it sofiens stools, retards "ater absorption and is said to lubricate hard ;,cybali by coating them. Dose: 15- 30 ml/day oil as such or in emulsi fied form.

Disadvantages (a) It is bland but very unplcasa111 to S\\ allow becau,e o f o ily consistency. (b) Small amount pa,ses into the intestinal muco,a is camed mto the lymph > may produce foreign body granulomas in the intcstma l submucosa, mesenterie lymph nodes. liver and spleen. (c) While swallowing it may trickle into lungs-90

70-SO

(OD)

400

beginning. Being bacteri cida l, the advantages of c iproAoxacin are quick defervescence, earl y relief fro m sympto ms and prevention o f ca rrier state. It can also be used to treat typhoid ca rriers (750 mg BO for 4-8 wee ks). Th is has been found to achieve 92% eradicatio n rate . (Fo r alternative drngs see box) Drugs for typhoid fever 1. Ceftriaxone (see p. 778): Currently, it is the most reliable and fastest acting bac tericidal drug for enteric fever. Practically all S . typhi isolates, including multidrug resistant ones, are susceptible. However, it has to be injected i.v. (2 g twice daily for 2 days followed by 2 g/day till 2 days after fever subsides; children 75 mg/kg/day) and is expensive. Generally 7- 1O days treatment is required. Being bactericidal, it also prevents relapses and carrier state. Ceftriaxone is to be preferred over FQs in children, pregnant women and in areas with FQ resistance. Cefoperazone and cefotaxime are the other third generation cephalosporins used in typhoid. 2. Fluoroqulnolones: Ciprofloxacin (750 mg BO) is mostly used. Ofloxacin (400 mg BO), levofloxacin (500 mg 00/80) are nearly equally efficacious alternatives. 3. Azithromycin (500 mg OD for 7 days) is a second line alternative in multidrug resistant typhoid. and in patients to whom the 1st line drugs cannot be given. 4. Chloramphenicol (seep. 792): Since majority of S. typhi strains are now chloramphenicol resistant, it has become clinically unreliable. 5. Cotrimoxazole (see p. 759): It was effective in typhoid till plasmid mediated multidrug resistance spread among S. typhi. Now it is rarely used.

763

SULFONAMIDES, COTR IMOXAZOLE AND QUINO LONES

6. Bone, soft /issue, gynaecological and wound infections: caused by resistant S1aph. and gramnegative bacteria mostly respond to ciprofloxacin. Hi gh cure rates have been obtained in osteomyel itis a nd join t infecti o ns but prolonged treatment (6-8 weeks) w ith high doses (750 mg BD) is required. Used along w ith clindamycin/ metronidazole (to cover a naerobes) it is a good drug for diabetic foot.

7. Respiratory infec1ions: C iprofloxacin should not be used as the primary drug because pneumococci and streptococci have low and variable s usceptibiI ity. H ow eve r, it can treat atyp ical pneumo nia due to Mycoplasma, Legionella, H. influenzae, Branh. catarrhalis, and Chlamydiae. Severa l 2nd ge neration FQs ( levofloxacin, gemifloxacin, moxifloxacin) have now become avail able for the treatment o f pneumon ias and chronic bronchitis. Anthrax: Ciprofloxacin is considered to be the drug of choice. The US-F DA has approved its use for post exposure treatment of inhalatio nal anthrax which may occur due to bioterrorism. 8. Tuberculosis Mox ifloxacin and levofloxacin are second I ine drugs which can be used as a compone nt of combination chemotherapy against multidrug resis tant tuberculos is (see Ch. 56). Ciprofloxacin is less active against M. tuberculosis, but is the most active FQ against Mycobact. avium complex (MAC). 9. Gram-nega1ive septicaemias: Parenteral c iprofloxacin may be com bined wi th a third generation cephalosporin or an aminog lycoside. I 0. Meningitis: Though penetration in CS F is not very good, ciprofloxacin has been successfully used in gram-negative bacteri al mening itis, especially that occurring in immunocompromised patients or those wi th CS F shunts. It can cure meningococcal carrier state. 11. Prophylaxis: o f infections in ne utropenic/ cancer a nd other susceptible patients. 12. Conjunctivitis: by gram-negative bacteria: topical therapy is effecti ve.

Norfloxacin It is the least potent FQ: MI C values for most gram-negative bacteria are 2- 8

times highe r than that of ciprofloxaci n. Many

Pseudomonas and g ram-positi ve o rganisms are not inhib ited. Moreover, it atta ins lower concentration in tissues which are non-therapeutic. ortloxacin is primari ly used for urinary tract infectio ns. Given for 8- 12 weeks, it can treat chronic UTI. It is also good for bacterial diarrhoeas, because high concentrations are present in the gut, and anaerobic flora of the g ut is not d isturbed. NORBACTIN, NORFLOX 200,400, 800 mg tab. 3 mg/ml eye drops; UROFLOX. NORlLET 200,400 mg tab. BACIGYL 400 mg tab, I 00 mg/5 ml susp.

Pefloxacin

It is the me thy l derivative of norfloxacin which is more lipid soluble, completely absorbed orally, penen-ates tissues bette r and attains higher plasma concentrations. Passage into CSF is g reater than other FQs--ca n be used for meningeal infections. It is highly metabo lized- partly to norfloxaci n which contributes to its activity. Pefloxacin has longer t½ : cumu lates on repeated dosing achieving plasma concentrations twice as high as a fter a s ingle dose. Because of thi s it is effective in some systemic infections as well. Dose of pefloxacin needs to be reduced in li ver disease, but not in rena l insuffic iency. PELOX, 200, 400 mg tab, to be taken \\ ith meals; 400 mg/5 m I mj (to be diluted in I 00-250 ml of glucose solution but not saline, because it precipitates in presence of ions).

Cr

Ofloxacin

T hi s FQ is som ew hat less ac tive than c iprofloxacin against gra m-negati ve bacteria , but equall y or mo re po te nt again st Strep. pyogene.1· and other g ram-positive cocci and certai n a naerobes. Good activity against Chlamydia a nd Mycoplasma has been noted. It is an alternati ve drug for nonspecific urethritis, ce rvicit is and aty pical pne umonia caused by Chlamydia trachomatis. lt a lso inhib its M. tuberculosis; can be used in resistalll cases of TB. Hi gh activ ity is exh ibited against M. leprae, and it is bei ng used in alternative m ultidrug therapy regimens. Otloxaci n is relati ve ly li pid soluble; oral bioavail ab ility is high , and hi g he r pl asma concentra ti o ns are attained. Food does no t interfere with its absorptio n. It is excre ted

764

ANTIMICROBIAL DRUGS

largely unchanged in urine; dose needs to be reduced in rena l fa ilure. Ofloxaci n is comparable to c iprofl oxacin in the therapy o f systemic and mixed infections. It is suitable for chronic bronchiti s a nd othe r respiratory or ENT infections as we ll as for chlamydia uretlu·itis as an alternative drug. Inhibition of th eophy lline metabolism is less marked. ZA OCt , TA RIYID 100, 200, 400 mg tab; 200 mg/100 ml i.v. infusion, ZE FLOX also 50 mg/5 ml susp. ZA ' OCI , OFLOX, EXOCIN 0.3% eye drops.

Levofloxacin lt is the active levo(S) isomer of ofloxaci n having improved activi ty against Srrep. pnewnoniae, M. ruberculosis and some other gram-positive and gram-negative bacteria. Anaerobes are mode rately susceptible. Oral bioavailabi lity of levofl oxacin is nearly 100%; ora l a nd i.v. doses are si milar. It is main ly excreted unchanged, and a si ng le daily dose is sufficient because of slower e li mination and hi gher potency. T heo phy lline, warfarin, cyc losporin e and zidovudine pharmacokinetics has been found to rema in unchanged during levofl oxacin treatment. The primary indi cati on of levoftoxacin is communi ty acqui red pneumonia and exacerbations of c hronic bronchitis in which upto 90% cure rate has been obtained. 1-1 igh cure rates have been noted in sin usitis, pyelonephritis, prostatitis and other UT I, as well as skin/soft tissue infectio ns. Levofloxacin is an alternative drug for chlamydia! urrethritis. It is the second most active FQ for TB, and in India it is a component of the standardized regi men for MOR-TB. TAVANIC, GLEVO 500 mg tab, 500 mg/100 ml inj. LOX OF, GLEVO, U :.VOFLOX, LEVODAY 250,500, 750 mg tabs, 500 mg/ 100 ml inj; GLEVO 0.5% eye drops. Lomefloxacin IL is a second genernt,on ditluorinated quinolonc, more ac1ivc agai11st some gram-nega1ive bacteria and chlamydia. However, due to higher incidence of phototoxic ity and Q-T prolongation, it has been withdrnwn in USA and some other countries. but is available in India though infrequently used. ' LOMEF-400, LOMEDON, LOMADAY 400 mg tab. LOMIBACT, LOX 0.3% eye drops.

Sparfloxacin

Another second genc:ration dinuorinated quinolone which has enhanced activity against gram-positive bacteria, Bac/e,vides fi'agilis, other anaero~ and mycobacteria. IIS major indications include pneumonia, exacerbations

of chronic bronchitis. sinusitis and other ENT infections. llo\\ e, er. it has frequently caused phototoxic reactions: recipients should be cautioned not to go out in the sun. Prolongation of QTc interval has been noted in 3% recipients; fata l arrh)'1hmias have occurred . It has been discontinued in many countries including USA, but not yet in India. Dose: 200-400 mg OD oral. TOROSPAR 200, 400 mg tab; SPARTA, S PARQUl , S PARDAC I 00, 200 mg tab, ZOSPAR, SPARC, EYPAR 0.3% eye drops.

Moxifloxacin

A long-acting 2nd generation FQ ha ving high activity against St,: pneumoniae, other gram-positi ve bacteria including ~-lactam/ macro Iide resistant ones and some anaerobes. It is the most potent FQ agai nst M. tuberculosis, and is extensively used in MDR-TB . Bacterial topo isomerase IV is the major target of action. Moxifloxacin is primarily used for pneumonias, bronchitis, sinusitis, otitis media, in w hich efficacy is co mparable to ~- lactam antibiotics. Because urinary concentrations are low, it is not used for urinary tract infections. It is primaril y metabolized in li ver; should not be given to li ver d isease patie nts . No dose modification is needed in renal disease. Side effects a re similar to other FQs. It should not be given to patients predisposed to seizures and to those receiving proarrhythmic drugs, because it can pro long Q-1: interval, and the combination may precipitate serious arrhythmias. Phototoxicity occurs rarely. Dose: 400 mg OD; MOXlf, MOXIC IP, STAXOM 400 mg tab, 400 mg/250 ml 1.v. mfusion. MOXIC IP, MILfLOX, VIGAMOX 0.5% eye drops for conj unc11vi1is caused by gram-pos i11,·c a~ \\ell as negative bacteria.

Gemifloxacin A nother broad spectrum FQ, active mainly agai nst aerobic g ram positi ve bacteria, especial ly Srrep. pneumoniae, H. influenzae, Moraxella, Mycoplasma pneumoniae, Chlamydia pneumophila, Klebsiella including some mu ltid rug resistant strains. Some anaerobes are also inhibited. Gem ifloxacin is rap idly absorbed, undergoes limited metabolism, and is excreted in urine as well as faeces, both as unchanged drug and as metabolites. Dose needs to be halved if creatininc clearance is

inhibits both bacterial D 'A gyrase and topoisomerase IV confering gram - ive and gram +he activity as well as lo" potential 10 select resistam mutams. Most enterobaeteriace.ae are highly susceptible while good activity has been noted agamsl resistant Pseudomonas. MRSA. ampicillin resistant H. influenzae and K.pneumoniae. Afler i. v. infusion pazufioxac in penetrates most body fluids including CSF, and is mainly excreted unchanged in urine with a t½ of 1.5-2 hours. Clinical indications of pa.wnoxacin are pneumonia. lung abscesses. upper and lo"er UTI, peritonitis. intraabdominal absce!>Ses. endo metritis. secondary infections after bums. inj uries and empirical therapy of nosocomial infections. Adverse effects are nausea. diarrhoea, rashes, rarely se izures. However, evidence o f its efficacy and safety arc limited. It is not approved in USA and in UK, etc. Risk of sei7ures may limit dosage . Dose: 500 mg (elderly 300 mg) infused i.v. over 34 weeks. Seroton in syndrome (see p. 488) has occurred when linezolid was given to patients taking serotonergic drugs, like selecti ve serotonin reuptake inh ibitors (SSRls), probably because linezo lid is a weak MAO inhibitor. However, no cytochrome P450 enzyme related interactions have been re ported. Tedizolld It is a new oxazolidinone, approved by USFDA in 201 4 for the t rea tment of acute s kin and sk in structure infections caused by gram +ive bacteria, including MRSA. Tedizolid is active only against gram +ive bacteria and exerts strong inhibi tory e ffect (4-8 fold more potent than linczolid) on Staphylococci, Enterococci. Streptococci including M RSA VRE as well as some linezolid-resistant pheno types. Its mechan is m o r acti on (bind ing to 23S ribosome) is similar to that of linczolid. In clinical trials on pat ients with acute skin and soft t issue in fections, tcdizo lid 200 mg once daily was foun d to be equally effective as linezolid 600 mg twice daily, both taken for 6 days. It has also been successfully used 10 treat HAP and ventilator-associated pneumonia (VAP) caused by gram +ive bacteria. Administered as inactive phosphate salt, free and active tedizolid is released by phosphatases. Oral bioavi lability is high (- 90%), and it is excreted by liver as sulfate conjugate with a t½ of 12 hours. No dose adjustment is needed in hepatic or renal disease. In clinical trials, tedi.wlid was better tolerated than line,:olid, and appears to have lower potential for causing haematological disorders or neuropathy. I lowever, it is also a weak MAO inhibitor. but there is no data on drug interactions. Thus. the distinctive features of tedizolid arc---enhanced activity against gram +ivc bacteria, once daily dosing, oral as well as i.v. route of administration, and possibly favourable adverse e ffect profile. MISCELLANEOUS ANTIBIO T ICS

Spectinomycin It is a chemically distinct (aminocyclitol), narrow spectrum. bacteriostatic antibiotic which inhibits a limited number of gram-negative bacteria. notably Neisseria gonorrhoeae. Spectinomycin acts by binding to 30S ribosome and inhibiting bacterial protein synthesis. but

the action is dis tinct from that of aminoglycosides. The single approved indication of spectinomycin is treatment of drug resistant gonorrhoea, or when the first line drugs (13-lactams/macrolides. etc.) can not be used due to allergy or other contraindication. Dose: 2.0 g i.m. single dose; for less responsive cases 4.0 g (2.0 g at 2 sites). SPECTIN. TROBIC II', 2.0 g/vial inj. The single dose produces pain at injection site; chills, fever and urticaria. Repeated doses may cause anaemia, renal and hepatic impairment.

Qulnupristin/Dalfopristin This is a combination of two semisynthctic streptogramins in 30:70 rat io which together exerts rapid bactericidal effect on many gram + ive organis ms. Quinu pristin binds to the same site on bacterial SOS ribosome as erythromycin, while dalfopristin binds to an adjacent site to exert synergistic inhibition of protei n synthesis. It is cidal against most gram +ive cocci including. MRSA, penicillin resistant Strep. pneumoniae and static against £. faecium, but not active on £. Jaecalis. Adm inistered by i.v. infusion over 60 mi n in a dose of 7.5 mg/kg every 8- 12 hours, it is used as a reserve drug for multidrug resistant nosocomial gram +ive infect ions, especially those caused by MR ' A, VRSA, Strep. p11e11111011iae, VRE, but not £. Jaeca/is. Both components are rapidly metabolized in the liver with a t½ of 40 50 min, and excreted in faeces. Dose reduction is needed in liver disease. Muscle, joint, and injection site pain is the common ad, crse effect. They inh ibit CY P3A4 and can internet with drugs metaboli,:ed by this enzyme. SYNERCID: Quinupnst1n 150 mg + dalfopristin 350 mg (total 500 mg) per vial inj. Daptomycin It is a lipopcptide antibiotic which has a unique mechanism of action. The lipoidal tail ofdaptomycin molecules gets inserted in the bacterial cell membrane followed by aggregation of several molecules to form a pore through which K • and other ions leak out and the membrane gets depolarized. ynthesis or DNA, RNA and proteins is consequently inhibited, and the cell is killed. It is bacteric idal against only gram +he bacteria, including MRSA, VRSA, multidrug resistant streptococci, VRE and coryncbacteria. Daptomycin is indicated for the treatment or complicated skin and soil tissue infections caused by Stoph a11re11s including MRSA. VRSA and staphylococcal bactememia, bu1 not for pneumonia, because the pulmonary surfactant binds and inactivates it.

Daptomycin can be used for VRSA endocarditis, but efficacy against enterococcal cndocarditis is not established. Resistance can develop during daptomycin therapy. Administered i.v. daptomycin is mostly excreted unchanged in urine with a bio logical t½ of 7 11 hours. The t½ is prolonged in renal insufficiency. The specific toxicity daptomycin is myopathy and risk of rhabdomyo lysis, especially if given with statins. It has also caused eosinophilic pneumonia when used for >2 weeks. Side effects are abdominal pain. arthralgia, insomnia. pruritus, dys pnoca and injection site reaction.

MACROLIDE, LIN COSAMIDE, GLYCOPEPTIDE AND OTHER ANTIBACTERIAL ANTIB IOTICS Dose: Ski n and sotl tissue infection 4 mg/kg i.v. OD; for bacteraemia and ~ndocarditis 6 mg/kg OD i.v. DA PTOCURE 350 mg/via l for s lo" i., . injection after reconstitution.

Muplrocln This topically used antibiotic obtained from a species of Pse11domo11as is active mainly against grampositive bacteria, including Strep. pyoge11es ( penicillin sensitive/resistant). Staph. a11re11s, MRSA. etc. It inhibits bacterial protein synthesis by blocking the production of I- RNA for isoleucin. As such, no cross resista nce with any other antibiotic is seen. T hougb primarily bacteriostatic, high concentrations applied topically may be bactericidal. It is indicated in furunculosis, folliculitis , impetigo, infected insect bit~s and small wounds. Local itching. irritation and redness may occur. BA CTRO BA • M UPI , T- BACT 2°10 oint. for topical application thri~c da ily. Fusldlc acid It is a narrow spectnim steroidal antibiotic, blocks bacterial protein synthesis. It is active against penicillinase producing Staphylococci and few other gram-positive bacteria. It is used only topically for boils, follicul itis, sycosis barbae and other cutaneous infections. FUCIDIN-L, rl., CIBAC.T. fUSIDE RM; 2% oint. and cream. POLYPEPTIDE ANTIBIOTICS These arc low molecular weight cationic polypeptide antibiotics. All are powerful bactericidal agents, but not used systemically due to tox icity. All arc produced by bacteria. Clinically used ones are: Po lymyxin B Colistin Bacitracin

Polymyxln B and Collstln Polymyxin and colistin are basic polypeptides which were obtained in the late 1940s from Bacillus polymyxa and B. colistinus respectively. They arc active against gram-negative bacteria only; all except Proteus, Serratia and Neisseria are inhibited. Both have very similar range o r activity, but colistin is more potent on Pseudomonas, Salmonella and Sltigella.

Mechanism of action Po lymyxin B and colistin are rapidly acting bactericidal agents. They have a detergentlike action on the cell membrane. Due to high affinity for phospholipids, the peptide molecules (or their aggregates) orient between the phospholipid and protein fi lms in gram-negative bacterial cell membrane causing membrane distortion or pseudoporc formation. As a result ions, amino acids, etc. leak out. Sensitive bacteria take up more of the antibiotic. T hese antibiotics may also inactivate the bacterial endotoxin. They exhibit synergism with many other AMAs by improving their penetration into the bacterial cell. Resistance Resistance to these antibiotics has never been a proble m. There is no cross resistance with any other AMA.

Adverse effects Littlt: or no absorption occurs from oral route or even from denuded skin (bum, ulcers). Applied topically, they arc safe- no systemic effect or sensitization occurs. A rash is rare. Given o rally. side e ffects are limited to the g.i.t.--0ccasional nausea. vomning, diarrhoea. Preparation and dose Polymyxin B: ( I mg = I 0,000 U) IOSPORJ~ POWDCR: 5000 U with neomycin ,ulf. 3400 U and bac1tracin 400 U per g. 1\1.0SPORN 1- YC DROPS: 5000 U "11!1neomycin ~ulf. 170-.JA 0.5. 0.75, 1.0 g tabs, 0.3 g kid tab: PZA-CIBA 0.5. 0.75 g tabs. 250 mg 5 ml syr; RIZAP 0.75. 1.0 g tabs.

Ethambutol (E) Ethambutol, a synthetic straight-chain compound. is selecti vely tuberculostatic and is active against MAC as well as some other mycobacteria, but not other types of bacteria. Fast mu ltipl ying bacilli are more suscepti ble. Added to the triple drug regimen of RH Z it has been found to hasten the rate of sputum conversion and to prevent development of resistance, the latter being the primary purpose of us ing it.

819

820

ANTIMICROBIAL DRUGS

The mechan ism of action of E is not fully understood, but it has been fou nd to inhibit a rabi nosyl tran s ferases (e ncoded by embAB genes) involved in arabinoga lactan synthes is thereby interfering with myco lic ac id incorporation in mycobacterial cell wall. Res istance to E develops slowly and is most commonl y associated with mutation in embB gene, reducing the affinity of the target enzyme fo r E. o cross resista nce w ith any othe r antituberc ular drug has been noted. Abou t 3/4 of an ora l dose of E is absorbed. It is distributed widely, but penetrates men inges incosiste111ly; more when they are inflamed. Less than ½ of E is metabolized. II is excreted in urine by g lomeru lar fi lt ra ti on and tubular secretion, as well as in faeces; plasma t½ is --4 hrs. Caution is required in its use in patients with renal disease. Patient accepta bility of E is very good and side effects are few. Loss of visua l acuity/ colou r vision, fie ld defects due to retrobulbar neuritis is the most important dose and duration of the ra py dependent toxici ty. Patients shoul d be instru cted to stop the drug at the first indication of visual impa irment. Because young c hildren may be unable to repor1 early visual impairment, it was contraindicated, but is now a llowed with d ue precaution . With early recognition a nd stoppage of the drug, v isual toxic ity is largely reversible. Ethambutol produces few other symptoms: nausea. rashes. feve r, rarel y peripheral ne urit is, and is safe during pregnancy. Ethambutol is used in MAC infection as we ll. MYCOBUTOL, MYAMBUTOL, C0\'1BUTOL 0 .2, 0.4, 0 .6. 0.8, 1.0 g tabs.

Streptomycin (S) The pharmacology of streptomycin is descri bed in C h. 54. It was the first clinically use ful antitubercular drug, and is tuberculocidal, but less effective than I H or rifampin. Streptomycin acts on ly on extracell ular bacilli (because or poor penetration into cells). T hus. other drugs and host defence mechan isms are needed to eradicate the disease. It penetrates tubercular

cavities. but does not cross to the CS F, and has poor action in ac idic medium. Resistance developed rapidly when streptomyc in was used alone in tuberculosis- most patients had a relapse. Recent studies indicate worldwide increase in resistance to S. In case of S-resistant infection, it must be stopped at the earliest because of risk of 5-dependence, in wh ich case the infection fl ourishes when the drug is continued. Most nontubercul ar mycobacteria are unaffected by S. Because of need for i. m. injections and lower margin of safety (ototoxicity and nephroloxicity, especially in the elderly and in those with impaired rena l function) S is used o nly in addition to other Ist line a nti-TB drugs or as a reserve a lternative. Use is restricted to a maximum of 2 months. It is thus also labe lled as a 'supplemental' 1st line drug.

SECOND LINE ANTI-TB DRUGS These are less e ffecti ve and/or less well tolerated anti-TB drugs that a re used only in case the baci IIi are resistant to one or more I st line drugs or whe n these are not tolerated/ a re contra indicated. 1. Kanamycin (Km), Amikacin (Am) TI1ese aminoglycoside antibiotics (described in Ch. 54) are, ery sim ilar to streptomycin in antitubercular activity. pharrnacokinetic properties and types of adverse effects. Many S resistant and \-!DR strains of M.111berr:ttlosis remain sensitive lo them. One of these is mostly included in the regimen for MOR-TB during the intcnsi, e phase. The R TCP standardi7ed regimen for MDR-TB includes Km (probably because it is less expen• Sl\e than Am). but in many countries Am is preferred, because it i, considered less toxic. Cross res istance between Km and Ami, very common. Both Km and Arn produce less vestibular toxicity than hearing loss, but are equally ncphrotoxic. Paucnts should be instructed to report vcttigo and tinnitus. Audiometry a nd monitoring of renal function is recommended. Do,e: 0.75 1.0 g/duy (10- 15 mg/kg/day) i.111.

2. Capreomycin (Cm) It i, a cyclic pept ide antibio tic, chemically very dinerent from aminoglyco,ides, but with similar rnycobactericidal activity. 01010xicity and nephrotoxicity. In addition. Cm often causes cosinophilia. rashes, fever and injection site p45 kg. CYCLORI E, COXERIN, MYSER 250 mg cap.

7. Terlzldone It contains 2 molecules of cycloserine and has antibacterial properties as well as mechanism of action similar 10 it; but is believed to be less neurotoxic; reported incidence o f adverse effects is lower. II is used as a s ubstilllte of Cs. especially in genitourinary TO, because it attains higher and lo nger lasting concentration in urine. Dosage arc s imilar 10 Cs; 500-750 mg/day. TERICOX 250 mg cap.

8.

Para-amino salicylic acid (PAS)

Introduced in 1946, PAS is related 10 sulfonamides and acts probably by the same mechanism, i.e. inhibition offolate syn• lhase. It is not active against other bacteria, and this selectivity may be due to diffe rence in the affinity for folate synthase of M.tuherculosis compared to that of other bacteria. However, other mechanis ms of action arc also possible. PAS is tubcrculos1a1ic a nd one of the least active drugs: does 1101 add to the efficacy of more active dmgs 1ha1 are given with it; o nly delays development of resistance-probably by directly inhibiting episoma l res istance trans fer. Resistance lo PAS is s low 10 develop. It is used as the sodium salt (large doses that a rc needed may cause a• overload) or calcium salt (better gastric tolerance is claimed). PAS is absorbed completely by the oral ro ute and dis• tributed all over except in CSF. About 50% PAS is acctylated; competes with accty latioo of INH and prolongs its t½. It is e xcreted rapidly by glo meru la r fi ltration and 1ubu Jar secretion; 1½ is shon , - I hour. Patient acceptability of PAS is poor because o f frequent anorexia, nausea and epigastric pain. Other adverse effects arc rashes, fever, malaise, hypokalaemia, goiter, liver dysfunction and rarely blood dyscras ias. PAS is used only in resistant TB. The RNTCP includes it in the standardized regimen for MDR-TB only wh~n one of the tubcrculocidal drugs (Km, O fx. Z. Eto) or both the static drugs (E, Cs) cannot be used. Dose: I 0-12 g (200 mg/kg) per day in divided doses: SODll , M·PAS 0.5 g tab. 80 g/100 g granules.

9. Rifabutln It is related to rifampin in structure and mechanism of action, but is less active against M. tuberculosis, a nd more active against 4AC. Majority of M.tuherculosis isolates resistant 10 R a re c ross resistant to ri fabu1in . Thus, it is 1101 a n option for 1rcatmem of M DR-TB. The only place o f rifabutin in the treatment of TB is as a substitute for R to minimise drug interactions due 10 strong en,:yme induc ing properly o f R. Rifabu1in is a much weake r inducer of CY P e111ymes than R. This is especially needed in HIV coinfectcd patients of TB who receive a protease

inhibitor ( Pl) and/or a non-nucleoside reverse transcriptasc inhibitor (NNRTI) whose metabolism is markedly induced by R re ndering the m ineffective. The dose of rifabutin needs to be red uced when it is used to treat TB in a HIV patiem receiving a protease inhibitor. The primary indication o f rifabutin is for pro phylaxis and treatment o f MAC infection in HIV-AIDS patients. For prophylaxis of MAC, rifabutin alone 300 mg/day is an alternative to azithrornycin/clarithro mycin, while for treatment o f MAC infecti on, it is combined with 2- 3 other anti-MAC drugs. Gastrointestinal intolerance, rashes, granulocytopenia. myalgia and uveitis have been reponed with rifabutin. Reactions similar 10 those caused by R can also occur. Oral bioavailability of rifabutin is low (- 20%), but t½ is much longer (>30 hours). Dose: 300 mg (5 mg/kg) OD oral; RIBUTIN 150 mg tab.

Rifapentine It is a rifampin congener, very snnilar to it is mechanism of action as well as in activity against M. tuberculosis and MAC. Cross res istance between the two is complete. Toxicity as well as drug interaction profile is also simi lar. Rifapentine is as potent an e nzyme inducer as rifampin. Ho" ever. the t½ o f r ifapentine ( 13-15 hr) is longer than that of r ifampin, so Lhlll it can be given once or twice weekly. The only indication of rifapenline (600 mg once/twice weekly) is in the continuation phase o f treatment o f TB, as a substitute of daily rifampin. It is not suitable for use during the intens ive phase. Once weekly rifape111ine combined with INH has also been employed to treat latent TB.

Bedaquiline (BDQ)

Introduced recently, th is d ia• ry lquino line anti-TB drug acts by a novel mechanism . Bedaquiline (B DQ) inhibits mycobactcrial ATP synthase, thereby limiting energy production within mycobacterial cell. The human ATP synthase is 20,000 fold less sensi• tive to BDQ than is the mycobactcrial e nzym e. It has exhibited strong bactericidal and sterilizing activity against M. tuberculosis in in vitro tests and in a nimal models by kill ing both rapidly multiply ing as well as dormant bacil li. The M ICs o f BDQ are very low and similar for dmg• sensit ive as well as for MDR-TB and XDR-TB s trains. Many nontubercular mycobacteria and M. leprae are also inhibi ted in •·itro. M. tuberculosis develops resistance to BDQ prima rily by mutation of ATP synthase e nzym e, reducing its affinity for BDO. Another mechanism is by acquisition o f BDQ efflux from the bac illi. However, no c ross resistance between BDQ and any 151 line or 2 nd line an ti-TB drug occurs. Bed:iqu iline is we ll ab;,orbed o rall y; fall y meal improves a bsorption. It is highly plasma protein bound and eX!ensively distributed in tissues. Metabolism occurs in liver, mainly by C YP3A4, and the principa l dcsme1hyl metabolite is 20% as active as BDQ. Clinically s ignifi. cant dmg interactions occur with CYP3A4 ind ucers and inhibitors. The tem1mal 1½ of BDQ is very long (- 160 days). probably due to redistri bution from tissues. It is excreted mainly in faeces.

ANTITUBERCULAR DRUGS In clinical trials on pulmonary MDR-TB patients, addition of BDQ to WHO regimen for MDR-TB reduced the time for sputum to become c ulture negative. However, in 2 clinical trials, despite better microbiological outcome, more patients died in the BDQ group than in those receiving only background regimen. Since all deaths occurred after BDQ was stopped, the deaths did not appear to be related 10 BDQ use. The US-FDA approved BDQ in 201 2, and it has been marketed internationally as SI RTURO 100 mg tab. The WHO in 2013 have issued guidelines for use of BDQ in drug resistant TB, following which the RNTCP (2016) has introduced BDQ in India for MDR-TB at selected centers through its 'conditional access programme'. The present recommendations regarding use o r BDQ arc: • It should be used only for pulmonary MDR-TB in adults (> 18 yr) • Women patients should be nonpregnant and willing to remain so during BDQ use. • It should be used only in combination with at least 3 other anti-TB drugs 10 which the bacilli of the patient arc shown to be susceptible in vitro. or at least 4 other drugs to which the patie nt's isolate is likely to be sansitive. • It should be used only when an effective regimen cannot otherwise be provided. • BDQ should be given for a maximum of 24 weeks in a dose or 400 mg/day for 2 weeks followed by 200 mg 3 times a week for the next 22 \\ eaks. • Each BDQ tablet should be S\\alloed whole with meals. • The background anti-TB drugs should be coniinucd after stopping BDQ for the total 24 months treatment of MDR-TB. This is to ensure that any surviving bacilli arc not expsed to BDQ alone, which persists in the body for > 5 months after stopping. • BDQ is not 10 be used for drug-sensitive TB , or extrapulmonary TB or for nontubercular mycobactcria. Adverse effects of BDQ arc nausea. headache, arthralgia and prolongation of QTc interval. Caution is required in using BDQ in patients taking other QTc prolonging drugs. BDQ has the potential to cause hcpatotoxicity.

TREATMENT OF TUBERCULOSIS The therapy of tuberc ulosis has undergone remarkable changes. The 'Conventional ' 18- 24 month treatment was abandoned in the 1990s and more effective as we ll as less tox ic 6 month (short course) treatment was introduced, wh ich also yielded higher completion rates. This has been possible due to better understanding of the biology of tubercular infection. Biology of tubercular Infection M. 1r1berculosis is an aerobic organism. In unfavourable conditions it grows only

mtermiuently or remains dormant for prolonged periods. Several subpopulations of bacilli, each with a distinctive metabolic state, could exist in an infected patient. e.g.: (a) Rapidly growing with high bacillary load as in the

wall of a cavitary lesion where oxygen tension is high and pH is neutral. These bacilli are highly susceptible to 11 and to a lesser extent to R, E and S. (b) Slow growing located intracellularly (inside macrophages) and at inflamed sites where pH is low. They are particularly vulnerable to Z, while 11. R and E arc less active. and S is inactive. (c) Spurters found mostly within caseous material where oxygen tension is low but pH is neutral: the baci lli grow intcrmiuently with occasional spurts of active metabolism. R is most active on this subpopulation. (d) Dormant some baci lli remain totally inactive for prolonged periods. No antitubcrcular drug (except probably bedaquiline) is significantly active against them. However, there is continuous shifting of bacilli between these subpopulations.

The goals of antitubercular c hemotherapy are: (a) Kill dividing bacilli Drugs with early bacteric idal action rap idly reduce baci Ilary load in the patient and ach ieve quick sputum negativity. This makes the patient non-contagious and affords quick symptom relief. (b) Kill persisting bacilli To effect cure, sterilize the patient and prevent relapse.

(c) Prevent emergence of resistance So that the bacilli remain susceptible to the drugs . The relative activity of the fi rst line drugs in achieving these goals differs, e.g. H and R are the most poten t bacteric idal drugs active against a ll populations of TB bacilli, whi le Z acts best on intracellular bacilli and those at inflamed sites. It thus has very good sterili zing activi ty. On the other hand S is act ive only against rapidly multiplying extracellular bacilli. E is bacteriostatic-main ly serves to prevent resistance and may hasten sputum conversion. The general principles of antitubercu lar chemotherapy are: • Use of any single drug in tuberculosis results in the emergence of resistant organisms and relapse in almost 314th patie nts. A combination of two or more drugs must be used. The rationale is: the incidence of resistant bacilli to most drugs ranges from I o~8 to I 0-6. Because an average

823

824

A NTIMIC ROB IA L DRUGS

patient of pulmonary tuberculosis harbours I 0 8 to 10 10 bacilli , the number of organisms that will not respond to a single drug is high and cannot be dealt by the host defence. During protracted treatment, these baci ll i multiply and become dom inant in 3- 4 months. Because insensitivity to one drug is independent of that to another, i.e. inc idence o f H resistance among bacilli resistant to R wi ll be IO'"° and vice versa; only few bacilli will be resistant to both; these ca n be handled by host de fence. By the same rationality, mass ive infection (> 10 10 organisms) has to be treated by 3 or more drugs; and a si ngle drug is sufficient for prophylaxis, because the number of bacilli is small. • Jsoniazid and R are the most efficacious drugs; their combination is clearly synergistic, so that duration of therapy is shortened from > 12 months to 9 months. Addition of Z for the initial 2 months further reduces duration of treatment to 6 months.

• A s ingle dai ly dose of all first line a ntitubercul ar drugs is preferred. The 'directly observed treatment short course' (DOTS) was recommended by the WHO in 1995. • Response is fas t in the first few weeks as the fast div iding bacilli are eliminated rapidly. Symptomatic relief is evident within 2-4 weeks. The rate of bacteriological, radi ological and clinical improvement declines subseq uently as the s low mu ltipl y ing organism s respond gradually. Bacteriological cure takes much longer. The adequacy of any regimen is assessed by sputum smear/culture conversion rates and 2- 5 year relapse rates after completion of treatment.

SHORT COURSE CHEMOTHERAPY The WHO introduced 6- 8 month multidrug ' short course' regimens in 1995 und er the DOTS programme. An expert group framed clearcut treatment guidelines in 1997 for different categories of TB patients. The dose of all first line drugs was standardized on body weight basis. These guidelines were implemented by India and other WHO member countries, making major

progress in global TB control. The 'Stop TB strategy' of WHO was launched in 2006 and the spread of MDR-TB was taken into account. On the basis o f experience gained. new guideline with revised categori7..ation of patients was brought out in 20 10, in which patients of TB were classified as 'New case· or 'Previously treated' patients and ·Drug reslSlant TB' (DR-TB) including MDR-TB patients. From 2016 the WHO has enhanced its goal and launched ' End TB strategy·. The RNTCP have bro ughtout their latest guidelines in 2016. in which extensive use of drugsensitivity testing has been incorporated to effectively treat DR-TB. Both liquid culture and drug susceptibility tests' (LC-DST) and the faster genotypic tests, e.g. 'line probe assay' (LPA) for R and H resistance, and 'Cartridge based nucleic acid amplification test· (CB AAT) for R resistance are now widely utilized.

All anti -TB regimens have an initial intensive phase (IP) wi th 4- 6 drugs aimed to rapidly kill the bacilli, bring about sputum conversion and afford fast symptomatic relief. This is followed by a continuation phase (CP) with 3-4 drugs during which the remaining bacilli are eliminated, so that relapse does not occur. For devising drug regimens, the RNTCP (2016) has classified TB cases into: I. Drug-sensitive TB: The patient's bacilli are susceptible to all 5 first line anti-TB drugs (AT Ds). All new TB patients who have never taken any ATD or have taken them for less than one month are presumed to have drug-sensitive TB. 2. Mullidrug resistant TB (MDR-TB): The patient's bacilli are resistant to both R and H with or without resistance to any number of other I st line drugs. 3. Rifampin resistant TB (RR-TB): The patient's bacilli are resistant to R but not to H, with or without resistance to other ATDs. These patients are to be treated as if they have MDR-TB. 4. Mono-resistant TB: The bacilli are resistant to one ) st line ATD, but not R-resistant. 5. Poly-drug resistanl TB (PDR-TB): The patient's bacilli are res istant to more than one 1st line ATD, except both R and H resistance. 6. Extensive drug resistant TB (XDR-TB): A

MDR-TB case whose bacilli are additionally resistant to a FQ and one 2 nd line injectable ATD.

Drug-sensitive TB

This is treated by I st line ATDs as per regimens summarized in Table 56.1. For new patients, the initial treatment with

825

ANTITUBERCULAR DRUG S

Table 56.1: Treatment regimens· for new patients and previously treated patients of pulmonary TB presumed to be drug sensItIve Type of patient

Intensive phase

Continuation phase

Total duration

New

2£ HRZE

4HRE



Previously treated

2 HRZES + 1 HRZE

5 HRE

8

• Based on RNTCP guidelines 2016. £ Duration of the phase/total duration in months. H,R,Z,E,S-Standard codes for lsoniazid, Rifampin Pyrazinamide, Ethambutol, Streptomycin.

4 drugs including 3 bactericidal drugs (RHZ) and one bacteriostatic drug (E) rapidy brings dovm the bacillary load and reduces the risk of selecting resistant bacilli, especially in the face of increasing primary H resistance which is now upto 18%. After 2 months Z is discontinued and the remaining 3 (RH E) are continued for anothe r 4 months to eliminate all bacilli from the body. While in the 20 10 RNTCP regimen, the continuation phase had only 2 drugs (RH), the 2016 regimen includes E as well. This is in consonence with the suggestion of WHO to include E in the continuation phase for areas with high incidence of primary H resistance. In the new guidelines, the frequency of dosing during both IP and CP has been changed from 3 times a week to daily administration, which is considered optimal. The thrice weekly dosing was a compromise in the face of operational constraints of the programme, which have now been overcome. For previously 1rea1ed patients of TB who are presumed not to have DR-TB, all 5 I s1 line ATDs arc administered daily for 2 months. Injected S is then stopped and remaining 4 (RHZE) are continued for I more month. The IP thus lasts 3 months. The CP is longer, i.e. 5 months with R1 I and E administered daily. In case of severe forms of extrapulmonary TB, the CP in both new as well as previously treated patients may be extended by 3 6 months on clinical grounds. The WHO has standardized the dai ly dose of all ! s t line ATDs on body weight basis

Table 56.2: Daily dose of 1st line ant1tubercular drugs on body weight basis' Dady dose (mg/kg)

Drug

5 (4-6)

1. lsoniazid (H) 2. Rifampin (R)

10 (8-12)

3. Pyrazinamide (Z)

25 (20-30)

4. Ethambutol (E)

15 (15-20)

5. Streptomycin (S)*

15 (12-1 8)

s Based on WHO (2010) guidelines • In patients above 50 years age, the maximum dose of streptomycin is 0.75 g/day

as given in Table 56.2. However, it is advisable to use fixed dose combinations (FDCs) of oral ATDs, because this ensures that the patient takes all the drugs; risk of bacilli being exposed to only I or 2 ATDs is eliminated, and resistance is not fostered. For operational ease the R TCP provides separate FDC tablets of Ist Iine drugs for use during IP and CP. The number of such FDC tablets for different body weight bands is given in Table 56.3. Multidrug-resistant (MDR) TB

MOR-TB has a more rapid course with worse outcomes. Its treatment requires complex multiple 2 nd line drug regimens which are longer in duration, more expensive and more tox ic. ln India MOR-TB accounts for 3% of all new TB cases and 12- 17% of retreatment cases in di !Terent states. These figures are close to the Table 56.3: Dally dose of 1st line oral ant1tubercular drugs as number of fixed dose comb1nat1on (FDC) tablets (based on RNTCP guidelines 2016) Bodyweight category

Intensive phase

Continuation phase

HRZE'

HREs

25-39 kg

2

2

40-54 kg

3

3

55-69 kg

4

4

2! 70 kg

5

5

• Intensive phase: each tablet contains H (75 mg) + R (150 mg)+ Z (400 mg)+ E (275 mg) s Continuation phase: each tablet contains H (75 mg) + R (150 mg)+ E (275 mg).

826

A NTIMIC ROB IA L DRUGS

global average incidence. As per WHO global TB report 20 15, India has the highest numbe r of MDR-T B cases and an estima ted 7 1,000 cases of M DR-TB emerge annually from the notifi ed cases of pulmonary T B in Ind ia. The general principles of treatment of M DR-TB are : • The regimen should have at least 4 drugs certain to be effective. Often 6 drngs are included, since efficacy of some may be uncertain. • Reliance about efficacy may be placed on DST results, and the anti-T B drugs used previously in that individual. • Avoid combining cross resista nce drugs, e.g. two FQs, Km w ith Am or Eto w ith Pto, or Cs w ith teriz idone. • Include d rugs from group I to gro up IV in a hie rarc hia l o rde r. Group I drugs (Z, E) ca n be incl uded , add o ne injectab le drug (group 11 ), one FQ (g roup Ill ) and two group IV d rugs. The RNTC P initiated the DOTS-plus programme in the yea r 2000 to cove r t he di agnos is and treatmen t of MDR -TB . It u pdated its st rategy in 20 I O acco rd ing to the W H O reco mmendations, and has further revised its g uide lines in 201 6. The 'standard' RNTC P re g ime n fo r M DR -TB c o ns ists o f 6 drugs intensive phase lasting 6- 9 months and 4 drugs co ntinuation phase of 18 months (see box). This regi men is used in all confi rm ed o r sus pect M DR-TB cases, un less DST results o r other specifics {intolerance, e tc.) of an individual case necessitate use of an ' indi vidualized regi men', which is constructed taking into account these individual specific features. Standard RNTCP regimen for MOR-TB•

Intensive phase (6- 9 months) 1. Kanamycin (Km) 2. Levofloxacin (Lfx) 3. Ethionamide (Eto) 4. Cycloserine (Cs) 5. Pyrazinamide (Z) 6. Ethambutol (E) + Pyridoxine

Continuation phase (18 months) 1. Levofloxacin 2. Ethionamide 3. Cycloserine 4. Ethambutol

100 mg/day

"Revised National Tuberculosis Control Programme Guidelines (2016).

The min imal 6 mo nth intensive p hase is exte nded by I month each time till a maximum of 9 months, if the s putum culture put up at the end of 4 lh, 51h and 6 th month respectively are posit ive. Pyridox ine I 00 mg/day is given to all patients during the whole course of therapy to prevent ne urotoxicity o f the anti-TB drugs. Th is standard reg imen ha s been fou nd to be highly uccessfu l.

Rifampin resistant TB (RR-TB) Accord ing to both W HO and RNTCP, a case of RR-TB is treated as MDR-TB. However, because the bacilli are sens it ive to H, this 151 line drug is adde d to the MO R-regi men , both in the intensive phase as well as in the continuati on phase, w ithout changing the duration o r the other drugs. T hus, fo r RR-TB, the inte nsive phase of 6 months has 7 drugs ( Km, Lfx, Eto, Cs, Z, E a nd I-1) , and the continuation phase ( 18 mo nths) has 5 drugs ( Lfx, Eto, Cs, E a nd H). Monodrug resistant TB When the liq uid culture drug sensitivity test ( LC-DST) o r line pro be assay ( LPA) reports resista nce to o ne first line drug, o the r than R resistance, the treatment regimen acco rding to RNTC P-201 6 consists of: • R + two of the I 51 line drugs to w hic h bacilli are sensitive + one injectable second line drug + o ne FQ to make a to tal of 5 drugs given dai ly in the intensive phase of 3- 6 months. Polydrug resistant TB W hen the bac ill i are res istant to mo re than o ne fi rst line d rugs, other than R resistance, the in itia l reg imen is : R + one injec table 2 nd line d rug + o ne FQ + any I st li ne d rug to w hic h the bacil li are sensitive - one of the oral 2nd line drugs (Eto/Cs/ PAS). T he durat ion of intensive phase is 3- 6 months. In the conti nuation phase for both mono and p oly d rug re sistant TB , the inj ectable d rug is stopped while the remaining 4 oral drugs are continued fo r a fixed d uratio n o f 6 mo nths . Thus, the tota l duration of regimens for mo no and po ly drug resistant TB is 9- 12 mo nths.

ANTITUBERCULAR DR UGS

/soniazid resistant TB lt has been observed th at w hen H resisLance is of low level ( in LC-DST) or is due to inhA m utation (in LPA), hi gh dose of I H (900 mg/day for 46-70 kg body weight) in place of 300 mg/ day, is effective and should be added to the regi men (along w ith pyridox ine). However, in case of high level H resista nce or that due to Kat C mutation, it is ineffective and should not be given. Extensive drug-resistant TB As a lready defined, these are M DR-TB cases that are resistant to at least 4 most effective cidal drugs, viz . H,R, FQ and one of Km/Am/Cm. The WIIO estimated that 9.7% ofM DR-TB patients had XDR-TB in 2015. The exact incidence ofXDR-TB in India is not known. but with expanding laboratory faci lities to conduct se nsitivity tests for 2nd line drugs, more XDR-TB cases arc likely 10 be confirmed.

The XDR-TB is very di ffic ult to treat, has a rapid course and high mortality. However, to prevent further amplification of resistance, the standard MDR regimen must be immedi ately stopped when XDR-TB is detected or suspected. An expert pane l may decide on inslituti ng a regimen including the grou p V drugs which are d ifficult to to lerate and expensive. The RNTCP (201 6) has recommended a treatment regimen for XDR-TB consisting of 7 drugs in the intensive phase (6-12 mon ths) and 6 drugs in the continuation phase ( 18 months). T he drugs and their adult daily doses (for 46-70 kg body weight) are: Capreomycin I000 mg 400 mg Moxifloxac in High dose isoniazid 900 mg PAS 12 g Clofazimi ne 200 mg 600 mg Linezolid Amoxicillin/clavulanate (875+ 125 mg tab) 2 tab morning + one ta b evening. In the continuation phase, injection capreomycin is stopped and the remaining 6 drugs are continued for another 18 months.

Tuberculosis in pregnant women The WHO and British Thoracic Society consider H, R, E and Z to be safe to the foetus and

recommend the standard 6 month (2 HRZE + 4HRE) regimen for pregnant women with TB. S is contraindicated because it is ototoxic to the foeLus. In india, the current (20 I 6) R TCP guideli nes also consider the 4 oral Is t line drugs to be safe, and recommend fu ll course of TB treatment as in nonpregnant. Treatment of TB should not be withheld or delayed because of pregnancy. All pregnant women being treated with IN H should receive pyridox ine 10- 25 mg/day.

Treatment of breastfeeding women All I st line ATDs are compatible with breastfeeding; full cou rse should be given to the mother, and breastfeeding should be conti nued. The infant should receive 6 month iso niazid preventive treatment after rul ing out active TB followed by BCG vaccination. Breast-fed infan ts whose mothers are taki ng I H, and th ose on IN H preventive therapy shou ld be supplemented with pyridoxine 5 mg/day. Management of patients with adverse drug reactions to antitubercular drugs The ! st li ne ATDs are ge nerall y well to lerated, bu t minor side effects do occur, and should be managed symptomatically without alteri ng medi cation; e.g. nausea, anorexia- give the drugs with small meals; drowsiness-give drugs before bed time; Z induced arthralgia can be treated by analgesic- SA!Ds; peripheral neuritis due to H can be mitigated by pyridoxine. If more severe reacti ons li ke skin rashes, itching develop, all drugs should be stopped promptly. A fter reso lution of the reaction, the drugs are to be rei ntroduced one at a Lime by challenging wi th small doses and increasing every 3 days. When the offendng drug is identified, it should be stopped and the regimen reconstitu ted. However, R should never be reintroduced in case of severe reaction such as hacmo lysis, thrombocytopenia or renal fa ilure. Ethambutol should be discontinued at the fi rs t sign of optic neuritis. Hepatotoxicity is the most common problem with anlitubercular drugs. Any one or more of H, R and Z could be causative and the reaction

827

828

ANTIMICROBIAL DRUGS

occurs more frequently when, as per standa rd protocol, combination of these drugs is used. In case hepatitis develops, all drugs should be stopped and the reaction a llowed to subside. If TB is severe, nonhepatotoxic drugs S + E + one FQ shou ld be started wh ile th e reaction clears. Subsequemly, the discontinued drugs are restarted one at a time. Generally, R is resumed first followed 7 days later by H. If hepatitis recurs, the last added drug is stopped pe,manently and the regimen is reconstructed. In case both R a nd H are tolerated- do not restart Z but prolong therapy with R, 1-1 and E to 9 months. If R is the culprit, HES may be given fo r 2 month followed by HE for IO months. If H is implicated, REZ may be given for 9 months.

Latent tubercular infection (Chemoprophylaxis) The purpose is to prevent progression of latent tubercular infection to active disease. This is indicated only in subjects at high risk of developing active TB, viz.: (a) Conta cts o f open cases who show recent Mantoux conversion. (b) Chi ldren with a sputum positive TB patien t in the family. (c) Neonate of tubercu lar mother. (d) Patients of leukaemia, diabetes, s ilicosis or those receiving imm unosuppressant medication or tho e on long-term corticosteroid therapy. (e) HI V infected contacts of sputum positive index cases. The standard drug fo r chemoprophy lax is of TB is IN H 300 mg (10 mg/kg in ch ildren) d aily for 6 mo nths. This is as effec ti ve in HIV patients as in those with normal immune function. The W HO ha ve recently (2014) reviewed the evidence on treatment of latent tubercular infection, and have concluded that 6 month [NH preventive treatment is optimal. Ex tending it by another 3 mon th s does not afford additional protection. Another regimen of once a week rifapenti ne (10 mg/kg) + h igh dose INH ( 15 mg/kg) given for 3 months is equally effective. Due to lack of qua lity evidence about the most appropriate drug(s) or duration of

prophy lax is that should be used in case of subjects exposed to MOR-TB, the W HO expert panel (20 14) recommend that MOR-TB contacts shou ld be watc hed c losely wi thout giv ing any prophylactic medication, and treated promptly if they develop active disease. evcrtheless. TB experts may institute individually tai lored preventive reg imens based on the drug susceptibility profile of the M OR-TB case from whom infections is like ly to have been contacted (index case).

Role of corticosteroids Corticosteroids should not be ordinari ly used in tubercular patie nts. However, they may be used u nd er adequate chemotherapeutic cover: • In seriously ill patients (m iliary TB or seve re pulmonary TB) to buy time for drugs to act. • When hypersensitiv ity reacti o ns occ ur to antitubercular drugs. • ln mcningeal/renal/pericardial TB or ple ural effusion-to reduce exudation, prevent its organ isati on and strictures, e tc. • In AIDS patients with severe manifestations of tuberculosis. Corticosteroids are contraindicated in intestinal tuberculosis because silent perforation can occur. Corticosteroids, if given, should be gradually withdrawn when the general condition of the patient improves. Tuberculosis in AIDS patients The association of HIV and TB infection is a serious problem. Risk of developing TB in HIV positive subjects increases by about 8 times. HJ V positive cases have a higher incidence of extrapulmonary, more severe, more lethal and more infectious TB. HIV infection is the stro ngest risk factor for unm asking latent TB. Moreover, adverse reactions to anti-TB drugs are more common in HIV patients. It is estimated that 2. 1 milli on Indians are cu rrently living with HIV, and that 1. 1 lac HIV associated TB cases occurred in 2014 and 31,000 died countrywide. About 1/4 of death s am ong A IDS patie nts in India is due to TB. India thus ranks 2 nd in the global burden of HIV-associated TB. On the other hand, instin,tion of 'antiretroviral therapy' (A RT) and improvement in CD4 cell

829

ANTI TUBERCULAR DRUGS count of the subject markedly reduces the inc idence of TB among H IV-AIDS patie nts. Whe n CD4 count is < 150 cell s/µL , extrapulmona ry and dua l TB is more commonly encou ntered. Risk of recurrent TB among cured TB s ubjects is a lso high in HI V patients. In case of M. tuberculosis infection, drug used are th e same as in non-HIV c ases, and at least 4 drugs are used. Initia l inte nsive phase th erapy w ith daily H RZE for 2 months is sta rte d im mediately on the di agnosis o f TB, and is fo llo wed by a continuatio n phase of H RE fo r 4- 7 mo nths (to ta l 6- 9 months). Th rice week ly regimen s ho uld not be used, because it is assoc ia ted with 2- 3 t imes highe r rate o f relapse a nd fa ilu re am o ng H IV pos iti ve pa tie nts . R isk o f acq uiri ng res is ta nce to R is in creased co mpa re d to dail y tre atme nt. Pyridoxine 25- 50 1ng/day is ro uti ne ly gi ven along w ith H to co unteract its ne urolo gica l side effects, wh ich are more like ly in AIDS patie nts . All HI V positi ve T B patients should also recei ve cotrimoxazole preve ntive therapy at least throug hout the anti-TB regimen. T his ha s been fou nd to red uce mo rta lity, probab ly by preventing Pneumocystis j irovecii a nd o the r infections. C ons ide rati o n a lso h as to be g iven t o possi ble drug interactions between an ti-T B and antiretroviral (ARV) drugs. Ri fa mpi n, a potent inducer o f CYP isoenzymes, markedly enhances the metabolism o f protease inhibitors ( Pis, vi::.. indinavir, nelfinavir, ritonavir) and o f RTl s, viz. ne virapine, making the m ineffect ive. In patients recei vin g these drugs, rifabutin (a less pote nt e nzyme induce r) g ive n for 9- 12 months may be substi tuted fo r rifam pi n. T he metabolism of nucleoside reverse transc ripta se in hibitors ( N RT ls, z ido vudine , etc .) is no t ind uced by ri fa mpin, a nd no dose a dj us tment is needed. A n a lternative regimen of 3 RT!s (zidovudine + lamivudine -"- abacavir) has been ad vocated fo r pat ie nts w ho are to be trea ted by rifampin. If 2 RT I + RT I is to be used, e fa virenz sho u ld be selected as the RT I because its metabolism is induced to a lesser exte nt.

MDR-T B in HIV-A IDS patie nts sho uld be treated in the sa me way as tha t in no n-H lV in fected patient for a to tal o f 24 months.

Mycobacterium avium complex (MAC} infection MAC is an oppo rtuni tic pathogen which causes disseminated and multi focal disease in immunocompromized (HI V-AI DS) patients. The disease develops when cell mediated immunity is markedly depressed, i.e. when C D4 count drops to 80% cases. It is particularly favoured for vaginitis because of a long lasting residual effect after once dail y appl ication. A 7 day course is generally used. For oropharyngeal candidiasis 10 mg troc he of clotri mazole is allowed to dissolve in the mouth 3-4 times a day, or the lotion/gel is app lied/swirled in the mouth for as long as possible. It is also effective in skin infections caused by CmJ·nebacteria, but like most topical anti fungals, has poor efficacy in tinea capitis (scalp) and tinea unguium (nails). Clotrimazole is wel l tolerated by mos t patients . Local irritation with stinging a nd burning sensation occurs in some. No systemic toxicity is seen after topical use. SL Rf AL, CLOD[RM l°'o lotion.cream, powder; !% ear drops ,1 ith 2% lidoca111c, 100 mg ,agmal tab. CANDI D 1% cream, mouth paint. pO\lder. I•·• car drops with 2% lidocaine.

Econazole It is s imi la r to c lotri mazo le; penetrates supe rficia l layers of the skin and is highl y effective in dermatophytosis, otomycosis, oral thrush, but is somewhat inferior to clotrimazole in vag initis. No adverse effects, except local irritation in few is reported. [CO,AZOLE 1% o int, 150 mg ,agmal tab; ECODER\,I 1°10 cream.

Miconazole It is a highly efficacious (>90% cure rate) drug for tinea, pi tyriasis versicolor, otomycosis, cutaneous and vulvovaginal candidias is. Becau e o f its good penetrating power,

843

844

ANTIMICRO BI A L DRUGS

single application on skin acts fo r a few days. Irritation after cutaneous application is infrequent, but a higher inc ide nce of vaginal i1Titation is reported in comparison to clotrimaz ole. DAKTARIN 2% gel, 2% powder and solution; GYNODAKTARI 2% vaginal gel; ZOLE 2% oint. lotion, dusting powder and spray, I% ear drops, I 00 mg vaginal ovules.

Oxiconazole A newer topica l im i.dazo le antifunga l effective in tinea and other dermatophytic infecti o n, as we ll as in vaginal cand idias is. Local irritation can occur in some patients. OXIZON, ZODERM : oxiconazole 1% with benzoic acid 0.25% cream/lotion; apply topically once or twice daily.

Ketoconazole (KTZ) It is the first orally effective broad-s pectrum antifungal drug, useful in both dennatophytosis a nd deep mycosis, but has been overshadowed by the newer triazoles. The oral absorptio n o f KTZ is faci litated by gastric acid ity. Hepatic metabolism is extensive; metabolites are excreted in urine and faeces with a variable t½ of 4- 8 hou rs . The oral dose is 200 mg OD, but systemic use of KTZ is infrequent now. FUNGICIDE, NIZRAL, KETOVATE 200 mg tab. FU"JGJNOC 2% oint, 2% shampoo (fordandrufl), KETOVATE 2% cream. NIZRAL 2% cream, 2% lotion; DA'JRUF 2% shampoo, HYPHORAL 2% lotion.

Adverse effects

Ketoconazo le produces more side effects than itr~conazole or fl uconazole.

The most common side effects are nausea and vo miting; others are- loss of appetite, headache, paresthesia, rashes and ha ir loss. The major drawback of KTZ is its hormonal effects. It decreases androgen productio n from testes, and displaces testosterone from protein bi nding sites. Gynaecomastia, loss of hair and libido, and oligozoospermia may deve lop when the drug is used for a few weeks. Menstrual irregul a rities occur in some women due to suppression of estradiol synthesis. A dose-dependent decrease in serum hydrocorti sone due to synthesis inhi bition has also been noted.

Interactions

Ketoconazole (and most azoles) interact with several drugs. Due conside ration

mu st be give n ·w hen azoles are coprescribed wi th other drugs. H 2 blockers, proton pump inhibitors and antacids decrease oral absorption of KTZ by reducing gastric acidi ty. Rifampin , phenobarbitone, carba maze pine and phenytoin induce KTZ metabolism and reduce its efficacy. Ketoconazo le in hi b its CYP450 enzymes, especially C YP3A4 , CY P2C9; CYP 2Cl9 and raises the blood leve ls of several drugs including : Digoxi n Phenytoin Carbamazepine Omeprazole Diazepam Cyclos pori ne Haloperido l ifedip ine and other DHPs Warfarin HIV protease inhibitors Sulfonylureas Statins

Use Orally adm inistere d KTZ is effective in dermatophy tosis because it is concentrated in the stratum corneum, but use is restricted due to potenti a l adverse effects . Used as a lotion or shampoo, KTZ is quite effective in seborrhoea of scalp a nd dandruff. Pity riasis versicolor in vo lvi ng limited area of skin can also be treated by to pical application of the lotion/shampoo fo rmulation . Weekly treatment may be continued to prevent recurrences. Though effecti ve in moniliol vaginitis, oral t he rapy (for 5- 7 days) with KTZ is reserved for recurrent cases or those not responding to topical agents. Oral KTZ is no longer used in systemic rnycosis. High-dose KTZ has been used in Cushing's syndrome to decrease corticosteroid production.

Fluconazole

It is a water-so luble triazole having a wider range of act ivity than KTZ; indications include c ryptococca l meni ngitis, systemic and mucosa! candid iasis in both normal a nd imm unocompromised patients, coccidioidal meningitis and some tinea infections. Fluconazole is 94% absorbed; o ra l bioavailabi Iity is not affected by food o r gastric pH. It is primarily excreted unchanged in urine with a t ½ of 25-30 hr. Fungicidal co ncentrations are achieved in nails, vagina and saliva. It is the preferred azole fo r fun gal meningitis because

845

ANTI FUNGAL DRUGS

Table 58.2: Properties of drugs used for systemic mycoses Characteristic AMB Caspo 5-FC

FLU

/TR

VORI

Antifungal spectrum

Broad

Narrow

Narrow

Broad

Broad

Broad

Oral absorption

Nil

Nil

High

High

Variable

High

Administered i.v.

Yes

Yes

No

Yes

Yes

Yes

Elimination route

H, R

H,R

A

A

H

H

Half life

15 days

10 hr

3-4 hrs

24-30 hrs

24-40 hrs

6 hrs

Resistance (in vivo)

No

No

Yes

Limited

Minor

No

Nephrotoxicity

Yes

No

No

No

No

No

Anaemia

Moderate

No

Mild

No

No

No

Gastrointestinal upset

Brief

Mild

Moderate

Mild

Moderate

Mild

Overall toxicity

High

Low

Medium

Low

Low

Low

AMB-AmphotericinB; 5-FC- Flucytosine; FLU- Fluconazole; ITR- ltraconazole; Caspo- Caspofungin; VORI- Voriconazole; H- Hepatic; A- Renal.

of good penetration into brain and CSF. Dose reductio n is needed in renal impaim,ent.

Adverse effects The gastrointes tinal tolerab ility of fluconazole is better than o ther azoles, but nausea, vomiting, abdom inal pain, rash and headache are produced. Incidence and severity of these side effects increase s wi th dose and duration of therapy. Selecti vity fo r fungal cytoc hrome P450 is higher. Unlike KTZ , it does not inhibit steroid synthes is in man: antiandrogeni c and other endocrine side effects have not occurred. Elevati on of hepatic transa minase has been noted in AIDS patients. Fluco nazole is no t reco mmended in pregnant and lactating women.

Interactions While fluconazole afTects hepatic drug metabolism to a lesser extent than KTZ. increased plasma levels of phenyto in, cyclosporine, warfarin, zidovudine and sulfonylureas have been observed. The drng interaction potentia l of fluconazole is the lowest among azole anti fungals, but caution needs to be applied in coadministering other drugs. Proton pump inhibitors and H 2 blockers do not affect its absorption. Use Fluconazole can be administered orally as we ll as i.v. ( in severe infections). A sing le 150 mg o ral dose can cure vagina l candidiasis with few re lapses.

Oral fluconazole ( 100 mg/day for 2 weeks) is highly effective in oropharyngeal candidiasis, but is reserved for cases not responding to topical anti fu nga ls. Fluconazole ( I 00 mg/day) for 2- 3 weeks is the first line treatment for candida esophagi tis. Most t inea in fec tions including pityriasis vers icolor involv in g large area of sk in a nd cutaneous candidiasis can be treated with 150 mg weekly fluconazole for 4 weeks. For disseminated candid ias is, cryptococcal or coccidioidal me ning itis and other systemic fu nga l infections the do se is 200-400 mg/ day for 4- 12 weeks or longer. In candidemia, therapy may be initiated with i.v. flucona zole and continued o rally. Long-term oral fluconazo le ma intenance therapy after initial treatment with i.v. AMB is used in AIDS patients w ith funga l meningitis to prevent re lapse. An eye drop is useful in fungal keratitis. Fluconazole is ineffecti ve in aspergillosis and mucormycosis, and inferior to itraconazole for histo plasmosis, blasto mycos is and sporotrichosis, as we ll as for tinea ung uium. SYSCAN, ZOCON. fORCAN, fLUZO'J 50, 100,150,200 mg caps, 200 mg/100 ml i.v. infu,ion. SYS(" AN 0.3% eye drops.

ltraconazole

This o rally active triazole antifungal has a broader spectrum of activity than KTZ or fluconazole; includes few moulds like

846

ANTIM ICROBIAL DRUGS Aspergiflus a well. Some fluconazole resistam Candida are susceptible. It is fungistatic, but effective in immunocompromised patients. Steroid hormone synthesis inhibition is absent in itraconazole, and serious hepatotoxicity is rare. Oral absorption of itraconazole is variable. It is enhanced by food and gastric acid. ltraconazole is highly protein bound, has a large volume of distribution ( IO L/Kg), accumulates in vaginal mucosa, skin and nails, but penetration into CSF is poor. It is largely metabolized in liver by CYP3A4; an active metabolite is produced which is excreted in faeces; t½ varies from 24-40 hours. ltraconazole is well tolerated in doses below 200 mg/day, but doses > 400 mg/day are poorly tolerated. Dizziness, pruritus, headache and hypokalaemia are the other common side effects. Unsteadiness and impotence are infrequent. Plasma transaminase may rise and liver damage is reported. However. antiandrogenic and other hormonal adverse effects are not seen. Impaired left ventricular function has been worsened in some patients after prolonged treatment with itraconazole.

Drug interactions Oral absorption of itraconazole is reduced by antacids, H2 blockers and proton pump inhibitors. Rifampin, phenobarbitonc, phcnytoin and carbamazepine induce itraconazole metabolism and reduce its efficacy. On the other hand, clarithromycin and HI V protease inhibitors reduce the metabolism of itraconazole and raise its blood levels. ltraconazole inhibits CYP3A4, but the effect on metabolism of other drugs is less marked than that of KTZ. evertheless, phenytoi n. digoxin, sulfon ylureas, statins, dihydropyridines, protease inhibitors, warfarin and cyclosporine levels are increased by itraconazole. Uses ltraconazole is th e preferred azole antifungal for most systemic mycosis (see Table 58. 1) that arc not associated with meningitis. It is superior to fluconazole for histoplasmosis. blastomycosis, sporotrichosis and is the drug of choice for the rare fungal infections-paracoccidioidomycos is and chromomycos is. It also affords some relief in chronic aspergillosis, but

voriconazole is superior. A dose of 200 mg OD/ 8D with meals is used for 3 months or more. Vaginal candidiasis: 200 mg OD oral for 3 days is as e ffective as intravaginal clotrimazole. Dermatophytosis: I00-200 mg OD for 7- 15 days. ltraconazole is more effective than griseofulvin, but systemic therapy is resorted to only when topica l antifungals fai l or in patients with extensive involvement or scalp infection. Onychomycosis: 200 mg/day for 3 months. An intem1ittcnt pulse regimen of 200 mg 8D for I week each month for 3 months is equally effective. Relapses have occurred after itraconazole therapy, though it remains in the nail for few months after completion of the course. Pityriasis versicolor: Oral itraconazole is the drug of choice when systemic therapy is needed for involvement of extensive area of skin. SPORANOX, CANDITRAL. CANDISTAT. ITASPOR. FLUCOVER 100 mgcap. lTASPOR 100 mg cap. 200 mg/201111 vial.

Voriconazole It is a secon d ge ne rat io n b roa d-s pectru m t ri azo le introdu ced for diffic ult to trea t fungal infec ti ons. Vo riconazole is the drug of c hoice for invasive aspergillosis, di semi nated infections caused by tl uconazole resista nt Candida, Fusari11111 infections, and feb ri le neutropenia not responding to anti bacterial therapy. It is also ac tive against histoplasmosis and blastomycosis. Serious cases arc first treated i. v. followed by oral voriconazo le. It is complete ly absorbed orally, except whe n taken with a fa tty mea l, w idely d istributed into tiss ues and metabolized extensively by CYP2C 19. CYP3A4. CYP2C9. Metabolites are excreted in urine. The t½ is 6 ho urs. Voriconazole is also an inhibitor of CYP isoenzymes and the drug interaction profi le is similar to itraconazole. Rashes, photosensitivity, rise in li ver enzymes and QTc prolongation can occur. Short lasting visual disturbances are common after i.v. injection, and an acute reaction is al o possible. Dose: 200 mg oral BD taken I hour before or I hour a Iler meal. Begin i.v. infusion with 6 mg/kg 12 hourly infused over 2 hours twice followed by 3-4 mg/kg 12 hourly.

ANTI FUNGAL DRUGS VFEND 50, 200 mg tab,. 40 mg/ml oral suspension: 200 mg/ vial 1nj .• FU 'GIVOR :!00 mg tab.

Posaconazole This recently introduced broad-spectrum triazole has potent anti fungal activity and is the only azolc which has shown efficacy in mucormycosis. Posaconazole is indicated for salvage therapy of this difficult to treat fungal infection. Because of its high cost and limited expe rie nce, it is reserved for nonrespons ive cases of aspergillosis and invasive candidiasis. Favourable results have been reported in febrile neutropenia and as a prophylactic in immunosuppressed patients. It has also been used as alternative to itraconazole for chromomycosis. Side effects ofposaconazole are common, but mostly limited to nausea, abdominal pain, loose motions, headache, d izziness and drowsiness. Anaemia, neutropenia, cardiac arrhythmi as and visua l disturbances are rare. Administered as an oral suspension, absorption of posaconazole is improved by low pH and fatty food. It is partly metabolized by CY P2C I 9 and g lucuronidation, but excreted most ly unchanged in faeces. The t½ is > 24 hours. It can increase levels of drugs metabolized by CY P3A4. Dm·e: 200 mg QID or 400 mg BD with meals. NOXAFIL 200 mg/5 ml susp.

ALLYLAMINE

Terbinafine This orally and topically active drug against dermatophytes and Candida belongs to a new allylamine class of anti fungals. In contrast to azoles wh ic h are primaril y fung istatic, terbinafine is fungicidal. It acts as a noncompetitive inhibitor of ·squalene epoxidase', an early ste p enzyme which generates squalene epoxide that is converted to lanosterol and then to ergosterol by fungi (s ee Fig. 58. I) . Accumulation of squalene within the fu ngal cells appears to be responsible for the fungicidal action. The mammalian enzyme is inhibited only by I 000-fold higher concentration of terbinafine.

Approximately 75% of orally administered terbinafi ne is absorbed, but only 5% or less from unbroken skjn. First pass metabolism reduces oral bioavailability to < 50%. It is widely distributed in tissues, strongly plasma protein bound and has high affinity for keratin. Therefore, it is concentrated in sebum , stratum corneum of skin and into nail plates. Inacti vation occurs by me tabolism, and it is excreted mainly in urine, but about 20% in faeces as well. Elimination t½ after single dose is 11 -16 hours, but is prolonged to IO days after repeated dosing. Oral terbinafine is usually well to lerated. Side effects are gastric upset, headac he, rashes, taste disturbance. Some cases of hepati c dysfunction, haematological disorder and severe cutaneous reaction are repo rted. Terbinafine does not inhibit CYP450. Topical terbinafine can cause erythema, itching, dryness, irritation, urticaria and rashes.

Use Terbinafine applied topically as I% cream tw ice daily is indicated in localized tinea pcdis/ cruris/corpo1is and pityriasis versicolor; 2-4 weeks treatment is required according to the site, yielding high efficacy. Oral treatment with 250 mg OD is reserved for onychomycosis, tinea capitis and wide spread lesions, but is not effective in pityriasis versicolor. Duration of treatment varies from 3--6 months or more depending on the site. Efficacy in nail infection is - 90%, wh ich is higher than with griseofulvin and itraconazole. Terbinafine is less effective against c utaneous and mucosal candid iasis: 2-4 weeks oral therapy may be used as an alternative to fluconazole. LAM ISI L. SEBIFIN. DASKIL 250 mg tab, 1% topical cream . EX IFINE 125, 250 mg tabs, 1% cream; TERBIDER"v1 1%

cream.

OTHER TOPICAL ANTI FUNGALS All these drugs are used for dermatophytosis.

1. Tolnaftate It is an effective drug for tinea cruris and tinea corporis, and most cases respond in 1- 3 weeks. Because of poor penetrab ility,

847

848

ANTIMICROBIAL DRUGS

it is less effective in tinea pedis and other hyperkerati nized lesions. For the same reason , it is ineffecti ve in tinea capitis (invol ving scalp) and tinea unguium (involving nails). Symptomatic re li ef occurs early, but if applications are discontinued before the fungus bearing tissue is shed- relapses are common. Tolnaftate causes little irritation, but is inferior in efficacy to imidazoles. It is not effective in candidiasis or other types of superficial mycosis. TINADERM, TINAVATE 1% lotion, TOLNADERM 1% cream.

2. Ciclopirox olamine It is a newer drug effective in tinea in fections, pityriasis versicolor and dermal candidiasis . High cure rates are reported. It penetrates superficial layers and reaches hair roots but systemic absorption is negligible. Local tolerance without irritation is good. Sensitization occurs occasionally. Form ulated as nail lacquer (painted like nail polish), it has been used in onychomycosis, but cure rate is low. Vaginal candidiasis can be treated by I% ciclopirox vaginal cream. BATRA FEN I% cream, I% topical solution, I% vaginal cream, OLAMN 1% cream; ONYLAC NAIL LACQUER 8% nail lacquer.

C"ir

3. Butenafine

It is a benzylamine congene r of terbinafine with the same mec hani sm of action. However, it is used only top ically in dermatophytosis. Efficacy in tinea cruris/ corporis/ pedis is s imi lar to that of topical terbinafine. BUTOP, FINTOP I% cream; apply locally once or twice daily. 4. Undecylenlc acid It is fun gistatic used topically, generally in combination with its zinc salt. It is inferi or to the drugs described above; cure rates are low even after prolonged treatment However, it is still used for tinea pedis, nappy rash and tinea cruris. Irritation and sensitization are infrequent. TINEAFAX: Zinc undeccnoate 8%, .i:inc naphthenate 8%. mcsulphcn 8%, methyl salicylate 2.5%, terpineol 2.5% oint. 5. Benzoic acid It has week antifon gal and ant ibacterial property in slightly acidic medium. Eradication of the fungus needs prolonged appl icat ion till in fected keratin is totally shed. On hyperkeratotic lesions, it is used in combination with sa licylic acid (as Whitfie ld ointment: benzoic acid 5% or 6% + sal icyli c ac id 3%). The latter, by its keratolytic action, helps to remove the in fected tissue and promotes the penetration of benzoic acid into the lesion. Irritation and burn ing sensation are experienced by many patients. RINGCUTTER, WHITEFIELDS oint/cream 20 g tube.

PROBLEM DIRECTED STUDY

58.1 A SO-year-old wo man presents with com pla ints of constant pain in the retrosternal region for the past 2 wee ks. The pa in is markedly aggravated during swallowing. The condition has prog ressively worsened, and now even dri nking water hurts. There is difficulty in swa llowing as well. She informs tha t she is a dia be tic and takes Tab. Glibe nclamide 5 mg twice a day for the past two years, but has not checked her blood glucose fo r t he last few months. Endoscopy reveals diffuse streaks of creamy yellow mucosal plaques and a few erosions in the esophagus. Scrapings from the plaq ue are sent for mi crobiological exami natio n. Fasting blood glucose is found to be 180 mg/ di. She is diagnosed as a case of esophageal candidiasis with poorly controlled diabetes mel litus. (a ) What drug/drugs sho uld be prescribed t o treat her esophageal condition? What should be the duration of therapy? (b) What are the aspects to be conside red in view of the fact that th e patie nt is a poorly co ntrolled diabetic t aking a sulfonylurea medication? (see Append ix-! for solution)

Chapter

59

Antiviral Drugs (Non-retroviral)

Viruses are the ultimate expression of obligate intrace ll ular parasitism. They not onl y take nutrition fro m the host cell but also direct its metabolic machinery to synthesize new virus particles. Vi ral c hemotherapy, therefore was cons ide red impossible, as it woul d requ ire interference with cellular metabolism in the host. However, in the past 50 years virus directed enzymes have been identified in the infected cell and some viruses have few enzymes of their own which may have higher affi nities for some antimetaboli tes or inhibito rs than the regu lar cellular enzymes. In addition, drugs have been developed which target virus specific steps like cell penetration, uncoating, reverse transcription, virus assembly or maturation and re lease from host cell, etc. Another stumbling block is that

in majority of acute infections v iral replication is already at its peak when symptoms appear. To be effective, therefo re, therapy has to be started in the incubation peri od, i.e. has to be prophylactic or preemptive.

ANTI-HERPES VIRUS DRUGS These a re drugs active aga inst the Herpes group of D A viruse wh ich include He,pes simplex virus-} (HSV- l ), Herpes simplex virus-2 (HSV2), Varicella-Zoster virus ( VZV), EpsteinBarr vin,s (E BV), and Cytomegalovirus (CMV). ldoxuridine It is 5-iodo-2-deoxyuridine ( IUDR), which acts as a thymidine analogue. It was the first pyrimidine antimetabolite to be used as antiviral drug. It competes with thymidine, gets incorporated into viral D A so that faulty D A is fonned which breaks down easily. ldoxuridine is

ANTIVIRAL DRUGS (Non-retroviral)

Anti-herpes virus drugs

Anti-influenza virus drugs

ldoxuridine Trifluridine Acyclovir Va lacyclovir Famciclovir Ganciclovir Valganciclovir Cidofovir Foscarnet

Amantadine Rimantadine Oseltamivir Zanamivir Peramivir

Anti-hepatitis virus drugs

B Lamivud ine Entecavir Adefovir dipivoxil Tenofovir Telbivudine

For hepatitis C Ribavirin Interferon ex

Sofosbuvir Simeprevir Daclatasvir Ledipasvir Velpatasvir

850

ANTIMICROBIAL DRUGS

effective only against DNA viruses and clinical uti lity is limited to topical treatmen1 o f Herpes s implex keratitis. Because of low virus selectivity, higher local toxicity and rapid development of viral resistance, use of idoxuridine is reslricted to superficial dendritic keratitis when rapid action is required. ldoxuridinc eye drops act faste r than acyclov ir eye ointment, which is more effective when there is stromal involvement of the cornea. Ocular irritation occurs with idoxuridine eye drops.

Dose: 0.1% eye drops to be instilled hourly, then 2 hourly and 4 hourly; apply 0. 1% eye ointment at night. IDURJ N, TOXIL 0. 1% eye drops and eye oint. Trifluridine it is a fl uorinated nucleoside which acts in the same way as idoxuridine, and inhibits HS V- 1, HSV-2, C 1 V and related viruses. However. virus selectivity is low and D A synthesis in host cells is also affected due to blockade of cellular kinascs. Trifluridine eye drop is approved for use in H. simplex keratitis. Higher efficacy than idoxuridine eye drops is reported. Ocu lar irritatio n and lid edema can occur.

Acyclovir This deoxiguanosine analogue requires a virus specific enzyme for conversion to the active metabolite th at inh ibits D A synthes is and viral replication. Acyclovir Herpes virus specific thymidine kinase Acyclovir monophosphate Cellular kinases Inhibits herpes virus D A polymerase competitively Acyclovir ~ triphosphate Gets incorporated in viral D A and stops lengthening of D A strand. The terminated D A inhibits DNA-polyme rase irreversibly.

been found to develop resistance to acyclovir during therapy; the forme r primarily due to mutants deficient in thymidine kinase activity a nd the latter primari ly by change in specificity of virus directed enzyme so that its affi ni ty for acyclov ir is decreased.

Pharmacokinetics Onl y about 20% of an oral dose of acyclovir is absorbed. It is little plasma protein bound a nd is w idely distributed atta in ing CSF concentration that is 50% of plasma concentration. A fter topical appl ication systemic absorption is negligib le, but it penetrates cornea well. Acyclovir is primarily excreted unchanged in urine, boLh by glomerular fi ltration and tubular secretion; p lasma t½ is 2- 3 hours. Rena l impairment necessitates dose reduction. /:OV IRAX 200, 800 mg tab, 250 mg/vial for i.v. inj ; CYCLOVIR 200 mg tab, 5% skin cream; HERPEX 200 mg tab, 3% eye o mt, 5% ~kin cream; OCUVIR 200, 400, 800 mg tab, 3% e)c oint, ACIVIR-DT 200, 400,800 mg tab, 250 mg mJ . ACIVIR EYE3%omt.

t

Use Acyc lovir is e ffecti ve for herpes virus infecti on in patients with normal as well as defic ient immu ne sta tu s . For better res ults, treatment should start as early as possible.

!

l . Genital Herpes simplex Genera lly caused by type-2 virus; can be treated by topical, oral or parenteral acyclov ir depend ing on stage and severity of disease.

Acyclovir is preferentia lly taken up by the virus infected cells. Because of selecti ve generation of the active inhibitor in the virus infected cell and its greater inhi bitory effect on vira l DNA synthesis, acyc lovir has low tox ic ity for host cells: a several hundred-fold che motherapeutic index has been noted. Acyclovir is act ive only against herpes group of viruses; HS V- 1 is most sensitive followed by H SV-2 > VZV=EBV, while CMV is prac tically not affected. HSY and VZV have

Primary di sease: Topical treatment has low efficacy; 5% o intment is applied locally 6 times a day for 10 days. This is useful only if started early and in mild cases. Late and more severe cases should receive oral therapy (2 g/day in 5 di vided doses or 800 mg TDS for IO days) in addition to local therapy. Both local and oral therapies afford symptomatic re lief and haste n healing of lesions, but do not prevent recurrences. Recurre nt d isease: Topical therapy is totall y ineffective. Response to oral treatment is slow and incomplete; severe cases may be treated parenterall y- 5 mg/kg i.v. infused over I hr, repeated 8 ho url y fo r IO days. Suppressive ora l therapy w ith 400 mg BO has been shown

ANTIVIRAL DRUG S (N ON-RETROVIRAL) to preven t recurrences as long as g ive n. It is recommended to stop treatment after I yr and ascerta in whether the patient is still having recurrences; if so restart treatment. After prolonged therapy frequency of recurrences is reduced. Continuous acyc lovi r proph ylaxis is generally advocated in patients with > 8 recurrences per year. However, suppressive therapy reduces, but does not toally prevent, disease transmission to sexual partner.

2. Mucocutaneous H. simplex It is a type- I virus disease, remains locali zed to lips and gums; does not usually require specific treatment. but acyclovir skin cream may provide some relief. Spreading lesions may be treated with IO day oral acyclovir. Prophylactic oral therapy may prevent sun exposure related recurrences. The disease often gets disseminated in immunocompromised individuals and may be treated with oral or i. v. acyc lovir ( 15 mg/kg/day) for 7 days, but recurrences are not prevented. 3. H. simplex encephalitis (type- I vi ru s): Acyclovir 10 to 20 mg/kg/8 hr i.v. for :::to days is the drug of choice. Treatment is effective on ly if started early: delay precludes salutary effect on morta lity and neurological complications. 4. H. simplex (type I) kerafitis: Acyclovir is equally efTective as idoxuridine in superficial dendritic corneal ulcer. and may be better for deep stromal infections because of good corneal penetration. Though acyclovir eye ointment acts slower than idoxuridinc eye drops, blindness can be prevented. The eye ointment should be applied 5 times dai ly till 3 days after healing.

5. ffetpes zoster: The varicella-zoster virus is less susceptible to acyclovir. As such, higher doses are needed and it should be used only in immunodeficient individuals or in severe cases: IO m g/kg/8 hr i. v. for 7 days. Oral therapy with 800 mg 5 times daily is beneficial only if started early. It affords symptomatic relief and faster healing of lesions. Postherpet ic neuralgia is not prevented, though its duration may be shortened. Acyclovi r skin cream may be applied on herpetic ulcers.

6. Chickenpox: Specific therapy is indicated only in patients with immunodeficiency and in neonates. Acyclovir ( 15 mg/kg/day i. v. x 7 days) is the drug of choice. It reduces fever, eruptions, hastens healing and prevents visceral comp I ications. Oral acyclovir is not advised in children. In susceptible contacts, oral acyclovir 400 mg 4 times a day for 7 days given during the incubation period may abort chickenpox.

Adverse effects Topical: Stinging and burning sensation after each application. Oral: The drug is well tolerated ; headache, nausea, malaise and some CNS effects are reported. Intravenous: Rashe , sweating, emesis and fall in BP occur only in few patients. Dose-dependent decrease in g. f.r. is the most important toxic ity; occurs especially in those with kidney disease; normal ises on discontinuation of the drug. Reversible neurological manifestations (tremors, lethargy, disorientation, hallucinations, convulsions and coma) have been ascri bed to higher doses. No tera togenic potential has been noted. Valacyclovir It is a valyl ester prodrug of acyclovir w ith improved oral bioavailability (55- 70%) due to active transport by peptide transporters in the intestine. During passage through intestine and liver, it is completely converted to acyclovir in the first passage by esterases. Thus, >3 fold higher plasma levels of acyc lovir are obtained improving clinical efficacy in certain conditions; e.g. it is the drug of choice in herpes zoster. Yalaciclovir is excreted in urin e as acyclovir with a t½ of 3 hours. Dol·e: For genital herpes simplex-first episode 1.0 g BD x 10 days; recurrent episode 1.0 g BD x 3 5 days: suppressive treatment 0.5 g OD x 6-12 months (max 5 years). For orolabial herpes 2 g 8D x 1 day; in immunocompromised pa1ic111 I g BD x 5 days. For herpes zos1cr I g TDS x 7 days.

\ 'ALCIVIR 0.5 g. 1.0 g lah~.

851

852

ANTIM ICROB IAL DRUGS

Famciclovir It is an ester prodrug of a guanine nucleoside analogue penciclovir, which has good oral bioavailabili ty and prolonged intracellular t½ (> 8 hrs) of the active Lriphosphate metabolite. Like acyclovir, it needs viral thymidine ki nase for generation of the active D A polymerase inhibitor. Famciclovir inhibits H. simplex, H. zoster but not acyclovir-resistant strains. Some activity against hepatitis B virus (HBV) has been noted. It is used as an alternati ve to acyclovi r for genital or orolabial herpes and herpes zoster. Early treatment of herpes zosler reduces the duration of post herpetic neuralgia, but not its incidence. Dose: Genital herpes (1st episode) 250 mg TDS x 5 days; recurrent cases 250 mg BD for up to I year. Herpes 1oster and orolabial herpes 500 mg TD fo r 7- 10 days. FAMTREX 250, 500 mg tabs.

Famciclovir is a less active alternative to lami vudine in chronic hepatitis B, but not in resistant cases. Side e ffects are headache, nausea. loose motions, itching, rashes and mental confusion. Drugs for cytomegalovirus (CMV) Ganciclovir II is an analogue of acyclovir which is most active again t CMV. but also inhibits other herpes viruses, viz. H.simplex, H. =osrer and EBV. Ganciclovir is also activated in tracellularly by vi rus s pecific thymidine kinase and its triphosphate nucleotide preferentially inhibits viral DNA polymerase. This active metabolite auains much higher concentration inside CM V infected cells. The precursor cells in bone marro" are also quite sensitive to ganciclovir, and this may account for its bone marrow toxicity. Due to poor oral absorption, bioavailability of ganciclovir is low (< 10%). Its prodrug Valga11cic/ovir, which is better absorbed orally; has replaced it for oral therapy. Ganciclovir and its active metabolite are mostly excreted unchanged in urine. The plasma t½ of ganciclO\ ir is 2-4 hrs, but that of its triphosphate inside CMV infected cells is > 24 hrs. CMV can develop ganciclovir resistance by mutation o f viral phosphokinasc and/or viral DNA polymerase. Systemic tox icity of ganciclovir is high (bone marrow depression, rash, fever, vomiting. neuropsychiacnc disn1rbances). Therefore. use is restricted to prophylaxis and treatment of severe CMV infections (pneumonia/colitis/retinitis) which occur in immunucompromiscd (A IDS, transplant n,:cipient) patients. Treatment may be initiated with i.v. infusion of ganciclov ir IO mg/kg/day "hich can prevent blindness in AIDS patients with CMV retinitis. Systemic therapy can be supplemented by intravitreal injection of ganciclovir. Colitis or pneumonia caused by CMV in immune deficient patients has also been treated by i.v. ganciclovir. GANGUA RO 250, 500 mg tnbs.

Valganciclovir It is the valyl prodrug of ganciclovir that is 60% bioavailable orally (about 8 fo ld higher than ga ncic lovir). Oral valganciclovir is nearly equally effective as i.v. ganciclovir in CMV retinitis, and has practically replaced the parent drug, except when vision is threatened. Valganciclovir is also suitable for long term suppressive therapy of CM V retinitis and other CMV infections. It is indicated for prophylaxis of CMV infections in organ transplant/immunosuppressed patients. Adverse effects of valganciclovir are similar lo those of ganciclovir. Dose: 900 mg OD, 8D. VALGAN, VALSTEAD 450 mg tab,

Cidofovir

It is a monophosphate nucleotide analogue of cytidine which inhibits most DNA viruses inc luding I-ISV. CMV. pox and adenoviruses. Many HSV resistant to acyclovir and many CMV resistant to ganciclovir are susceptible. Because it is a monophosphate. it does not require viral phosphokinase for activation, and is converted to the active diphosphate by cellular enzymes. C idofovir diphosphate rema ins intracell ularly for long periods in virus infected cells to inhibit viral DNA polymerase, as "ell as acts as its a lternative s ubstrate. Weekly therapy is. therefore, possible desp ite short plasma 1\/, (2-3 hours) o f cidofovir itself. CMV develops cidofovir resistance by mutation o f its DNA polymerase. Very little cidofovir is absorbed orally. It is administered by i. v. infusion with pre and post dose oral probenecid which inhibits its tubular secretion and improves its availability for e111ering into cells. as well as reduces nephrotoxicity. Cidofo, ir S mg/kg i. v. weekly and then every 15 days is used for CMV retinitis in A IDS patients. particularly those who have fai led ganciclovir therapy. It can also be used for acyclovir-resistant mucocutaneous herpes simplex in immunosuppressed patients. Cidofovir can be applied topically on anogenital "ans. The primary toxicity of cidofovir is dose related kidney damage and it is contraindicated in patients with kidney disease. Gastric disturbances, constitut ional symptoms. hypersensitivity reactions. ncutropcnia and uveitis are the other adverse e ffects.

Foscarnet

It 1s a si mple s traight chain phosphonate unrelated to any nucleic acid precursor which inhibits viral DNA polymerase and reverse transcriptase. It is active against H. simµlex (including strains resistant to acyclovir), CMV ( including gnneiclovir-resistant ones), other herpes group viruses and HI V. Viral resistance to foscarnet is minimal. However. viral select ivity of foscarnet is low. Oral absorption is poor. Its t½ is 4- 8 hours. and it is not metabolised. Toxicity of J'oscarnet is high: uamagcs kidney-produces a renal diabetes like condition. acute renal fai lure can also

AN TIVIRAL DRUGS (NON-RETROVIRAL) occur. Anaemia. phlebitis, tremor. convulsions and other neurological as we ll as constitutional symptoms due to hypocalcaemia are frequent. Administered by i.v. infusion, foscarnct has been used for: I. CMV retinitis and other CMV infections in AIDS patients: efficacy is similar to ganc iclovir, includes resistant case , but produces more adverse enects. 2. Acyclovir-resistant mucocutancous H. simplex type 2 and varicella-zoster infections in AID patients. When used to treat associated CMV/H. simplex/VZV inlection in AIDS patient, it decreases HIV viral titre, and may improve outcome in patients receiving antiretroviral therapy (ART).

ANTI-INFLUENZA VIRUS DRUGS T he anti-influenza virus d rugs exert type a nd strain specific action on infl uenza virus. lnflucna virus is an R A virus with segmented genome and core prote ins whic h defi ne its type (A, 8 or C). Maj ority of human infections and epidemics are caused by influenza A, while the B type produces sporadic seasonal cases. Several subtypes of influenza A characteri7cd by 11s haemagglutinin ( H) and neuraminidase (N) surface glycoproteins have produced epidemics and pandemics from Lime-to-time. Of particular importance are the IISN I (bird flu ) subtype which got transferred from poultry in 1997 and caused severe epidemics in cas t Asia, and the 111 I subtype (swine fl u) which produced a pandemic in 2009 and several epidemics thene afier. Amantadine Introduced in the I 960s, it is a tricyclic am ine unrelated to any nuc leic acid precurso r. w hich inhibits inn uen7a A virus, but not innuenza 13 or the HSN I and H I I strains of influcnn A. Amantadinc acts by inhibiting the viral M2 protein (an ion channel). and prevents uncoating of the viral genome within the infected cell. Viral resistance to amantadinc developed by mutation of the M2 protein. Due to the emergence of 115N I and other strains (since the 1990s) which were unresponsive to aman tadine. a;, "ell as due to a, ailability of newer broad spectrum and more efficacious drugs. like Osaltamivir, the use of amantadinc declined. Currently it is seldom used for influenza. Its use for parkinsonism is also occasional no\\. DosC'· prophylactic 100 mg OD, therapeutic 100 mg 13D for 5 days. A YIA 'JTREL I CX) mg Lah. Rlmantadine It is the methyl dcrivut1vc of amantadine; s1m1 lar to it in mechanism of action, but longer acting and bcner to lerated. However, it exhibits complete crossresistance "ith amantadine. and is inactive against IHI , NSH I strams of influenza A as well as influenza B viruses. As such. rimantadine has also gone into disuse. Dose: prophylactic I 00 mg OD. therapeutic I 00 mg 130. FLUI\IAOINE 100 mg tah.

Oseltamivir

This is the most commonly used anti-influenza virus drug now. It is a sialic acid analogue with broad spectmm act ivity covering influenza A (ama ntadine sensiti ve as we ll as resistant), H5N I (bird fl u), H I I (swi ne flu ) strains and infl uenza B. Oseltamivir is an ester prodrug that is rapidly a nd nearly completely hyd rolysed d uri ng absorptio n in intestine and by liver esterases to the active form oseltamivir carboxylate with an oral bioavailability of ~ 80%. The active metabolite is not further metabolized and is exc reted by the kidney w ith a t½ of 6- 10 ho urs. Dose reduction is needed in renal. insufficie ncy. Oselta mi vir acts by in hibiti ng infl uenza virus ncurami nidase enzyme w hich is needed for release of progeny virions from the infected cell . Spread of the vims in the body is thus checked. Resistance can develop by mutation of the viral neuraminidase enzyme. In many areas oseltam ivir-resistant in fl uenza A (seasona l infl uenza) and H5 I have been encountered, though swine fl u (HI N I) is still mos tl y sensi ti ve. Some oseltam iv ir- res istant strains remain s usceptible to zanamivir. Oseltamivir (as well as zanamivir) is the most effective drug for influenza A, inc luding H5 I and HI N I stra ins, as well as influenza B. Sta rted w ithin few hours ( maximum 48 hours) of sympto m onset, it red uces seve rity, duration and complications o f the ill ness. Early initiation of therapy is c ri tical because viral replication is max imal during the fi rst 2 days of symptom onset. Oseltamivir may no t be elTective in infants, because they may fail to generate the active metaboli te. Prophy lactic use w ithin 2 days of exposure prevents illness in contacts of influenza pat ients. Dose: therapeutic 75 mg oral BO for 5 days; prophylactic 75 mg OD. TAMIFLl . ANTIFLU 75 mg cap, 12 mg/ml su,p., Fl.l VIR 75 mg cap.

Side elTects a re nausea and abdomina l pain due to gastric irri tation (reduced by taking the drug with food), headache, weakness, sadness, diarrhoea, coug h and insom nia. Skin reactions have been reported.

853

854

ANTIMICROB IAL DRUGS

Zanamivir Another influenza A ( including amantadine-resistant, HI I, H5 I strains) and influenza B virus neuraminidase inhibitor that is administered by inhalation as a powder due to very low oral bioavailability. Small amount that is absorbed after inhalation is excreted by the ki dney with a t½ or 2- 5 hours. The mechanism of action, clinical ut ility and efficacy of zanamivir are similar to that of oseltamivir. Some strains resistant to oscltamivir remain sensiti ve to zanami vi r. Therefore, it should be reserved for oseltamivir-resistant cases, or when the latter cannot be given, or fo r severely immunocompromized patients. Prophylactic use may be made for 7- 10 days in household contacts. Dose: IO mg through breath actuated inhaler or rotacaps, 13D >< 5 days for treatment and OD for prophylaxis. RELENZA. VIRENZA 5 mg/actuatio n powder inhaler rota caps.

The inhaled powder can induce bronchospasm in some individuals. This may be severe in asth matics, therefore contraindicated in them. Headache, dizziness, nausea and rashes are mild and infrequent side elTects. Peramivir It is a new s ingle dose i.v. treatment for in nuenza in adults approved by the US-FDA in 2014. Peramivir is active against human influen7a A and B. as well as bird flu ( H5Nl). swine nu (111 1) and se veral emerging strains of influenza A virus, including those resistant to oseltamivir. Like oseltamivir, it inhibits influenza viral neuraminidase e,u.yme, pre, e nts spread of the virus in hos t body, and is effective if administered soon after symptom onset (max. 48 hrs). Peramivir is more active in vi1ro against innue n-:a 13, which is intrinsical ly less susceptible Lo oseltamivir and nnamivir. In multicentric comrolled study, s ing le dose i.v. peramivir administered within 48 hours of symptom onset was found Lo be as cfTcctivc as 5 day ose ltamivir in reducing severity and duration of influenza illness. Oral bioavai lability of preramivir is ,ery Im, (::,3%) in humans. It is almost entirely eliminated unchanged by glomerular filtra tion and its clearance paralle ls C Lcr. Dose needs to be reduced in re nal insufficiency. but not in liver disease. The elimination t½ in hea lthy adults is 20 hr. The long t½ allows single or infrequent dosing. Adverse elTecl of peramivir arc mild and similar to those of oral osel tamivir. Self limiting diarrhoea, nausea and vomit ing arc the usua l complaints. Decreased neulrophi l count peaking on 3"' day with ,pontaneous recovery has been noted. Because of well tolerat~d and safe sing le dose i.v. use. peramivir appears Lo he a valuable option for treating severely ill inAucn1,a patients, or those

unable to tolerate other drugs, as well as for oseltami vir res istant cases.

Dose: 600 mg 10 be diluted and injected i.,. over 1520 m in, within 4 8 hours of innuenza symptom o nset ( internationally marketed as RAPIVAB 600 mg inj.).

ANTI-HEPATITIS VIRUS DRUGS While hepatitis B virus (HBV) is a DNA virus which, like retroviruses, can integrate into host chromosoma l D A to estab lish pennanent infection, the hepatitis C virus (HCV) is a RNA virus, which does not integrate into chromosomal DNA, does not establish noncurable infection, but frequently causes chronic hepatitis. Some antiviral drugs (e.g. ribavirin, interferon a) used primarily for hepatitis viruses are nonselective and inhibit several other viru es, includi ng both RNA and DNA viruses.

DRUGS FOR HEPATITIS B Since the hepatitis B virus cannot be eradicated from the body, the treatment is aimed as suppression or ihe virus and its inflammatory and hepatocyte damaging response. This resul ts in improved liver function and reduction of risk for developmem of cirrhosis and hepatic carcinoma. Sudden stoppage of anti-HBV drugs can results in acute exacerbation of hepatitis. Most expert opine that un less loss of efficacy or toxicity limits use, the nucleoside/nucleotide anti-I IB V drugs should be continued till 6 months after seroconversion. Combination therapy of HBV has not proven superior, and drugs are generally used sequentially. Lam ivudine Th is nuc leoside analogue is active against HB V as well as HI V, and is described w ith anlirctroviral drugs on p. 862.

Entecavir This newer guanosine nucleoside analogue is currently the most active and Ist Iine option for treating chronic hepat itis B. Like other nucleoside analogues, entecavir inhibits llBVD A polymerase after activation by intracellular phosphorylation. ll brings down HBV titre more rapidly and more profoundly than lamivudine, and this is attended by clinical, biomecha nical and histological improvement. The response is

ANTIVIRAL DRUGS (N ON-RETROVIRAL) well sustained and H BY-resistance to entecavir is rare upto 5 years of use, but is more li kel y in cases a lready resistant to lamivudine. It has weak a nti- HI V acti vity as well . Entecavir is nearly completely absorbed after oral dosing, but food decreases bioavailability; it should be taken in empty stomach. It is not metabolized, and is excreted unchanged by the kidney with a biologial t½ of 128- I48 hours. Entecavi r is well tolerate d; side effects are mild dyspepsia, nausea, diarrhoea, fatigue, and disturbed sleep. Lactic acidosis can occur in patients with decompensated cirrhosis of liver. Dose: 0.5 mg OD; for lamivudine resistant cases 1.0 mg OD. ENTAVIR 0.5 mg, 1.0 mg tabs.

Adefovir dipivoxil

Adefovir is a mo nophosphate analogue of AMP that is active against HBV and some other D A as well as R A viruses, but is used only for hepatitis caused by HBV. Esterases in the intestine and liver release the active drug during absorption to a ttain ora l bioavailability of ~60% in terms of adefovir. On entering cells, adefovir is phosphorylated to the diphosphate which has high affinity for HBV DNA polymerase compared to host cell D A polymerase. Adefovir itself gets incorporated in the viral DNA resulti ng in termination of the DNA chain. It is primarily excreted by the kidney. While adefovir plasma t½ is 7 hours, intracellular t½ of its diphosphate is upto 18 hours. Adefovir is indicated in chronic hepatitis B, inc lud ing lamivudine-resistant cases and those havi ng concurrent HIV infection. Clinical, biochemical, histological, sero logical and virologica l response occurs in nearly 50% patients within I year. More cases respond with continued treatment. Virologic response to adefovir is slower than to lamivudine and tenofovi r and it is the least active nucleotide ana logue against HBV, therefore not a first line drug. Adefovir resistance occurs in about 30% patients. Dose: IO mg/day; A DESERA, ADf OVIR 10 mg tab.

At IO mg/day dose adefovir is well tolerated. Side effects are sore throat, headache, weakness, abdominal pain and fl u syndrome.

ephrotoxicity occurs at hi gher doses and in those with preex isting renal insufficiency.

Tenofovir disoproxil fumarate (TDF)

It is a monophosp hate nu c leotide re lated to AM P, which is active against HBV as well as HIV. Due to very low oral absorption, it is used as the disoproxil ester prodrug, which not only improves bioavailability, but also intracellular passage of the acti ve form. Tenofovir released from hydrolysis of the prodrug is diphosphorylated by cellu lar kinases into tenofovi r d iphosphate which preferentia lly inhibits HBY-DNA polymerase and HI V-reverse transcriptase (see p. 863). Affinity for host DNA-polymerase is very low. It also gets incorporated in the vira l D A to cause chain termination. Tenofovir disoproxil is incompletel y, but adequately absorbed after oral intake, and is largery excreted by the kidney with a plasma t½ of ~ I 6 hours. It produces few side effects, which are mostly limited to the g. i. tract viz. nausea, fl atulence, abdom inal discomfort, loose motions and headache. Slight increase in serum creatinine is common and acute rena l injury or chronic renal disease is possi ble. Drug interactions are not significant. Adm inistered in a dose of 300 mg daily, tenofovir disoproxil has produced good clinical and viro logical response in c hronic hepatitis B. In a comparative study, higher percentage of patients responded within one year of use than with adefovir. A response rate of > 90% has been reported among HBe antigen negati ve patients. Tenofovir-resistance bas not developed during treatment of chronic hepatitis B, a nd it is effective in lamivudine-resistant cases. Due to its high efficacy, good tolerability and low risk o f resistance, tenofovir is one of the first line drugs fo r chronic he patitis due to HBV. T ENT IDE. TENOF. T ENVIR 300 mg tah; I tab OD.

Telbivudine

T his newer anti-HBV drug is a thymidine nuc leoside analogue. Telbivudine is absorbed ora lly and its bioavailability is not affected by food. It is not metaboli zed and is excreted unchanged by the kidney with an average plasma t½ of 15 hours. Telbivudine

855

856

ANTIM ICROBIA L DRUGS

enters cells and is phosphorylated by cellular kinases to generate the active triphosp hate nucleotide, which competitively inhibits HB VDNA polymerase, a nd also gets incorporeted into HBV-DNA resulting in chain te rmination. l;elb ivudine causes faster and more complete suppression of HBV-D A titre than lamivudine or adefovi r, but resistance o ften develops resulting in return of viraemia. Telbivudine-resistance is more likely in lamivudine-resista nt cases. As such, it not a first li ne drug for chronic hepatitis B. Tolerability of telbivudine is good; side effects are abdominal pain, d ia rrhoea, cough, headache. dizz iness and m yalgia, which are usually mil d. Serum amylase may rise.

persists for months after di scontinuati on; long term t½ is > 10 days. The most common therapeutic use of oral rib avirin is in c hronic hepa titis C, bu t it is not used alone, and is traditio nally combined with inj ected peginterferon for 6- 12 months. This is acti ve aga inst all genotypes of HCV and ach ieves SVR in 50- 80% cases. It is also used along with sofosbuvir based combination therapy in decompensated cirrhotics. ebulized ribavirin is used fo r respiratory syncytial virus bronchi olitis in infa nts and chi ldren, particularly those with congenital heatt disease, prematurity or other high risk conditions. Ribavirin has shown efficacy in some rare viral infections as well.

Dose: 600 mg OD: SEBIVOL 600 mg lllb.

Dose: 200 mg QID; 400 mg TDS for body weight ::: 75 kg (children 15 mg/kg/day). VIRAZIOC, RIBAVN 100. 200 mg caps, 50 mg,5 ml syr.

DRUGS FOR HEPATITIS C The aim of treatment for chronic hepati tis C is to attain sustained viral response (SVR), meanin g undetectable HCV-RNA in blood for at least 6 months after completion of the rapy. If this is achieved, only ~5% patients relapse. Oral ribavirin combined with injected PegINFa is conventionally the standard therapy for HCV infection, that is still widely used in India. However, several novel direct acting oral antiviral drugs have been developed over the past 5- 10 years which have revolutionized the treatment of hepati tis C. These new drugs have largely replaced the older ones in the developed world, but are accessible only to few patients in India due to constraints of cost and other logistics.

Ribavirin This purine nucleoside analogue has broad-spectrum antiviral activ ity, including that against HCV, influenza A and B, respiratory syncytial virus and many other D A and double stranded RNA viruses. Its mono- and triphosphate derivatives generated intracellularly by host kinases inhibit GTP synthesis and viral R A synthesis. They have other s ites of action as well. Viral resistance to ribavi rin is rare. Oral bioavailabi lity of ri bavirin is - 50%. 1t is partly metabolized and elim inated mainly by the kidney in a multiexponential manner. It accumulates in the body on daily dosing and

The major toxic effect is dose-dependent haemo lytic anaemia. Others a re bone marrow depression, CNS a nd g.i. symptoms. It is also tera togenic; fema le patients shou ld prac tice contraception during and ti 11 3 mo nths after riba v irin treatment. The aerosol can cause irri ta tion of mucosae and bronchospasm.

Interferon a Interferons (IF s) are low mo lecul ar weight glycoprotein cytokines produced by host cells in response to vi ral infections, TN Fa, IL- I and some othe r inducers. They have nonspecific anti viral as well as other complex effects on immunity a nd cell proliferation. Inte rferons b ind to speci fie cell surface receptors a nd a ffect viral replication a t multiple steps, viz. viral pene tration, syn thesis of vira l mR A, assembly of viral parti cles and their release, but the most w idespread effect is direct or indirect suppression of v ira l protein synthesis, i.e. inhibition o f translation . Interferon receptors are JA K-STAT tyros ine protein kinase receptors whi c h on activation phosphory late cellular proteins. The phosphory lated proteins then migrate to the nuc leus and induce transcription of' interferon-induced-proteins ' which exert antiviral effects.

ANTIVIRAL DRUGS (NON-RETROVIRAL) Interferons inhibit many R A and D A viruses, but they are host specific: those produced by another s pecies have poor acti v ity in man. Three types of human lF s (a, P and y) are known to have antiv iral activity. Only IFNa 2A and IFNo.28 produced by recombinant technology are available and are c linically used. Both are n o ng lycosylated low MW proteins administered by i.m. or s.c. injection. Their pegylated (polyethylene glycol complexed) forms are meant for s.c. injection at weekly intervals. Plasma levels of pegl F a 2A are sustained twice longer than those of peglF a 28 . After i.m./s.c. lllJection, interferon is distributed to tissues. It is degraded mainly in kidney and to some extent in liver, so that it remai ns detectable in plas ma for < 24 hours. However, cel lular effects arc longer lasting because the interferon induced proteins persist, so that IF is generally administered thrice week ly. Peginterferon is absorbed more s lowlyexerts more sustained efTccts. perm iti ng week ly administration and improving clinical efficacy. Peg IF has replaced IF , except for consideration of cost. ALFERON: Interferon a ~, JMU/vial mj. ZAVINEX 3MU, 5MU vials for 111j. llEAL~A-213, SIIANFEll01\. VIRAFERON: Interferon a,.. 3MU. 5MU vials for inj. PEGASYS: Pcginlcrferon a,. 135 µg/,ial and 180 µg,,1al for weekly s.c. IOJCClion.

Uses I. Chronic hepatitis B: I FNa2A 2.5- 5 MU/1112 or IF a 213 5-10 MU g iven 3 times per week for 4-6 months causes di sappearance of HB VD A from plasma and improvement in liver function tests/histology in nearly hat f of the patients. High doses ( IO MU) injected thrice weekly for 6 months often produce prolonged remission, but re lapses do occur. Pcgl FNa2A 180 µg s.c. once weekly for 24- 48 week s produces better and more s ustained responses.

2. Chronic hepatitis C: IF a 28 3MU 3 times weekly fo r 6-12 months has p roduced remiss ion in 50 70% patients. Viral R A becomes

undetectable and li ver function tes ts return to norma l. Histology improves if response is sustained. However, relapses occur in majority of patients. Peg lF a 2A 180 µg/week is more effective and induces longer lasti ng rem issions. Comb ination with oral ribavirin increases number of responders, decreases chances of relapse and is the standard therapy.

3. AIDS-related Kaposi's sarcoma: IF

is used to treat A I OS-re lated Kapos i's sarcoma, but not to treat HIV as s uc h. However, interferon accentuates haematolog ical toxicity ofzidovudine.

Condyloma acuminata: caused by papilloma v irus is us ually treated with topical podophyllin. Intralesiona l interferon injection may be used m refractory cases.

4.

5. H. simplex, H. zoster and CMV: IFN may be added to acyclovir/ganciclovir as an adjuvant in immunocompromised patients. 6. Interferons are a lso used in chronic myeloid leukaem ia, follicu lar lymphoma, cutaneous T-ce ll lymphoma and multiple myeloma. C linical utility of s.c. or i.m. injected interferon is limited by s ub tantial adverse effects.

Adverse effects Side effects of IF / Pegl FN are freque nt and di s tressing. They are contraindicated during pregnancy. in decompensated cirrhotics and in autoimmune disease patients. • Flu- like symptoms- fatigue, aches and pains, ma laise, fever, di zziness, anorexia, nausea, taste and visual disturbances develop few hours after each injection. but become milder later. • Neurotox ici ty-num bness, neuropathy, altered behaviour, mental depression, tremor, s leepine s. rarely convu lsions. • Myelosuppress ion: dose dependent neutropenia, thrombocytopen ia. • Thy roid dysfunction (hy po as well as hyper). • Hypotension, trans ie nt arrhythmias, a lopec ia and reversible liver dys function .

857

858

AN TIMICROB IAL DRUGS

New direct acting anti-HCV drugs As indicated above, several novel direct-acting antiviral dru gs for HCV are now avai lable. They target spec ific nonstructural (NS) viral proteins that play essential role in replication of HCV ins ide hepatocytes. All of them are used in combination, either among themselves or wi th ri bavirin ± Pegl Fa, because of lower efficacy and development of resistance if used as monotherapy. Used in combination, the newer drugs have achieved SVR of upto 99%, and have shortened the duration of therapy to 12- 24 weeks. They are also much less toxic than ribavirin or rNFa., but most of them do have important drug interacti ons. Therapy of HCV infection is at present rapidly evolving. The newer drugs may be gro uped according to their target S protein. I. NS5B polymerase inhibitor: Sofosbuvir. 2. NS3 protease inhibitor: Simeprcvir. 3. NS5A inhibitor: Dac latasv ir, Ledipasvi r, Velpatasvir.

NSSB polymerase inhibitor Sofosbuvir It is a uridine analogue prodrug which is converted into its triphosphate nucleotide within the hcpatocytes and inhibits the HCV nonstructural protein 58 ( S58) which is a HCV-R A polymerase. The active metabolite also enters the viral RNA and serves as a chain terminator. ofosbuvir is active against all (1-6) HCV genotypes, e pecially genotype I, but is always used in com binati on. either with one of the SSA inhibitors or with simeprevir, or with ribavirin ± PeglNFa. Such combinati ons have ac hieved SVR o f 85%- 9 9% after 12 weeks therapy in noncirrhotics and upto 93% after 24 weeks therapy in cirrhotics. Remarkably, resistance to sofosbuvir does not develop easily. Recent studies have shown that it can be employed in patients with decompensated li ver disease as well. The oral bioava ilability o f sofosbuvir is - 80% and is improved if taken with fatty meal. It is rapidly metabolized in liver and excreted in urine with a t½ of 27 hr. It is a substrate

for Pgp e fflux transporter, therefore should not be used with Pgp inducers like phenytoin, ri fampin, etc. and in patients with severe renal impairment. Adverse effects of sofosbt1vir are abdominal pa in, fa tigue, agitati on, joint pain and anaem ia. Severe bradycardi a and arrest has occurred in patients taking amiodarone. Dose: 400 mg dai ly with meals. MYHEP, SOFOCURE, SOVTII EP, HEPCINAT 400 mg tab.

NS3/4A Protease inhibitor Simeprevir This newer drng is a HCV protease 3 inhibitor wh ich blocks the cleavage of HCV polyprote in complex, so that functional viral R A is not formed and viral replication is halted . Simeprevir is acti ve against HCV ge no type I and 4 and is recom mended for use along with sofosbuvir or with ribav irin + PegINFa. The simeprevir- sofosbuvir combinatio n has ach ieved SVR in 83- 97% noncirrhotic patients after 12 weeks therapy, and in cirrhotic patients after 24 weeks therapy. Jt is orall y acti ve and should be taken with food to increase absorption . The terminal t½ is 13 hours in normal subjects, but is increased upto 41 hrs in IICV infected patients. Simeprevir is metabolized by CYP3A4. It is a substrate for, as well as inhibitor of the effl ux transporter Pgp. As such. clinically significant interactions occur with rifampi n, statins, NN RTI and protease inhibitor antiretroviral drugs, wh ich are an important limitation to simeprevir use. Its adverse effects are nausea, headache, dyspnoea, fatigue, rashes and photosensitivity. Dose: 150 mg/day OLYSIO, GALEXOS 150 mg cap.

NSSA inhibitors The NS5A is a multifunctiona l protein that serves essential role in the replication of HCV. Daclatasvir Ct is an orally active NS5A inhibitor which blocks 1-I CV-RNA replicatio n as well as assembly of progeny virions. Daclatasvir is metabolized by CYP3A and is a substrate for effl ux transporter Pgp. It also inhibits Pgp and other transporters. The usual dose 60 mg/day needs to be halved in patients receiving CYP3A inhibitor and increased to 90 mg/day in those

AN TI VIRAL DR UGS (NON-R ETROVIRAL)

taking CY P3A inducers. It is active agai nst all geno types of HCV. Coadministered w ith sofosbuvir, it is indicated in I-ICY genotypes 1- 6 infection and has achieved SVR upto 90% after 12 week therapy in no nc irrhotic patients, but the response rate is lower in cirrhotics. Add ition o f ribavirin to the regimen is recommended. In patients w ith HIV coinfection or with advanced liver disease, it can be employed regardless of th e HCV genotype. T he t½ of daclatasvir is 12- 15 hr. It is genera lly we ll tolerated; adverse effects are headache, fatigue, abdominal pain, alopecia, anaemia and rarely allergy. HEPECDAC, DAC IHEP, MYDEKLA 60 mg tab.

Ledipasvir T his NSSA inhibitor is approved for use in combination with sofosbuvir, and is marketed only as fixed-dose combination (LDV/ SOF) tablets for o nce daily dosing. Led ipasvir is strongly active against HCV genotypes 1,4,5,6 w ith o nl y week activity on genotypes 2 and 3. As such, LDV/SOF combination is indicated for genotypes 1,4,5,6 HCV, but can be used in H IV coinfected, as well as in those with decornpensated cirrhosi s. In noncirrhotic patients LDV/SOF combination has ach ieved SVR in 95- 99% cases after 12 weeks therapy, and in c irrhotics after 24 weeks therapy. Add ition of ribavirin to the regimen achieves the sa me SVR among c irrhotics in 12 weeks, and is effective in genotype 3 HCV in fection as well . Ledi-

pasvir absorption is dependent on gastric acid and is impaired in patients taking H 2 blockers or PPi s. Only - 30 ledi pasvir is metaboli zed, and it is largely excreted unchanged in faeces. The t½ is 48 hr. The LDV/SOF combination should not be used in patients being treated with Pgp inducers. HEPCl AT-LP, SOFAB-LP: ledipasvir 90 mg + sofosbuvir 400 mg tab; one tab OD.

Velpatasvir Availa bl e only as a fi xed dose combina ti on of velpatasvir 100 mg + sofosbuvir 400 mg (VEL/SOF), this SSA inhibito r is indicated in a ll ( 1- 6) genotypes of H CV hepatit is . In noncirrhotics it has ach ieved SVR of 95- 99% after 12 wee ks . T he SVR rates we re lower in decom pensated patients and could be improved either by addition of riba viri n or by ex tending the therapy to 24 weeks. Like Iedipasvir, the absorption of velpatasvir is also impai red m patients receiving acid suppressant drugs. It is partly metaboli zed and excreted mainly in faeces. The t ½ is 15 ho urs a nd liver disease has modest effect o n its c learance. Inducers of CY P3A and Pgp lower velpatasvir blood levels and should not be used with it. The usual adverse e ffects a re headache, fatigue, weakness and nausea. VE LPANAT: Ve lpatasvir 100 mg +- sofosbuvir 400 mg tab; one tablet/day.

859

Chapter

60

Antivi ral Drugs (Anti-retrovirus)

These are drugs active against human immunodeficiency virus ( HIV) wh ich is a retrovirus. They are useful in prolonging and improving the quality of life a nd postponing complications of acquired immunodeficiency syndrome (AIDS) or AIDS-related complex (ARC), but do not cure the infection. The clinical efficacy of antiretrovirus drugs is monitored primarily by plasma HIV-RNA assays and C D4 lymphocyte count carri ed out at regular intervals. HIV is a s ing le s tranded RNA retrovirus which unique ly carries out reverse transcript ion o f prov iral DNA fro m v iral R A (normally R A is transcri pted from D A) w ith the he lp of a viral RNA-dependent DNA polymerase (reverse transcriptasc). The primary cel l type attacked by HIV is t he CD4+ he lper T-ly mphocytc, but later macrophages and some other cell types may also be infected. When population of C D4 cells declines markedly (50% patients became nonresponsive to AZT within 1- 2 years therapy due to growth of resistant mutants.

free combination A RT to eligible patients. ln 20 I 5, an estimated 86000 new HIV infections occurred in India, which is a decline of32% compared to new HIV cases in 2007. After 2007, AIDS related deaths have steadily declined in India, and the annual death rate in 2015 was 54% lower than that in 2007, which in large measure. is due to effective use of combination ART.

Nucleoside reverse transcriptase inhibitors (NRTls) Zidovudine (AZT) It is a thymidine ana logue (azidothymidine), the first and prototype NRTL After phospho rylation in the host cell- z idovudine triphosphate selecti vely inh ib its viral reverse transcriptase in preference ro cellu lar DNA polymerase. S ingle-stranded v iral RNA

l

Vin1s d irected reverse transcriptasc (inlribi!ed by :idovudine lriphosphale)

Double-stranded proviral D A On the temp late of single-stranded RNA genome of HIV, a double-stranded D A copy is produced by vira l reverse transcriptase. This proviral DNA translocates to the nucleus and is integrated w ith chromosomal D A of the host

Pharmacokinetics The oral absorption of AZT is rapid, but bioavailability is ~65%. It is quickly cleared by hepatic gl ucuronidation (t½ I hr); 15- 20% of the unchanged drug along with

ANTIRETROVIRUS DRUGS

I I

I

I

Nucleoside reverse transcrlptase Inhibitors (NRTls)

Non-nucleoslde reverse transcriptase Inhibitors (NNRTls)

Zidovudine (AZT) Didanosine (dd.l) Stavudine (d4T) Lamivud i.ne (3TC) Abacavir (ABC) Emtricitabine (FTC) Tenofovir (IDF)

Nev irapine (NVP) Efavirenz (EFV) Delavirdine (DL V) Etravirine (ETV) Rilpivirine

I

IEnfuvirtide (T-20) I

I CCR-5 receptor inhibitor I Maraviroc I

*www.who.111t/gho/h1v/en (20 17) 1 www.naco.gov.in/documents/annual

Ritonavir (RTV) Atazanavir (ATV) Tndinavir (INV) Nelfinavir (NFV) Saqu.i.navir (SQV) Fosam p renavir (FPV) Lo pi.nav ir (LPV) Darunavir (DRV)

I I

I [Entry Inhibitor

I

Protease inhibitors (Pis)

rcpons (July 201 7)

I

I lntegrase Inhibitor Raltegravir Dolutegravir (DTG)

862

ANTIM ICROB IAL DRUGS

the metabolite is excreted in urine. It crosses placenta and is fou nd in milk. Dose Adults 300 mg BD; Children 180 mg/m 2 (max 200 mg) 8 hourly. RETROVIR, ZIDOVIR 100 mg cap. 300 mg tab, 50 mg/5 ml syr VIRO-Z, Z IDOMAX, ZYDOWJN 100 mg cap, 300 mg tab. (to be taken w ith ple nty of water).

Adverse effects Anaemia and neutropenia are the most important and dose-related adverse effects. ausea, anorexia, abdominal pain, headache, insomnia and myalgia are common at the start of therapy, but diminish later. Myopathy, pigmentation of nails, lactic acidosis, hepatomegaly are infrequent. Interactions Paracetamol increases AZT toxicity, probably by competing for glucuronidation. Azole antifungals also inhibit AZT metabolism. Other nephrotoxic and myelosuppressive drugs and probenecid enhance tox icity. Stavudine and zidovudine exhibit mutual antagoni sm by competing for the same activation pathway. Use Z idovudine is used in HIV infected patients onl y in combination with at least 2 other ARV drugs. However, it is no longer included in the preferred Ist line WHO (2016) regimen, but is a component of one alternative regimen. Didanosine (ddl) It is a purine nuc leoside analogue which a fter intracellular conversion lo didanosinc triphosphate competes with ATP for incorporation into viral DNA, inhibits HIV reverse lranscriptase and terminates proviral DNA. Mutational resistance develops, but only few AZT resistant mutants are non-responsive to didanosine a lso. It is infrequently used now due to higher toxicity than other N RTls. Dose: 400 mg/day (for ;;: 60 kg BW), 250 mg/day (< 50 kg BW) I hour before or 2 hour after meals. DINEX EC, DD RETRO, VI ROSINE DR 250 mg, 400 mg tabs. In contrast to AZT, it does not cause mye losuppress ion. T he major dose-related toxicity is peripheral (stocking and glove) ne uropathy, which may be irreversible. Rarely acute pancreatitis occurs. Diarrhoea, abdominal pain, dry mo uth and nausea arc the side e!Tects.

Stavudine (d4T) It is also a thymidine analogue which acts in the same way as AZT. Because of long term serious metabolic complications like lipodystrophy, lactic acidosis and peripheral neuropathy stavudine is no longe r employed in WHO as well as NACO regimens. except in exceptional circumstances. Dose: 30 mg BD STAG , STAVIR, \IIROSTAV 30 mg caps.

Lamivud ine (3TC) This deoxycytidine analogue is phosphorylated intracellularly and inhibits HIV reverse transcriptase as well as HB V DNA polymerase. Its incorporation into viral D A results in chain termination. Most human DNA polymerases are not affected and system ic toxicity of 3TC is low. Point mutation in HIV-reverse transcriptase and HBV-DNA polymerase gives rise to rapid lamivudine resistance. Ora l bioavailability of J TC is high and plasma t½ longer (6-8 hours). Intracellular t½ is sti 11 longer (> 12 hr). It is mainly excreted unchanged in urine, and it can be em ployed in patients with hepatic insufficiency. Lamivud ine is used in combination wi th other anti-H IV dru gs, and is as effective as AZT. lt synergises with most other NRTls for HJV, and is an essential component of most first line triple drug WHO and NACO regimens. Lamivudine is also effective in chronic hepatitis B; in which lower doses are required than for HI Y. This may be due to longer intracellular t½ of lami vudine in HBV infected cells than in HIV infected cells. HBV-DNA titre is markedly reduced and biochemical as well as histological indices of liver function improve. However, viral titres rise again after discontinuation. Even with continued medication HBV viraemia tends to return and lamiv udine-resistance develops in upto 70% patients within 1- 5 years. Because of this, it is no longer a Ist line drug for chronic hepatitis B. Lamivudine-resistant HBV remain responsive to tenofovir and adefovir, but usually not to entecavir or to telbivudine. Dose: For chronic hepatit is B- 100 mg OD For HIV infection- 150 mg B D, or 300 mg OD. LAM IVIR 150 mg tab, 150 mg/ 5 ml soln ; LA M IVIR- HB\I I 00 mg tab; H EPTAVIR, LA M IOAC , LAM UV JD, V IRO LAM 100. 150 mg tabs;

Lamivudine is generally well to lerated and has good safety profi le, because of which it is accorded high priority in use for HIV. Side effects are few- headache, fatigue, rashes, nausea, anorexia, abdominal pain. Pancreatitis and neuropathy are rare. Hematological tox icity does not occur. Abacavir (ABC) This guanosine ana logue is a clinically potent ARV drug that acts after

AN TI-RETROVIR US DRUGS intrace llular conversion to carbovir triphosphate, w hich gets incorporated in proviral DNA and terminates c hain e longation. Rapid reduction in plasma HIV-RNA count and rapid rise in C D4 cell count has been noted when a bacavir was given to A IDS patients. Resistance to ABC develo ps s lowly, and it exhibits little cross resistance with other NRTls. Its o ral bioavailability is 80% and it is mainl y e liminated by metabo lism. The plas ma t ½ is 1- 1.5 hou r, but intrace llula r t½ of the active me tabol ite is > 12 hours. Hy persens itivity reactions such as rashes, fever, abdomina l pain, bowel upset, flu-like respiratory and constitutional symptoms occur in 2- 5% adult patients and are the major problems. Abacavir must be promptly stopped when the reaction occurs, because fatalities are on record when patients developing the reaction were g ive n further doses of A BC. A genetic basis and mass ive release of TN Fa have been related to th is reaction. Abacav ir s hou ld never be given again to a patient w ho has developed this reacti on . IL s hould a lso be avo ided in patients with cardiovascular risk factors. Other side effects are not serious. Lypodystrophy is least likely. Avo idance of alcohol is adv ised. Combinat ion regimens including abacavi r a re freq uently used, and it is a component of the preferred I" line WHO regimen for children. Dose: 300 mg BD or 600 mg OD. ABAVIR, ABAMUNE 300 mg tab.

Tenofovir (TDF) This is the only nucleotide (not nucleoside) analogue that is a commo nly used a nti- HI V drug. It is a lso active aga inst HBV, and its pharmacology is described o n p. 855. Tenofov ir was initia ll y used on ly in previously treated patien ts, but because of good tolerability profile, it is now be ing included in first line regimens as well. Tenofovir containing regimens have been fou nd at least as effective and less tox ic as othe r first line regimens. It is included in the preferred I" li ne W HO regimen for adults and adolescents. Renal toxic ity is to be watched. TE VIRAL-L: Tcnofovir 300 mg + Lamivudine 300 mg tab TRIO DAY: Tenofovir 300 mg + Larnivudine 300 mg+ Efavirenz 600 mg tab.

Emtricitabine (FTC) It is a fluorinated cytid ine analogue which is converted intracellularly by cellular kinases into its triphosphate which acts as the HIV reverse transcriptase inhibitor. Emtricitab ine is we ll absorbed orally, little metabolized and largely excreted unchanged by the kidney with a t½ of 10 Im. However, the intracellu lar t½ of the active triphosphate is much longer (-40 hrs), perm itting once daily dosing. Dose reductio n is needed in re nal impairment. Some cross resistance with lamivudine occurs, bu t not w ith other N RTls. Like lami vudine, it is also active against HB V, but should not be used in H IV-HBV coninfected patie nts because sudde n stoppage of therapy can cause rebound exacerbation of hepatitis. Emtric itabine is one of the l " line anti-HIV drugs, and is a component of the prefe1Ted WHO regimen for treating adults and adolescent HI V patients. In combination w ith tenofovir, it is advocated by WHO fo r pre-exposure prophylaxis of HIV in high risk adults. Emtri c itabine is o ne of the least toxic ARV drugs; side effects are fat igue, headache, nausea, diarrhoea, and di scoloration of exposed skin. It is marketed only as fi xed dose combination tablets. Dose: 200 mg OD, as FDC tablets. TENVIR-EM. TAVJN-EM, TENOF-EM: Emtricitabine200 mg + Tenofnvir disoproxil fumarate (TDF) 300 mg tab. TRUSTIVA, VI RA DAY, VONAVIR: Emtricitabine 200 mg + tenofovir (TDF) 300 mg + Efaviranz 600 mg tab.

Non-nucleoside reverse transcriptase inhibitors (NNRTls) Nevirapine (NVP), Efavirenz (EFV) These are nucleoside unre lated compounds which directly inhib it JII V reve rse transcri ptase w ithout the need for intracel lular phos pho rylation. Their locus of action on the enzyme is different fro m that of N RTi s, and they are non -competitive inhibitors. T hey are more potent than AZT on HIV-1 , but do not inh ibit HI V-2. Accordingly, they are not indicated in infect io ns ca used by HIV-2 ( HI V-2 infecti ons a re infrequent and mi lder). If used a lone, vira l resistance to 1 RTl s develops rap idly by poi nt mutation o f the enzyme; they should always be combi ned

863

864

A NTIM ICROBI A L DRU GS

with 2 other effecti ve drugs. C ross-resistance between NVP and EFV is common, but not w ith RTls or P is. A patient failing a regimen contaning NV P should not be treated with EFV a nd vice versa, but ca n be put on etravi rine regi men. P is well absorbed orally; is extensively metabolized, mainly by CY P3A4 and to a lesser extent by CY P2B6, with a t½ of ~ 30 hours. Oral absorption of EFV is - 50% , but the t½ is longer (48 hours). It is complete ly metabolized, mainly by CYP28 6 and a sma ller fraction by CYP3A4. Both are enzyme inducers, and cause autoinduction of their own metabolism. However, EFV inhibits CYP3A4 as we ll. evi rapine is sta rted at a lower dose (200 mg/day); the dose is doubled after 2 weeks when its blood levels go down due to auto induction. Such dose escalation is not required for EFY. Ri fa mpi n induces VP me tabo lism and makes it ineffective, but has litlle e ffe c t on EF V leve ls. If a patie nt being treated w ith VP develops TB and is put on rifampi n, NVP should be replaced by EFY. The RTls are indicated in combina tion regime ns for HIV. Eit he r V P or EFV is incl uded in the fi rst line triple drug regime n used by ACO, but the WHO (2016) regimen prefers EFV and advoca tes PV as an alternative. These drugs have succeeded in reduc ing HI V-RNA levels w hen an earlier regimen (not including an NNRTI) has fa iled.

Nevirapine (NVP) Dose: Initially 200 mg/day, to be increased after 2 weeks to 200 mg twice daily (because autoinduction reduces levels). NEVIMUNE, NEVJVIR. NEVIPA!\ . '-IEVIRETRO 200 mg tab.

Rashes are the commonest adverse effect, followed by nausea and headache. Occasiona lly skin reactions are severe. Fever a nd ri se in transa minases occurs dose dependently. NVP is potenti ally hepatotox ic. 1n patients developi ng VP toxicity, it should be replaced by EFV w hic h has low hepatotoxic ity. NVP should not be used in patients with he patic dysfu nction.

Efavirenz (EFV)

Its side effects are headache, ras hes, di zziness, insomnia and a variety of

ne uropsychi a tric symptoms . However, these sympto ms decrease over time and d iscontinuation rate (due to adverse effect) is low. Gynaecomastia can occur. On the basis of a nimal stud ies, EFV was considered to be teratogeni c, but recent human data have exone rated it; and the WHO (20 16) regimen inc ludes EFV in the preferre d I st line regimen for pregnant and breast- feed ing women. Beca use o f its longer plasma t½, occasional missed doses of EFV are less damaging. Dose: 600 mg OD on empty stomach. EFFERVl::.N. VIKANZ, EVIRE:-.lZ 200 mg cap, 600 mg tab.

Etravirine T his is a second generation NNRTI w hich is active agai nst HJ V-1 mu tants that are resistant to other RTls. Etravirine is indicated in adults and children >6 yr w ho have already been treated and are resista nt/intolerant to other RTls. In one study, a fte r 24 weeks etravirine conta ining regimen, 74% recipients achi eved < 400 copies of HIV-RNA, compa red to 5 1% of p lacebo recipien ts . T he o ra l a bsorpt ion of etravi rinc is e nhanced by food a nd it is completely meta bo lized by CYP3A4, CYP2C9 a nd CYP2C 19 with a plasma t½ of 40 hours. Metabo lites a re excreted in urine. Etrav irine ind uces CY P3A4, but in hibits C YP2C9 a nd 2C 19. As such it interacts with many drugs. Etraviri ne is appro ved fo r use only in combination with a RTI + one of Pis, e.g. darunavir/r or lopinavir/r or saqui navir/r, but not wi th fosamp re navir/r or atazanav ir/r. Adverse e ffects are nausea and ski n rashes, whic h can rarely be very serious. Dose: 200 mg 8D after meals. INTRAVIR 100 mg tab. Delavirdine and Rilpivirine arc the other infrequently.

' RTls used

Retroviral protease inhibitors (Pis) An aspartic protease enzyme encoded by HI V is involved in the production of structural proteins a nd enzymes (i ncl udi ng reverse transc riptase and in tegrase) of the virus from the la rge vira l po lyprote in synthesized in the infected cell. The polyprotei n is broken into va rious fun ctiona l components by thi s protease enzyme. It acts at

AN TI-RETROVIRU S DRUGS

a late step in HIV replication, i.e. maturation of the new virus particles when the R A genome acquires the core proteins and enzymes. Eight protease inhibitors-Atazanavir (ATV), Indinavir (JDV), Nelfinavir (NFV}, Saquinavir (SQV) , Ritonavir (RTV), lopinavir (in combination with ritonavir LPV/r), Fosamprenavir (FPV) and Darunavir (DR V) are available for use against

HIV. They bind to the active site of the protease molecule, interfere with its cleaving function, and are more effective viral inhibitors than AZT. The Pis do not need intracellular activation by phosphorylation. Because they act at a late step of viral cycle, they are effecti ve in both newly as well as chronically infected cells. Under their influence, HIV-infected cells produce immature non infectious viral progeny- hence prevent further rounds of infection. Oral bioavailability of Pls is variable (IDV and RIV -65%, FY >20%, SQV 15%) and their plasma t½ ranges from 2- 8 hours. All are extensively metabolized mainly by CYP3A4, except NFV whic h is mainly a substrate of CYP2C 19. All P is (es pecially ritonavir and lopinavir) are potent inhibitors of CYP3A4, while some other CY P isoenzymes are induced. The Pis interact with many drugs. elfinavi r, lopinavir and ritonavir induce their own metabolism. The Pis are not used as monotherapy, because resistance develops over months due to selection of resistant mutants. Combination of NRTls with Pis is more effective than either drug given alone, and triple therapy is more effective than double therapy. Current recommendation is to use a Pl in combi nation with either two RTls or one JU I + one NNRTI. However, Pls are avoided in 1st line regimens, because their use in initial regimens markedly restricts second line regimen options. Most guidelines, including that of WHO and ACO, reserve them for failure cases. Because different Pis inhibit as well as induce specific CYP isoenzymes to different extents, drug interactions with them are common and often unpredictable. Manufacturer's package inserts

should be consulted while coprescribing any other drug. Specifically, metabolism of Pls is induced by rifampin and other enzyme inducers rendering them ineffective. Another problem in their use is the large tablet load. In case of different Pls, 6-18 tablets are to be taken da ily, some on empty stomach, but others with meals; and this has to go on for months and years. Therefore, patient acceptability and compliance are often low. One of the strategies adopted to reduce the dose of Pis (except that of FV) is to combine them with a low and subtherapeutic dose ( I00 mg) of ritonavir. By reducing fi rst pass metabolism, ritonavir increases the bioavailability and by slowing systemic metabolism decreases clearance of the companion Pl. This 'boosted Pl regimen' permits reduction in the number/frequency of tablets to be taken each day. Nelfinavir is not to be combined with ritonavir because it is metabolized mainly by CYP2C l9 that is not inhibited. All other Pis are mostly administered with low dose RTV. The most pro minent adve rse effects of Pis are gastrointestinal into lerance, asthenia, headache, dizziness, limb and facial tingling, numbness and rashes. or particular concern are lipodystrophy (abdominal obesity, buffalo hump with wasting of limbs and face), dyslipidaemia (raised tr iglycerides and cholesterol) which may necessitate restriction of lipids in diet. Insulin resistance occurs and diabetes may be exacerbated. l ndinavir crystalises in urine and increases risk of urinary calculi. Atazanavir (A TV) This PI is administered with light meal which improves absorption, while acid suppressant drugs decrease its absorption. ATV is metabolized primarily by CYP3A4, which is also moderately inhi bited by it. Bioavailabil ity and efficacy of ATV is improved by combi ning with RTV. The t½ is 6-8 hours. Loose motions, nausea and abdominal pain arc the side effects. Dyslipidaemia and other metabolic complications are minimal wit h ATV, but jaundice occurs in some patients without liver damage due to inhibition of hepatic glucuronyl transferase. Dose: 300 mg OD with ritonavir 100 mg taken at meal time. ATAZOR 100, 150, 200, 300 mg caps.

865

866

AN T IM ICROBIAL DRUGS

lndinavir (IDV) It is to be taken on empty s tomach: g.i. intolerance is common; excess fluid s must be consumed to avoid nephrolithiasis. 1-lyperbilirubinaemia occurs. It is infrequent ly used now.

Dose: 800 mg T DS (BD if taken with 100 mg RTV). INDIVAN, IN DlY IR , VIROD I 400 mg cap.

Nelfinavir (NFV)

It is to be taken with meals, since food increases absorption, but bioavai lability is erratic. NFV is mainly meuibolized by CYP2Cl9. Oflen produces diarrhoea and flatulence; clinical efficacy is lower than other boosted Pis; less popular now. Dose· 750 mg TDS; ' ELFIN. NELV IR, NElVEX 250 mg tab.

Ritona vir (RTV)

It is a po tent Pl ; also a po te nt C YPJA4 inhib ito r. Drug in terac ti o ns, nausea, dia rrhoea , paresthesias, fatigue and lipid abno1111a lities are promine nt. Tho ugh RTV (600 mg twice daily) can be used as an anti re troviral drug. it is more commonly employed in a low dose (100 mg BO) to boost all o ther Pis except FY. Low dose RTV produces no addi tional side effect. RITO 'vt U E, RITOMAX 100 mg cap: RITOVIR 250 mg tab.

Saquinavir (SQV) Its oral bioavai lability is low, the tablet load is large and s ide effects arc frequent; pho1osensi1ivi1y can occur. Importantly, it is a weak inhibitor of CYP3A4. Dose: 1200 mg TDS on full stomach: I 000 mg BD (with RTV 100 mg). SAQUfN 200 mg tab.

Lopinavir Jt is available only in combinatio n wi th RTV to improve bioavailability, thoug h it is itself a C YP3A4 inh ib ito r. Diarrhoea, abdo minal pa in, nausea and dyslipi dae mi as a re mo re co mmo n. ECG abnorma lities have been noted . Its dose needs to be increased by I/3 rd if e it her NV P or EFV is used concurrently. Dose: 400 mg (with ritonavir I 00 mg) BD w ith food. RITOMAX- L: V- LETRA : lopmavir 133.3 mg ... ritona, ir 33.3 mg cap.

Darunavir

It is a potent newer Pl active against both HI V-1 and H IV-2, includ ing severa l strains res istant to o ther Pis. Oral bioava ila bility of darunavir is improved by coadm iniste ri ng ritonavir, whic h retards its Ist pass metabolism as we ll as systemic clearance by inhibiting CY P3A4. It is meta bo lized extensive ly by CY P3A4 and excreted in urine with a t½ of 15 hrs. Rifampin and o ther CYP3A4 inducers render it ineffective. Da runavi r is approved for use only in com bination with ritonavir or another booster

Cobicistat. Side effects are nausea, diarrhoea, rise in hepatic enzymes, rashes and a lle rgic reactions, pa rticularly in those with sul fo namide a lle rgy, but overall to lerability is good. Dose: 600 mg BO along with ritonavir I 00 mg 13D. DA RUVIR 300 mg tab. Fosamprenavir It is a phosphorylatcd prod rug of amprenavir that has better oral bioavailability and better tolerability than the parent drug. As such. it has replaced amprenavir. Fosamprcnavir is active against both HIV- I and HIV-2, and is effective in treatment-naive as \\ ell as previously Pl treated patients. It is extensively metabolized. mainly by CYP3A4 and is a moderate inhibitor o f CY 1'3A4. The plasma t½ is - 8 hours. Side effects arc nausea, diarrhoea, fat igue and rashes, but lipid abnormalities are less mar ked. Dose: 1400 mg twice daily, or 1400 mg once da ily with ritonavir 200 mg daily.

lntegrase inhibitors The I IIV integrase enzyme enters the host cell alongwith the geno mic RNA. After the HIVproviral D A is transcripted in the cytoplasm of host cell , th is enzyme trans locates to the nucleus alongwith the provira l DNA, nicks host chromosoma l DN A, integrates the proviral DNA w ith it and resea ls it. Thus the proviral DNA becomes a part of the chromosomal DNA, and the cell gets permanently in fec ted. Interference with this virus specific function provides an important target for action of ARV drugs.

Raltegravir

I t is the first integrase inhi bito r to beco me availab le fo r treating HIV infectio n. Raltegrav ir prevents the integrati on of proviral DNA into the host chromoso me and thus interrupts the vira l cycle. It is active against both H IV- I and HI V-2, a nd there is no cross resistance with other classes of ARV drugs. Its clinical efficacy in rapidly c learing HI V-R A from circulation and in improving CD4 cell cou nt has been de monstrated in treatmen t-naive as wel I as previo usly treated patients. Ra ltegrav ir is rapid ly a bso rbed orally and extensively glucu ronide conj ugated in liver. The terminal p lasma t ½ is ~ 12 ho urs. Rifamp in induces meta bo lism of ra ltegravi r; its dose needs to be doub led. Admin istrat ion of Ca2 and Mg~• containing antacids and Ca2+ or iron supplement should be avoided with raltegravir

ANTI-RETROVIRUS DRUGS or staggered from it, because of its chelating property. Raltegra vir produces fe w side effects li ke nausea, headache, diarrhoea, a nd weakness; myopathy is a potential toxicity. Dose: 400 mg BD (a long with 2 N RTls). ISENTRESS, ZEPODOM 400 mg tab.

Dolutegravir This is a second gene ration integrase inhibitor, w hic h like ra ltegravi r, is acti ve against both HIV- I and HlV-2, exhibits no cross res istance w ith other classes of ARV drugs. In addition it reta ins some activity against raltegra vir-resistant viruses. Dolutegravir combined w ith 2N RTl s has yielded superior results than raltegravi r-containing regimens. It is absorbed rather slow ly, gluc uronide conjugated and e lim inated w ith a t ½ of 12 hours. The dose of do lutegravir s hould be doubled when given w ith enzyme inducers like rifampin and efavirenz. but its combination w ith nevirapine and etravirine should be avoided . Cationic medication (Ca2 , Mg2+ salts) should be avoided or staggered with dolutegravir. lt has good to lera bility profile w ith rashes and hypersensitivity as the infrequent adverse effects. Hepatotox ic ity is occas ional. Gi ven its potent activ ity, good tolerability, once a day dosing, dolutegravi r alongwith 2 RTls is gaining popularity as a I st line regimen for treatment-naive HIV patie nts, if cost is not a consideration. By do ubling the dose, it is also advocated fo r patients w ho have been treated with raltegravi r, whe n integrase-resistance is suspected. Dose: 50 mg once daily; 50 mg twice dai ly with enzyme

CCRS receptor inhibitor Maraviroc The globular glycoprotein gp 120 of the HI V em elope anchors to the CD4 site of host cell by binding to a cell membrane receptor, which mostly is the CCRS chemokine receptor ( most HI V arc CCR5-tropic). Maraviroc is a novel anti-I-I IV drug which targets the host cell CCRS receptor and blocks it. Attachment of the virus and subsequent entry of viral genome into the cell is thus interfered. It has no effect on HIV strains that are CXCR4 receptor tropic (CXCR4 is an a lternative chemokine receptor which also can bind gp 120). or dual CCR5/CXCR4 tropic. Added to optimized background therapy in patients who have already been treated with several regimens and who have CCR5-tropic HI V infection, maraviroc has resulted in marked reduction in HIV-RNA load. and improvement in C D4 count. It is active orally and there is no cross rt:sistance with any other ARV drug. However, CCR5-tropis m must be proven before using it. Side effects are cough, fever, abdominal pain, rashes and postural hypotension, but tolerability of maraviror in genera l is satisfactory. Some antiretroviral combinations I.

2.

3. 4. 5.

6.

7.

inducers, or if integrase-resistance is suspected. TNSTGRA, NAIVEX 50 mg tab.

Entry (fusion) inhibitor Enfuvirtide This HIV-de rived synthetic peptide acts by binding to HIV-I envelo pe transmembrane glycoprotein (gp4 I) which is involved in fusion of viral and cellular membranes. Fusion of' the two rnembrances is thus prevented and entry of the virus into the cell is blocked. It is not active against H IV-2. No cross resistance w ith other c lasses of ARV drugs occurs. Administered s.c. twice dai ly, it is used as add-on drug to an optimized regimen in selected patients who have failed many earl ier regimens or have multidrug resistant HI V, and for whom there is no other treatment option. The injections are painful and cause local nodules/cysts. The cost and inconvenience are prohibitive.

8.

Tcnofovir disoproxil fumarate 300 mg -'- Emtricitabine 200 mg tab: TENVJR- EM, TAVIN- EM, TE OF-EM tab. Tcnofovir disoproxil fumarate 300 mg + Emtricitabine 200 mg + Efavirenz 600 mg tab: TRUSTIVA , VIRADAY, VONAV IR tab. Tenofovir disoprox il fltmarate 300 mg + Lamivudine 300 mg tab: TENVIR-L tab. Tenofov ir disoprox il fumurale 300 mg + Lamivudine 300 mg + Efavirenz 600 mg tab: TRIODAY tab. Lamivudine 150 mg + Zidovudine 300 mg tab ( I tab BD); COMBI\IIR, CYTOCOM , DUOVIR, LAMUZID tab. Lamivudine 150 mg + Zidovudinc 300 mg + Nevirapine 200 mg tab (I tab BD); DUOVIR-N, CYTOCOM-N, NEXIVIR-Z. Lamivudine 150 mg + Stavudine 30 mg + Nevi rapine 200 mg tab (I tab BD); LAMOSTAD-N . TROMUNE, VlROLANS. Lamivudine 150 mg + Zidovudine 300 mg 2 tab and Efavirenz 600 mg I tab kit; CYTOCOM-E kit.

HIV TREATMENT PRINCIPLES AND GUIDELINES T he treatment of HIV infection and its complications is comp lex, li fe- long, needs expertise, strong motivation and commitment of the patient, resources and is expensive. Antiretroviral thera py (ART) is only 30 years old, and is still evolving. In itia lly, anti-HI V drugs were used singly o ne a fter the other as each fa iled in a patient

867

868

ANTIMICROBIAL DRUGS

due to emergence of resistance. Understanding the biology of HI V infecti on and availabi lity of several potent drugs belonging to diITerent classes has mandated ' highly active antiretroviral therapy ' (HAA RT) with combination of 3 or more drugs w hen ever indicated. Monotherapy is contraindicated. It has been realized that even with 3 drug ART, which rapidly kills > 99% virions, a small number survive within the resting CD4 lymphocytes and invariably give rise to relapse when treatment is discontinued despite complete absence ofdetectable viraem ia and nom1al CD4 cell count for years. Relapses occur even i fthe sameA RT is continued afler disappearance ofviraemia and immune reconstitution. l11is is because HIV-reverse transcriptase and other enqmes are highly copying error prone, implying that viral replication produces changes at some base pairs (and codons) with high frequency-rate of mutation is high. Some mutations confer resistance to one or the other amiretroviral drugs. The resistant mutants are selected by ART and in time an apparently sensitive population is replaced by resistant virions. As the disease progresses in the individual (and several ARV regimens are used) the HI V population becomes genetically complex and diverse with respect to susceptibility to drugs. Each fai ling regimen limits future treatment options. Even primary drug resistance (i.e. in w1treatcd patients) is being detected in 5- 20% 1 -nv patients. In developed countries, drug resistance studies are being recommended before initiating ART.

Since none or the c urrently ava ilable regimens can eradicate II IV from the body of the patient, the goal of therapy is to max imall y and durably inhibit viral replication so that the patient can attain and maintain effective immune response towards potential microbial pathogens. Greater the suppres ion of viral replication, lesser is the chance or eme rgence of drug resistant virus . Effecti ve A RT prevents exual transmission of HIV Therapy in the infected partner nearly e liminates risk of transmission to the uninfec ted partner. It also reduces infec tivity of the patient for other persons, thu s serves to limit transmission.

Initiating antiretroviral therapy

Till recently, various professiona l bodies and health organi za tions like WHO and ACO recommended delaying start of ART in asymptomatic HJV infected subjects untill the CD4 count fell to 95% plasmodia. They leave only a small biomass of the paras ites to be e li minated by the lon g t½ drug, reduc in g the c hanc e s of selecting resistant mutants. Advantages of ACT over other antimalarials a re: • Rapid clinical a nd parasito logical cure. • High c ure rates (>95%) and low recrudescence rate. • Absence of parasite resistance (the components prevent development of resistance to each other). • Good tolerability profi le. T he ACT regimens fo r oral treatment of unco mplicated falci parum ma laria that are already in use in Ind ia, or are WHO approved, a re g iven in the box on p. 890. Oral ACTs are not to be used in severe or compli cated malaria, for which parenteral drugs are needed.

1. Artemether-lumefantrine

Lumefantrine is an orally active, h igh e fficacy, long-acting crythrocytic schizontocide, re lated chemi cally and in mechanism of action to ha lofantrine and MQ. Like the other schi zontocides, vivax hypnozoites are not affected. Lume fantrine is highly lipophilic; absorption starts after 2 hours of ingestion and peaks at 6-8 hours. Antima laria l action is slower than CQ. Plasma protein binding is 99%, and it is metabolized predominantly by CYP3A4. Terminal t½ is 2- 3 days, which is prolonged to 4 6 days in malaria patients. Lumefantrine is used only in combi nation with artemethcr, as FDC tablets. The two components protect each other from plasmodial resistance. As such, no c linically relevant resistance has developed so far. C linical e fficacy is high achieving 95- 99% cure rate. Artemether-lumefantrine is act ive even in mu ltidrug resistant Pf areas inc luding MQ-resistant. It has been extensive ly employed in Southeast Asia and Africa and is now popular in India. The VBDC P uses this ACT for falciparum malaria in the northeastern

889

890

ANTIMICROBIAL DRUG S

ACT regimens for uncomplicated falciparum malaria* 1. Artemether-lumefantrine ( 1:6f s Artemether (80 mg BD) + lumefantrine (480 mg BD) x 3 days COARTEM, COMBITHER, LUMETHER (artemether 20 mg + lumefantrine 120 mg tab.) to be taken with fatty meal. Adult and child >35 kg 4 tab BD; child 25-35 kg 3 tab BD; 15- 25 kg 2 tab BD; 5-15 kg 1 tab BD, all for 3 days. FALCI MAX PLUS, ARTE PLUS (artemether 80 mg + lumefantnne 480 mg tab) 1 tab BD x 3 days for adults.

2. Artesunate-mefloquine (AS/MQf·s Artesunate 100 mg BD (4 mg/kg/day) x 3 days+ mefloquine 750 mg (15 mg/kg) on 2nd day and 500 mg (10 mg/kg) on 3rd day (total 25 mg/kg). MEFLIAM PLUS: Artesunate 100 mg+ mefloquine 200 mg pack of 6 FDC tabs; LARINATE-MF Kit: Artesunate 200 mg (3 tabs)+ mefloquine 250 mg (6 tabs) kit; FALCIGO PLUS kit (Artesunate 50 mg tab+ Mefloquine 250 mg tab kit) 3. Artesunate-amodiaquine (AS/AO.JS Artesunate 200 mg (4 mg/kg)+ amodiaquine 600 mg (10 mg/kg) per day x 3 days Artesunate 25 mg/50 mg/ 100 mg +Amodiaquine 67.5 mg/135 mg/270 mg fixed dose combination tablets; ASAQ, FALCINIL-Aq: 25 + 67.5 mg, 50 + 135 mg, 100 + 270 mg FDC tabs. 4. Artesunate-sulfadoxine + pyrimethamine (AS-S/Pf s Artesunate 100 mg BD (4 mg/kg/day) x 3 days + sulfadoxine 1500 mg (25 mg/kg) and pyrimethamine 75 mg (1.25 mg/kg) single dose. ZESUNATE kit, MASUNATE kit, FALCIART kit (Artesunate 100 mg x 6 tab+ sulfadoxine 500 mg/pyrimetham,ne 25 mg x 3 tab kit) 5 . Dihydroartemisinin-piperaquine (DHAIPPQ 1:BY s DHA 120 mg (2 mg/kg) + piperaquine 960 mg (16 mg/kg) daily x 3 days; for children < 25 kg body weight, DHA not less than 2.5 mg/kg+ piperaquine 20 mg/kg daily x 3 days. PALUDOSE PLUS: Dihydroartemisinin 40 mg+ piperaquine 320 mg tab.; DYSURE: DHA 80 mg+ piperaquine 640 mg per 5 ml suspension. 6. Arterolane-piperaquine Arterolane (as maleate) 150 mg + piperaquine 750 mg daily x 3 days (approved only for adults). SYNRIAM (arterolane 150 mg + piperaquine 750 mg) cap, 1 cap OD x 3 days ' All drugs are administered orally r wHO approved ACTs; s Approved in India

region. Whi le artemether quickly reduces parasite biomass and resolves symptoms, lumefantri ne prevents recrudescence. Gametocyte population is reduced. checking transmi ssion. Artemether-lumefantrine must be adm inistered w ith fatty food or milk, which markedly enhances lumefantrine (and to some extent artemether) absorption. and ensures adequate blood levels. Fai lure to take it with fa t rich food limits absorption and may result in recrudescence. This ACT is generally well tolerated; side efTects arc- headache, dizzi ness, sleep disturbances, abdom ina l pain, arthralgia, myalgia, pruritus and rash. Comparative studies indicate that it is bette r tol e rated tha n AS/ MQ. Artemctherlumefantrine should not be given w ith drugs

metabolized by C YP2D6 (metoprolo l, neuroleptics, tricyclic a ntidepressants, etc.), because lumefantrine inhibits the isoenzyme CYP2D6. Lumefantrine causes minimal prolongation of QTc. Since anemether can also prolong Q-Tc to some extent, artemether-lumefantrine ACT should not be given to patients receiving Q-Tc prolonging drugs. It is to be avoided in fi rst trimester of pregnancy and during breastfeeding.

2. Artesunate-mefloquine (AS/MO)

This was the most extensively used ACT in Thailand, Myanmar and other Southeast Asian countries as we ll as in South-America and Africa; and was found highly effective in uncomplicated falciparum malaria. Howeve r, many areas in far-East already have MQ-res istant Pf, but

A NT IM ALAR IAL DRUGS

by combining w ith AS, further spread of MQ resistance was checked. evertheless, some of the fa r- East countri es have switched over to a lternative ACTs. T he adverse effect potential o f AS/MQ is less favourable than other ACTs. T herefore, it is less popular now. In Ind ia AS/ MQ has been used lo a limited extent, but sma ll studies have shown - I00% efficacy. A kit with separate AS and MQ tablets as well as a FDC form ulation is now ava ilable (see box on p. 890). Side efTects of MQ are more troublesome, and need to be watched for.

3. Artesunate-amodiaquine(AS/AQ) Amodiaquine (AQ; see p. 880) has long been used pa ra lle l to CQ. Whil e AQ itself has a sho rt t½ due to rapid metabolism, its metabolite, an equa lly potent anti malaria l has long t½ of I 0- 18 days. Because of c lose structural resemblance of AQ to CQ, it was apprehended that AQ may not be an effective antima larial in areas with CQ-resistant Pf However, trials in Africa showed that AQ produced satisfactory response in such areas. Addition of artesunate further improved the cure rate. Trials were conducted in Africa with AS/AQ coformulated as FDC tablets, which produced high c ure rates, and now this ACT has become the first- line therapy of uncomplicated falciparum malaria in many A fr ican co untries. Recent tria l in India a lso yie lded - 97% cure of falciparum ma laria. This ACT has now been marketed in India as FDC tablets in 3 strengths for di ITcrent age groups (see box on p. 890), to be taken twice daily for 3 day treatmem of uncompl icated falc ipa rum ma laria. 4. Artesunate-sulfadoxine + pyrimethamine (AS-S/P) This ACT has been adopted as the first li ne treatment for uncompli cated falc iparu m ma lar ia in Ind ia, except in the north-eastern states bordering Myanmar, from w here S/P resista nce has been reported. Th is does not imply that AS-S/P is the best ACT. because it is not effecti ve agai nst multidrugres istant strains whic h are nonresponsivc lo S/P. However, in rest of I ndia so far this ACT a ppears to be worki ng satisfactorily w ith >96%

success rate. As s uc h , NV B DC P c on t inues to use AS-S/P ACT as the fi rstline therapy, inc luding that during 2nd and 3 rd trimester of pregnancy. T his ACT appears to produce fewer side effects than artesunate/mefloquine. Private c li nics, however, are using a lternative ACTs. Some othe r malaria endem ic count ries have fou nd AS-S/P ACT to be inferi or.

5. Dihydroartemisinin (DHA)-piperaquine Piperaqu ine is a bisqu inolinc congener of CQ developed in C hi na as a hig h efficacy lo ng acti ng (t½ 3-4 weeks) erythrocytic schizontocide with a slower onset of action because of larger volume of distribution. The mecha ni sm of action is similar to CQ. It is more active agai nst CQ-resistant P falciparum and retains equa l acti v ity aga inst CQ sensitive orga nisms. The efflux transporters of the resistant parasite are believed to be unable to pump out the bulky molecule of piperaquine. In 1978, piperaquine replaced CQ in C hina, where it has been extensively used for mass prophylax is as well as treatment of ma laria. P iperaq uine is coformulated w ith DH A in a dose ratio of 8: I and extensively eval uated in multidrug resistant Pf areas of Cambodia, Thailand, Vietnam, etc. with high success rate. In c linical trials, efficacy of DH A-pi peraquinc FDC has been found comparable to artemetherlumefantrine. Safety profile of DHA-piperaquine is good and it is we ll tolerated even by children. However, dizziness, vomiting and other g.i. symptoms are common; rashes arc rare. T he DII A-piperaquine FDC has achieved >98% response rate in clinical trials on uncomplicated fa lciparu m malaria patients in Indi a, and has been approved. WHO (2015) has recommended a higher pe r kg dose fo r ch ild ren weigh ing < 25 kg.

6. Arterolane-piperaquine

Artcrolane is a novel orally active synthetic trioxolane congener of a11cmisinin that has been developed in India and marketed in combination wi th piperaquine. Arterolane acts rapidly at all stages of asexual schi:wgony of ma laria l parasite including

891

892

ANTIMICROBIAL DRUGS

mul tidrug resistant Pf , but has no effect on the hepatic stages. It accumulates in the food vacuole of the parasite, and thus differs from artemisinins which do not accumulate at this site. It also has moderate gametocidal activiry similar to that of artemether-lumefantrine. Both arterolane and piperaquine are wel l absorbed orally, and absorption is unaffected by food. Peak plasma arterolane concentration is reached in 3- 5 hours and it has a large volume of distribution. T he major metabolic pathway is oxidation, mainly by CYP3A4, which is also the primary isoenzyme responsible for piperaquine metabolism. Anerolane is short acting and its plasma t½ varies between 1- 3 hours. Artero lane-piperaquine FDC ha undergone multicentric clinical trials in India, Bangladesh and T hailand. In uncompl icated falciparum 95% cure mal aria this A CT has produced rate wi th a fever and parasitaemia clearance time of 24-48 hours. In a comparative trial its

efficacy and tolerability was found equivalent to artemetber- lumefantr ine. Side effects are genera lly mild headache, postural dizziness, vomi ting, abdominal pain and diarrhoea. Thus, arterolane-piperaquine appears to be an effective and well tolerated alternative ACT. 7. Artesunate-pyronaridine Pyronaridine is a watersoluble Mannich base resembling strncturally Lo amodiaquine. ll is an erythrocyt ic schi,:onlocide w ilh high ci'ficacy and mechanism of action similar to CQ. It has been used in China for - 40 years, and is active against both CQ-sen itive and CQ-resistant P. falciparum. Other ma larial species are also sensiti,·e. The onset of action i slower and duration long. It is concentrated in RBCs and metabolized with a terminal t½ of 7 days. Weak analgcsic-antipyrelic action is produced al higher doses. Clinica l efficacy of anc unale-pyronaridinc FDC (dose ratio I :3) has been lestcd in fa lciparum ma laria in China, Thai land and Africa wi th > 95% success. Multid rugresistant P. falciparum and P. vivax also respond. Clinical trials have been completed in India with > 95% cure rate. However. this ACT has nol yet been approved for use in India.

r::r PROBLEM DIRECTED STUDY

61.1 A 20-year-old girl reported to the district hospital OPD with irregular episodes of high fever for the pa st 3 days. Th e fever is preceded by chills and shivering and attended by headache, body ache, pain in abdomen, nausea and weakness. The fever lasts 4- 6 hours and subsides after sweating. On enquiry she informed that she belongs to a vi llage in th e tribal area of Madhya Pradesh. About a month back she had returned from her home after a 3 weeks vacation and she works as a house maid in the city. Blood smear examination showed presence of intraerythrocytic P. vivax parasites. She was treated with the standard 1.5 g chloroquine (ba se) course over 3 days, and was given primaquine 15 mg tab to be taken once daily for 14 days, after she tested negative for G-6PD deficiency. She was afebri le on the 4th day, but returned back 7 days later with similar episode of chills and fever. Finger prick blood smear was positive for P. vivax. She confirmed continuing to take daily primaquine medication. (a) What is the most likely cause of recurrence of fever and parasitaemia? (b) How should the 2nd episode of fever be treated? (c) Should primaquine medication be continued or stopped ? (see Appendix-1 for solution)

Chapter

62

Antiamoebic and Other Antiprotozoal Drugs

ANTIAMOEB IC DRUGS These are drugs useful in infection caused by the anaerobic protozoa E111a111oeba histoly tica. Amoebiasis has a worldwide di stributi on (over 50 million people uffer invasive disease, but many more harbour it). It is endemic in most parts of Ind ia (about 15% people are infected) and other developing countries. Poor environmental sanitation and low socio-economic status are important factors in the spread of the disease, wh ich occurs by faecal contamination of food and water. Amoebic cysts reaching

INTESTINAL LUMEN~

r.1 .

the intestine tra nsform into trophozoites which either live on the su rface of colonic mucosa as commensals-form cysts that pass into the stools (luminal cycle) and serve to propagate the disease, or invade the mucosa- form amoebic ulcers ( Fig. 62. 1) and cause acute dysente ry (with blood and mucus in stools) or chronic in testinal amoeb iasis (with vague abdom inal symptoms, amoeboma). Occasionall y the trophozoites pass into the bloodstream, reach the li ver via porta l vei n and cause amoebic liver abscess. Other organs like lung, spleen, kidney and brain are rare ly

0 --+ W

~ 1 ) \_ L Um ina 1eye~

- - -- -

0 C

- - + FAECES (Cyst passer)

T

BLOOD STREAM ULCER (Dysentery) Fig. 62.1: The luminal cycle and invasive forms of amoebiasis. T-trophozoite; C-cyst

894

ANTIMICROB IAL DRUGS

ANTIAMOEBIC DRUGS

I e • e

INI I



"'

I For Intestinal + extralntestlnal amoeblasis

For extraIntestinal amoeblasls only C hloroquine

.,-.

• [ ;.I t

.. I

I

IAntibiotics

! Amides Diloxanidc furoate Nitazoxanide

Tetracycline Paromomycin

I 8-Hydroxyqulnollnes INitrolmldazoles Metronidazole Tinidaz.ole Secnidazole Ornidazo le Satranidazole

Quiniod ochlor (lodochlo ro hydroxyquin) Diiodohydroxyquin (lodoquinol)

Alkaloid Eme tine Dehydroemetine

involved in extraintestinal amoebiasis. In the tissues, only trophozoites are present; cyst formation does not occur. Tissue phase is always secondary to intestinal amoebiasis, which may be asymptomatic. In fact, most chronic cyst passers are asymptomatic. In the colonic lumen, the Entamoebae live in symbiotic relationship with bacteria, and a reduction in colonic bacteria red uces the amoebic population. The • Braz iI root' or Cephaelis ipecacua nha was used for the treatment o f dysentery in the 17th century. The pure alkaloid emctinc obtained fro m it was fo und lo be a potent antiamoebic in 1912. Emeline remained the most efficacious and commonly used drug for amocbiasis till 1960. Many 8-hydroxyq11inoli11es (quiniodochlor. elc.) became ve ry popular drugs for diarrhoea and amoebi c dysentery, but have come under a cloud since lhey were held respo nsible for causing epidemics of 'Subacute myelooptic neuropathy' (SMON) in Japan in 1970s. Soon after its triumph as an anc imalaria l in 1948, c hforoq11i11e was found to be an effecti, e and safe drug for hepacic amoebiasis. Diloxan ide furoatc \\3S a useful addition in 1960, covering mainly chronic intestinal forms o f the disease. However, the most remarkable developrncm was the demonstration of antiamoebic property of 111e1ro11idazole in the early I 960s. This drug had been introduced a few years back as a well tolerated, o rally effective agent for Lrichomona vagini11s. Of the many congeners of metronidaLole that were tested, tinidazole has emerged in the 1970s as a good alternative, and others have been added subsequently.

NITROIMIDAZOLES Metronidazole It is the prototype nitroimidazo le introduced in 1959 for trichomoniasis and later found to be a highly active amoebicide. It has broadspectrum cidal activity against anaerobic protozoa, including Giardia lamblia in addition to the above two. Many anaerobic and microaerophilic bacteria, such as Bae/. fragilis, Fusobacterium,

Clostridium petfringens, Cl. d(lficile, flelicohactel' pylori, Campylobaclet; peptococci , spirochetes and anaerobic Stl'eptococci are sensitive. Metronidazolc does not affect aerobic bacteria. Clinically significant resistance has not developed among E. histolytica, but decreased responsiveness or T. vagina/is has been observed in some areas. Anaerobic bacteria and G. lamblia a lso can develop metronidazole resistance, but this is a clinical problem onl y in the case or

H. py lori. Metronidazole is selectively tox ic to anaerobic and microaerophi lic microorganisms. After entering the cell by d iffusion, its nitro group is reduced by certain redox proteins operative only in anaerobic microbes to a highly reactive nitro radical which exerts cytotoxicity. The nitro radical

AN TI AMO EB IC AND OTHER ANTIPROTOZOAL DR UGS of metronidazole acts as an e lectron sink which competes with the biological e lectron acceptors of the anaerobic organism for the electrons generated by the pyruvate : ferredoxin oxidoreductase (PFOR) enzyme pathway of pyruvatc oxidati on. The energy metabolism of anaerobes that have no mitochondria is thus, disrupted. Aerobic environment attenuates cytotoxicity of metronidazole by inhibiting its reductive acti vation. Moreover, 0 2 competes with the nitro radical of metronidazole for the free e lectro ns generated during energy metabolism of anaerobes. Anaerobes which deve lop metronidazole resistance become deficient in the mechanism that generates the reactive nitro radical from it or have lower levels of PFOR. Mctronidazolc, in addition, ha;, been found to inhibit cell mediated immunity, to induce mutagenesis and to cause radioscnsiti7.ation.

Pharmacokinetics

Metro nidazolc is almost co mpl etely absorbed from the sma ll intestines; little unabsorbed drug reaches the colon. IL is wi dely distributed in the body, atta ining therapeutic concentration in vaginal secretion, semen, saliva and CS F. Metabol ism occurs in li ver primarily by ox idati on and glucuronide conj ugati on followed by renal excretion. Plasma t½ is 8 hrs.

Adverse effects

S ide effects of metronidazole are relatively freque nt and unpleasant. but mostly nonserious. • Anorexia, nausea, metallic taste and abdominal cramps arc the most common. Looseness of stool is occasional. • Less frequent side effects are- headache, glossitis, dryness of mouth and impairment of concentration. • Urti cari a, flushing, heat, itching, rashes and fixed drug eruption occur in a llergic subj ects, warrant discontinuation of the drug and preclude future use of nitroimidazoles. • Prolonged administration may cause peripheral neuropathy and CNS effects. Seizures have followed very high doses. Leucopenia is likel y with repeated courses. • Thrombophlebitis o f the injected vein occurs if the solution is not we ll diluted.

Contraindications Metronidazole is contraindicated in ne urological disease. blood dyscrasias, first trimester of pregnancy (though no teratogenic effect has yet been demonstrated, its mutagenic potential warrants caution). Cautious use in chronic alcoholics.

Interactions A disulfiram-like intolerance to alcoho l occurs in some patients taki ng metronidazole. Alcohol-metronida,m le interaction occurs only in some individuals, while majority of those taking it can con~ume alcohol without any reaction.

Enzyme inducers (phenobarbitone, ri fampi n) may redu ce its therapeutic effecl. Cimetidine can reduce metronidazole metabolism: its dose may need to be decreased. Metronidazole en hances warfa rin action by inhibiting its metabolism. It can decrease renal e limination of lithium and precipitate toxicity. FLAGYL, METROGYL, METRON, ARISTOGY L 200, 400 rng tab, 200 mg/5 ml susp. (as metronidazolc: tastele~~); 500 mg/ 100 ml i.\ . infusion; U IME/,OL 200,400 mg tab~, 200 mg/5 ml sus1i. METROGYLGEL, LU PIGY L GE L: 1% gel for vagmal/topical use.

Uses

1. Amoebiasis: Metronidazole is a first line drug for all forms of amoebic infection. Many dosage regime ns have been tried; the c urrent recommendations are: For invasive dysentery and liver abscess-800 mg TDS (children 30- 50 mg/kg/day) for 7- 10 days. In severe cases of amoebic dysentery or Ii ver abscess 500 mg (chi ldren I 0-15 mg/kg) may be infused i.v. slowly every 6-8 hours for 7- 10 days or till oral therapy can be instituted. For mild in testinal disease-400 mg TDS for 5-7 days. Metronidazole is less effective than many luminal amoebicides in eradicating amoebic cysts from the colon, because it is nearly completely absorbed from the upper bowel. 2. Giardiasis It is highly effective in a dose of 400 mg TDS for 7 days. A shorter course of 3 days with 2 g/day is equally effecti ve. 3. Trichomonas vaginitis It is the drug o f choice; 2.0 g si ngle dose is preferred.

895

896

ANTIM ICROB IAL DRUGS Alternatively 400 mg BD- TDS may be used for 7 days. Additional intravaginal treatment is needed only in refractory cases. Repeated courses may be necessary in some patients, but should be given with gaps o f 4-6 weeks. T he male partner should be treated concurrently in cases o f recurrent infections.

Nonspecific bacterial vaginosis also responds.

2- 3 days; but treatment must be continued for at least 5 days. 7. Helicobacter pylori gastritis/peptic ulcer (see p. 705- 06) Metronidazole 400 mg TDS or tinidazole 500 mg 8D are combined with amoxicill in/clarithromycin and a proton pump inhibitor in trip le drug regimens to eradicate H. pylori.

4. Anaerobic bacterial infections Anaerobic bacterial infecti ons occur mostly after colorectal or pel vic surgery, appendicectomy, etc. Brain abscesses and endocard itis may be caused by anaerobic organisms. Metronidazole is an effective drug for these a nd is generally used in combination with gentamicin or cephalospori ns (because many are mixed infections). For serious cases i.v. administration is recommended: 15 mg/ kg infused over I hr followed by 7.5 mg/kg every 6 hrs till oral therapy can be instituted with 400- 800 mg TDS. Prophylactic use in high risk s ituations (colorectal/bi liary surge ry) is recommended. Other drugs e ITective in anaerobic infections are clindamyc in and chloramphenicol.

Tinidazole H is an equa lly eflicacious congener of metronidazole, si milar to it in every way except: • Metabolism is slower; t½ is ~ 12 hr; duration of action is longer; dosage sc hedules a re simpler. Thus, it is more suited for si ngle dose or once daily therapy. • Some comparative trials in amoebiasis have reported higher c ure rates. • Generally better tolerated; the incidence of side effects is lower: metallic taste (2%), nausea (1 % ), rash (0.2%).

5 . Pseudomembranous enterocolitis caused by Cl. difficile is generally associated with use of antibiotics. Oral metronidazole 400-800

Intestinal amoebiasis: 2 g OD oral fo r 3-6 days (children 30-50 mg/kg/day), or 0.6 g BO for 5- 10 days.

mg BD-TDS fo r I 0- 14 days is more effective, more conven ient, less toxic, and the refore preferred over vancomycin whic h may be used in nonresponsive cases, or when the infection recurs.

Amoebic liver abscess: The 2 g oral daily

6. Acute necrotizing ulcerative ging1v1tis (ANUG) Metronidazole/ ti nidazole are

Trichomoniasis and giardiasis: 2 g si ngle oral dose or 0.6 g OD for 7 days.

the drugs of choice for ANUG (a lso called ' tre nc h mou th ') which is caused by mixed fiora of anaerobes like fusobacteria, spirochetes and bacteroides. Metronida zo le 200- 400 mg T DS ( 15- 30 mg/ kg/ day) is often combined wi th amoxicillin, tetracycline or erythromycin . The respo nse is rapid with di sappearance of the spirochetc-fusobacterium complex from the lesions and resolution of pain, bleeding, ulceration and bad breath within

Anaerobic bacterial infections: prophylactic-2 g single oral dose before

T IN IRA 300,500, 1000 mg tabs; 800 mg/400 ml i., . infusion; TRIDAZOLE 300,500 mg tab: l'AS IGY'\ 0.5 g and I g tab.. T l I 0.3 g. 0.5 g, 1.0 g tabs. 75 mg/5 ml and 150 mg/5 ml oral susp.

Recommended schedules a re-

dose may be continued for 3-6 days. In case oral therapy is not possible, tinidazole 800 mg in 400 ml may be infused i.v. over 40-60 min dai ly until oral treatment can be instituted.

colorectal/biliary surgery;

therapeutic-2 g followed by 0.5 g BD ora l for 5 days. l n severe cases, i. v. tin idazole may be used as described above.

pylori: 500 mg 80 oral for 1- 2 weeks in triple combination.

fl.

Secnidazole

A conge ne r of metronidazo le with the same spectrum of activity a nd

ANTIAMOEB IC AND OTHER ANTIPROTOZOAL DRUGS potency. Absorption after ora l administration is rapid and complete, but metabolism is slower resulting in a plasma t ½ of 17- 29 hours. After 48 hr of a sing le 2 g dose, plasma secnidazole concentratio n may still remain within the range of M IC val ues aga inst sensitive organis ms. In intestinal amoeb iasis a sing le 2 g dose has been found to yie ld high cure rates. Side e ffect profile is similar to metron idazole and reported incidence is 2- 10%.

60 mg OD. It is a local irrirant and has high systemic toxicity, viz, nausea, vomiting (due to CTZ stimulation and gastric irritation), abdomina l cramps, diarrhoea, weakness, stiffness of muscles, myositis, hypotens ion, ECG changes and myocarditis.

Dose: 2 g single dose (children 30 mg/kg) for mild intestinal amoebiasis, giardiasis. trichomonas vaginitis and nonspecific bacterial vaginosis. For acute amoebic dysentery 0.5 g TDS for 5 days is recommended. SEC!\IL, SECZOL 0.5. 1.0 g tabs; 1\OA~IEBA-DS 1.0 g tab.

Dehydroemetine II is equally effective bu less cumulative and less toxic to the heart Thus, it is usually preferred over emetine. Dose: 60-100 mg s.cJ i.m.OD for not more than 10 days. DEIIYDROCMETINE HCI: 30 mg,ml inJ. I and 2 ml amps.

Ornidazole

Chloroquine The pharmacology of chloroquine is described in Ch. 6 1. It kills trophozoites of £. histolytica and is highly concentrated in liver. Therefore, it is used in hepatic amoebiasis only. Because it is completely absorbed from the upper intestine and not so highly concentrated in the intestina l wa ll- it is neither effective in invasive dysentery nor in controlling the luminal cycle (cyst passers). Efficacy of chloroquine in amoebic liver absce ss approaches that of emetine, but duration o f treat me nt is longer and relapses are relatively more frequent, but amoebae do not develop resistance to chloroquine. A luminal amoebicide must always be gi,·en with or after chloroquine to abolish the luminal cycle. Dose for amoebic liver abscess: 600 mg (base) for two days followed by 300 mg daily for 2- 3 weeks. Though chloroquine is relatively safe, side effects are freq uent. The 2-3 week course is poorly tolerated. In amoeb ic liver abscess, chloroquine is generally employed following a course of emetine or as an a lternative to it. In case of bigger abscess, it may also be used after a course of mctronidazole.

It has activity similar to metronidazole, but it is slowly me tabolized- has longer t½ ( 12-14 hr). Dose and durat ion of regimens fo r amoeb iasis, giardiasis, trichomoniasis, anaerobic infections and bacterial vaginosis resemble th ose for tinidazole. In c hronic intestinal amoebias is and asympto mati c cyst passers 0.5 tw ice dai ly for 5 to 7 days has a lso been used. Side effect profile is similar to tinidazole. DAZOLIC 500 mg tab, 500 mg/100 ml vial for i., . infusion. ORI\IDA 500 mg tab. 125 mg/5 ml susp.

Satranidazole Another nitroirnidazole having lo nger t½ ( 14 hr). Ad vantages claimed are: better to lerab ility-m inimal nau sea, vomiting or meta llic taste , absence of neurological and dis ul firam-like reactions and that it does not produce the acetamide metabo lite which is a weak carcinogen. Dose: Amoebiasis 300 mg BD for 3-5 days, giardiasis and trichomoniasis 600 mg single dose. SATROGYL300 mg tab. Emetine It is an alkaloid from Cephacl,s ipecacuanlw. Emctine is a potent and directly acting amoebicidc- kills trophozoites but has no effect on cysts. It acts by inhibiting protein synthesis in amoebae by arresting intraribosomal translocation of tRNA-amino acid complex. In acute dysentery the stool is rapidly cleared of the 1rophozo i1es and symptomatic relie f occurs in 1- 3 days (even faster than mctronidazole), but ii is 1101 curative in the sense that the patient continues to pass cysts in the stool. II is highly efficacious in amoebic liver abscess also. Emeline cannot be g iven oral ly because it wi ll be vomited out. It is administered by s.c. or i.m. injection:

897

Use Emeline is now rarely used for acute amoebic dysentery or for amoebic liver abscess, only in patients not tolerating metronidazole. A luminal amoebicide must always follow emctine 10 eradicate the cyst form ing tropho,wites. It is also effective in liver fl uke in festation. EMCTl"JE HC'I: 60 mg /2 ml inj; for not more than 10 day, to a,01d cumulau,c 10=-icity.

Diloxanide furoate It is a hig hl y effective luminal amoebicide wh ic h directly kill s tropho zoi tes res ponsible for producti on of cysts. The furoate ester is hydrolysed in intestine and the released d iloxanide is la rgely absorbed. Diloxanide as s uch is a weaker arnocbicide th an its furoate ester. No systemic antiamoebic activity is evident despite its absorption. lt is primarily metabo lized by g lucuronidation a nd is excreted in urine. Diloxan ide fu roate exe rts no antibacteri al action. It is less effective in invasive amoebic dysentery, because of poor tissue amoeb icidal acti o n. However, a single course prod uces

898

ANTIM ICRO BIAL DRUGS

high (80- 90%) c ure rate in mild intestin al amoebiasis and in asymptomatic cyst passers.

Giardia, Trichomonas, some fungi (dermatophytes, Candida) and some bacteria. They kill the cyst

Dose: 500 mg TDS for 5- 10 days; children 20 mg/kg/day. FURAMIDE,AMICLINE 0.5 g tab: in Tl'JIBA-DF diloxanide furoate 250 mg • 150 mg tinidazole and TINIRA-DF FORTE 500 mg + 300 mg tabs; in ENTAM IZOLE 250 mg + 200 mg 111etronida70lc and E, TA 'v1 IZOLE FORTE 500 mg+ 400 mg tabs.

forming amoeb ic trophozoites in the intestine, but do not have tissue amoebicidal action. Like d iloxanide furoatc, they are not very effective in acute amoebic dysentery but afford relief in chronic intestinal amoebiasis. The ir efficacy to eradicate cysts from asymptomatic carriers is rated lower than that of diloxan ide furoate. They are totally valueless in extraintestinal amoebiasis. Absorption o f 8-hydroxyquinol ines from the intestine is variable. The absorbed fraction is conjugated in liver with glucuronic acid and sulfate and excreted in urine; t½ is ~ 12 hours. Therapeutic concentrations a re not attained in the intestinal wa ll or in liver. The unabsorbed part reaches lower bowel and acts on luminal cycle of amoebae. Being inexpensive, these drugs were w ide ly and injudiciously used for the prophylaxis and treatment of nonspecific diarrhoeas, traveller's di a rrhoea, dietary ind iscretion, etc .. but are infrequently prescribed now, except in some poor localities. 8-Hydroxyqu ino lines produce few side effects- nausea, tran sient loose and gree n stools, pruritus, etc. but carry toxic potential if improperly used. lodi sm (furuncu losis, inflamma ti on of mucous membranes) and goiter may develop o n prolonged intake. Indi vid uals sensi tive to iodine may experience acute reaction with c hills, fe ver, angioedema and cutaneous haemorrhages.

Diloxanide furoate is very well tolerated; the only side effects are fl atulence, occasiona l nausea, itching and rarely urti ca ria. It is a pre fe rred drug for mild intestinal and asym ptomatic amoebiasis, and is given after or along with any tissue amoebicide to eradicate cysts. Combined use with metronidazole or tinidazole is quite popular. Some chro nic cases require repeat courses for eradi cation.

Nitazoxanide

This salicylamide congener of the anthe lmintic niclosamide, introduced for the treatment of giardiasis and cryptosporidi osis is also active against many other protozoa including E. histolytica, T vagina/is, and helm inthsAscaris, II. nana, e tc. It is a prodrug which o n absorption is converted to the active form tizoxanide, an inhibitor of PFOR enzyme that is an essential pathway of electron transport energy metabolism in a naerobic orga ni s ms. Activi ty against metronidazole-resistant Giardia has also been demonstrated. Tizoxan ide generated from nitazoxanide is gl uc uronidc conjugated and excreted in urine as well as bile. N itazoxa nide is the most elTective drug for Cryptosporidium parvum infection (upto 88% cure), w hich causes diarrhoea, especiall y in children and in AlDS patients. It is also indicated in giard iasis, and may be used as an alternative luminal amoebic ide in amoebic dysentery. A bdom inal pain, vomiting and headache a re mild and infreq uent side effects. Dose: 500 mg (children 7.5 mg/kg) BD x 3 days NITA CURE, NITCOL, N ITA RID 200 mg, 500 mg wbs, I 00 mg/5 ml dry syrup.

8-HYDROXYQU INOLINES Several 8-hydroxyquinolines including Quiniodochlor and Jodoquinol were widely employed in the past. They are acti ve aga inst Entamoeba,

Prolonged/repeated use of relati, ely high doses o f quiniodochlor caused a ncuropa1hic syndrome called ·subacute myelo-op1ic neuropathy' (SMO ) in Japan in an epidemic form, alTectmg several thousand peo ple in 1970. Other 8-hydroxyquinolines have also produced neuropathy and visual impairment. llowever, despi te w idespread use in the past, only sporadic and unconfirmed cases have been reported frorn Ind ia. The 8-hydroxyquinolincs have been banned in Japan and fow other countries, but in India they are prohibited only for pediatric patients, because the ir use for chronic diarrhoeas in children has caused blindness.

8-H ydroxyquinolines are cheap and have good pa tient acceptabi lity. They may be employed in intestinal amoebiasis as alternative 10 di loxanide furoate.

ANTIAMO EBIC AND OTHER ANTIPROTOZOAL DRUGS Other uses are- g iardiasis; local treatment of rnonilial and trichomonas vaginitis, fungal and bacterial skin infections. Quniodochlor(lodochlorohydroxyquin, C lioquinol): 250--500 mg TDS: (not to exceed 1.5 g/day for 14 days). ENTEROQU INOL, Q Ul!\OFORM, DEQUINOL250 mg tab. Diiodohydroxyquin ( lodoquinol): 650 mg TDS; (not to exceed 2.0 g/day for 14 days). DIODOQUIN 650 mg tab, 2 10 mg/5 ml susp.

Dose: Oral: 500 mg (children 10 mg/kg) T DS, for 7 days for amocbiasis/giardiasis/cryptosporidiosis.

Side effects are lim ited to the g.i.t.; nausea. vomiting, abdominal cramps, diarrhoea; rarely rashes, renal impairment on prolonged use.

NOTES ON THE TREATMENT OF AMOEBIASIS

Tetracyclines

1. Acute amoebic dysentery Most cas-

Tetracyclines have modest direct inhibitory action on Entamoeba. The older tetracyclines are incompletely absorbed in the small intestine, reach the colon in la rge amounts and inhibit the bacterial flora with which Entamoebae live symbi otically. Thus, they indirectly reduce proliferation of entamoebae i.n the colon and arc especially valuable in chronic cases who have only the luminal cycle w ith little mucosal invasion. Tetracyclines have an adjuvam role in conjunction with a more efficacious luminal amoebicide. They may be added as the third drug along w ith a nitroimidazole + a luminal amoebicidc.

es of amoebic dysentery respond to a single adequate course of treatment. Metronidazole/ tinida zole are the drugs of c hoic e; afford quick symptomatic relief and kill the trophozoi tcs in 5 IO days. Secnidazole, omidazole, are the a lternat ives. Adjuvant measures for diarrhoea and abdominal pain may be needed. Dehydroemetine is rarely used in the most severe cases o r when metronidazole produces severe allergic reaction or ncurotoxicity. Dehyd roemetine should be discontinued as soon as acute symptoms arc controlled (2-3 days) and metronidazole started, except in case of metronidazole intolerance. The above treatment should be followed by a course of luminal amocbicide to e ns ure erad ication of £. histolytica from the colon and to prevent carrier (cyst passing) state. A tetracycl ine, added as the third drug, may have adjuvant value.

Paromomycin It is an ammoglycoside antibiotic which closely resembles neomycin and is described in Ch. 54. Distinctively, paromomycin is acti, c agamst many protozoa like Emwmoeba, Giardia, C1:1ptosporidi11m, Trichomonas, l.eishmania and some tape worms, in addi tion to having antibacterial spectrum like neomycm. In the 1960s an oral formulation of paromomycin "as introduced as a luminal amoebicide and wa;, briefly marketed in India as \\ell. However. it was soon overshadowed by mctrunidazole, became commercially unviablc and was discontinued. It has gain«d popularity again and is being frequently used in USA and some other countries. In India and Africa, parc:nteral (i.m.) paromomycin is being used in resistant Kalaanr (see p. 904}. Orally administer.:d paromomycin acts only in the gut lumen. It is neither absorbed nor degraded in the intestine!.. and is eliminated unchanged in the faeces. Thus, it is largely free from systemic toxicity. Its effect on gut flora resembles that of neomycin. Paromomycin is an cnicacious luminal amoeb1cu.Je, ach ieving s imilar o r even belier dearing of cysts fro m stools compared to diloxanidc furoate in asymptomatic cyst passers. Good symptomatic relief and cyst clearance is obtained in chronic amoebic colitis. Paromomycin is an alternativ« drug for giardiasis, especially during Isl trimester of pregnancy when metronidazole and other dn1gs arc contraindicated.

2. Mild intestinal amoebiasis/asymptomatic cyst passers Nitroimidazoles afford rapid symptomatic relief in mi ldly symptomatic intestinal amoebiasis and arc the first line drugs. However, they mostly fail to clear cysts. Therefore, the standard practice is to give diloxanide furoate or anothe r lu minal amoebicidc, either concu rrentl y or imm edi ate ly a fter. Luminal amoebicides alone are genera ll y s lower in action, but avoid side etTects of rnetronidazolc. Asymptomatic cyst passers a rc mostly treated wtih only lumina l a moebicide. C hronic cases may require 2-3 repeated courses in which drugs may be alternated. A tetracycline may be given concurrently w ith the luminal amoebicide in cases which fail to clear completely.

899

900

ANT IMICROB IAL DRUGS

Treatment of different forms of amoebic infection

Drugs of Choice

Alternative Drugs Acute Amoebic Dysentery

• Metronidazole 800 mg oral TDS x 7-1 0 days • Ornidazole 2.0 g oral daily x 3 days or (in severe cases 500 mg slow i.v. 6 hourly till • Secnidazole 0.5 g oral TDS x 5 days oral therapy can be instituted) Altemat,ve /uminal amoebicides or • Quiniodochlor 250-500 mg oral TDS x 7- 14 days or • Tinidazole 2.0 g oral daily x 3-6 days • lodoquinol 650 mg oral TDS x 7-14 days or + Lummal amoeb,cide • Paromomycin 500 mg oral TDS x 7-10 days • Diloxanide turoate 500 mg TDS x 5-10 days

±

• Tetracycline 250 mg TDS x 7-10 days (adjuvant)

Mild Intestinal amoebiasis/Asymptomatic cyst passers• • Metronidazole 400 mg oral TDS x 5-7 days or • Tinidazole 2.0 g oral OD x 2-3 days

+ Lummal amoebicide (as above)

• Ornidazole 0.5 g oral BO x 5-7 days or • Secnidazole 2.0 g oral single dose Altemattve /um,nal amoeb1c1des (as above) ± • Tetracycline 250 mg TDS x 7-10 days (adjuvant)

Amoebic liver abscess • Metronidazole 800 mg oral TDS x 1Odays (in serious cases 500 mg slow i.v. infusion 6 hourly x 10 days) or • T inidazole 2.0 g oral daily x 3--6 days (severe cases: 800 mg i.v. daily x 6 days or till oral can be given)

+ Lummal amoebicide (as above)

• Emetine/Dehydroemetine 60 mg i.m./s.c. x 8-1 Odays followed by or alternatively • Chloroquine 600 mg (base) oral daily x 2 days, followed by 300 mg daily for 2-3 weeks. Altemat,ve luminal amoeb1c1des (as above, but no role of tetracycline)

·1n asymptomatic cases, a luminal amoebicide alone may be used (the nitroimidazole may be omitted). Repeat courses after a gap of 2-3 weeks may be needed with the same or alternative drugs to eradicate the chronic luminal cycle.

3. Amoebic liver abscess It is a serious disease; complete eradication of trophozoites fro m the liver is esse ntia l to avoid re la pses. Mctronidazol c and ti nidazole are the fi rst choice drugs effective in > 95% ca es. Critically ill patients may be treated with i.v. meLronidazol e for the entire course, or at least initial ly, fo ll owed by oral dosing . Dehydroeme tine is Lo be used only if metronidazol e cannot be given for one reason or the other, and in patients not cured by metronidazol e. Large abscesses usually take months to resolve, even if a ll tropho7oites arc ki lled. If a big abscess has formed. it may be aspirated.

A luminal amoeb icide must be given later to finish the intestinal reservoir o f infection. A course o f chloroquine may be adm ini stered after that of meLronidazo le/dehydroe metine in those with incomplete response or to en ure that no motile forms survive in the li ver.

DRUGS FOR GIARDIASIS Ciardia la 111bl ia is a fla g e l la te protozoo n which infects children and ad ults by oro-faecal contaminati on and mostly lives as a commcnsal in t he in tes tine. It so meti mes invades the mucosa and cau e acute watery hort duration

901

ANTIAMOEB IC AND OTHER ANTIPROTOZO AL DRUGS diarrhoea with foul smellling stool , gas and abdom inal cram ps. If untreated, it may pass on to chronic diarrhoea with greasy or frothy stools but no blood or mucus. Ma ny d ru gs usefu l in a moebiasis are also effective in giardi asi .

l. Metronidazole 400 mg T DS (children 15 mg/ kg/day) for 5- 7 days or 2 g daily fo r 3 days, Or tinidazole0.6 gdaily fo r7 daysor 2gsingle dose, Or secnida;::o/e 2 g single dose These may be conside red as the drugs of choice. but - I0% patients may not be cured, and a second course or alternati ve drug may be needed. 2 . Nitazoxanide (see p. 898) This prodrug of the PFOR enzyme inhibitor tizoxan ide has become ava ilab le for the treatment of diarrhoea and dysentery caused by C,yptosporidium parvum, Giardia lambfia and £. histolytica. The dosage schedule is convenient- 500 mg (chi ldren 7.5 mg/kg) tw ice daily for 3 days. Effi cacy (~80% cure) approaches that of metronidazole. 3. Quiniodochlor 250 mg TDS for 7 days is a somewhat less efTective alternative. 4. Furazolidone It is a nitrofuran compound active against many gram-negative baci lli inc luding Salmonella and Shigetla. also Giardia and Tric/10111011as. For giardiasis I 00 mg TDS for 5 7 days has been used, but is inferior to metronidazolc or tinida.lOle. II has also been used in bacterial enteritis. but is not a first line treatment. Furazolidone is partly absorbed from intestines and excreted in u rine which turns orange- patients should be told about it. Side e ffects are mild and infrequent- nausea. headache, dizziness. FU ROXONC 100 mg tab, 25 mg/5 1111 susp.

DRUGS FOR TRICHOMONIASIS Trichomonas vagina/is is a no the r m icroaerophilic flagellate protozoo n which causes vulvovaginitis. It is a common sexuall y transmitted di sease affecting - I0% sexually active women. Several drugs are partly effective by vagina l appl ication, but may not entirely clear the infection; recurrences are freq uent; repeat courses are required.

1. Drugs used orally Metronidazo/e 400 mg T DS fo r 7 days or 2 g single dose, or Tinida::ole 600 mg daily for 7 days or 2 g single dose or Secnidazole 2 g single dose, a re the drugs of choice. They produce upto 90% c ure. However, vaginitis due to nitroimidazole resistant T vagina/is is bei ng reported. Some resistant cases respond to higher doses, pa rti cularly oftinidazole. Addi tional intravaginal treatment is required only in refractory cases. A hard core of rec urren t cases may remain. A repeat course can be given afte r 6 weeks. In some cases recurrences are due to rei nfection fro m the male partner who harbours the parasite in the semi na l vesicles but remains asym ptoma tic. In such case , both part ners should be treated conc utTently to prevent cross infection of each other.

2. Drugs used intravaginally I. Diiodohydroxyquin 200 mg inserted intravagina lly at bed time fo r 1- 2 weeks; FLORAQUN 100 mg \"aginal pessaries.

2. Quiniodochlor 200 mg inserted in the vagina every night for 1- 3 weeks; GYNOSAN 200 mg vaginal tab.

3. Povidone-iodine 400 mg insert ed vagina daily at night for 2 weeks;

111

the

BETADINE VAGIN AL 200 mg pessaries.

DRUGS FOR LEISHMANIASIS Visceral leishmaniasis (VL; kala-azar) caused by leishmania donovani (and o!her Lcishmania species) occurs in several tropical and subtropical reg ions of 1he world. Though prevalence of the disease globally. including India, is continuously declining over the past decade, still according to WHO estimate made in 20 I 5. about 0.3 million cases of VL occur annually worldwide with > 20,000 deaths each year. About 90% of the cases occur in India. BraLil. Ethiopia, Kenya. Somalia and Sudan, but the disease i~ also present in other countries of East Africa, South America, Mediterranean basin and central Asia. In india. leishmaniasis is prevalent in Dihar. West Bengal, Jharkhand and eastern UP; the worst affected being Dihar which contributes 50% cases that occur world over. In treating VL. the geographical location is important. because the species o r leish111a11ia causing VL and its responsiveness to d ifferen t drugs differs between diffe rent regions. The

902

ANTIMICROB IAL DRUGS

disease is h ighly concentrated in North Bihar and t he parasite is resistam to sodmm stibogluconate (SSG). the first line drug in many other countries.

Leishmaniasis is transmitted by the bi te of the rcma le sa ndfly ph lebotomus. In the fl y the parasi te exists in the flage llate extracellular (promastigote) fo rm, while in man it is fou nd only intracellularly w ith in macrophages in the nonflagellate (amastigote) form. Mucocutaneous and dermal leishmaniasis are ca used respectively by L. brazi!iensis and L. tropica (a lso other species). Visceral leishm aniasis ( VL) is fata l unless treated. The curTently used drugs for treatment or VL are: I. Sodium stibogl uconate (SSG ) (or Meglumine antimonate- in French speaking countries of Latin and south America) 2. Amphoteric in B (A MB) 3. Miltefosine 4. Paromomyc in . Pentamidine was used in resistant kala-aLar till 15 years back but not now. Ketoconazole and Allopurinol have weak antileishmania action, but are not used now.

Kal a - azar is prima ril y a di sease of the economica ll y poor c lass , a nd the areas affected are underdeve lo ped . India launched the ' Kal a-azar Control Programme ' in 1990, which was upgraded in the year 2000 to aim at e limination of the disease. The prog ramme is implemented under the VBDCP, wh ich has laid down its own treatment guidelines, and provides free treatme nt. Under the programme over 33,000 cases of V L were treated in 20 11 w ith 90 deaths. The number o f cases dec lined by 33% in 20 13 and only 20 died. H owever, the actual number of cases is much g reater. O nl y confirmed cases (by 'rapid d iagnostic test' or splenic aspirate examination) are to be treated with antileishmania drugs. The choice of drugs, doses and regimens as c urrently recomme nded by WHO and VB DC P are s ummarized in the box (p. 903). Cure is ind icated by clinical improvement and absence or relapse w ithin 6 months. This can be confirmed by absence of leishmania m s plenic aspirate smear examination.

1. Sodium stibogluconate (SSG) It has been the standard fi rst line drug for VL in most parts of the world achieving > 90% cure rate, and is still used in East Africa, Central Asia. Mediterranean basin and South America, but is no longer efTectivc in India and epal because of extensive resistance. Over 60% cases in Bihar are unresponsi~e. T herefore, SSG is no longer used in India. It is a water soluble pentavalent antimonial, the supplied solution contains I 0% ( I 00 mg/ml) antimony, and doses are expressed in terms of elemental Sb. The mechanism of action and the basis of selective toxicity to the leishmania amastigotc is unclear. It was believed that -SH dcpcndem enzymes are inhibited by antimony and bioenergctics of the parasite is interfered w11h. This occurs due to blocking of glycolytic and fatty acid oxidation pathways. However, recent evidence indicates that a specific reductasc enzyme present in leishmania amastigots reduces pentavalent-Sb of SSG to the toxic trivalent fom1, which then promotes efflux of g lutathione and other reduced thiols from the parasite residing within macrophages, exposing them to oxidative damage. Resistance to SSG may involve reduced capacity of the parasite to conven it to the trivalent fonn, and/or alteration in thiol metabolism of the parasite. Sod. stibogluconate is rapid ly absorbed from the site of i.m. injection and excreted unchanged in urine within 6-12 hrs. A small ti-action enters tissues and remains s tored for long periods. Repeated doses are cumulative. Accumulation ofSSG within macrophages accounts for its prolonged inhibitory efTect on leishmania residing there. SSG is administered by deep i.m. injection (into buttocks) or by slow i.v. injection daily or on alternate days. Adverse effects Though, antimonials are toxic drugs, but the penta,alcnt compounds (panicularly SSG) are beuer tolerated. Nausea, vomitmg, meta llic taste, cough, pain abdomen. pain and s tifTness o f injected mu cle, sterile abscesses, and mental symptoms may occur. Q-T prolongation may herald arrhythmias. Few cases of shock and death are on record. Used alone or in combination with paromomyc in, SSG is sti ll a 1• line drug in East Africa, Central Asia and South America.

2. Amphotericin B (AMB) (see Ch. 58) This antifunga l antibi otic is ava ilable in two types of preparations. The older and less expensive one is form ulated w ith deoxycholate (A M BDOC), while in the newer one it is incorporated in liposomes (L-AM B), and is expensive. Like fungi , leishmania has high percentage of ergosterol a nd is s uscepti ble to th is antibiotic wh ich has high affi nity for ergosterol and acts by binding to it. Presently, AMB is the most effective drug with highest cure rate in ka laazar: up to 99% c linical and parasito logical cure has been reported from Ind ia in SSG resistant

ANTIAMOEBIC AND OTHER ANTIPROTOZOA L DRUGS

• Recommended treatment regimens for visceral leishmaniasls (Kala-azar) caused by L. donovani in the Indian subcontinent t1. Amphotericin B deoxycholate (AMB-DOC): 0.75-1 .0 mg/kg i.v. infusion over 4 hours daily or on alternate days till 15 mg/kg total dose. •2. Liposomal amphotericin B (L-AMB): 3-5 mg/kg i.v. infusion daily for 3-5 days (total dose 15 mg/kg) or L-AMB 10 mg/kg single dose i.v. infusion s3_ Miltefosine (all doses given orally with meals for 28 days) Adults (>12 years) weighing> 25 kg: 100 mg/day (50 mg cap twice daily)' Adults (>12 years) weighing < 25 kg: 50 mg/day (50 mg cap once daily) Children (2-11 years): 2.5 mg/kg/day (as 1O mg caps) 4. Paromomycin sulfate: 15 mg (11 mg base) per kg/day i.m. for 21 days 5. Sodium stibogluconate (SSG): 20 mg (Sb5•)/kg i.m. or slow i.v. daily for 30 days (only in areas where the Leishmania is still sensitive to SSG). COMBINATIONS (co-administered drugs) 1. L-AMB (5 mg/kg i.v. infusion single dose)+ Miltefosine (as above) for 7 days. 2. L-AMB (5 mg/kg i.v. infusion single dose) + Paromomycin (as above) for 10 days. 3. Miltefosine (as above) daily for 10 days+ Paromomycin (as above) daily for 10 days. RESCUE TREATMENT (of failure/non-responsive cases) 1. AMB-DOC or L-AMB at higher doses. 'Adopted from Report of meeting of the WHO Expert Committee on control of leishmaniasis, March 22-26, 2010; WHO Technical Report Series No. 949 (2010) s Used as 1'' line treatment under NVBDCP £Used as 2nd line treatment by NVBDCP. ' 1st preference regimen recommended by WHO. e The WHO in additon recommends miltefosine dose to be 150 mg/day for adults weighing > 50 kg, while NVBDCP treats all adults above 25 kg with 100 mg/day

cases, and it is treated as the 'reference drug' whi le comparing the efficacy of other drugs. However, high toxicity and need for prolonged hospitalization, monitoring and repeated slow i. v. infusions limit its application. Therefore, it is the 2 nd line treatment of VL under NVBDCP, though the WHO recommends it as the drug of choice and accords it higher preference over miltefosine. Because miltefosine is teratogenic, AMB is the drug of choice in pregnant women and breast feeding mothers. Liposomal AMB is particularly suited for kala-azar because it delivers the drug inside the reticuloendothelial ce ll s in spleen and liver where the amastigotes live, but high cost is prohibitive. As such, use of L-AMB in India is limi ted. Using L-AMB, the total dose of 15 mg/ kg can be administered over 3- 5 days with - 98% cure, and WHO has accorded it the highest preference. Even a single dose treatment has been tried. reporting 90% cure at 5 mg/kg, and 98% cure at IO mg/kg.

AMB is also useful m mucocutaneous and derma l leishmaniasis.

3. Miltefosine

It is a derivative of alkyl phosphocholine wi th potent antile ishm a nia activity that has been tested in India since the 1980s, but was approved o nly in 2002 as the first orally active drug for kala-azar. Due to spread of SSG resistance in Bihar and nei g hbouring areas, NVBDC P is now using miltefosine as the )51 line treatment of VL in India. A 4 week course of mil tefos ine has achieved > 95% cu re rate in Ind ia and 90% in Ethiopia. Cutaneous leishmaniasis can also be cured by 4 week thrapy. Miltefosine is available in India only through NVBDCP. It is also avai lable in few other countri es of the Indian region, in South America and has been recently US-FDA approved. The mechanism of antilcishmania action of mi ltefosine is not known. However, studies suggest that it may be interferi ng wi th lipid metabolism of the parasite or prevent synthesis

903

904

ANTIMICROBIAL DRUGS

of some critical cell surface anchor molecules, or alte r signal transducti o n. Leishmania can develop resistance to miltefosine and this may be due to mutation limiting trans port of the drug into the parasi te cell.' Recently, decreased responsiveness has been sus pected in India. To prevent development of resistance to th is useful drug. combination therapy w ith paromomycin o r AMB is being promoted. Miltefosine is rapidly absorbed after oral medication, and widely distributed in the body. It is a long acting drug with biphasic elimination. In the early phase, t½ is ~7 days whi le the terminal t½ i ~ 4 weeks. Anorexia, vomiting and diarrhoea are the commonest side effects occurring in over 50% patients. However, these are general ly brief and resolve with continued use. Skin allergy and rise in hepatic transaminases occurs in some recipients indicating hepatic derangement, but this is usually mild and reverses on stopping the drug. Reversible kidney dysfunction with rise in serum creatinine has also been noted. Miltefosine is teratogenic. It is contraindicated in pregnant women. When miltefosine is given, it should be ensured that female patients do not get pregnant during and till 3 months after miltefosine course.

4. Paromomycin

This a minoglycoside antibiotic is described in C h. 54 and with antiamoebic drugs o n p.899. For intestinal protozoa! infections, it is used by the oral route and remains confi ned to the gut. In 2006 it was approved in Indi a for use in VL by the i.m. route. Over the past 2 decades paromomycin has been tried in India and Africa fo r kala-azar and fou nd to be effective in SSG-resistant cases. The WHO recommends 21 day paromomycin treatment as an a lternative to miltcfosine, but is con idered to have lower efficacy, particularly in Africa. In a phase 111 trial on 667 kala-uar patients in Bihar.' parornornycin 11 mg/kg/day x 21 days has yielded 95% cure rate, which was statistically not inferior to 99% cure rate obtained with AMB I mg/kg x 15 injections over 30 days. Mortality was < I% with both the drugs. Several other trials have confinncd the efficacy ofparomomycin in VL.

Th o ugh paromomycin produces ototoxicity (in 2% recipients), reversible e levation of serum transamina e and injection site pain, but renal toxicity is rare. Paromomyein is a less expensive and eas ier to use, but some what less effective alternative to AM B in kala-azar. Topical paromomycin is effective in dermal leis hmania sis.

Combination therapy Like in the case o f TB, leprosy, HIV and malari a, combinatio n therapy w ith 2 e ffective drugs has several advantages in the treatment of VL. These are: • L imitin g ri s k o f development of dru g resistance, thereby prolonging the effective life-time of availa ble med ic ines. • Atta ining higher efficacy and cure rate. • Shortenin g of duration o f therapeutic regimen; better compliance and convenience. • Reduction o f o veral I dose; lower toxicity and cost. Clinical studies in India have testified to the hig h efficacy of drug combinati ons, and have shown that duration o f treatment can be reduced by ha lf or more. The 3 combinations tested were: • L-AMB (5 mg/ kg i.v. s ing le dose) + Miltefosine (oral) daily x 7 days • L- AMB (5 mg / kg i.v. s in g le dose) + Paro momycin (i.m.) daily x IO days • Miltefos ine (o ral) daily x I O days + Paromomycin (i.m.) daily x IO days. Eac h of these combinatio ns yie lded 98- 99% c ure rate. A s such , these combinatio ns are recommended with high preference by WHO. However, cost of L-AMB (even sing le dose) is high.

HIV and Leishmania coinfection H IV and le is hmania infectio ns wo rsen each other. In endemic areas. HIV pos itive s ubjects

*Sundar S. Chakravarty J. Leishmaniasis: an update of current phannacotherapy. Expert Opi11 Pharmacother;. 16, 237-52, 20 15. 5 Sundar S, et al: Injectable paromomycin for visceral lcishmaniasis in India. N Engl J Med. 2007;356: 257 1-258 I.

ANTIAMOEBIC AND OTHER ANTIPROTOZOAL DRUGS are more likely to develop VL, harbour higher and disseminated leishmania load, and have poorer prognosi s. Simil arl y. occurrence of VL in a I ll V positive subject is regarded 10 be a si gn of A IDS and warrants intensi ve therapy. Coin fected patients have a poorer response to anti lei shmania as well as anti-HIV drugs. Therapy of such patients is difficult and mortality is high.

Drugs used locally for dermal leishmaniasis (oriental sore) Dermal leishmaniasis is not a life-threatening condit i on; many cases are treated by l ocal application of drugs.

I . Sodium stibog/11cona1e: Infiltrate 2 ml of the solution ( I 00 mg antimony/ ml) around the sore. 2. Paromomycin ( 15%) ointmen1: applied top ically on the sore, twice daily fo r 20 days. Small and mild lesion may heal by itsel f in a few months. Multiple sores and severe cases should be treated by systemic drugs as for ka la-azar. A nt ibiotics may be needed for secondary infection of the sore.

r:r PROBLEM DIRECTED STUDY 62.1 A SO-year-old gardener weighing 58 kg was admitted to the hospital with fever for 4 days, severe pain in right upper part of abdomen, loss of appetite, vomiting and marked weakness. He was not wel l for the past 2-3 weeks and had lost weight. There was no history of chronic diarrhoea. Palpation of abdomen revealed soft tender enlargement of liver 2 cm below costal margin. Marked tenderness was noted in the lower right intercostal region. Ultrasound showed a solitary 2.5 cm diameter abscess with sharp margins in t he right lobe of liver. Stool examination was negative for any kind of ova and cysts. A clinical diagnosis of amoebic liver abscess was made and he was treated with: Injection Metronidazole 500 mg i.v. over 1 hour every 8 hours for 5 days along with infusion of glucose-saline and vitamins. The fever and vomiting subsided and he started eating food. The injections were substituted by oral metronidazole 800 mg 3 times a day for another 5 days, and the patient became well, except weakness and mild tenderness in the right lower chest. Repeat ultrasound showed abscess cavity size to decrease to 1.5 cm. The patient was discharged with advise for vitamins and food . (a) Was the choice of medication and route correct, or a better drug/route of administration is available? (b) Should metronidazole therapy be extended or a repeat course given? (c) Should the patient be given any other antiamoebic medication in addition to or following metronidazole? (see Appendix-! for solution)

905

Chapter

63

Anthelmintic Drugs

Anthelmintics are drugs that e ither kill (vermicide) or expel (venn ifuge) infesting helminths. He lminthiasis is prevalent globally ( I/3rd of world 's population harbours them), but is more common in de veloping countri es with poorer personal and environmenta l hygiene. Multiple infestation in the same indiv idual are not infrequent. In the human body, g.i.t. is the abode of ma ny helminths, but some worms also live in tissues, or their larvae migrate into tissues. They harm the host by depriving him of food, causing blood loss, injury to organs, intes tina l or lymphatic obstruction and by secreting tox ins. Helm inthiasis is rarely fa tal, but is a major cause of ill health.

The choice of drug for each worm infestation is based not only on efficacy, but also on lack of side effects/toxicity, ease of administration (preferably single dose) and low cost. Development of resistance has not been a problem in the c linical use of anthelmintics. A therapeuti c grouping of the anthel mintics is presented in the chart, and the current choice of drugs for worm infestations common in the lndian subcontinent is given in Table 63. 1.

Mebendazole It is a benz imidazole in t roduced in 1972. This congener o f thiabendazo le became very popular because it retained the broad-spectrum

ANTHELMINTIC DRUGS

I

I

For Roundwonn, Hookworm, Plnwonn

For Whlpworm, Trlchlnella BDlralls

Albendazole Mebendazole Py rantel pamoate Piperazine Levamisole

Albendazole Mebendazole

, For Threadworm

lvermcctin Albendazole

I

I For Tapeworms Praziquantel Niclosamide Albendazole

I For Filariaals Diethylcarbamazine lvermectin Albendazole

I For Hydatld disease

Albendazole Mebendazole

907

ANTHELMINTIC DRUGS

Table 63.1 : Choice of drugs for helm1nth1as1s Worm

First choice drugs

Alternative drugs

1. ROUNDWORM Ascaris lumbricoides

Mebendazole, Albendazole, Pyrantel

Piperazine, Levamisole lvermectin

2. HOOKWORM Ancylostoma duodenale Necator americanus

Pyrantel, Mebendazole, Albendazole Mebendazole, Albendazole

Levamisole Pyrantel

3. PIN WORM Enterobius (Oxyuris) vermicularis

Pyrantel, Mebendazole, Albendazole

Piperazine

4. THREAD WORM Strongyloides stercorahs

lvermectin

Albendazole

5. WHIPWORM

Mebendazole

Albendazole

Trichuris trichiura 6. Tr/chine/la spiralis

Albendazole

Mebendazole

7 . FILARIA

Diethyl carbamazine, lvermectin

Albendazole

Albendazole

lvermectin

9. TAPEWORMS Taenia saginata Taenia solium Hymenolepis nana Neurocysticercosis

Praziquantel Praziquantel Praziquantel Albendazole

Niclosamide, Albendazole Niclosamide, Albendazole Niclosamide, Nitazoxanide Praziquantel

10. HYDATID DISEASE Echinococcus granu/osus, E. multilocularis

Albendazole Albendazole

Mebendazole

Wuchereria bancrofti, Brugia malayi

8. CUTANEOUS LARVA MIGRANS Ancylostoma caninum

anthe lmintic act1v1ty but not the toxicity o f its pre dece ssor. Mebendazole has produced nearly I00% cure rate/reduc tion in egg count in ro undwo rm , hoo k worm (both s pec ies), £nterobius and Trich11ris infestations, but is much less active on Strongyloides. Upto 75% cu re has been reported in tapeworms, but H. nana is re lati vely insensitive. It ex pe ls Trichinella spiralis from intestines, but clncacy in killi ng larvae that have m igrated to muscles is uncerta in. Prolo nged trea tment has bee n shown to cause regression of hydatid cysts in the li ver. Treatment after resection of the cyst may prevent its regrowth. The immobili z in g and letha l actio n o f mebendaz ole o n worms is rather slow: takes

2-3 days to develop. The maj or site of action of mebendazole appears to be the microtubular protein ' P-tubulin" of the parasite. It binds to P-tubulin of s usceptible worms with hig h affinity and inhibits its polymerization. Intracellular microtubules in the cells of the worm are gradual ly lost. In addi tion, it blocks glucose uptake in the parasite, inh ibits some mi tochondrial enzymes a nd depletes its glycogen stores. I-latching or nematode eggs and the ir larvae are a lso inhibited. Ascaris ova are killed.

Pharmacokinetics Absorption of mebendazole from intestines is minima l; 75- 90% of an oral dose is passed in the faeces. The fraction absorbed is excreted mainly as inactive metabolites in urine/faeces.

908

ANTIM ICROB IAL DRUGS

Adverse effects

has the advantage of si ngle dose administration in many infestations. One dose treat ment has produced cure rates in ascariasis, hookworm (bot h species) and enterobiasis which arc comparable to 3 day treatment wi th mebcndazole. Results in trichuriasis have been in ferior to mebendazole. In strongy lo idosis , it is more effective than mebendazole. A 3 day cou rse has achieved nearly 50% c ure, and a second cou rse repealed after 3 weeks cured practically a lI patients. Th ree day treatment has been found necessary for tapeworms including H. nana. Results in hydatid disease and hookworm have Uses and administration Mebendazo le is been s uperior to mebendazole. Albendazole has available as: MEBEX, WORMIN 100 mg chewable weak microfilaricidal action. kills cysticerci, tab and 100 mg/5 ml suspension. MEBAZOLE 100 mg hydatid larvae, ova of ascaris/hookworm and is tab. The dose and duration of treatment is the also effective in cutaneous larva migrans. The same for children above 2 year as for adults; mechanism or action of a lbendazole is similar ½ dose for 1- 2 yr age. Roundworm } I 00 mg twice a day for 3 consecutive to that of mebendazole. days. No fasting, purging or any other Pharmacokinetics Absorption of a lbendazole Hookworm Whipworm preparation of the patiems is needed. after oral admini stration is s ig niri ca nt, but

Mebendazole is well tolerated even by patients in poor health. Diarrhoea, nausea and abdominal pain have allended its use in heavy infestation. Incidents of expulsion o r Ascaris from mouth o r nose have occurred, probably due to starvation or the parasite and their s low death. Allergic reactions, loss of ha ir and gran ul ocytopen ia have been reported w ith hi gh doses. Safety of mebend azo le during pregnancy is not known, but it is contraindicated on the basis of animal data.

Upto 7 day treatm e nt may be needed in heavy tri churiasis, and c ure rates are lower, than in ascariasis. Pin worn, (Enterohius) I00 mg single dose, repeated after 2-3 weeks (to kill the ova that have developed later). Strict hygien ic measures and simultaneous treatment of all children in the family or class is advocated to cut down autoinfection and person to person infection. This holds true of enterobiasis, irrespective of drug used. Trich inosis: 200 mg BO for 4 days; less effec tive than a lbendazo lc. Hydatid disease: 200-400 mg BO or TDS for 3-4 weeks; less effective than albendazole. Mebendazole is one of the preferred drngs for treatment or multiple infestations and is more effective than albendazole in trichuriasis. It has also been used for mass treatment, but need for multipl e doses is a drawback.

Albendazole It is a s ubseq uentl y introduced congener of mebendazole: reta ins the broad-spectrum activity and excellent to lerabil ity or its predecesso r, and

inconsistent. Absorption is enhanced when the drug is taken with fatty meal (this may he lp in treating neurocysticercosis and hydatid disease). The fraction absorbed is converted by first pass metabolism to its sulfoxide metabolite which has potent anth elmintic action. In contrast, the metabolites of mebendazole and thiabendazole are inactive. Albendazole sulfoxide is widely di stributed in the body, enters b rain and is excreted in urine with a t ½ of 8.5 hours. Thus, albendazole is able to exert antihelmintic acti vity in tissues as we ll.

Side effects Albendazole is we lI tolerated; only gastrointestinal s ide effects have been noted. Few pa tients have felt dizzines . Prolonged use, as in hydatid or in cysticcrcosis, has caused headache, fever, alopecia, jaundice a nd neutropenia. ZENTEL, AL"1 1t\TH. A LBEZOLE, COMBANTRI -A 400 mg tab, 200 mg/5 ml suspension.

Uses No preparation o r postdrug fasting/ purging is required. For intestinal worms it should be g iven o n empty stomach, while for cysti cercosis, hydat id and c utaneous larva migrans it shou ld be given with a fatty meal.

ANT HELMINT IC DRUGS

• Ascaris, hookworm, Enterobius and Trichuris: a si ng le dose of 400 mg ( for adu lts and children above 2 yrs: 200 mg for 1- 2 yr age) is sufficient. Three day treatment may be needed in heavy tric huriasis. • Tapeworms and Strongyloides: 400 mg twice daily for 3 consecutive days. It is a second choice drug to praziquantel and niclo amide for tapeworm infestation. Efficacy in strongyloidosis is low, and it is the 2nd choice drug to ivermectin. • Tric hinosis: Three day treatment w ith 400 mg twice daily expels the adult worm from intestine, but has limited effect on larvae that have migrated to muscles, for which there is not effective specific treatme nt. La rvae are not killed but symptomatic relief occurs. Cort icosteroids and analgesics are added if systemic manifestations are severe. eurocysticercosis: Albendazole is the anthelminti c of choice for the treatment of neurocysticercosis (see p. 9 I 3). Usuall y 8- 15 days cou rse of 400 mg BD ( 15 mg/ kg/ day) is employed. Cysticercosis of ot her tissues (m uscles, su bcutaneous area) also responds, but no drug should be given for ocular cysticercosis, because blindness can occur due to the reaction to dead cysticerci. • C utaneous larva migrans: Albendazole 400 mg dail y for 3-5 days is the drug of choice; kills larvae and relieves symptoms. Albendazo le 400 mg twice da ily fo r 5 da ys is the most effective therapy for visceral larva migrans as well. • Hydatid disease: 400 mg 8D for 4 weeks, repeat after 2 weeks (if required), up to 3 courses. It is the preferred treatment give n before and after s urgery as we l I as fo r inoperable cases. • Filariasis: Added to dicthylcarbamazine ( DEC) or ivcrmectin, albendazole has adjuvant value in treating lymphatic fila riasis. A single dose of its combination with either DEC or ivermectin g iven yearly has been used in mass programmes to suppress microfi laraemia and disease transmission.

Yearly mass treatment of children with 400 mg albendazole has been shown to lower worm burden, particularly in developing countries, and has been recommended by WHO. Because albendazole has exhibited embryotoxicity in animals, use in pregnant women is contraindicated. It should be given w ith caution to patients with hepatic or renal disease.

Pyrantel pamoate 111 1969 fo r pin wo rm infestatio n in childre n; use soon ex te nded to roundworm and hookworm as we ll. Efficacy against Ascaris, Enrerobius and Ancylostoma is high and comparable to that of mebendazole. Lower cure rates (abo ut 60%) have been obtained in case of Necaror infestation. It is less active aga inst Srrongyloides and inactive against Trichuris and o ther worms. Py rantel act ivates nicotinic c holinergic receptors in the worms resulting in persistent depolarization, giving rise to slowly developing contractu re and s pastic para lys is. Wo rms are then expelled. An anticholinesterase action has also been demonstrated. Because pi perazine causes hyperpolarization and flaccid paralys is, it antagoni zes the actio n of pyrantel. Chol inergic receptors in mamm a lian skeletal muscle have very low affin ity for pyrantel. Only I 0-15% of an oral dose of pyrante l pamoate is absorbed: thi s is partly metabolized and excreted in urine.

It was int roduced

Adverse effects Pyrantel pamoate is remarkably free of side effects: occasional g.i. symptoms, headache a nd di zziness are reported. Other advantages are it is tasteless, non-irritant and does not provoke abnormal migration of worms. Safety of pyrantcl pamoate in pregnant women and in c hildren below 2 years has not been established. Use and administration For Ascaris. Ancylosroma and Enterobius: a si ng le dose of 11 mg/kg is recommended. A 3 day course for Necator and fo r Strongyloides is needed. o fasting, purging or other preparation of the patient is required.

909

ANTIM ICROB IAL DRUGS

910

EMOC ID, ANTIMINTII, EXPENT 250 mg tab, 50 mg/ml suspension ( IO m l bou le).

ulcers and recurrent herpes, but repeated doses produce severe reactions; not used now.

Piperazine

Adverse effects One or two doses used in hclminthiasis are well tolerated. Incidence of side effects-nausea. abdominal

Introduced in 1950, it is a highl y active drug against Ascaris and £nterobi11s; achieves ~90% cure rates. However. because of availability of more convenient and better tolerated a lbendazole/mebendazole it is now infrequently used. Piperazinc causes hyperpolarization of Ascaris muscle by channels causes a GABA agonistic action. Opening of relaxation and depresses responsiveness to contractile action of ACh. Flaccid paralysis occurs and worms are expelled alive. They recover if placed in piperazine free medium. Therefore, often a purgative (senna) is given with it, but is not mandatory. No fasting or patient preparation is required. Piperazine does not excite Ascaris to abnormal migration. It docs not affect neuromuscular transmission in man. A considerable fraction of the oral dose of piperazine is absorbed; partly metaboli zed in liver and excreted in urine. Piperazine is safe, but nausea, vom it ing, abdomina l discomfort and urticaria may be felt. DiZLiness and excitement occur at high doses; toxic doses produce convulsions; death is due to respiratory failure. lt is contraindicated in renal insufficiency and in epileptics, but is safe in the pregnant.

er

Dose: For roundworm infestation 4 g once a day for 2 consecutive days; children 0.75 g/year of age (max. 4 g) is considered curative. Because of its ability to relax ascarids, it is of particular value in intestinal obstruction due to roundworms. It can be used during pregnancy while safety of other drugs is not established. Pin worm: 50 mg/kg (max. 2 g) once a day for 7 days or 75 mg/kg (max. 4 g) single dose, repeated after 3 weeks. PI PERAZlNE CITRAT E 0.75 g/5 ml elixir in 30 ml, 115 ml bottle; 0 .5 g (as phosphate) tablets; Combination ofany other anthelmintic (except piperazine) with a purgat ive in the same formulation is banned in India.

Levamisole, Tetramisole Tetramiso le was developed in the late 1960s. It is racemic; its levo isomer (levamisole) was found to be more active and preferable. Both are active against many nematodes, but use is restricted to ascariasis and ancylostomiasis as a second line drug. The ganglia in worms are stimulated causing tonic paralysis and expulsion of live worms. Interference with carbohydrate metabolism (i nh ibition of fi.11narate reductase) may also be contributing. Dose: Ascariasis-Single dose 150 for adults, I 00 mg for children 20-39 kg body weight, 50 mg for 10-19 kg. Ancylostomiasis- Two doses at 12 hour intervals. It is less e ffective against Necca/or. Levamiso le : DEWORMIS, VERMISOL 50, 150 mg tab, 50 mg/5 ml syr. Lcvamisole is an immunomodulator as well: restores depressed T cell function. It was used as a disease modifying drug in rheumatoid arthritis and as an adjunct in malignancies, aphthous

pain, giddi ness, fatigue, drowsiness or insomnia is low.

Oiethylcarbamazine citrate (DEC} De ve lo ped in 1948, it is the first drug for fi larias is caused by the nematodes Wuchereria bancrofti (90% cases) and Brugia malayi . DEC is absorbed after oral ingestion, distributed all over the body (V = 3- 5 L/kg), metaboli zed in li ver and excreted in urine. Excretion is faste r in acidic urine. Plasma t ½ of us ual clinical doses is 4- 12 hours, dependi ng on urinary pH. Diethylcarbamazine is microfi laric idal. lt has a hig hly selecti ve effect on microfilariae (Mt). A dose of 2 mg/kg TDS clears Mf of W bancrofti and B. malavi from peripheral blood in 7 days. However, Mf present in nodu les and transudates (hydrocoele) a re not killed. The most important action of DEC a ppears to be alteration of organell e me mbranes of the Mf promoting cell death. It is also suggested that muscul ar activity o f M f and adult worms is affected so that they are dislodged. Prolonged treatment may kill adult B. malayi and probably W bancrofti worms a lso. Th us, DEC is slow acting macrofilaricidal. DEC is acti ve against Mf of Loa loa a nd Onchocerca volvulus as well. The adult worm of L. loa but not 0. volvulus is killed. DEC reduces worm burden in ascariasis, but efficacy is low.

Uses I. Filariasis : DEC 2 mg/kg IDS is the first line drug: produces rapid symptomatic rel ief; Mf disappear from blood and patient becomes noninfecti ve to mosquitoes in 7 days. However, th e adu lt worm s urvives in lymphatics and gives rise to intermittent microfilaraemia and symptoms. Prolonged treatme nt with d ifferent sched ules has been fou nd to achieve radical

ANTH ELMINTIC DRUGS

cure m most patients. A total dose of 72- 126 mg/kg spread over 12 days to 3 weeks has been found satisfactory, but more than one cou rse may be needed with a gap of 3-4 weeks. Elephantiasis due to chronic lymphatic obstruction is not affected by DEC, because fibrosis of lymphatics is irreversible. Yearly treatment with a combination of DEC (6 mg/ kg) and albendazole (400 mg) single dose on mass scale has brought down transmission of filariasis by reducing microfi laraemia.

2. Tropical p11/mona,y evsinophi/ia: DEC (2-4 mg/kg T DS) for 2- 3 weeks produces dramatic improvement in the signs and symptoms of eosinophilic lung or tropical eosinophi lia. The benefit probably reflects anti-microfilarial action: the symptoms of the disease bei ng presumab ly due to reaction to the Mf. The associated cough may respond to inhaled corticosteroids. HETRAZAN. 13ANOCIDE 50, 100 mg tab, 120 mg/5 ml ,yr; 50 mg/ 5 ml pediatric syr; to be taken uficr meals.

Loa loa and 0. volvulus infections can also be treated with DEC, but the risk of life-threatening reaction to dying Mf is high. DEC is better avoided initially, or a small (25-50 mg) test dose is given first which is un likely to produce severe reaction. lvennectin does not produce such severe reactions and is preferred for initial treatment, but is less effective against adult worms than DEC.

Adverse effects

Side effects are common but general ly not serious. Nausea, loss of appetite, headache, weakness and dizziness are the usual complaints. A febrile reaction with rash, pruritus, enlargement of lymph nodes, bronchospasm and fall in BP may occur due to mass destruction of Mf and adult worms. This is usually mild, but may be severe. The reaction can be minimized by starting with a low dose (0.5 mg/kg) and escalation to full dose over the next 4 days. When the reaction occ urs, DEC should be temporarily withheld and antihistaminics and/or corticosteroids given. Subsequent adm inistration of DEC does not cause such reaction. Leukocytosis and mild albuminuria are also noted.

lvermectin ll is an extremely potent semisynthetic derivative o f the anti nematodal principle obtained from Streptomyces avermiti/is. lvermectin is the drug of choice for single dose treatment of onchocerciasis and strongyloidosis, and is comparable to DEC for bancroflian and brugian filaria. 1t is microfilari cidal but not macrofilaricidal. Jvermectin (0.2 mg/kg sing le dose) is also highly effective in cutaneous larva migrans and ascariasis, while efficacy against Enterobius and Trichuris is moderate. Jt has been used as add-on drug to albendazole/mebendazole in heavy trichuriasis. Certain insects, notably scabies and head lice are killed by ivermectin. ematodes develop tonic paralys is when exposed to ivennectin. It acts through a special type of glutamate gated Cl- channel fou nd only in invertebrates. Such channels are not involved in the motor control of Aukes and tapeworms which are unaffected by ivennectin. Potentiation of GA BAergic transmission in the worm has also been observed. The lack of GABA-related actions in man could be due to its low a ffinity for mammalian GABA receptors and its excl usion from the brain by P-glycoprotein mediated efflux at the blood-brain barrier. A si ngle I 0- 15 mg (0.2 mg/kg) oral dose of ivermectin, preferably with 400 mg albendazole, given annually for 5-6 years has been used for filaria is. Single 0.15- 0.2 mg/kg dose has yielded highest cure rate in strongyloidosis, but repeat dose may be given if stool remains positive. For Strongyloides hyper-infec tion syndrome occurring in immunocompromised patients, daily treatment till the syndrome subisdes and larvae are eradicated is needed. Jvermectin reduces burden of other intestinal nematodes as we ll. lvennec1in has replaced DEC for onchocerciasis and has been used in the 'river blindness' control programme of WHO in Africa and Lalin America. One dose of ivcm1ec1in is given a1 6-12 month intcrvals--produces long lasting reduction of Mf counts in eye and skin, without affecting the adult wonn. Though it docs not cure 0 . volvu/11s infection. ocular inflammation/damage as well as lymphadenopa1hy are suppressed with only mild ocular or systemic reactions.

lvenncctin is the onl y drug effective orally in scabies and pediculosis. Single 0.2 mg/kg dose cures most patients.

911

ANTIMICROBIAL DRUGS

912

IVERMECTOL, IVER\.IEC, VER1v1 1N 3. 6 mg tabs; to be taken on empty stomach.

Ivermectin is well absorbed ora lly, widely distributed in the body, but does not enter CNS, sequestrated in liver and fat, and has a long terminal t½ of 48---60 hours. It is metabolized by CYP3A4, but no drug interactions related to this isoenzyme are known. Side e ffects have been mild- pruritus, giddiness, nausea, abdominal pain, constipation, lethargy and transient ECG changes, but more important are the reactions due to degeneration products of the Mf, which are fever, urticaria, myalgia, edema and tender lymph nodes. Safety of ivermectin in p regnant women and young children is not established. Doxycycline ln a recent innovative approach, doxycycline (100-200 mg/day for 4-6 weeks) has been used to treat filariasis. It acts indirectly by kil ling Wolbachia which are obligate symbiotic gram negative bacteria that are essential for adult Wbancrofli and 0 . volvu/us worms.

Niclosamide N iclosarnide is a highly effective drug against cestodes infesting man- Ta enia saginata, T solium, Diphyllobothrium latum a nd Hymenolepis nana, as we ll as pin worm (Enterobius), but is infrequently used now due to availability of praziquante l. The drug appears to act by inhibi ting oxidative phosphorylation in mitochondria and interfering with anaerobic generation of ATP by the tapewom,. Injured by niclosamide, the tapeworms are partly d igested in the intestine. In cases of T solium, digestion of the dead segments can be hazardous, because the ova re leased from them may develop into larvae in the intestine, penetrate its wall and cause visceral cysticercosis. Though, the magnitude of such risk is uncertain , many experts d o not use ni c losamide no w for T solium infestation. Regimen for tapeworm Niclosarn ide is available as 0.5 g tab (N IC LOSAN). After a light breakfast, 2 tablets are to be chewed and swallowed with water, followed by another 2 tablets a fter I hr (total 2 g); total dose for childre n 2-{i years is I g. A saline purge is given 2 hours after the late r dose to wash off the worm. Tbe scolex shou ld be searched in the stools to be sure that the worm will not grow again. C ure rate of 85-95% has been obtained by one day treatment. A thorough purge is essential in the

cases of T. solium so that all segments are passed out and cysticercosis does not occur. Because praziquantel does not lead to digestion of the worm and kills encysted larvae as well, it is the drug of choice for T. soliwn. For H. nana, the 2 g dose is repeated daily for 5 days. This is needed because cysticerci of H. nana (which are not affected by niclosarnide) develop in the jejuna! villi of the same host and worms appear in the intestinal lumen afte r 4 days. However, no purgative is required. In some cases treatment may have to be repeated after JO days. Praziquantel is now preferred due to single dose treatment.

Adverse effects N iclosa mide is tasteless and nonirri tati ng. It is min imally absorbed from g.i.t.- no systemic toxicity occurs. It is well tolerated; minor abdominal symptoms are produce d occasionally. Malaise, pruritus and light headedness are rare. Niclosamide is safe during pregnancy .and in poor heal th patients. Praziquantel Ibis anthelmintic has w ide ranging acti vity against Schistosomes, other trematodes, cestodes and their larval forms but not nematodes. It is rapidly taken up by susceptible worms and appears to act by causing leakage of intracellu lar calcium from the membranes producing contracture and paralysis of the worms. Selectiv ity of action of praz iquantel on tapeworms and flukes may be dependent on the presence of a specific variant of Ca2+ channel sensitive to praziquantel in these worms. The tapeworms lose grip of the intestinal mucosa and are expelled. Flukes and schistosomes are a lso dislodged in tissues and ve ins . Praziquantel is act ive against adult as well as juvenile and larval stages o f tapeworms. At relatively higher concentrations, it cau ses vacuolization of the tegument and release of the contents of tapeworms and flukes followed by their destruction by immune mechani sms of the host. Th is ac tion appears to be more important in cases of schistosomes and flu kes.

Pharmacokinetics Praziq ua ntel is rapidly absorbed from intestines; absorption is enhanced if it is ingested with food. Hi gh first pass metabolism in liver limits its systemic bioavailability. Phenytoin, carbamazepine and dexamethasone induce p raziquantel metabolism and further

ANTHELM INTIC DRUGS

decrease its bioavailability. Patients of neurocysticercosis are mostly receiving these drugs which may contribute to therapeutic failure of praziqua ntel. It crosses blood-brain barrier and attains therapeutic concentrations in the brain and CS F. The plasma t½ is short ( 1.5 hours). Metabolites are excreted chiefl y in urine.

Adverse effects Desp ite systemic absorption, praziquantel has ex hibited no systemic toxicity. It tastes bitter: can produce nausea and abdom ina l pain. Other side effects are headache, dizziness and malaise. When used for schistosomes and viscera l flukes, symptoms like itch ing, urticaria, rashes, fever and bodyache occur as a reacti on to the destroyed parasites. Destruction of cysticerci in the brain may produce neurological compli cati ons (see below). No interaction with food, a lcoho l or tobacco has been noted. Uses I . Tapeworms: Praziquantel admin istered as a single dose has achieved 90-100% cure rate in all human tapeworms. This level of efficacy is simi lar to or better than that of niclosam ide, particularly in case of H. nana.

T saginata, T solium: IO mg/kg single dose in the morni ng. It is especially valuable in case of T solium, because it kills the tapeworm larvae wi thin the cysts and there are no chances of systemic cysticercosis developing. H. nana, D. /alum: 15-25 mg/kg single dose in the morn ing. T his is much simpler compared to 5 day treatment needed with niclosamide for eradication of H. nana. In case of heavy infestation, retreatment after one week is desirable.

2 . Ne11roc:rsticercosis: The ro le of anthelminti cs in this condition is controversial; only selected cases should be treated w ith the m. Praziquantel was the fi rst drug found to be effective in ne urocyst icercosis: 50 mg/ kg da ily in 3 divided doses for 15- 30 days kills the larvae lodged in brain and other tissues. However, it is now the 2nd choice drug to albendazole (see below).

Praziquante l or a lbendazole are contraindicated in ocular cysticercosis because reaction to the dead cysticerc i may lead to blindness. 3. Schistosomes: All 3 species can be treated wi th 40- 75 mg/kg given once or in instal me nts over one day.

4. Other fluke.\: Praziquante l is the drug of choice for all schistosome and fl uke infestations except Fasciola hepatica. The flukes respond to 75 mg/kg single day treatment in most cases, and on two occasions in the remaining. CYSTICIDE 500 mg lab, DI STOCIDE 600 mg lab.

Anthelmintic treatment of neurocysticercosis Cysticercosis of various organs, including brain, occurs in r solium infestation due to migration of the larvae from the gut to various tissues via bloodstream. Anthelmintic treatment of neurocysticercosis may or may not be appropriate, because the cysts do not cause any problem unless the larva dies and its products induce an intense focal reaction. The anthelmintic ki lls the larvae and precipitates the reaction, resulting in meningeal irritation, rise in intracranial pressure, seizures and other neurological phenomena. However, it prevents future episodes due to spontaneous death of the cysticerci. The decision whether or not to give the anthel mintic may be taken depending on the number, location and viab ility of the cysts. Active cysts, multiple parenchymal cysts, or intraventricu lar cysts likely to enlarge and cause hydrocephalus are better treated. Inactive and calcified cysts may be left alone. Out of the two anthel m intics effective in kill ing cysticerci, albendazole is now preferred over praziquantel for the following reasons: • The course of treatment is shorte r (8-15 day) compared to praziquanrel ( 15- 30 days). • Cure rates in terms of resolution of symptoms and disappearance of cysts are higher (75-85% with a lbendazole) than praziquantel (50- 60%). • Corticosteroids (which have to be given concurrently) e nh ance the absorption of

913

914

AN T IMICROB IAL DRUGS

albendazole, but lower the blood levels of praziquantel. Phenytoin and carbamazepine al so lower praziquantel levels. • A lbendazole is cheaper. Whichever anthelmintic is used, corti costeroids (prednisol one 40-60 mg/day or dexamethasone 8- 12 mg/day) must be started 2 days before and continued till 2 weeks after completing the anthelmintic course. This is necessary t• suppress th e inflammatory reacti on to the products of killed larvae. A bsorption of both albendazole and praziquantel i s enhanced by ingesting th em with food, particularly fatty food. For patients w ith seizures (as most of th em are),

50% patie nts relapse on discontinuat ion. Systemic absorption and toxicity can occur if it is applied to inflamed or damaged skin. Some ind ividuals develop sensitivity reactions. SELSUN 2.5% susp.. SELDRUFr PLUS 2.5% sus p. with clomma7ole t%.

Zinc pyrithione It reduces e pidermal turnover a nd inhibits P ovale. Weekl y shampoo ( 1%)

949

950

MISCE LLANEOUS DRUGS

Methoxsalen

reduces dandru !T, but symptoms do not resolve co mpletely. It is often combined with kctoconazole.

solution. MALANOCYL 10 mg tab, 0.75% soln.)

SCALPE: Zinc pyrithione 1%, Ketocona,olc 2% shampoo.

0.2% lotion)

Corticosteroids

These agents sensitize the skin 10 sunlight which then induces erythema. inflam mation and pigmentation. They are applied topically as well as given orally. Methoxsalcn is absorbed belier, undergoes less first pass metabolism and is more effective than trioxsalcn. Their plasma 1½ is short (- I hr): ~ensitization of skin is maximal at I 2 hours, but lasts for 8 hours or more.

Massaged in the scalp as a lotion, topical steroids arc highly e !Tective in relieving symptoms of seborrhe ic dermat itis including dandruff. Pityrosporal yeasts are reduced in the affected skin. However. relapse rates arc high on discontinuation and prolonged use can produce adverse effects like atrophy, poor healing, purpura, etc.

lmidazole antifungals Among several of these compounds, kctoconazolc ( KTZ) was found to be the most effective against P ovate. Orally (200 mg/day for 4 weeks) it has been found to improve seborrhoea. But because this is often a chronic relapsing condition and prolonged oral KTZ therapy is considered unwarranted, KTZ has been fo m1ul ated into 2% cream/shampoo/ scalp gel. Good to excellent results have been obtai ne d with these preparatio ns without skin irritation, contact sensitivity, phototoxicity or systemic adverse etTects. KETOVATC, '\/IZRAL, OCO A 2% cream, 2% shampoo.

Clotrimazole 1% solution may be used in its place. Sulfur, Resorcinol, Coaltar, Ammoniated mercury These drugs are mildly efTectiv.:. They have minimal antiyeast action: may benefit seborrhoea hy keratolytic and a111iscp1ic propc11 ics.

Salicylic acid It is keratolytic, has mild effect in seborrhoea, probably by removing the scales and by improving penetration of other drugs.

MELANIZING AGENTS Me lan iz in g agents are drugs that increase sen sitivity to so lar radiation and p romote repigmentat ion of vi til iginous areas o f skin. Psoralcns arc furocoumarins which on photoactivation stimulate me lanocytes and induce their pro Ii fera tion.

Psoralen majus.

It is obtained from fruit of Ammi

\1A 'ADER\ 1 10 mg tablet. 1% omtment, PSORLr.-JC 5 mg tablet. 0.25% »olution and ointment.

Trioxsafen

( MACSO RALEN 10 mg tab, 1%

and

(NEOSORALEN 5 mg, 25 mg tablets and

arc synthetic psoralcns.

Topical the rapy The solution/ointment is carefully painted on the ~mall well defined vitiliginous lesion which is then exposed to sunlight for I minute and then occluded by bandage or sun screen ointme111. Weekly treatment with longer exposures is given. Pigmcntntion usually begins lo appear afler a few weeks; months are needed for satisfactory results. Then periodic maintenance treatment may be needed. This therapy should be undcnaken only under direct supervision of physician because longer exposure causes buming and blistering. Oral therapy On alternate days after 2 hours of a 0.3-0.6 mg/kg (usually 20 mg) oral dose of u psoralen. skin is ex posed to s unlight (or artificial UV hgh1), initially for 15 minutes- gradually increasing to 30 minutes over days. Eyes, lips and other normally pigmented areas should be protected during exposure to sunlight.

DRUGS FOR PSORIASIS Psoriasis is an immunological disorder manifest ing as localized or w idespread erythematous scaling lesions or plaques. There is excessive ep idermal prol ifera ti on attended by dermal infla mmati on. Periodic fla re ups are common. Drugs can diminish the lesions, but can not cure the disease. Therapy has to be prolonged and adj usted to the body surface area involved as well as to the severity of disease. Topically applied emollients, keratolytics. antifungals afford variable symptomatic re lief, but topical corticoste,vids are the primary drugs used. They are very effective in mild-to-moderate disease. and ini tia ll y even in severe cases. Most patients respond within 3 weeks, and the response may be hastened by applying the steroid under occlusion. Therapy is started w ith a high potency steroid which is substituted aner improvement by either weekly application or by a milder preparation. I lowever, topical steroids carry their

DRU GS ACTING O N SKIN AND M UCOU S M EMBRAN ES

ow n local and systemic adverse effects, and lesions may progressively become refractory. Syste mi c therapy with corticosteroids and/or immunosuppressants is reserved for severe a nd refractory cases. Other topically used drugs are:

Calcipotriol

It is a synthetic nonhypercalcaemi c vit D analogue effective topicall y in plaque ty pe psoriasis. C alc ipotrio l bi nd s to the intracell ula r vit D receptor in epidermal keratinocytes and suppresses the ir proliferation while enhancing diffe rentiation. On absorption through the skin, it is inact ivated rapid ly by me tabolism so that little systemic effec t on ca lc ium metaboli sm is e xe rted . Benefi t in psoriasis is slow; but most cases respond in 4-8 weeks. Response is maintained til l treatment is continued. Efficacy of calcipotriol in p soriasis is rated co mparable to a moderate potency topical steroid. Co mbination w ith a steroid is more effecti ve than ei ther drug alone. Side effects are skin irritation, erythema and scaling. Hypercalcaemia is rare. It is a safe and effective alternative to steroids, but expensive. DAIVONEX 0 .005% oint; apply over psoriatic lesions twice daily.

Tazarotene This synthetic retinoid applied as a topical gel (0.05-0. 1%) is moderately eJTective in psoriasis. It is a prodrug which is hydrolysed in the skin to tc7arotcnic acid that exerts antiproliferative and antiinflammatory action by binding to the intracellular retinoic acid receptor and modification of gene functio n. Combination with a topical steroid/calcipotriol may bene fit refractory cases. Skin irritation, burning sensation, peeling are common. These side e ffects can be min imized by careful application to the plaques only. It is teratogenic. LATEZ 0 .05%, 0. 1% gel ; TAZRET 0.05% gel, 0. 1% cream; 0.05-0.1% application once daily in the evening. Coaltar This crude preparation containing many phenolic compounds exerts a phototoxic action oa the skin when exposed to light, especially UVA, and retards epidermal turnover. Applied as ointment or alcoholic solution on psoriatic plaques (generally with sa licylic acid) and exposed to sunlight daily, it induces resolution of psoriatic lesions in majority o f cases, but relapses arc common. Use of coaltor has declined now because of strong smell, cosmetic unacceptabil ity, skin irritation, a llergy, and potential for photosensitivity and carcinogenicity. EXETAR: coaltar 6%, salicylic acid 3%, sul fur ppt. 3%. o int. TARSYL : coaltar 1%, salicylic acid 3% lotion. IONAX-T: coaltar 4.25%, salicylic acid 2% scalp lotion.

Photochemotherapy (PUV A: Psoralen ultraviolet A) Photoactivated pso ra len undergoes 0 2 independent as well as 0 2 dependent reactions and binds to pyrimidine bases, wh ich interfere with D A synth esis and epithelial cell turnover. PUVA th erapy has produced gratifying res ults in severe ly debilitating p soriasis, but relapses occur when treatment is stopped. Oral methoxasalen is followed 1- 2 hours la te r by UVA expos ure on alternate days. However, there are serious concerns regarding potential of P UVA to cau se skin cancer, cataracts and immunological damage. Being inconvenient and carrying risks, it is reserved for severe cases or extensive surface area of psoriasis only. Psoralens have also been used to accelerate tann ing- a max imum o f 2 weeks treatment has been adv ised for this purpose. Other applicat ions of PUVA are in lichen planus, urticaria pigmentosa, atop ic dermatitis and cutaneous T cell lymphoma.

Adverse effects: Mottling, ery the ma, burns, blister ing, premature agei ng of skin , gastr ic discomfort, nervousness and inso mnia. Acitretin It is a synthetic retinoid for oral use in psoriasis, lichen planus, severe ichthyosis, etc. Acitretin acts by binding to 'retinoic acid receptor' in epidermal cells and regulating their proliferation and maturation. Inflammation is suppressed. Because of frequent and potentia lly serious adverse effects, use of acitretin is restricted to recalcitrant, pustular and other forms of severe psoriasis. Combination with topical antipsoriatic drugs is advised. Dose: 0.5- 0.75 mg/kg/day oral; ACIT RIN, ACETEC, ACERET 10, 25 mg tab. Dryness ofskin and eyes, gingivitis, erythema and scaling of skin, alopecia, arthralgia, myalgia, lipid abnorn1alities and liver damage are the important adverse effects. Elimi nation of acitretin is very slow (taking months) because of accumulation in body fat. It is high ly teratogen ic. Women taking acitretin must not conceive during and ti ll 3 years after stopping it. Drinking alcohol should be prohibited during a• d till 3 months after acitretin use. £tanercept, lnfliximab

These arc TN Fa inhibitor i mmuno-

suppressive drugs, also used in psoriasis. They are described in Ch. 15 and Ch. 65.

DEMELANIZING AGENTS These drugs lighten hyperpigmented patches on skin.

951

952

M ISCE LLAN EOUS DRUGS

Hydroquinone It is a weak hypopigmenting agent. It in hibits tyrosinase and other melanin forming enzymes, decreases formation of and increases degradation of melanosomes. Regular application (as 2-6% lotion or cream) for months is required in melasma, chloasma of pregnancy, etc. The response is often incomplete and pigmentation may recur when it is discontinued, esp ecially if exposed to sunlight; sunscreens are freq ue ntl y combin ed with hydroq uin one. Skin irritation, rashes and allergy are possible. Care is to be taken to avoid its entry in eyes. EUKROMA4% cream, MELALITE: Hydroquinone 2% with glycery lester of PABA 2 .8% cream. BRITE: hydroquinonc 4%, glyceryl PABA 2.8% cream.

Monobenzone This deri vative of hydroquinone is a potent demelanizing agent- 5%) cause b urn in g and bl isteri n g of ski n . Tincture iodine (2% in a lcoh o l) stings on abrasions.

These are s o lub le complexes of iodine with large molecular organic compound s that serve as carriers- re lease free iodine s lowly.

The most popu lar is:

Povidone (Polyvinylpyrrolidone) iodine: It is nonirritating, nontox ic, nonstaining and exerts prolonged germicidal action. Treated areas can be bandaged or occluded w ithout risk of blis tering. It is used on boils, furunculosis, bums, otitis extema, u lcers. tinea, monilial/trichomonal/ nonspecific vaginitis and for s urgica l scrubbing, disi nfection of e ndoscopes and instruments. BETADI E 5% ,olution. 5% omtment. 7.5% scrub solution, 200 mg vaginal pessary; PIODIN 10% solution, 10% cream, 1% mouthwash; RANVIDONJ:. AEROSOL 5% spra) with freon prupellant. Chlorine A highly reactive element and a rapidly acting potent germicide. 0. 1--0.25 ppm kills most pathogens (but not M 111berc11/osis) in 30 sec. I Im, ever, the dcgcrming action is soon exhausted, and it lacks substantivity. It is used to disinfect urban "ater supplies. Organic mailer binds ch lorine, so that excess has to be added to obtain free chlorine concentration of 0.2-0.4 ppm. This i, known as the 'chlorine d~mand' of water. Chlorine is more active in acidic or 111:utral medium. Chlorophores These arc compounds that slowly release hypochlorous acid ( HOCI). Decause or case of handling. they are used in preference 10 gaseous chlorine.

(i) Chlorinated lime (bleaching powder) It is obtained by the action of chlorine on hme (CaO); resulting in a mixture of calcium chloride and calcium hypoch loritc. On exposure. it decomposes releasing 30- 35% W/ W chlorine. Chlorinated lime is used as disinfectant for drinking water, S\\ imming pools and sanitiLer for privies, etc. (ii) Sodium hypochlorite solution Contains 4- 6% sodium hypochlorite. that is a powerful disinfectant used in dairies fo r milk cans, other equipment and for infant feeding bottles. It is unstable and too irritant to be used as antiseptic, except for root cana l therapy in dentistry.

It is used on cuts, for degermin g s kin

before surgery, and to treat ring worm.

4. BIGUANIDE

Mandel 's paint ( 1.25% iodi ne d issolved w ith Lhe

Chlorhexi d ine A p owerful , non irr itat in g,

help of P ot. iodide forming soluble !~ -ions) is

cationic antiseptic that d isrupts bacteria l cel l membrane. A secondary action is denaturat io n of microbia l prote ins. It is relatively more acti ve against gram-posit ive bacte ria . L ike hexachl orophe ne, chlorh exidine per is ts on the s kin. P resent in SAVLON (see below), it is exten s ively u sed

applied on sore throat.

A nonstaining iodine ointment (IODEX 4%) is po p u lar as antiseptic and cou nterirritant. Some individ uals a re sensit ive to iodine-

rashes and

system ic mani festa tio ns occu r in them.

959

960

M ISC ELLANEO US DRUGS fo r surgical scrub, neonatal bath, mouthwash, obstetrics and as general skin antiseptic. Chlorhexidine is the most widely employed a ntiseptic in dent istry. As 0. 12- 0.2% oral rinse or 0.5- 1% toothpaste, it is highly active in preventing/treating g ing ivitis. Twice daily chlorhexidine oral rinse markedly reduces oral infectio ns in immunocompromised patients, inc luding AIDS. Ho we ver, it may leave an unpleasant after taste, and repeated application causes brownish discolourati on of teeth.

5. QUATERNARY AMMONIUM (CATIONIC) ANTISEPTICS T hese are detergents; cidal to bacteria, rungi and v iruses. However, many gram-negative bacte ri a (es peciall y Pseudomonas), M. tu berculosis and bacteri al spores are relatively resistant. Cationic antiseptics act by altering permea bility of cell membranes and denaturing o r bacter ial prote ins. Soaps, be ing anionic, neutra lize the ir action, w hile a lcohol pote ntiates. They sp read th rough o iI and g rease, ha ve c le an s in g and emu lge nt propert ies. They are nonirritating and mildl y keratolytic. However, the germicida l action is rather s lo w and bacteria may thri ve under a fi lm formed by them on the skin. Pus, debris and porous material like cotto n, polyethy lene reduce their ac ti vity. Occasionall y sens iti za tion occurs. These disadvantages no t withstanding, they are wide ly used as sani tizers, antiseptic and d isinfectant fo r su rgical instruments, gloves. etc, but should not be considered sterilizing.

Cetrimide It is a soapy powder with a fa int fi shy odo ur. Used as I 3% soluLi on. cctrimide has good cleans ing action. efficiently remo ving d irt, grease, ta r and congea led blood from roa d s id e accident wou nds. A lo ne or in combination with chlorhexidine, it is one of the most popular hospital antiseptic and dis infectant for surgical instruments, utensils, baths, etc. CETAVLON CONCENTRATE: Cctrimide 20% SAVLON LIQUID ANTISEPTIC: Chlorhexidine gluconate 1.5% + Cetrimide 3%. SAVLON/CETAVLEX CREMt : Chlorhcxidinc HCI 0.1% .._ C~trimide 0.5%.

SAVLON HOSPITAL CONCENTRATE: C hlorhexidine gluconate 7.5% + Cetrimide 15%.

Benzalkonium chloride (Zephlran) It is highly soluble in water and alcohol. A I: IOOO solution is used for sterile storage of instruments and I in 5000 to I in I 0,000 for douches, irrigation, etc.

Dequalinium chloride Has been used in gum paints and lozenges. DEQUADIN 0.25 mg lozenges.

6. SOAPS Soaps are anionic detergents with weak ant iseptic action. They affect only gram-positive bacteria. Their usefulness primarily res ides in 1heir cleansing action. Wash ing with soap and warm water is one of the most e nective methods of preventing trans mission of infection by removing or di luting pathogenic bacteria. Soaps can be medicated hy 01hcr antiseptics.

7. ALCOHOLS Ethanol It is an effecti ve an tiseptic and cleansing agent at 40- 90% concentration. The ra pidity of acLio n increases with concentration upto 70% and decreases above 90%. IL acts by precipitating bacteri al proteins. A cotton swab soaked in 70% ethanol ru bbed on th e skin ki lls 90% bacte ria in 2 min. T his has been used before hy podermi c injection and on minor cuts. Alcohol-based hand rubs are recommended fo r decontamination of hands in the health-care setting, as we ll as for personal hyg iene. Low concentrations enhance the antiseptic acti vity of iodine and chlorhexidine when used as solvent for these. Alcohol is an irri tant and should not be applied to muco us membranes o r to de licate skin (scrotum ), ulcers, etc. On open wounds it prod uces a burning sensatio n, inj ures the surface and forms a coagulu m unde r which bacteria cou ld grow. It is a poor disinfectant for ins trum e nts- does no t ki 11 spo res a nd promotes rusting. lsopropanol It is less volatile; can be used in place of ethanol. 8. ALDEHYDE$

Formaldehyde It is a pungent gas- sometimes used for fumigation. A 37% aqueous solu1ion called Formalin is diluted to 4% and used for hardening and preserving dead

ANT ISEPTICS, DISINFECTANTS AND ECTOPARASITI CIDES tissues. Fonnaldehyde denatures proteins and is a general protoplasmic poison, but acts slowly. It is a broad-spectrum germicide, but use as antiseptic is restricted by its irritating nature and pungent odour. It is occasionally employed to disinfect instruments and excreta. Those who handle forma lin can develop ec£ematoid reactions. The urinary antiseptic methenaminc acts by releasing forma ldehyde in acidic urine (see p. 8 10). Formaline is a lso used to precipitate toxoids from toxins.

Glutaraldehyde It is less volatile, less pungent, less irritating and better sterili£ing agent than formali n, but needs to be activated by alkalinization o f the solution. Glutaraldehydc exerts broad-spectrum activity against bacteria, fungi and viruses. Organic matter does not inactivate it. A 2% solution is used to disinfect surgical instruments and endoscopes, but prolonged contact is needed. Repeated application on skin can cause sensitization. The alkalinized solution has a short shel f life (2 weeks) unless stabili£ing agents are added. 9. ACIDS Boric acid It is o nly bacteriostatic and a very weak antiseptic. But being nonirritating even to delicate structures, saturated aqueous soluuons (4%) have been used for irrigating eyes and as mouthwash, douche, etc. Boroglycerine paint (30%) is used for stomutitis and glossitis. A I 0% ointment (BOROCIDE) is availab le for cuts and abrasion. Boric acid is included in prickly heat powders and ear drops. However. it is not innocuous; systemic absorption causes vomiting. abdominal pain, diarrhoea, visual disturbances and kidney damage. I lence its use fo r irrigating bladder. large wounds and as ointment on extensive burnt areas, liberal use of the powder for infants is not recommended. 10. METALLIC SALTS Silver compounds These are astringent and caustic. They react with SH. COOH. PO, and N H, groups of proteins.

(i) Silver mtrate It rapidly kills microbes, action persisting for long periods because of slow release of Ag ions from silver proteinate formed by interaction with tissue proteins. Tissues get stained black due to deposition of reduced silver. Silver nitrate touch is used for hypertrophied tonsillitis and aphthous ulcers. It i, highly active against gonococci I% solution is used for ophthalmia neonatorum. (ii) Silver sulfadiazine (see p. 757) It is high ly active against Pseudomonar and has been used on bums. Zinc salts They are as1ri ngent and mild antisep1ics.

(i) Zinc sulfate: It is high ly water soluble, 0.1 - 1% is used for eyewash and in eye/car drops (Zinc-boric ac id drops- in ZINCO-SULFJ\ 0. 1% eye drop). Applied to skin, it decreases perspiration. White lotion containing 4% each of zinc sulfate and sulfu rated potash has been used for acne and impetigo; (THIOSOL 2.5%, THIOSOL fORTE 4% lotion).

(ii) Calamine and zinc oxide: T hese are insoluble. In addition to being mildly antiseptic, they are popular dermal protectives and adsorbants. 11 . DYES Gentian violet (crystal violet) A rosanil ine dye active against staphylococci. other gram-positive bacteria and fungi, but gram-negative organisms and mycobacteria arc insensitive. Aqueous or alcoholic solution (0.5-1%) has been used on furunculosis, bedsores, chronic ulcers, infected eczema, thrush, Vincent's angina, ringworm, etc. However, it has become unpopular due to deep staining. Acriflavine and Proflavine These are orange-yellow acridine dyes active against gram-positive bacteria and gonococci. Their efficacy is not reduced by organic matter and is enhanced in alkaline medium. Solutions lose efficacy on exposure to light; therefore needs to be stored in amber bottles. They are nonirritant, do not retard healing, and arc particularly suitable for chronic ulcers and wounds. Bandage impregnated with acriflavine-vaseline is used for bum dressing; ACRl'1 episode per day, he must be put on regular prophylactic antiinflammatory drug therapy. His present treatment with short acting P2 agonist (salbutamol) inhalation alone is not adequate for his condition. The first line treatment of this patient would be a low-dose inhaled corticosteroid + a long acting P2 agonist on a regular twice daily schedule. For example, he may be prescribed- Fluticasone 125 µg + salmeterol 25 µg per puff inhaler, one puff to be taken twice daily. A medium dose of the inhaled corticosteroid (Fluticasone 125 µg per puff, 2 puffs twice daily) alone may also be tried if prolonged use of long-acting p2 agonist is to be avoided. b. The patient should continue to use the short acting p2 agonist (salbutamol) inhalation to abort/ terminate each episode of asthma when it occurs despite instituting prophylactic therapy. c. Since no other drug is as effective prophylactic as a corticosteroid, it should not be replaced in this case of medium severity bronchial asthma. However, the long acting p2 agonist can be replaced by a leukotriene antagonist, like Montelukast 10 mg 1 tab daily in the evening. However, some patients may not benefit from it. As a third option, sustained release theophylline 400 mg tab OD may be tried, with dose titration over time, as add on drug to the low dose inhaled corticosteroid.

SOLUTION 16.2 a. The recently developed symptoms of the patient are indicative of early stage theophylline toxicity. Erythromycin is an inhibitor of several hepatic microsomal enzymes, including those that metabolize theophylline. As such, when the patient took erythromycin, metabolism of theophylline appears to have been retarded, causing rise in its plasma concentration over t he next 2 days and producing overdose symptoms. b. This complication could have been prevented in two ways, viz. i. When erythromycin was prescribed, the daily dose of theophylline shou ld have been reduced from 800 mg to 500 mg, and maintained at this level till the patient was taking erythromycin. Or ii. An alternative antibiotic (e.g. a P-lactam like amoxicillin or cephalexin) which does not inhibit theophylline metabolism but is effective in sore throat, could have been selected for this patient.

1001

1002

APPEND ICES SOLUTION 18.1 a. Since carbimazole inhibits further synt hesis of thyroid hormones (T3, T4 ) without affecting their release or action, the hormone stored in the gland continues to be released and produce effects. Moreover, thyroxine has a long plasma t½ of 6-7 days. Thus, the effect of carbimazole starts manifesting only after 2-3 weeks and peaks after 2-3 months. Many of the symptoms of thyrotoxicosis are due to sympathetic overactivity. Blockade of p adrenergic receptors (~1 and p2) by propranolol or similar drug affords rapid symptomatic reli ef, without affecting thyroid status. A nonselective P-blocker given to her along with carbimazole could have controlled palpitation, tremor, etc. w ithin a few days. This drug could be withdrawn when carbimazole had taken effect. b. The reappearance of neck swelling without any symptom of thyrotoxicosis indicates that it is due to deficient feedback inhibition of TSH by a suboptimal thyroid hormone level as a result of higher maintenance dose of carbimazole. This is supported by the mild hypothyroid symptoms experienced by the patient and the raised TSH level alongwith low normal FT4 level. The raised TSH is stimulating the thyroid so that despite its low functional status, deficiency is not marked. Since the disease activity in Graves' disease may decline after some time, the maintenance dose of carbimazole needs to be adjusted from time-to-time according to the assessed clinical and laboratory thyroid status of the patient. This patient requires temporary discontinuation of carbimazole followed by a lower maintenance dose as assessed later.

SOLUTION 19.1 a. According to the current recommendation of professional guidelines, the patient should be prescribed metformin therapy concurrently with dietary and lifestyle measures. This is based on the finding that metformin can delay progression of diabetes and prevent microvascular as well as macrovascular (heart attack, stroke) complications. Metformin does not increase circulating insulin, reduces insulin resistance, is unlikely to induce hypoglycaemia and may have a positive influence on pancreatic p cell health. Lack of serious toxicity over several decades of use of metformin is well established. No other antidiabetic drug has all these favourable features, and therefore, it is considered the first-choice drug. Metformin is particularly suitable for this patient who is overweight, because it can help weight reduction. A combination of antidiabetic drugs is not indicated at this stage. Another drug needs to be added only when the target blood glucose and HbA1, levels are not attained by metformin alone.

SOLUTION 20.1 a. The patient has received supraphysiological doses of a corticosteroid for more than 3 weeks, and is likely to have developed hypothalamo-pituitary-adrenal (HPA) suppression. The injury and surgery are a stress which need excess corticoid activity. The depressed HRA axis may not be able to cope up with increased demand, and there is risk of developing acute adrenal insufficiency. As such, hydrocortisone hemisuccinate 100 mg should be infused i.v. during surgery and repeated 8 hourly till the patient is stable. b. Prednisolone therapy must not be stopped in the postoperative period apprehending spread of infection and delayed healing. Effective antibiotic medication to prevent wound infection should be given and prednisolone dose should be increased temporarily (for a week or so) to 20 mg/ day, till the stress of th e t rauma and surgery subsides.

1003

SOLUTION TO PROBLEM DIR ECTED STUDY SOLUTION 21.1 a. Th is is a case of advanced metastatic prostate carcinoma, fo r w hich only pa lliative therapy with androgen depr ivation (tumour cells remain androgen dependent ) is possible. When orchidectomy has been refused, the most effective method of androgen deprivation is to give a long acti ng GnRH agonist. Thus, t he choice of triptorelin is correct. The GnRH agonists initially increase LH (also FSH) release for 1-2 weeks, fol lowed by receptor desensitization and nearly total blockade of LH secretion by 3-4 weeks. The raised LH levels in the beginning stimulate testis to secrete more testosterone which activates t umour cells resulting in increased bone pain and bladder obstruction noticed after 1 week of therapy in this patient. b. The initial fla ring of symptoms can be avoided by pretreating with an antiandrogen bicalutamide 50 mg orally daily for 3 days before st arting triptorelin injection and then continuing both drugs together. The stimulatory effect of excess testosterone on tumour cells would be blocked by bicalutamide so that no flaring of symptoms would occur. The combined androgen blockade with GnRH agonist + androgen antagonist is the favoured approach. c. The patient can be given an antiresorptive drug in addition to relieve bone pain. A potent parenteral bisphosphonate like zoledronate infused i.v. over 15 min every 1-4 weeks is the most effective drug for this purpose. It may also retard growth of the bony metastasis for some time.

SOLUTION 22.1 a. The most likely cause of endometrial thickening in this patient is tamoxifen therapy. Tamoxifen is a selective estrogen receptor modulator (SERM) which has estrogen antagonistic action in th e breast (basis of its use in breast carcinoma), but agonistic action on the endometrium which stimulates proliferation. Such unopposed (by progestin) hyperproliferation can produce thickening and predisposes t o endometrial carcinoma. b. For the reason stated above, tamoxifen should not be continued in this patient. Total stoppage of adjuvant therapy is not advisable, because estrogen suppression therapy has been shown to exert protective effect for at least 5 years. Aromatase inhibitors, which block synthesis of estrogens in the body, have been clearly demonstrated to prevent recurrence of breast cancer, without stimulating endometrial prol iferation or predisposing to endometrial carcinoma. Therefore, in thi s case, tamoxifen should be replaced by letrozole 2.5 mg /day or anastrozole 1.0 mg/day for the next 5 years. Due precautions to prevent osteoporosis and measu res to address arthritic symptoms, if they develop, should be taken concurrently.

SOLUTION 22.2 a. Al l diseases and conditions which contraindicate use of oral contraceptives or need caution in th eir use have to be ruled out before prescribing one to this subject. Full medical history, including menstrual history and past pregnancy details should be elicited. Any thromboembolic episode, j aundice or toxaemia of pregnancy should be ascertained. History of smoking, diabetes, hypertension, migraine, tuberculosis and gallbladder disease should be specifically asked. Any medication that she is taking and the reason for it should be taken into account to foresee possible interactions with the contraceptive. Whether she is obese or very lean also matters in selecting the contraceptive preparation. General physical examination, including palpation of breast, for any lump and a per vaginum examination fo r fibroid/other tumour, should be done. Blood pressure should be recorded to rule out hypertension. Fasting and postprandial blood glucose,

Contd...

1004

A PPENDI CES lipid profile should be ordered to detect diabet es and dyslipidaemia. Ultrasound examination of pelvic organs should be perfo rmed for uterus size, fibro id, ovarian cyst or malignancy. Only after all the above find ings are favourable that an oral contraceptive be selected and prescribed.

SOLUTION 23.1 a. Though t he progress of labour in t his case is tardy and uterine contractions are relatively weak, there are signs of foetal distress (passage of meconium stained liquor, rapid foeta l heart becoming irregular during uterine contraction). Moreover, t he mother is dehydrated and exhaust ed. As such, t he best course of action is to deliver t he baby by caesarean section. b. The mother should not be administered an oxytocic drug, because stronger uterine contractions are likely to worsen foetal distress and pose risk to t he baby. The mother is also not in a fit conditi on to endure t he stress of a difficult labour.

SOLUTION 25.1 a. Rocuronium is t he preferred muscle relaxant for t racheal intubation and short lasting muscle relaxation in t his patient . Succinylcholine (SCh), t he fastest and shortest acting muscle relaxant which is most commonly used for aiding tracheal intubation, is not suitable for this patient, because it is a depolarizing blocker and releases K• from skeletal muscles. Since this patient has extensive burns and tissue injury, which itself causes hyperkalemia due to leakage of K• from injured cells, t he K• released by SCh will accentuate t he hyperkalemia and expose the patient to risk of cardiac arrhythmias and other complications. Rocuronium, on the other hand, is a nondepolarizing blocker which does not trigger loss of intracellu lar K•. It is the fastest acting nondepolarizing blocker wi t h speed of action approaching that of SCh. Intubating conditions can be obtained in 60- 90 sec. It also provides surgical grade relaxation for 25-40 min, along with good cardiovascular stability.

SOLUTION 26.1 a. Labour pain as well as that due to stretching of the birth canal can be largely relieved by spinal as well as epidural anaesthesia. It is desirable, at the same time, not to produce motor block so t hat the mother can actively participate in the process of labour. Since motor fibres are less sensitive to local anaesthetics (LAs) than sensory fibres, motor block of a lower level is usually produced during spinal anaesthesia. Such separation is more pronounced with epidural anaesthesia. Lidocaine and bupivacaine are the two LAs commonly used for epidural anaesthesia. Out of t hese, bupivacaine is more suitable for this purpose for the following reasons: • It provides greater separation of sensory from motor block. Separation is still larger when lower concentration {0.25% bupivacaine) is used. • Because of higher li pid solubility, its tissue distribution is large and maternal blood levels are lower. Less drug is likely to cross to the foetus, reducing chances of neonat al depression. • It is longer acting. Thus, epidural anaesthesia with 0.25% bupivacaine is most suitable for this patient.

1005

SOLUTIO N TO PROBLEM DI RECTED STUDY SOLUTION 28.1 a. Alco hol exerts an t iconvulsant action w hile it s concentration in the brai n is ri sing or is maintained. This is followed by lowering of seizure threshold when the concentration fa lls and becomes zero. Thus, recurrence of seizures in this patient could most likely be due to the temporarily increased susceptibilit y to seizures caused by withdrawal of alcohol from the brai n. b. Since this lowering of seizure threshold is a short-term problem, no abrupt change in antiepileptic medication or alteration of dose is warranted at this stage. The patient should be kept under observation for few days/weeks and decision about further antiepileptic therapy taken only on the basis of the subsequent course of events. Th e patient also must be advised to st rictly avoid alcoholic drinks in future.

SOLUTION 29.1 a. Since this patient does not require a hypnotic on regular basis, there is no identifiable cause of occasional sleep onset difficulty and he has tried non-drug measures, he can be prescribed a hypnotic to be kept handy for use when required. Because there is only sleep onset difficulty, and he will take the drug only later at night (after going to bed as usual), he needs a short acting hypnotic which woul d be free of residual effect next morning. Za leplon would be suitable for t his patient, as it has a short t½ (1 hour), does not cause next morning drowsiness, day time anxiety or rebound insomnia. Tolerance is unlikely to develop, because use is going to be occasional.

SOLUTION 30.1 a. The husband of the patient should be instructed that at the first sign of a seizu re attack the patient should be laid on bed or ground in t he prone or lateral position with neck extended to ensure free airway. No emergency medicine is requi red during or just after the fit. Only reassurance and moral support are needed. These instructions should be shared will other family members, so that anyone who is closeby may do the needful. b. Because the patient has a history of head trauma and two seizure attacks have occur red within one week, the probability of developing epilepsy is high. As such, antiepileptic medication should be started rightaway without waiting for test reports or further fit s to occur. c. Therapy should be initiated with a single antiepileptic drug. Antiepileptics with proven efficacy in post-head injury tonic-clonic seizures are phenobarbitone, phenytoin, carbamazepine and valproate. Since the patient is a young active lady, phenobarbitone with sedative/cognitive side effects, phenytoin with gum hyperplasia, hirsutism and other cosmetic side effects as well as problems related t o maintaining optimum blood levels of the drug, and valproate with tremor and weight gain would be less suitable. Carbamazepine appears to be the most appropriate initial drug in this case.

SOLUTION 31 .1 a. The Pa rkinson's disease of this patient appears to have advanced over the last 5 years and he is now experiencing 'wearing off effect' of levodopa-carbidopa. He is also developing dyskinesia, a late adverse effect of the drug. At this stage, antiparkinsonian medication cannot be withdrawn, becau se he will develop marked rigidity, immobility and tremor hampering life activities. He is

Contd ...

1006

APPENDICES

already experiencing an adverse effect of his medication; therefore, the dose should not be increased further. Since levodopa-carbidopa is the most efficacious and cheapest medication for parkinsonism, it may be prudent to continue it at a reduced dose and supplement it with anot her longer acting drug to smoothen the therapeutic effect. The options available as supplementary medication are: • A direct dopamine agonist like ropini ro le/pram ipexo le can be gradually added to levodopa-carbidopa w hose dose should be reduced in steps. Both drugs can be taken concurrently 3 times a day. Ropinirole/pramipexole being longer acting will smoothen symptom control. They also produce less dyskinesia. • A MA0-8 inhibitor like selegiline 5 mg twice a day or rasagiline 1 mg once a day in the morning will prevent degradation of dopamine in the brain, prolonging and smoothening effect of levodopa-carbidopa. • Entacapone 200 mg with each dose of levodopa-carbidopa can also potentiat e and prolong levodopa action by inhibiting another metabolizing enzyme COMT. It can also be an additional third drug to levodopa-carbidopa + selegiline for greater symptomatic relief.

SOLUTION 32.1 a. The most likely cause of the motor restlessness exhibited by the patient after 4 weeks of haloperidol therapy is appearance of a common extrapyram idal side effect of the antipsychotic drug called 'akathisia'. The symptom does not appear to be due to inadequate dose of haloperidol, because the psychotic symptoms have been relieved and the initial psychomotor restlessness had been controlled. There is no return of anxiety, hallucinations, etc. As such, there is no need to increase the dose of ha loperidol. Dose reduction may be tried but return of psychotic symptoms is a risk. One of the atypical antipsychotic drugs may be substituted for haloperidol. Quetiapine with its sleep promoting effect will be more suitable in this case. The atypical antipsychotics have a low propensity to cause extrapyramidal motor side effects, including akathisia. b. For early resol ution of motor restlessness, a benzodiazepine, e.g. clonazepam 1 mg or diazepam (5 mg) 2-3 times a day may be given. This may be supplemented by trihexyphenidyl 2 mg 3 times/day. In case the akathisia persists, propranolol 40 mg 2-3 times a day may be ad ded.

SOLUTION 33.1 a. Sertraline is a selective serotonin reuptake inhibitor (SSRI) and a first-line drug for major depression. The choice of drug is correct and the SO mg twice daily an average dose for initiation of therapy. However, since antidepressant action of any drug (including sertraline) takes 2-4 weeks to manifest, it is too early at 1 week to expect any improvement in depressive symptoms. The dose is not subtherapeutic as indicated by appearance of mild side effects, which nevertheless are quick to appear, but to whom gradual tolerance usually develops. Restlessness, nausea, dyspepsia, epigastric distress and anorexia are expected side effects of SSRls. The patient and his family members should be counselled to continue the medication for another 3-4 weeks by which time symptoms should start improving. The choice of drug is appropriate, and at this stage, there is no reason to change the medicine or its dose. No additional drug needs to be added at this stage.

1007

SO LUTION TO PROBLEM DIR ECTE D STU DY SOLUTION 34.1 a. Symptoms and signs indicate t hat t he patient is going into neurogenic shock due to the excruciating pain of the crush injury. As such, the first priority is to relieve the acute pain. Morphine 5 mg should be injected i.v. immediately. It will not only lessen the pain and suffering of the patient, but also allay apprehension and counteract neurogenic shock. It will facilitate proper examination and first aid measures as well. Supplemental doses may be given every 2-3 hours. An i.v. infusion of saline should be started at the earliest to restore blood pressure and maintain tissue perfusion.

SOLUTION 35.1 a. The primary reason for no improvement in the state of the patient is that all medicines, including donepezil, take weeks and months before any perceptible improvement in Alzheimer's symptoms become apparent. Moreover, donepezi l (or any other drug) is not effective in a significant number of patients. However, one week is too short a time to know whether this patient is going to benefit or not. Since this patient has developed intolerable cholinergic side effects, they are due to donepezil which should be discontinued. No other anticholinesterase drug is likely to be tolerated by this patient. Therefore, a drug which acts by a different mechanism could be used in th is patient. Memantine is the only other drug, with documented efficacy in moderate to severe Alzheimer's disease, which is not a cholinergic drug, and which probably acts by blocking glutamate excitotoxicity. It is better tolerated and does not produce cholinergic side effects. However, improvement in memory and cognitive function is less likely, and it may only serve to slow the functional decline.

SOLUTION 36.1 a. The weakness, nausea, sweating and fainting suffered by the patient is due to t he marked and rapid fall in BP caused by captopril (and augmented by furosemide). Congestive heart failu re patients have an overactive renin-angiotensin system (RAS) which helps in maintaining haemodynamics in the face of low cardiac output. Captopril is a rapidly acting ACE inhibitor, which, given in doses used for hypertension, removes the RAS support (angiotensin II is not formed) and causes marked fall in BP. This is aggravated by Na• loss caused by the diuretic. b. Though, a slower acting ACE i nhibitor, e.g. enalapril, would be less likely to cause rapid fall in BP, captopril cannot be considered a wrong choice of drug, provided it is initiated at 1/4th dose. In CHF, captopril therapy should be initiated at 6.25 mg dose, which can be gradually increased as haemodynamic adj ustment s take place. c. The reaction cou ld have been avoided by initiating captopril at 6.25 mg twice daily dose. A slower acting ACE inhibitor (at low starting dose) could be still less likely to produce acute hypotension. d. The first measure to be taken in this case is to put the patient in 15° head low position. This could be supplemented by short-term fluid and electrolyte infusion. A presser agent is rarely needed.

SOLUTION 38.1 a. This patient of moderate CHF is in a decompensated state with dilated heart. Though the diuretic (furosemide) and ACE inhibitor (enalapri l) will relieve symptoms slowly, they may not be sufficient to restore a compensated cardiac status. Digoxin should be prescribed concurrently as it is the

Contd...

1008

APPEND ICES most effective drug for restoring com pen sation by increasing ca rdiac contractility. The features of this patient do not indicate any urgency. Therefore, the patient may be started with an average maintenance dose 0.25 mg/day of digoxin. It is expected to produce peak effect after 5-7 days. Dosage adjust ment may be done after that depending on the response. b. Enalapril dose of 5 mg twice a day should be increased by 5 mg/day at 1- 2 week intervals ti ll hypotension or other side effects appear or 40 mg/day dose is reached. For maximum prognostic benefit, ACE inhibitors have to be used at or near the highest permissible dosage. Enalapril should not be stopped un less compelled by an adverse effect, because it continues to retard worsening of CHF and avoid complications. c. Since the patient is in a decompensated state, a p blocker cannot be added at this stage, because chances of deterioration of cardiac status are high. However, after compensation has been restored by digoxin, diuretic and enalapril and t he patient is in a stable condition, a suitable P blocker may be started at a very low dose, to be upward titrated later, because p blockers afford further morbidity and mortality benefits.

SOLUTION

39.1

a. The patient has been having atrial fibrillation (AF) for at least the past one month. He is likely to have developed thrombi in the fibrillating atria, and is at risk of em bolic stroke when sinus rhythm (SR) is restored. Therefore, he has been put on anticoagulant medication wit h warfarin to prevent thromboembolism . b. His heart (vent ricu lar) rate can be controlled by a drug which depresses A-V co nduction. For this purpose verapamil or diltiazem or propranolol should be given orally and dose adjusted to maintain a heart rate between 60-70/min. Digoxin (0.25 mg/day) may be prescribed in addition if t he ta rget heart rate is not achieved by monotherapy. c. If electri cal cardioversion does not succeed, amiodarone 200 mg injected i.v. over 60 min may be tried for reversa l to SR. d. After restoration of SR, the same may be maintained by continued t reatment with one of the following drugs, viz. sotalol/propafenone/amiodarone/dronedarone or disopyramide.

SOLUTION 40.1 a. This patient is having one or more episodes of angina practically every day; therefore, he should be prescribed regular medication to prevent the episodes. The first line drugs for this purpose which can be given to this patient are: 1. A long-acting nitrate viz. oral sustained release isosorbide mononitrate or similar drug morning and afternoon or transder mal glyceryl trinitrate patch applied in the morning and taken off at night. 2. A long-acting calcium channel blocker, like amlodipine once a day. Alternative second line or add-on drugs are: Nicorandil (K• channel opener), or ranolazine (IN• current inhibitor), or t rimetazidine (LC3-KAT inhibitor) or ivabradine (Ir current inhibitor). Since he is also suffering from COPD, he cannot be given a p blocker which is likely to precipitate severe breathlessness. b. This patient is having coronary artery disease (CAD). None of the above drugs can alter the course of CAD or prevent complications like Ml or death. He should in addition be put on Contd...

1009

SOLUTION TO PROBLEM DIRECTED STUDY long-term treatment with the followi ng to arrest/delay the progression of CAD and to afford cardioprotection: 1. An antiplatelet drug, such as low-dose aspirin or clopidogrel. 2. A hypolipidaemic statin, such as atorvastatin. 3. An angiotensin converting enzyme (ACE) inhibitor, such as enalapril.

SOLUTION 41 .1 a. Since the systolic BP is above 140 mm Hg and diastolic BP is below 90 mm Hg, this patient has 'isolated systolic hypertension'. Repeated measurements have confirmed the raised BP, therefore, antihypertensive medication is indicated. Therapy should be initiated with a single drug because he is stage I hypertensive (systolic BP